(Medbook4u Com) Pre21

Download as pdf or txt
Download as pdf or txt
You are on page 1of 912

PREP ® Self-Assessment PREPSA 2021

Question 1
A 5-year-old boy is seen for evaluation of a rash. He has a history of atopic dermatitis that is
typically well controlled. He reports falling on the playground and skinning his knee 4 days ago.
This morning, he awoke to find redness, swelling, and tenderness surrounding the abrasion. The
boy’s mother is a physician, and his younger sister has been treated for several skin abscesses in
the past. The boy appears well and has age-appropriate vital signs with no fever. He has a 2×1–
cm shallow abrasion overlying his left patella with approximately 3 cm of surrounding erythema
and edema. The area is tender, with no focal fluctuance. He has full range of motion of his knee.
He has several enlarged, nontender lymph nodes at the left inguinal canal. The remainder of his
physical examination findings are normal.

Of the following, the MOST appropriate treatment is


A. amoxicillin
B. cephalexin
C. clindamycin
D. doxycycline

American Academy of Pediatrics 2


PREP ® Self-Assessment PREPSA 2021
Correct Answer: C
The boy in the vignette has cellulitis that needs treatment with antibiotics. He has risk factors for
methicillin-resistant Staphylococcus aureus (MRSA), including a history of skin trauma, a parent
in the health-care field, and a family member with a history of skin abscesses. Therefore,
antibiotics must be chosen that provide coverage for common skin flora (eg, β-
hemolytic Streptococcus, methicillin-sensitive Staphylococcus aureus), as well as MRSA. Of the
response choices, clindamycin is the most appropriate treatment. Amoxicillin and cephalexin are
not effective coverage for MRSA, and doxycycline does not provide adequate coverage against
β-hemolytic Streptococcus.

When evaluating children with skin and soft tissue infections, bacterial culture and sensitivities
should guide antimicrobial choice if purulent material can be obtained. When a bacterial culture
cannot be obtained (eg, in cases of cellulitis or erysipelas), empiric antibiotic choice must be
guided by the type and location of the skin and soft tissue infection, the severity of illness, the
presence or absence of risk factors for MRSA infection, and local antibiotic sensitivities. Risk
factors for MRSA infection include:
• Skin trauma
• Frequent exposure to antibiotics
• Chronic disease, recent surgery or hospitalization, residence in a long-term care facility,
indwelling medical device
• Crowded living conditions, poor hygiene, sharing of potentially contaminated items such
as razors or towels
• History of abscesses or other skin infections in patient or household contact
• History of MRSA infection in the patient or household contact
• Household contact with identified risk factor

For hemodynamically stable children with cellulitis and risk factors for MRSA, initial antibiotic
choice should include coverage for MRSA and β-hemolytic Streptococcus. Clindamycin is the
preferred first-line agent. In communities where the prevalence of clindamycin-resistant MRSA
is greater than 15%, an alternative antibiotic strategy such as combination therapy (eg,
amoxicillin or cephalexin plus trimethoprim-sulfamethoxazole or doxycycline), vancomycin, or
linezolid must be considered. For hemodynamically stable children with cellulitis and no risk
factors for MRSA, antibiotic coverage to treat methicillin-sensitive Staphylococcus aureus and
β-hemolytic Streptococcus such as cephalexin, cefadroxil, or cefuroxime should be initiated.

PREP Pearls
• For hemodynamically stable children with cellulitis and risk factors for methicillin-
resistant Staphylococcus aureus, initial antibiotic choice should include coverage for
methicillin-resistant Staphylococcus aureus and β-hemolytic Streptococcus. Clindamycin
is the preferred first-line agent.
• For hemodynamically stable children with cellulitis and no risk factors for methicillin-
resistant Staphylococcus aureus, coverage should include methicillin-
sensitive Staphylococcus aureus and β-hemolytic Streptococcus. Cephalexin, cefadroxil,
and cefuroxime are options for this clinical situation.
• Risk factors for methicillin-resistant Staphylococcus aureus infection include skin
trauma, frequent exposure to antibiotics, chronic disease, recent surgery or
American Academy of Pediatrics 3
PREP ® Self-Assessment PREPSA 2021
hospitalization, sharing of potentially contaminated items such as razors or towels,
history of abscesses or other skin infections in patient or household contact, and history
of methicillin-resistant Staphylococcus aureus infection in patient or household contact.

ABP Content Specifications(s)


• Understand the epidemiology of Staphylococcus aureus
• Plan the appropriate management of methicillin-sensitive and methicillin-
resistant Staphylococcus aureus infection
• Plan the appropriate diagnostic evaluation of Staphylococcus infection
• Recognize the clinical features associated with Staphylococcus aureus infection

Suggested Readings
• American Academy of Pediatrics. Staphylococcus aureus. In: Kimberlin DW, Brady MT,
Jackson MA, Long SS, eds. Red Book 2018-2021: Report of the Committee on Infectious
Diseases. 31st ed. Itasca, IL: American Academy of Pediatrics; 2018:733-745. Red Book
Online .
• American Academy of Pediatrics. Group A Streptococcal infections. In: Kimberlin DW,
Brady MT, Jackson MA, Long SS, eds. Red Book 2018-2021: Report of the Committee
on Infectious Diseases. 31st ed. Itasca, IL: American Academy of Pediatrics; 2018:748-
761.
• Stevens DL, Bisno AL, Chambers HF, et al. Practice guidelines for the diagnosis and
management of skin and soft tissue infections: 2014 update by the infectious diseases
society of America. Clin Infect Dis. 2014;59(2):147-159. doi: 10.1093/cid/ciu296.

American Academy of Pediatrics 4


PREP ® Self-Assessment PREPSA 2021
Question 2
A 20-month-old, previously healthy boy is being evaluated for poor weight gain for 4 to 5
months, and watery diarrhea, bloating, and poor appetite for 3 weeks. He currently passes stools
4 to 6 times daily, wakes up with stool in his diaper, and has a significant diaper rash. He has
had no vomiting or hematochezia. The boy prefers to drink whole milk (generally 20 oz/day) and
water. He does not eat many sugary foods and does not drink juice. His father has lactose
intolerance and his older brother has type 1 diabetes. The boy’s physical examination findings
demonstrate a weight of 10 kg (7th percentile), length of 87 cm (54th percentile), and
weight/length less than the 1st percentile. He appears tired and has thin extremities. His
abdomen is soft, moderately distended, and nontender, without mass or
organomegaly. Moderate diaper dermatitis is present.

Stool testing shows the following results:


Laboratory Test Result
Fecal ova and parasite examination Negative
Giardia antigen Negative
Fecal bacterial culture Negative
Fecal fat 43% (reference range <20%)
Pancreatic elastase >500 μg/g (reference range >200)
Fecal occult blood Negative
Fecal white blood cells Negative

Of the following, the MOST likely cause of this boy’s diarrhea is


A. celiac disease
B. functional diarrhea
C. Shwachman Diamond syndrome
D. ulcerative colitis

American Academy of Pediatrics 5


PREP ® Self-Assessment PREPSA 2021
Correct Answer: A
The boy in the vignette has fat malabsorption due to celiac disease. This diagnosis is supported
by the presence of excessive fat in the stool in the absence of pancreatic insufficiency (normal
pancreatic elastase) and inflammation (no evidence of blood or white blood cells in the stool).
The boy has signs and symptoms suggestive of malabsorption including poor weight gain/failure
to thrive, watery diarrhea, abdominal distention, bloating, and diaper dermatitis. Other signs and
symptoms that may be seen with malabsorption include abdominal pain, anemia, edema, and
easy bleeding. Thus, the presence of any of these findings should prompt consideration of
evaluation for malabsorption.

Malabsorption should be considered in infants and children with failure to thrive/poor weight
gain, vitamin deficiencies, and evidence of gastrointestinal losses (vomiting and/or diarrhea).
Evaluation should be tailored to the history and suspected macronutrient(s) involved.
Carbohydrate malabsorption can cause watery and acidic stool; this may result from conditions
including lactose intolerance, excessive fructose ingestion, fructose intolerance, or congenital
intestinal transport/enzyme deficiencies (eg, sucrase-isomaltase deficiency). Failure to thrive
may be seen in severe cases. Protein malabsorption may present with hypoalbuminemia, failure
to thrive, and edema in protein-losing enteropathies (eg, Crohn disease and celiac disease). Fat
malabsorption may be caused by enteropathies (eg, Crohn disease and celiac disease); this may
present with failure to thrive, steatorrhea (greasy stools which may float in the toilet water), and
fat-soluble vitamin deficiencies. Causes of steatorrhea include fat maldigestion (as in pancreatic
insufficiency from cystic fibrosis or Shwachman Diamond syndrome or impaired bile
production/secretion because of cholestatic liver diseases) or fat malabsorption.

Functional diarrhea (or chronic nonspecific diarrhea of childhood) is unlikely in this boy because
of his several-month history of poor weight gain coupled with examination findings of thin
extremities and significant diaper rash. With a normal pancreatic elastase, Shwachman Diamond
syndrome is excluded as a possibility because there is no evidence of pancreatic insufficiency.
Ulcerative colitis is not likely in this boy because of the absence of hematochezia and fecal
occult blood.

PREP Pearls
• Malabsorption should be considered in infants and children with failure to thrive/poor
weight gain, vitamin deficiencies, and evidence of gastrointestinal losses (vomiting
and/or diarrhea).
• Evaluation for malabsorption should be tailored to the history and suspected
macronutrient(s) involved.

ABP Content Specifications(s)


• Formulate an age-related differential diagnosis for malabsorption

American Academy of Pediatrics 6


PREP ® Self-Assessment PREPSA 2021
Suggested Readings
• Ammoury, RF, Croffie JM. Malabsorptive disorders of childhood. Pediatr
Rev. 2010;31(10):407-416. doi: 10.1542/pir.31-10-407.
• Ediger TR, Hill ID. Celiac disease. Pediatr Rev. 2014;35(10):409-416.
doi: 10.1542/pir.35-10-409.
• Ulshen MH. Diarrhea and steatorrhea. In: McInerny TK, Adam HM, Campbell DE,
DeWitt TG, Foy JM, Kamat DM, eds. American Academy of Pediatrics Textbook of
Pediatric Care. 2nd ed. Itasca, IL: American Academy of Pediatrics; 2016;chap
138:1267-1281. Pediatric Care Online.
• Zella GC, Israel EJ. Chronic diarrhea in children. Pediatr Rev. 2012;33(5):207-218.
doi: 10.1542/pir.33-5-207.

American Academy of Pediatrics 7


PREP ® Self-Assessment PREPSA 2021
Question 3
A 35 4/7-week-gestation neonate is admitted to the neonatal intensive care unit with respiratory
distress. Maternal group B Streptococcus status is negative. The mother’s pregnancy was
complicated by mild gestational hypertension. A primary cesarean delivery was performed after
failed induction of labor. Meconium was noted in the amniotic fluid. The Apgar score was 8 at 1
and 5 minutes. Maternal temperature was 38°C for 1 hour after delivery. The neonate is placed
on continuous positive airway pressure. Ampicillin and gentamicin are started after a blood
culture specimen is drawn. Early-onset sepsis is a concern.

Of the following, the factor in this neonate’s history MOST closely associated with this
diagnosis is
A. cesarean delivery
B. maternal gestational hypertension
C. meconium-stained amniotic fluid
D. preterm gestational age

American Academy of Pediatrics 8


PREP ® Self-Assessment PREPSA 2021

Correct Answer: D
Early-onset sepsis (EOS) continues to be a significant source of neonatal morbidity and
mortality. Since the introduction of intrapartum antibiotic prophylaxis for maternal group
B Streptococcus (GBS) colonization, rates of EOS resulting from GBS have decreased
dramatically. The risk factors for GBS sepsis have changed with the administration of penicillin
to women in labor. Being born prematurely, between 34 and 36 weeks’ gestation, or post-term at
more than 41 weeks of gestation increases the risk for EOS. Maternal fever, particularly if the
highest temperature is greater than 38°C, and prolonged rupture of membranes are also
associated with EOS.

Puopolo et al combined maternal and neonatal factors to create an online calculator to estimate
the risk for EOS. Practitioners must enter their institution-specific rate of EOS and the maternal
risk factors; the calculator will estimate risk of infection stratified by the neonate’s clinical
condition. Many institutions now use this online calculator
(https://neonatalsepsiscalculator.kaiserpermanente.org/) with documented reduction in antibiotic
exposure without missing neonates with EOS. The accuracy and reliability of clinical
assessments must be considered when using this tool.

Meconium-stained amniotic fluid indicates fetal distress, but is not independently associated with
EOS. Cesarean delivery and maternal gestational hypertension are not associated with increased
risk of EOS.

PREP Pearls
• Risk of early-onset sepsis due to group B Streptococcus is higher among neonates born at
less than 36 weeks’ or greater than 41 weeks’ gestation.
• Highest maternal temperature greater than 38°C and prolonged rupture of membranes are
associated with increased risk of early-onset sepsis.
• Intrapartum prophylaxis for mothers positive for group B Streptococcus (GBS) has
decreased overall rates of neonatal GBS sepsis.

MOCA-Peds Objective
• Prevent and manage the complications and risks associated with late pre-term birth.

ABP Content Specifications(s)


• Plan the appropriate management of an infant born to a mother with chorioamnionitis

American Academy of Pediatrics 9


PREP ® Self-Assessment PREPSA 2021
Suggested Readings
• Escobar GJ, Puopolo KM, Wi S, et al. Stratification of risk of early-onset sepsis in
newborns =34 weeks' gestation. Pediatrics. 2014;133:30. doi: 10.1542/peds.2013-1689.
• Kojaoghlanian T. The newborn at risk for infection. In: McInerny TK, Adam HM,
Campbell DE, DeWitt TG, Foy JM, Kamat DM, eds. American Academy of Pediatrics
Textbook of Pediatric Care. 2nd ed. Itasca, IL: American Academy of Pediatrics;
2016;chap 102:899-908. Pediatric Care Online.
• Puopolo KM, Benitz WE, Zaoutis ZE, et al; Committee on Fetus and Newborn and
Committee on Infectious Diseases. Management of neonates born at =35 0/7 weeks'
gestation with suspected or proven early-onset bacterial sepsis. Pediatrics.
2018;142(6):e20182894. doi: 10.1542/peds.2018-2894.
• Puopolo KM, Draper D, Wi S, et al. Estimating the probability of neonatal early-onset
infection on the basis of maternal risk factors. Pediatrics. 2011;128:e1155.
doi: 10.1542/peds.2010-3464.

American Academy of Pediatrics 10


PREP ® Self-Assessment PREPSA 2021
Question 4
A 10-year-old boy is seen in the office for evaluation of sudden onset of cola-colored urine. He
reports no pain, burning, or passage of blood clots with urination. He has had a sore throat and
mild cough for 2 days. He does not have flank pain, fever, rash, or joint pains. He had a similar
episode of painless cola-colored urine 6 months ago that resolved without treatment. There is no
family history of renal disease. He has a heart rate of 86 beats/min, a respiratory rate of 14
breaths/min, and a blood pressure of 128/82 mm Hg. His chest is clear to auscultation. The rest
of his physical examination findings are unremarkable.

Laboratory data are shown:


Laboratory Test Result
Blood
White blood cell count 7,400/µL (7.4 × 10⁹/L)
Hemoglobin 12.8 g/dL (128 g/L)
Platelet count 360 × 10³/µL (360 × 10⁹/L)
Blood urea nitrogen 36 mg/dL (12.9 mmol/L)
Creatinine 0.9 mg/dL (80 µmol/L)
Complement C3 110 mg/dL (1.10 g/L) (reference range 85-142 mg/dL [0.85-1.42 g/L])
Complement C4 24 mg/dL (0.24 g/L) (reference range 12-41 mg/dL [0.12-0.41 g/L])
Antistreptolysin O < 200 IU
Urine
Blood 3+
White blood cells 5-10/HPF
Red blood cells 50-100/HPF
Protein 3+

Of the following, the MOST likely diagnosis for this child is


A. acute hemorrhagic cystitis
B. immunoglobulin A nephritis
C. poststreptococcal glomerulonephritis
D. systemic lupus erythematosus nephritis

American Academy of Pediatrics 11


PREP ® Self-Assessment PREPSA 2021
Correct Answer: B
The boy in this vignette has clinical and laboratory features suggestive of immunoglobulin A
(IgA) nephritis. A history of recurrent painless gross hematuria, proteinuria, hypertension, and
azotemia favor a diagnosis of acute glomerulonephritis (AGN). A normal serum complement
level in this child further supports a diagnosis of IgA nephritis.

Acute glomerulonephritis or nephritic syndrome is characterized by sudden-onset hematuria


(macroscopic or microscopic), proteinuria, hypertension, edema, and acute kidney injury. The
presence of cola-colored or tea-colored urine without clots and red blood cell casts on urinalysis
suggest hematuria of a glomerular origin. Laboratory evaluation in a child presenting with AGN
includes urinalysis with microscopy, serum electrolytes, blood urea nitrogen, serum creatinine,
serum albumin, and a complete blood count. The serum complement levels (C3 and C4)
differentiate causes of hypocomplementemic AGN from those with normal complement levels
(Item C4).

American Academy of Pediatrics 12


PREP ® Self-Assessment PREPSA 2021

Immunoglobulin A nephritis characteristically presents with recurrent episodes of gross


hematuria triggered by an upper respiratory tract or gastrointestinal illness. The synpharyngitic
(concomitant with an upper respiratory illness) presentation of IgA nephritis differentiates it
from acute poststreptococcal glomerulonephritis, in which a prior history of sore throat is
present. The gross hematuria usually resolves within a few days to a week. The microscopic
hematuria may persist or completely clear in between episodes of recurrent gross hematuria.
American Academy of Pediatrics 13
PREP ® Self-Assessment PREPSA 2021
Occasionally, children with IgA nephritis may present only with persistent microscopic
hematuria and worsening proteinuria.

The laboratory evaluation of IgA nephritis may show features of AGN (azotemia, proteinuria,
and hematuria) and a normal serum level of C3 and C4, as seen in the boy in the vignette. The
serum IgA level can be high or normal and is not diagnostic of IgA nephritis. Children with acute
kidney injury, nephrotic range proteinuria, and persistent hematuria may need a renal biopsy.
The diagnosis is confirmed by immunofluorescence staining of IgA in the glomerular mesangial
cells.

Acute hemorrhagic cystitis, commonly caused by adenovirus, is unlikely for the child in the
vignette. Acute hemorrhagic cystitis presents with acute onset of dysuria and urinary frequency
followed by hematuria. The urine may be bright red and blood clots may be present.
Hypertension and azotemia are absent in acute hemorrhagic cystitis. Poststreptococcal
glomerulonephritis is characterized by a history of sore throat a few weeks before the onset of
hematuria. In children with poststreptococcal glomerulonephritis, ASO titers may be elevated,
C3 level is low, and C4 level is normal. Systemic lupus erythematosus nephritis is characterized
by both C3 and C4 levels being low.

Immunoglobulin A nephritis is treated by using different regimens based on severity of


presentation and findings on the renal biopsy. Children may need intravenous
methylprednisolone pulses, oral prednisone, other immunosuppressive medications, or
angiotensin-converting enzyme inhibitors. Poor prognostic markers are proteinuria,
hypertension, acute kidney injury at presentation, and severe histologic changes on the renal
biopsy.

PREP Pearls
• Immunoglobulin A nephritis characteristically presents with recurrent episodes of gross
hematuria concomitant with an upper respiratory illness.
• The serum complement levels (C3 and C4) are normal in immunoglobulin A nephritis
and helpful in differentiating it from other causes of hypocomplementemic acute
glomerulonephritis.

ABP Content Specifications(s)


• Recognize the clinical findings associated with IgA nephropathy

American Academy of Pediatrics 14


PREP ® Self-Assessment PREPSA 2021
Suggested Readings
• Goodman S, Reid-Adam J. Immunoglobulin A nephropathy. Pediatr Rev.
2019;40(8):439-441. doi:10.1542/pir.2018-0119.
• VanDeVoorde RG III. Acute poststreptococcal glomerulonephritis: the most common
acute glomerulonephritis. Pediatr Rev. 2015;36(1):3-13. doi:10.1542/pir.36-1-3.
• Varade WS. Nephritis. In: McInerny TK, Adam HM, Campbell DE, DeWitt TG, Foy JM,
Kamat DM, eds. American Academy of Pediatrics Textbook of Pediatric Care. 2nd ed.
American Academy of Pediatrics; 2017:2358-2367. Pediatric Care Online.
• Viteri B, Reid-Adam J. Hematuria and proteinuria in children. Pediatr
Rev. 2018;39(12):573-587. doi:10.1542/pir.2017-0300.

American Academy of Pediatrics 15


PREP ® Self-Assessment PREPSA 2021
Question 5
A 7-year-old previously healthy boy is seen in the emergency department for concern about
black stools for 1 day. For 2 days before the onset of the black stool, he had multiple episodes of
brown, watery, nonbloody diarrhea and nonbloody, nonbilious emesis. He complains of
abdominal pain only when he has a bowel movement or vomits. He has had no fever. He has had
no known sick contacts and has not traveled. On the day of presentation, the boy had a large
black stool with no visible blood. His vital signs and physical examination findings are normal.
His mother reports giving the boy several over-the-counter medications for his diarrhea and
vomiting.

Of the following, the medication MOST likely responsible for the boy’s stool finding is
A. aluminum hydroxide
B. bismuth subsalicylate
C. calcium carbonate
D. magnesium hydroxide

American Academy of Pediatrics 16


PREP ® Self-Assessment PREPSA 2021
Correct Answer: B
The child in the vignette was most likely given bismuth subsalicylate by his mother in an attempt
to alleviate symptoms of acute gastroenteritis. The bismuth component may react with small
amounts of sulfur in the gastrointestinal tract and result in black stools, which can be mistaken
for melena. Bismuth also reacts with sulfur in saliva, which can result in a black tongue. Both
discolorations resolve after cessation of the medication. The wintergreen smell of these products
is because of the salicylate component. The presence of salicylates in this medication places the
child at risk for the same toxidrome associated with aspirin—salicylism—especially with chronic
ingestion. Children with varicella who take bismuth subsalicylate may be at risk for Reye
syndrome.

Over-the-counter (ie, nonprescription) medications present a unique toxicologic risk for the child
because they are readily available for purchase, often in extremely large quantities, and parents
may believe nonprescription medications do not pose a significant risk. Common analgesics such
as acetaminophen, ibuprofen, and salicylates run the risk of significant toxicity in and of
themselves, and are often a component of combination medications targeted toward the treatment
of upper respiratory tract infections. These combination medications often contain classes of
medications that can pose a significant risk to the pediatric patient. Item C5 lists common
components in over-the-counter cough and cold preparations and their potential adverse effects.
Because there is no evidence to support the efficacy of common over-the-counter cough and cold
medications for alleviation of symptoms in infants, toddlers, and young children, and the risks
are recognized, current recommendations are to not use them for children younger than 6 years.

American Academy of Pediatrics 17


PREP ® Self-Assessment PREPSA 2021

Recent regulatory actions have led to a decrease in dosing errors and unintentional ingestions of
over-the-counter cough and cold medications in children. Nonetheless, the potential risk of
toxicity from all classes of over-the-counter medications expands each year as additional
American Academy of Pediatrics 18
PREP ® Self-Assessment PREPSA 2021
products become available without prescription. It is important to advise parents of these risks
and that nonprescription medications should be kept out of reach of young children and only
administered as listed on the package or as instructed by a health-care professional.

Aluminum hydroxide is an antacid used to relieve indigestion that also has laxative properties.
Calcium carbonate is an antacid that works by decreasing the amount of stomach acid.
Magnesium hydroxide is an antacid used to relieve indigestion, which also has laxative
properties. Magnesium hydroxide and aluminium hydroxide are also combined into a single
product marketed toward the treatment of indigestion. None of these products would produce
black stools.

PREP Pearls
• The bismuth component of bismuth subsalicylate may react with small amounts of sulfur
in the gastrointestinal tract and result in black stools.
• Ingestion of bismuth subsalicylate can result in salicylism, especially with chronic
ingestions, and children with varicella may be at risk for Reye syndrome.
• Over-the-counter cough and cold medications have not been shown to be effective in
young children, and can pose a significant risk to children younger than 6 years.

ABP Content Specifications(s)


• Know the components and the common adverse effects and toxicities of common over-
the-counter preparations, and advise regarding their appropriate use

Suggested Readings
• Gunn VL, Taha SH, Liebelt EL, Serwint JR. Toxicity of over-the-counter cough and cold
medications. Pediatrics. 2001;108(3):E52. doi: 10.1542/peds.108.3.e52.
• Lowry JA, Leeder JS. Over-the-counter medications: update on cough and cold
preparations. Pediatr Rev. 2015;36(7):286-298. doi: 10.1542/pir.36-7-286.

American Academy of Pediatrics 19


PREP ® Self-Assessment PREPSA 2021
Question 6
A previously healthy 2-year-old girl is seen in the office for evaluation of bloody discharge from
her left ear. She was seen earlier in the day at an urgent care center for ear pain. The urgent care
provider could not visualize the left tympanic membrane because of the impacted cerumen. She
used an ear curette to remove the cerumen and subsequently visualized a normal left tympanic
membrane. After the visit, her mother noted blood coming from the left ear. The girl's vital signs
are normal. Her physical examination findings are normal except for crusted blood in her left ear
canal obscuring the tympanic membrane.

Of the following, the BEST next step in management is to


A. irrigate the left ear canal
B. prescribe antibiotic ear drops
C. provide supportive care
D. refer her to an otolaryngologist

American Academy of Pediatrics 20


PREP ® Self-Assessment PREPSA 2021
Correct Answer: C
The child in the vignette should be given supportive care. Irrigation could further irritate the ear
canal and exacerbate the bleeding. Antibiotic ear drops are not needed at this time because an
infection has not been identified. Referral to an otolaryngologist is also unnecessary because she
has a simple superficial irritation that will resolve.

Blood coming from the ear can be alarming to parents. Fortunately, the most common causes are
relatively benign and readily treatable. Superficial ear canal irritation such as from an ear curette
or scratching from seborrheic dermatitis is usually self-limiting, and parents can be reassured that
hearing will not be impaired. Counseling on appropriate hygiene and treatment of any underlying
skin condition may prevent further irritation. Otitis media and otitis externa may sometimes
produce blood-tinged fluid. Published guidelines outline treatment recommendations for these
conditions including for those who have underlying anatomical or medical abnormalities.

Foreign bodies are also a common cause of bloody otorrhea. If the foreign body is a button
battery, emergent evaluation by an otolaryngologist is necessary for removal because of possible
severe soft tissue and bony degradation from the corrosive electrolytes contained within the
battery. Other foreign bodies may be removed in a less urgent outpatient setting.

Although usually benign, some more concerning causes of otorrhea need to be considered. Head
trauma resulting in a basilar skull fracture may also result in bloody otorrhea, even without the
classic signs of raccoon eyes or Battle sign. Any child who has head trauma should undergo an
ear examination to look for hemotympanum or blood-tinged ear drainage. Clinicians should be
aware that a cerebrospinal fluid leak may present as clear otorrhea.

Bloody otorrhea may also be the result of a cholesteatoma or other otic mass, as well as
granulomatosis with polyangiitis. The latter will usually also have systemic symptoms. An
otolaryngologist should be consulted if these diagnoses are suspected.

PREP Pearls
• Bloody otorrhea that results from superficial irritation of the ear canal requires only
reassurance and supportive care.
• Counseling patients on appropriate ear hygiene can prevent ear canal irritation.
• Button batteries in the ear canal require emergent removal by an otolaryngologist.

MOCA-Peds Objective
• Evaluate and manage ear trauma.

ABP Content Specifications(s)


• Identify the various causes of bloody otorrhea

American Academy of Pediatrics 21


PREP ® Self-Assessment PREPSA 2021
Suggested Readings
• American Academy of Pediatrics healthchildren.org website. Button batteries: damaging
if swallowed, put in ears or nose. https://www.healthychildren.org/English/safety-
prevention/at-home/Pages/Button-Batteries-Damaging-if-Swallowed-Put-in-Ears-or-
Nose.aspx.
• Rosa-Olivares J, Porro A, Rodriguez-Varela M, Riefkohl G, Niroomand-Rad I. Otitis
media: to treat, to refer, to do nothing: a review for the practitioner. Pediatr Rev.
2015;36(11):480-488. doi: 10.1542/pir.36-11-480.
• Rosenfeld RM, Schwartz SR, Cannon CR. Clinical practice guideline: acute otitis
externa. Otolaryngol Head Neck Surg. 2014;150(1 suppl):S1-S24.
doi: 10.1177/0194599813517083.
• Schwartz SR, Magin AE, Rosenfeld RM. Clinical practice guideline (update): earwax
(cerumen impaction). Otolaryngol Head Neck Surg. 2017;156(1 suppl):S1-S29.
doi: 10.1177/0194599816671491.

American Academy of Pediatrics 22


PREP ® Self-Assessment PREPSA 2021
Question 7
An 8-year-old girl is referred to her primary care provider by her dentist for evaluation of tongue
and mouth lesions (Item Q7A and Item Q7B). Her mother recalls the lesions as being present
for at least the past 3 years. They have not changed and are asymptomatic. The girl has no known
medical problems. A review of systems is significant for intermittent constipation and diarrhea.
No one in her family has similar lesions. Her weight is at the 25th percentile, and her height is at
the 90th percentile. Her face is long and thin, and she has full lips. Her joints are
hyperextensible, and pes planus is present. The remainder of her physical examination findings
are unremarkable.

American Academy of Pediatrics 23


PREP ® Self-Assessment PREPSA 2021

Of the following, the BEST next step is to evaluate for


A. an insulinoma
B. a medullary thyroid carcinoma
C. an optic glioma
D. a pheochromocytoma

American Academy of Pediatrics 24


PREP ® Self-Assessment PREPSA 2021
Correct Answer: B
The girl in the vignette has multiple endocrine neoplasia type 2B (MEN2B). Her mouth lesions
are mucosal neuromas. The mucosal neuromas; tall, thin body habitus; full lips; and joint laxity
are all phenotypic features of MEN2B. Her intermittent constipation and diarrhea are typical
symptoms of associated gastrointestinal tract ganglioneuromas. Although not mentioned for the
girl in the vignette, alacrima from birth is also a feature of MEN2B.

The best next step is to evaluate for medullary thyroid carcinoma (MTC). Medullary thyroid
carcinoma occurs in 100% of individuals with MEN2B, is aggressive, and metastasizes early.
Other than resection, effective treatment options for MTC are limited. The American Thyroid
Association recommends prophylactic thyroidectomy in the first year after birth for those with
MEN2B. It is important to recognize the clinical features of MEN2B for timely prevention,
detection, and treatment of the associated MTC.

Evaluation for MTC includes thyroid ultrasonography and a serum calcitonin level test.
Calcitonin is produced by the neuroendocrine C cells of the thyroid and is elevated in MTC.
Pheochromocytoma is also associated with MEN2B, but the onset is later and the prognosis is
significantly better than for MTC. Insulinoma and optic glioma are not associated with MEN2B.
Insulinoma is associated with MEN type 1. Optic glioma is associated with neurofibromatosis
type 1.

Multiple endocrine neoplasia type 2 (2A and 2B) is caused by heterozygous pathogenic variants
in the RET proto-oncogene. They are inherited in an autosomal dominant fashion. At-risk family
members of those with a pathogenic RET proto-oncogene variant should also be tested. Most of
those with MEN2B, however, harbor a de novo pathogenic variant.

There is significant genotype-phenotype correlation with RET pathogenic variants, which are
stratified according to risk level. Although the penetrance of MTC in MEN type 2 (2A and 2B) is
essentially 100%, the age of onset varies according to risk level. Multiple endocrine neoplasia
type 2B is associated with the highest risk variants. Multiple endocrine neoplasia type 2A and
familial medullary thyroid carcinoma are associated with less high, moderate, and lower risk
variants. Features of MEN type 2A include hyperparathyroidism in addition to MTC and
pheochromocytoma, but not the physical phenotype. The American Thyroid Association
recommends timing of prophylactic thyroidectomy based on the risk level of the
specific RET proto-oncogene pathogenic variant. Item C7 summarizes the clinical features of the
conditions associated with RET proto-oncogene pathogenic variants.

American Academy of Pediatrics 25


PREP ® Self-Assessment PREPSA 2021

Risk for multiple endocrine neoplasia type 2A is often identified in children after genetic testing
is performed because of family history. Papillary thyroid carcinoma is the most common type of
thyroid cancer occurring in children and commonly presents as a thyroid nodule, cervical
lymphadenopathy, or both.

PREP Pearls
• Phenotypic features of multiple endocrine neoplasia type 2B include mucosal neuromas;
a tall, thin body habitus; full lips; joint laxity; and alacrima. Gastrointestinal tract
ganglioneuromas may cause intermittent constipation and diarrhea.
• Multiple endocrine neoplasia type 2B is associated with early-onset and aggressive
medullary thyroid carcinoma, in addition to pheochromocytoma and the physical
phenotype.
American Academy of Pediatrics 26
PREP ® Self-Assessment PREPSA 2021
• It is important to recognize the clinical features of multiple endocrine neoplasia type 2B
for timely prevention (prophylactic thyroidectomy), detection, and treatment of the
associated medullary thyroid carcinoma.

ABP Content Specifications(s)


• Recognize the clinical features associated with thyroid carcinoma

Suggested Readings
• Anisowicz SK, McIver H, Pedersen AM. Visual diagnosis: exophytic lesions on tongue
and oral mucosa. Pediatr Rev. 2018;39(9):e43-e46. doi:10.1542/pir.2017-0184.
• Francis GL, Waguespack SG, Bauer AJ, et al; American Thyroid Association Guidelines
Task Force. Management guidelines for children with thyroid nodules and differentiated
thyroid cancer. Thyroid. 2015;25(7):716-759. doi:10.1089/thy.2014.0460.
• Kochmann M, Bansal S, Umpaichitra V, Perez-Colon S. Visual diagnosis: an 11-year-old
girl with swollen lips and oral bumps. Pediatr Rev. 2017;38(10):e38-e40.
doi:10.1542/pir.2016-0059.
• Nagaoka R, Sugitani I, Sanada M, et al. The reality of multiple endocrine neoplasia type
2B diagnosis: awareness of unique physical appearance is important. J Nippon Med Sch.
2018;85(3):178-182. doi:10.1272/jnms.JNMS.2018_85-26.
• Osipoff JN, Wilson TA. Consultation with the specialist: thyroid nodules. Pediatr
Rev. 2012;33(2):75-81. doi:10.1542/pir.33-2-75.

American Academy of Pediatrics 27


PREP ® Self-Assessment PREPSA 2021
Question 8
A 30-week-gestation pregnant woman is seen at the obstetrician’s office for a febrile illness. Her
symptoms include headache and joint pain. She has had routine prenatal care and no
complications of pregnancy. Physical examination reveals the rash shown in Item Q8. She asks
how this illness may affect her newborn.

Item Q8: Rash seen in the woman described in in the vignette.


American Academy of Pediatrics 28
PREP ® Self-Assessment PREPSA 2021

Courtesy of George Nankervis and Kimberlin DW, et al, eds. Red Book Online. Itasca, IL:
American Academy of Pediatrics; 2018

Of the following, the MOST accurate response is that a likely sequela is


A. central nervous system calcification
B. chorioretinitis
C. generalized edema
D. hearing loss

American Academy of Pediatrics 29


PREP ® Self-Assessment PREPSA 2021
Correct Answer: C
The most likely sequela for the newborn in the vignette is generalized edema. Maternal infection
with parvovirus B19 is suggested by arthralgia and a lacy rash. Maternal infection can lead to
fetal hydrops, isolated pleural or pericardial effusions in the fetus, intrauterine growth restriction,
anemia, and fetal demise. Pregnant women with acute parvovirus B19 infection should be
monitored closely; fetal infection is thought to occur in approximately 30% of cases. Fetal deaths
have only been described with maternal infections in the first half of pregnancy; they are
estimated to occur in 3% to 11% of maternal infections before 20 weeks’ gestation.

Parvovirus B19 infections are often asymptomatic. Clinical manifestations include erythema
infectiosum (Fifth disease), characterized by a facial rash with a “slapped cheek” appearance that
is followed by a symmetric, lacy or maculopapular, pruritic exanthem of the trunk and
extremities. Additional clinical manifestations may include polyarthropathy, or a distinct
exanthem termed papular-purpuric gloves-and-socks syndrome in which the morphology of the
rash can range from papules, petechiae, and pustules, to bullae. In adults, the lacy exanthem is
less common than in children, whereas arthralgias are more prominent. Severe anemia resulting
from pure red cell aplasia can occur in immunodeficient children and aplastic crises may occur in
children with hemoglobinopathies or chronic hemolytic anemia. Encephalitis, hepatitis, and
myocarditis can also occur. Parvovirus B19 infection during pregnancy does not cause
congenital anomalies.

The other response options can result from congenital infections, but the maternal signs and
symptoms would differ from those noted in the vignette. Chorioretinitis and central nervous
system calcifications can be found in infants with congenital cytomegalovirus infection and
toxoplasmosis. Hearing loss is most often associated with congenital cytomegalovirus infection
and congenital rubella infection.

PREP Pearls
• Maternal infection with parvovirus B19 can lead to fetal hydrops, intrauterine growth
restriction, and fetal death.
• Clinical manifestations of parvovirus B19 infections include erythema infectiosum,
arthropathy, severe anemia caused by pure red cell aplasia in immunodeficient patients
and aplastic crises in patients with hemoglobinopathies.
• Parvovirus B19 infection during pregnancy is not a cause of congenital anomalies.

ABP Content Specifications(s)


• Recognize the complications of parvovirus B19 infection
• Recognize the clinical features associated with erythema infectiosum

Suggested Readings
• American Academy of Pediatrics. Parvovirus B19. In: Kimberlin DW, Brady MT,
Jackson MA, Long SS, eds. Red Book: 2018 Report of the Committee on Infectious
Diseases. Itasca, IL: American Academy of Pediatrics; 2018:602-606. Red Book Online.
• Kojaoghlanian T. The newborn at risk for infection. In: McInerny TK, Adam HM,
Campbell DE, DeWitt TG, Foy JM, Kamat DM, eds. American Academy of Pediatrics

American Academy of Pediatrics 30


PREP ® Self-Assessment PREPSA 2021
Textbook of Pediatric Care. 2nd ed. Itasca, IL: American Academy of Pediatrics; 2016;
chap 102. Pediatric Care Online.
• Qiu J, Soderlund-Venermo M, Young N. Human parvoviruses. Clin Microbiol Rev.
2016;30(1):43-113. doi: 10.1128/CMR.00040-16.

American Academy of Pediatrics 31


PREP ® Self-Assessment PREPSA 2021
Question 9
An 11-year-old boy is being evaluated for bilateral anterior knee pain. He reports gradual onset
of pain after beginning training with a local soccer development program. He denies any acute
injury. The pain is progressive, and over the past several weeks he has developed focal swelling
several inches below his kneecaps. His medical history is remarkable for multiple episodes of
chronic heel and lower leg pain over the past several years. He is otherwise healthy with normal
growth and development. The boy began training year round with a soccer travel league last
year. In addition, he enjoys participating in basketball and baseball. His soccer coach recently
recommended that the boy focus more on soccer and give up other sports activities.

On physical examination, the boy has a significant limp. There is focal swelling and tenderness
bilaterally over the tibial tuberosities and pain with resisted extension of the knees. His
examination findings are otherwise unremarkable. The boy is counseled on ways to address his
current knee pain and minimize his risk of future injury.

Of the following, the BEST recommendation for this boy would be to


A. comply with his coach’s recommendation regarding sports participation
B. obtain cushioned orthotics for use with his cleats
C. seek additional training with a certified fitness specialist
D. spend at least 2-3 months per year away from structured sports participation

American Academy of Pediatrics 32


PREP ® Self-Assessment PREPSA 2021
Correct Answer: D
The boy in the vignette has findings typically seen with apophysitis of the tibial tuberosity, also
known as Osgood-Schlatter disease (OSD). Osgood-Schlatter disease is an overuse syndrome
that occurs with repetitive traction of the patellar tendon on the open apophysis. Although OSD
can occur in any child with open growth plates, it is most commonly seen in boys around the
time of the adolescent growth spurt. Participation in a single sport throughout the year is a
significant risk factor for OSD, thus a recommendation that he spend at least 2 to 3 months per
year away from structured sports participation is the best option for this boy.

The increases in linear growth and strength that occur during adolescence place additional
mechanical strain across apophyseal tendon origins and insertions, making these structures prone
to overuse injury. Susceptible regions include the patellar tendon insertion on the tibial tuberosity
(OSD); achilles insertion on the calcaneus (Sever disease); peroneus brevis insertion on the 5th
metatarsal (Iselin disease); and flexor/pronator tendon origin on the medial epicondyle of the
humerus (Youth baseball-related elbow injury, also known as “little league elbow”).

Overuse occurs when periods of recovery are insufficient to support the volume and intensity of
physical training. Rigorous physical activity results in microtrauma of bones and soft tissue, and
at the end of a given training session, overall strength and tissue integrity are compromised. A
subsequent period of rest and recovery is necessary for the body to repair this damage. Adequate
recovery results in a period of “supercompensation” where strength and performance are
enhanced. Improvements in strength and skill are built by these repeated cycles of training,
recovery, and supercompensation.

When subsequent training sessions occur before recovery is complete, tissue damage
accumulates and overuse injury occurs. Many young athletes and their families are focused on
training frequency and intensity, and do not understand the importance of recovery in terms of
building performance and reducing injury risk. Item C9 describes current recommendations for
recovery in child and adolescent athletes.

American Academy of Pediatrics 33


PREP ® Self-Assessment PREPSA 2021

American Academy of Pediatrics 34


PREP ® Self-Assessment PREPSA 2021
When recovery periods are inadequate, overuse injuries typically follow this 4-step progression
as microtrauma accumulates:
1. Pain after activity
2. Pain during activity that does not interfere with performance
3. Pain during activity that results in decreased performance or participation
4. Persistent pain, even during periods of rest.

Injuries identified early in this process may resolve with a period of “relative rest” when activity
levels are kept below that which aggravates symptoms. Injuries diagnosed in the later stages, or
that are recurrent, often benefit from physical therapy in addition to rest to help address any
biomechanical or other predisposing factors.

Although adding a cushioned orthotic to an athlete’s cleats might be of benefit in relieving the
symptoms of Sever disease, it would not be expected to benefit an athlete with OSD.
The coach’s recommendation that the boy focus efforts on a single sport is in direct contrast to
current recommendations regarding youth sports participation. Sport specialization in the
preadolescent markedly increases risk for overuse injury; a recent study found an almost 4-fold
increase in lower extremity overuse injuries in specialized versus nonspecialized athletes of ages
10 to 14 years.

Some athletes and families seek out additional training opportunities in an effort to augment
athletic development. General conditioning and resistance training programs may enhance
movement efficiency and assist with skill development, but are not a substitute for adequate rest
and recovery. In addition, families should be cautioned that certifications in fitness instruction
are highly variable, and most do not provide specific training on working with young athletes.
Some certification programs require only minimal online training, while others are more
rigorous. Families who seek to work with personal trainers or fitness instructors should be
encouraged to confirm that their certifications are recognized by the National Commission of
Certifying Agencies (https://www.credentialingexcellence.org/nccadirectory).

PREP Pearls
• Overuse occurs when periods of recovery are insufficient to support the volume and
intensity of physical training.
• Young athletes need 1 to 2 days per week and 2 to 3 months per year of recovery time
away from structured sports participation.

MOCA-Peds Objective
• Identify contraindications for sports participation.

ABP Content Specifications(s)


• Identify the common overuse injuries in athletes
• Plan the appropriate management of an athlete with an overuse injury

American Academy of Pediatrics 35


PREP ® Self-Assessment PREPSA 2021
Suggested Readings
• Brenner JS. Overuse injuries, overtraining, and burnout in child and adolescent
athletes. Pediatrics. 2007;119(6):1242-1245. doi: 10.1542/peds.2007-0887.
• DiFiori JP, Benjamin HJ, Brenner J, Gregory A, et al. Overuse injuries and burnout in
youth sports: a position statement from the American Medical Society for Sports
Medicine. Clin J Sport Med. 2014; 24(1):3-20. doi: 10.1097/JSM.0000000000000060.

American Academy of Pediatrics 36


PREP ® Self-Assessment PREPSA 2021
Question 10
An expectant mother is seen for a prenatal visit. Prenatal genetic testing performed on her fetus
showed karyotype 45,X. She will be meeting with a genetic counselor in the coming days.

Of the following, the MOST likely cardiac disorder to be seen in this fetus is
A. aortopulmonary window
B. coarctation of the aorta
C. tetralogy of Fallot
D. truncus arteriosus

American Academy of Pediatrics 37


PREP ® Self-Assessment PREPSA 2021
Correct Answer: B
A karyotype of 45,X, is consistent with Turner syndrome (TS). Patients with TS are known to
have an increased incidence of left-sided cardiac lesions consisting of bicuspid aortic valve and
coarctation of the aorta. Other vascular anomalies that have been noted in TS include anomalous
pulmonary venous connection and persistent left superior vena cava. Dilation of the aorta in
general, as well as dilation of other vessels such as the brachial and carotid arteries, may also be
seen. Tetralogy of Fallot, truncus arteriosus, and aortopulmonary window have not been
associated with TS. Affected patients can have resting sinus tachycardia and prolonged QT
syndrome.

In addition to the cardiac diagnoses noted above, patients with TS are at risk of experiencing
aortic dissection. Bicuspid aortic valve, coarctation of the aorta, and hypertension are risk factors
for the development of aortic dissection. An aortic dissection is very rare but is typically fatal.
The average age of dissection is 30 years, but dissection has also been seen in early childhood.
Patients and their parents should be educated regarding the signs and symptoms of aortic
dissection, should follow suggestions regarding imaging screening, and should undergo
aggressive treatment for risk factors such as hypertension. At the time of diagnosis, patients
should be referred to pediatric cardiology for an evaluation that includes echocardiography and
electrocardiography. A magnetic resonance imaging scan should also be used in the surveillance
process, once a patient is old enough to have it without sedation, unless there is reason to obtain
one before that time. Interval timing of repeat imaging studies will be dictated on the basis of
screening study findings (Item C10A).

American Academy of Pediatrics 38


PREP ® Self-Assessment PREPSA 2021

Item C10A: Cardiology evaluation and screening in Turner syndrome. Echo, echocardiography:,
MRI, magnetic resonance imaging: ECG, electrocardiography; TS, Turner syndrome.
Reprinted with permission from Loscalzo ML. Turner syndrome. Pediatr Rev. 2008;29(7):223.

Turner syndrome can affect many organ systems (Item C10B details recommended screening).
Lymphedema can present in the neonatal period; webbed neck and cystic hygroma also can be
seen. About one-third of children with TS will have kidney or collecting system problems (ie,
horseshoe kidney). External eye anomalies can consist of epicanthal folds, upslanting palpebral
fissures, ptosis, and hypertelorism. Strabismus is common. Children should be screened for
hearing loss as well. Those with TS can develop thyroid dysfunction as well as celiac disease.
These children are at risk of developing acquired melanocytic nevi. Girls with TS are of short
stature; their typical growth pattern is mild intrauterine growth restriction, slow growth in
infancy, delayed onset of childhood growth, and absence of pubertal growth spurt. Growth
deficiency of the radius, cubitus valgus, genu valgum, short fourth metacarpals, and scoliosis are
also noted. Although about one-third of girls with TS can have some progression of puberty, and
2% to 5% can have a spontaneous pregnancy, more than 90% have gonadal failure. Most girls

American Academy of Pediatrics 39


PREP ® Self-Assessment PREPSA 2021
with TS have normal cognitive and motor development and normal intelligence, although some
can have intellectual disability.

American Academy of Pediatrics 40


PREP ® Self-Assessment PREPSA 2021

American Academy of Pediatrics 41


PREP ® Self-Assessment PREPSA 2021
PREP Pearls
• Individuals with Turner syndrome can have congenital heart disease in the form of
coarctation of the aorta, bicuspid aortic valve, aortic dilation, anomalous pulmonary
venous drainage, persistent left superior vena cava, sinus tachycardia, and prolonged QT.
• Although the occurrence is rare, individuals with Turner syndrome can have aortic
dissection, which typically is fatal.
• Individuals with Turner syndrome can have involvement of multiple organ systems,
including renal, endocrine, skeletal, auditory, and visual.
• Individuals with Turner syndrome typically do not have intellectual or motor delays.

MOCA-Peds Objective
• Recognize genetic causes of congenital heart disease.

ABP Content Specifications(s)


• Recognize cardiac conditions associated with Turner syndrome

Suggested Readings
• Fuchs MM, Attenhofer Jost C, Babovic-Vuksanovic D, Connolly HM, Egbe A. Long-
term outcomes in patients with Turner syndrome: a 68-year follow up. J Am Heart Assoc.
2019;8(11):e011501. doi:10.1161/JAHA.118.011501.
• Gravholt CH, Andersen NH, Conway GS, et al; International Turner Syndrome
Consensus Group. Clinical practice guidelines for the care of girls and women with
Turner syndrome: proceedings from the 2016 Cincinnati International Turner Syndrome
Meeting. Eur J Endocrinol. 2017;177(3):G1-G70. doi:10.1530/EJE-17-0430.
• Howell JC, Backeljauw P. Turner syndrome and Noonan syndrome. In: McInerny TK,
Adam HM, Campbell DE, DeWitt TG, Foy JM, Kamat DM, eds. American Academy of
Pediatrics Textbook of Pediatric Care. 2nd ed. Elk Grove Village, IL: American
Academy of Pediatrics; 2017:2732-2744. Pediatric Care Online .
• Loscalzo ML. Turner syndrome. Pediatr Rev. 2008;29(7):219-227. doi:10.1542/pir.29-7-
219.

American Academy of Pediatrics 42


PREP ® Self-Assessment PREPSA 2021
Question 11
A 17 year old is seen for chronic nasal obstruction. She reports inability to breathe through her
nose for several months. She has a history of recurrent bronchitis, sinusitis, and chronic cough.
Review of systems is pertinent for bulky stools with a greasy film left in the toilet. The family
history is noncontributory. On physical examination, her weight is at the 80th percentile and
body mass index is at the 78th percentile. There is bilateral nasal obstruction with polypoid
material filling both antra. Her oropharynx is clear. Chest examination is notable only for a few
inspiratory crackles in the left upper chest posteriorly; no wheezing is heard. Point of maximum
cardiac impulse is in the left midclavicular line. Her abdomen is soft and there is no
hepatosplenomegaly. There is no digital clubbing.

Serum IgE values are normal and radioallergosorbent test for multiple inhalant antigens shows
negative results. Pulmonary function testing shows a mild combined restrictive and obstructive
pattern with no bronchodilator response.

Of the following, the MOST appropriate next diagnostic step is


A. CFTR mutation analysis
B. computed tomography of the sinuses
C. quantitative immunoglobulin determination
D. sweat chloride measurement

American Academy of Pediatrics 43


PREP ® Self-Assessment PREPSA 2021
Correct Answer: D
The presence of nasal polyposis, chronic cough with nonreversible obstruction on pulmonary
function testing, and signs of malabsorption make cystic fibrosis (CF) the most likely diagnosis
for this adolescent. Although genetic testing for CF (CFTR mutation analysis) is indicated after
the diagnosis is made, it is not the most appropriate diagnostic test. Measurement of chloride in a
sample of sweat obtained via pilocarpine iontophoresis remains the gold standard for the
diagnosis of CF.

It is likely that the girl will need surgical polypectomy and debridement of her sinuses.
Computed tomography of the sinuses would be appropriate before surgery, but will not lead to
the underlying diagnosis for this patient. Although she has had recurrent respiratory infections,
and deficiency or dysfunction of immunoglobulins may be implicated in recurrent
sinopulmonary infections, the presence of nasal polyps makes CF a more likely diagnosis.
Patients with CF usually have elevated quantitative immunoglobulins, representing an
appropriate immune response to recurrent infections and chronic colonization with bacterial
organisms; however, this test would not be diagnostic of CF.

In the era of newborn screening for CF, it is relatively unusual to encounter a child diagnosed
based on symptoms beyond infancy. Although newborn screening for CF was the standard in
most states by 2007-2008, it was not universal throughout the United States until 2010. Thus,
children born before 2010 may be at risk for late diagnosis of CF, depending on the standard of
care in the state in which they were born. A negative newborn screening result in an affected
patient is also a possibility. Methods of screening are not uniform across all states, therefore the
specificity and sensitivity of the tests vary. Vigilance is needed in the face of symptoms
suggestive of CF, even in those with negative newborn screening results.

PREP Pearls
• In the presence of suggestive symptoms, cystic fibrosis must be considered in all
children, even those who have undergone newborn screening.
• The best diagnostic test for cystic fibrosis is measurement of chloride in a sample of
sweat obtained with pilocarpine iontophoresis.

ABP Content Specifications(s)


• Plan the appropriate diagnostic evaluation and management of cystic fibrosis

American Academy of Pediatrics 44


PREP ® Self-Assessment PREPSA 2021
Suggested Readings
• Farrell PM, White TB, Ren CL, et al. Diagnosis of cystic fibrosis: consensus guidelines
from the Cystic Fibrosis Foundation. J Pediatr. 2017;181S:S4-15.
doi: 10.1016/j.jpeds.2016.09.064.
• Neemuchwala P, Taki M, Secord E, Nasr SZ. Newborn screening saves lives but cannot
replace the need for clinical vigilance. Case Rep Pediatr. 2018;7217326.
doi: 10.1155/2018/7217326.
• Sosnay PR, White TB, Farrell PM, et al. Diagnosis of cystic fibrosis in nonscreened
populations. J Pediatr. 2017;181S:S52-57. doi: 10.1016/j.jpeds.2016.09.068.
• Tumin D, Hayes D, Kirby SE, et al. Safety of endoscopic sinus surgery in children with
cystic fibrosis. Int J Pediatr Otorhinolaryngol. 2017;98:25-18.
doi: 10.1016/j.ijporl.2017.04.034.
• Willey-Courand DB, Marshall BC. Cystic fibrosis. In: McInerny TK, Adam HM,
Campbell DE, DeWitt TG, Foy JM, Kamat DM, eds. American Academy of Pediatrics
Textbook of Pediatric Care. 2nd ed. Itasca, IL: American Academy of Pediatrics;
2016;chap 239:1936-1947. Pediatric Care Online.

American Academy of Pediatrics 45


PREP ® Self-Assessment PREPSA 2021
Question 12
A 14-year-old adolescent boy is seen for his annual health supervision visit. He will be starting
ninth grade. His mother is concerned that he has difficulty making friends and worries that he
will become isolated in the large high school he will be attending. During the confidential
interview with his mother outside the examination room, he does not report engaging in any
high-risk behaviors. He prefers to play video games or watch television, and he has no interest in
joining a sports team. He also discloses that he has been doing research online to determine
whether it is normal that he feels attracted to both boys and girls, and he asks for his provider’s
opinion.

Of the following, the MOST appropriate statement about this adolescent’s attraction is that these
are feelings that occur during puberty and
A. he may find himself more attracted to one sex over time
B. may be related to a traumatic incident that occurred during childhood
C. should be discussed with a psychologist
D. will define his future sexual preference

American Academy of Pediatrics 46


PREP ® Self-Assessment PREPSA 2021
Correct Answer: A
Adolescence is the transitional time between childhood and adulthood that consists of biological,
cognitive, psychological, and social changes. The goal of adolescence is to become an
autonomous adult who is capable of making independent decisions and is a productive member
of society. It is also a time during which adolescents define their sense of self and create their
own personal identity. Part of creating an identity is self-concept, what a teen believes to be true
about themself in terms of goals, beliefs, and talents. The other component of creating an identity
is self-esteem, how one regards his or her own self-concept. An adolescent may view themself as
a good person and have high self-regard or may have negative views about who they are. Self-
esteem is influenced by family, friends, teachers, coaches, and other adults.

Adolescents transition through defined stages. Early adolescence, from ages 10 to 13 years, is
characterized by becoming independent, testing authority with parents and teachers, starting to
talk about future goals, and becoming aware of having sexual feelings. Middle adolescence, from
ages 14 to 17 years, is characterized by starting to move from concrete to abstract thought, being
better able to examine others’ thoughts and opinions, further exploring career goals, and often
having feelings of immortality. Middle adolescence is a time of increased risk-taking behaviors,
including drug use, sexual activity, higher rates of accidents, and more depression and suicidal
thoughts. In middle adolescence, individuals are highly influenced by their friends. Late
adolescence, from ages 17 to 21 years, is characterized by abstract thought and being able to
consider consequences of behavior, focusing on one career or vocational skill, refining morals
and beliefs, and clearly establishing sexual identity.

The boy in the vignette is in middle adolescence, which is usually a time when a teen is more
concerned about becoming independent and establishing friendships and has less interest in
family-related activities. This boy does not have many friends and is not interested in making
friends. It would be important to screen him for depression. He may be isolating himself or have
poor self-esteem owing to concern about being attracted to both sexes. It is very common for
young adolescents to develop strong feelings toward their peers of the same sex, and as they get
older they may differentiate into attraction towards the same sex, the opposite sex, or both sexes.
The role of the primary care provider is to be nonjudgmental and provide reassurance that such
feelings are part of the normal process of becoming aware of one’s sexual orientation, and that it
is normal to have attraction to individuals of both sexes while determining one’s sexual identity.
The primary care provider should inquire about sexual orientation and behaviors without
applying any labels such as “gay,” “lesbian,” “bisexual,” or “straight.” To inquire about abuse or
other traumatic experiences, discuss the need for therapy, and make statements about the
adolescent’s future sexual preferences cast a stigma that these thoughts are abnormal. This
negative interaction may potentially lower this adolescent’s self-esteem and cause him to
internalize his thoughts and feelings, which may lead to depression and suicidal ideation.

American Academy of Pediatrics 47


PREP ® Self-Assessment PREPSA 2021
PREP Pearls
• Identity is a combination of self-concept and self-esteem.
• Adolescents establish their identity as they transition through the stages of early, middle,
and late adolescence.
• The role of the primary care provider is to be nonjudgmental and provide reassurance and
support as an adolescent goes through the process of becoming aware of his or her sexual
orientation.

ABP Content Specifications(s)


• Recognize the risks associated with adolescents who do not identify with any peers
(“loners”)
• Understand features associated with an adolescent’s search for identity

Suggested Readings
• Alderman EM, Breuner CC; AAP Committee on Adolescence. Unique needs of the
adolescent. Pediatrics. 2019;144(6):e20193150. doi:10.1542/peds.2019-3150.
• Kreipe RE. Challenges of health care delivery to adolescents. In: McInerny TK, Adam
HM, Campbell DE, DeWitt TG, Foy JM, Kamat DM, eds. American Academy of
Pediatrics Textbook of Pediatric Care. 2nd ed. Elk Grove Village, IL: American
Academy of Pediatrics; 2017:1135-1140. Pediatric Care Online .
• Sanders RA. Adolescent psychosocial, social, and cognitive development. Pediatr Rev.
2013; 34(8):354-358. doi:10.1542/pir.34-8-354.
• Sherer S, Radzik M. Psychosocial development in normal adolescents and young adults.
In: Neinstein LS, Katzman DK, Callahan ST, Gordon CM, Joffe A, Rickert VI,
eds. Neinstein’s Adolescent and Young Adult Health Care: A Practical Guide. 6th ed.
Philadelphia, PA: Wolters Kluwer; 2016:38-42.

American Academy of Pediatrics 48


PREP ® Self-Assessment PREPSA 2021
Question 13
A 12-year-old previously healthy, developmentally normal girl is seen in the office after an
emergency department visit for a first-time seizure. The previous evening her mother heard a
thud and found her daughter on the bedroom floor shaking, with her eyes open and rolled back.
This lasted for about 2 minutes after which she fell asleep. Her mother called 911 immediately
after the shaking ended. On arrival at the emergency department, the girl’s physical examination
findings and laboratory results were normal, including a complete blood cell count, bedside
glucose level, comprehensive metabolic panel, and serum and urine toxicology screen. She was
discharged after several hours of observation.

In the office, the girl and her mother report no prior history of paroxysmal events, family history
of seizures or epilepsy, and no recent medication changes. The girl denies drug or alcohol use.
Findings of her physical examination, including vital signs, and neurologic examination are
normal

Of the following, the BEST next step in this girl’s diagnostic evaluation would be
A. computed tomography of the head
B. electroencephalography
C. electrocardiography
D. magnetic resonance imaging of the brain

American Academy of Pediatrics 49


PREP ® Self-Assessment PREPSA 2021

Correct Answer: B
Electroencephalography (EEG) is the best next step in this girl’s diagnostic evaluation for new-
onset seizure. Electroencephalography is an important electrophysiologic test that can provide
evidence in support of a seizure tendency or a specific epilepsy syndrome with demonstration of
epileptiform discharges interictally or with provoking maneuvers such as hyperventilation or
photic stimulation. In addition, EEG can demonstrate nonspecific changes in brain
electrophysiology such as slowing or background asymmetry, which could suggest a focal
structural or physiologic change warranting further investigation. A normal EEG finding does
not exclude the diagnosis of seizure, which is ultimately a clinical diagnosis, but when abnormal,
this test can support the diagnosis.

Seizures are common in childhood, occurring in approximately 1% of children, with the highest
incidence occurring in the first year after birth. Obtaining a detailed history is critical in the
evaluation of seizure-like events. The goal of the history is to: 1) determine the type of the event:
seizure or nonseizure (eg, syncope, breath-holding, cardiac arrhythmias, or paroxysmal
nonepileptic event), and 2) categorize the type, if the event is consistent with a seizure, to guide
evaluation and management. A child with a new-onset seizure should receive a thorough
neurologic examination in addition to a general physical examination aimed at assessing for the
presence of systemic illness (eg, meningeal signs) or inherited disorder (eg, tuberous sclerosis).
Key history and physical examination components are summarized in Item C13. Following
initial stabilization of the child’s condition, laboratory investigations are often performed at the
time of the seizure to determine reversible or triggering causes. Blood glucose and serum
electrolyte levels, toxicology, and complete blood cell counts are often obtained, as was done for
the girl in the vignette. Further laboratory testing may be warranted by the clinical situation.

American Academy of Pediatrics 50


PREP ® Self-Assessment PREPSA 2021

American Academy of Pediatrics 51


PREP ® Self-Assessment PREPSA 2021

Electrocardiography is important if cardiac arrhythmias are a suspected cause of a paroxysmal


event. The timing and type of neuroimaging recommended, if any, is dependent on the clinical
scenario. For children with status epilepticus, focal seizure, postictal paralysis, lack of return to
baseline condition, or newly abnormal neurologic examination, computed tomography of the
head may be appropriate to assess for acute neurologic conditions such as hemorrhage, mass, or
hydrocephalus. In such cases, brain magnetic resonance imaging (MRI) may be considered; MRI
provides a detailed structural image, allowing for evaluation of brain malformations, mass
lesions, ischemic stroke, or demyelination.

PREP Pearls
• In first-time unprovoked seizure cases, electroencephalography is a key part of the
diagnostic evaluation for evidence of abnormalities in support of a seizure tendency
(epilepsy), specific epilepsy syndrome, and to guide further diagnostic and therapeutic
management.
• Seizure is a clinical diagnosis; the history and physical examination are critical for
diagnosis and appropriate care.

ABP Content Specifications(s)


• Recognize factors associated with an increased risk of a seizure disorder
• Plan the appropriate management of a first seizure
• Differentiate the features of epileptic seizures from those of paroxysmal non-epileptic
events

Suggested Readings
• Roddy SM, McBride MC. Seizure disorders. In: McInerny TK, Adam HM, Campbell
DE, DeWitt TG, Foy JM, Kamat DM, eds. In: American Academy of Pediatrics Textbook
of Pediatric Care. 2nd ed. Itasca, IL: American Academy of Pediatrics; 2017;chap
329:2620-2627. Pediatric Care Online.
• Sidhu R, Velayudam K, Barnes G. Pediatric seizures. Pediatr Rev. 2013;34:333-342.
doi: 10.1542/pir.34-8-333.

American Academy of Pediatrics 52


PREP ® Self-Assessment PREPSA 2021
Question 14
A 6-month-old male infant who is limp and exhibiting obtundation in the morning is brought to
the emergency department. Bedside blood analysis reveals a significant metabolic acidosis,
including an elevated lactate level and a capillary blood glucose level of 35 mg/dL (1.9 mmol/L)
and 3+ ketones in the urine. His newborn screening results were normal. His parents report that
he only recently has been able to sleep through the night without waking for a feeding and that
he has had a few episodes of morning lethargy that improved after his first morning feeding. The
infant appears small and nearly comatose. He has chubby cheeks, a short nose, and a relatively
small chin. He has a temperature of 37.8°C, a heart rate of 155 beats/min, a respiratory rate of 15
breaths/min, and a blood pressure of 75/45 mm Hg. Cardiac examination reveals no murmur and
a normal S1 and S2. The liver edge is palpable 3 cm below the right costal margin. The
remainder of the physical examination findings are unremarkable.

Of the following, the MOST likely diagnosis is


A. fatty acid oxidation disorder
B. glycogen storage disorder
C. mitochondrial disorder
D. organic acid disorder

American Academy of Pediatrics 53


PREP ® Self-Assessment PREPSA 2021
Correct Answer: B
The infant in the vignette, with obtundation after prolonged overnight fasting and hypoglycemia
with lactic acidosis and ketosis, most likely has a glycogen storage disorder (GSD). Glycogen
storage disease type 1a (also known as von Gierke disease) is caused by a deficiency in glucose-
6-phosphatase. This type often presents in infancy and results in hepatomegaly, growth failure,
and recurrent episodes of hypoglycemia with ketosis. Although some neonates with glycogen
storage disease present with severe hypoglycemia, it is more common for infants aged 3 to 4
months to manifest hepatomegaly, lactic acidosis, hypoglycemic seizures, or a combination of
these. Additional laboratory abnormalities include elevated uric acid levels and hyperlipidemia.
The physical findings of chubby cheeks, a short nose, and a relatively small chin are nonspecific
but can be found in children with GSD. The diagnosis is made by means of genetic analysis
identifying pathologic variants in specific genes, including G6PC, SLC37A4, or both. These
genes encode for glucose-6-phosphatase activity and glucose-6-phosphate
exchanger SLC37A4 activity, respectively.

Inborn errors of metabolism, in general, are insidious and involve abnormalities in the processing
of carbohydrate, lipid, or protein that result in a toxic accumulation of energy precursors or
metabolites or in a deficiency of nutritional elements. Because early intervention and appropriate
therapy are required to ensure optimal outcomes, clinicians must remain aware of the physical
findings and characteristic histories of different inborn errors of metabolism.
Metabolic disorders involving protein include amino acid, organic acid, and urea cycle disorders.
When amino acids are metabolized, deamination removes the amine group and leaves ammonia
and an organic acid. Disorders of the first step in amino acid metabolism result in accumulation
of unmetabolized precursor amino acids. If the abnormality occurs further along the processing
pathway, organic acids that are unable to be metabolized will accumulate. Finally, if the child
has a urea cycle defect, ammonia will not be metabolized to urea, resulting in hyperammonemia.
Patients with organic and amino acid disorders often experience neonatal lethargy, vomiting,
coma, strokes, and, if unrecognized, death. Patients may have elevations of specific amino acids,
organic acids, or ammonia in the serum or urine.

Disorders of lipid metabolism, or fatty acid oxidation disorders, result from abnormal β-
oxidation of fatty acids. A hallmark of fatty acid oxidation disorders is the development of
hypoglycemia without the concurrent development of ketones, which normally results from the
breakdown of fats. Hypoglycemia generally occurs during a period of fasting when glycogen and
gluconeogenesis normally ensure maintenance of serum glucose levels. To produce glucose from
gluconeogenesis, fats must be oxidized; thus, patients with fatty acid oxidation defects cannot
metabolize fatty acids to produce ketone bodies and cannot release stored energy from glycogen.
Presenting signs of fatty acid oxidation disorders may include nonketotic hypoglycemia,
seizures, rhabdomyolysis, cardiomyopathy, liver dysfunction, and sudden infant death.
Mitochondrial disorders result from mutations in the mitochondrial genome required for
mitochondrial function. This leads to decreased production of energy in the form of adenosine
triphosphate and intracellular acidosis. Metabolically active tissues, including the brain, skeletal
muscles, and cardiac muscles, are most affected. Diagnosis is made via fresh muscle biopsy
microscopic analysis revealing ragged red fibers. Patients develop nonspecific lactic acidosis and
concurrent organ-specific findings such as strokes, seizures, cardiac conduction abnormalities,
hypotonia, or weakness.
American Academy of Pediatrics 54
PREP ® Self-Assessment PREPSA 2021

The 6-month-old in the vignette does not have characteristic neurological features of a
mitochondrial disorder, including hypotonia. Although lactic acidosis is present in mitochondrial
disorders, the other laboratory findings and clinical history are more consistent with GSD.
The infant in the vignette has ketonuria, indicating that the β-oxidation of fatty acids is normal,
making a fatty acid oxidation disorder an incorrect choice.
Organic acid disorders generally present in the neonatal period.

PREP Pearls
• Inborn errors of metabolism are generally insidious; they involve abnormalities of
carbohydrate, lipid, or protein processing that result in a toxic accumulation of energy
precursors or metabolites or in a deficiency of nutritional elements.
• Glycogen storage disease often presents in infancy and results in hepatomegaly, growth
failure, and recurrent episodes of hypoglycemia with ketosis.
• Children with organic and amino acid disorders often experience neonatal lethargy,
vomiting, coma, strokes, and, if unrecognized, even death.
• A hallmark of fatty acid oxidation disorders is the development of hypoglycemia without
the concurrent development of ketones.

ABP Content Specifications(s)


• Plan the evaluation of a patient with suspected metabolic disease who is acidotic
• Plan the evaluation of a patient with suspected metabolic disease who is comatose
• Plan the evaluation of a patient with suspected metabolic disease who has hypoglycemia,
and manage appropriately

Suggested Readings
• Bali DS, Chen YT, Austin S, Goldstein JL. Glycogen storage disease type I. In: Adam
MP, Ardinger HH, Pagon RA, et al, eds. GeneReviews. Seattle, WA: University of
Washington, Seattle; 1993–2016. 2006 Apr 19 (updated 2016 Aug 25).
• Rice GM, Steiner RD. Inborn errors of metabolism (metabolic disorders). Pediatr Rev.
2016;37(1):3-15. doi:10.1542/pir.2014-0122.
• Rios A, Adams DJ. Specific congenital metabolic diseases. In: McInerny TK, Adam HM,
Campbell DE, DeWitt TG, Foy JM, Kamat DM, eds. American Academy of Pediatrics
Textbook of Pediatric Care. 2nd ed. Elk Grove Village, IL: American Academy of
Pediatrics; 2017:938-962. Pediatric Care Online.

American Academy of Pediatrics 55


PREP ® Self-Assessment PREPSA 2021
Question 15
A 9-month-old male infant is seen for a health supervision visit. He is pulling to a stand,
cruising, and taking one or two independent steps. His parents would like to encourage his gross
motor development. They seek advice on the use of infant walkers.

Of the following, the MOST accurate response is that this device


A. accelerates the development of unassisted walking
B. is not recommended
C. is safe with parental supervision
D. should be used with a helmet

American Academy of Pediatrics 56


PREP ® Self-Assessment PREPSA 2021
Correct Answer: B
Infant walkers are devices that support infants before the development of ambulation, providing
increased mobility, and thereby increasing exposure to hazardous situations, such as stairs, hot or
sharp objects, poisons, and water. Despite recent declines in infant walker–related injuries, these
devices continue to be a significant source of child injury and should be avoided entirely. Serious
injuries include skull fractures, brain and neck injuries, burns, drowning, ingestion of toxins, and
even fatality.

Despite parents’ perceptions that infant walkers accelerate the development of unassisted
walking, no data support such a benefit. In fact, one study showed that infants aged 6 to 15
months who used infant walkers demonstrated delayed motor development and scored lower on
the Bayley scales of mental and motor development. Furthermore, adult supervision has not been
shown to prevent walker-related injuries. One study demonstrated that at the time they
experienced a walker-related injury, 78% of infants and toddlers were supervised. Although
helmets are recommended when roller skating, skateboarding, and riding a bicycle, scooter, or
motorbike, helmets do not prevent all walker-related injuries. Safer alternatives to infant walkers
include stationary play stations and play yards.

PREP Pearls
• Infant walker–related injuries are a significant source of child injury.
• Despite parents’ perceptions that infant walkers accelerate the development of unassisted
walking, they should not be used.
• Adult supervision has not been shown to prevent walker-related injuries.

ABP Content Specifications(s)


• Counsel parents and children regarding safety and protection with the use of recreational
equipment
• Counsel parents regarding stairway safety

Suggested Readings
• American Academy of Pediatrics, Committee on Injury and Poison Prevention. Injuries
associated with infant walkers. Pediatrics. 2001;108(3):790-792.
doi:10.1542/peds.108.3.790.
• Siegel AC, Burton RV. Effects of baby walkers on motor and mental development in
human infants. J Dev Behav Pediatr. 1999;20(5):355-361. doi:10.1097/00004703-
199910000-00010.
• Sims A, Chounthirath T, Yang J, Hodges NL, Smith GA. Infant walker–related injuries in
the United States. Pediatrics. 2018;142(4):e20174332. doi:10.1542/peds.2017-4332.
• Smith GA, Bowman MJ, Luria JW, Shields BJ. Baby walker-related injuries continue
despite warning labels and public education. Pediatrics. 1997;100(2):e1.
doi:10.1542/peds.100.2.e1.

American Academy of Pediatrics 57


PREP ® Self-Assessment PREPSA 2021
Question 16
A female neonate born at term is evaluated in the neonatal intensive care unit. The pregnancy
was complicated by maternal gestational diabetes and alcohol exposure. Prenatal
ultrasonography at 20 weeks showed intrauterine growth retardation. The neonate is presently
intubated. Her birth weight is 2,200 g. She has a small jaw, clenched hands with overlapping
fingers (Item Q16), rocker-bottom feet, a short sternum, and hypoplastic nails.
Echocardiography shows a ventricular septal defect. Renal ultrasonography shows a horseshoe
kidney.

Item Q16: Neonate’s hand.


Reprinted with permission from Crotwell PL, Hoyme HE. NeoReviews. 2012;13 (1):e32.

Of the following, the MOST likely diagnosis is


A. fetal alcohol spectrum disorder
B. trisomy 13
C. trisomy 18
D. trisomy 21

American Academy of Pediatrics 58


PREP ® Self-Assessment PREPSA 2021
Correct Answer: C
The neonate in the vignette has trisomy 18. Trisomy 18 is characterized by intrauterine growth
retardation and by congenital cardiac, ocular, and renal anomalies. Dysmorphic features include
small jaw, clenched hands with overlapping fingers (index overlapping the third, fifth
overlapping the fourth), short sternum, and hypoplastic nails. The majority of pregnancies
affected by trisomy 18 result in fetal loss during the first trimester. Infants who make it to term
have a high rate of infant mortality, with one-half not surviving 1 week and only 5% to 10%
living beyond the first year after birth. Care of neonates with trisomy 18 involves a
multidisciplinary approach with emphasis on growth and feeding, development, respiratory
requirements, and specialty-specific care according to the congenital anomalies. The most
frequent causes of death include central apnea, complications of cardiac malformations, and
respiratory failure related to obstruction and aspiration.

Fetal alcohol spectrum disorder is manifested in characteristic facial features—small palpebral


fissures, smooth philtrum, and a thin upper lip—in addition to failure to thrive and being small
for gestational age. Although congenital anomalies can be seen in fetal alcohol spectrum
disorder, the dysmorphic features described in the vignette are classic characteristics of trisomy
18. Intrauterine growth retardation, small size for gestational age, and congenital cardiac and
renal defects can be seen in neonates with trisomy 13 or trisomy 18. Neonates with trisomy 13
also have a scalp defect called "cutis aplasia" and midline defects such as holoprosencephaly and
cleft lip, cleft palate, or both. Clenched hands with overlapping fingers and rocker-bottom feet
can be seen in trisomy 13, but they are more common in trisomy 18. Trisomy 21 manifests in a
distinctive facial profile with up-slanted palpebral fissures, midface hypoplasia, and epicanthal
folds. Other characteristic features are hypotonia, congenital heart defects, single transverse
palmar crease, and a wide space between the first and second toes. See Item C16 for a
comparison of trisomy 13, trisomy 18, trisomy 21, and fetal alcohol spectrum disorder.

American Academy of Pediatrics 59


PREP ® Self-Assessment PREPSA 2021

American Academy of Pediatrics 60


PREP ® Self-Assessment PREPSA 2021

PREP Pearls
• Trisomy 18 is characterized by distinctive dysmorphic features: clenched hands with
overlapping fingers, rocker-bottom feet, and short sternum.
• Trisomy 13 and trisomy 18 can both manifest in failure to thrive and congenital
anomalies.
• Trisomy 13 is characterized by midline and scalp defects and rocker-bottom feet.

MOCA-Peds Objective
• Recognize genetic causes of congenital heart disease.

ABP Content Specifications(s)


• Recognize the clinical features associated with trisomy 18

Suggested Readings
• Cereda A, Carey JC. The trisomy 18 syndrome. Orphanet J Rare Dis. 2012;7:81.
doi:10.1186/1750-1172-7-81.
• Levy PA, Marion M. Trisomies. Pediatr Rev. 2018;39(2):104-106. doi:10.1542/pir.2016-
0198.
• Rosen O, Marion RW, Samonich J. Common congenital anomalies. In: McInerny TK,
Adam HM, Campbell DE, DeWitt TG, Foy JM, Kamat DM, eds. American Academy of
Pediatrics Textbook of Pediatric Care. 2nd ed. Elk Grove Village, IL: American
Academy of Pediatrics; 2017:828-836. Pediatric Care Online .
• Thackray H, Tifft C. Fetal alcohol syndrome. Pediatr Rev. 2001;22(2):47-55.
doi:10.1542/pir.22-2-47.

American Academy of Pediatrics 61


PREP ® Self-Assessment PREPSA 2021
Question 17
A 4-year-old girl with short bowel syndrome and on long-term total parenteral nutrition is seen
in the emergency department for evaluation of fever, chills, redness, and swelling within 2 cm of
her central venous catheter site. She appears ill and has a temperature of 39.1°C, tachycardia, and
hypotension. There is marked erythema and induration surrounding the exit site of the central
venous catheter. Blood cultures are obtained from the catheter and a peripheral site, and
antibiotic therapy with vancomycin and ceftazidime is initiated. Within 12 hours, both central
and peripheral blood cultures grow gram-positive cocci in clusters.

Of the following, the BEST next step in management is


A. addition of gentamicin and rifampin
B. antibiotic lock therapy
C. echocardiography
D. removal of the central venous catheter

American Academy of Pediatrics 62


PREP ® Self-Assessment PREPSA 2021
Correct Answer: D
The child in the vignette has short gut syndrome and is dependent on total parenteral nutrition
delivered by an indwelling central venous catheter (CVC). She is experiencing fever,
hypotension, erythema at the catheter exit site, and blood cultures with gram-positive cocci in
clusters. This scenario is suggestive of a diagnosis of central line-associated bloodstream
infection (CLABSI) associated with an exit site infection. The best next step in the management
of this girl’s condition is removal of the CVC.

The term CLABSI is used by the Centers for Disease Control and Prevention National
Healthcare Safety Network for surveillance purposes. Catheter-related bloodstream infection is
defined as bacteremia or fungemia in which an intravascular catheter is identified as the source
of infection. Positive blood culture drawn simultaneously from the CVC and peripheral blood
growing the same microorganism is indicative of CLABSI. In the United States, CLABSI
represents the most common device-associated infection among hospitalized children.
The common pathogens causing intravascular catheter-related infections are gram-positive
bacteria, including Staphylococcus species (eg, coagulase-negative Staphylococcus and S
aureus, including methicillin-resistant S aureus) and enterococci, followed by gram-negative
bacilli (eg, Enterobacter spp, Pseudomonas aeruginosa, Klebsiella pneumoniae, Escherichia
coli) and Candida spp. Other organisms implicated
include Corynebacterium spp, Bacillus spp, Serratia
marcescens, Acinetobacter spp, Citrobacter spp, and nontuberculous mycobacteria.

The origin of CLABSI may include inoculation at the time of catheter insertion (endogenous host
flora or environmental source), inoculation during catheter manipulation (break in antiseptic
technique), hematogenous seeding (transient bacteremia from translocation of organisms from
colonized sites, breach in gastrointestinal mucosa) or extension of local infection. Several types
of catheter-related infection may occur including exit site, subcutaneous tunnel or pocket, or in
the bloodstream.

Risk factors for CLABSI include catheter-related factors (eg, placement site, manipulations,
duration of catheterization, development of thrombosis, tunneled vs not), type of infusants (eg,
parenteral nutrition, lipids, blood transfusion), and host factors (eg, skin integrity, skin flora,
immunocompromised host).

The clinical manifestations of CLABSI include fever and local site infection. Fever may be
absent with localized exit site, pocket, or tunnel infection. The exit site of the catheter may
demonstrate signs of inflammation or purulence; however, normal appearance of the exit site
does not exclude catheter-associated local or systemic infection. Thrombosis increases the risk of
developing infection.

The diagnosis of CLABSI is made by obtaining blood cultures from the CVC and a peripheral
site. Positive catheter tip cultures suggest that the infection started with the catheter; however,
the tip frequently becomes contaminated with skin flora when being removed, these cultures
overpredict infection, and they are not useful in asymptomatic patients.

American Academy of Pediatrics 63


PREP ® Self-Assessment PREPSA 2021
Complications of catheter-related bloodstream infection include sepsis, endocarditis, thrombosis
and thrombophlebitis, and metastatic infection involving many organs. Overall, case fatality rates
for CLABSI range from 10% to 25%, with rates up to 50% in critically ill patients and neonates.
Empiric antimicrobial therapy for children with suspected CLABSI must include an agent with
excellent activity against gram-positive bacteria (eg, vancomycin, oxacillin) in combination with
an agent active against gram-negative bacteria (eg, ceftazidime or cefepime). The Infectious
Disease Society of America has published guidelines for diagnosis and management of
CLABSI.

The need for removal of the catheter in patients with CLABSI is determined by many factors.
Removal of the CVC is recommended if
• the catheter is no longer needed
• exit site, tunnel, or pocket infection is found
• the patient is critically ill (eg, hypotension)
• infection caused by certain microorganisms (eg, candida, atypical mycobacteria, S
aureus) is present
• Bacteremia fails to clear in 48 to 72 hours
• underlying valvular heart disease (eg, increased risk of developing endocarditis) is
present
• known endocarditis
• development of metastatic infection
• the finding of septic thrombophlebitis

Treating CLABSI with antibiotics administered through the infected CVL may at times be
successful according to the pathogen identified, but ongoing bacteremia increases the risk of
complications and relapse rate up to 20% have been reported. Emergence of resistant flora with
administration of prolonged, multiple courses of broad-spectrum antibiotics is a concern.
Antibiotic lock therapy for tunneled CVCs, in which antibiotics are administered into the
catheter and allowed to remain in the catheter for varied lengths of time, has been successful in
adults to salvage the catheter, but experience with this therapy is limited in children. Addition of
rifampin may also be considered in patients with device-associated invasive infections,
especially when removal of the device is not feasible. Addition of gentamicin and rifampin to a
vancomycin regimen would be recommended for prosthetic valve endocarditis due to
methicillin-resistant S aureus, but the preferred management for CLABSI with exit site infection
in the ill-appearing patient described in the vignette is removal of the CVC. Echocardiogram to
exclude endocarditis may be indicated as part of the evaluation of CLABSI if there is persistent
bacteremia while receiving appropriate treatment, new murmur noted on cardiac examination, or
presence of underlying congenital heart defect.

Further information regarding education, training, and appropriate staffing of health care
personnel for prevention of CLABSI can be found in the guidelines (O’Grady et al).

American Academy of Pediatrics 64


PREP ® Self-Assessment PREPSA 2021
PREP Pearls
• Catheter-related bloodstream infection is defined as bacteremia or fungemia in which the
catheter is identified as the source of infection.
• The common pathogens causing intravascular catheter-related infections
include Staphylococcus species, enterococci, gram-negative bacilli, and Candida spp.
• In patients with catheter-related infection, removal of the central venous catheter is
recommended in patients who are critically ill (eg, hypotension).

ABP Content Specifications(s)


• Plan the appropriate treatment of a patient with an intravenous catheter-associated
infection

Suggested Readings
• Mermel LA, Allon M, Bouza E, et al. Clinical practice guidelines for the diagnosis and
management of intravascular catheter-related infection: 2009 Update by the Infectious
Diseases Society of America. Clin Infect Dis. 2009;49(1):1-45. doi:10.1086/599376.
• O'Grady NP, Alexander M, Burns LA, et al; Healthcare Infection Control Practices
Advisory Committee (HICPAC). Guidelines for the prevention of intravascular catheter-
related infections. Clin Infect Dis. 2011;52(9):e162-e193. doi:10.1093/cid/cir257.
• Sammons JS, Gerber JS. Clinical syndromes of device-associated infections. In: Long
SS, Prober CG, Fisher M, eds. Principles and Practice of Pediatric Infectious
Diseases. 5th ed. Philadelphia, PA: Elsevier; 2018:600-612.

American Academy of Pediatrics 65


PREP ® Self-Assessment PREPSA 2021
Question 18
A 3-year-old boy is seen for a health supervision visit. His parents express concern that their son
is not speaking at the same level as his 3-year-old cousin. They state that he can "understand
everything," but his language does not include combinations of more than 2 words. His parents
understand approximately half of what he says. The boy enjoys running around with the other
children at his preschool. He can draw lines but not circles and can stack up to 4 blocks. He
needs assistance with removing his clothing. The boy is friendly, makes good eye contact, and
smiles on being greeted. He looks confused when asked to hand over his jacket, but does so
willingly when the jacket is pointed to and a hand held out for it. He looks to his parents for
reassurance during the physical examination. The boy’s growth has been appropriate. His vital
signs and physical examination findings are unremarkable. When offered a choice of stickers, he
points to the one he wants and says "Thank you."

Of the following, the BEST next management step for this boy is to
A. administer a general developmental screen
B. order a chromosomal microarray
C. perform an office-based hearing screen
D. refer the boy to state early intervention services

American Academy of Pediatrics 66


PREP ® Self-Assessment PREPSA 2021
Correct Answer: A
The parents of the child described in the vignette are concerned about speech or language delay.
However, it is important to consider that the differential diagnosis of a child with speech or
language delay includes global developmental delay, autism spectrum disorder, and hearing
impairment. A general developmental screen should be administered to determine if he has any
additional delays in development and if a full developmental evaluation is needed.

This 3-year-old boy should have expressive language of at least 3-word sentences, receptive
language to allow following of 3-step commands, and speech that is 75% understandable to
strangers. He should be able to draw a circle, stack at least 6 blocks, undress, and put on some
clothing items. Item C18A lists developmental milestones for 3 year olds. Item C18B lists
specific language developmental milestones. This boy’s history indicates delays in multiple areas
of development. If a full developmental evaluation confirms global developmental delay, a
chromosomal microarray and DNA analysis for fragile X are then indicated to look for possible
causes.

American Academy of Pediatrics 67


PREP ® Self-Assessment PREPSA 2021

A formal audiology evaluation is needed for this child with significant delays in language
development. An office-based hearing screen is not sufficient because of the risk for false-
negative results; milder cases of clinically significant hearing impairment may not be identified.
This boy is in need of developmental therapies but has already passed his third birthday and no
longer qualifies for state early intervention services (0-3 years). Services may now be accessed
through the boy’s school district or his insurance.

Language delay affects approximately 10% to 15% of preschool children. About half of children
with delayed language development at age 2 years catch up to their peers by age 3 years.
Children with both receptive and expressive language delays are more likely to have persistent
problems in language development. Those whose language delay resolves are at risk for
language-based learning problems (eg, reading). A language disorder occurs when language
delay persists to school age, limits learning or communication, is severe, or has an atypical
American Academy of Pediatrics 68
PREP ® Self-Assessment PREPSA 2021
pattern. Language disorders can be primary (eg, because of a genetic contribution) or secondary
(eg, such as part of an autism spectrum disorder). They can be because of limited developmental
enrichment of the child’s environment, hearing impairment, neurologic problems (eg, stroke,
seizure disorder), or oral dysfunction (eg, cleft palate).

In a child with a language disorder, obtaining a good developmental history and performing
close observation of the child’s verbal and nonverbal communication are essential. The language
history should include questions about the child’s expressive language (what the child says),
receptive language (what the child understands), and speech (articulation, intelligibility). It is
important to ask about other areas of development (ie, cognition, fine motor, gross motor, social,
adaptive) as well as any unusual behaviors that might indicate an autism spectrum disorder (eg,
repetitive, stereotypic, sensory behaviors). A physical and neurologic examination should
evaluate for possible apraxia or other genetic, developmental, or physical conditions such as
autism, cerebral palsy, or neuromotor disorder.

Formal screening with a standardized tool, such as the Ages and Stages Questionnaire or Parents
Evaluation of Developmental Status, should be conducted at the recommended health
supervision visits of 9 months, 18 months, 24 months, or 30 months, or whenever there is a
developmental concern. Formal screening for autism spectrum disorder should also be conducted
at 18 months and 24 months. Standardized developmental and autism screening can determine
the need for further evaluation of language by a speech pathologist, and when multiple areas of
development are delayed, by a developmental psychologist, developmental-behavioral
pediatrician, or pediatric neurologist. Referral should also be made to an audiologist, and the
child should be referred to early intervention if younger than 3 years or to special education if
older than 3 years.

PREP Pearls
• For a child with speech or language delay, a general developmental screen should be
administered to determine if he/she has any additional delays in development and if a full
developmental evaluation is needed.
• A formal audiology evaluation is needed for a child with significant delays in language
development. An office-based hearing screen is not sufficient because of the risk for
false-negative results; milder cases of clinically significant hearing impairment may not
be identified.
• Children whose language delay resolves are at risk for language-based learning problems
(eg, reading).

ABP Content Specifications(s)


• Plan the appropriate evaluation of language disorders in patients of various ages
• Identify the various etiologies of delayed language development

American Academy of Pediatrics 69


PREP ® Self-Assessment PREPSA 2021
Suggested Readings
• Feldman HM, Youssef JN. Language delays. In: Augustyn M, Zuckerman B, eds.
Zuckerman Parker Handbook of Developmental and Behavioral Pediatrics for Primary
Care. 4th ed. Philadelphia, PA: Wolters Kluwer; 2019:307-316.
• Feldman HM. How young children learn language and speech. Pediatr
Rev. 2019;40(8):398-410. doi: 10.1542/pir.2017-0325.
• Lipkin PH, Macias MM, Council on Children with Disabilities, Section on
Developmental and Behavioral Pediatrics. Promoting Optimal Development: Identifying
Infants and Young Children with Developmental Disorders Through Developmental
Surveillance and Screening. Pediatrics. 2020;145(1):e20193449.
doi: https://doi.org/10.1542/peds.2019-3449. 398-410.
• Macias MM, LaRosa AC, Mittal S. Speech and language development and disorders. In:
Voigt RG, Macias MM, Myers SM, Tapia CD, eds. Developmental and Behavioral
Pediatrics. 2nd ed. Itasca, IL: American Academy of Pediatrics; 2018:347-366.
• Scharf RJ, Scharf GJ, Stroustrup A. Developmental milestones. Pediatr
Rev. 2016;37(1):25-38. doi: 10.1542/pir.2014-0103.

American Academy of Pediatrics 70


PREP ® Self-Assessment PREPSA 2021
Question 19
A mother requests early discharge of her neonate, who is 36 hours of age, so she can be home
with her 2-year-old daughter and husband. Her mother is also staying at her home to help. The
boy was born at 38 weeks 6 days in a normal spontaneous vaginal delivery without any
complications after an uneventful pregnancy. He has been nursing well, voided four times, and
passed stools three times. His temperature is 37.2°C, his heart rate is 120 beats/min, his
respiratory rate is 40 breaths/min, and his oxygen saturation is 98% on room air. His physical
examination findings are normal.

Of the following, the BEST next management step for this neonate’s care is to
A. discharge him at 72 hours of age
B. discharge now with follow-up within 48 hours
C. obtain a complete blood cell count
D. obtain a serum bilirubin level

American Academy of Pediatrics 71


PREP ® Self-Assessment PREPSA 2021
Correct Answer: B
The neonate in the vignette meets all the criteria necessary for an early discharge (before 48
hours of age). Neonates discharged early should be seen by a care provider within 48 hours. If
newborn testing is completed before 24 hours, a repeat newborn screening should be obtained at
that follow-up visit.

Establishing a discharge plan after delivery includes many factors, including social support,
ability to follow up with a pediatric provider within 48 hours of discharge, and medical stability
of both the mother and infant. Parents are often less comfortable with newborn care when the
infant is their firstborn child. Mothers should have support from a spouse or partner, as well as
from another family member or close friend. Coordination of a follow-up appointment, including
transportation, should be arranged before discharge.

Neonates born to mothers with chronic medical conditions, excessive maternal bleeding, or
complications from delivery, including the need for instrument assistance or cesarean delivery,
should not be considered for early discharge.

As neonates transition from intrauterine to extrauterine physiology, cardiopulmonary


complications can arise, usually within 12 hours of delivery. Other concerning conditions that
can present in the newborn period include infection, ductal-dependent cardiac lesions, bowel
obstructions, and feeding difficulties. A list of reassuring criteria, which place the neonate in a
low-risk category, should be used when considering discharge for a term neonate (gestational age
37 0/7-41 6/7 weeks):
• Normal physical examination findings, including vital signs for 12 hours before discharge
o Temperature 36.5°C to 37.4°C while lying in an open crib
o Respiratory rate less than 60 breaths/min without any increased work of breathing
o Heart rate 100 to 190 beats/min while awake and more than 70 beats/min while
asleep, with normal circulation and appropriate response to stimuli
• One stool and normal urination
• Two feedings with good latch and suck/swallow pattern, with one feeding observed
• No excessive bleeding for 2 hours after a circumcision
• Appropriate treatment plan for hyperbilirubinemia, if present; negative result on Coombs
test
• Reassuring sepsis work-up if risk factors are identified
• Normal maternal laboratory findings, including negative results for syphilis, hepatitis B
surface antigen, and human immunodeficiency virus
• Receipt of intramuscular vitamin K

The neonate in the vignette was delivered vaginally without complications, meets the criteria for
early discharge, and can safely be discharged before 72 hours of age. A complete blood cell
count is not indicated because he has no risk factors for early sepsis. This neonate does not have
any risk factors for hyperbilirubinemia and his physical examination findings are normal, making
it unnecessary to obtain a bilirubin level.

American Academy of Pediatrics 72


PREP ® Self-Assessment PREPSA 2021

PREP Pearls
• Neonates at low risk of experiencing complications can be safely discharged 24 hours
after birth.
• Neonates discharged before 48 hours of age should follow up with a pediatric provider
within 48 hours.
• Social support, access to care, and maternal and newborn health should be considered
when planning for discharge from the newborn nursery.

ABP Content Specifications(s)


• Plan the early discharge of a newborn infant, including follow-up evaluation

Suggested Readings
• Benitz WE; Committee on Fetus and Newborn. Hospital stay for healthy term newborn
infants. Pediatrics. 2015;135(5):948-953. doi:10.1542/peds.2015-0699.
• Sullivan CK, Dela Cruz-Rivera S. Hospital discharge of the healthy term and late preterm
infant. In: McInerny TK, Adam HM, Campbell DE, DeWitt TG, Foy JM, Kamat DM,
eds. American Academy of Pediatrics Textbook of Pediatric Care. 2nd ed. Elk Grove
Village, IL: American Academy of Pediatrics; 2016:779-789. Pediatric Care Online.

American Academy of Pediatrics 73


PREP ® Self-Assessment PREPSA 2021
Question 20
A 16-year-old adolescent girl with worsening fatigue is brought to the emergency department.
She is a member of the varsity soccer team and had been in her usual state of good health until 2
days ago, when she experienced worsening exercise intolerance. Over the last 48 hours, her
condition has progressed. She reports that her heart is racing and that it is hard to catch her
breath after climbing a flight of stairs. She has had normal growth and development with no
remarkable medical or surgical history. She has a temperature of 37.8°C, a heart rate of 132
beats/min, a respiratory rate of 26 breaths/min, a blood pressure of 98/70 mm Hg, and oxygen
saturation of 97% on room air. Her height is at the 60th percentile and her weight is at the 40th
percentile for age. Her conjunctivae are pale and slightly icteric. Her lungs are clear to
auscultation bilaterally, and her heart examination reveals a normal rhythm with tachycardia. The
tip of her spleen is palpable just below the costal margin.

Laboratory data are shown:


Laboratory Test Result
White blood cell count 9,900/µL (9.9 × 109/L)
Hemoglobin 4.1 g/dL (41 g/L)
Platelet count 432 × 103/µL (432 × 109/L)
Reticulocyte count 13.1%
Neutrophils 45%
Lymphocytes 38%
Monocytes 11%
Nucleated red blood cells Present
Lactate dehydrogenase 430 U/L (reference range, 135-225 U/L)
Bilirubin, total 3.2 mg/dL (54.7 µmol/L)
Direct antibody test IgG positive

Of the following, the MOST appropriate next step in management is to urgently administer
A. 2 mg/kg intravenous methylprednisolone
B. 20 mL/kg intravenous normal saline
C. 0.2 mg/kg intravenous rasburicase
D. 15 mL/kg O-negative packed red blood cells

American Academy of Pediatrics 74


PREP ® Self-Assessment PREPSA 2021
Correct Answer: A
The girl in the vignette has severe, symptomatic anemia with a normal white blood cell count
and differential and an elevated platelet count. Healthy red blood cells have an average lifespan
of 120 days, so a normal reticulocyte count (the rate of replacement of senescent red blood cells)
is ~1%. Severe, isolated anemia is caused either by inadequate red blood cell production, which
would manifest with a low reticulocyte count, or by increased destruction that cannot be
adequately compensated for by increasing red cell production, which would manifest with an
increased reticulocyte count (Item C20). The girl in the vignette has a markedly elevated
reticulocyte count, elevated total bilirubin, and an elevated lactate dehydrogenase level, all
suggesting a destructive (hemolytic) disorder. The nucleated red blood cells suggest a stressed
hematopoiesis as the marrow attempts to compensate for the severe anemia. Her unremarkable
personal medical history suggests that she does not have a heritable hemolytic disorder, and the
positive direct antibody test finding for IgG is diagnostic of a warm antibody-mediated
autoimmune hemolytic anemia (AIHA). Whereas AIHA can be associated with medications, the
girl in the vignette has an unremarkable medical history, suggesting that she has idiopathic
AIHA. Autoimmune hemolytic anemia can rapidly lead to life-threatening anemia. The
definitive therapy for AIHA is immune suppression, which can typically be slowly tapered over
time and eventually discontinued. Glucocorticoids are the first-line treatment of choice.

Although infusing crystalloid may be an appropriate first step in treatment for a


hemodynamically unstable adolescent, increasing the intravascular volume with crystalloid in the
setting of severe anemia will exacerbate the anemia, decrease the oxygen-carrying capacity, and
potentially precipitate heart failure.

American Academy of Pediatrics 75


PREP ® Self-Assessment PREPSA 2021
Rasburicase is a uricase; it breaks down uric acid and would be appropriate to use in a patient
with acute leukemia or lymphoma who is at high risk of developing tumor lysis syndrome. There
is no indication that the girl in the vignette has leukemia or lymphoma.

If the girl in the vignette was having symptoms suggestive of inadequate oxygen delivery to a
critical organ (eg, brain [confusion]) or heart [chest pain]), then transfusion of a least
incompatible unit of packed red blood cells should be considered. This will be only a
temporizing measure, as the autoimmune antibody in AIHA is typically a pan agglutinin and will
lead to hemolysis of the transfused blood as well. In the vignette, the girl has tachycardia, but
without other symptoms suggestive of an immediately impending event. As such, a transfusion
of packed red blood cells should be avoided.

PREP Pearls
• Severe, isolated anemia is caused either by inadequate red blood cell production, which
would manifest with a low reticulocyte count, or by increased destruction, which would
manifest with an increased reticulocyte count.
• The definitive therapy for autoimmune hemolytic anemia is immune suppression.
Glucocorticoids are the first-line treatment of choice.
• In the setting of autoimmune hemolytic anemia, symptoms suggestive of inadequate
oxygen delivery to a critical organ (eg, brain [confusion]) or heart [chest pain]) would
make transfusion of a least incompatible unit of packed red blood cells an appropriate
first step in treatment.

ABP Content Specifications(s)


• Plan the appropriate diagnostic evaluation of acute-onset anemia
• Recognize the clinical findings associated with autoimmune hemolytic anemia

Suggested Readings
• Brodsky RA. Warm autoimmune hemolytic anemia. N Engl J Med. 2019;381(7):647-654.
doi:10.1056/NEJMcp1900554.
• McFarren AK, Levy AS. Anemia and pallor. In: McInerny TK, Adam HM, Campbell
DE, DeWitt TG, Foy JM, Kamat DM, eds. American Academy of Pediatrics Textbook of
Pediatric Care. 2nd ed. Elk Grove Village, IL: American Academy of Pediatrics;
2017:1199-1208. Pediatric Care Online .
• Noronha SA. Acquired and congenital hemolytic anemia. Pediatr Rev. 2016;37(6):235-
246. doi:10.1542/pir.2015-0053.
• Sackey K. Hemolytic anemia: part 1. Pediatr Rev. 1999;20(5):152-158.
doi:10.1542/pir.20-5-152.
• Sackey K. Hemolytic anemia: part 2. Pediatr Rev. 1999;20(6):204-208.
doi:10.1542/pir.20-6-204.

American Academy of Pediatrics 76


PREP ® Self-Assessment PREPSA 2021
Question 21
A pediatrician receives a phone call from the state’s newborn screening program regarding a 7-
day-old neonate with an elevated trypsinogen, which is concerning for a diagnosis of cystic
fibrosis. The neonate was seen by the pediatrician’s partner 1 day ago and had appropriate
feeding and growth parameters as well as normal vital signs and physical examination findings.
The partner noted that there is no significant family history of diseases in childhood.

Of the following, the MOST appropriate next step is to


A. inform the parents of the diagnosis of cystic fibrosis
B. discuss the result at the 2-month health supervision visit
C. reassure the parents that this is likely a false-positive result
D. schedule the neonate for diagnostic testing

American Academy of Pediatrics 77


PREP ® Self-Assessment PREPSA 2021
Correct Answer: D
This infant’s newborn screening test resulted in an elevated immunoreactive trypsinogen (IRT), a
pancreatic enzyme that is elevated in most newborns with cystic fibrosis (CF). An elevated IRT
is a sensitive but not a specific screen for CF; further diagnostic testing with a sweat chloride test
is necessary. It would, therefore, not be appropriate to notify the neonate’s parents of a diagnosis
at this time. Early identification and treatment of CF has been associated with maximal growth
and improved lung function; it would not be appropriate to wait for the 2-month health
supervision visit to address this laboratory result. Although false-positive IRT elevations can
occur, especially in premature or ill neonates, follow-up testing is necessary before any result can
be labeled a false positive. The most appropriate next step is to schedule diagnostic testing with a
sweat chloride test.

The goal of newborn screening is to identify and initiate treatment for conditions with significant
morbidity and/or mortality before a patient is clinically symptomatic. In the United States,
newborn screening began in the 1960s with phenylketonuria. Although each state’s newborn
screen is different, the US Department of Health & Human Services maintains a Recommended
Uniform Screening Panel which contains 35 core conditions and 26 secondary conditions as of
winter 2019. All 50 states also have screening requirements for critical congenital heart defects
and congenital hearing loss. For a condition to be included on the newborn screening panel, the
disorder must be associated with significant morbidity and/or mortality and be treatable, with
evidence that early treatment improves outcomes. There must be an inexpensive and sensitive
test that can be performed on a dried bloodspot sample and a definitive follow-up testing plan to
differentiate true positives from false positives. It is important to remember that no screening test
is 100% sensitive; even with the history of a normal newborn screening test, diagnostic testing
for a condition should be considered when clinically indicated. The web site babysfirsttest.org is
maintained by the US Department of Health & Human Services and contains information for
parents and medical providers about newborn screening.

PREP Pearls
• By design, newborn screening tests have high sensitivity and may have lower specificity;
positive results must be promptly followed up with diagnostic testing.
• Even with the history of a normal newborn screening test, evaluation for a condition
should be considered when clinically suggested.
• For a condition to be included on the newborn screening panel, the disorder must be
associated with significant morbidity and/or mortality and be treatable, with evidence that
early treatment improves outcomes. There must be an inexpensive and sensitive test that
can be performed on a dried bloodspot sample and a definitive follow-up testing plan to
differentiate true positives from false positives.

ABP Content Specifications(s)


• Plan the appropriate initial response to a positive neonatal screening test for metabolic
diseases

American Academy of Pediatrics 78


PREP ® Self-Assessment PREPSA 2021
Suggested Readings
• American Academy of Pediatrics Newborn Screening Authoring Committee. Newborn
screening expands: recommendations for pediatricians and medical homes—implications
for the system. Pediatrics. 2008;121(1):192-217. doi: 10.1542/peds.2007-3021.
• Farrel PM, White TB, Ren CL, et al. Diagnosis of cystic fibrosis: consensus guidelines
from the Cystic Fibrosis Foundation. J Pediatr. 2017;181S:S4-S15.e.1.
doi: 10.1016/j.jpeds.2016.09.064.
• Health & Human Services Administration. Recommended uniform screening
panel. https://www.hrsa.gov/advisory-committees/heritable-disorders/rusp/index.html.

American Academy of Pediatrics 79


PREP ® Self-Assessment PREPSA 2021
Question 22
A male neonate, born at 34 weeks’ gestation, is being evaluated for respiratory distress shortly
after delivery. The mother reports no medical problems during the pregnancy; prenatal care
records are not available. Maternal group B Streptococcus status is unknown. The mother was
febrile, with a maximum temperature of 39°C, for 6 hours before delivery. The neonate was
delivered vaginally through green amniotic fluid. On physical examination, he is noted to have
grunting and tachypnea. The remainder of his examination findings are unremarkable. He is
started on continuous positive airway pressure +5 cm H2O and 30% oxygen in the delivery room.
Chest radiography shows bilateral streaky infiltrates.

Of the following, the pathogen MOST likely to be responsible for this neonate’s findings is
A. Chlamydia trachomatis
B. Escherichia coli
C. Listeria monocytogenes
D. Streptococcus agalactiae

American Academy of Pediatrics 80


PREP ® Self-Assessment PREPSA 2021
Correct Answer: C
For the late preterm neonate in the vignette, the most likely pathogen is Listeria
monocytogenes. Listeria is an uncommon cause of neonatal infection, with mortality ranging
from 14% to 56%. Because immunity against Listeria is dependent on T-cell–mediated
immunity and granuloma formation, pregnant women and other patients with
immunosuppression are at increased risk of infection.

Typically, pregnant women acquire Listeria via contaminated food such as ready-to-eat deli
meat, unpasteurized milk, soft cheese, ice cream, or frozen fruits and vegetables. Affected
pregnant women may report a flu-like illness with fever before delivery. Up to 22% of affected
pregnancies will result in a stillbirth.

Neonatal Listeria may present as early-onset disease in the first week after birth or as late-onset
disease (Item C22). In early-onset disease, following maternal bacteremia, Listeria crosses the
placenta into the fetal circulation. More than 70% of women with Listeria deliver before 35
weeks’ gestation. Among premature infants, meconium-stained amniotic fluid is uncommon.
However, green or brown-stained amniotic fluid is often noted with Listeria infection. Affected
neonates may present with respiratory distress because of pneumonia; chest radiographic
findings show bilateral streaky infiltrates or a miliary pattern. With severe infection, neonates
may have an erythematous rash with small, pale papules called granulomatosis infantisepticum,
which is caused by disseminated granulomas. Pustular lesions may also be noted on the skin and
pharynx.

American Academy of Pediatrics 81


PREP ® Self-Assessment PREPSA 2021

In late-onset Listeria infection, neonates are more often of term gestation. Two weeks after birth,
these neonates develop nonspecific symptoms such as poor feeding. Transmission is thought to
be perinatal or nosocomial. Transmission via breast milk has also been reported. Meningitis is
more common among neonates with late-onset disease. Antibiotic therapy for Listeria should
include ampicillin plus an aminoglycoside, typically gentamicin.

Pneumonia caused by Streptococcus agalactaie has a reticulogranular pattern on chest


radiography. Neonatal chlamydial infection presents later, typically between 4 and 12 weeks
after birth, not in the immediate postnatal period. Neonatal infection with Escherichia coli most
commonly manifests as bloodstream infection rather than pneumonia.

American Academy of Pediatrics 82


PREP ® Self-Assessment PREPSA 2021
PREP Pearls
• Neonates born to mothers with Listeria infection are often delivered prematurely with
brown- or green-stained amniotic fluid. Among premature infants, meconium-stained
amniotic fluid is uncommon.
• Early-onset neonatal Listeria is uncommon, with a high rate of mortality. These neonates
may present with respiratory distress and an erythematous papular rash.
• Late-onset neonatal Listeria typically presents with nonspecific findings among term
infants at 14 days after birth. Meningitis is more common with late-onset
neonatal Listeria.

ABP Content Specifications(s)


• Differentiate respiratory distress syndrome from congenital pneumonia in a newborn
infant
• Recognize the characteristic clinical and radiographic appearance of respiratory distress
syndrome in a newborn infant, and manage appropriately

Suggested Readings
• American Academy of Pediatrics. Listeria monocytogenes infections. In: Kimberlin DW,
Brady MT, Jackson MA, Long SS, eds. Red Book: 2018 Report of the Committee on
Infectious Diseases. Itasca, IL: American Academy of Pediatrics; 2018:511-515 Red
Book Online.
• Lee PJ, Krilov LR. Listeria. Pediatr Rev. 2018;39;153-155. doi: 10.1542/pir.2017-0062.
• Posfay-Barbe KM, Wald ER. Listeriosis. Pediatr Rev. 2004;25;151-159.
doi: 10.1542/pir.25-5-151.

American Academy of Pediatrics 83


PREP ® Self-Assessment PREPSA 2021
Question 23
A 7-year-old boy who had surgery for craniopharyngioma 2 days ago now has increased thirst
and frequent urination. He has no vomiting, blurry vision, or seizures. His current medication
includes phenytoin. He has a heart rate of 102 beats/min, a respiratory rate of 16 breaths/min,
and a blood pressure of 102/60 mm Hg. He has no swelling over his body. His craniotomy
incision site is clean and healing. He has no neurological deficits, and the rest of his physical
examination findings are unremarkable.

Laboratory data are shown:


Laboratory Test Result
Blood
Sodium 150 mEq/L (150 mmol/L)
Potassium 4.2 mEq/L (4.2 mmol/L)
Chloride 110 mEq/L (110 mmol/L)
Bicarbonate 23 mEq/L (23 mmol/L)
Blood urea nitrogen 28 mg/dL (10.0 mmol/L)
Creatinine 0.5 mg/dL (44 µmol/L)
Glucose 90 mg/dL (5.0 mmol/L)
Serum osmolality 325 mOsm/kg (325 mmol/kg)
Urine
Specific gravity 1.006
Leukocyte esterase Negative
Nitrite Negative
Blood Negative
Protein Negative
Glucose Negative

Of the following, the MOST likely diagnosis for this child is


A. acute kidney injury
B. central diabetes insipidus
C. cerebral salt wasting
D. syndrome of inappropriate secretion of antidiuretic hormone

American Academy of Pediatrics 84


PREP ® Self-Assessment PREPSA 2021
Correct Answer: B
The boy in this vignette is experiencing polydipsia and polyuria immediately after undergoing
cranial surgery. His laboratory evaluation shows dilute urine and hypernatremia, favoring a
diagnosis of central diabetes insipidus (CDI).

Central diabetes insipidus is a condition resulting from inadequate secretion of antidiuretic


hormone (ADH) from the posterior lobe of the pituitary gland. It is commonly a result of a
primary brain tumor or cranial surgery or injury. In infants and children, other causes of CDI
include familial history, Langerhans cell histiocytosis, head trauma, associated brain
malformations, and other central nervous system injuries (infectious, vascular, and
granulomatous).

In CDI, lack of ADH causes large amounts of free water to be lost from the collecting duct of the
kidney, resulting in polyuria. New-onset nocturia can sometimes be the presenting symptom of
CDI. In children with CDI secondary to head trauma or cranial surgery, a triple-phase response is
seen: initial intense transient polyuria lasting hours to several days, followed by an antidiuretic
period and then by persistent polyuria. Children old enough to ask for water will try to
compensate for the urinary losses with increased thirst and intake of water and typically will
have a craving for ice-cold water. Infants and young children too young to ask for or get free
water on their own will develop hypernatremic dehydration. Untreated CDI can also lead to
growth failure in children.

A 24-hour urine collection and intake recording is done to confirm polyuria and polydipsia.
Laboratory evaluation in CDI shows low urine specific gravity and low urine osmolality on a
first-voided morning specimen. Normal or high serum sodium (hypernatremia) is seen,
depending on the child’s free water intake. When the diagnosis is unclear, a water deprivation
test is performed to differentiate CDI from nephrogenic diabetes insipidus. Nephrogenic diabetes
insipidus results from an inability of the renal tubules to respond to ADH and concentrate the
urine despite normal ADH secretion (ADH insensitivity). Urine specific gravity will increase
after exogenous ADH administration during a water deprivation test in CDI but will remain
unchanged in children with nephrogenic diabetes insipidus.

Although the child in this vignette has an elevated blood urea nitrogen:serum creatinine ratio,
acute kidney injury is unlikely because the child does not have oliguria and concentrated urine.
Cerebral salt wasting and the syndrome of inappropriate antidiuretic hormone can present after
cranial surgery. However, both of these conditions are characterized by inappropriately
concentrated urine in a setting of hypo-osmolality (hyponatremia).

Children with central DI are frequently referred to an endocrinologist to determine the cause and
to investigate other pituitary functions. A magnetic resonance imaging of the brain may reveal
absence of the normal hyperintense signal of the posterior pituitary gland. Management includes
desmopressin administration, free access to water, and a close monitoring of serum sodium and
urine specific gravity.

American Academy of Pediatrics 85


PREP ® Self-Assessment PREPSA 2021
PREP Pearls
• Diabetes insipidus presents with polyuria, new-onset nocturia, and polydipsia.
• Laboratory evaluation in diabetes insipidus shows low urine specific gravity on the first-
voided morning urine specimen and a normal to high serum sodium level.
• Central diabetes insipidus is commonly a result of a primary brain tumor, cranial surgery,
or brain injury

ABP Content Specifications(s)


• Recognize the signs and symptoms of diabetes insipidus in patients of various ages
• Interpret the laboratory findings associated with diabetes insipidus
• Recognize the association between cranial injury/surgery and diabetes insipidus

Suggested Readings
• Miller RS, Libber SM, Plotnick L. Polyuria. In: McInerny TK, Adam HM, Campbell DE,
DeWitt TG, Foy JM, Kamat DM, eds. American Academy of Pediatrics Textbook of
Pediatric Care. 2nd ed. American Academy of Pediatrics; 2017:1528-1532. Pediatric
Care Online.
• Saborio P, Tipton GA, Chan JCM. Diabetes insipidus. Pediatr Rev. 2000;21(4):122-129.
doi:10.1542/pir.21-4-122 .
• Weiner A, Vuguin P. Diabetes insipidus. Pediatr Rev. 2020;41(2):96-99.
doi:10.1542/pir.2018-0337.

American Academy of Pediatrics 86


PREP ® Self-Assessment PREPSA 2021
Question 24
A 16-year-old previously healthy adolescent is seen in the emergency department for evaluation
after 3 hours of sharp left-sided testicular pain that radiates to the lower abdomen. The pain
started suddenly while he was playing video games at home. There was no antecedent trauma,
and he is not sexually active. He has no fever, urinary symptoms, or penile discharge. He has
vomited twice since the onset of pain. On physical examination, the left hemiscrotum is
edematous and exquisitely tender. The left testis appears to be elevated and has a horizontal
orientation. The left cremasteric reflex is absent. The right hemiscrotum examination findings are
normal.

Of the following, the MOST appropriate next step in management is


A. incision and drainage
B. scrotal ultrasonography
C. urine testing for gonorrhea and chlamydia
D. urological consultation

American Academy of Pediatrics 87


PREP ® Self-Assessment PREPSA 2021
Correct Answer: D
The adolescent in the vignette has a classic presentation of testicular torsion. Testicular torsion is
a urologic emergency that requires immediate urological consultation and detorsion of the
testicle. In this scenario imaging is unnecessary because it will not yield additional information
and will potentially delay appropriate treatment.

Testicular torsion has a biphasic peak incidence, first in the neonatal period and a larger peak
during puberty. The presentation of testicular torsion is typically acute onset of unilateral
testicular pain; however, abdominal pain may be the only complaint, particularly in younger
children. Inguinal or abdominal pain, pain with ambulation, nausea, and/or vomiting may also be
present. On physical examination, one hemiscrotum will be swollen and tender and may be
elevated with a horizontal lie of the testicle because of a change in testicular orientation that
occurs as the spermatic cord shortens during the torsion process. If the diagnosis is uncertain,
ultrasonography of the testicle with Doppler blood flow can assist in the diagnosis. Torsion is
confirmed if blood flow to the affected testicle is significantly decreased or absent.
Management of testicular torsion requires detorsion. Torsion time is an independent risk factor
for testicle survival. One recent study demonstrated that for every 10 minutes, the risk of having
a nonviable testis increased 4.8%. Testicular detorsion is best accomplished in the operating
room where an orchiopexy can also be performed to minimize the risk of retorsion. However, if a
significant delay is anticipated, manual detorsion can be attempted. A torsed testis usually is
twisted medially. For detorsion, after ensuring appropriate analgesia, the affected testis is rotated
laterally, for at least 1 full rotation.

Incision and drainage is the appropriate treatment for a scrotal abscess, which has a more
insidious presentation than testicular torsion. Urine tests for gonorrhea and chlamydia have a
high sensitivity and specificity and have replaced urethral swabs for diagnosis in men. This can
be helpful in determining a causative agent for epididymitis, the most common cause of
testicular pain. Unlike testicular torsion, the symptoms of epididymitis typically develop over
several days, as opposed to several hours; there will be a normal cremasteric reflex and more
focal tenderness at the posterior aspect of the testicle. In severe cases epididymitis can develop
into epididymo-orchitis with involvement of the ipsilateral testicle.

PREP Pearls
• Testicular torsion typically presents with acute onset of unilateral scrotal pain with a
swollen and tender hemicrostum on physical examination.
• If the diagnosis of testicular torsion can be confirmed with the history and physical
examination, urology should be consulted emergently; there is no need for imaging.
• Detorsion should take place in the operating room in conjunction with orchiopexy;
however, manual detorsion should be attempted if there will be a significant delay.

ABP Content Specifications(s)


• Recognize the clinical findings associated with testicular torsion, and manage
appropriately
• Plan the appropriate diagnostic evaluation of testicular torsion

American Academy of Pediatrics 88


PREP ® Self-Assessment PREPSA 2021
Suggested Readings
• Gatti JM, Murphy JP. Acute testicular disorders. Pediatr Rev. 2008;29(7):235-241.
doi: 10.1542/pir.29-7-235.
• Gold DD, Lorber A, Levine H, et al. Door to detorsion time determine testicular
survival. Urology. 2019. pii: S0090-4295(19)30711-3.
doi: 10.1016/j.urology.2019.08.003.
• Palmer LS. Scrotal swelling and pain. In: McInerny TK, Adam HM, Campbell DE,
DeWitt TG, Foy JM, Kamat DM, eds. American Academy of Pediatrics Textbook of
Pediatric Care. 2nd ed. Itasca, IL: American Academy of Pediatrics; 2016; chap
190:1571-1577. Pediatric Care Online.
• Wang CL, Aryal B, Oto A, et al. ACR appropriateness criteria: acute onset of scrotal
pain-without trauma, without antecedent mass. J Am Coll Radiol. 2019;16(5S):S38-S43.
doi: 10.1016/j.jacr.2019.02.016.

American Academy of Pediatrics 89


PREP ® Self-Assessment PREPSA 2021
Question 25
A previously healthy 2-year-old boy is seen in the office for a worsening cough. The illness
started 2 days ago with cough, rhinorrhea, and a fever of 38.3°C. His mother describes the cough
as having a harsh, seal-like quality. The boy’s temperature is 38°C, heart rate is 110 beats/min,
respiratory rate is 25 breaths/min, oxygen saturation is 99% on room air, and blood pressure is
90/60 mm Hg. At rest, he has a frequent seal-like cough; he is not in respiratory distress and has
no stridor. His lungs are clear to auscultation bilaterally with good aeration, and he has no chest
wall retractions. When he becomes more active, the boy has mild stridor and mild retractions.

Of the following, the BEST next step in management of the boy’s symptoms is to
A. administer dexamethasone
B. continue supportive care
C. prescribe azithromycin
D. refer for laryngoscopy

American Academy of Pediatrics 90


PREP ® Self-Assessment PREPSA 2021
Correct Answer: A
The boy in the vignette has signs and symptoms consistent with a diagnosis of croup
(laryngotracheitis). Because he has signs of worsening upper airway inflammation and
obstruction, administering dexamethasone is the best next step to manage his symptoms.
Continuing supportive care is inadequate because of his worsening symptoms. Treatment with
azithromycin is inappropriate because he has no signs of bacterial infection. Laryngoscopy is
also not indicated at this time because he does not have recurrent, prolonged, or severe disease.
The term "croup" most commonly refers to stridor caused by a viral infection. Item C25A lists
the most common viruses responsible for croup. Croup is primarily a clinical diagnosis.
Laboratory testing (typically real-time polymerase chain reaction from a nasopharyngeal swab)
and imaging are usually unnecessary except when placing patients in cohorts or when the
diagnosis is uncertain. If imaging is performed, neck radiography is preferred and will

demonstrate the classic “steeple sign” that signifies an edematous subglottis (Item C25B).

American Academy of Pediatrics 91


PREP ® Self-Assessment PREPSA 2021

American Academy of Pediatrics 92


PREP ® Self-Assessment PREPSA 2021

Item C25B: Radiograph depicting classic “steeple sign” and signifying an edematous subglottis.
Reprinted with permission from Breese Hall C, Hall WJ. Croup (acute
laryngotracheobronchitis). In: McInerny TK, Adam HM, Campbell DE, DeWitt TG, Foy JM,
Kamat DM, eds. American Academy of Pediatrics Textbook of Pediatric Care. 2nd ed. Itasca,
IL: American Academy of Pediatrics; 2016.

The stridor of croup is more commonly observed in children younger than 5 years because of
their smaller airways. The classic barking or brassy cough is usually preceded by 1 to 2 days of
upper respiratory tract symptoms. Some parents are more alarmed by the intensity of their child’s
hoarseness than by their cough. Fever may reach 40°C, but is usually of lower grade. Respiratory
symptoms usually progress for 3 to 4 days; they may wax and wane throughout the day, but
typically worsen at night, perhaps because of the natural nocturnal nadir in endogenous
glucocorticoids.

Exogenous glucocorticoids, such as dexamethasone or prednisone, are the mainstay of treatment


for croup (Item C25C). Steroids should be given orally, if possible, to avoid the increased work
of breathing that accompanies injections. Medication is usually not needed in children who do
not have stridor or retractions at rest. Children who have these symptoms with increased activity
may also not need steroids, but if they present early in the course of their illness, as did the child
in this vignette, steroids can offer significant clinical improvement. Some parents and physicians
may opt to not administer steroids to children with mild croup, given the potential risk, although
small, of side effects, especially if they are further along in their course of illness and likely to
show improvement soon without treatment.

American Academy of Pediatrics 93


PREP ® Self-Assessment PREPSA 2021

American Academy of Pediatrics 94


PREP ® Self-Assessment PREPSA 2021
Children with moderate croup who have stridor and retractions at rest without evidence of
respiratory distress should receive steroids, and clinicians may consider administering nebulized
epinephrine. Observation for a minimum of 2 hours after epinephrine treatment is required
because of the risk of return of symptoms. If symptoms persist or recur after 2 epinephrine
treatments, the child should be hospitalized for continued epinephrine treatments; heliox
administration may be considered.

Children with severe croup and those with evidence of respiratory distress should receive both
steroids and nebulized epinephrine. If the symptoms resolve, these children may be discharged
from the hospital if they exhibit no return of symptoms after the 2-hour observation period.
Bacterial tracheitis and epiglottitis should be considered in the differential diagnosis of croup.
These illnesses may initially mimic the presentation of croup; however, affected children will
appear more toxic and deteriorate more rapidly. Children with bacterial tracheitis or epiglottitis
should receive antibiotics and may need urgent laryngoscopy and possibly endotracheal
intubation.

PREP Pearls
• Children with worsening mild croup and children with moderate or severe croup should
be treated with oral glucocorticoids.
• Nebulized epinephrine may be considered for treatment of children with moderate croup
and should be administered to those with severe croup.
• Bacterial tracheitis and epiglottitis must be considered in the differential diagnosis of
viral croup.

ABP Content Specifications(s)


• Plan the appropriate clinical and laboratory evaluation of croup of various etiologies
• Plan the appropriate management of croup

Suggested Readings
• Breese Hall C, Hall WJ. Croup (acute laryngotracheobronchitis). In: McInerny TK,
Adam HM, Campbell DE, DeWitt TG, Foy JM, Kamat DM, eds. American Academy of
Pediatrics Textbook of Pediatric Care. 2nd ed. Itasca, IL: American Academy of
Pediatrics; 2016;chap 352:2799-2804. Pediatric Care Online.
• Virbalis J, Smith L. Upper airway obstruction. Pediatr Rev. 2015;36(2):62-73.
doi:10.1542/pir.36-2-62.

American Academy of Pediatrics 95


PREP ® Self-Assessment PREPSA 2021
Question 26
A 14-year-old adolescent boy is seen for a sports preparticipation physical examination. He and
his mother are concerned about his small size. He has no known medical problems and takes no
medication. A comprehensive review of systems is unremarkable. His mother is 165 cm tall and
had menarche at age 12 years. His father is 178 cm tall and completed his linear growth while in
college. His 12-year-old sister is taller than he is at 157 cm and recently had menarche. His
growth chart is shown in Item Q26. His body mass index is 17.3 kg/m2 (18th percentile). His
vital signs are normal for age. He appears younger than his chronological age. His sexual
maturity rating is 1 for pubic hair and genital development. His testicular volume is 3 mL
bilaterally. The remainder of his physical examination findings are normal. A bone age
radiograph has a result of 11 years.

American Academy of Pediatrics 96


PREP ® Self-Assessment PREPSA 2021

American Academy of Pediatrics 97


PREP ® Self-Assessment PREPSA 2021
Laboratory data are shown:
Laboratory Test Result
Luteinizing hormone 0.4 IU/L (0.4 IU/L) (reference rangea, < 0.1-0.3 IU/L)
Follicle-stimulating hormone 2.4 IU/L (reference rangea, 0.26-3 IU/L)
15 ng/dL (0.5 nmol/L) (reference rangea,
Testosterone
< 2.5-10 ng/dL [0.09-0.4 nmol/L])

a
Reference ranges are for individuals with a sexual maturity rating of 1.

Of the following, the BEST next step is


A. advising the patient to increase the calories in his diet
B. brain magnetic resonance imaging
C. referral for consideration of growth hormone therapy
D. referral for consideration of testosterone therapy

American Academy of Pediatrics 98


PREP ® Self-Assessment PREPSA 2021
Correct Answer: D
The adolescent boy in the vignette has constitutional delay of puberty, also known as
constitutional delay of growth and puberty. The best next step is referral for consideration of
testosterone therapy. A short course of a testosterone ester intramuscularly once per month for 3
to 4 months can accelerate pubertal development and promote earlier initiation of the pubertal
growth spurt. It can have a positive effect on the psychosocial distress that can accompany
constitutional delay. There is no significant effect on final adult height. Reassurance with
observation would also be an acceptable management option.

Puberty is considered delayed in boys when there is a lack of testicular growth ≥ 4 mL in volume
(or 2.5 cm in length) by age 14 years. In girls, puberty is delayed when there is a lack of breast
development by age 13 years. Constitutional delay of puberty is the most common cause of
delayed puberty, especially in boys. The typical growth curve of a child with constitutional delay
of growth and puberty shows normal size at birth, re-channeling to a height percentile lower than
that expected for the adjusted mid-parental height by age 3 years, then normal prepubertal height
velocity until the pubertal growth spurt. The pubertal growth spurt occurs at a later than average
age, and final adult height is in the normal range. These individuals are otherwise healthy, and
often there is a family history of constitutional delay. Bone age is delayed and during
adolescence more closely matches the pubertal stage than chronological age. Many children and
adolescents with constitutional delay present with concerns regarding short stature. When
typically developing adolescents are experiencing their pubertal growth spurt, the height
discrepancy for those with constitutional delay increases.

The boy in the vignette is otherwise healthy and has a delayed bone age. He likely has a family
history of constitutional delay, because his father completed his own linear growth in college.
Boys on average stop growing in height by age 17 years. His height has fallen below the 5th
percentile because he continues to grow at a normal prepubertal height velocity while his
typically developing peers are experiencing their pubertal growth spurt. Although he has no
physical signs of puberty, his biochemical profile shows evidence of early activation of the
hypothalamic-pituitary-gonadal axis, with a luteinizing hormone and testosterone level just
above the normal range for a male with a sexual maturity rating of 1. These levels make
hypogonadotropic hypogonadism unlikely. His bone age of 11 years matches this biochemical
profile.

The boy’s delayed bone age also predicts catch-up growth. Using his current height and bone
age, his predicted adult height is 179 cm. This predicted adult height matches his adjusted mid-
parental height of 178 cm.

Adjusted mid-parental height (target height) can be calculated in boys as


[mother’s height + father’s height + 13 cm]/2

For girls, 13 cm is subtracted from the sum of the parents’ heights. Most children end up within 5
cm of this calculated target height.

The boy’s body mass index is normal at the 18th percentile and there is no evidence of
undernutrition, so advising him to increase the calories in his diet is not the best answer. With a
American Academy of Pediatrics 99
PREP ® Self-Assessment PREPSA 2021
normal height velocity for his pubertal stage, evidence of early activation of the hypothalamic-
pituitary-gonadal axis, and no signs or symptoms of a central nervous system mass, brain
magnetic resonance imaging is not indicated. Referral for consideration of growth hormone
therapy is not correct because growth hormone is not indicated for this boy. Even without
testosterone therapy, his height is expected to catch up to the normal range.

PREP Pearls
• Constitutional delay of puberty is the most common cause of delayed puberty, especially
in boys.
• Constitutional delay of puberty in boys with associated psychosocial distress is an
indication for a short course of testosterone to accelerate pubertal development and
promote earlier initiation of the pubertal growth spurt.
• Puberty is considered delayed in boys when there is a lack of testicular growth = 4 mL in
volume (or 2.5 cm in length) by age 14 years. In girls, puberty is delayed when there is a
lack of breast development by age 13 years.

ABP Content Specifications(s)


• Plan the appropriate diagnostic evaluation of constitutional delay of puberty
• Plan the appropriate management of constitutional delay of puberty

Suggested Readings
• Braun LR, Marino R. Disorders of growth and stature. Pediatr Rev. 2017;38(7):293-304.
doi:10.1542/pir.2016-0178.
• Kaplowitz PB. Delayed puberty. Pediatr Rev. 2010;31(5):189-195. doi:10.1542/pir.31-5-
189.
• Kritzler RK, Long D, Plotnick L. Puberty: normal and abnormal. In: McInerny TK,
Adam HM, Campbell DE, DeWitt TG, Foy JM, Kamat DM, eds. American Academy of
Pediatrics Textbook of Pediatric Care. 2nd ed. Elk Grove Village, IL: American
Academy of Pediatrics; 2017:1540-1544. Pediatric Care Online .
• Rose SR, Vogiatzi MG, Copeland KC. A general pediatric approach to evaluating a short
child. Pediatr Rev. 2005;26(11):410-420. doi:10.1542/pir.26-11-410.

American Academy of Pediatrics 100


PREP ® Self-Assessment PREPSA 2021
Question 27
An 11-month-old girl is seen by her primary care provider for evaluation after a daycare
exposure. Two and a half weeks ago, her room teacher developed fever, nausea, and vomiting,
and 1 week later developed jaundice. She was subsequently diagnosed with hepatitis A infection.
The teacher has been out of work since the onset of this illness. Yesterday, the daycare program
notified families of the possible exposure and recommended medical evaluation. The girl has
been eating, voiding, and sleeping well. She has no underlying medical conditions and is
scheduled for her 12-month health supervision visit in a few weeks. Her vital signs include a
temperature of 37.1°C, heart rate of 110 beats/min, respiratory rate of 28 breaths/min, and blood
pressure of 88/46 mm Hg. Physical examination findings reveal a well-appearing child with no
jaundice, abdominal tenderness, or hepatomegaly.

Of the following, the BEST next management step for this child is to administer
A. hepatitis A vaccine
B. immune globulin
C. interferon
D. no prophylaxis

American Academy of Pediatrics 101


PREP ® Self-Assessment PREPSA 2021
Correct Answer: D
Hepatitis A prophylaxis with either hepatitis A vaccine or immune globulin is not indicated for
the girl in the vignette. Though she is susceptible to infection with hepatitis A virus, she is too
young to be vaccinated (<12 months of age), and immune globulin is not effective if given 2
weeks after the exposure.

Childcare settings are unique in their vulnerability to hepatitis A exposures and outbreaks.
Because hepatitis A virus is transmitted via the fecal-oral route, the care of incontinent children
creates a potentially high-risk environment. Hepatitis A infection in children is often
asymptomatic; only 30% of young children develop symptoms, which can include fever,
malaise, nausea, vomiting, and jaundice. In contrast, approximately 70% of adults with hepatitis
A develop jaundice and 80% develop hepatomegaly. Exposures in childcare settings are often
not recognized until an adult staff member becomes symptomatic. Because the average
incubation period is 28 days, there may be a significant delay between exposure to the index
patient and recognition of symptoms in an adult.

Prophylaxis with hepatitis A vaccine (recommended between the ages of 12 months and 40
years) or immune globulin is indicated for exposed individuals who have not previously received
hepatitis A vaccine. Although vaccination and immune globulin have similar efficacy,
vaccination is preferred when appropriate, because it provides active and more durable
immunity. Immune globulin should be used for individuals younger than 12 months, older than
40 years, those who have vaccine contraindications, immunocompromised individuals, and
individuals with chronic liver disease. When given during the appropriate time frame (within 14
days), immune globulin is more than 85% effective at preventing symptomatic infections. If
immune globulin is not available, hepatitis A vaccine can be administered to individuals older
than 40 years who do not have vaccine contraindications. Either means of prophylaxis should be
administered within 14 days of the exposure.

Interferons were the first therapeutic agents for chronic viral hepatitides. Although used in the
treatment of infection with hepatitis B and C, interferon does not have a role in postexposure
prophylaxis of hepatitis A.

PREP Pearls
• Hepatitis A vaccine or immune globulin can be used for hepatitis A postexposure
prophylaxis and should be administered within 14 days of the exposure.
• Hepatitis A vaccine can be used in individuals who are vaccine eligible between the ages
of 1 and 40 years, and is preferred over immune globulin because it provides active and
more durable immunity.
• Immune globulin should be used for hepatitis A postexposure prophylaxis in individuals
younger than 12 months, older than 40 years, those who have vaccine contraindications,
immunocompromised individuals, and individuals with chronic liver disease.

ABP Content Specifications(s)


• Initiate appropriate post-exposure prophylaxis for hepatitis A virus infection

American Academy of Pediatrics 102


PREP ® Self-Assessment PREPSA 2021
Suggested Readings
• American Academy of Pediatrics. Hepatitis A. In: Kimberlin DW, Brady MT, Jackson
MA, Long SS, eds. Red Book: 2018 Report of the Committee on Infectious
Diseases. Itasca, IL: American Academy of Pediatrics; 2018:392-400. Red Book Online.
• Christenson J, Manaloor J. Hepatitis A, B, and C. Pediatr Rev. 2016;37(10):426-438.
DOI: 10.1542/pir.2015-0075.
• Hardikar W, Schwarz KB. Hepatitis. In: McInerny TK, Adam HM, Campbell DE,
DeWitt TG, Foy JM, Kamat DM, eds. American Academy of Pediatrics Textbook of
Pediatric Care. 2nd ed. Itasca, IL: American Academy of Pediatrics; 2016; chap
265:2131-2142. Pediatric Care Online

American Academy of Pediatrics 103


PREP ® Self-Assessment PREPSA 2021
Question 28
A mother brings her 6-year-old daughter for an evaluation for concerns about “knock knees.”
Her parents noticed that her knees seem to bump together when she runs. The girl denies any
pain. She is otherwise healthy with normal growth and development. Physical examination
reveals full range of motion and normal strength throughout both lower extremities. Genu
valgum is apparent on stance and gait evaluation. Supine intermalleolar measurement is 7 cm.

Of the following, the BEST next management step for this girl is
A. orthopedic referral
B. physical therapy
C. reassurance
D. reevaluation in 6 months

American Academy of Pediatrics 104


PREP ® Self-Assessment PREPSA 2021
Correct Answer: D
Parents often raise concerns about the potential development of varus (“bow-leg”) or valgus
(“knock-knee”) knee deformities in young children. Most of these cases are physiologic and
parents are often reassured when educated about the predictable pattern of leg alignment changes
in young children. However, an intermalleolar distance of 7 cm in a 6-year-old girl is at the
upper limits of normal and should be re-evaluated in 6 months. Parental reassurance is not
sufficient in this case.

Infants normally demonstrate varus knee alignment (genu varum) caused by intrauterine
positioning. Knee varus increases as children start to walk, but typically resolves by 2 years of
age, at which point the knees should transition to valgus alignment (genu valgum). In many
populations, physiologic knee valgus peaks at 4 years of age and then decreases until final
alignment is reached at about 7 years of age. However, there may be some racial and ethnic
variation in this timing, with recent studies showing that peak valgus occurs at 5 to 6 years of age
in Indian children and age 6 to 7 years in Turkish children. Most older children and adults have
mild valgus knee alignment.

Office assessment can help determine whether knee alignment is physiologic or represents
underlying pathology. The history should include:
• Timing of onset and progression of knee valgus
o Valgus onset before age 2 years, or which worsens after expected peak age,
warrants additional evaluation
• Associated symptoms
o Any pain or swelling suggests possible pathology
• Previous lower-extremity trauma, infection, or surgical procedures
o These may change patterns of growth and increase the risk of pathologic
malalignment
• Review of growth curve
o Obese children have higher rates of progressive valgus
o Short stature may be an indicator of underlying systemic contributors (such as
rickets, renal osteodystrophy, skeletal dysplasias)
• Review of timing of walking and motor development
o Children who learn to walk early often increase knee varus alignment
• Review and acknowledgment of concerns regarding perceived impact of knee alignment
on gait, movement, and appearance
• Family history of significant malalignment or associated medical conditions

Physical examination should include:


• Inspection of the knees, including with weight-bearing and walking, in ambulatory
children.
o Significant asymmetry in appearance or alignment warrants additional evaluation
• Measurement of knee alignment by measuring intercondylar or intermalleolar distances
(see Item C28A ).
o The child should stand or lie supine with knees fully extended and patellae facing
forward (patellar positioning is important to ensure accuracy)

American Academy of Pediatrics 105


PREP ® Self-Assessment PREPSA 2021
o In children with varus alignment, the medial malleoli of the ankles are placed
together, and the intercondylar distance is measured between the bony
prominences of the medial femoral condyles
o In children with valgus alignment, the medial femoral condyles are placed next to
each other, and the intermalleolar distance is determined

Most current recommendations for the evaluation and treatment of varus and valgus knee
deformities are based on normative values for knee alignment shown in Item C28B, which
depicts the normal progression of knee angle in white children in the United States. At present,
there are no specific recommendations that accommodate racial and ethnic variations. Providers
should periodically monitor children in whom knee alignment is of concern; apply their
understanding of the usual progression of knee alignment (varus → progressive valgus to peak
→ regression of valgus to final alignment); and consider additional evaluation for children who
are not following the expected course. For children at the outer limits of normal, like the girl in
this vignette, re-evaluation in 6 months is indicated.

Radiography in genu valgum can assist pediatric providers in determining whether there are any
associated medical conditions of concern, and whether orthopedic or other specialist referral is
needed. Radiography for genu valgum is indicated for children:
• Older than age 7 years with intermalleolar distances greater than 8 cm
• With intermalleolar measurements outside 2 standard deviations from the mean for age
(Item C28B)
• With other indicators described earlier that increase the risk of associated pathology

American Academy of Pediatrics 106


PREP ® Self-Assessment PREPSA 2021

Item C28B: Normal ranges of knee angle and intercondylar and intermalleolar distances in
children.
Reprinted with permission from Heath CH, Staheli LT. Normal limits of knee angle in white
children - denu varum and genu valgum. J Pediatr Orthop 1993:13:261.

Appropriate radiographic ordering and technique help to ensure accurate portrayal of knee
alignment; providers should indicate the following when ordering radiography for these
children:
• Bilateral full-length weight-bearing radiographs of the legs (standard knee radiographs
are not sufficient)
• Patellar alignment is important; both should be straight ahead, regardless of impact on
foot alignment

In older children, knee valgus is associated with increased risk for acute and overuse injuries (eg,
anterior knee pain and patellofemoral syndrome). Although bony alignment is static,
strengthening of the hip musculature can decrease the dynamic valgus stress across the knee and
may decrease associated symptoms. Physical therapy may be appropriate for children with knee
valgus who have knee pain, but would not be expected to affect the degree of angular deformity,
and would not be indicated for the asymptomatic young girl in the vignette.

PREP Pearls
• Physiologic genu valgus (“knock knee”) peaks in early childhood and then regresses to
adult values over the next several years.
• Radiography should be performed in children older than 7 years with intermalleolar
distances greater than 8 cm.

ABP Content Specifications(s)


• Recognize the clinical findings associated with various valgus and varus deformities, and
understand when referral is appropriate

Suggested Readings
• Sarwark JF, LaBella CR. Rotational and angular deformities: general treatment
guidelines. In: Pediatric Orthopedics and Sports Injuries: A Quick Reference
Guide. Itasca, IL: American Academy of Pediatrics; 2010:201-212.
• Weintraub B. Foot and leg problems. In: McInerny TK, Adam HM, Campbell DE,
DeWitt TG, Foy JM, Kamat DM, eds. American Academy of Pediatrics Textbook of
Pediatric Care. 2nd ed. Itasca, IL: American Academy of Pediatrics; 2016;chap
154:1366-1378. Pediatric Care Online.

American Academy of Pediatrics 107


PREP ® Self-Assessment PREPSA 2021
Question 29
A 5-year-old boy who is otherwise healthy is brought to the emergency department after 7 days
of fever and fatigue. He has not had cough, congestion, rhinorrhea, vomiting, or diarrhea. He
appears ill but is not in distress. He has a temperature of 39°C, a heart rate of 140 beats/min, a
blood pressure of 90/60 mm Hg, a respiratory rate of 40 breaths/min, and oxygen saturation of
95%. He has clear bilateral breath sounds. His heart rate and rhythm are regular. He has a grade
2/4 diastolic murmur that is not indicated in prior clinic notes. He has petechiae on his distal
extremities. Blood cultures and an echocardiogram are obtained. A vegetation is noted on his
mitral valve.

Of the following, the pathogen that is MOST likely to be the cause of this child’s infection is
A. Eikenella corrodens
B. Kingella kingae
C. Staphylococcus aureus
D. Streptococcus pneumoniae

American Academy of Pediatrics 108


PREP ® Self-Assessment PREPSA 2021
Correct Answer: C
The child in the vignette has signs and symptoms that are consistent with infective endocarditis
(IE). The most likely pathogen would be Staphylococcus aureus. The other listed pathogens are
known to cause IE but are not the most common pathogens to do so. Although endocarditis is
rare in children, it is associated with significant morbidity and mortality. The epidemiology of IE
has changed as the survival of those with congenital heart disease has improved, the use of
central venous catheters in general has increased, and the incidence of rheumatic heart disease
has decreased.

The most common pathogens associated with IE are streptococcus viridans group
(including Streptococcus sanguis, Streptococcus mitis, Streptococcus oralis, and Streptococcus
anginosus) and S aureus. The streptococcus viridans group has been associated with infecting
abnormal valves as seen in congenital heart disease and rheumatic fever. Staphylococcus spp.
have been seen in both structurally normal and abnormal valves. The AACEK organisms
(Aggregatibacter parainfluenzae [formerly Haemophilus parainfluenzae], Aggregatibacter
actinomycetemcomitans, Cardiobacterium hominis, Eikenella corrodens, and Kingella species)
are also known to cause IE, although less frequently. Candida and Aspergillus species can also
cause IE, particularly in hospitalized patients or those with central venous catheters.
Additionally, culture-negative IE is not uncommonly seen. Streptococcus pneumoniae is a
possible pathogen but is quite rare.

Clinically, IE presents as either a subacute or an acute process. In subacute IE, patients have
nonspecific symptoms for weeks, whereas acute IE is a rapidly progressive illness. Item
C29A lists the common manifestations of pediatric IE.

American Academy of Pediatrics 109


PREP ® Self-Assessment PREPSA 2021

In acute or subacute IE, the presentation is often nonspecific. A validated tool, the Duke criteria
(Item C29B), has been developed and modified to aid in the diagnosis of IE. The major criteria
needed to make the diagnosis include evidence of bloodstream infection and of endocardial
involvement. From a practical perspective, it is important to remember that skin organisms can
cause IE, so multiple blood cultures (three or more of adequate volume) are needed to distinguish
between contaminant and pathogen. Inflammatory markers can be elevated. Echocardiography is
needed to evaluate for vegetations, cardiac function, and valve injury. In children, transthoracic
echocardiography is often sensitive enough with adequate visualization. At times, in children for
whom visualization is poorer or who have more complex disease, transesophageal
echocardiography may be needed.

American Academy of Pediatrics 110


PREP ® Self-Assessment PREPSA 2021

American Academy of Pediatrics 111


PREP ® Self-Assessment PREPSA 2021

The initial management of IE includes broad-spectrum bactericidal antibiotics at high serum


levels (ie, vancomycin and gentamicin). Thereafter, the antibiotics can be tailored to fit the
patient’s age, presentation, cardiac status, organism, and sensitivities. A prolonged course is
necessary. Surgical procedures are sometimes warranted and should be considered in those with
heart failure symptoms that cannot be managed medically, prosthetic valves, embolic events, or
positive blood cultures for more than 5 to 7 days.

Antibiotic prophylaxis (Bragg L, Alvarez A) is recommended for procedures that have the
greatest potential to cause bacteremia with an organism that can cause IE in patients who are at
the greatest potential risk. Procedures involving the mucosa of the oropharynx, respiratory, skin,
and musculoskeletal systems (but not the gastrointestinal or genitourinary systems) should be
considered. Patients who have prosthetic material on a cardiac valve, unrepaired cyanotic heart
disease, or residual defects after repair are among those who are at highest risk and will need
prophylaxis.

PREP Pearls
• The most common pathogens associated with infective endocarditis are streptococcus
viridans group and Staphylococcus aureus.
• The clinical presentation of infective endocarditis can be subacute or acute and is often
nonspecific. A high index of suspicion is needed.
• The modified Duke criteria can aid in the diagnosis of infective endocarditis.
• Treatment initially includes broad-spectrum antibiotics that can then be tailored to the
clinical situation.

ABP Content Specifications(s)


• Recognize the clinical findings associated with infective endocarditis and provide
appropriate initial management
• Recognize pathogens commonly associated with infective endocarditis
• Plan appropriate prophylaxis for infective endocarditis
• Plan an appropriate diagnostic evaluation of infective endocarditis
• Understand the natural history of infective endocarditis

American Academy of Pediatrics 112


PREP ® Self-Assessment PREPSA 2021
Suggested Readings
• Baltimore RS, Gewitz M, Baddour LM, et al; American Heart Association Rheumatic
Fever, Endocarditis, and Kawasaki Disease Committee of the Council on Cardiovascular
Disease in the Young and the Council on Cardiovascular and Stroke Nursing. Infective
endocarditis in childhood: 2015 update: a scientific statement from the American Heart
Association. Circulation. 2015;132(15):1487-1515.
doi:10.1161/CIR.0000000000000298.
• Bragg L, Alvarez A. Endocarditis. Pediatr Rev. 2014;35(4):162-167. doi:10.1542/pir.35-
4-162.
• McCulloch MA, Gajarski RJ. Congenital and acquired heart disease. In: McInerny TK,
Adam HM, Campbell DE, DeWitt TG, Foy JM, Karnat DM, eds. American Academy of
Pediatrics Textbook of Pediatric Care. 2nd ed. Elk Grove Village, IL: American
Academy of Pediatrics; 2017:1883-1916. Pediatric Care Online.
• Wilson W, Taubert KA, Gewitz M, et al; American Heart Association Rheumatic Fever,
Endocarditis, and Kawasaki Disease Committee; American Heart Association Council on
Cardiovascular Disease in the Young; American Heart Association Council on Clinical
Cardiology; American Heart Association Council on Cardiovascular Surgery and
Anesthesia; Quality of Care and Outcomes Research Interdisciplinary Working Group.
Prevention of infective endocarditis: guidelines from the American Heart Association: a
guideline from the American Heart Association Rheumatic Fever, Endocarditis, and
Kawasaki Disease Committee, Council on Cardiovascular Disease in the Young, and the
Council on Clinical Cardiology, Council on Cardiovascular Surgery and Anesthesia, and
the Quality of Care and Outcomes Research Interdisciplinary Working Group (published
correction appears in Circulation. 2007;116[15]:e376-e377). Circulation.
2007;116(15):1736-1754.

American Academy of Pediatrics 113


PREP ® Self-Assessment PREPSA 2021
Question 30
A 12-year-old boy with myotonic dystrophy requires nocturnal ventilator support via
tracheostomy. He takes oral nutrition supplemented with feedings via gastrostomy tube. He has
had intermittent episodes of fever associated with increased tracheal secretions. A modified
barium swallow study demonstrates oropharyngeal dysphagia; the vocal cords are penetrated by
thin liquids but not by semisolids.

Of the following, the MOST appropriate next management step for this boy is
A. limitation of oral intake to solids and semisolids
B. pharmacologic manipulation of oral secretions
C. placement of a cuffed tracheostomy tube
D. use of a Passey-Muir speaking valve

American Academy of Pediatrics 114


PREP ® Self-Assessment PREPSA 2021
Correct Answer: A
The boy in the vignette is at risk for aspiration of thin liquids, as demonstrated by the modified
barium swallow, though he does not appear to aspirate semisolids. He should have an oral diet
consisting of those densities that he can safely swallow, with supplemental liquids given by
gastrostomy tube. Thickening of his secretions pharmacologically will not decrease his risk of
aspiration with exogenous liquids, nor will the use of a cuffed tracheostomy tube or a Passey-
Muir speaking valve.

Although some studies suggest that oral feedings are possible in children with tracheostomies
and dysphagia, many studies show that just the presence of a tracheostomy promotes dysphagia
and potential aspiration. Even oral secretions may be aspirated by children with a tracheostomy,
both with or without a cuffed tube. Cuffed tracheostomy tubes are helpful in limiting the driving
pressure and volume needed to deliver positive pressure ventilation without a large air leak, but
they do not prevent aspiration. Secretions and oral liquids may pool in the valleculae and be
aspirated into the lower airway. Barium and methylene blue dye studies have shown that 30% to
50% of patients with tracheostomies aspirate oral liquids, and 50% to 75% of these aspirations
are silent, with no cough or attempt to clear the aspirate.

PREP Pearls
• Most patients with a tracheostomy experience at least minimal aspiration of liquids.
• Children with a tracheostomy may safely swallow feedings of some consistencies; a
modified swallow study should be performed to assess swallowing ability.
• A cuffed tracheostomy tube does not prevent aspiration.

ABP Content Specifications(s)


• Understand the effect of a tracheostomy on aspiration

Suggested Readings
• Lepainteur M, Ogne A, Claire B, et al. Risk factors for respiratory tract bacterial
colonization in adults with neuromuscular or neurologic disorders and chronic
tracheostomy. Respir Med. 2019;152:32-36.
• Sanders CD, Guimbellot J, Muhlebach, MS, Lin F-C, Gilligan P, Esther CR Jr.
Tracheostomy in children: epidemiology and clinical outcomes. Pediatr Pulmonol.
2018;53:1269-1275. doi: 10.1002/ppul.24071.
• Streppel M, Veder LL, Pullens B, Joosten KFM. Swallowing problems in children with
tracheostomy tubes. Int J Pediatr Otorhinolaryngol. 2019;124:30-33.
doi: 10.1016/j.ijporl.2019.05.003.

American Academy of Pediatrics 115


PREP ® Self-Assessment PREPSA 2021
Question 31
A 16-year-old adolescent girl with a 6-week history of abdominal pain is seen in the office. The
pain is dull and intermittent and occurs daily. It is primarily localized to the left lower quadrant
and self-resolves after a few minutes. She has occasional diarrhea but no nausea, vomiting, fever,
or dysuria. She reports having a decreased appetite and a 1.4-kg weight loss over the past 6
months. She has never been sexually active. During the confidential psychosocial interview, she
discloses that she is worried about her upcoming standardized college admissions test. Her three
older siblings have done extremely well on their testing, but she does not feel prepared and does
not want to disappoint her parents. She states that she would never discuss how she is feeling
with her parents because she has always been taught to attempt to think of a solution to a
problem before asking for help.

Of the following, the BEST next step in management is to


A. order abdominal ultrasonography
B. order a complete blood cell count and erythrocyte sedimentation rate
C. provide reassurance that she will do well on her college admissions testing
D. refer her for individual and family therapy

American Academy of Pediatrics 116


PREP ® Self-Assessment PREPSA 2021
Correct Answer: D
The adolescent girl described in the vignette is displaying symptoms related to anxiety. An
abdominal ultrasonogram, complete blood cell count, and erythrocyte sedimentation rate would
be appropriate to rule out an organic cause for the girl’s pain. However, with her sharing
concerns about the upcoming test, the best step would be to facilitate individual and family
therapy so she has an outlet to express her feelings and develop coping skills, as well as improve
communication with her parents. It would be presumptive to assure her that she will do well on
her college admissions test if she is unable to deal with her anxiety.

Anxiety is a physiological response to stress. Anxiety disorders are the most common psychiatric
illnesses in children and adolescents, affecting up to 10% of this population. Females are twice
as likely as males to have an anxiety-related disorder. Anxiety can begin during childhood,
become more pronounced during the adolescent years, and continue into adulthood. According
to Diagnostic and Statistical Manual of Mental Disorders, 5th edition, general anxiety disorder
is characterized by excessive worry of at least 6 months’ duration. Adolescents with anxiety may
exhibit fatigue, restlessness, difficulty concentrating, irritability, and sleep disturbance. Affected
children and adolescents often have physical complaints such as headache and abdominal pain.
Individuals with anxiety may experience academic performance anxiety that impedes completion
of assignments and test taking owing to fear of not performing well.

Parents are important influences on their children, and effective parenting plays a role in how
adolescents deal with adversity. Adolescents often model the behavior of their parents in how
they approach life issues, whether it be dealing with drugs and alcohol or addressing stress,
depression, and anxiety. It is imperative to have the parents on board in deciding the best plan of
care for their child. The primary care provider often plays the role of mediator and has to assess
how a parent feels about therapy or mental health care, while also taking into consideration
cultural differences. Many ethnic groups do not openly speak about mental health issues or
believe in involving outsiders in family-related issues. For adolescents who are experiencing
physical symptoms without any organic disease, it is important to acknowledge their symptoms
and reassure them that they are physically healthy and can participate in everyday activities such
as school, clubs, and sports. Individual and family therapy have been useful tools in reducing
illness behavior.

PREP Pearls
• Anxiety disorders are the most common psychiatric illnesses in children and adolescents.
• Parents are important influences on their children, and effective parenting plays a role in
how adolescents deal with adversity.
• Individual and family therapy are useful tools in reducing illness behavior.

ABP Content Specifications(s)


• Understand the importance of evaluating family dynamics in adolescent patients,
including stressors and methods of coping with stress
• Recognize the importance of the family in modeling adolescent behaviors

American Academy of Pediatrics 117


PREP ® Self-Assessment PREPSA 2021
Suggested Readings
• Alderman EM, Breuner CC; Committee on Adolescence. Unique needs of the
adolescent. Pediatrics. 2019;144(6):e20193150. doi:10.1542/peds.2019-3150.
• Korczak DJ. Monga S. Depression and anxiety disorders. In: Neinstein LS, Katzman DK,
Callahan ST, Gordon CM, Joffe A, Rickert VI, eds. Neinstein’s Adolescent and Young
Adult Health Care: A Practical Guide. 6th ed. Philadelphia, PA: Wolters Kluwer;
2016:578-588.
• Murry VM, Lippold MA. Parenting practices in diverse family structures: examination of
adolescents' development and adjustment. J Res Adolesc. 2018;28(3):650-664.
doi:10.1111/jora.12390.
• Wissow LS. Anxiety. In: McInerny TK, Adam HM, Campbell DE, DeWitt TG, Foy JM,
Kamat DM, eds. American Academy of Pediatrics Textbook of Pediatric Care. 2nd ed.
Elk Grove Village, IL: American Academy of Pediatrics; 2017:1209-1216. Pediatric Care
Online .

American Academy of Pediatrics 118


PREP ® Self-Assessment PREPSA 2021
Question 32
A 15-year-old adolescent is seen in the office for evaluation of recurrent headaches for the past 5
years. Her headaches are described as a bifrontal throbbing pain with associated photophobia,
phonophobia, nausea, and vomiting. She feels fatigued and experiences visual distortions with
sparkling lights before headache onset. Her headaches occur approximately once weekly and
respond to appropriately dosed nonsteroidal anti-inflammatory drugs (NSAIDs). The girl misses
school approximately once per semester because of her headaches. She denies waking at night
with headache, initiation of headache with valsalva maneuver, or associated focal neurologic
deficit. No triggers have been identified. Her neurologic examination findings in the office are
normal. Fundoscopy reveals sharp disc margins with no evidence of papilledema. Magnetic
resonance imaging of the brain, performed 3 years earlier, was normal.

Of the following, in addition to continued use of NSAIDs, the BEST next management step for
this girl is
A. initiation of vitamin therapy
B. initiation of prophylactic medication
C. initiation of triptan
D. lifestyle trigger identification and modification

American Academy of Pediatrics 119


PREP ® Self-Assessment PREPSA 2021
Correct Answer: D
The best next step for the adolescent in the vignette is identification and modification of lifestyle
triggers for her headache. She meets diagnostic criteria for episodic migraine with aura. The
International Classification for Headache defines migraine with aura as an episodic, moderately
severe, pulsatile, bilateral, frontoparietal headache with 2 of 4 associated symptoms:
photophobia, phonophobia, nausea, and vomiting. For diagnosis of episodic migraine there must
be at least 4 lifetime attacks lasting 2 to 72 hours, not better explained by another condition. An
aura can include visual, sensory, motor, language, or brainstem symptoms which are reversible
and often accompanied by a headache. Concerning features, such as age less than 6 years,
waking at night with headache, initiation of headache with valsalva maneuver, or associated
focal neurologic deficit, would indicate a need for additional investigation. The complete
International Classification of Headache Disorders criteria can be found at: https://ichd-3.org.

Management of migraine headache is multimodal and includes 3 components:


1. Lifestyle migraine management
2. Abortive medications and plan
3. Preventive medications

Lifestyle migraine management provides the foundation for treatment. Identification and
anticipatory guidance around common headache triggers and how to address them constitute a
critical part of migraine care. Lack of sleep, diet factors, inadequate hydration, lack of exercise,
and stress are common triggers for pediatric migraine. A headache diary can allow for
identification of triggers; the pediatrician can provide feedback on lifestyle changes that would
benefit the child. Online resources such as “Migraine
Buddy” https://migrainebuddy.com and www.headachereliefguide.com can be helpful tools for
trigger identification and lifestyle management.

Abortive medications, including nonsteroidal anti-inflammatory drugs (NSAIDs) and triptans,


are most effective when used early in a migraine attack, if there are no contraindications.
Associated symptoms, such as nausea and vomiting, can be treated with antiemetics. Children
should be counseled regarding medication overuse, which can itself cause headaches; this
becomes a risk when using acetaminophen or NSAIDs more than 14 days in a month and triptans
more than 9 days in a month. The 2019 guidelines for acute management of pediatric migraine
can be found at https://www.aan.com/Guidelines/home/GetGuidelineContent/970.

For frequent headaches, preventive medications may be considered. Topiramate is approved by


the US Food and Drug Administration in the prevention of pediatric migraine. Other commonly
used medications include amitriptyline, cyproheptadine, and propranolol. However, clinical trials
have failed to consistently demonstrate the efficacy of common preventive medications over
placebo in pediatric migraine management. High rates of placebo responses in pediatric
populations, selection criteria, and active cointerventions (lifestyle coaching) have been
identified as potential contributing factors to trial results. Cognitive behavioral therapy in
combination with preventive medications has been shown to be effective in adolescents.
Magnesium, riboflavin (B2), and melatonin have been shown in adult randomized control trials
to be more effective than placebo, with a favorable side effect profile. The 2019 guidelines for

American Academy of Pediatrics 120


PREP ® Self-Assessment PREPSA 2021
preventive management of pediatric migraine is available
at https://www.aan.com/Guidelines/home/GetGuidelineContent/971.
The adolescent in this vignette has episodic migraines that are not yet frequent or disabling
enough to require a trial of vitamins or prophylactic medications. She responds appropriately to
NSAIDs during migraine attacks, negating the need for triptans. Lifestyle migraine management
and trigger identification will be beneficial as part of her ongoing headache management.

PREP Pearls
• Migraine headaches are moderately severe, episodic, pulsatile pain events with associated
photophobia, phonophobia, nausea, and vomiting.
• Migraine management includes 3 components: abortive medications, preventive
medications, and lifestyle management.
• Trigger identification and lifestyle management form the foundation of migraine care.

ABP Content Specifications(s)


• Plan appropriate abortive therapy for acute migraine
• Plan the appropriate management of headache of various origins
• Plan appropriate prophylaxis for recurrent migraine

Suggested Readings
• Gladstein J. Headache. In: McInerny TK, Adam HM, Campbell DE, DeWitt TG, Foy JM,
Kamat DM, eds. American Academy of Pediatrics Textbook of Pediatric Care. 2nd ed.
Itasca, IL: American Academy of Pediatrics; 2017chap 157:1404-1407. Pediatric Care
Online.
• Klein J, Koch T. Headache in Children. Pediatr Rev. 2020;41(4): 159-171;
doi: 10.1542/pir.2017-0012.
• Oskoui M, Pringsheim T, Holler-Managan Y, et al. Practice guideline update summary:
acute treatment of migraine in children and adolescents. Neurology. 2019;93(11):487-
499. doi: 10.1212/WNL.0000000000008095.
• Oskoui M, Pringsheim T, Lori Billinghurst L, et al. Practice guideline update summary:
pharmacologic treatment for pediatric migraine prevention. Neurology. 2019;93(11):500-
509. doi: 10.1212/WNL.0000000000008105.
• Powers SW, Coffey CS, Chamberlin LA, et al. Trial of amitriptyline, topiramate, and
placebo for pediatric migraine. N Engl J Med. 2017;376(2):115-124.
doi: 10.1056/NEJMoa1610384.

American Academy of Pediatrics 121


PREP ® Self-Assessment PREPSA 2021
Question 33
A 4-month-old infant is seen for evaluation of excessive tearing and blinking since birth. His
parents report wiping clear discharge from his eyes and nares multiple times per day. He blinks
often and prefers to keep his eyes closed when exposed to bright lights. There has been no fever,
eye redness, eye swelling, yellow eye discharge, or cough. He is afebrile and in the 50th
percentile for weight and height. On eye examination, he does not fixate on objects or track.
There is no periorbital edema or erythema, and his conjunctivae and sclerae are clear. He has
bilateral clear discharge, opaque red reflexes, and mild corneal clouding.

Of the following, the MOST likely diagnosis for this infant is


A. acute dacryocystitis
B. cataract
C. glaucoma
D. nasolacrimal duct obstruction

American Academy of Pediatrics 122


PREP ® Self-Assessment PREPSA 2021
Correct Answer: C
The infant in this vignette with corneal clouding, photophobia, and chronic or intermittent
tearing is demonstrating the classic presentation of congenital or early-onset glaucoma.
Glaucoma is a type of optic neuropathy that may be associated with elevated intraocular
pressure. Primary congenital, infantile, and juvenile glaucoma are rare; secondary glaucoma may
develop in the setting of Sturge-Weber syndrome, aniridia, or retinopathy of prematurity or as a
consequence of cataract surgery, tumors, trauma, or exposure to steroids.

Prompt recognition and management of congenital glaucoma is crucial for prevention of


blindness. Every infant should be evaluated for a red reflex in a darkened room with an
ophthalmoscope. First, each eye should be examined individually from a distance of 1 to 2 feet,
followed by Brückner reflex testing, during which both eyes are examined simultaneously from a
distance of 2 to 3 feet. Immediate referral to an ophthalmologist should occur for findings of
asymmetric, absent, dulled, or opaque red reflex; dark spots in the red reflex; or leukocoria
(white reflex). After the age of 3 months, poor tracking, lack of fixation, nystagmus, or squinting
should warrant a referral. The definitive treatment for most pediatric glaucomas is surgical
intervention.

Acute dacryocystitis is often a complication of a dacryocystocele and presents with edema,


erythema, and tenderness of the lacrimal sac located inferior to the medial canthus. Purulent
drainage may be present. In consultation with a pediatric ophthalmologist, the clinician should
ensure that patients with acute dacryocystitis receive empiric antibiotics while awaiting culture
results. Congenital cataracts may be caused by congenital infections, teratogens, genetic or
metabolic diseases, and primary ocular disorders. The presentation of congenital cataracts is
highly variable, but findings may include asymmetric red reflex, leukocoria, nystagmus,
strabismus, and photophobia. The most common cause of tearing in newborns and infants is
nasolacrimal duct obstruction. However, unlike patients with glaucoma, nasolacrimal duct
obstruction does not present with photophobia, corneal clouding, or nasal discharge.

PREP Pearls
• Every newborn should be evaluated with red reflex testing and Brückner reflex testing
before discharge and at each health supervision visit.
• Prompt recognition and management of congenital glaucoma is crucial for prevention of
blindness.
• The classic presentation of congenital or early-onset glaucoma is corneal clouding,
photophobia, and chronic or intermittent tearing.

ABP Content Specifications(s)


• Recognize the clinical findings associated with congenital glaucoma

American Academy of Pediatrics 123


PREP ® Self-Assessment PREPSA 2021
Suggested Readings
• Ali MJ. Pediatric acute dacryocystitis. Ophthal Plast Reconstr Surg. 2015;31(5):341-347.
doi:10.1097/IOP.0000000000000472.
• Davenport KM, Patel AA. Cataracts. Pediatr Rev. 2011;32(2):82-83. doi:10.1542/pir.32-
2-82.
• Donahue SP, Baker CN; Section on Ophthalmology, American Academy of Pediatrics;
Committee on Practice and Ambulatory Medicine, American Academy of Pediatrics;
American Academy of Ophthalmology; American Association for Pediatric
Ophthalmology and Strabismus; American Association of Certified Orthoptists. Visual
system assessment in infants, children, and young adults by pediatricians. Pediatrics.
2016;137(1):28-30. doi:10.1542/peds.2015-3596.
• Schnall BM. Pediatric nasolacrimal duct obstruction. Curr Opin Ophthalmol.
2013;24(5):421-424.

American Academy of Pediatrics 124


PREP ® Self-Assessment PREPSA 2021
Question 34
A 2-month-old male infant is seen for his health supervision visit. He is tracking, smiling
socially, and holding his head up without support for a few seconds. He has a 4-year-old sister
who is developmentally normal. His mother’s brother died at 20 years of age with complications
related to a muscle disease. He had calf hypertrophy and used his hands to crawl from a lying to
a standing position. He required the use of a wheelchair by 10 years of age. The mother reports
that she has no muscle weakness or cramps. The mother underwent echocardiography 5 years
ago for chest pain that showed mild dilated cardiomyopathy felt to be related to her brother’s
disorder. The parents are concerned about their son’s risk of experiencing the same condition.

Of the following, the MOST accurate percentage chance that this infant has the same condition
as his uncle is
A. 25%
B. 50%
C. 75%
D. 100%

American Academy of Pediatrics 125


PREP ® Self-Assessment PREPSA 2021
Correct Answer: B
The condition seen in the family described in the vignette is Duchenne muscular dystrophy
(DMD). This disorder of the muscle protein dystrophin is inherited in an X-linked recessive
manner (Item C34). Clinical features in affected male patients include delayed achievement of
motor milestones, calf pseudohypertrophy in early stages, and proximal muscle weakness that
leads to wheelchair dependence by the age of 10 to 12 years. Gowers sign, which is described in
the vignette, indicates proximal muscle weakness whereby a patient has to use his arms to
support the trunk when rising from a lying to a standing position. Cardiac muscle is also affected,
with cardiomyopathy in all affected men after age 18 years. Common causes of death associated
with DMD include progressive cardiomyopathy and respiratory complications, as described for
the patient’s uncle. Tests to confirm the diagnosis include elevated serum creatine kinase and
molecular analysis of the DMD gene to look for the disease-causing variant. Skeletal muscle
biopsy was used in the past in assessing for nonspecific dystrophic changes on histology and the
quantity of the dystrophin protein via Western blot analysis. Female carriers are reported to have
mild muscle weakness and dilated cardiomyopathy; hence, cardiac surveillance also should be
offered to female carriers in the family. In the vignette, the patient’s uncle had DMD, the mother
is symptomatic with dilated cardiomyopathy, and the maternal grandmother is an obligate carrier
because two of her children are affected. The risk of the infant’s having DMD is 50%, because
the mother could pass on either the mutated or the normal DMD allele to her son.

American Academy of Pediatrics 126


PREP ® Self-Assessment PREPSA 2021

The other response choices are incorrect because the risk of the mother’s passing the
mutated DMD allele on to the patient is 50%.

PREP Pearls
• Duchenne muscular dystrophy is an X-linked recessive disorder.
• Males with Duchenne muscular dystrophy have calf pseudohypertrophy, elevated
creatine kinase level, proximal muscle weakness leading to Gowers sign when standing
from a lying position, and cardiomyopathy.
• Cardiac surveillance is recommended for female carriers, who may develop
cardiomyopathy.

ABP Content Specifications(s)


• Recognize the inheritance pattern associated with X-linked recessive disorders

American Academy of Pediatrics 127


PREP ® Self-Assessment PREPSA 2021
Suggested Readings
• Darras BT, Urion DK, Ghosh PS. Dystrophinopathies. In: Adam MP, Ardinger HH,
Pagon RA, et al, eds. GeneReviews. Seattle, WA: University of Washington, Seattle;
1993-2020. 2000 Sep 5 [updated 2018 Apr
26]. https://www.ncbi.nlm.nih.gov/books/NBK1119/ .
• Moxley RT III, Ciafaloni E. Muscular dystrophy. In: McInerny TK, Adam HM,
Campbell DE, DeWitt TG, Foy JM, Kamat DM, eds. American Academy of Pediatrics
Textbook of Pediatric Care. 2nd ed. Elk Grove Village, IL: American Academy of
Pediatrics; 2017:2344-2357. Pediatric Care Online .
• Tsao CY. Muscle disease (published correction appears in Pediatr Rev. 2014;35[3]:137).
Pediatr Rev. 2014;35(2):49-61.doi:10.1542/pir.35-2-49.

American Academy of Pediatrics 128


PREP ® Self-Assessment PREPSA 2021
Question 35
An 8-year-old boy with a 3-day history of low-grade fever, sore throat, nasal discharge, cough,
and malaise is seen in the office. He recently returned to school after summer break. He appears
well and has a temperature of 38.2°C, a heart rate of 102 beats/min, a respiratory rate of 18
breaths/min, and a blood pressure of 108/67 mm Hg. There is redness of the pharyngeal mucosa
without exudate. There is nontender, shotty, anterior cervical adenopathy. His right tympanic
membrane is erythematous. The remainder of his physical examination findings are normal.

Of the following, the MOST likely etiology of this child’s illness is


A. adenovirus
B. influenza
C. respiratory syncytial virus
D. rhinovirus

American Academy of Pediatrics 129


PREP ® Self-Assessment PREPSA 2021
Correct Answer: D
The patient described in the vignette has clinical features that are consistent with the diagnosis of
the common cold (or rhinosinusitis). Although all the community-acquired respiratory viruses
noted in the responses can cause a common cold, rhinovirus is the most frequently implicated
pathogen in children and adults, accounting for almost 50% of cases of upper respiratory tract
infections. Rhinovirus infection occurs throughout the year, with peak activity from autumn
through spring. There are many serotypes (>150) of rhinovirus and although immunity is type-
specific, the duration of protection is temporary and short-lived. The serotypes of rhinovirus in
any community vary from season to season. Children often experience at least two episodes of
rhinovirus infections annually, and more than 90% of individuals have at least one rhinovirus
infection annually. Transmission of rhinovirus infection occurs via person-to-person contact by
respiratory tract secretions or large-particle aerosol spread.

The incubation period of rhinovirus infection ranges from 2 to 3 days. Common clinical features
include nasal discharge, sore throat, cough with associated malaise, headache, and low-grade
fever. The typical duration of upper respiratory tract illness is around 5 to 7 days, but it may
persist for 10 to 14 days in 25% of affected individuals. In children with asthma, infection with
rhinovirus may result in a severe and prolonged illness. In addition to the common cold,
rhinoviruses can cause otitis media, bronchiolitis, and pneumonia in infants and children. In
immunocompromised hosts, the most common manifestation of rhinovirus infection is
pneumonia.

As with other community-acquired respiratory viruses, rapid diagnosis by reverse-transcriptase


polymerase chain reaction assay of nasopharyngeal specimen (as part of multiplex polymerase
chain reaction) is the preferred diagnostic method for detecting rhinovirus with reported
sensitivity of 90% to 100% and specificity around 95%. Owing to the self-limited nature of this
infection, diagnostic testing is rarely indicated in immunocompetent hosts. Given the genetic
similarity of primers used in detection of rhinovirus and enterovirus, laboratories often report a
positive rhinovirus test result in conjunction with enterovirus in the respiratory virus panel.
Because of the ubiquitous occurrence of rhinovirus infection and prolonged virus shedding (up to
7 weeks or longer), detection of virus by itself does not confer causality in patients. Detection of
positive rhinovirus in infants usually represents acquisition of infection within the past month.
Rhinovirus infections are most often self-limited, and treatment is supportive. In hospital
settings, droplet precautions for the duration of illness are recommended for patients admitted
with symptomatic disease. Contact precautions may be indicated in hospitalized children
(especially infants) with copious nasal secretions and potential close contacts. Virus shedding
may be prolonged in immunocompromised patients, thus necessitating extended periods of
contact precautions. Control measures include hand hygiene.

Of the community-acquired respiratory viruses listed in the responses, respiratory syncytial virus
and influenza are responsible for most of the respiratory infections seen during the winter
months. Although adenovirus infections can occur throughout the year with a variety of clinical
presentations, rhinovirus is the preferred response choice given the clinical presentation of mild
upper respiratory tract illness in a child returning to school after summer vacation.

American Academy of Pediatrics 130


PREP ® Self-Assessment PREPSA 2021
PREP Pearls
• Rhinovirus is the most frequently implicated community respiratory viral pathogen
causing the common cold in children and adults.
• Rhinovirus infection occurs throughout the year, with peak activity from autumn through
spring.
• In addition to the common cold, rhinoviruses can cause otitis media, bronchiolitis, and
pneumonia in infants and children.

ABP Content Specifications(s)


• Recognize the clinical features associated with rhinovirus infection
• Recognize the epidemiology of rhinovirus infection

Suggested Readings
• Fischer H. Common cold. In: McInerny TK, Adam HM, Campbell DE, DeWitt TG, Foy
JM, Kamat DM, eds. American Academy of Pediatrics Textbook of Pediatric Care. 2nd
ed. Elk Grove Village, IL: American Academy of Pediatrics; 2017:1881-1882. Pediatric
Care Online .
• Noor A, Fiorito T, Krilov LR. Cold weather viruses. Pediatr Rev. 2019;40(10):497-507.
doi:10.1542/pir.2018-0237.

American Academy of Pediatrics 131


PREP ® Self-Assessment PREPSA 2021
Question 36
A 5-year-old boy is brought to the office by his mother who is concerned that her son has been
intentionally banging his head. When he is watching videos, he may bang his head several times
against the back of his chair. He may bang his head against the wall when he does not want to do
something or when he is upset. At bedtime, the boy bangs his head on his pillow until he falls
asleep. His mother is worried that he is having headaches or may cause himself serious injury.

Of the following, the MOST appropriate next step is


A. applied behavioral analysis therapy
B. magnetic resonance imaging of the brain
C. reassurance that these behaviors will resolve with time
D. screening for a developmental disability

American Academy of Pediatrics 132


PREP ® Self-Assessment PREPSA 2021
Correct Answer: D
Head-banging is a repetitive motor behavior (stereotypy) that usually resolves by age 4 years in
typically developing children. Persistence after age 5 years is associated with developmental
disorders such as autism or intellectual disability. Thus, the most appropriate next step would be
to screen for those conditions in this 5-year-old boy.

Head-banging occurs in 5% to 15% of typically developing children. It usually begins at 9 to 18


months of age and stops by age 4 years. It occurs more commonly in boys. Some children will
bang their forehead on walls, the floor, a mattress, or pillow; others will bang their occiput on a
carseat. The behavior can last for less than 15 minutes to more than an hour. Head-banging may
occur at bedtime or when the child is upset. It can result in bruising, abrasions, calluses, or hair
loss. Significant head injury is rare.

The pediatric provider should provide reassurance about the head-banging to the parents of the
toddler or preschooler, who is otherwise healthy and typically developing. Parents should be
instructed to redirect their child’s behavior and to avoid giving undue attention to the head-
banging because they may inadvertently reinforce it. Providing reassurance would not be the
appropriate next step for the 5-year-old child in the vignette.

Head-banging can be more persistent and severe in a child with developmental disability or
severe sensory impairment (ie, vision, hearing). In the context of a child with significantly
impaired communication skills, self-injurious behaviors such as head banging, should prompt
evaluation for painful medical conditions such as otitis media, sinusitis, or constipation,
particularly when the behavior is acute. Applied behavioral analysis therapy may be an
appropriate future step if this child were confirmed to have an autism spectrum disorder.
Magnetic resonance imaging of the brain is not warranted at this time; it is rare for children with
head-banging behavior to cause themselves significant injury.

PREP Pearls
• Head-banging is a repetitive motor behavior (stereotypy) that usually resolves by age 4
years in typically developing children. Persistence after age 5 years is associated with
developmental disorders such as autism or intellectual disability.
• Parents of typically developing toddlers or preschoolers who bang their heads should be
instructed to redirect their child’s behavior and to avoid giving undue attention to the
head-banging because they may inadvertently reinforce it.
• In the context of a child with significantly impaired communication skills, self-injurious
behaviors such as head-banging should prompt evaluation for painful medical conditions
such as otitis media, sinusitis, or constipation, particularly when the behavior is acute.

ABP Content Specifications(s)


• Plan the appropriate management of head banging in toddlers and preschool-age children

American Academy of Pediatrics 133


PREP ® Self-Assessment PREPSA 2021
Suggested Readings
• Blum NJ, Pipan ME. Basics of child behavior and primary care management of common
behavioral problems. In: Voight RG, Macias MM, Myers SM, Tapia CD,
eds. Developmental and Behavioral Pediatrics. Itasca, IL: American Academy of
Pediatrics; 2018:91-110.
• Sarles RM, Edwards S. Self-stimulating behaviors. In: McInerny TK, Adam HM,
Campbell DE, DeWitt TG, Foy JM, Kamat DM, eds. American Academy of Pediatrics
Textbook of Pediatric Care. 2nd ed. Itasca, IL: American Academy of Pediatrics;
2016;chap 192:1582-1584. Pediatric Care Online.
• Senturias YSN. Behavior patterns in infancy and preschool. In: Feld LG, Mahan JD,
eds. Succinct Pediatrics Book 3: Evaluation and Management for Newborn, Genetic,
Neurologic, and Developmental-Behavioral Disorders. Elk Grove Village, IL: American
Academy of Pediatrics; 2018:435-446.
• Stein MS, Blum NJ, Lukasik MK. Self-injury and mental retardation in a 7-year-old
boy. J Dev Behav Pediatr. 2005;26(3):241-245. doi: 0.1097/00004703-200506000-
00013.

American Academy of Pediatrics 134


PREP ® Self-Assessment PREPSA 2021
Question 37
A prospective randomized controlled study is being conducted to assess whether patients with
bronchiolitis benefit from nebulized hypertonic saline treatments given at home. A cohort of 600
patients diagnosed with bronchiolitis is randomly assigned to 3 groups receiving either no
treatment, once-daily treatment, or treatment 3 times per day. Symptoms are analyzed by means
of a symptom checklist daily over 7 days. Of the 600 enrolled patients, 510 appropriately
completed all 7 daily checklists. There is concern about how this might influence the results of
the study.

Of the following, the BEST way to maintain study power and avoid compliance bias in this study
is to
A. decrease the duration of the study
B. increase study enrollment
C. use intention-to-treat analysis
D. use per-protocol analysis

American Academy of Pediatrics 135


PREP ® Self-Assessment PREPSA 2021

Correct Answer: C
The best way to maintain study power and avoid compliance bias in the study described in the
vignette is to use intention-to-treat analysis. Changing the length of the study or increasing the
number of patients enrolled at the beginning of the study will not improve the percentage of
appropriately completed forms.

When per-protocol analysis is used, only those who complete the study are included in the final
analysis. When intention-to-treat analysis is used in a prospective randomized controlled study,
the analysis includes data from all patients who were randomly assigned to a group even if they
did not complete the study. Using intention-to-treat analysis can eliminate bias that arises from
additional factors, such as earlier discharge of children with mild symptoms who are not
receiving treatment. In this example, using intention-to-treat analysis, all patients enrolled would
be analyzed in the group to which they were originally randomly assigned—no treatment, once-
daily treatment, or treatment 3 times per day—regardless of whether or not the checklist was
completed. The sample size that drives the power of a study is preserved and remains at 600 even
after 200 patients do not complete the symptom checklist.

For example, sample data in Item C37A (based on the study in the vignette) are studied with
per-protocol analysis and Item C37B with intention-to-treat analysis. In Item C37A, the final
results appear as though more children have moderate to severe disease, because children with
mild disease were discharged and excluded from the final analysis. Item C37B, in which these
patients are included in the final statistics, has a larger percentage of mild disease. The intention-
to-treat analysis, therefore, has less bias and is a more stringent analysis of the data.

American Academy of Pediatrics 136


PREP ® Self-Assessment PREPSA 2021

PREP Pearls
• Intention-to-treat analysis includes data from all patients who were randomly assigned to
a group even if they did not complete the study.
• Intention-to-treat analysis preserves sample size and improves the power of a study.
• Intention-to-treat analysis can help reduce bias.

ABP Content Specifications(s)


• Understand the concept of intention-to-treat analysis to maintain the power of a study

Suggested Readings
• Gupta SK. Intention-to-treat concept: a review. Perspect Clin Res. 2011;2(3):109-112.
doi:10.4103/2229-3485.83221.
• McCoy CE. Understanding the intention-to-treat principle in randomized controlled
trials. West J Emerg Med. 2017;18(6):1075-1078. doi:10.5811/westjem.2017.8.35985.
• Tschudy MM, Rowe PC. Research and statistics: number needed to treat and intention to
treat analysis. Pediatr Rev. 2010 Sep;31(9):380-2. doi:10.1542/pir.31-9-380.

American Academy of Pediatrics 137


PREP ® Self-Assessment PREPSA 2021
Question 38
A 3-day-old female neonate is seen in the emergency department for bleeding from the umbilical
stump. Her mother had an uncomplicated pregnancy and a planned full-term delivery at home.
Her mother reports that the umbilicus began bleeding 12 hours ago and has bled continuously
despite pressure applied with a clean towel. There is no family history of a bleeding diathesis.
The neonate is crying vigorously and appears pale. She has a weight of 3.4 kg, a temperature of
37°C, a heart rate of 178 beats/min, a respiratory rate of 28 breaths/min, a blood pressure of
66/42 mm Hg, and an oxygen saturation on room air of 95%. The lungs are clear bilaterally, and
the heart rhythm is normal but tachycardic. There is no hepatomegaly or splenomegaly. There is
a steady trickle of frank blood emanating from the umbilicus. There is no other evidence of
bleeding. Laboratory data are shown:

Laboratory Test Result


White blood cell count 11,400/µL (11.4 × 109/L)
Hemoglobin 11.2 g/dL (112 g/L)
Platelet count 134 × 103/µL (134 × 109/L)
Neutrophils 53%
Lymphocytes 42%
Monocytes 5%
Prothrombin time 25.2 s
Partial thromboplastin time 62 s

Of the following, the MOST likely condition to be contributing to this patient’s presentation is
A. disseminated intravascular coagulation
B. hereditary factor IX deficiency
C. maternally derived antiplatelet antibodies
D. a vitamin deficiency

American Academy of Pediatrics 138


PREP ® Self-Assessment PREPSA 2021

Correct Answer: D
The neonate in the vignette has had frank bleeding from the umbilical stump that started at 3
days after birth, and it has lasted for at least 12 hours, strongly suggesting a coagulopathy. The
absence of a family history of bleeding disorders decreases the likelihood of a genetic
coagulopathy, and the history of a home birth is a risk factor for having not received vitamin K
supplementation. Neonates who do not receive vitamin K supplementation shortly after birth are
at risk of experiencing vitamin K deficiency bleeding (VKDB), also known as hemorrhagic
disease of the newborn. Vitamin K is essential for the function of factors II, VII, IX, and X,
affecting both the intrinsic and extrinsic coagulation pathway and thereby prolonging both the
prothrombin time and the partial thromboplastin time (Item C38). Neonates are at particular risk
of experiencing VKDB, as very little vitamin K crosses the placenta, the microbiome of neonatal
intestines is not populated by the bacteria that produce vitamin K, and breast milk is a poor
source of vitamin K. Neonatal VKDB can occur at any time from birth to 6 months of age and
can be classified as early (within 24 hours), classical (1-7 days after birth) or late (2 weeks–6
months after birth). Early VKDB is typically severe and can present with hemorrhage into the
intestines or head. Classical neonatal VKDB often presents with bruising and bleeding from the
umbilical stump. Late VKDB can present with any type of bleeding, although intracranial bleeds
are common. The infant in the vignette is exhibiting a classical presentation of neonatal VKDB.

Coagulopathies can be related to platelet abnormalities (abnormal number or function) or to


dysfunction of the coagulation cascade (hereditary or acquired). Disseminated intravascular
coagulation is an acquired coagulopathy that can result in both thrombocytopenia and decreased
levels of coagulation factors, but the neonate in the vignette is not exhibiting the stigmata of
sepsis or disseminated intravascular coagulation. The female sex essentially eliminates
hemophilia A (factor VIII deficiency) and hemophilia B (factor IX deficiency), because both are
X-linked recessive disorders. She has mild thrombocytopenia on her complete blood count, but a
platelet count of more than 100,000/µL is not low enough to permit bleeding from the umbilical
stump. The mild thrombocytopenia is likely due to platelet consumption given the ongoing
bleeding, which makes thrombocytopenia due to a maternally derived platelet antibody unlikely.
Platelet function disorders are very rare, and the absence of a family history of a bleeding
diathesis makes a platelet function disorder very unlikely in this scenario.

American Academy of Pediatrics 139


PREP ® Self-Assessment PREPSA 2021
PREP Pearls
• Neonates who do not receive vitamin K supplementation shortly after birth are at risk of
experiencing vitamin K deficiency bleeding, also known as hemorrhagic disease of the
newborn.
• Vitamin K is essential for the function of factors II, VII, IX, and X, affecting both the
intrinsic and extrinsic coagulation pathways and thereby prolonging both the prothrombin
time and the partial thromboplastin time.
• Neonatal vitamin K deficiency bleeding can occur at any time from birth to 6 months of
age and can be classified as early (within 24 hours), classical (1-7 days after birth), or late
(2 weeks–6 months after birth).

MOCA-Peds Objective
• Evaluate and manage a patient with the new onset of thrombocytopenia.

ABP Content Specifications(s)


• Plan the appropriate evaluation and management of an acquired bleeding or thrombotic
disorder

Suggested Readings
• Callaghan MU, Rajpurkar M. Coagulation disorders. In: McInerny TK, Adam HM,
Campbell DE, DeWitt TG, Foy JM, Kamat DM, eds. American Academy of Pediatrics
Textbook of Pediatric Care. 2nd ed. Elk Grove Village, IL: American Academy of
Pediatrics; 2017:1858-1867. Pediatric Care Online .
• Lauer B, Spector N. Vitamins. Pediatr Rev. 2012;33(8):339-351. doi:10.1542/pir.33-8-
339.
• Pramanik AK. Bleeding disorders in neonates. Pediatr Rev. 1992;13(5):163-173.
doi:10.1542/pir.13-5-163.
• Stokowski LA. Reemergence of vitamin K deficiency bleeding. Adv Neonatal
Care. 2014;14(2):75. doi:10.1097/ANC.0000000000000064.

American Academy of Pediatrics 140


PREP ® Self-Assessment PREPSA 2021
Question 39
A 15-year-old adolescent is being evaluated for acne. She has used an over-the-counter salicylic
acid wash without benefit. She has been in good health and takes no medications. There are
approximately 8 active inflammatory lesions on each cheek and her chin. There are several open
comedones (Item Q39), but no scarring. The forehead, chest, and back are free of acne lesions.

Item Q39: Acne lesions of the patient described in the vignette.


Reprinted with permission from Mancini AJ, Krowchuk DP eds. Pediatric Dermatology: A Quick
Reference Guide. 3rd ed. Elk Grove Village, IL: American Academy of Pediatrics; 2016

Of the following, the MOST appropriate treatment is


A. benzoyl peroxide administered topically
B. benzoyl peroxide/clindamycin administered topically
C. benzoyl peroxide/clindamycin and a retinoid administered topically
D. doxycycline administered orally and benzoyl peroxide administered topically

American Academy of Pediatrics 141


PREP ® Self-Assessment PREPSA 2021
Correct Answer: C
The girl in the vignette has moderate inflammatory and comedonal acne involving the face (Item
C39A). There are several inflammatory papules (red arrow) and open comedones (yellow
arrow). No scarring is evident, but there are several resolving inflammatory lesions (blue arrow).
These lesions may remain erythematous or violaceous for months and are often confused with
scars. The most appropriate treatment, therefore, is benzoyl peroxide (BPO)/clindamycin applied
to the face each morning and a retinoid (eg, tretinoin or adapalene) applied at bedtime. Therapy
with BPO, BPO/clindamycin, or doxycycline orally combined with BPO topically would address
the inflammatory component of her disease, but not follicular obstruction. In addition, because
the girl has moderate acne limited to the face, has no scarring, and is using no medication, an
attempt to manage the inflammatory component of her disease with topical agents is reasonable.

Several factors contribute to the development of acne. Key among these are disordered
keratinization (leading to obstruction within pilosebaceous follicles), increased sebum production
(which contributes to obstruction), and inflammation (caused, in large part, by activation of the
immune system by the bacterium Cutibacterium acnes). As obstruction increases, follicles may
rupture, contributing to the inflammatory process. In some patients, the inflammatory process
results in scarring. On the face, scars appear as small pits, whereas on the trunk they are
hypopigmented macules.

Item C39A: Moderate inflammatory and comedonal acne on the face. There are inflammatory
papules and pustules (red arrow), open comedones (yellow arrow), and resolving inflammatory
lesions (blue arrow).

American Academy of Pediatrics 142


PREP ® Self-Assessment PREPSA 2021
Reprinted with permission from Reprinted with permission from Mancini AJ, Krowchuk DP eds.
Pediatric Dermatology: A Quick Reference Guide. 3rd ed. Elk Grove Village, IL: American
Academy of Pediatrics; 2016

Treatment plans should be designed to affect as many aspects of disease pathophysiology as


possible. Benzoyl peroxide, antibiotics (topical or oral), and dapsone address the inflammatory
process, topical retinoids improve follicular obstruction, and oral contraceptives lessen the
impact of androgens (reducing sebum production and follicular obstruction). It is important to
note that particularly in moderate or severe acne, follicular obstruction is present and should be
addressed therapeutically, even if blackheads (open comedones) and whiteheads (closed
comedones) are not observed.
Guidelines for acne management exist, but treatment should be individualized according to the
patient’s perception of disease severity, past experiences with medications, and the ability to
adhere to therapy. Suggested treatment plans for mild, moderate, and severe acne are presented
in Item C39B, Item C39C, and Item C39D.

American Academy of Pediatrics 143


PREP ® Self-Assessment PREPSA 2021

American Academy of Pediatrics 144


PREP ® Self-Assessment PREPSA 2021

American Academy of Pediatrics 145


PREP ® Self-Assessment PREPSA 2021

PREP Pearls
• A topical retinoid should be included in the treatment of adolescents who have moderate
or severe acne.
• Obstruction within follicles should be addressed, even if blackheads and whiteheads are
not observed.
• Extensive inflammatory acne (ie, involving the trunk, as well as the face) requires
treatment with an oral antibiotic.

ABP Content Specifications(s)


• Recognize the clinical findings associated with acne
• Plan the appropriate management of acne

American Academy of Pediatrics 146


PREP ® Self-Assessment PREPSA 2021
Suggested Readings
• Chen C, Williams JV. Acne. In: McInerny TK, Adam HM, Campbell DE, DeWitt TG,
Foy JM, Kamat DM, eds. American Academy of Pediatrics Textbook of Pediatric Care.
2nd ed. Elk Grove Village, IL: American Academy of Pediatrics; 2017:1681-
1687. Pediatric Care Online .
• Eichenfield LF, Krakowski AC, Piggott C, et al; American Acne and Rosacea Society.
Evidence-based recommendations for the diagnosis and treatment of pediatric
acne. Pediatrics. 2013;131(suppl 3):S163-S186. doi:10.1542/peds.2013-0490B.
• Mancini AJ, Krowchuk DP, eds. Pediatric Dermatology: A Quick Reference Guide. 3rd
ed. Elk Grove Village, IL: American Academy of Pediatrics; 2016.
• Thiboutot DM, Dréno B, Abanmi A, et al. Practical management of acne for clinicians:
an international consensus from the Global Alliance to Improve Outcomes in Acne. J Am
Acad Dermatol. 2018;78(2 suppl 1):S1-S23.e1. doi:10.1016/j.jaad.2017.09.078.

American Academy of Pediatrics 147


PREP ® Self-Assessment PREPSA 2021
Question 40
A 15-year-old adolescent girl is seen for evaluation of hair loss. She first noted hair loss several
months ago. At that time, the area was slightly itchy, tender, and pink. She has no significant past
medical or family history and has no other signs or symptoms. She is afebrile with age-
appropriate vital signs. Her scalp is shown (Item Q40). The remainder of her physical
examination findings are normal, including full range of motion of all of her joints with no
evidence of effusion and no other skin lesions.

American Academy of Pediatrics 148


PREP ® Self-Assessment PREPSA 2021

American Academy of Pediatrics 149


PREP ® Self-Assessment PREPSA 2021
Item Q40: Scalp lesion.
Reprinted with permission from Mancini AJ, Krowchuk DP, eds. Pediatric Dermatology: A
Quick Reference Guide. 3rd ed. Elk Grove Village, IL: American Academy of Pediatrics; 2016.

Of the following, the MOST likely diagnosis is


A. alopecia areata
B. linear scleroderma
C. tinea capitis
D. trichotillomania

American Academy of Pediatrics 150


PREP ® Self-Assessment PREPSA 2021
Correct Answer: B
The girl in the vignette has linear localized scleroderma (LS), a subtype of LS. When linear LS
involves the scalp or face it is also called “en coup de sabre.” Localized scleroderma (also called
morphea) involves cutaneous tissue and sometimes underlying muscle and bone, and needs to be
differentiated from systemic sclerosis, a multisystem disorder that includes nervous system and
visceral organ abnormalities. During the active phase, LS lesions have a red border and a white
“waxy” center. As the lesions become inactive, they can be either hyper- or hypopigmented.
Active linear LS is typically treated with systemic corticosteroids and/or methotrexate to reduce
the chances of permanent disfigurement, though sometimes small lesions that do not involve
joints or the face can be treated with localized phototherapy, topical corticosteroids, or topical
immunomodulators.

Alopecia areata is an autoimmune disease. Physical examination findings include smooth areas
of nonscarring hair loss, often with short vellus hairs, broken hairs, and tapering or “exclamation
point” hairs (where the proximal end of the hair is narrower than the distal end). Tinea capitis is
a fungal infection of the scalp that presents with patches of hair loss with overlying scale and
sometimes with black dots indicating broken hairs. Trichotillomania, a disorder in which the
patient compulsively pulls out their hair, is characterized by irregular patches of hair loss with
broken hairs and signs of scalp trauma.

PREP Pearls
• Localized scleroderma (also called morphea) involves cutaneous tissue and sometimes
underlying muscle and bone, whereas systemic sclerosis is a multisystem disorder that
includes nervous system and visceral organ abnormalities.
• Active localized scleroderma lesions have a red border and a white “waxy” center. As the
lesions become inactive, they can be either hyper- or hypopigmented.

MOCA-Peds Objective
• Evaluate a child with hyperpigmentation.

ABP Content Specifications(s)


• Recognize the clinical findings associated with localized scleroderma
• Understand the prognosis of patients with scleroderma as opposed to those with systemic
sclerosis

Suggested Readings
• Fett N, Werth VP. Update on morphea: part I—epidemiology, clinical presentation, and
pathogenesis. J Am Acad Dermatol. 2011;64(2):217-228.
doi: 10.1016/j.jaad.2010.05.045.
• Fett N, Werth VP. Update on morphea: part II—outcome measures and treatment. J Am
Acad Dermatol. 2011;64(2):231-242. doi: 10.1016/j.jaad.2010.05.046.
• Mancini AJ, Krowchuk DP, eds. Pediatric Dermatology: A Quick Reference Guide. 3rd
ed. Elk Grove Village, IL: American Academy of Pediatrics; 2016:656.
• Zulian F, Culpo R, Sperotto F, et al. Consensus-based recommendations for the
management of juvenile localised scleroderma. Ann Rheum Dis. 2019;78(8):1019-1024.
doi: 10.1136/annrheumdis-2018-214697.
American Academy of Pediatrics 151
PREP ® Self-Assessment PREPSA 2021
Question 41
An 11-year-old girl is discharged from the hospital with a prescription for levetiracetam after
hospitalization for newly diagnosed generalized epilepsy. She returns to the emergency
department 2 weeks later with suicidal ideation and agitation and is admitted to the child
psychiatry unit. Upon medication reconciliation, it is discovered that she is taking 4 times the
intended dose of levetiracetam. Investigation shows that her discharge prescription was written
incorrectly by a resident physician. Her neurologist is contacted and the correct dose is
confirmed. The correct dose is administered in the hospital and her psychiatric symptoms
resolve.

Of the following, the BEST person to disclose the error to the family would be the
A. child psychiatrist
B. hospital risk management supervisor
C. prescribing resident physician
D. supervising attending neurologist

American Academy of Pediatrics 152


PREP ® Self-Assessment PREPSA 2021
Correct Answer: D
The supervising attending neurologist, ideally along with the prescribing resident physician,
would be the most appropriate person to disclose the medication error. The Institute of Medicine
(IOM) and American Academy of Pediatrics (AAP) both affirm the practice of timely and
transparent disclosure of medical errors as the standard of care and ethical obligation.

Medical errors are a common occurrence in health care and can lead to morbidity and mortality.
Research has shown that the majority of parents want to know when an error has occurred in the
care of their child, regardless of whether it resulted in harm. Pediatric physicians and resident
trainees agree that they have an obligation to disclose errors when they occur, with resident
physicians more likely to have received training in error disclosure.

Disclosure, or communication of a medical error from the physician to the patient, is grounded in
the ethical principle of autonomy. Autonomy refers to a patient’s ability to make informed health
care decisions without coercion. By disclosing an error, the physician is providing the patient and
family with the facts needed to understand what has occurred. It is grounded in the profession’s
responsibility to be truthful and have integrity—the foundation of a patient-physician
relationship.

When disclosing an error, the physician responsible for the care of the patient, in this case the
attending neurologist, is the most appropriate person to lead the discussion. In this situation, the
prescribing resident physician should have the opportunity to be part of the disclosure with the
support of the attending. Approaching the disclosure as a learning opportunity through
discussion, modeling, and debriefing in a supportive way has been shown to be beneficial.
Standardized approaches, such as the I-HELP model, provide a framework for this educational
opportunity (Item C41). The supervising psychiatrist may be present for the discussion, but was
not the responsible physician at the time the error occurred. Hospital risk management may be
involved in reporting systems and should be available for professional advice regarding
disclosure.

American Academy of Pediatrics 153


PREP ® Self-Assessment PREPSA 2021

American Academy of Pediatrics 154


PREP ® Self-Assessment PREPSA 2021
PREP Pearls
• Timely and transparent disclosure of medical errors is the standard of care and an ethical
obligation of the physician.
• Disclosure should be led by the physician responsible for the patient’s care at the time of
the medical error.
• Medical error disclosure supports the principle of autonomy by providing patients with
the information needed to understand their care.

ABP Content Specifications(s)


• Recognize and apply ethical principles regarding malpractice
• Recognize and apply ethical principles regarding medical errors

Suggested Readings
• AAP Committee on Medical Liability and Risk Management and AAP Council on
Quality Improvement and Patient Safety. Disclosure of adverse events in
pediatrics. Pediatrics. 2016;138(6):e2016321. doi: 10.1542/peds.2016-3215.
• Beck J, McGrath C, Toncray K, Rooholamini S. Failure Is an option: using errors as
teaching opportunities. Pediatrics. 2018;141. doi: 10.1542/peds.2017-4222.
• Garbutt J, Brownstein DR, Klein EJ, et al. Reporting and disclosing medical errors:
pediatricians' attitudes and behaviors. Arch Pediatr Adolesc Med. 2007;161(2):179-185.
doi: 10.1001/archpedi.161.2.179.
• Lin M, Famiglietti. Closing the disclosure gap: medical errors in pediatrics. Pediatrics.
2019;143(4):e20190221. doi: 10.1542/peds.2019-0221.
• Neuspiel DR. Medical errors, adverse events, and patient safety. In: McInerny TK, Adam
HM, Campbell DE, DeWitt TG, Foy JM, Kamat DM, eds. American Academy of
Pediatrics Textbook of Pediatric Care. 2nd ed. Itasca, IL: American Academy of
Pediatrics; 2016;chap 288:2287-2294. Pediatric Care Online.

American Academy of Pediatrics 155


PREP ® Self-Assessment PREPSA 2021
Question 42
A male neonate is delivered at 37 weeks’ gestation because of maternal preeclampsia. His
mother’s history is significant for obesity and chronic hypertension. He weighs 2.2 kg at birth. At
18 hours after birth, shortly after breastfeeding and supplementing with formula, his bedside
glucose level is 35 mg/dL (1.9 mmol/L). He is breathing comfortably in room air and has good
perfusion.

Of the following, the MOST likely explanation for this neonate’s glucose level is
A. congenital hyperinsulinemia
B. decreased glycogen stores
C. maternal hyperglycemia
D. physiologic hypoglycemia

American Academy of Pediatrics 156


PREP ® Self-Assessment PREPSA 2021
Correct Answer: B
For the growth-restricted neonate in the vignette, limited glycogenolysis because of decreased
glycogen stores is the most likely cause of hypoglycemia. In utero, a fetus relies on maternal
glucose supply. After delivery, the maternal glucose is no longer available, resulting in transient
neonatal hypoglycemia. In response, there is a surge in glucagon, epinephrine, and cortisol,
which promotes glycogenolysis, gluconeogenesis, and lipolysis and ultimately restores serum
glucose values. Increased glucagon stimulates the production of phosphoenolpyruvate
carboxykinase, the rate-limiting enzyme for gluconeogenesis. At the same time, muscle tissue
can use free fatty acids and ketone bodies, leaving serum glucose available for brain utilization.
The causes of neonatal hypoglycemia are shown in Item C42A.

American Academy of Pediatrics 157


PREP ® Self-Assessment PREPSA 2021

American Academy of Pediatrics 158


PREP ® Self-Assessment PREPSA 2021
The brain relies on glucose as a primary energy source. Periods of prolonged neonatal
hypoglycemia have been associated with seizures, cerebral palsy, and cognitive impairment. In
particular, low serum glucose because of elevated insulin levels presents multiple challenges.
Hyperinsulinemia decreases glycogenolysis, fat metabolism, and the formation of ketone bodies,
limiting the availability of alternate energy sources for the brain. Congenital hyperinsulinemia
because of mutations in the glucose transporter genes can cause prolonged hypoglycemia, which
is associated with adverse neurodevelopmental outcomes.

Neonates at risk for hypoglycemia because of growth restriction, prematurity, family history of
hypoglycemia, and inherited diseases of metabolism, should be monitored closely. The definition
of neonatal hypoglycemia is currently being debated (Item C42B). The American Academy of
Pediatrics recommends maintaining serum glucose levels at more than 45 mg/dL (2.5 mmol/L)
by 4 hours after birth. The Pediatric Endocrine Society recommends serum glucose levels greater
than 50 mg/dL (2.7 mmol/L) for the first 48 hours after birth and greater than 60 mg/dL (3.3
mmol/L) by 48 hours after birth.

Item C42B: Pediatric Endocrine Society (PES) and American Academy of Pediatrics (AAP)
neonatal hypoglycemia guidelines in the first 48 hours after birth and beyond.
Reprinted with permission from Thompson-Branch A, Havranek T. Neonatal hypoglycemia.
Pediatr Rev. 2017;38(4):150

Physiologic hypoglycemia occurs in the first four hours of life and would not explain this
neonate’s hypoglycemia at 18 hours of life. His mother does not have diabetes making maternal
hyperglycemia unlikely. While congenital hyperinsulinemia is a possible explanation, this
neonate’s hypoglycemia is more likely the result of inadequate glycogen stores.

PREP Pearls
• Periods of prolonged neonatal hypoglycemia have been associated with seizures, cerebral
palsy, and cognitive impairment.
• Neonates with hypoglycemia because of elevated insulin levels have less glycogenolysis,
fat metabolism, and formation of ketone bodies, decreasing the available alternate
sources of energy.
• Neonates at risk for hypoglycemia because of growth restriction, prematurity, family
history of hypoglycemia, or inherited diseases of metabolism should be monitored
closely.

American Academy of Pediatrics 159


PREP ® Self-Assessment PREPSA 2021
ABP Content Specifications(s)
• Recognize the clinical features associated with hyperinsulinism
• Plan the evaluation of a patient with suspected metabolic disease who has hypoglycemia,
and manage appropriately

Suggested Readings
• Adamkin DH; Committee on Fetus and Newborn. Postnatal glucose homeostasis in late-
preterm and term infants. Pediatrics. 2011;127:575-579. doi: 10.1542/peds.2010-3851.
• Sperling MA. Hypoglycemia. In: Kliegman R, St Geme JW, Blum NJ, Shah SS, Tasker
RC, Wilson Karen, eds. Nelson Textbook of Pediatrics. Philadelphia, PA: Elsevier;
2020:848-862.e1.
• Thompson-Branch A, Havranek T. Neonatal hypoglycemia. Pediatr Rev.
2017;38(4):147-155. doi: 10.1542/pir.2016-0063.
• Thornton PS, Stanley CA, De Leon DD, et al. Recommendations from the Pediatric
Endocrine Society for Evaluation and Management of Persistent Hypoglycemia in
Neonates, Infants, and Children. J Pediatr. 2015;167(2):238-245.
doi:10.1016/j.jpeds.2015.03.057.

American Academy of Pediatrics 160


PREP ® Self-Assessment PREPSA 2021
Question 43
A 2-month-old female infant with poor feeding and failure to gain weight is seen for evaluation.
She is fed 24 kcal/oz of appropriately prepared infant formula. She was born at 39 weeks’
gestation, and her birth weight was 3 kg. She failed her newborn hearing screening. She had one
hospital admission with vomiting and diarrhea at 6 weeks after birth. Her weight is currently at
the third percentile, and her height is at the fifth percentile. Her cardiac examination reveals no
murmur, and her chest is clear to auscultation. The rest of her physical examination findings are
unremarkable.

Laboratory data are shown:


Laboratory Test Result
Blood
Sodium 134 mEq/L (134 mmol/L)
Potassium 3.2 mEq/L (3.2 mmol/L)
Chloride 110 mEq/L (110 mmol/L)
Bicarbonate 14 mEq/L (14 mmol/L)
Blood urea nitrogen 10 mg/dL (3.6 mmol/L)
Creatinine 0.2 mg/dL (18 µmol/L)
Glucose 90 mg/dL (5.0 mmol/L)
Urine
pH 7.0
Specific gravity 1.015
Urinalysis with microscopy Normal
Sodium 60 mEq/L (60 mmol/L)
Potassium 10 mEq/L (10 mmol/L)
Chloride 55 mEq/L (55 mmol/L)

Of the following, the MOST likely diagnosis for this child is


A. congenital adrenal hyperplasia
B. distal renal tubular acidosis
C. methylmalonic acidemia
D. postinfectious diarrhea

American Academy of Pediatrics 161


PREP ® Self-Assessment PREPSA 2021
Correct Answer: B
The infant in the vignette has failure to thrive and normal anion gap metabolic acidosis. The
higher-than-expected urinary pH and positive urine anion gap favors a diagnosis of distal renal
tubular acidosis.

Metabolic acidosis is defined as low serum bicarbonate (HCO3−) concentration. Metabolic


acidosis results from either a net gain in hydrogen (H+) ion or a net loss of HCO3−. A systematic
approach is required to diagnose and treat metabolic acidosis. The first step in evaluation of
metabolic acidosis is to calculate the serum anion gap (the difference between the major cation
and anions).

Anion gap (mEq/L) = sodium (Na+) − [chloride (Cl−) + bicarbonate (HCO3−)]


A normal anion gap ranges from 8 mEq/L to 12 mEq/L. Metabolic acidosis can be classified as
elevated anion gap acidosis or normal anion gap acidosis (hyperchloremic metabolic acidosis)
(Item C43A ).

Normal anion gap acidosis is most commonly due to diarrhea or renal tubular acidosis (RTA). In
diarrhea, gastrointestinal loss of HCO3− occurs, whereas in RTA there is impaired renal excretion
American Academy of Pediatrics 162
PREP ® Self-Assessment PREPSA 2021
of hydrogen ion (type 1 distal) or impaired reabsorption of HCO3− (type 2 proximal). To
differentiate between diarrhea and RTA, urine electrolytes should be obtained and the urine
anion gap (UAG) calculated.

Urine anion gap = {urine sodium (Na+) + urine potassium (K+)} - urine chloride (Cl−)
The UAG can be positive (urine [Na+] + urine [K+] is > urine [Cl−]) or negative (urine [Na+] +
urine [K+] is < urine [Cl−]). Unlike the serum anion gap, the magnitude of the gap is not
clinically relevant. The UAG serves as a surrogate marker of NH4+ production. Diarrhea leads to
a negative UAG, as there is increased excretion of H+ ion in the form of ammonium chloride
(NH4+Cl−) in the urine to counteract the loss of HCO3− in the gastrointestinal tract. Therefore, in
diarrhea, urinary Cl− is higher than the sum of urinary Na+ and K+.

Renal tubular acidosis can be further classified into type 1 (distal; failure to excrete H+ ion), type
2 (proximal; failure to reabsorb HCO3−), and type 4 (hyperkalemic, disruption of renin-
angiotensin-aldosterone axis). Children with distal RTA have a distal tubular acidification defect,
resulting in low NH4+ excretion in urine. Therefore, urinary Cl− is lower than the sum of urinary
Na+ and K+, and the UAG is positive. Children with proximal RTA have a reduced capacity to
reabsorb HCO3− in the proximal tubule. However, the distal acidification mechanism is intact;
therefore, urinary acidification is appropriate (pH < 5.5) and the UAG is negative. On
supplementation with HCO3−, the threshold for filtered HCO3− is exceeded, causing an alkaline
urine (pH > 5.5). Further clinical and laboratory differences in various types of RTA are shown
in Item C43B. Urinary alkalizing agents (sodium bicarbonate or potassium citrate) are used as
first-line treatment for all types of RTA. Serum electrolytes and growth are closely monitored
while the patient is receiving treatment.

American Academy of Pediatrics 163


PREP ® Self-Assessment PREPSA 2021

American Academy of Pediatrics 164


PREP ® Self-Assessment PREPSA 2021

Congenital adrenal hyperplasia presents with failure to thrive, atypical genitalia, metabolic
acidosis or alkalosis, hyponatremia, and hyperkalemia. The child in this vignette has
hypokalemia and, thus, congenital adrenal hyperplasia is an unlikely diagnosis. Methylmalonic
acidemia is an inborn error of metabolism associated with an elevated anion gap acidosis. The
infant in this vignette has a normal anion gap acidosis, making methylmalonic acidemia unlikely.
In diarrheal illness, the renal acidification mechanism is intact, urinary ammonium excretion is
high, and the urine anion gap is negative, making a diarrheal illness an unlikely diagnosis for this
child.

PREP Pearls
• Diarrhea and renal tubular acidosis are the most common causes of normal anion gap
acidosis in children.
• The urine anion gap helps differentiate diarrhea from renal tubular acidosis.
• In distal renal tubular acidosis, the urine anion gap is positive and ammonium (NH4+)
excretion is low.

ABP Content Specifications(s)


• Understand the clinical and laboratory findings associated with renal tubular acidosis
• Formulate a differential diagnosis of renal tubular acidosis

Suggested Readings
• Carmody JB. Urine electrolytes. Pediatr Rev. 2011;32(2):65-68. doi:10.1542/pir.32-2-65.
• Hsu BS, Lakhani SA, Wilhelm M. Acid-base disorders. Pediatr Rev. 2016;37(9):361-
369. doi:10.1542/pir.2015-0093.
• Kallen R. Renal tubular acidosis. In: McInerny TK, Adam HM, Campbell DE, DeWitt
TG, Foy JM, Kamat DM, eds. American Academy of Pediatrics Textbook of Pediatric
Care. 2nd ed. Elk Grove Village, IL: American Academy of Pediatrics; 2017:2554-
2571. Pediatric Care Online .
• Pelletier J, Gbadegesin R, Staples B. Renal tubular acidosis. Pediatr
Rev. 2017;38(11):537-539. doi:10.1542/pir.2016-0231.

American Academy of Pediatrics 165


PREP ® Self-Assessment PREPSA 2021
Question 44
A previously healthy, 5-year-old boy is seen in the emergency department 1 hour after a
witnessed 2.5-foot fall from playground equipment. The child hit the right side of his head but
did not lose consciousness and has not vomited. He is afebrile. He has a heart rate of 108
beats/min, blood pressure of 108/64 mm Hg, and respiratory rate of 23 breaths/min. He is awake,
alert, interactive, and responds appropriately to questions. He has a 2 × 2, cm mildly tender right
parietal hematoma; there is no palpable fracture. He has no other signs of injury.

Of the following, the BEST next management step for this boy is
A. computed tomography of the head
B. magnetic resonance imaging of the brain
C. neurosurgical consultation
D. supportive care

American Academy of Pediatrics 166


PREP ® Self-Assessment PREPSA 2021
Correct Answer: D
The primary clinical decision that must be made during the evaluation of the child who has
sustained head trauma is whether to perform neuroimaging. For the boy in this vignette, the most
appropriate next management step is supportive care. Several head trauma clinical decision aids
are available to identify children who have sustained blunt head trauma and are at low risk for
significant sequelae. The most well-validated is the Pediatric Emergency Care Applied Research
Network (PECARN) pediatric head trauma prediction rule. That rule states that children older
than 2 years who sustained blunt head trauma are considered to be at low risk and do not warrant
neuroimaging if they have:
• Glasgow Coma Scale (GCS) score of 15 (Item C44A)
• No signs of basilar skull fracture: hemotympanum, periorbital ecchymosis (“raccoon
eyes”), perimastoid ecchymosis (“battle sign”), and cerebrospinal fluid otorrhea
• No loss of consciousness
• No vomiting
• The absence of a severe headache
• A nonsevere mechanism of injury (in this case, a fall of <5 feet)

American Academy of Pediatrics 167


PREP ® Self-Assessment PREPSA 2021

Other pediatric head trauma prediction rules include the Canadian Assessment of Tomography
for Childhood Head Injury (CATCH) and Children’s Head Injury Algorithm for the Prediction of
Important Clinical Events (CHALICE) rules. A comparison of the low-risk criteria for these
rules can be found in Item C44B. If neuroimaging is indicated, computed tomography (CT) is
the modality of choice. Magnetic resonance imaging is not the preferred initial imaging modality
for evaluation of the child with acute head trauma because CT is more rapidly obtained and
allows for better resolution of the skull. Neurosurgical consultation is inappropriate in the setting
of low-risk acute head trauma where neuroimaging is not warranted.

American Academy of Pediatrics 168


PREP ® Self-Assessment PREPSA 2021

American Academy of Pediatrics 169


PREP ® Self-Assessment PREPSA 2021

Traumatic brain injury remains a leading cause of death and disability in children. Blunt head
trauma can cause vascular injuries, leading to intracranial bleeding in the epidural, subdural, or
subarachnoid spaces; direct injury at the site of trauma (coup injury) and/or 180 degrees from the
site of trauma (contracoup injury); and direct axonal injury. The immediate life-threatening
complications of head trauma result from intracranial bleeding or cerebral edema, causing
increased intracranial pressure and resultant herniation of brain tissue. Bradycardia,
hypertension, and irregular respirations, known as the Cushing triad, suggest impending brain
herniation. Unilateral mydriasis resulting from compression of the third cranial nerve (“blown
pupil”) is indicative of uncal herniation under the tentorium cerebelli. Item C44C depicts the
different intracranial hemorrhages and herniations.

Item C44C: Intracranial hemorrhages and herniations.

Reprinted with permission Gelineau-Morel RN, Zinkus TP, Le Pichon JB Pediatric Head
Trauma: A Review and Update. Pediatr Rev. 2019 Sep;40(9):475.

For the child who has sustained head trauma, evaluation of the airway, breathing, and circulation
is the first step. Formal assessment of the patient’s neurologic status (“disability”) should be
performed, including the GCS score, evaluation of pupil size, reactivity and symmetry, and a
blood glucose level. A GCS score of 8 or less is an indication for endotracheal intubation.
Cervical spine stabilization should be maintained with a hard cervical collar until the absence of
cervical spine injury can be confirmed. If increased intracranial pressure is suspected, the head of
American Academy of Pediatrics 170
PREP ® Self-Assessment PREPSA 2021
the bed should be raised to 30 degrees and the patient’s head should be kept in the midline.
Maintenance of normoglycemia, normothermia, and euvolemia is crucial. Management of
increased intracranial pressure may include medical interventions such as mannitol and
hypertonic saline. Hyperventilation can decrease the partial pressure of carbon dioxide, which
may transiently improve symptoms of increased intracranial pressure. Large intracranial
hemorrhages may necessitate operative evacuation.

Of note, it is important to always consider nonaccidental trauma when evaluating the child who
has blunt head trauma. Care should be taken to obtain a detailed history of the mechanism of
injury and to ascertain if the injuries noted on physical examination are consistent with the story
provided.

PREP Pearls
• Several head trauma prediction rules can be used to help identify children at low risk for
significant intracranial injuries for whom neuroimaging is not indicated after blunt head
trauma.
• Computed tomography of the head is the initial imaging modality of choice for
evaluation after blunt head trauma, when neuroimaging is clinically indicated.
• The triad of bradycardia, hypertension, and altered respirations (Cushing triad) is
suggestive of impending brain herniation.

ABP Content Specifications(s)


• Plan the appropriate diagnostic evaluation of closed-head injury and brief loss of
consciousness
• Recognize the signs and symptoms associated with closed-head trauma (eg,
hemotympanum, ecchymoses, postauricular bruise, etc)
• Recognize the significance of a linear skull fracture in infants
• Recognize the immediate life-threatening complications of closed-head trauma
• Plan the appropriate initial evaluation and management of acute central nervous system
trauma

Suggested Readings
• Babl FE, Borland ML, Phillips N, et al. Accuracy of PECARN, CATCH, and CHALICE
head injury decision rules in children: a prospective cohort study. Lancet.
2017;389(10087):2393-2402. doi: 10.1016/S0140-6736(17)30555-X.
• Fein DM, Fagan MJ. Overall approach to trauma in the emergency department. Pediatr
Rev. 2018;39(10):479-489. doi: 10.1542/pir.2017-0246.
• Gelineau-Morel RN, Zinkus TP, Le Pichon JB. Pediatric head trauma: a review and
update. Pediatr Rev. 2019;40(9):468-481. doi: 10.1542/pir.2018-0257.
• Kuppermann N, Holmes JF, Dayan PS, et al. Identification of children at very low risk of
clinically-important brain injuries after head trauma: a prospective cohort study. Lancet.
2009;374(9696):1160-1170. doi: 10.1016/S0140-6736(09)61558-0.
• Mahajan PV, Indra S. Head injuries. In: McInerny TK, Adam HM, Campbell DE, DeWitt
TG, Foy JM, Kamat DM, eds. American Academy of Pediatrics Textbook of Pediatric
Care. 2nd ed. Itasca, IL: American Academy of Pediatrics; 2016; chap 360:2862-
2867. Pediatric Care Online.
American Academy of Pediatrics 171
PREP ® Self-Assessment PREPSA 2021
Question 45
A 3-year-old boy with trisomy 21 and autism is seen for a health supervision visit. His mother
reports that their neighbor, who is a naturopath, suggested administering high doses of vitamin
B9 (folic acid) supplements 3 times a day. The mother asks about the benefits and risks of this
high intake.

Of the following, the BEST response is that this treatment may


A. improve his IQ
B. increase his risk of malignancy
C. mask vitamin B12 deficiency
D. prevent worsening of autistic behaviors

American Academy of Pediatrics 172


PREP ® Self-Assessment PREPSA 2021
Correct Answer: C
The mother in the vignette may be advised that high doses of folic acid may mask vitamin
B12 deficiency. Evidence thus far is not adequate to show that folate consistently improves
cognition (IQ) or stabilization of autistic behaviors. Folic acid has likewise not been reliably
shown to increase the risk for malignancy.

Questions about vitamin intake are common for healthy children who reportedly refuse to eat
vegetables and for children who have conditions that legitimately restrict their diet. Some
families may use vitamins without their providers’ knowledge and reveal their use only upon
specific questioning. Children and families may be unaware of the vitamin content of the
supplements they use, such as the megadoses of vitamin B12 in widely available energy drinks.
Many vitamins interact with medications used to treat disorders ranging from common viral
illnesses to rare forms of cancer. Item C45 lists vitamins with their drug interactions and adverse
reactions resulting from high doses. Normal usage of most over-the-counter vitamins usually
does not approach toxicity, but some families use higher-than-recommended doses of nutritional
supplements for unconventional indications.

American Academy of Pediatrics 173


PREP ® Self-Assessment PREPSA 2021

Providers should always ask about such usage to prevent adverse reactions. Nutritional
supplements including vitamins are not subject to the same regulations by the Food and Drug
Administration as over-the-counter or prescription medications. They are therefore more subject
to variability in content, purity, and labeling of their ingredients.

American Academy of Pediatrics 174


PREP ® Self-Assessment PREPSA 2021
Guidance on appropriate vitamin usage is especially needed for families who feel helpless and
are seeking a way to gain control of their children’s health, such as those with autism and other
conditions for which there is currently no evidence-based cure. Some children with autism have
a deficit in the metabolism of vitamins B6, B9, and B12, leading families to supplement with these
vitamins, but the evidence for clinical effectiveness is poor.

Conditions besides deficiency, in which higher doses of vitamins or vitamin derivatives may be
used as part of evidence-based treatment, include acne (isotretinoin from vitamin A), measles
(vitamin A), fat malabsorption (fat-soluble vitamins), and pyridoxine-dependent seizure disorder
(vitamin B6). Counseling is essential when vitamins are used as treatment, to prevent toxicity.

PREP Pearls
• High doses of vitamin B9 (folic acid) may mask vitamin B12 deficiency.
• Providers should proactively ask children and families about their usage of vitamins and
other supplements.
• Providers should counsel children and families on the risks of hypervitaminosis.

ABP Content Specifications(s)


• Recognize the signs and symptoms of hypervitaminosis

Suggested Readings
• American Academy of Pediatrics Committee on Nutrition. Vitamins. In: Kleinman RE,
Greer FR, eds. Pediatric Nutrition. 8th ed. Itasca, IL: American Academy of Pediatrics;
2019:625-677.
• Diab L, Krebs NF. Vitamin excess and deficiency. Pediatr Rev. 2018;39(4):161-179.
doi: 10.1542/pir.2016-0068.

American Academy of Pediatrics 175


PREP ® Self-Assessment PREPSA 2021
Question 46
A 13-year-old adolescent boy is seen for a preparticipation physical examination for summer
camp. He and his mother have no concerns. He has no known medical problems and takes no
medication. His father has hypertension. His paternal grandfather has type 2 diabetes and had a
myocardial infarction at age 68 years. The patient has a blood pressure of 128/86 mm Hg and a
heart rate of 102 beats/min. He has a weight of 80 kg (> 95th percentile), height of 162 cm (75th
percentile), and body mass index of 30.5 kg/m2 (121% of the 95th percentile). He has acanthosis
nigricans over the nape of his neck. His sexual maturity rating is 3 for pubic hair and genital
development. The remainder of his physical examination findings are unremarkable.

Data from a fasting laboratory evaluation are shown:


Laboratory Test Result
Glucose 96 mg/dL (5.3 mmol/L)
Hemoglobin A1c 5.7%
Aspartate aminotransferase 42 U/L (reference range, 10-40 U/L)
Alanine aminotransferase 65 U/L (reference range, 7-55 U/L)
Total cholesterol 214 mg/dL (5.54 mmol/L)
Triglycerides 130 mg/dL (1.5 mmol/L)
High-density lipoprotein 35 mg/dL (0.91 mmol/L)
Low-density lipoprotein 120 mg/dL (3.11 mmol/L)
Thyroid-stimulating hormone 6.0 mIU/L (reference range, 0.3-4.0 mIU/L)

Of the following, the BEST next treatment step for this adolescent is initiation of
A. atorvastatin
B. levothyroxine
C. lifestyle changes for weight loss
D. metformin

American Academy of Pediatrics 176


PREP ® Self-Assessment PREPSA 2021
Correct Answer: C
The boy described in the vignette has obesity and metabolic syndrome. His borderline elevated
hemoglobin A1c level, mildly elevated transaminase levels, dyslipidemia, and mildly elevated
thyroid stimulating hormone (TSH) level are all most likely secondary to his obesity. The best
next management step is initiation of lifestyle changes for weight loss. Weight loss to achieve
even small reductions in body mass index (BMI) percentile can have beneficial metabolic effects
and is associated with improvement in metabolic parameters.

Metabolic syndrome is a constellation of risk factors for cardiovascular disease and type 2
diabetes. Components include obesity, hypertension, dyslipidemia, and glucose intolerance.
Obesity and insulin resistance play key roles in the pathogenesis of metabolic syndrome.
Adults meet criteria for metabolic syndrome when three of the following are present:
• High waist circumference
• High triglyceride level (≥ 150 mg/dL [1.7 mmol/L])
• Low high-density lipoprotein (HDL) cholesterol (< 40 mg/dL [1 mmol/L] in males, < 50
mg/dL [1.3 mmol/L] in females)
• High blood pressure (≥ 130 mm Hg systolic and/or ≥ 85 mm Hg diastolic)
• High fasting glucose level (≥ 100 mg/dL [5.6 mmol/L])

Although there is no consensus definition for metabolic syndrome in children and adolescents,
the adult criteria are often modified to better reflect age-specific normative data and
measurements.

The boy in the vignette has obesity, borderline high triglyceride level, low high-density
lipoprotein cholesterol, and elevated blood pressure. He has evidence of insulin resistance, with
his acanthosis nigricans and his hemoglobin A1c level is in the prediabetes range of 5.7% to
6.4%. His elevated heart rate may be a marker of poor fitness level and may also be a marker of
insulin resistance. His elevated liver transaminases suggest nonalcoholic fatty liver disease, a
complication of obesity. His family history of hypertension, type 2 diabetes, and cardiovascular
disease also confers risk.

Obesity in children and adolescents is defined as a BMI in the 95th percentile or higher for age
and sex. Overweight is defined as BMI in the 85th percentile or higher. For those with
significantly high BMI levels, BMI can be expressed as a percentage of the 95th percentile to
better clarify the degree of obesity and to better track changes in BMI percentile over time. A
BMI of 120% of the 95th percentile or higher is classified as extreme obesity. Online calculators
are available to determine the percentage of the 95th percentile for a given BMI.

Guidelines recommend screening children and adolescents with a BMI in the 85th percentile or
higher for comorbidities. This screening includes glycemic indicators (fasting glucose,
hemoglobin A1c, or oral glucose tolerance test), fasting lipids, and liver transaminases, as well as
screening for polycystic ovary syndrome (in females), obstructive sleep apnea, and psychiatric
comorbidities. The American Academy of Pediatrics Section on Endocrinology made a Choosing
Wisely recommendation against routinely obtaining thyroid function tests and fasting insulin
levels for children with obesity. Thyroid-stimulating hormone levels are often mildly elevated in

American Academy of Pediatrics 177


PREP ® Self-Assessment PREPSA 2021
the setting of obesity and not related to a thyroid disorder. Fasting insulin levels have significant
limitations in the evaluation of insulin resistance.

Healthy lifestyle changes that promote weight loss are the primary management for obesity and
metabolic syndrome. Motivational interviewing can be used to help set SMART (specific,
measurable, attainable, relevant, timely) goals. The 9-5-2-1-0 principle provides a framework for
counseling and goal setting:
• 9 hours of sleep per night
• 5 servings of fruits and vegetables per day
• Less than 2 hours of screen time per day (television, computer, electronic devices)
• 1 hour of physical activity per day
• 0 sugary drinks (soda, juice)

For the boy in the vignette, atorvastatin is not indicated. Guidelines recommend considering a
statin in those with risk factors who have a low-density lipoprotein level persistently 160 mg/dL
or higher despite dietary management. Levothyroxine is not indicated without other indications
of thyroid disease. The boy’s mildly elevated TSH level is most likely secondary to his obesity.
There is no evidence that treatment of a mildly elevated TSH level in the context of obesity is
beneficial. Metformin is approved for type 2 diabetes in those aged 10 years or older. This boy
does not meet criteria for type 2 diabetes.

PREP Pearls
• Components of metabolic syndrome, a constellation of risk factors for cardiovascular
disease and type 2 diabetes, include obesity, hypertension, dyslipidemia, and glucose
intolerance.
• Recommended screening in children and adolescents with a BMI at the 85th percentile or
higher includes glycemic indicators (fasting glucose, hemoglobin A1c, or oral glucose
tolerance test), fasting lipids, and liver transaminases, as well as polycystic ovary
syndrome (in females), obstructive sleep apnea, and psychiatric comorbidities.
• Healthy lifestyle changes that promote weight loss constitute the primary management
for obesity and metabolic syndrome.

ABP Content Specifications(s)


• Plan appropriate initial management of a patient with metabolic syndrome
• Recognize the clinical features associated with metabolic syndrome
• Plan an appropriate screening evaluation for metabolic syndrome, considering risk factors
that necessitate such screening

American Academy of Pediatrics 178


PREP ® Self-Assessment PREPSA 2021
Suggested Readings
• Styne DM, Arslanian SA, Connor EL, et al. Pediatric obesity—assessment, treatment,
and prevention: an Endocrine Society clinical practice guideline. J Clin Endocrinol
Metab. 2017;102(3):709-757. doi:10.1210/jc.2016-2573.
• Wang HC, Gahagan S. Obesity and metabolic syndrome. In: McInerny TK, Adam HM,
Campbell DE, DeWitt TG, Foy JM, Kamat DM, eds. American Academy of Pediatrics
Textbook of Pediatric Care. 2nd ed. Elk Grove Village, IL: American Academy of
Pediatrics; 2017:2396-2405. Pediatric Care Online.
• Wittcopp C, Conroy R. Metabolic syndrome in children and adolescents. Pediatr
Rev. 2016;37(5):193-202. doi:10.1542/pir.2014-0095.

American Academy of Pediatrics 179


PREP ® Self-Assessment PREPSA 2021
Question 47
A 3-year-old boy is seen by the otolaryngologist for progressive hoarseness over the last few
months. When crying, his voice is weak and he has respiratory distress. He has an intermittent
cough without fever, rhinorrhea, congestion, emesis, or abdominal pain. He was born at term via
vaginal delivery and does not have any chronic medical conditions. Vital signs include a
temperature of 37°C, heart rate of 115 beats/min, respiratory rate of 30 breaths/min, and blood
pressure of 95/52 mm Hg. On physical examination there is moderate stridor.
Nasopharyngoscopy is performed (Item Q47 ).

Item Q47: Findings for the boy described in the vignette.


Reprinted with permission from Kimberlin DW, et al, eds. Red Book Online. Itasca, IL:
American Academy of Pediatrics; 2018.

Of the following, the MOST likely cause of this boy’s symptoms is


A. laryngeal web
B. respiratory papillomatosis
C. subglottic hemangioma
D. vocal fold nodule

American Academy of Pediatrics 180


PREP ® Self-Assessment PREPSA 2021
Correct Answer: B
The most likely cause of the boy’s symptoms is respiratory papillomatosis, suggested by the
polypoid lesion seen on nasopharyngoscopy in the context of the child’s age and sex.
Respiratory papillomatosis that occurs before 18 years of age is considered juvenile-onset
disease. It is typically diagnosed between the ages of 2 and 5 years. This condition is thought to
result from vertical transmission of human papillomavirus (HPV; types 6 or 11) from the genital
tract of a mother to her child. Risk factors include being a firstborn child, vaginal delivery, and
birth to a teenage mother with genital papillomatosis. On endoscopy, the lesions have a
verrucous or polypoid appearance.

Symptoms of respiratory papillomatosis include hoarseness, stridor, and abnormal cry. Severe
forms can lead to airway obstruction. The lesions are managed with surgical excision.
Investigational treatments have included intralesional interferon, oral indole-3-carbinol,
photodynamic therapy, and intralesional cidofovir. Human papillomavirus types 6 and 11 have
low potential for malignant transformation. In addition to respiratory papillomatosis, they are
associated with genital warts and conjunctival papillomatosis.

Human papillomavirus can lead to oropharyngeal cancer, which is the most common HPV-
associated cancer in men. Although approximately 70% of oropharyngeal cancers are HPV-
associated, they are usually attributable to HPV type 16.

Laryngeal webs and subglottic hemangiomas are congenital laryngeal anomalies. In laryngeal
webs, the vocal folds are connected by a web that can be located either anteriorly or posteriorly.
Symptoms can mimic those of respiratory papillomatosis. Subglottic hemangiomas have a higher
prevalence in females. Like other hemangiomas, they undergo a rapid proliferation phase
followed by stabilization and then involution. This boy’s lesion does not have the appearance of
a hemangioma. Vocal fold nodules result from repeated abuse of the vocal folds from screaming
and shouting. They are the most common cause of hoarseness in school-aged children, and most
commonly occur in boys at the age of 9 years. Vocal fold nodules are visualized directly on the
vocal folds and are bilateral.

PREP Pearls
• Respiratory papillomatosis is thought to result from vertical transmission of human
papillomavirus (types 6 or 11) from the genital tract of a mother to her child.
• Symptoms of respiratory papillomatosis include hoarseness, stridor, and abnormal cry;
severe forms can lead to airway obstruction.
• Human papillomavirus types 6 and 11 have low potential for malignant transformation.

ABP Content Specifications(s)


• Recognize the clinical features associated with human papillomavirus infection
• Understand the epidemiology of human papillomavirus infection

American Academy of Pediatrics 181


PREP ® Self-Assessment PREPSA 2021
Suggested Readings
• American Academy of Pediatrics. Human papillomaviruses. In: Kimberlin DW, Brady
MT, Jackson MA, Long SS, eds. Red Book: 2018 Report of the Committee on Infectious
Diseases. American Academy of Pediatrics; 2018:582-590. Red Book Online.
• Conrad C, David N. Cornfield DN. Airway obstruction. In: McInerny TK, Adam HM,
Campbell DE, DeWitt TG, Foy JM, Kamat DM, eds. American Academy of Pediatrics
Textbook of Pediatric Care. 2nd ed. Itasca, IL: American Academy of Pediatrics;
2016;chap 348:2777-2785. Pediatric Care Online.
• Eliscu A. Human papillomavirus and HPV vaccines. Pediatr Rev. 2017;38(9):443- 445.
doi: 10.1542/pir.2016-0018.

American Academy of Pediatrics 182


PREP ® Self-Assessment PREPSA 2021
Question 48
A 15-year-old adolescent is seen for a health supervision visit. She is a competitive dancer who
participates in ballet, tap, modern, and lyrical dance, with classes and rehearsals year round, for
12 to 18 hours each week. Over the past 2 years, she has sustained stress fractures in her foot and
lower leg, which healed after an appropriate period of rest and progressive return to activity. She
does not have any symptoms at present. The adolescent is otherwise healthy. Her growth has
been tracking along the 50th percentile for height, weight, and body mass index without recent
change. Her menses are regular. She has no dietary restrictions. Her physical examination
findings are unremarkable; gait, strength, and range of motion are normal. The adolescent asks
how she can minimize her risk of another injury this year.

Of the following, the test MOST likely to provide information useful in addressing this
adolescent’s concern is
A. foot and lower leg radiography
B. magnetic resonance imaging
C. serum calcium level
D. total 25-hydroxyvitamin D level

American Academy of Pediatrics 183


PREP ® Self-Assessment PREPSA 2021
Correct Answer: D
The test most likely to provide information useful in addressing this patient’s concern is a serum
total 25-hydroxyvitamin D level (25-OH-D). The 2014 American Academy of Pediatrics (AAP)
clinical report “Optimizing Bone Health in Children and Adolescents” recommends vitamin D
screening for those with conditions associated with low bone mass or those with multiple low-
impact fractures, such as the girl in the vignette.

Stress fractures occur when recovery and nutritional support are inadequate to heal the bony
microtrauma that occurs during episodes of high-intensity physical activity. Prevention of stress
fractures requires assessment and management of those issues that increase microtrauma
development or decrease the body’s capacity for bony recovery. These include:
• Avoiding rapid increases in training volume or intensity
• Adequate recovery between training sessions
o Hard workouts should be followed by 36 to 48 hours of easy workouts or rest
• Maintaining sufficient calorie intake to support good menstrual function and adequate
estrogen levels in female athletes
• Ensuring adequate vitamin D and calcium intake

Although there has been some disagreement as to how to define an “adequate” serum level of 25-
OH-D, a 2016 international consensus statement made the following recommendations:
• Deficiency: <12 ng/mL (<30 nmol/L)
• Insufficiency: 12-20 ng/mL (30–50 nmol/L)
• Sufficiency: >20 ng/mL (>50 nmol/L)

It is important to recognize that these levels are based on a goal of avoidance of nutritional
rickets, and do not necessarily represent optimal levels for bone health. For example, osteoclasts
are activated and calcium bioavailability is decreased when 25-OH-D levels are less than 30
ng/mL (74 nmol/L). Evidence demonstrates that in athletes and military populations, stress
fracture rates are lower when 25-OH-D levels are greater than or equal to 30 ng/mL. Based on
these findings, the current recommendation for athletes to minimize injury risk and optimize
athletic performance is to maintain 25-OH-D levels between 32 and 50 ng/mL (80-125 nmol/L).

Providers should also recognize that many adolescents are not maintaining adequate vitamin D
levels. Cross-sectional studies of adolescents have found 25-OH-D deficiency rates between 17%
and 47%. Certain subpopulations are at particular risk for deficiency, including those who:
• Live north of the 35th parallel (which runs along the southern border of Tennessee and
through Santa Fe, NM)
• Minimize absorption of UVB radiation with clothing and/or sunscreen
• Have dark skin pigmentation
• Are overweight or have obesity
• Are athletes who usually train indoors (such as dancers and gymnasts)

Screening radiography does not have a role in looking for bone stress injury in asymptomatic
children. Although magnetic resonance imaging (MRI) is more sensitive than radiography for
detecting stress fractures and reactions (precursor to a stress fracture), dancers often have

American Academy of Pediatrics 184


PREP ® Self-Assessment PREPSA 2021
asymptomatic areas of bone stress response; therefore, performing MRI without a specific
symptomatic indication would lead to overdiagnosis of these as areas of pathology.

Decisions regarding whether to perform bone densitometry in children with fractures are more
nuanced. In otherwise healthy children with fractures, the AAP currently recommends bone
densitometry for children who have sustained clinically significant fractures (≥2 long bone
fractures before age 10 years, ≥3 long bone fractures before age 19 years, or a vertebral fracture
occurring without significant trauma or local disease). The specific role of stress fractures in
determining the need for densitometry is not clearly delineated in the AAP report. However, the
stress fractures of the girl in the vignette should raise concerns about the female athlete triad. The
triad is defined as caloric intake that is insufficient to support both healthy physiologic function
and the energy needs of training, menstrual dysfunction, and loss of bone mineral density.
Although many athletes with the triad do have low body weight, some athletes with inadequate
energy availability suppress physiologic function even though body weight is maintained.
Recommendations for bone densitometry in female athletes at risk for the female athlete triad are
outlined in Item C48.

American Academy of Pediatrics 185


PREP ® Self-Assessment PREPSA 2021

American Academy of Pediatrics 186


PREP ® Self-Assessment PREPSA 2021

PREP Pearls
• Measurement of serum total 25-hydroxy vitamin D should be considered for athletes with
repeat stress fractures.
• Current vitamin D level recommendations for optimal bone health in athletes are higher
than the global consensus recommendations for avoidance of vitamin D deficiency and
insufficiency.

ABP Content Specifications(s)


• Identify the nutritional needs and complications associated with sports and recreational
activities, including cheer leading and dancing

Suggested Readings
• Bachrach LK, Gordon CM; SECTION ON ENDOCRINOLOGY. Bone Densitometry in
Children and Adolescents. Pediatrics. 2016;138(4):e20162398. doi:10.1542/peds.2016-
2398.
• De Souza MJ, Nattiv A, Joy E, et al. 2014 Female Athlete Triad Coalition consensus
statement on treatment and return to play of the female athlete triad: 1st International
Conference held in San Francisco, California, May 2012 and 2nd International
Conference held in Indianapolis, Indiana, May 2013. Br J Sports Med. 2014;48:289.
doi: 10.1136/bjsports-2013-093218.
• Golden NH, Abrams, SA; Committee on Nutrition. Optimizing bone health in children
and adolescents. Pediatrics. 2014;134(4);e1229-e1243 doi: 10.1542/peds.2014-217 .
• Munns CF, Shaw N, Kiely M, et al. Global consensus recommendations on prevention
and management of nutritional rickets. J Clin Endocrinol Metab. 2016;359:394-415.
doi: 10.1210/jc.2015-2175.
• Thomas DT, Erdman KA, Burke LM. American College of Sports Medicine joint
position statement: nutrition and athletic performance. Med Sci Sports
Exerc. 2016;48(3):543-568. doi: 10.1249/MSS.0000000000000852.

American Academy of Pediatrics 187


PREP ® Self-Assessment PREPSA 2021
Question 49
A 2-month-old boy is seen in the office for a sick visit. His mother is concerned that he is
breathing fast and eating less. Over the last few days, while breastfeeding, he has tired out
sooner, breathed a little faster, and become sweaty. He now seems to be breathing faster when he
is at rest and not eating. He does not have rhinorrhea, cough, congestion, fevers, vomiting,
diarrhea, or rash. He is afebrile. He has a heart rate of 158 beats/min, a blood pressure of 75/45
mm Hg, and a respiratory rate of 60 breaths/min. He appears comfortable. His oxygen saturation
is 95%. His weight remains at the 50th percentile for age. He has a regular heart rate and rhythm
with a normal S1 and physiologically split S2. He has a grade 2/6 holosystolic murmur at the left
lower sternal border. He has good perfusion and normal pulses. His liver edge is palpated 3 cm
below the right costal margin.

Of the following, the BEST next step(s) in treatment is (are) to


A. admit the infant to the hospital for a septic evaluation
B. increase the infant’s caloric density of nutrition and refer him to a pediatric cardiologist
C. prescribe furosemide and refer the infant to a pediatric cardiologist
D. provide reassurance to the mother and follow up in 2 weeks for a weight check

American Academy of Pediatrics 188


PREP ® Self-Assessment PREPSA 2021
Correct Answer: C
The baby in the vignette has a clinical history and physical examination findings consistent with
a hemodynamically significant ventricular septal defect. At 2 months of age, he is demonstrating
that his pulmonary vascular resistance has dropped, making it far easier for blood to be shunted
through the ventricular septal defect and pulmonary outflow tract instead of to the systemic
circulation. Because of this, he has congested lungs with pulmonary edema and is breathing
faster. The next course of action would be treatment with furosemide to help relieve some of the
congestion and to refer him to a pediatric cardiologist for further care. He does not have signs or
symptoms of sepsis, and he has maintained his growth trajectory and does not need increased
calories at this time. Following up in 2 weeks after providing reassurance to the mother will only
delay care.

Acyanotic congenital heart disease can be categorized into two broad topics, left-to-right shunts
and obstruction to blood flow. Left-to-right shunts include lesions such as atrial septal defects,
ventricular septal defects, atrioventricular septal defects, and patent ductus arteriosus. Pulmonary
and aortic valve stenosis as well as coarctation of the aorta can be categorized as obstruction to
blood flow.

The cardiac lesions that result in left-to-right shunting do so through a defect that allows for
oxygenated blood from the lungs (present on the left side of the heart) to mix with deoxygenated
blood returning from the body to the right side of the heart. Oxygenated blood then returns to the
lungs instead of going out to the body. This ineffective blood flow results in pulmonary
congestion and edema, which in turn lead to tachypnea and increased work of breathing. The
degree of increased pulmonary blood flow is determined by the relative resistances between the
pulmonary and systemic vascular beds as well as the effective size of the ventricular septal
defect (ie, whether it is large or small). An atrial level shunt will result in right ventricular
enlargement, whereas a ventricular septal defect, atrioventricular septal defect, aortopulmonary
window, and patent ductus arteriosus result in left heart enlargement. Findings that can aid in
diagnosis include specific murmurs and hepatomegaly found on physical examination, cardiac
enlargement seen on chest radiograph, specific electrocardiographic findings dependent on the
lesion, and the appearance of the defect itself seen on echocardiogram. When hemodynamically
significant, these lesions can be repaired through cardiac catheterization or cardiac surgery.

Cardiac lesions that result in outflow obstruction include aortic and pulmonary valve stenosis, as
well as coarctation of the aorta. These lesions vary in severity from mild to severe. Those labeled
“critical” require a patent ductus arteriosus to maintain adequate pulmonary or systemic cardiac
output. If the ductus is inadequate, prostaglandin infusion will be required (pulmonary valve
stenosis, when critical, will actually be a cyanotic lesion). Both pulmonary and aortic valve
stenosis can present as an asymptomatic lesion with a murmur, ventricular hypertrophy, or both
on an electrocardiogram (or echocardiogram); diagnosis is made via echocardiography. These
lesions can be accompanied by fatigue and exercise intolerance. Significant aortic stenosis can
also present with chest pain, signs of ischemia, or both. Both types of valve stenosis can be
treated with balloon dilation in the cardiac catheterization laboratory or cardiac surgical
procedure. Coarctation of the aorta can present as a ductal-dependent (ie, critical coarctation)
lesion in the neonatal period with murmur, decreased femoral pulses, and tachypnea or
cardiogenic shock as the ductus begins to close or closes completely. In this instance,
American Academy of Pediatrics 189
PREP ® Self-Assessment PREPSA 2021
prostaglandin infusion will need to be initiated. In some children, however, coarctation of the
aorta will present later in life with murmur or hypertension. Coarctation of the aorta can be
treated with cardiac catheterization or cardiac surgery.

PREP Pearls
• Cardiac lesions with left-to-right shunting can include atrial septal defects, ventricular
septal defects, atrioventricular septal defects, aortopulmonary window, and patent ductus
arteriosus. Hemodynamically significant lesions result in cardiac enlargement and
pulmonary congestion and warrant repair.Pulmonary or aortic valvar stenosis results in
varying degrees of obstruction to blood flow and ventricular hypertrophy; lesions can be
treated by balloon dilation in the cardiac catheterization laboratory.
• Pulmonary or aortic valvar stenosis results in varying degrees of obstruction to blood
flow and ventricular hypertrophy; lesions can be treated by balloon dilation in the cardiac
catheterization laboratory.
• Coarctation of the aorta results in varying degrees of obstruction to blood flow and can
cause cardiogenic shock in the neonate. Coarctation can be treated with cardiac surgery
or in the cardiac catheterization laboratory.

MOCA-Peds Objective
• Recognize genetic causes of congenital heart disease.

ABP Content Specifications(s)


• Recognize the major clinical findings associated with the various types of acyanotic
congenital heart disease

Suggested Readings
• McCulloch MA, Gajarski RJ. Congenital and acquired heart disease. In: McInerny TK,
Adam HM, Campbell DE, DeWitt TG, Foy JM, Kamat DM, eds. American Academy of
Pediatrics Textbook of Pediatric Care. 2nd ed. Elk Grove Village, IL: American
Academy of Pediatrics; 2017:1883-1916. Pediatric Care Online .
• Price JF. Congestive heart failure in children. Pediatr Rev. 2019;40(2):60-70.
doi:10.1542/pir.2016-0168.
• Puri K, Allen HD, Qureshi AM. Congenital heart disease. Pediatr Rev. 2017;38(10):471-
486. doi:10.1542/pir.2017-0032.

American Academy of Pediatrics 190


PREP ® Self-Assessment PREPSA 2021
Question 50
A 10-year-old girl has had a chronic wet cough, present day and night, for 6 months. She has no
identified triggers. It is not exacerbated by exercise and does not limit her in any way. There is
no seasonal pattern and no tobacco smoke or other irritant exposure. There is a pet dog in the
home. Review of systems finds nasal congestion without rhinorrhea. The family history is
negative for asthma, allergies, and immunodeficiency.

Physical examination shows mildly edematous nasal mucosa without discharge. The oropharynx
and chest wall anatomy are normal. Breath sounds are somewhat coarse with a few scattered
rhonchi and no fine rales or wheezes.

Spirometry findings are normal. Chest radiography demonstrates a mild increase in central
peribronchial markings with no other abnormalities.

Of the following, the BEST next management step for this patient is
A. computed tomography of the chest
B. measurement of quantitative immunoglobulin levels
C. removal of the dog from the home
D. 3-week course of amoxicillin/clavulanic acid

American Academy of Pediatrics 191


PREP ® Self-Assessment PREPSA 2021
Correct Answer: D
The girl in the vignette most likely has persistent bacterial bronchitis, a relatively recently
described cause of chronic cough in children. Before initiation of usually fruitless diagnostic
tests, an extended course of antibiotics is indicated in children who have no risk factors for
underlying disease. In this child, there is nothing to suggest asthma other than the cough and
nothing to suggest an immunodeficiency or structural anomaly of the chest or airway. There is no
family history of allergy or other signs or symptoms of allergy in her history or on physical
examination. Thus, computed tomography of the chest, measurement of immunoglobulins, or
changes in family lifestyle are not indicated.

After obtaining a complete history and performing a thorough physical examination, cough in
children can be divided into acute and chronic categories. A chronic cough is one that has been
present for at least 4 continuous weeks. Both acute and chronic coughs can be further divided
into specific and nonspecific categories. A cough for which there is a recognizable trigger or
characteristic pattern is considered to be a specific cough; intervention is guided by the specific
pointers of cough (specific signs and/or symptoms) that may be obtained from history or physical
examination (Item C50). For example, a chronic brassy, barking cough that disappears with
distraction and when the child is asleep is most likely a habit cough. A chronic wet cough
associated with recurrent otitis and/or sinusitis suggests immunodeficiency or ciliary dyskinesia.
A dry cough that is exacerbated by exercise and associated with frequent night awakenings is
often caused by asthma or airway reactivity, especially when there is a family history of asthma.

American Academy of Pediatrics 192


PREP ® Self-Assessment PREPSA 2021

American Academy of Pediatrics 193


PREP ® Self-Assessment PREPSA 2021
A chronic cough with no abnormal physical examination findings and no history findings
suggestive of a specific etiology is considered a nonspecific cough. Appropriate diagnostic
investigations in this context include chest radiography and a pulmonary function test. No
additional investigations are indicated unless results of these tests point to a specific diagnosis
requiring further evaluation. For the child in the vignette, the chronic wet cough with no other
abnormal findings suggests persistent bacterial bronchitis; the most appropriate management is a
2- to 4-week course of amoxicillin/clavulanate or equivalent narrow-spectrum antibiotic
appropriate for respiratory flora.

PREP Pearls
• Chronic cough is a cough that has been present for at least 4 continuous weeks.
• Specific cough pointers (signs and/or symptoms) are findings from the history or physical
examination that suggest a specific etiology for the cough and help direct management.
• Nonspecific cough has no findings on history or physical examination that suggest an
etiology. Appropriate evaluation includes chest radiography and pulmonary function
testing (in children >6 years old); no additional testing is needed unless an abnormality is
suggested by the results of these tests.

ABP Content Specifications(s)


• Plan effective screening evaluation of chronic cough
• Plan the appropriate clinical and diagnostic evaluation of cough of various etiologies

Suggested Readings
• Chang AB, Oppenheimer JJ, Weinberger M, Grant CC, Rubin BK, Irwin RS; CHEST
Expert Cough Panel. Etiologies of chronic cough in pediatric cohorts: CHEST guidelines
and expert panel report. Chest. 2017;152:607-617. doi: 10.1016/j.chest.2017.06.006.
• Goldsobel AB, Chipps BE. Cough in the pediatric population. J Pediatr. 2010;156:352-
358. doi: 10.1016/j.jpeds.2009.12.004.
• Kasi AS, Kamerman-Kretzmer RJ. Cough. Pediatr Rev. 2019;40:157-165.
doi: 10.1542/pir.2018-0116.
• Marcus MG. Cough. In: McInerny TK, Adam HM, Campbell DE, DeWitt TG, Foy JM,
Kamat DM, eds. American Academy of Pediatrics Textbook of Pediatric Care. 2nd ed.
Itasca, IL: American Academy of Pediatrics; 2016;chap 135:1247-1251. Pediatric Care
Online.

American Academy of Pediatrics 194


PREP ® Self-Assessment PREPSA 2021
Question 51
A 13-year-old adolescent girl is brought to the office by her mother. The mother is concerned
because her daughter’s period has lasted for 2 weeks. She is using 5 to 6 overnight pads per day
and occasionally soaks through her pads at school. She had menarche at age 12 years and had 2
regular periods followed by no bleeding for 5 months. She reports no fatigue, headaches,
palpitations, or dizziness. She has no other significant medical history except for nosebleeds
associated with seasonal allergies during the spring and summer. She has a temperature of
37.1°C, a heart rate of 96 beats/min, and a blood pressure of 110/76 mm Hg. She is at the 50th
percentile for height and weight. Her physical examination findings are significant for
conjunctival pallor. Her lungs are clear bilaterally. Her heart examination is significant for
tachycardia but no murmurs or gallops. She has no hepatosplenomegaly, petechiae, or lesions on
her skin.

Of the following, the MOST appropriate initial test in determining the etiology of this patient’s
symptoms is
A. complete blood cell count with differential
B. thyroid-stimulating hormone test
C. urine pregnancy test
D. von Willebrand disease panel

American Academy of Pediatrics 195


PREP ® Self-Assessment PREPSA 2021

Correct Answer: C
The adolescent girl in the vignette most likely has abnormal uterine bleeding (AUB) owing to
anovulatory cycles. The initial test in her evaluation would be a urine pregnancy test to evaluate
for a pregnancy and any associated complications that could be a potential medical emergency.
A complete blood cell count with differential, a thyroid-stimulating hormone test, and a von
Willebrand disease panel would be drawn as part of this girl’s evaluation, but pregnancy should
be ruled out first.

The average age of menarche is 12.5 years. Compared with the adult menstrual cycle, which
ranges between 21 and 35 days, the adolescent menstrual cycle is more variable, ranging from 21
to 45 days. The duration of normal flow is between 3 and 7 days, and normal blood loss is 30 to
40 mL per period (3 to 6 pads or tampons per day; 10 to 15 soaked pads per cycle). In contrast,
AUB manifests with heavy menstrual bleeding characterized by blood loss greater than 80 mL in
one cycle; soaking more than one tampon or pad per hour; passing large clots > 1 cm; or soaking
through clothing at night.

The universally accepted classification system for causes of AUB in women of reproductive age
is PALM-COEIN, which stands for
• Polyp
• Adenomyosis
• Leiomyoma
• Malignancy/hyperplasia
• Coagulopathy
• Ovulatory dysfunction
• Endometrial
• Iatrogenic
• Not yet classified

The PALM component encompasses structural etiologies, whereas the COEIN component
focuses on nonstructural etiologies. The majority of cases of AUB in adolescents are
nonstructural. Younger adolescent girls in the first several years after menarche often have
anovulatory menstrual cycles, one of the most common etiologies of AUB. Anovulatory cycles
may be due to a combination of incomplete shedding of a proliferative endometrium and
immaturity of the hypothalamic pituitary ovarian axis. There can be a delay in the maturation of
the normal negative feedback loop, which affects levels of follicle-stimulating hormone and
luteinizing hormone. The differential diagnosis of AUB is extensive and can include pregnancy
and related complications, endocrinopathies such as polycystic ovarian syndrome and thyroid
disease, infection (cervicitis), bleeding disorders (Von Willebrand disease and platelet
dysfunction disorders), medications (use of hormonal birth control methods and anticoagulants),
foreign bodies, and trauma.

Evaluation of AUB begins with history, both with and without the parent in the room, to get a
detailed menstrual, medical, family, and sexual history, as well as a physical examination to
establish hemodynamic stability. Laboratory testing should include a pregnancy test, complete
blood count, thyroid-stimulating hormone level test, prothrombin time, and partial
American Academy of Pediatrics 196
PREP ® Self-Assessment PREPSA 2021
thromboplastin time. A von Willebrand disease panel should be drawn for adolescents with
excessive bleeding within the first two menstrual cycles or a personal or family history of
bleeding or coagulopathy. Heavy menstrual bleeding at menarche is often the first sign of a
bleeding disorder. For adolescents who are receiving an estrogen-containing hormonal
medication, a von Willebrand disease panel should be drawn when the patient has not received
hormones for at least 1 week. Estrogen can elevate the von Willebrand factor to normal levels.
Screening for Chlamydia trachomatis and Neisseria gonorrhœae, pelvic examination, and pelvic
ultrasonography may be warranted in sexually active females.

Management of AUB is based on the severity of anemia and bleeding and involves ensuring that
the patient is hemodynamically stable. Treatment of adolescents with mild bleeding and no
anemia can be limited to observation alone. Combined oral contraceptive pill regimens are often
used because the estrogen improves hemostasis of the endometrial lining. Progestin-only pill
regimens can also be used in adolescents with moderate anemia or contraindications to the use of
estrogen. An adolescent with a hemoglobin level less than 7 g/dL, heavy bleeding, and vital sign
instability (tachycardia, hypotension, and orthostatic changes) must be admitted to the hospital
for observation and treatment. If a patient with severe anemia and bleeding does not respond to
traditional therapy involving use of a combined oral contraceptive, intravenous conjugated
estrogen, and antifibrinolytic medications, an examination under anesthetic with therapeutic
curettage may be warranted in some cases. Item C51 offers a summary of treatment options.

American Academy of Pediatrics 197


PREP ® Self-Assessment PREPSA 2021

American Academy of Pediatrics 198


PREP ® Self-Assessment PREPSA 2021

PREP Pearls
• Abnormal uterine bleeding is the most common gynecologic complaint in adolescent
females.
• The most common cause of abnormal uterine bleeding in adolescents is anovulatory
cycles.
• The PALM-COEIN classification system defines causes of abnormal uterine bleeding
and differentiates between structural and nonstructural etiologies. (PALM-COEIN stands
for polyp, adenomyosis, leiomyoma, malignancy/hyperplasia, coagulopathy, ovulatory
dysfunction, endometrial, iatrogenic, not yet classified.)
• In adolescents with abnormal uterine bleeding, a pregnancy test must be done as the
initial step in evaluation.

ABP Content Specifications(s)


• Formulate a differential diagnosis of dysfunctional uterine bleeding
• Plan the appropriate management of dysfunctional uterine bleeding
• Recognize the various etiologies of menometrorrhagia
• Plan the appropriate evaluation of dysfunctional uterine bleeding

Suggested Readings
• Committee on Practice Bulletins—Gynecology. Practice bulletin no. 128: diagnosis of
abnormal uterine bleeding in reproductive-aged women. Obstet
Gynecol. 2012;120(1):197-206. doi:10.1097/AOG.0b013e318262e320.
• Elmaogullari S, Aycan Z. Abnormal uterine bleeding in adolescents. J Clin Res Pediatr
Endocrinol. 2018;10(3):191-197.
• Munro MG. Practical aspects of the two FIGO systems for management of abnormal
uterine bleeding in the reproductive years. Best Pract Res Clin Obstet
Gynæcol. 2017;40:3-22. doi:10.1016/j.bpobgyn.2016.09.011.
• Trent M, Joffe A. Vaginal bleeding. In: McInerny TK, Adam HM, Campbell DE, DeWitt
TG, Foy JM, Kamat DM, eds. American Academy of Pediatrics Textbook of Pediatric
Care. 2nd ed. Elk Grove Village, IL: American Academy of Pediatrics; 2017:1653-
1656. Pediatric Care Online .

American Academy of Pediatrics 199


PREP ® Self-Assessment PREPSA 2021
Question 52
A 5-year-old girl is seen in the emergency department for acute onset of left arm weakness. Two
weeks earlier she had a nonspecific viral illness that self-resolved. She awoke on the day of
admission with inability to move her left arm. She denies pain, numbness, headache, bowel or
bladder involvement, or prior trauma.

In the emergency department, the girl is alert, pleasant, and interactive with no evidence of
encephalopathy. Her vital signs and general physical examination findings are normal. On
neurologic examination, cranial nerve findings, fundoscopy, sensation, and gait are normal.
Motor examination is notable for flaccid paralysis of the left upper extremity with areflexia;
strength, reflexes, and coordination are preserved in her other extremities.

Brain magnetic resonance imaging (MRI) is normal. Spine MRI scans are shown (Item
Q52A and Q52B). Lumbar puncture reveals clear cerebrospinal fluid with a lymphocytic
pleocytosis. Cerebrospinal fluid is sent for viral testing and bacterial culture.

Item Q52A: Axial T2 image of the thoracic spine.


Courtesy of J Goldstein

American Academy of Pediatrics 200


PREP ® Self-Assessment PREPSA 2021

Item Q52B: Sagittal STIR image of the spine.


Courtesy of J Goldstein

Of the following, the MOST likely diagnosis for this girl is


A. acute flaccid myelitis
B. acute disseminated encephalomyelitis
C. Guillain-Barré syndrome
D. idiopathic transverse myelitis

American Academy of Pediatrics 201


PREP ® Self-Assessment PREPSA 2021
Correct Answer: A
The clinical presentation, imaging, and laboratory findings of the girl in the vignette are most
consistent with a diagnosis of acute flaccid myelitis (AFM). Acute flaccid myelitis is a rare
neurologic disorder presenting with acute paralysis of 1 or more limbs, preserved sensation, and
normal mental status, typically within a week of onset of viral symptoms. Clinical severity is
variable. Magnetic resonance imaging (MRI) of the spine will show extensive longitudinal
centrally located inflammation affecting the gray matter of the cord. Cerebrospinal fluid analysis
typically reveals a lymphocytic pleocytosis.

Clusters of AFM have been reported since 2014 with a biennial pattern, usually occurring in the
late summer and early fall. Although no single pathogen has been identified, enteroviruses,
specifically enterovirus D68, A71, and coxsackie A16, are implicated. The Centers for Disease
Control should be notified when a case of AFM is suspected. Information, reporting forms, and
interim clinical management guidelines can be found at: https://www.cdc.gov/acute-flaccid-
myelitis/index.html. Treatment is supportive; there is currently no recommendation for
adjunctive therapies such as intravenous immunoglobulin, corticosteroids, or plasmapheresis.
Recovery is variable and often incomplete.

The differential diagnosis of AFM can be broad and is shaped by history and physical
examination findings. Acute disseminated encephalomyelitis (ADEM) is a postinfectious
inflammatory disorder with variable presentation determined by the parts of the brain and spinal
cord affected. Encephalopathy is required for the diagnosis, with symptoms ranging from
seizures, headache, ataxia, paralysis, and paresthesias. Brain MRI will demonstrate evidence of
demyelination, with focal or multifocal lesions of the white matter without evidence of prior
white matter lesions. In addition to supportive and symptomatic care, immunotherapy such as
corticosteroids are used in the treatment of ADEM.

Guillain-Barré syndrome is a postinfectious inflammatory demyelinating polyneuropathy,


typically presenting with an ascending paralysis characterized by areflexia and weakness, with
the potential to progress to respiratory failure. Diagnosis is based on clinical findings; the results
of cerebrospinal fluid analysis, imaging, and nerve conduction studies may lend support.
Treatment includes a combination of supportive care, with close neurologic monitoring and
administration of intravenous immunoglobulin or plasmapheresis.

Idiopathic transverse myelitis is a postinfectious inflammatory disorder of the spinal cord


typically presenting with myelopathy symptoms over hours to days. Symptoms include bilateral
motor and sensory deficits. Bowel and bladder dysfunction and autonomic dysregulation may
also occur. Spinal MRI demonstrates a longitudinal lesion traversing multiple contiguous
segments involving both the white and gray matter of the spinal cord. Early neuroimaging can be
normal; treatment with corticosteroids should be initiated if clinical suspicion is high.

American Academy of Pediatrics 202


PREP ® Self-Assessment PREPSA 2021
PREP Pearls
• Acute flaccid myelitis is a rare neurologic disorder presenting with acute paralysis of 1 or
more limbs, preserved sensation, and normal mental status, typically within a week of
viral symptoms.
• Treatment of acute flaccid myelitis is supportive; recovery is variable and often
incomplete.
• Suspected cases of acute flaccid myelitis should be reported to the Centers for Disease
Control and Prevention with samples sent for viral testing.

MOCA-Peds Objective
• Recognize the child with spinal cord compression from a mass.

ABP Content Specifications(s)


• Understand the benefits and limitations of neurodiagnostic tests in the evaluation of
weakness
• Differentiate the causes of acute, subacute, and chronic weakness

Suggested Readings
• Ayers T, Lopez A, Lee A, et al. Acute flaccid myelitis in the United States: 2015-
2017. Pediatrics. 2019;144(5):e20191619. doi: 10.1542/peds.2019-1619.
• Centers for Disease Control. Acute flaccid myelitis. https://www.cdc.gov/acute-flaccid-
myelitis/index.html.
• Fatemi Y, Chakraborty R. Acute flaccid myelitis: a clinical overview for 2019. Mayo
Clinic Proc. 2019;94(5):875-881. doi: 10.1016/j.mayocp.2019.03.011.

American Academy of Pediatrics 203


PREP ® Self-Assessment PREPSA 2021
Question 53
A 3-year-old boy is brought to the emergency department after sustaining a scald burn to the
majority of his torso. His family states that he reached up to the stove and pulled on the handle of
a pot of boiling water that spilled on the anterior surface of his chest. The total body surface area
burned is 40%. After the patient has undergone initial resuscitation and fluid management, the
burn service asks about the plan for nutritional support.

Of the following, the MOST appropriate nutritional regimen for this patient is administration of
A. dextrose-containing intravenous fluids at 30% above maintenance rate for 24 hours,
followed by initiation of half-strength enteral formula with daily multivitamin
B. full enteral feedings with high-protein formula supplemented with vitamins C and A and
zinc sulfate to provide 30% above resting energy expenditure
C. isotonic saline at maintenance rate with bowel rest for 72 hours followed by trophic
feedings with 30% of full-volume enteral formula
D. total parenteral nutrition with the recommended dietary allowance of protein,
carbohydrates, and fats to meet the daily resting energy expenditure

American Academy of Pediatrics 204


PREP ® Self-Assessment PREPSA 2021
Correct Answer: B
The boy in this vignette has sustained a burn to 40% of his body surface area. Pediatric thermal
injuries are common and constitute one of the main reasons for trauma care in children younger
than 5 years. In patients who have significant burns, nutritional support is known to be an
important factor in ensuring a positive outcome. Unlike other traumatic injuries, burns result in
an increase in resting energy expenditure (REE) and hypermetabolic state. Thus, the prescription
of nutrition must address the increase in caloric requirements. Of the choices given, providing
full enteral feedings with high-protein formula supplemented with vitamins C and A and zinc
sulfate to provide 30% above REE is the best nutritional plan to provide appropriate calories and
nutrition for promotion of optimal wound healing.

Children with burns over less than 10% of their total body surface area (TBSA) have REE that is
nearly normal; however, those with burns that affect more than 40% TBSA have REE that may
be as high as 1.5 times the normal REE. In children with significant TBSA burns, adequate
nutrition is required, and the current recommendations include initiation of enteral nutrition as
soon as possible to ensure positive nitrogen balance and reduce gastrointestinal bacterial
translocation. In rare circumstances in which enteral nutrition cannot be administered owing to
severe malabsorption or feeding intolerance, parenteral nutrition can be used, but it should still
provide the appropriate calories to meet the increase in REE. Nutritional supplementation with
vitamins C and A and zinc sulfate will ensure adequate wound healing and should be prescribed.
Half-strength formula will not provide adequate caloric intake for the boy in the vignette. There
is no indication for bowel rest, nor for restriction to 30% of goal enteral feeding volume.
Although parenteral nutrition may be required, this option is incorrect because it does not
provide increased REE, and the first-line nutrition option in children with burns should be enteral
unless otherwise contraindicated.

PREP Pearls
• Adequate nutritional support is required for optimal wound healing in pediatric patients
with burns, and the enteral route is preferred, unless otherwise contraindicated, to ensure
positive nitrogen balance and reduce gastrointestinal bacterial translocation.
• For children with large total body surface area burns, caloric needs will be higher than
usual resting energy expenditure.
• Vitamins C and A and zinc sulfate should be prescribed in burn management to ensure
adequate wound healing.

ABP Content Specifications(s)


• Recognize the specific nutritional problems in children with burns

American Academy of Pediatrics 205


PREP ® Self-Assessment PREPSA 2021

Suggested Readings
• Abdullahi A, Jeschke MG. Nutrition and anabolic pharmacotherapies in the care of burn
patients. Nutr Clin Pract. 2014;29(5):621-630. doi:10.1177/0884533614533129.
• Sheridan RL. Burn care for children. Pediatr Rev. 2018;39(6):273-286.
doi:10.1542/pir.2016-0179.
• Sheridan RL. Thermal injuries. In: McInerny TK, Adam HM, Campbell DE, DeWitt TG,
Foy JM, Kamat DM, eds. American Academy of Pediatrics Textbook of Pediatric
Care. 2nd ed. Elk Grove Village, IL: American Academy of Pediatrics; 2017:2987-
2994. Pediatric Care Online .

American Academy of Pediatrics 206


PREP ® Self-Assessment PREPSA 2021
Question 54
A 10-month-old male infant is seen for follow-up. He was diagnosed with bilateral acute otitis
media 3 days ago after developing fever and otalgia with abnormal tympanic membrane findings.
He was prescribed a 10-day course of high-dose amoxicillin and ibuprofen as needed. He has
received 6 doses of amoxicillin, but the fever persists despite administration of ibuprofen every 6
hours. He has had a poor appetite and a few episodes of nonbloody, nonbilious emesis. His
parents have also noted that his eyes seem swollen and he is urinating less frequently. His history
includes a previous history of otitis media treated successfully with amoxicillin at 4 months of
age. He appears to be nontoxic and well hydrated. His weight and length are in the 75th
percentile, and his blood pressure is in the 87th percentile. There is mild periorbital edema
without conjunctival or scleral injection and no discharge. Other significant findings include
erythematous, opaque, and bulging tympanic membranes bilaterally. The remainder of the
physical examination findings are within normal limits. Laboratory findings reveal sterile pyuria,
proteinuria, hematuria, and elevated levels of blood urea nitrogen and creatinine. Findings of a
urine culture are negative.

Of the following, the BEST initial step in management is to


A. admit for intravenous fluid administration
B. admit for intravenous steroid administration
C. discontinue amoxicillin
D. discontinue ibuprofen

American Academy of Pediatrics 207


PREP ® Self-Assessment PREPSA 2021
Correct Answer: D
The clinical presentation of the infant in this vignette is consistent with acute interstitial nephritis
(AIN) caused by the nonsteroidal anti-inflammatory drug (NSAID) ibuprofen, which can result
in acute kidney dysfunction (Item C54 ). Acute kidney injury induced by NSAIDs may be
hemodynamically mediated or secondary to AIN. Nonsteroidal anti-inflammatory drugs lead to a
decrease in glomerular filtration rate by causing reversible renal ischemia, even more so in a
setting of decreased intravascular volume (dehydration, vomiting, and diarrhea). The child in the
vignette has decreased appetite and vomiting, and is at risk of experiencing acute kidney injury
secondary to NSAID-induced renal vasoconstriction. However, the presence of hematuria,
proteinuria, and sterile pyuria favors a diagnosis of AIN. Acute interstitial nephritis should be
included in the differential for sterile pyuria.

American Academy of Pediatrics 208


PREP ® Self-Assessment PREPSA 2021

Amoxicillin may also cause AIN, but in this vignette the infant had received amoxicillin in the
past without issue. Reactions may recur with re-exposure to the same class of medications and,
therefore, ibuprofen should be added to this patient’s allergy list.

American Academy of Pediatrics 209


PREP ® Self-Assessment PREPSA 2021
The mainstay of treatment for drug-induced AIN is discontinuation of the offending agent, which
in this case is ibuprofen. The majority of patients improve after the offending medication is
withdrawn. However, for patients with AIN-caused acute kidney injury that is severe enough to
warrant dialysis, glucocorticoids may be initiated after a renal biopsy. Nonsteroidal anti-
inflammatory drug–induced AIN does not typically respond to glucocorticoid therapy. Other
supportive interventions may include fluid and electrolyte management, adequate hydration, and
avoidance of nephrotoxic drugs. Increasing hydration alone without removal of the NSAID
would not be adequate treatment for this patient.

PREP Pearls
• Drug-induced acute interstitial nephritis is commonly caused by nonsteroidal anti-
inflammatory drugs, such as ibuprofen.
• The mainstay of initial treatment for drug-induced acute interstitial nephritis is
discontinuation of the offending agent. The majority of patients improve after the
offending medication is withdrawn.
• Supportive interventions for acute interstitial nephritis may include fluid and electrolyte
management, adequate hydration, and avoidance of nephrotoxic drugs.

ABP Content Specifications(s)


• Recognize the risks associated with the use of nonsteroidal anti-inflammatory drugs

Suggested Readings
• Joyce E, Glasner P, Ranganathan S, Swiatecka-Urban A. Tubulointerstitial nephritis:
diagnosis, treatment, and monitoring. Pediatr Nephrol. 2017;32(4):577-587.
doi:10.1007/s00467-016-3394-5.
• Kodner CM, Kudrimoti A. Diagnosis and management of acute interstitial nephritis. Am
Fam Physician. 2003;67(12):2527-
2534. https://www.aafp.org/afp/2003/0615/p2527.html.
• Ruebner RL, Fadrowski JJ. Tubulointerstitial nephritis. Pediatr Clin N Am.
2019;66:(1)111-119. doi:10.1016/j.pcl.2018.08.009.
• Varade WS. Nephritis. In: McInerny TK, Adam HM, Campbell DE, DeWitt TG, Foy JM,
Kamat DM, eds. American Academy of Pediatrics Textbook of Pediatric Care. 2nd ed.
Elk Grove Village, IL: American Academy of Pediatrics; 2017:2358-2367. Pediatric Care
Online .

American Academy of Pediatrics 210


PREP ® Self-Assessment PREPSA 2021
Question 55
A 16-year-old adolescent boy diagnosed with acute myeloid leukemia 8 weeks ago has remained
hospitalized for fever and neutropenia since his last chemotherapy course. He has had persistent
fever and neutropenia for the last 5 days despite treatment with broad-spectrum antibiotics,
including vancomycin and ceftazidime. He is febrile and has a heart rate of 110 beats/min, a
respiratory rate of 24 breaths/min, a blood pressure of 110/86 mm Hg, and an oxygen saturation
of 92% on room air. He has decreased breath sounds and dullness to percussion at the right lower
lung field and in the right axillary area. Laboratory findings are notable for an absolute
neutrophil count of 200/µL (0.2 × 109/L), anemia, and thrombocytopenia. Blood culture results
have remained negative. Chest radiography and computed tomography with contrast (Item Q55)
show diffuse nodular densities of various sizes in both lung fields.

Item Q55: Computed tomography for the adolescent described in the vignette.
Courtesy J Fish

Of the following, the organism MOST likely responsible for the patient’s findings is
A. Aspergillus
B. mycoplasma
C. Pneumocystis jirovecii
D. Staphylococcus aureus

American Academy of Pediatrics 211


PREP ® Self-Assessment PREPSA 2021

Correct Answer: A
For the adolescent boy in the vignette—who has acute myelogenous leukemia, persistent
neutropenia, and fever while receiving broad-spectrum antibiotics; negative blood culture results;
and a finding of bilateral pulmonary nodules—the greatest concern regards invasive mold
infection, especially Aspergillus species. Aspergillus is a ubiquitous mold that grows in soil. In
humans, Aspergillus colonizes the respiratory tract. Acquisition of infection occurs via inhalation
of airborne conidia (spores) from various environmental sources and soil. The most common
cause of invasive aspergillosis is Aspergillus fumigatus, followed by Aspergillus flavus.

Invasive aspergillosis may complicate the clinical course of immunocompromised patients (eg,
those with acute myeloid leukemia, with aplastic anemia, undergoing intensive chemotherapy)
with prolonged neutropenia; stem-cell transplant recipients during periods of neutropenia and
graft-versus-host disease; patients with phagocyte dysfunction (eg, chronic granulomatous
disease); or those receiving T-lymphocyte immunosuppressive therapy (eg, steroids, calcineurin
inhibitors, tumor necrosis factor-ɑ inhibitors).

Pulmonary infection is the most common manifestation of invasive aspergillosis, but other sites,
including the sinus, skin, and brain, may be involved. Invasive pulmonary aspergillosis often
presents with unremitting fever in high-risk patients who are immunocompromised; other early
symptoms may include dry cough or chest pain. Dyspnea is noted in individuals with diffuse
pulmonary disease. Hemoptysis can occur and may be fatal, given the angioinvasive nature
of Aspergillus spp.

The diagnosis of invasive pulmonary aspergillosis is suggested by positive galactomannan assay


finding in the serum or bronchoalveolar lavage fluid (BAL) and evidence of dichotomously
branched and septate hyphae on fungal smears of tissue or BAL specimens. Negative fungal
culture results on a BAL specimen does not exclude disease. Definitive diagnosis of invasive
aspergillosis is made via isolation of the mold in tissue biopsy specimens (eg, skin, lung, sinus).
Chest radiography may show multiple, ill-defined, peripheral nodules that may coalesce to larger
masses or lobar or diffuse consolidative lesions. High-resolution computed tomographic scan is
the imaging modality of choice for evaluation of invasive pulmonary aspergillosis and may
demonstrate multiple pulmonary nodules, early halo sign (ie, rim of ground-glass opacity
surrounding the nodule seen in neutropenic hosts reflecting hemorrhagic nodule from
angioinvasion), followed later by cavitation or the “air crescent” sign after resolution of
neutropenia. These radiographic findings are nonspecific and may be seen in other invasive mold
infections such as mucormycosis. Magnetic resonance imaging is the recommended imaging
standard for central nervous system disease.

Voriconazole is the drug of choice for primary treatment of invasive Aspergillus infection.
Measurement of voriconazole serum trough concentration is recommended to monitor safety and
efficacy, given the high variation in drug metabolism among patients. Alternative agents to treat
invasive Aspergillus infection include lipid formulations of amphotericin B. Combination
therapy with voriconazole in conjunction with an echinocandin is a consideration in selected
patients. Posaconazole may be indicated in refractory disease. Other key strategies for
management include reduction of immunosuppression if feasible.
American Academy of Pediatrics 212
PREP ® Self-Assessment PREPSA 2021
The differential diagnosis of bilateral pulmonary disease in immunocompromised patients is
broad and must include viral, bacterial, and atypical pathogens (eg, Mycoplasma) and other
fungal pathogens, such as Pneumocystis jirovecii. The diagnosis of Pneumocystis
jirovecii pneumonia is less likely in the case described in the vignette owing to absence of
significant hypoxemia, respiratory distress, and a characteristic ground-glass appearance of the
chest imaging. The diagnosis of mycoplasma pneumonia is less likely given the clinical
presentation, presence of bilateral pulmonary nodules, and absence of pleural effusion on
computed tomographic chest imaging. Development of severe staphylococcal pneumonia would
be unusual in light of the patient’s negative blood cultures and progression of pulmonary disease
while receiving vancomycin therapy.

PREP Pearls
• Invasive aspergillosis may complicate the clinical course of immunocompromised
patients.
• Pulmonary infection is the most common manifestation of invasive aspergillosis, but
other sites, including the sinus, skin, and brain, may be involved.
• Voriconazole is the preferred drug of choice for primary treatment of
invasive Aspergillus infection.

ABP Content Specifications(s)


• Understand the epidemiology of Aspergillus
• Recognize the clinical features associated with Aspergillus infection

Suggested Readings
• Alexander K, Cadilla A, Qureshi N. Fungal infections (systemic). In: McInerny TK,
Adam HM, Campbell DE, DeWitt TG, Foy JM, Kamat DM, eds. American Academy of
Pediatrics Textbook of Pediatric Care. 2nd ed. Elk Grove Village, IL: American
Academy of Pediatrics; 2017:2036-2062. Pediatric Care Online .
• American Academy of Pediatrics. Aspergillosis. In: Kimberlin DW, Brady MT, Jackson
MA, Long SS, eds. Red Book: 2018-2021 Report of the Committee on Infectious
Diseases. 31st ed. Elk Grove Village, IL: American Academy of Pediatrics; 2018:230-
234. Red Book Online.
• Choi SH, Lee SY, Hwang JY, et al. Importance of voriconazole therapeutic drug
monitoring in pediatric cancer patients with invasive aspergillosis. Pediatr Blood
Cancer. 2013;60(1):82-87. doi:10.1002/pbc.24262.
• Patterson TF, Thompson GR III, Denning DW, et al. Practice guidelines for the diagnosis
and management of aspergillosis: 2016 update by the Infectious Disease Society of
America. Clin Infect Dis. 2016;63(4):e1-e60. doi:10.1093/cid/ciw326.

American Academy of Pediatrics 213


PREP ® Self-Assessment PREPSA 2021
Question 56
An 8-year-old boy is seen for a health supervision visit. He is in the third grade in a special
education classroom. His parents report that he walked at 2 years, started putting 2 words
together at 3 years, and has been receiving physical, occupational, and speech therapy for his
entire life. His birth and postnatal history are significant for hypotonia and failure to thrive, and
he required a gastrointestinal tube for feeding. His parents are currently concerned about his
excessive eating behavior. His weight and height are at the 98th and 2nd percentile, respectively.
He has almond-shaped eyes, a thin upper lip, small hands and feet, and a small penis with a
hypoplastic scrotum. The remainder of his physical examination findings are normal.

Of the following, the MOST appropriate therapy for this condition is


A. glucocorticoid therapy in the first year after birth with nutrition management
B. growth hormone therapy in the first year after birth with nutrition management
C. growth hormone therapy in the first year after birth with supplementation of carnitine
D. levothyroxine therapy in the first year after birth with nutrition management

American Academy of Pediatrics 214


PREP ® Self-Assessment PREPSA 2021
Correct Answer: B
The boy in the vignette has Prader-Willi syndrome (PWS). Classic presentation of PWS in a
neonate involves hypotonia with failure to thrive owing to poor sucking and swallowing.
Affected neonates almost universally require assisted feeding with special nipples, nasogastric
tubes, or gastrostomy tubes to ensure adequate nutrition. Later, when the child is aged 4 to 6
years, hypothalamic hyperphagia occurs, primarily owing to decreased satiety, which can lead to
significant obesity if food intake is not strictly controlled. Motor delay occurs, with achievement
of motor milestones occurring at about twice the typical age (eg, walking at about 24 months).
Language delay and mild to moderate cognitive impairment are characteristic. Common
behavioral issues include obsessive-compulsive behaviors, anxiety, stubbornness, temper
tantrums, and skin picking. Facial features include almond-shaped eyes, a thin upper lip, and
small hands and feet. Hypogonadism is a prominent feature in both males and females. In male
infants, the most common manifestation is cryptorchidism, which occurs in nearly all males with
PWS. Endocrine abnormalities result from hypothalamic-pituitary dysfunction and include
growth hormone (GH) deficiency, which is considered universal, and variable central adrenal
insufficiency and central hypothyroidism. Other features include central and obstructive sleep
apnea, temperature instability, strabismus, and scoliosis. Many of these features are thought to be
related to hypothalamic dysfunction.

Diagnosis is made by PWS methylation analysis. Prader-Willi syndrome is an imprinting


disorder caused by an absence of expression of paternally derived genes in the PWS/Angelman
syndrome region of chromosome 15q11.2-q13. This can occur via one of three mechanisms:
paternal deletion of the 15q11.2-q13 region (accounting for the majority [about 70%] of cases),
maternal uniparental disomy of chromosome 15 (accounting for about 25%-30% of cases of
PWS), or an imprinting defect (rare; the imprinting center controls the expression of genes in this
region). Recurrence risk, the risk that parents will have another child with PWS, is very low for
the first two mechanisms at <1% (the same as in the general population), whereas it is 50% if the
underlying etiology is an imprinting center defect.

With increased knowledge and the advent of molecular testing methodologies, the diagnosis is
now frequently made within the first few months after birth. Early diagnosis, allowing for earlier
access to team-based multidisciplinary management, has improved long-term health outcomes.
Treatment in the first year after birth includes GH therapy with nutrition management. Also
important in the first year of life is respiratory support, early-intervention developmental
services, and appropriate anticipatory guidance. The recommendation is that the clinician discuss
GH therapy at the time of diagnosis. Early intervention with GH therapy has been shown to
improve body composition and increase lean body mass. In addition, GH therapy initiated within
the first year after birth has been shown to have a positive impact on respiratory drive, central
apnea, verbal intelligence quotient, language, and cognitive skills.

Nutrition management is required for adequate supplementation in infancy. After a period of


feeding difficulties, early establishment of food rules and a meal plan with strictly controlled
food intake is effective in preventing obesity. Implementing principles of food security, both
physical (preventing unauthorized access to food, locking up food) and psychological
(establishing food routines, assuring the person with PWS that the next meal is coming), is of
paramount importance in the management of PWS in childhood and adulthood. Other important
American Academy of Pediatrics 215
PREP ® Self-Assessment PREPSA 2021
management modalities include regular exercise, as well as strategies to help with behavioral
issues throughout the lifespan.

Central adrenal insufficiency can be seen in patients with PWS and requires glucocorticoid
therapy, especially during times of stress. However, glucocorticoid therapy without GH therapy
is not the standard of care for the management of PWS. Carnitine supplementation can be
provided if secondary deficiency is present because of inadequate intake. However, carnitine
supplementation for all patients with PWS is not recommended if dietary needs are met
adequately. Hypothyroidism has been documented in patients with PWS. Treatment with
levothyroxine should not be routinely prescribed to patients with PWS who do not have
abnormal thyroid function test findings and would not replace GH therapy.

PREP Pearls
• Prader-Willi syndrome is characterized by neonatal hypotonia with failure to thrive. This
is replaced by hyperphagia and obesity in early childhood.
• Early diagnosis of Prader-Willi syndrome is important; growth hormone therapy in
conjunction with nutrition management should begin in the first year after birth, along
with appropriate anticipatory guidance.

ABP Content Specifications(s)


• Understand the role of imprinting in genetic disorders

Suggested Readings
• Butler MG, Manzardo AM, Forster JL. Prader-Willi syndrome: clinical genetics and
diagnostic aspects with treatment approaches. Curr Pediatr Rev. 2016;12(2):136-166.
• Dolan E, Wiley S. Prader-Willi syndrome. In: McInerny TK, Adam HM, Campbell DE,
DeWitt TG, Foy JM, Kamat DM, eds. AAP Textbook of Pediatric Care. 2nd ed. Elk
Grove Village, IL: American Academy of Pediatrics:2531-2536. Pediatric Care Online.
• National Library of Medicine. Genetics Home Reference: Prader-Willi
syndrome. https://ghr.nlm.nih.gov/condition/prader-willi-syndrome#diagnosis.

American Academy of Pediatrics 216


PREP ® Self-Assessment PREPSA 2021
Question 57
A 15-year-old adolescent is brought to the emergency department by his friends who became
concerned when he complained of nausea, dizziness, and weakness. He had a hard time
describing how he was feeling to his friends except to say that he was hearing colors and that
time seemed to have slowed down. In the emergency department, the adolescent is euphoric. His
vital sign assessment is significant for tachycardia and hypertension.

Of the following, the MOST appropriate statement regarding this boy’s condition is that he
A. is at risk for serotonin toxicity
B. is likely to experience hepatic injury
C. should be treated with naloxone
D. will experience physical dependence

American Academy of Pediatrics 217


PREP ® Self-Assessment PREPSA 2021
Correct Answer: A
This adolescent’s symptoms are consistent with acute intoxication with a hallucinogen, which
causes euphoria, tachycardia, hypertension, nausea, dizziness, weakness, vomiting, and diarrhea.
Synesthesia, a state of altered perception where one sense is perceived as another (eg, hearing
colors), is the most distinctive symptom indicating hallucinogen use. Hallucinogens act at
serotonin receptors and there is a risk for serotonin toxicity with their use.

Hallucinogens are drugs that alter sensory perception, thoughts, and mood. They include lysergic
acid diethylamide (LSD), mescaline, psilocybin, N,N-dimethyltryptamine (DMT), lysergic acid
amide (LSA), Salvia divinorum, and recreational nutmeg. Neurologic effects include weakness,
dizziness, drowsiness, paresthesias, and nystagmus. Neuropsychiatric effects include a distorted
sense of time and altered sensory perception. Colors or shapes may appear different. Visual and
auditory hallucinations may be present. Mood enhancements range from fear to euphoria.
Patients may become agitated and react unpredictably. Psychosis can occur. Sympathomimetic
effects from hallucinogens are generally mild and include tachycardia, hypertension, diaphoresis,
mydriasis, and hyperthermia.

Signs and symptoms of acute hallucinogen intoxication are self-limited. Treatment is primarily
supportive; morbidity primarily includes the person’s behaviors while in the altered state (eg,
accidents). Patients should be around familiar people and placed in a calm and unstimulating
environment. Serotonergic agents should be avoided. Benzodiazepines may be used for
significant agitation and dysphoria.

Hallucinogens do not cause liver damage. Naloxone is not useful to treat the signs or symptoms
of hallucinogen intoxication. Hallucinogens are not physiologically addictive and do not cause
withdrawal or physical dependence.

PREP Pearls
• Synesthesia, a state of altered perception where one sense is perceived as another (eg,
hearing colors), is the most distinctive symptom indicating hallucinogen use.
• Hallucinogens act at serotonin receptors and their use carries a risk for serotonin toxicity.
• Treatment of acute hallucinogen intoxication is primarily supportive, and morbidity from
hallucinogen use is primarily from the person’s behaviors while in the altered state (eg,
accidents).

ABP Content Specifications(s)


• Recognize the major behavioral consequences of hallucinogen use/abuse
• Identify the major physiologic consequences associated with hallucinogen use/abuse,
including those associated with the various means of administration
• Recognize the clinical findings associated with an acute hallucinogen intoxication, and
manage appropriately

American Academy of Pediatrics 218


PREP ® Self-Assessment PREPSA 2021
Suggested Readings
• Eldridge DL, Hillenbrand K. Hallucinogens. Pediatr Rev. 2006;27(8):314-315.
doi: 10.1542/pir.27-8-314.
• Fine JS. Poisoning. In: McInerny TK, Adam HM, Campbell DE, DeWitt TG, Foy JM,
Kamat DM, eds. American Academy of Pediatrics Textbook of Pediatric Care. 2nd ed.
Itasca, IL: American Academy of Pediatrics; 2016;chap 369:2924-2929. Pediatric Care
Online.
• Wang GS, Hoyte C. Common substances of abuse. Pediatr Rev. 2008;39(8):403-412.
doi: 10.1542/pir.2017-0267.
• Wang GS, Hoyte C. Novel drugs of abuse. Pediatr Rev. 2019;40(2):71-78.
doi: 10.1542/pir.2018-0050.

American Academy of Pediatrics 219


PREP ® Self-Assessment PREPSA 2021
Question 58
A 1-month-old infant is being seen for a health supervision visit. She is growing well and
developing normally. She was born at term without any complications, and her mother had
appropriate prenatal care. She received erythromycin ophthalmic ointment before discharge from
the newborn nursery. Her mother often notices mild clear drainage from the girl’s left eye that is
easily wiped away with a warm damp cloth. The girl has not had any fevers or other systemic
symptoms. The eye does not seem painful and is not injected. There is no lid swelling,
tenderness, or erythema. The remainder of her examination findings are normal.

Of the following, the MOST likely underlying cause of this infant’s eye drainage is
A. allergic reaction
B. bacterial infection
C. lacrimal duct obstruction
D. viral infection

American Academy of Pediatrics 220


PREP ® Self-Assessment PREPSA 2021

Correct Answer: C
The most likely cause of the eye drainage for the infant in the vignette is nasolacrimal duct
obstruction, which is present in up to 5% of infants. The symptom of eye drainage without
inflammation or pain occurs because of incomplete opening of the valve of Hasner, the valve
between the nasolacrimal duct and inferior nasal meatus.

Providers often encourage applying gentle pressure to the lacrimal sac (at the inner corner of the
eye) to increase the pressure through the nasolacrimal duct and overcome the obstruction. Gentle
pressure is thought to be sufficient, without downward massage. Using a clean washcloth to
avoid the introduction of infection is important. In some cases, when the obstruction does not
resolve by 12 months of age, an ophthalmologist may probe the duct to open the obstruction.

A bacterial or viral infection is not likely for the infant in this vignette, given the lack of injection
or irritation. In addition, the infant has no risk factors for a chlamydia or gonorrhea ocular
infection; the mother had negative prenatal testing results and the infant received ophthalmic
ointment at delivery. Irritation or photophobia is not consistent with nasolacrimal duct
obstruction and, if present, should prompt further evaluation, including for glaucoma. Allergies
are not likely in this age group and would also likely cause irritation, along with other systemic
symptoms of allergies such as rhinorrhea.

PREP Pearls
• Nasolacrimal duct stenosis is common, seen in up to 5% of infants.
• Photophobia or eye irritation can be the result of glaucoma.
• Injected sclera can occur because of an infection.

ABP Content Specifications(s)


• Recognize the clinical findings associated with obstruction of the nasolacrimal duct
• Plan the appropriate management of obstruction of the nasolacrimal duct

Suggested Readings
• Centers for Disease Control and Prevention. Conjunctivitis (pink eye) in
newborns. https://www.cdc.gov/conjunctivitis/newborns.html.
• Lueder GT. Nasolacrimal duct obstruction in children.
2015. https://www.aao.org/disease-review/nasolacrimal-duct-obstruction-4.

American Academy of Pediatrics 221


PREP ® Self-Assessment PREPSA 2021
Question 59
A 3-year-old boy with fatigue and bruising is brought to the emergency department. His parents
state that he was well until about 2 days ago, when his energy level decreased. The boy had been
previously well, with normal growth and development.

He appears pale and quietly clings to his mother. He is responsive but sleepy. He has a
temperature of 38.6°C, a heart rate of 136 beats/min, a blood pressure of 78/56 mm Hg, a
respiratory rate of 32 breaths/min, and an oxygen saturation of 94% on room air. The heart
rhythm is normal but rapid, and the lungs are clear to auscultation. The spleen and liver are both
markedly enlarged. There are petechiae and purpura over the chest, back, arms, and legs.

Laboratory data are shown:


Laboratory Test Result
White blood cell count 91,900/µL (91.9 × 109/L)
Hemoglobin 4.2 g/dL (42 g/L)
Platelet count 46,000/µL (46 × 109/L)

Of the following, the pattern of results MOST likely to be seen on further laboratory evaluation
is

A.
Phosphorus Level Potassium Level Uric Acid Level

High High High

B.
Phosphorus Level Potassium Level Uric Acid Level

Low High High

C.
Phosphorus Level Potassium Level Uric Acid Level

High Low High

D.
Phosphorus Level Potassium Level Uric Acid Level

High High Low

American Academy of Pediatrics 222


PREP ® Self-Assessment PREPSA 2021
Correct Answer: A
The boy in the vignette has many stigmata raising concern about acute leukemia, including an
elevated white blood cell count, severe anemia, thrombocytopenia, an enlarged spleen and liver,
petechiae and purpura, and fatigue. Acute lymphoblastic leukemia is far more common in
children than is acute myeloblastic leukemia, and children with acute lymphoblastic leukemia
and an elevated white blood cell count are at risk of experiencing tumor lysis syndrome. In this
syndrome, tumor cells multiply so quickly that they outstrip their nutrient supply and experience
premature death and lysis. As the tumor cells lyse, they release their intracellular contents into
the serum, leading to alterations in serum electrolytes. Because there are high intracellular
concentrations of potassium and phosphorus relative to the extracellular compartment, tumor
lysis will result in elevated levels of serum potassium and phosphorus. As the phosphorus
concentration rises, it will bind with serum calcium, resulting in a commensurate decrease in the
calcium concentration. Therefore, tumor lysis is associated with hyperkalemia,
hyperphosphatemia, and hypocalcemia. In addition, the free DNA released from the tumor cells
will be metabolized to uric acid, resulting in hyperuricemia.

There are many possible clinical consequences of tumor lysis syndrome, including cardiac
dysrhythmias due to hyperkalemia and renal dysfunction due to precipitation of uric acid or
calcium phosphate crystals in the nephrons. Hyperkalemia can be managed with administration
of intravenous calcium, insulin, and inhaled albuterol; however, caution should be used with the
administration of calcium, because it may increase the risk of renal dysfunction due to the
precipitation of calcium phosphate. Ultimately, if these management strategies are inadequate,
dialysis may be needed. Whereas renal dysfunction due to hyperuricemia can be prevented
through the use of allopurinol or rasburicase, renal dysfunction due to hyperphosphatemia can be
effectively countered only through removal of the phosphorus via dialysis. Management of the
electrolyte abnormalities associated with tumor lysis is temporizing, and prevention of organ
dysfunction depends on definitive treatment of the underlying leukemia.

PREP Pearls
• Tumor lysis is associated with hyperkalemia, hyperphosphatemia, and hypocalcemia; free
DNA released from tumor cells is metabolized to uric acid, resulting in hyperuricemia.
• Clinical consequences of tumor lysis syndrome may include cardiac dysrhythmias due to
hyperkalemia and renal dysfunction due to precipitation of uric acid or calcium phosphate
crystals in the nephrons.
• Management of electrolyte abnormalities associated with tumor lysis is temporizing;
prevention of organ dysfunction requires definitive treatment of the underlying leukemia.

ABP Content Specifications(s)


• Plan an appropriate diagnostic evaluation to exclude tumor lysis syndrome in patients
suspected of having leukemia

American Academy of Pediatrics 223


PREP ® Self-Assessment PREPSA 2021

Suggested Readings
• Coiffier B, Altman A, Pui CH, Younes A, Cairo MS. Guidelines for the management of
pediatric and adult tumor lysis syndrome: an evidence-based review. J Clin
Oncol. 2008;26(16):2767-2778. doi:10.1200/JCO.2007.15.0177.
• Howard SC, Jones DP, Pui CH. The tumor lysis syndrome (published correction appears
in N Engl J Med. 2018;379[11]:1094). N Engl J Med. 2011;364(19):1844-1854.
doi:10.1056/NEJMra0904569.
• Polishchuk V, Roth M, Kolb EA. Leukemias. In: McInerny TK, Adam HM, Campbell
DE, DeWitt TG, Foy JM, Kamat DM, eds. American Academy of Pediatrics Textbook of
Pediatric Care. 2nd ed. Elk Grove Village, IL: American Academy of Pediatrics;
2017:2253-2268. Pediatric Care Online .
• Stephanos K, Picard L. Pediatric oncologic emergencies. Emerg Med Clin North Am.
2018 Aug;36(3):527-535. doi:10.1016/j.emc.2018.04.007.

American Academy of Pediatrics 224


PREP ® Self-Assessment PREPSA 2021

Question 60
A 9-month-old infant is seen in the clinic for urgent evaluation. Her parents report that she is
sleepy, weak, and listless. She developed a runny nose and increased drooling yesterday. Today,
she has had 5 loose stools and decreased oral intake. She has had a normal number of wet
diapers. She lives with her parents on a farm. She has no known sick contacts and does not attend
daycare. She has a temperature of 37.5°C, heart rate of 80 beats/min, respiratory rate of 50
breaths/min, and blood pressure of 90/50 mm Hg. She has decreased tone and is sleepy, but
responds appropriately to painful stimuli. Her pupils are constricted (1-2 mm diameter), and she
is tearing and drooling. Her lungs are clear to auscultation. Intermittent intercostal retractions are
noted. The heart has a regular rate and rhythm, and her abdomen is soft and nontender without
distention or hepatosplenomegaly. Her skin is dry, and she has a capillary refill time of 3
seconds.

Of the following, the MOST likely source of the toxin causing her symptoms is
A. chicken droppings
B. goat milk
C. parents’ clothing
D. unpasteurized honey

American Academy of Pediatrics 225


PREP ® Self-Assessment PREPSA 2021
Correct Answer: C
The infant in the vignette has symptoms of cholinergic excess consistent with organophosphate
poisoning. The most likely source of her exposure is an agricultural insecticide. Children can be
exposed to insecticides through accidental ingestion, inhalation of insecticide sprays, ingestion of
contaminated food, or transmission on clothing or fabric. Organophosphates can contaminate
clothing even after they are washed. In this case, her farmworker parents’ clothing is the most
likely source.

Organophosphates have been used as insecticides for decades. They are well absorbed through
the skin, gut, and respiratory tract and bind to acetylcholinesterase, leading to an acetylcholine
excess at the neuronal synapse and neuromuscular junction. Symptoms of acute cholinergic
toxicity include:
• Bradycardia
• Miosis
• Lacrimation
• Salivation
• Bronchospasm and bronchorrhea
• Urination
• Emesis and diarrhea

Goat milk, unpasteurized honey, or chicken droppings are not a likely source of organophosphate
exposure. Infants who are fed goat milk instead of breast milk or formula can develop electrolyte
imbalances and folate deficiency, and if the milk is unpasteurized, are at risk for infections such
as brucellosis. Unpasteurized honey is a potential source of Clostridium botulinum spores that
can cause botulism (symptoms include constipation, weakness, feeding difficulties, and
descending hypotonia). Chicken droppings can transmit bacteria including Campylobacter,
Escherichia coli, and Salmonella.

PREP Pearls
• Exposure to pesticides that contain organophosphates can cause symptoms of cholinergic
excess, including bradycardia, miosis, lacrimation, salivation, bronchospasm and
bronchorrhea, urination, emesis, and diarrhea.
• Organophosphates are well absorbed through the skin, gut, and respiratory tract. Children
can be exposed to organophosphate insecticides through accidental ingestion, inhalation
of insecticide sprays, ingestion of contaminated food, or transmission on clothing or
fabric.

ABP Content Specifications(s)


• Know the potential exposures that result from a parent’s occupation that directly or
indirectly affect the health of their children

American Academy of Pediatrics 226


PREP ® Self-Assessment PREPSA 2021
Suggested Readings
• Council on Environmental Health. Pesticide exposure in children. Pediatrics.
2012;130(6):e1757-e1763. doi: 10.1542/peds.2012-2757.
• Fine JS. Poisoning. In: McInerny TK, Adam HM, Campbell DE, DeWitt TG, Foy JM,
Kamat DM. American Academy of Pediatrics Textbook of Pediatric Care. 2nd ed. Elk
Grove Village, IL: American Academy of Pediatrics; 2017:2924-2949. Pediatric Care
Online.
• Galvez MP, Balk SJ. Environmental risks to children: prioritizing health messages in
pediatric practice. Pediatr Rev. 2017;38(6):263-279. doi: 10.1542/pir.2015-0165.

American Academy of Pediatrics 227


PREP ® Self-Assessment PREPSA 2021
Question 61
A 3-year-old girl is brought to the clinic for concerns of rectal bleeding. Her symptoms began
about 6 months ago. Her parents had noticed bright red blood intermittently on the toilet paper
and in the toilet; however, over the last few weeks, the bleeding has intensified. Now the girl has
blood with each bowel movement. She has no history of constipation and has a soft bowel
movement 1 to 2 times every day. She is toilet trained and does not seem to experience painful
bowel movements. She is otherwise healthy, with excellent growth and development. Her
physical examination findings demonstrate normal vital signs; the abdomen is soft and
nontender, without masses or hepatosplenomegaly; digital rectal examination findings are
normal and there is no gross blood present. Meckel scan result is negative.

Laboratory data include the following:


Hemoccult Positive
White blood cell 9,000/µL (9.0 × 109/L)
Hemoglobin 11.8 g/dL (118 g/L)
Hematocrit 32.8%
Platelet 310 × 103/µL (310 × 109/L)
Erythrocyte sedimentation rate 3 mm/h

Of the following, the MOST likely cause of this girl’s findings is


A. cow milk protein allergy
B. infectious colitis
C. juvenile polyp
D. very early-onset inflammatory bowel disease

American Academy of Pediatrics 228


PREP ® Self-Assessment PREPSA 2021
Correct Answer: C
The girl in the vignette has a juvenile polyp, which has resulted in intermittent rectal bleeding.
The polyp was identified during colonoscopy and removed during the procedure (Item C61).
Juvenile polyp(s) should be suspected in a young child (typically ages 1-7 years) with
intermittent and painless rectal bleeding. Most juvenile polyps are found in the distal colon
(sigmoid and rectum); once resected, follow-up colonoscopy is not indicated. However, if more
than 5 polyps are detected, or if the location of the polyps is in the proximal colon, juvenile
polyposis syndrome should be suspected. Evaluation should include genetics and oncology
consultation, and repeat colonoscopy every 1 to 3 years.

Item C61: Juvenile Polyp. Courtesy of J Sullivan

Children with gastrointestinal (GI) bleeding may exhibit various symptoms. Upper GI tract
bleeding can manifest as hematemesis (bright red blood or coffee-ground appearance) or melena
(dark purple/black appearing stools). Rarely, hematochezia (bright red blood in stool) can be
caused by brisk upper GI tract bleeding. Lower GI tract bleeding may present as melena or
hematochezia. History and physical examination should be performed in all children with GI
bleeding to determine the severity of the bleeding. Children with evidence of hemodynamic
instability (tachycardia, orthostasis/hypotension) should be emergently treated in a hospital

American Academy of Pediatrics 229


PREP ® Self-Assessment PREPSA 2021
setting (emergency department, intensive care unit), and adequate intravenous access obtained.
Fluids and packed red blood cells may be necessary in severe presentations.

Causes of rectal bleeding vary based on age and severity of symptoms. Neonates may exhibit
significant rectal bleeding because of coagulopathies, necrotizing enterocolitis, Hirschsprung
disease, midgut volvulus, or vascular malformations. Neonates with mild symptoms may have
hematochezia because of swallowed maternal milk, milk protein allergy, or an anal fissure. In
infants and toddlers, causes of severe rectal bleeding include Meckel diverticulum,
intussusception, midgut volvulus, Henoch-Schönlein purpura, and vascular malformations.
Infants and toddlers with mild symptoms may have milk protein allergy or anal fissure. Older
children can have severe bleeding with Meckel diverticulum, inflammatory bowel disease,
vascular malformations, or Henoch-Schönlein purpura. Older children with milder symptoms
may have infectious colitis, polyp (juvenile or other type), or inflammatory bowel disease.
Causes of painless rectal bleeding include polyps, Meckel diverticulum, and vascular
malformations.

Cow milk protein allergy is most often seen in the first few months after birth and generally
resolves by age 1 year; thus it is not likely in this 3-year-old child described in the vignette.
Infectious colitis would be common in this age group, but it would be unlikely without diarrhea
and colicky abdominal pain. In addition, infectious colitis is commonly self-limiting, so it would
not likely cause hematochezia for 6 months. Very early-onset inflammatory bowel disease could
occur at this age but would likely be associated with signs of colitis (diarrhea, colicky abdominal
pain), perianal disease (fistulae, deep fissures), and/or poor weight gain.

PREP Pearls
• Gastrointestinal bleeding may present as hematemesis, melena, and/or hematochezia.
• Hemodynamic status should be quickly assessed in children with gastrointestinal
bleeding.
• Causes of painless rectal bleeding include polyps, Meckel diverticulum, and vascular
malformations.

ABP Content Specifications(s)


• Recognize the clinical features associated with Meckel diverticulum, and manage
appropriately
• Plan the appropriate evaluation of rectal bleeding
• Distinguish among the etiologies of occult blood and bright red blood per rectum
• Formulate an age-appropriate differential diagnosis for rectal bleeding

American Academy of Pediatrics 230


PREP ® Self-Assessment PREPSA 2021
Suggested Readings
• Avner JR. Gastrointestinal hemorrhage. In: McInerny TK, Adam HM, Campbell DE,
DeWitt TG, Foy JM, Kamat DM, eds. American Academy of Pediatrics Textbook of
Pediatric Care. 2nd ed. Itasca, IL: American Academy of Pediatrics; 2016;chap
155:1379-1385. Pediatric Care Online.
• Neidich GA, Cole SR. Gastrointestinal bleeding. Pediatr Rev. 2014;35(6):243-254.
doi: 10.1542/pir.35-6-243.
• Padilla BE, Moses W. Lower gastrointestinal bleeding and intussusception. Surg Clin
North Am. 2017;97:173-188. doi: 10.1016/j.suc.2016.08.015.

American Academy of Pediatrics 231


PREP ® Self-Assessment PREPSA 2021
Question 62
A 24-hour-old term neonate is being evaluated. He was born at 38 weeks’ gestation by cesarean
delivery after failed induction for maternal preeclampsia; his birthweight was 3.6 kg. Since birth,
he has breastfed 8 times. His nurse reports a good latch. He has had 1 meconium stool and 2 wet
diapers. His weight is 3.2 kg and his physical examination findings are unremarkable.

Of the following, the clinical information that SUPPORTS the decision not to supplement with
formula is
A. the neonate’s sex
B. gestational age
C. history of maternal preeclampsia
D. mode of delivery

American Academy of Pediatrics 232


PREP ® Self-Assessment PREPSA 2021
Correct Answer: D
The mode of delivery may be taken into consideration when deciding whether to recommend
formula supplementation for a breastfed term neonate in the first few days after birth. In the first
week after birth, most full-term neonates lose up to 10% of their birth weight. In the first 24 to
48 hours after birth, as adequate renal function is established, there is a natural diuresis, likely
from the extracellular space. This is followed by an expected weight gain of 15 g/day with
adequate oral intake.

Other factors may contribute to this initial weight loss. A recent study of term neonates
demonstrated that at 48 hours after birth, more neonates born via cesarean section had a loss of
greater than 10% of birthweight compared with those delivered vaginally. The higher weight loss
among neonates delivered via cesarean section may be the result of maternal intravenous fluid
administration or delayed lactogenesis.

In an exclusively breastfed neonate, weight loss of greater than 10% in the first 24 to 48 hours
after birth should alert clinicians to the possibility of problems with breastfeeding. A thorough
assessment of adequate breastfeeding must include assessment of latch and transfer as well as the
number of diapers with urine and meconium.

For neonates born before 37 weeks, gestational age is associated with higher weight loss.
However, for this neonate born at 38 weeks’ gestation, gestational age does not predict weight
loss. There are no data to suggest that sex or maternal history of preeclampsia has an impact on
weight loss after birth.

PREP Pearls
• For full-term neonates, loss of 7% to 10% of birthweight is acceptable and not
unexpected over the first week after birth.
• Neonates born via cesarean section may lose more weight in the first 2 days after birth
than those delivered vaginally.
• In an exclusively breastfed neonate, weight loss of greater than 10% in the first 24 to 48
hours after birth should alert clinicians to the possibility of problems with breastfeeding.

ABP Content Specifications(s)


• Know the normal pattern of feeding and stool frequency in breast-fed infants

Suggested Readings
• Feld LG, Neuspiel DR, Foster BA, et al; Subcommittee on Fluid and Electrolyte Therapy.
Clinical practice guideline: maintenance intravenous fluids in children. Pediatrics.
2018;142(6):e20183083. doi: 10.1542/peds.2018-3083.
• Flaherman VJ, Schaefer EW, Kuzniewicz MW, Li SX, Walsh EM, Paul IM. Early weight
loss nomograms for exclusively breastfed newborns. Pediatrics. 2015;135;e16-e23.
doi: 10.1542/peds.2014-1532.
• Parks EP, Shaikhkhalil A, Sainath NN, Mitchell JA, Brownell JN, Stallings VA. Feeding
healthy infants, children and adolescents. In: Kliegman R, St Geme JW, Blum NJ, Shah
SS, Tasker RC, Wilson Karen, eds. Nelson Textbook of Pediatrics. Philadelphia, PA:
Elsevier; 2020:321-331.e1.
American Academy of Pediatrics 233
PREP ® Self-Assessment PREPSA 2021
Question 63
A 5-week-old male infant is brought to the emergency department after 2 days of vomiting. He is
not tolerating 2-oz feedings. His vomit is projectile and yellow. He has not had diarrhea,
constipation, or abdominal distension. He is alert and active. He has a heart rate of 136
beats/min, a respiratory rate of 20 breaths/min, and a blood pressure of 86/52 mm Hg. His
mucous membranes are dry, and his capillary refill time is less than 2 seconds. His abdomen is
not distended. The liver edge is palpable 2 cm below the costal margin, and bowel sounds are
present. Abdominal ultrasonography shows thickening of the pylorus muscle with no other
abnormality.

Of the following, the MOST likely laboratory finding in this infant is


A. high serum chloride level and metabolic acidosis
B. high urine chloride level and metabolic alkalosis
C. low serum chloride level and metabolic alkalosis
D. low urine chloride level and metabolic acidosis

American Academy of Pediatrics 234


PREP ® Self-Assessment PREPSA 2021
Correct Answer: C
The infant in this vignette has projectile vomiting, clinical features of dehydration, and
thickening of the pylorus muscle on ultrasonography, all of which favor a diagnosis of
hypertrophic pyloric stenosis. The excessive vomiting leads to loss of large amounts of gastric
hydrochloric acid (hydrogen and chloride ions). Of the options shown, the laboratory evaluation
most likely to be found is a low serum chloride level and metabolic alkalosis.

Metabolic alkalosis is defined as an increase in serum bicarbonate content, resulting in an


increase in serum pH. Metabolic alkalosis is due to either loss of chloride or a net gain of cations
or bicarbonate. Gastrointestinal losses of chloride occur through emesis (pyloric stenosis) or
diarrhea (chloride losing diarrhea). Renal losses of chloride occur with diuretic use (contraction
alkalosis) or with Bartter syndrome or Gitelman syndrome. The chloride loss occurs through skin
in children with cystic fibrosis. In these scenarios involving loss of chloride, compensatory
increase in bicarbonate buffer occurs to maintain electroneutrality, leading to metabolic
alkalosis. Other causes of metabolic alkalosis are net gain of cations (as occurs with infusion of
large amounts of lactated ringer solution and milk alkali syndrome) or bicarbonate infusions.
The cause of metabolic alkalosis is usually evident in the history. In cases in which the cause is
not apparent, urine chloride is a helpful test to distinguish renal from extra-renal chloride losses
(Item C63). Pyloric stenosis, chloride losing diarrhea, and cystic fibrosis cause extra-renal loss
of chloride, resulting in a low urine chloride level and metabolic alkalosis. However, diuretic use
and Bartter syndrome involve renal loss of chloride and thus lead to high levels of urine chloride
along with metabolic alkalosis.

American Academy of Pediatrics 235


PREP ® Self-Assessment PREPSA 2021
In normal anion gap acidosis (renal tubular acidosis, diarrhea), the loss of bicarbonate is buffered
by an increase in chloride to maintain electroneutrality. Thus, high serum chloride
(hyperchloremia) is seen in various causes of normal anion gap acidosis. In acidosis secondary to
intravascular depletion (dehydration, burns), renal mechanisms try to conserve sodium chloride
and water, resulting in a low urine chloride level along with metabolic acidosis.

PREP Pearls
• Low serum chloride levels and metabolic alkalosis are seen in hypertrophic pyloric
stenosis.
• High serum chloride levels and metabolic acidosis are seen in diarrhea and renal tubular
acidosis.
• High urine chloride levels and metabolic alkalosis are seen in diuretic use, Bartter
syndrome, and Gitelman syndrome.

ABP Content Specifications(s)


• Recognize the various etiologies of hypochloremia
• Recognize the association of chloride and acidosis in the differential diagnosis of
metabolic acidosis

Suggested Readings
• Carmody JB. Urine electrolytes. Pediatr Rev. 2011;32(2):65-68. doi:10.1542/pir.32-2-65.
• Hsu BS, Lakhani SA, Wilhelm M. Acid-base disorders. Pediatr Rev. 2016;37(9):361-
369. doi.10.1542/pir.2015-0093.
• Mahajan P, Felt JR. Fluids, electrolytes, and acid-base composition. In: McInerny TK,
Adam HM, Campbell DE, DeWitt TG, Foy JM, Kamat DM, eds. American Academy of
Pediatrics Textbook of Pediatric Care. 2nd ed. Elk Grove Village, IL: American
Academy of Pediatrics; 2017:419-432. Pediatric Care Online .

American Academy of Pediatrics 236


PREP ® Self-Assessment PREPSA 2021
Question 64
A fully immunized 5-year-old girl is being evaluated for a puncture wound to the sole of her
right foot. The child was walking with her parents when she suddenly fell to the ground crying.
When her father picked her up, he noticed a solitary nail protruding from the sole of her sneaker.
He removed the nail and brought the child to the emergency department. On physical
examination, there is a solitary puncture wound on the sole of her right foot that is oozing blood.
The surrounding skin is intact with minimal erythema and minimal tenderness. The girl has full
range of motion of her right ankle and toes without any pain. The father provides the intact rusty
nail for inspection.

Of the following, the MOST appropriate next management step for this girl is
A. irrigation of the wound
B. oral antibiotic administration
C. tetanus postexposure prophylaxis
D. wound debridement

American Academy of Pediatrics 237


PREP ® Self-Assessment PREPSA 2021
Correct Answer: A
The most appropriate next step in the management of this child’s injury is irrigation of the
wound. One of the hallmarks of wound care is irrigation under a mild to moderate amount of
pressure to remove loose tissue and foreign material, and possibly reduce the bacterial load in the
wound. A recent Cochrane review found that using tap water for irrigation of acute wounds does
not increase infection rates compared with irrigation with other solutions.

Evaluation of a puncture wound commences with obtaining a thorough history and discerning
the exact mechanism of injury. The physical examination includes evaluation of the wound and
all anatomically associated joints and tendons. Identifying the causative agent of the puncture
wound is important because one must ensure that there is no retained foreign body. Superficial
foriegn bodies can be easily removed by the pediatric provider; however, deeper foreign bodies
may require surgical consultation. Wounds should be copiously irrigated under pressure and the
need for tetanus postexposure prophylaxis should be assessed.

Puncture wounds tend to be found on either the hand or foot, occurring when an individual
reaches for something or steps on something, respectively. Because these injuries are typically
deeper than simple lacerations, they present a higher risk for infection; however, prophylactic
antibiotics need not be prescribed for a simple superficial puncture wound without any signs of
surrounding infection. Puncture wounds should be closely monitored for signs of deep infection
because of the potential for deeper penetration than what one can appreciate from careful
inspection of the wound. Complications that can arise from a puncture wound to the hand or foot
include abscess, tenosynovitis, septic arthritis, and osteomyelitis. These should be suspected if
there is worsening of symptoms several days after the initial injury, pain out of proportion to skin
findings, spreading erythema, purulent discharge, or pain with passive movement of the affected
extremity.

The organisms primarily responsible for infections occurring after a puncture wound
are Staphylococcus aureus and β-hemolytic Streptococcus. Puncture wounds on the foot which
penetrate through the bottom of a sneaker run the risk of
introducing Pseudomonas species, which tends to grow in the moist inner sole of the shoe. Cat
bites may introduce Pasteurella species or Bartonella henselae; however, cat bites typically have
2 puncture sites.

Given that the nail penetrated the sole of the girl’s sneaker, the threshold for initiation of
prophylactic antibiotics should be low; however, that does not take precedence over cleaning and
irrigation of the wound. Tetanus postexposure prophylaxis is not warranted in this case, because
this child has received her primary series of tetanus vaccines including a 4th dose within the
previous 5 years. There is no indication for debridement of a puncture wound that is not clearly
infected.

American Academy of Pediatrics 238


PREP ® Self-Assessment PREPSA 2021
PREP Pearls
• Puncture wounds are at higher risk for infection than simple lacerations.
• Wound irrigation can be done with normal saline, sterile water, or tap water and should
be done under mild to moderate pressure.
• Puncture wounds through sneaker soles can introduce Pseudomonas species deep into the
soft tissues.

ABP Content Specifications(s)


• Identify the sequelae of puncture wounds of various etiologies
• Plan the appropriate evaluation of various puncture wounds, including a puncture wound
through a sneaker, and manage appropriately

Suggested Readings
• Black KD, Cico SJ, Caglar D. Wound management. Pediatr Rev. 2015;36(5):207-216.
doi: 10.1542/pir.36-5-207.
• Fernandez R, Griffiths R. Water for wound cleansing. Cochrane Database Syst Rev.
2012;(2):CD003861. doi: 10.1002/14651858.CD003861.pub3.

American Academy of Pediatrics 239


PREP ® Self-Assessment PREPSA 2021
Question 65
A healthy 16-year-old adolescent is seen for a health supervision visit. Her mother reports no
concerns. Vital signs, growth parameters, and physical examination findings are normal. Her
sexual maturity rating is stage IV; she underwent menarche at 12 years of age. Her mother
reports that their religion prohibits receiving vaccines after menarche. Privately, the adolescent
reports that she has researched vaccines on her own and asks that she receive all recommended
vaccines.

Of the following, the BEST next step is to


A. administer all recommended vaccines confidentially
B. contact the local ethics committee for further assistance
C. further explore the mother’s decision regarding vaccination
D. report the mother to child protective services for neglect

American Academy of Pediatrics 240


PREP ® Self-Assessment PREPSA 2021
Correct Answer: C
The best next step for the family in the vignette is to further explore the mother’s decision
regarding vaccination to demonstrate respect and desire for open communication. Administering
all vaccinations confidentially is inappropriate because adolescents in the United States do not
uniformly have the legal ability to consent to vaccination. Contacting the local ethics committee
may be necessary, but would not be the first action step. Reporting the mother to child protective
services for neglect is inadvisable because her refusal does not clearly qualify as neglect.

Vaccination hesitation and refusal has been increasing in the United States for several decades.
Motivations are multifactorial, but common reasons include religion or personal belief. Some
states have eliminated personal belief exemptions to vaccination, leading some families to use
the religious exemption as a loophole. Most religions do not oppose it, and many actually
encourage vaccination. These religions include Christianity, Judaism, Islam, Hinduism, and
Buddhism. Catholicism permits vaccination using vaccines manufactured with cell lines derived
from aborted human fetal cells (rubella, varicella, and hepatitis A) as long as no other alternative
exists. Other groups, such as Christian Scientists, specifically prohibit vaccination.

When families express vaccine refusal or hesitancy, providers should respect their autonomy to
make medical decisions without coercion while also exercising beneficence, doing what is best
for the child. The best practice is to engage in open dialogue, even when the initial reason given
is religious. Most religious objections to vaccinations have been addressed by religious leaders,
including the use of human fetal cells, pork products, or other animal products, so directing
families to these statements may alleviate those concerns. Open discussion may also reveal that
the religious objection is highly individualized or somewhat removed from the original doctrine.
Even if the family cannot be persuaded, bestowing respect and dignity helps build a trusting
patient-physician relationship that may allow for discussion at subsequent visits.

The same technique should be used with personal belief objections to vaccination. Dismissal
from a practice based on vaccination refusal should be used as a last resort, and usually only after
multiple attempts at persuasion have been made, carefully weighing the risks and benefits to the
child, physician, and community, and potentially involving local ethics committees. Even after
dismissal, the physician is obligated to provide emergency and continuing care for a specified
period, depending on state law.

Parental refusal to vaccinate may not meet criteria for medical neglect. In fact, involving child
protective services solely on this basis may serve only to erode the patient-physician relationship
and enforce distrust in the medical system. If, however, the child is at significant risk of harm
such as during an outbreak, the legal system may need to be used to protect the community.
Adolescents are increasingly obtaining legal rights to medical services in confidence. Some
states have allowed mature minors to consent not only to care related to sexual and mental
health, but also to vaccination regardless of their legal guardians’ desires. Awareness of
changing local regulation is important, but perhaps more so, is actively advocating for the
elimination of nonmedical exemptions.

American Academy of Pediatrics 241


PREP ® Self-Assessment PREPSA 2021
PREP Pearls
• Open communication is critical when addressing or discussing vaccine or medical
treatment refusal or hesitancy, even when the initial reason is religious.
• Refusal of routine vaccination may not meet criteria for medical neglect, and dismissal of
such families from a medical practice should be used as a last resort.

ABP Content Specifications(s)
• Recognize and apply ethical principles involved in the patient-parent-pediatrician
relationship regarding religious (philosophical) exemptions for medical
treatment/immunizations

Suggested Readings
• Edwards KM, Hackell JM; Committee on Infectious Disease, Committee on Practice and
Ambulatory Medicine. Countering vaccine hesitancy. Pediatrics.
2016;138(3):e20162146. doi:10.1542/peds.2016-2146.
• Matheny Antommaria AH, Weise KL; Committee on Bioethics. Conflicts between
religious or spiritual beliefs and pediatric care: informed refusal, exemptions, and public
funding. Pediatrics. 2013;132(5):962-965. doi:10.1542/peds.2013-271.

American Academy of Pediatrics 242


PREP ® Self-Assessment PREPSA 2021
Question 66
A 9-year-old boy is seen for a routine follow-up for asthma that was diagnosed at age 3 years. He
has been treated with inhaled corticosteroids since the time of diagnosis. His current medications
include mometasone furoate 100 μg with formoterol fumarate 5 μg two inhalations twice daily,
fluticasone propionate one spray in each nostril daily, and albuterol two puffs inhaled every 4
hours as needed for cough, shortness of breath, or wheeze. His asthma has been well controlled
since his inhaled corticosteroid therapy was intensified 3 years ago. His last asthma exacerbation
and last course of oral corticosteroids was more than 1 year ago. His vital signs are normal for
his age. His growth chart is shown in Item Q66. His body mass index is 15.9 kg/m2 (40th
percentile). Examination of his respiratory system yields normal findings. The remainder of his
physical examination findings are unremarkable.

American Academy of Pediatrics 243


PREP ® Self-Assessment PREPSA 2021

American Academy of Pediatrics 244


PREP ® Self-Assessment PREPSA 2021
Of the following, the MOST likely cause of this boy’s physical findings is
A. constitutional delay of growth
B. exogenous glucocorticoid
C. growth hormone deficiency
D. pituitary adenoma

American Academy of Pediatrics 245


PREP ® Self-Assessment PREPSA 2021
Correct Answer: B
The boy in the vignette has linear growth failure that coincides with intensification of his inhaled
glucocorticoid therapy regimen. The most likely cause of his linear growth failure is exogenous
glucocorticoid. Excess glucocorticoid by any route can have a profound effect on linear growth.
In this case, the route is inhalation.

The boy’s growth curve is not consistent with constitutional delay of growth. In constitutional
delay of growth, growth velocity at this age should be normal. His growth curve could be
consistent with growth hormone deficiency, but given his history, exogenous glucocorticoid is
more likely. An adrenocorticotropic hormone (ACTH)-producing pituitary adenoma can cause
growth failure due to excess endogenous glucocorticoid, but the boy has a history of exogenous
inhaled glucocorticoid exposure, making the exogenous form the more likely culprit.
Excess inhaled glucocorticoid, as with excess glucocorticoid acquired by any route, can also
suppress the endogenous hypothalamic-pituitary-adrenal (HPA) axis. This HPA axis suppression
becomes a problem when the axis cannot respond appropriately to stress and symptoms of
adrenal insufficiency ensue.

Frank Cushingoid features can also occur with excess inhaled corticosteroids. Cushing syndrome
is characterized by weight gain, centripetal obesity, linear growth failure, violaceous striae, and
osteopenia. The most common cause of Cushing syndrome is exogenous glucocorticoid. Cushing
syndrome caused by endogenous glucocorticoid excess is rare.

Endogenous Cushing syndrome can be divided into adrenocorticotropic hormone (ACTH)-


dependent and ACTH-independent etiologies. Adrenocorticotropic hormone–dependent causes
include an ACTH-secreting pituitary adenoma and ectopic ACTH secretion. Adrenocorticotropic
hormone–independent causes are due to adrenal tumors or other adrenal hyperfunction.
After the clinician excludes exogenous glucocorticoid exposure, the first step in the evaluation of
Cushing syndrome is to confirm hypercortisolism. Options for initial testing for hypercortisolism
include a 24-hour urine free cortisol test, a low-dose (1-mg) overnight dexamethasone
suppression test, or a midnight salivary cortisol test. Positive test findings should be confirmed
with a second measurement. A morning cortisol level that fails to suppress after administration
of low-dose dexamethasone the night before is consistent with hypercortisolism. Similarly, an
elevated midnight salivary cortisol level is consistent with hypercortisolism and disruption of the
normal circadian rhythm.

Once hypercortisolism is confirmed, the next step is to distinguish an ACTH-dependent versus


ACTH-independent etiology and further determine the source. These second-line tests include an
ACTH level, a high-dose dexamethasone suppression test, and pituitary or adrenal imaging as
indicated. Cortisol levels are suppressed after administration of high-dose dexamethasone with
an ACTH-dependent pituitary source of Cushing but not with adrenal sources. Pituitary imaging
may show an adenoma. Adrenal imaging may show an adrenal tumor or nodular adrenal disease.

American Academy of Pediatrics 246


PREP ® Self-Assessment PREPSA 2021
PREP Pearls
• The most common cause of Cushing syndrome is exogenous glucocorticoid.
• Excess glucocorticoid acquired by any route can have a profound effect on linear growth.
• Suppression of the hypothalamic-pituitary-adrenal axis by excess glucocorticoid acquired
by any route can cause an adrenal crisis during times of stress.

ABP Content Specifications(s)


• Plan appropriate diagnostic evaluation of Cushing syndrome
• Identify the clinical features associated with Cushing syndrome, including that associated
with exogenous corticosteroid therapy

Suggested Readings
• Kapadia CR, Nebesio TD, Myers SE, et al; Drugs and Therapeutics Committee of the
Pediatric Endocrine Society. Endocrine effects of inhaled corticosteroids in
children. JAMA Pediatr. 2016;170(2):163-170. doi:10.1001/jamapediatrics.2015.3526.
• Klein J, Vuguin P, Hyman S. Cushing syndrome. Pediatr Rev. 2014;35(9):405-407.
doi:10.1542/pir.35-9-405.
• Smith A, Doan ML, Roy D, Pinsker JE. Adrenal insufficiency and growth failure
secondary to inhaled corticosteroids: a paradoxical complication. Clin Pediatr (Phila).
2012;51(12):1194-1196. doi:10.1177/0009922812437932.
• Speiser PW. Adrenal dysfunction. In: McInerny TK, Adam HM, Campbell DE, DeWitt
TG, Foy JM, Kamat DM, eds. American Academy of Pediatrics Textbook of Pediatric
Care. 2nd ed. Elk Grove Village, IL: American Academy of Pediatrics; 2017:1692-
1700. Pediatric Care Online.

American Academy of Pediatrics 247


PREP ® Self-Assessment PREPSA 2021

Question 67
A 7-year-old girl is being evaluated for a 3-day history of an itchy rash. She is otherwise well, is
receiving no medications, and has no history of a similar eruption. Her vital signs are normal,
and her physical examination findings are unremarkable aside from a rash in a patchy
distribution on the face and upper extremities (Item Q67).

Item Q67: Rash for the girl described in the vignette.


Reprinted with permission from Krowchuk D. Miscellaneous Skin Conditions. In: Neinstein, LS,
Katzman DK, Callahan T, Gordon, CM, Joffe, A, Rickert,V, eds. Neinstein’s Adolescent and
Young Adult Health Care. 6th ed. Wolters Kluwer. 2016.

Of the following, the MOST likely diagnosis is


A. allergic contact dermatitis
B. atopic dermatitis
C. irritant contact dermatitis
D. seborrheic dermatitis

American Academy of Pediatrics 248


PREP ® Self-Assessment PREPSA 2021
Correct Answer: A
The girl in the vignette has an acquired eczematous dermatitis—that is, one composed of
inflamed papules or plaques and associated with pruritus. Some of her lesions are grouped, and
others are in a linear distribution (Item C67A). The latter finding is a key to recognizing allergic
contact dermatitis (ACD) caused by poison ivy, oak, or sumac. This linear arrangement results
from brushing against a damaged plant, during which act plant resin is applied to the skin. If the
resin is applied indirectly (eg, from a pet or fomites such as clothing), lesions are not linear.
Rather, one will observe an asymmetric and patchy distribution of fine papules with a
background of erythema (Item C67B). Other eczematous eruptions, such as atopic dermatitis,
irritant contact dermatitis, and seborrheic dermatitis, are not characterized by lesions in a linear
array.

Item C67A: Allergic contact dermatitis caused by poison ivy. In some areas (arrows)
erythematous papules are distributed in a linear arrangement.
Reprinted with permission from Krowchuk D. Miscellaneous Skin Conditions. In: Neinstein, LS,
Katzman DK, Callahan T, Gordon, CM, Joffe, A, Rickert,V, eds. Neinstein’s Adolescent and
Young Adult Health Care. 6th ed. Wolters Kluwer. 2016.

American Academy of Pediatrics 249


PREP ® Self-Assessment PREPSA 2021

Item C67B: Allergic contact dermatitis caused by poison ivy. In this patient, the resin was not
applied directly by the plant. As a result, the lesions are not arranged in a linear distribution.
Courtesy of D Krowchuk

In the United States, poison ivy, oak, and sumac are the most common causes of ACD. These
plants are members of the genus Toxicodendron (formerly Rhus) and contain the allergen
urushiol. Poison ivy occurs throughout the United States and Canada; poison oak is found along
the pacific coast of North America and in the central and southeast regions of the United States,
and poison sumac in eastern North America and the south-central United States
(https://plants.usda.gov/java/nameSearch). The adage “leaves of three, let them be” is useful in
identifying poison ivy and oak (Item C67C), although both may have more leaves (5 or 7)
depending on the species, geographic location, and climate. Poison sumac typically has 7 to 13
leaves arranged in pairs. Black spots may be observed on the leaves of Toxicodendron species;
this is oxidized urushiol.

American Academy of Pediatrics 250


PREP ® Self-Assessment PREPSA 2021

Item C67C: Poison ivy: note the “leaves of three.”


Courtesy of D Krowchuk

Allergic contact dermatitis is the result of a type IV immunologic (delayed hypersensitivity or


cell-mediated) reaction. Once sensitization occurs, which may occur after a single exposure (as
may be the case for a potent antigen like urushiol) or after longer periods of repeated contact,
reexposure to the allergen results in memory T-cell activation and an inflammatory response.
The result is a rash that typically appears 1 to 3 days after exposure but as early as after 8 hours
in those who are highly sensitive; it may persist for 1 to 3 weeks. If the antigen involved is
potent, like urushiol in poison ivy, and depending on the amount that contacts the skin, the result
is an acute dermatitis with erythema, edema, vesicle or bulla formation; oozing (when vesicles
rupture); and crusting (when fluid dries) (Item C67D). Less potent antigens (eg, nickel) produce
features of subacute or chronic dermatitis with lichenification and scaling (Item C67E).
The diagnosis of ACD is usually made clinically on the basis of the eruption’s appearance and
location. Another clue is a dermatitis that persists despite topical corticosteroid treatment. In
uncertain cases, patch testing to a panel of allergens may be performed. Clues to the recognition
of various forms of ACD according to the location of the dermatitis are summarized in Item
C67F.

American Academy of Pediatrics 251


PREP ® Self-Assessment PREPSA 2021

Item C67D: Acute contact dermatitis caused by poison ivy. Vesicles in a linear array are seen on
the forearm.
Reprinted with permission from Mancini AJ, Krowchuk DP, eds. Pediatric Dermatology. A
Quick Reference Guide. 3rd ed. Elk Grove Village, IL: American Academy of Pediatrics; 2016.

American Academy of Pediatrics 252


PREP ® Self-Assessment PREPSA 2021
Item C67E: Acute contact dermatitis on the lateral neck caused by nickel in a necklace. Nickel is
a less potent antigen; erythema and vesicle formation are not present.
Courtesy of D Krowchuk

American Academy of Pediatrics 253


PREP ® Self-Assessment PREPSA 2021

American Academy of Pediatrics 254


PREP ® Self-Assessment PREPSA 2021
If ACD is diagnosed, avoidance of the allergen is important. In the case of plant ACD
(ie, Rhus dermatitis), patients and families should be encouraged to become familiar with the
appearance of poison ivy, oak, or sumac. When avoidance is less feasible (eg, when gardening,
hiking, or camping), wearing protective clothing (long sleeves, long pants, socks, gloves) can be
helpful. However, it is important to remember that urushiol on clothing may be spread to the
skin. Topical barrier agents (eg, bentoquatam) can prevent skin contact with urushiol, but none
are completely effective. If plant contact occurs and is recognized, immediate washing with soap
and water (ideally within 5-10 min) may remove the resin and limit or prevent dermatitis.

The treatment of mild ACD is aimed at reducing pruritus. Options include a topical antipruritic
agent (a lotion such as calamine or a product containing pramoxine), a midpotency topical
corticosteroid, an oral antihistamine, or cool tap water compresses (these are soothing, promote
drying of lesions through evaporation, and remove crust). Topical products containing
diphenhydramine or benzocaine are best avoided, because they are potentially sensitizing (ie,
may induce an ACD). For more widespread involvement or severe involvement of the face or
genitalia, systemic corticosteroid therapy is useful. Prednisone (beginning at 1 mg/kg per day or
its equivalent) is tapered gradually over 2 to 3 weeks; earlier cessation may result in a rebound of
the dermatitis.

PREP Pearls
• Plant contact dermatitis (Rhus dermatitis) (ie, poison ivy) is the most common form of
allergic contact dermatitis. A linear distribution of lesions supports the diagnosis but is
not always present.
• The presence of a persistent dermatitis despite topical corticosteroid treatment should
raise suspicion of allergic contact dermatitis. The allergen responsible often results in a
rash with a characteristic location.
• The treatment of mild allergic contact (Rhus) dermatitis is aimed at reducing pruritus.

ABP Content Specifications(s)


• Recognize the clinical findings associated with contact dermatitis
• Understand the pathophysiology of rhus dermatitis

Suggested Readings
• Jacob SE, Brankov N, Kerr A. Diagnosis and management of allergic contact dermatitis
in children: common allergens that can be easily missed. Curr Opin Pediatr.
2017;29:443-447. doi:10.1097/MOP.0000000000000513.
• Mancini AJ, Krowchuk DP, eds. Pediatric Dermatology. A Quick Reference Guide. 3rd
ed. Elk Grove Village, IL: American Academy of Pediatrics; 2016.
• Paller AS, Mancini AJ. Hurwitz Clinical Pediatric Dermatology. 5th ed. Edinburgh, UK:
Elsevier; 2016.
• Silverberg NB, Pelletier JL, Jacob SE, et al. AAP Section on Dermatology, Section on
Allergy and Immunology. Nickel Allergic Contact Dermatitis: Identification, Treatment,
and Prevention. Pediatrics. 2020;145(5):e20200628 DOI: 10.1542/peds.2020-0628 .

American Academy of Pediatrics 255


PREP ® Self-Assessment PREPSA 2021
Question 68
A 14-year-old adolescent who recently immigrated from El Salvador is seen in the pediatric
clinic for evaluation of a rash. He was in good health until 3 days ago when he developed
malaise and sore throat. Yesterday, he noticed an itchy and painful rash over his chest that has
progressed. He does not have any chronic medical conditions. His immunization history is
unknown. On physical examination, his vital signs include a temperature of 37.9°C, heart rate of
98 beats/min, respiratory rate of 16 breaths/min, and blood pressure of 128/65 mm Hg. Multiple
papular to vesicular lesions are noted over his face, thorax, and extremities (Item Q68).
The boy was in the clinic waiting area for 20 minutes before being placed in an examination
room. There was 1 other child, a healthy 6-month-old infant, in the waiting room at the same
time.

Item Q68: Lesions noted on the adolescent described in the vignette.


Courtesy of L Marquez

Of the following, the MOST appropriate next step in management for this infant’s exposure is
A. disease-specific immune globulin
B. disease-specific vaccination
C. no prophylaxis
D. oral acyclovir

American Academy of Pediatrics 256


PREP ® Self-Assessment PREPSA 2021
Correct Answer: C
Prophylaxis is not indicated for the infant in the vignette. The adolescent’s illness is most
consistent with primary varicella virus infection, given his unknown vaccination history and the
findings of widespread papular-vesicular pruritic lesions in different stages of development.
Although varicella is highly contagious and the infant is too young to have been vaccinated, the
infant does not have underlying medical conditions that would make him a candidate for
postexposure prophylaxis.

Interventions for varicella-zoster virus exposures are only considered for individuals without
evidence of immunity (Item C68). Interventions can include varicella vaccine, varicella-zoster
immune globulin, and antiviral medications including acyclovir and valacyclovir.
For individuals who are vaccine-eligible (have no vaccine contraindications and are ≥12 months
of age), vaccination should be administered ideally within 3 days but up to 5 days after the
exposure.

American Academy of Pediatrics 257


PREP ® Self-Assessment PREPSA 2021

American Academy of Pediatrics 258


PREP ® Self-Assessment PREPSA 2021

Varicella-zoster immune globulin is indicated for immunocompromised individuals, pregnant


women, and certain newborns. Ideally, it should be administered within 96 hours of exposure,
but can be administered up to 10 days afterward. Immunocompromised individuals include those
with T-cell immunodeficiencies (congenital, HIV), neoplasms of the bone marrow or lymphatic
system, hematopoietic stem cell transplant recipients, and those receiving immunosuppressive
therapy. Newborn candidates include infants whose mothers developed primary varicella disease
between 5 days before and 48 hours after delivery, hospitalized preterm infants (≥28 weeks’
gestation) whose mothers lack evidence of varicella immunity, and all hospitalized preterm
infants of less than 28 weeks of gestation. If varicella-zoster immune globulin is unavailable,
intravenous immune globulin can be substituted.

Antiviral medication can be considered for patients who are unable to receive vaccine or immune
globulin such as adolescents, adults, and individuals with chronic underlying conditions in which
varicella disease would be problematic (eg, mild immunocompromised states that do not qualify
for varicella-zoster immune globulin). The medication should be started 7 to 10 days after the
exposure and be administered for 7 days.

PREP Pearls
• Interventions for varicella-zoster virus exposures, including varicella vaccine, varicella-
zoster immune globulin, and antivirals, are only considered for individuals without
evidence of varicella immunity.
• Postexposure prophylaxis with varicella vaccine should be provided for individuals who
are vaccine-eligible, ideally within 3 days but up to 5 days after the exposure.
• Postexposure prophylaxis with varicella-zoster immune globulin should be provided for
immunocompromised individuals, pregnant women, and eligible newborns ideally within
96 hours but up to 10 days after the exposure.

ABP Content Specifications(s)


• Understand the epidemiology of varicella-zoster virus
• Plan appropriate control measures to prevent the spread of varicella-zoster
• Recognize the clinical features associated with varicella-zoster infections in normal and
immunocompromised children of various ages

Suggested Readings
• American Academy of Pediatrics. Varicella-zoster virus infections. In: Kimberlin DW,
Brady MT, Jackson MA, Long SS, eds. Red Book: 2018 Report of the Committee on
Infectious Diseases. Itasca, IL: American Academy of Pediatrics; 2018:869-883.
• Blair R. Varicella-zoster virus. Pediatr Rev. 2019;40(7):375-377. doi: 10.1542/pir.2017-
0242.
• Kimberlin DW, Price N. Chickenpox In: McInerny TK, Adam HM, Campbell DE,
DeWitt TG, Foy JM, Kamat DM, eds. American Academy of Pediatrics Textbook of
Pediatric Care. 2nd ed. Itasca, IL: American Academy of Pediatrics; 2016; chap
227:1835-1845. Pediatric Care Online.

American Academy of Pediatrics 259


PREP ® Self-Assessment PREPSA 2021
Question 69
A 16-year-old adolescent is having a sports preparticipation physical examination 1 week before
the start of the cross country running season. Based on her responses on the required history and
physical examination form (Item Q69), additional information is obtained about a syncopal
event that occurred while she was running in the first meet of her cross country season last year.
She recovered immediately, and finished the remainder of her season without incident. The girl
and her mother attribute this episode to dehydration and low blood sugar, though no medical
evaluation was performed. Her medical history is significant for anxiety; the mother reports that
her daughter gets very nervous before her track meets.

On physical examination, the girl’s lungs are clear to auscultation, cardiac findings are normal,
and pulses are symmetrically present in all extremities. The remainder of her physical
examination findings are within normal limits.

Of the following, the BEST next management step for this girl is to
A. provide clearance for full participation and counseling on pre-exercise nutrition and
hydration
B. provide provisional clearance with the girl’s commitment to pursue cognitive behavioral
therapy
C. withhold clearance for all sports participation pending cardiac evaluation
D. withhold clearance pending results of fasting blood glucose level, electrolyte levels, and
complete blood cell count

American Academy of Pediatrics 260


PREP ® Self-Assessment PREPSA 2021
Correct Answer: C
Syncope during exertion is a potentially ominous event, and may be the only presenting
symptom before sudden cardiac death. Therefore, for the adolescent in the vignette, clearance
should be withheld for participation in any activity requiring significant exertion until she has
had a full cardiac evaluation.

Syncope is common during adolescence. The vast majority of these are vasovagal events that do
not warrant intervention beyond reassurance of the patient and caregivers. The timing of collapse
is a crucial distinction. Collapse after high levels of exertion, particularly at the finish line of
running events, most commonly occurs because of blood pooling in the legs. This can be
prevented by encouraging athletes to perform an active cool-down; treatment involves placing
athletes supine, with their legs elevated above the heart. Episodes of syncope that occur during
physical exertion need a full evaluation to rule out cardiac pathology, because vagal tone is
reduced during exercise (Item C69).

The history obtained from athletes experiencing syncope should include the overall state of the
patient’s health preceding the event and the patient’s recollection of the event, particularly any
American Academy of Pediatrics 261
PREP ® Self-Assessment PREPSA 2021
presyncopal symptoms such as chest pain, palpitations, or shortness of breath. Any available
information from witnesses should be obtained as well. The American Academy of Pediatrics
preparticipation history form contains 10 questions regarding personal and family medical
history that are important to review for any athlete presenting with exercise-associated syncope
(https://www.aap.org/en-us/Documents/PPE-History-Form-%28English%29.pdf).

For a child who has had a syncopal event, physical examination with a full cardiac assessment is
recommended, including:
• Vital signs
• Peripheral pulses: symmetry, amplitude, and timing
• Cardiac auscultation when supine and standing, with or without the Valsalva maneuver
Hypertrophic cardiomyopathy may produce a murmur because of turbulent flow through a
relatively obstructed outflow tract. The degree of obstruction is dynamic. As cardiac preload
increases, the obstruction is reduced, and the murmur decreases in intensity. Therefore, the
murmur of hypertrophic cardiomyopathy increases with the Valsalva maneuver and decreases
when supine as compared to sitting or standing. These effects are opposite to those seen with
most benign cardiac murmurs.

In patients with a family history and/or physical examination findings suggestive of Marfan
syndrome, the 2010 revised Ghent nosology should be used for making the diagnosis. Marfan
stigmata include high-arched palate, pectus excavatum, kyphoscoliosis, long and slender fingers
and toes, mitral valve prolapse, aortic insufficiency, myopia, and generalized laxity of skin and
ligaments. The Marfan Foundation has compiled a diagnostic algorithm that can be accessed
at https://www.marfan.org/resources/professionals/marfandx.

Cardiac conduction and structure should also be evaluated in patients with exertional syncope,
and cardiology referral should be considered. Electrocardiography (ECG) and echocardiography
are typically the initial studies obtained. A resting 12-lead ECG can identify arrhythmias and
depolarization/repolarization abnormalities. Well-trained athletes often develop physiologic
cardiac changes known as the athletic heart syndrome, which can mimic pathologic changes on
ECG. These changes include voltage criteria for left ventricular hypertrophy, early
repolarization, right bundle branch block, sinus bradycardia, and first-degree AV block.
Although tutorials on ECG assessment in athletes have been developed for noncardiology
providers, most pediatric providers do not have significant experience reading ECGs and would
benefit from cardiology input. Providers should not rely on electronic interpretations generated
by the ECG machine itself.

Echocardiography is the best initial test to evaluate cardiac structure. Hypertrophic


cardiomyopathy and arrhythmogenic right ventricular dysplasia are often readily detected on
echocardiography; however, delineation of coronary artery anatomy is more difficult, and may
require advanced imaging, such as cardiac magnetic resonance imaging, computed tomography,
or angiography. In cases in which ECG and echocardiography are unrevealing, additional testing
may be indicated. Testing may include event monitoring, exercise testing, electrophysiologic
studies, and/or genetic testing.

American Academy of Pediatrics 262


PREP ® Self-Assessment PREPSA 2021
Noncardiac causes of exertional syncope are often suggested by the event history. Heat stroke
typically results in progressive changes in level of consciousness (eg, confusion, agitation) rather
than frank and sudden syncope. Heat syncope is a less severe form of heat-related illness that
typically occurs with prolonged standing and is less common during exercise. Exertional
hyponatremia is typically described in endurance events longer than 4 hours in duration, and is
much less common in events of shorter duration. However, some athletes who aggressively
rehydrate with hypotonic fluid and do not replenish dietary sodium in between exercise sessions
may be at increased risk with multiple rigorous exercise sessions per day (eg, during tournaments
with multiple games/matches per day).

Patients and caregivers often attribute exertional syncope to dehydration or poor nutrition (eg,
“low blood sugar”). Although these issues may produce fatigue and presyncopal symptoms, they
typically do not produce frank loss of consciousness. Although many young athletes benefit from
nutritional counseling, this would not be sufficient to clear the adolescent in this vignette for a
return to sports. Laboratory evaluation is unlikely to add to diagnostic or treatment decision
making and is not indicated at this time for the girl in the vignette.

Female adolescents with anxiety may be more prone to episodes of vasovagal syncope but, as
described earlier, these episodes are less likely to occur during exercise. Cognitive behavioral
therapy may be warranted once cardiac causation is ruled out, but is not the most appropriate
next step for the girl in the vignette.

PREP Pearls
• Syncope during exertion is a potentially ominous event, and children with exercise-
associated syncope should be withheld from any activity requiring significant exertion
until they have had a full cardiac evaluation.
• Collapse during high levels of exertion is more concerning than collapse after completion
of high level exertion activity.
• Children who have had a syncopal event should undergo a physical examination with a
cardiac assessment including:
▪ Vital signs
▪ Peripheral pulses: symmetry, amplitude, and timing
▪ Cardiac auscultation when supine and standing, with or without the
Valsalva maneuver.

MOCA-Peds Objective
• Evaluate and manage the adolescent with syncope.

ABP Content Specifications(s)


• Recognize the cardiac risks associated with sports participation and when cardiac
evaluation is required

American Academy of Pediatrics 263


PREP ® Self-Assessment PREPSA 2021
Suggested Readings
• American Academy of Family Physicians, American Academy of Pediatrics, American
College of Sports Medicine, American Medical Society for Sports Medicine, American
Orthopedic Society for Sports Medicine, American Osteopathic Academy of Sports
Medicine. PPE: Preparticipation Physical Evaluation. 5th ed. Itasca, IL: American
Academy of Pediatrics; 2019.
• Drezner JA. Standardised criteria for ECG interpretation in athletes: a practical tool. Br J
Sports Med. 2012;46 (suppl 1):i6-i8. doi: 10.1136/bjsports-2012-091703.
• Madan S, Chung EH. Expert analysis: the syncopal athlete. American College of
Cardiology website. 2016. https://www.acc.org/latest-in-
cardiology/articles/2016/04/29/19/06/the-syncopal-athlete.

American Academy of Pediatrics 264


PREP ® Self-Assessment PREPSA 2021
Question 70
A 5-year-old boy remains hospitalized after experiencing septic shock complicated by acute
kidney injury. He is improving but has continued to receive diuretics. On routine laboratory
surveillance, he was found to have hypokalemia. The team decided to replace his potassium with
an intravenous bolus dose. Repeat testing after the dose was administered demonstrated
hyperkalemia. A review revealed that he had received a larger-than-appropriate dose of
potassium.

Of the following, the strategy MOST likely to decrease this type of error is to
A. ensure family-centered rounds
B. enter orders on a computer
C. limit trainee order prescribing
D. use verbal orders

American Academy of Pediatrics 265


PREP ® Self-Assessment PREPSA 2021

Correct Answer: B
The patient in the vignette received an inappropriate dose of potassium. Computerized order
entry has been shown to decrease medication prescribing errors and augment patient safety.
Conducting family-centered rounds, although a desirable form of communication, is not
associated with decreasing medication errors. Limiting trainee order prescribing has not been
shown to decrease medication errors and would harm education. Use of verbal orders would not
decrease medication prescribing errors; in fact, data suggest verbal orders may increase the
likelihood of medication prescribing errors.

Medication administration errors are common and can cause important morbidity and mortality.
Pediatric medicine presents a unique challenge to medication safety given the weight-based
nature of dosing. Studies have demonstrated decreased error rates with computer order entry,
increased education, preprinted order sheets, implementation of a barcode medication
administration system, and increased pharmacist involvement. The data available are
heterogeneous and varying definitions of terms are used, making comparisons difficult. More
research is needed in this area.

PREP Pearls
• Pharmacist involvement as well as computerized order entry, education, and preprinted
order sheets have been shown to decrease medication errors.
• A barcode medication administration system has been shown to decrease medication
administration errors.

ABP Content Specifications(s)


• Understand the role of ancillary services such as the pharmacy in the prevention of
medication errors
• Understand the role of computerized order entry and dose-range checking in reducing
medication errors

Suggested Readings
• Alex S, Adenew AB, Arundel C, Maron DD, Kerns JC. Medication errors despite using
electronic health records: the value of a clinical pharmacist service in reducing discharge-
related medication errors. Qual Manag Health Care. 2016;25(1):32-37.
doi:10.1097/QMH.0000000000000080.
• Maaskant JM, Vermeulen H, Apampa B, et al. Interventions for reducing medication
errors in children in hospital. Cochrane Database Syst Rev. 2015;(3):CD006208.
doi:10.1002/14651858.CD006208.pub3.
• Rinke ML, Bundy DG, Velasquez CA, et al. Interventions to reduce pediatric medication
errors: a systematic review (published correction appears in Pediatrics.
2015;136[3]:583). Pediatrics. 2014;134(2):338-360. doi: 10.1542/peds.2013-3531 .

American Academy of Pediatrics 266


PREP ® Self-Assessment PREPSA 2021
Question 71
A 15-year-old adolescent girl is evaluated in the office for a 2-month history of chronic
nonproductive cough associated with fatigue, 5- to 10-lb weight loss, low-grade fever, and
mildly painful nodules on her arms and legs. She has not traveled out of the state or had any
known sick exposures. On pulse oximetry, her oxygen saturation is 98% on room air. On
physical examination, the girl has no respiratory distress, and no crackles or wheezes are heard.
She has several 1- to 3-cm diameter, tender, nonerythematous, nonexcoriated nodules on the
extensor surfaces of both arms and legs.

Chest radiography suggests hilar adenopathy. Computed tomography additionally demonstrates


interstitial changes in the pulmonary parenchyma. Pulmonary function testing shows a mild
restrictive pattern with a decrease in forced vital capacity and total lung capacity. Carbon
monoxide diffusion is slightly decreased for age and hemoglobin concentration. A tuberculin
skin test has a negative result 60 hours after placement.

Of the following, the BEST next step in the evaluation of this adolescent is
A. biopsy of a skin nodule
B. bronchoscopy and bronchoalveolar lavage
C. repeat spirometry after administration of albuterol
D. 6-minute walk test

American Academy of Pediatrics 267


PREP ® Self-Assessment PREPSA 2021

Correct Answer: A
The girl in the vignette likely has sarcoidosis, with both pulmonary and nonpulmonary
manifestations. The presence of cough and fatigue with hilar adenopathy and restrictive lung
disease may be nonspecific, but for this adolescent with tender peripheral nodules, sarcoidosis is
the most likely diagnosis. Of the response choices listed, the most appropriate next step in
evaluation is biopsy of one of the nodules to demonstrate noncaseating granulomas.
Bronchoscopy will not produce a diagnosis, though the bronchoalveolar lavage findings may
show lymphocytosis with a reduced number of CD8 lymphocytes. However, this is not specific
for sarcoidosis and not necessary for the diagnosis.

Bronchodilator responsiveness on pulmonary function testing would be most helpful in a patient


with an obstructive rather than restrictive pattern of findings. The pathophysiology of sarcoidosis
does not involve airway reactivity, rather, interstitial inflammation can be seen, which is not
sensitive to bronchodilators. The girl in the vignette likely would have reduced endurance in a 6-
minute walk test, but that finding is nonspecific and would not help determine her diagnosis.
Sarcoidosis is a multisystem disease that usually presents between 20 and 60 years of age.
Sarcoidosis is 4 times more common in black than white populations, and more common in
women than men. Black women are most commonly affected; they may develop disease up to 10
years earlier than white patients. It is estimated that the lifetime risk for sarcoidosis in blacks in
the United States is 2.4% compared with a lifetime risk of 0.85% in whites. The most specific
diagnostic test for sarcoidosis is biopsy of an affected organ; for the adolescent in the vignette,
biopsy of a skin nodule is the least invasive approach to obtaining a tissue diagnosis. Biopsy of a
hilar lymph node would be a more invasive approach than necessary. Sarcoidosis can affect all
organ systems; the lungs are involved in most cases. Extrapulmonary sites may be affected
exclusively or may be involved before pulmonary findings. After the lungs, the skin, eyes, and
lymph nodes are the most commonly involved organs. The liver, spleen, parotid glands, and bone
marrow may also be sites of noncaseating granulomas characteristic of sarcoidosis.

PREP Pearls
• Sarcoidosis is diagnosed based on the presence of noncaseating granulomas in affected
tissues.
• The lungs, skin, eyes, and lymph nodes are the organs most commonly affected by
sarcoidosis.
• Sarcoidosis is 4 times more common in black than white populations, and more common
in women than men.

ABP Content Specifications(s)


• Recognize the clinical findings associated with sarcoidosis

American Academy of Pediatrics 268


PREP ® Self-Assessment PREPSA 2021
Suggested Readings
• Iannuzzi MC, Rybicki BA, Teirstein AS. Sarcoidosis. N Engl J Med. 2007;357:2153-
2165. doi: 10.1056/NEJMra071714.
• Nathan N, Marcelo P, Houdouin V, et al. Lung sarcoidosis in children: update on disease
expression and management. Thorax. 2015;70:537-542. doi: 10.1136/thoraxjnl-2015-
206825.
• Shetty AD, Gedalia A. Sarcoidosis in children. Curr Prob Pediatr. 2000;30:149-176.
doi: 10.1067/mps.2000.105929.
• Sileo C, Epaud R, Mahloul M, et al. Sarcoidosis in children: HRCT findings and
correlation with pulmonary function tests. Pediatr Pulmonol. 2014;49:1223.
doi: 10.1002/ppul.22956.

American Academy of Pediatrics 269


PREP ® Self-Assessment PREPSA 2021
Question 72
An 18-year-old patient is seen in the office to request sexually transmitted infection screening.
He noticed that his 19-year-old male partner, whom he has been dating for the past 10 months,
has a rash on the palms of his hands. He did an online search and is worried he may have been
exposed to an infection. He has engaged in oral and anal sex (insertive and receptive) and uses
condoms infrequently. He has never been screened for sexually transmitted infections. He reports
no dysuria, penile discharge, lesions, recent fever, or viral illnesses. His physical examination
findings are normal, and he does not have any skin lesions.

Of the following, the BEST initial step in treatment is to


A. administer azithromycin 1 g orally once and ceftriaxone 250 mg intramuscularly once
B. administer penicillin G benzathine 2.4 million units intramuscularly once
C. discuss pre-exposure prophylaxis and safer sex practices to reduce risk of sexually
transmitted infections
D. order a rapid plasma reagin test and tests for Neisseria gonorrhœae, Chlamydia
trachomatis, and HIV

American Academy of Pediatrics 270


PREP ® Self-Assessment PREPSA 2021

Correct Answer: D
The asymptomatic adolescent boy described in the vignette is at risk of acquiring sexually
transmitted infections (STIs). The first step in his treatment would be to order screening tests for
gonorrhea, chlamydia, HIV, and syphilis. He does not have a partner who has been confirmed to
have an STI, and he has not yet been screened, so he would not be treated presumptively for
syphilis, gonorrhea, or chlamydia infections. Although he would benefit from a discussion about
his options for reducing his risk of acquiring STIs, he has never been screened and is engaging in
high-risk behaviors.

Syphilis is caused by the spirochete Treponema pallidum and transmitted through sexual contact.
Syphilis infections can be congenital or acquired. Congenital syphilis occurs as a result of
intrauterine infection and can be spread transplacentally. Early congenital syphilis is defined as
diagnosis before age 2 years; late congenital syphilis is diagnosed at age 2 years or older. Infants
with early congenital syphilis may present with a rash, snuffles, hepatosplenomegaly, and
thrombocytopenia. Infants diagnosed with late congenital syphilis may present with
developmental delay, anterior bowing of the shins on plain radiographs, and Hutchinson teeth.

Acquired syphilis infections in childhood and adulthood are divided into primary, secondary,
latent, and tertiary-stage syphilis. Primary syphilis is characterized by a painless ulcer or chancre
that occurs at the site of inoculation and spontaneously resolves in 3 to 6 weeks without any
treatment. Secondary syphilis can occur weeks to months after inoculation and may present with
a diffuse maculopapular rash that may affect the palms and soles, condyloma lata, fever, malaise,
and lymphadenopathy. These symptoms can also spontaneously resolve in 3 weeks to 3 months.
Latent syphilis is asymptomatic; however, if affected patients are screened, they will be
seropositive. Latent syphilis is further divided into early (acquired within the preceding 12
months) and late (acquired more than 12 months prior) types. Tertiary-stage syphilis occurs
decades after initial infection and is characterized by dementia, cardiovascular involvement such
as aortic aneurysms, and development of granulomas or gummas.

Syphilis is diagnosed in a stepwise fashion, first with a nontreponemal screening test (rapid
plasmin reagin or Venereal Disease Research Laboratory test) and, if that first test result is
positive, a treponema-specific test (fluorescent treponemal antibody absorption test). A second-
step confirmatory test is necessary because false-positive nontreponemal test results can occur
with pregnancy, tuberculosis, other viral infections, and autoimmune disease. The treatment for
primary, secondary, and early latent syphilis is one dose of penicillin G benzathine 2.4 million
units intramuscularly; the treatment for late latent syphilis and tertiary syphilis is penicillin G
benzathine 2.4 million units intramuscularly once per week for 3 doses.

There has been an increase in the number of cases of syphilis in the United States in all stages. In
2018, there were more than 35,000 cases of primary and secondary syphilis reported to the
Centers for Disease Control and Prevention, and 64% of the cases were among men who have
sex with men.

American Academy of Pediatrics 271


PREP ® Self-Assessment PREPSA 2021
PREP Pearls
• There has been an increase in syphilis cases in the United States, with the majority among
men who have sex with men.
• The four stages of syphilis are primary, secondary, latent, and tertiary, each with varying
clinical manifestations.
• Syphilis is diagnosed in a stepwise fashion, first with a non treponemal screening test
(rapid plasmin reagin or Venereal Disease Research Laboratory test); if that result is
positive, a treponema-specific test (fluorescent treponemal antibody absorption test).

MOCA-Peds Objective
• Screen an adolescent for sexually transmitted diseases and manage appropriately.

ABP Content Specifications(s)


• Recognize the clinical features associated with congenital and acquired Treponema
pallidum infection
• Plan appropriate management for a patient with Treponema pallidum infection

Suggested Readings
• American Academy of Pediatrics. Syphilis. In: Kimberlin DW, Brady MT, Jackson MA,
Long SS, eds. Red Book: 2018 Report of the Committee on Infectious Diseases. 31st ed.
Itasca, IL; American Academy of Pediatrics; 2018:773-788. Red Book Online .
• Centers for Disease Control and Prevention. Sexually Transmitted Disease Surveillance
2018. Atlanta, GA: U.S. Department of Health and Human Services; 2019.
doi:10.15620/cdc.79370.
• Peeling RW, Mabey D, Kamb ML, Chen XS, Radolf JD, Benzaken AS. Syphilis. Nat Rev
Dis Primers. 2017;12(3):1-21. doi:10.1038/nrdp.2017.73.

American Academy of Pediatrics 272


PREP ® Self-Assessment PREPSA 2021
Question 73
A 15-year-old adolescent is brought to the emergency department for a new-onset generalized
tonic-clonic seizure that occurred while asleep. She was having a sleepover party and had stayed
up until 3 am. Around 7 am, her friend found her unresponsive with generalized shaking, her
eyes rolled up and open, and foaming at the mouth. The event lasted approximately 2 minutes
before self-resolving.

On arrival at the emergency department, the girl is drowsy but arousable. She has otherwise
normal neurologic findings. Her mother reports that the adolescent has had several episodes of
staring spells over the past few years which she attributed to her daughter ignoring her. Over the
past year, she has periodically complained of “spasms” where her hand will suddenly jump or
jerk. She denies substance use, recent or intercurrent illnesses, or trauma. There is no family
history of seizures.

Of the following, the MOST likely diagnosis is


A. benign focal epilepsy of childhood
B. juvenile absence epilepsy
C. juvenile myoclonic epilepsy
D. progressive myoclonic epilepsy

American Academy of Pediatrics 273


PREP ® Self-Assessment PREPSA 2021
Correct Answer: C
The adolescent in the vignette likely has juvenile myoclonic epilepsy (JME). Juvenile myoclonic
epilepsy is a common genetic generalized epilepsy usually presenting in early adolescence.
Affected children may have a combination of myoclonic jerks, generalized tonic-clonic seizures,
and/or absence seizures. Seizures typically occur in the early morning on waking. Often
myoclonic seizures are mistaken for clumsiness; if there is a suspicion of myoclonic seizures, the
history should include questions about early morning clumsiness, jitteriness, or dropping objects.
The most common triggers for seizures in JME are sleep deprivation, alcohol consumption,
stress, and flashing lights.

A diagnosis of JME is made based on the clinical history and electroencephalography (EEG)
findings. Interictal generalized polyspike and wave discharges at 4- to 6-Hz frequency are seen
on EEG. Photic stimulation can provoke typical EEG findings or electrographic seizure in some
children. Magnetic resonance imaging (MRI) of the brain results is normal; MRI is not required
if JME is suspected. Valproic acid is effective in the treatment of JME; however, because of
concerns for teratogenicity, valproic acid is not first-line treatment for adolescent girls. Other
treatment options include zonisamide, topiramate, and levetiracetam. Lamotrigine can be used,
but is known to exacerbate myoclonus in some children. Juvenile myoclonic epilepsy is a
lifelong form of epilepsy, requiring maintenance antiepileptic medication.

Benign focal epilepsy of childhood (benign rolandic epilepsy) is the most common childhood
epilepsy, usually presenting between 5 and 10 years of age. Seizures are focal, occur during
sleep, and consist of unilateral facial twitching, orofacial paresthesias, and speech arrest, with
preservation of consciousness. These focal seizures secondarily generalize in approximately 50%
of patients. Diagnosis is confirmed by EEG findings of benign focal epileptiform discharges of
childhood independently occurring in the centrotemporal regions bilaterally. Most children
outgrow the condition in adolescence. Antiepileptic medications are not usually required unless
seizures are frequent.

Juvenile absence epilepsy is a genetic generalized epilepsy presenting between 9 and 13 years of
age with absence seizures and brief (seconds) episodes of unresponsiveness and staring that
occur multiple times per day. Seizures can be mistaken for inattentiveness or daydreaming.
Absence seizures can be provoked with hyperventilation. Interictal 3-Hz spike and slow wave
discharges can be seen on EEG. Treatment with antiepileptic medication (ethosuximide, valproic
acid, or lamotrigine) is recommended and often lifelong, in contrast to childhood absence
epilepsy. In some the condition may evolve to JME.

Progressive myoclonic epilepsy is a rare group of genetic disorders characterized by myoclonic,


generalized tonic-clonic seizures and accompanying progressive neurodegeneration. Clinical
findings include cognitive decline, ataxia, neuropathies, and myopathies. Treatment is
supportive, aimed at symptom management and control.

American Academy of Pediatrics 274


PREP ® Self-Assessment PREPSA 2021
PREP Pearls
• Juvenile myoclonic epilepsy is a lifelong genetic generalized epilepsy presenting in early
adolescence with myoclonus, generalized tonic-clonic, and/or absence seizures; seizures
often occur in the morning.
• Seizures in juvenile myoclonic epilepsy can be provoked by sleep deprivation, photic
stimulation, alcohol consumption, or stress.
• Benign rolandic epilepsy (benign focal epilepsy of childhood) is the most common
childhood epilepsy. It typically presents between 5 and 10 years of age and resolves by
adolescence.

ABP Content Specifications(s)


• Recognize the clinical findings associated with juvenile myoclonic epilepsy, and manage
appropriately

Suggested Readings
• Park JT, Shahid AM, Jammoul A. Common pediatric epilepsy syndromes. Pediatr Ann.
2015;44(2):e30-e35. doi: 10.3928/00904481-20150203-09.
• Roddy SM, McBride M. Seizure disorders. In: McInerny TK, Adam HM, Campbell DE,
DeWitt TG, Foy JM, Kamat DM, eds. American Academy of Pediatrics Textbook of
Pediatric Care. 2nd ed. Itasca, IL: American Academy of Pediatrics; 2016;chap
327:2599-2616. Pediatric Care Online.
• Sidhu R, Velayudam K, Barnes G. Pediatric seizures. Pediatr Rev. 2013;34(8):333-342.
doi: 10.1542/pir.34-8-333.

American Academy of Pediatrics 275


PREP ® Self-Assessment PREPSA 2021
Question 74
An 11-year-old boy is brought to the pediatric intensive care unit with severe hypoxic brain
injury after self-inflicted hanging. He is initially treated with conservative neurological
protection. After 72 hours of care, his neurological examination shows apnea on the ventilator,
decorticate posturing to painful stimulation that is determined via electroencephalography not to
be a spinal reflex, fixed and dilated pupils, and diabetes insipidus. After a family conference to
discuss the child’s current status, the mother asks if her son could be an organ donor.

Of the following, the BEST response to the mother is that the child
A. is brain dead, so organ donation is an option and consultation with organ procurement
should proceed as requested
B. is not a candidate for organ donation because he did not give prior assent
C. is not a candidate for organ donation because the mechanism of injury was attempted
suicide
D. would be a candidate for donation after cardiac death

American Academy of Pediatrics 276


PREP ® Self-Assessment PREPSA 2021
Correct Answer: D
The best response to the mother of the boy in this vignette is that he would be a candidate for
organ donation after cardiac death.

In 1968, the Uniform Anatomical Gift Act provided the legal foundation on which human organs
and tissues can be donated for transplantation. The Uniform Brain Death Act of 1978 aimed to
clarify legal ambiguity surrounding the determination of death and established that the
"irreversible cessation of all functioning of the brain, including the brain stem" is death. In 1987,
guidelines for the determination of brain death in children were published cooperatively and in
2011 were revised and again promulgated by the Society of Critical Care Medicine, the
American Academy of Pediatrics, and the Child Neurology Society. The question of organ
donation can be an emotionally challenging discussion for any physician and family.
The diagnosis of brain death requires confirmation of irreversible loss of all functions of the
brain, including the brainstem. In clinical practice, the requisite findings in brain death are
persistent coma of a known cause in the absence of metabolic derangements or pharmacologic
confounders, absence of brainstem reflexes, and apnea. Reversible conditions that can interfere
with the neurological examination must be excluded before brain death testing is performed for
the clinical determination of brain death to be valid. A patient determined to be brain dead is
legally and clinically dead and, thus, his or her organs and tissues are eligible for donation (Item
C74).

American Academy of Pediatrics 277


PREP ® Self-Assessment PREPSA 2021

Item C74: Algorithm for Determination of Brain Death.


Reprinted with permission from AAP Guidelines for Brain Death in Children Toolkit.

American Academy of Pediatrics 278


PREP ® Self-Assessment PREPSA 2021

In situations in which brain death is not achieved, patients may be eligible for organ donation via
an alternative pathway known as donation after cardiac death. This may be an option for patients
who have a severe neurological injury and irreversible brain damage but still have minimal brain
function and do not meet brain death criteria. It is important to note that donation after cardiac
death is an option only for patients whose families have opted for withdrawal of life support. In
cases of donation after cardiac death, all life support is withdrawn in a controlled setting and the
patient must then progress to cardiopulmonary arrest; death is determined by lack of
cardiopulmonary function for at least 5 min. Once death is pronounced by a physician who is not
a part of the transplant team, the organs for transplantation are surgically removed by the
transplant team. In the event that the patient does not die after a period, donation is no longer
permitted.

The child in the vignette has sustained a severe hypoxic brain injury as a result of his self-
inflicted injury; however, he does not meet criteria for brain death. In addition to decorticate
posturing, which indicates intact brainstem reflexes, the child has not undergone formal apnea
testing and thus determination of brain death cannot yet be made. Although the child is a minor,
assent is not required with respect to organ donation decisions, and consent would be obtained
from the parent or guardian around the time of end-of-life decision making. Finally, suicide does
not restrict the ability for organ donation; however, medical examiner notification and
involvement will be required. The local organ procurement organization usually coordinates with
the medical examiner to coordinate donation in cases in which legally mandated autopsy may be
warranted.

PREP Pearls
• Brain death requires clinical evidence of irreversible cessation of all functioning of the
brain, including the brainstem.
• The Uniform Anatomical Gift Act provides the legal foundation on which human organs
and tissues can be donated for transplantation.

MOCA-Peds Objective
• Recognize and apply ethical principles involved in end-of-life care.

ABP Content Specifications(s)


• Recognize and apply ethical principles involved in decisions regarding organ
transplantation and donation

American Academy of Pediatrics 279


PREP ® Self-Assessment PREPSA 2021
Suggested Readings
• American Academy of Pediatrics. Guidelines for brain death in children:
toolkit. https://www.aap.org/en-us/Documents/socc_pediatric_bd_guideline_tool.pdf..
• Crain N. Pediatric brain death. Pediatr Rev. 2002;23(6):222-223. doi:10.1542/pir.23-6-
222.
• Nakagawa TA, Ashwal S, Mathur M, Mysore M; Society of Critical Care Medicine,
Section on Critical Care and Section on Neurology of American Academy of Pediatrics;
Child Neurology Society. Clinical report—guidelines for the determination of brain death
in infants and children: an update of the 1987 task force recommendations. Pediatrics.
2011;128(3):e720-e740. doi:10.1542/peds.2011-1511.
• Ross LF, Thistlethwaite JR; Committee on Bioethics. Minors as living solid-organ
donors. Pediatrics. 2008;122(2):454-461. doi:10.1542/peds.2008-1525.
• van der Jagt EW. Appendix A: Pediatric cardiopulmonary resuscitation. In: McInerny
TK, Adam HM, Campbell DE, DeWitt TG, Foy JM, Kamat DM, eds. American Academy
of Pediatrics Textbook of Pediatric Care. 2nd ed. Elk Grove Village, IL: American
Academy of Pediatrics; 2017:2995-3026. Pediatric Care Online .

American Academy of Pediatrics 280


PREP ® Self-Assessment PREPSA 2021
Question 75
A physician is conducting a follow-up visit with a 12-year-old girl who has been experiencing
painless vomiting. The girl has a 6-month history of effortless, nonbloody, nonbilious
regurgitation occurring 30 min after meals at least once per day. She has had a 4-kg weight loss
in the last 6 months. She reports that she enjoys food and is not intentionally losing weight. She
reports no nausea, abdominal pain, bloating, diarrhea, or constipation. She was prescribed a
proton pump inhibitor for 6 weeks without improvement. On physical examination, her heart rate
is within normal range and her dentition is normal.

Of the following, the MOST likely diagnosis is


A. cyclic vomiting syndrome
B. eating disorder
C. Helicobacter pylori gastritis
D. rumination syndrome

American Academy of Pediatrics 281


PREP ® Self-Assessment PREPSA 2021
Correct Answer: D
The girl in this vignette has rumination syndrome, which is characterized by effortless and
painless nonbilious, nonbloody regurgitation of food within minutes to hours of eating, after
which partially digested food is then rechewed, re-swallowed or expulsed (see Hyams JS, Di
Lorenzo C, Saps M, Shulman RJ, Staiano A, van Tilburg M). Neither nausea nor retching
usually precede regurgitation in rumination syndrome. Whereas pain is not a predominant feature
of rumination syndrome, dyspeptic symptoms such as epigastric or pharyngeal burning may
exist. Mild weight loss is relatively common in rumination syndrome, but other features
indicative of an eating disorder—such as malnutrition, dental enamel erosion, electrolyte
abnormalities, and vital sign abnormalities—are absent.

Although infections due to Helicobacter pylori may be asymptomatic, the bacterium has been
associated with dyspepsia, causing epigastric abdominal pain, bloating, nausea, and vomiting;
acute and chronic gastritis; and peptic ulcer disease. Complications can include gastrointestinal
bleeding, bowel perforation, and gastric outlet obstruction. Cyclic vomiting syndrome is a
diagnosis of exclusion characterized by recurrent, discrete stereotypical episodes of vomiting and
baseline health between episodes. Cyclic vomiting syndrome is often triggered by psychological
or infectious events (see Li BU, Lefevre F, Chelimsky GG, et al).

Medication therapy suppressing acid production, including the use of proton pump inhibitors,
provides little, if any, relief for rumination syndrome. The mainstay of treatment for rumination
syndrome is behavioral modification consisting of habit reversal. Diaphragmatic or abdominal
breathing techniques may be effective treatment because they compete with the urge to
regurgitate.

PREP Pearls
• Rumination syndrome should be considered in patients who experience effortless and
painless vomiting for 2 months.
• Rumination syndrome can be treated with behavioral therapy.

ABP Content Specifications(s)


• Recognize the clinical manifestations of rumination, and manage appropriately

American Academy of Pediatrics 282


PREP ® Self-Assessment PREPSA 2021
Suggested Readings
• Chial HJ, Camilleri M, Williams DE, Litzinger K, Perrault J. Rumination syndrome in
children and adolescents: diagnosis, treatment and prognosis. Pediatrics.
2003;111(1):158-162. doi:10.1542/peds.111.1.158.
• Hyams JS, Di Lorenzo C, Saps M, Shulman RJ, Staiano A, van Tilburg M. Functional
disorders: children and adolescents. Gastroenterology. 2016;150(6):1456-1468.
doi:10.1053/j.gastro.2016.02.015.
• Li BU, Lefevre F, Chelimsky GG, et al; North American Society for Pediatric
Gastroenterology, Hepatology, and Nutrition consensus statement on the diagnosis and
management of cyclic vomiting syndrome. J Pediatr Gastroenterol
Nutr. 2008;47(3):379-393. doi:10.1097/MPG.0b013e318173ed39.
• Lightdale JR. Gastroesophageal reflux disease. In: McInerny TK, Adam HM, Campbell
DE, DeWitt TG, Foy JM, Kamat DM, eds. American Academy of Pediatrics Textbook of
Pediatric Care. 2nd ed. Elk Grove Village, IL: American Academy of Pediatrics;
2017:2063-2075. Pediatric Care Online .
• Stanghellini V, Chan FK, Hasler WL, et al. Gastroduodenal disorders. Gastroenterology.
2016;150(6):1380-1392. doi:10.1053/j.gastro.2016.02.011.

American Academy of Pediatrics 283


PREP ® Self-Assessment PREPSA 2021
Question 76
A 4-year-old boy (Item Q76) is seen for a health supervision visit. He was born at full term to a
35-year-old mother. The mother reports that at age 30 years she underwent fertility treatments
because of premature ovarian failure. The boy has delayed speech and can say only 10 words. He
is very active and runs around the room during the entire visit. The mother expresses concern
about his behavior, frequent temper tantrums, hand flapping, and biting his 2-year-old sister. The
family history is significant for bipolar disorder in the maternal grandmother and Parkinson
disease in the maternal grandfather. The family history on the paternal side is unknown.

Item Q76: Boy described in the vignette.


Reprinted with permission from Saul RA, ed. Medical Genetics in Pediatric Practice. Elk Grove
Village, IL: American Academy of Pediatrics; 2013

Of the following, the MOST likely underlying etiology of this boy’s clinical disorder is
A. CAG repeats in HTT
B. CGG repeats in FMR1
C. CTG repeats in DMPK
D. GAA repeats in FXN

American Academy of Pediatrics 284


PREP ® Self-Assessment PREPSA 2021
Correct Answer: B
The condition seen in the family described in the vignette is FMR1-related disorder caused by
expansion of CGG repeats in the FMR1 gene. The CGG repeats are present in the promoter
region of the FMR1 gene located on the X-chromosome. The clinical picture depends on the
number of CGG repeats present in the individual (Item C76). Males with a full mutation (> 200
repeats) exhibit features of fragile X syndrome with developmental delay, behavioral issues such
as attention-deficit/hyperactivity disorder, and autism-spectrum disorder. Physical examination
findings include a facial profile with long face, prominent forehead, prominent jaw, and large
ears. Macroorchidism can be seen in postpubertal males. Females with full mutation can have a
variable presentation according to the degree of tissue-specific X-inactivation of the affected
allele.

Fragile X syndrome is the most common cause of X-linked intellectual disability. Males with
fragile X syndrome inherit an expanded FMR1 allele from their premutation carrier mother.
Males and females with CGG repeat size in the premutation range (55-200 repeats) are at risk of
developing fragile X–associated tremor/ataxia syndrome, which is characterized by progressive
cerebellar ataxia with intention tremor. Affected individuals can be misdiagnosed as having
Parkinson disease. Executive function deficits, dementia, peripheral neuropathy, and cognitive
decline can also be seen. In addition to fragile X–associated tremor/ataxia syndrome,
premutation carrier females are at risk of experiencing premature ovarian failure, exhibited as
infertility and early menopause, as in the patient’s mother in the vignette. Intermediate alleles

American Academy of Pediatrics 285


PREP ® Self-Assessment PREPSA 2021
(45-54 repeats) are unstable and are at risk of expansion into the premutation range when
transmitted by the mother. Previously, it was thought that females with intermediate allele were
at increased risk of experiencing premature ovarian failure, but this was not supported by
subsequent larger studies. FMR1-related disorders are diagnosed by CGG repeat analysis of the
gene. Management is aimed only at symptoms; no specific treatment exists for the condition.

The other response choices are incorrect. Expansion of CAG repeats in the HTT gene cause
Huntington disease, CTG repeats in the DMPK gene cause type 1 myotonic dystrophy, and GAA
repeats in the FXN gene are associated with Friedreich ataxia.

PREP Pearls
• Fragile X syndrome is caused by expansion of CGG repeats in the FMR1 gene.
• Fragile X syndrome is the most common cause of X-linked intellectual disability.
• Both male and female premutation carriers are at increased risk of experiencing fragile
X–associated tremor/ataxia syndrome. Female premutation carriers are at increased risk
of experiencing premature ovarian failure.

ABP Content Specifications(s)


• Know the genetic abnormalities and phenotypic expression associated with fragile X
syndrome

Suggested Readings
• Hersh JH, Saul RA; Committee on Genetics. Health supervision for children with fragile
X syndrome. Pediatrics. 2011;127(5):994-1006. doi:10.1542/peds.2010-3500.
• Raspa M, Wheeler AC, Riley C. Public health literature review of fragile-X
syndrome. Pediatrics. 2017;139(suppl 3):S153-S171. doi:10.1542/peds.2016-1159C.
• Saul RA. Fragile X syndrome. In: McInerny TK, Adam HM, Campbell DE, DeWitt TG,
Foy JM, Kamat DM, eds. American Academy of Pediatrics Textbook of Pediatric Care.
2nd ed. Elk Grove Village, IL: American Academy of Pediatrics; 2017:2031-
2035. Pediatric Care Online .

American Academy of Pediatrics 286


PREP ® Self-Assessment PREPSA 2021
Question 77
A 2-year-old girl with no underlying medical illness is brought to the emergency department for
evaluation of a painful left forearm 12 hours after she was scratched by a pet cat. She has a
temperature of 38.2°C and a puncture mark on her left forearm. There is erythema, mild
swelling, and tenderness to touch in the area of the puncture mark.

Of the following, the BEST choice of antimicrobial therapy for this girl is oral
A. amoxicillin-clavulanate
B. cephalexin
C. clindamycin
D. trimethoprim-sulfamethoxazole

American Academy of Pediatrics 287


PREP ® Self-Assessment PREPSA 2021
Correct Answer: A
The girl described in the vignette has a clinical picture suggestive of cellulitis of the left forearm
after a cat scratch. In the setting of cat scratch or bite, the most frequent pathogen is Pasteurella
multocida. Amoxicillin-clavulanate is the antimicrobial agent of choice for the treatment of
infected cat bite wounds, given its excellent coverage against Pasteurella and other aerobic-
anaerobic bacteria often found in animal bites.

Pasteurella is a facultative anaerobic gram-negative coccobacillus. The principal reservoir


of Pasteurella species is the oral flora of many animals, especially cats (70%-90%) and dogs
(25%-50%). Pasteurella multocida causes most human infections, followed by other species.
Infection typically occurs after a bite, scratch, or lick from cats, dogs, or other animals. In
addition to Pasteurella, a variety of mixed aerobic-anaerobic bacteria may be found in infected
animal bite wounds: Staphylococcus aureus, Streptococcus spp, anaerobes,
Capnocytophaga spp, Moraxella spp, Corynebacterium spp, and Neisseria spp.

Risk factors for severe infection are immunodeficiency, receipt of solid organ transplants, or
liver disease. Although unusual, transmission of Pasteurella from human to human has been
reported via vertical transmission from mother to baby, horizontal transmission from colonized
individuals, and receipt of contaminated blood products.

Patients may present with progressive skin and soft-tissue inflammation at the site of a bite or
scratch of a cat. The onset of cellulitis is usually within 24 hours of the injury. Regional
adenopathy in conjunction with systemic symptoms such as fever and chills may be present. The
clinical course may be complicated by the development of soft-tissue abscesses, tenosynovitis,
and, rarely, septic arthritis and osteomyelitis. Invasive disease including bacteremia, sepsis,
meningitis, ocular infection, pneumonia, endocarditis, peritonitis, and pyelonephritis has been
documented.

The organism may be isolated from wound or tissue culture, often as part of polymicrobial
bacterial infection. Cultures for aerobic and anaerobic bacteria should be obtained from infected
wounds. Blood cultures may be indicated in patients with fever and other signs of systemic
infection or in high-risk patients (eg, immune deficiency).

Treatment of choice for animal bite infections include amoxicillin-clavulanate in the outpatient
setting or ampicillin-sulbactam in the inpatient setting for severe, complicated infections.
Amoxicillin-clavulanate is active against Pasteurella spp and other aerobic and anaerobic
bacteria implicated in animal bite infections. Pasteurella is usually susceptible to penicillin, but
rarely β-lactamase–producing strains have been reported. Antistaphylococcal penicillins (eg,
cephalexin) are not active against Pasteurella spp. Pasteurella is resistant to clindamycin and
vancomycin. In patients allergic to penicillin, alternative antibiotic regimens include TMP-SMX
or azithromycin, but data on the use of these agents are limited. Doxycycline is considered as an
alternative agent for Pasteurella multocida infection.

The duration of therapy for bite infection is determined by location, severity, and isolated
pathogen. Cellulitis is usually treated for 10 to 14 days. Extended treatment up to 3 weeks is
recommended for tenosynovitis, 4 weeks for septic arthritis, and 6 weeks for osteomyelitis. The
American Academy of Pediatrics 288
PREP ® Self-Assessment PREPSA 2021
general and antimicrobial management of animal or human bite wounds is summarized in Item
C77A and Item C77B, respectively.

American Academy of Pediatrics 289


PREP ® Self-Assessment PREPSA 2021

PREP Pearls
• The most frequently implicated pathogen in infected cat bite infections is Pasteurella
multocida; in addition to Pasteurella, a variety of mixed aerobic-anaerobic bacteria may
be found in infected animal bite wounds.
• The most common presentation of Pasteurella infection in children is cellulitis at the site
of a bite or scratch of a cat; the onset of cellulitis is usually within 24 hours of the injury.
• Treatment of choice for animal bite infections in the outpatient setting is amoxicillin-
clavulanate.

ABP Content Specifications(s)


• Recognize the common clinical features associated with Pasteurella multocida infection
• Plan appropriate management for a patient with Pasteurella multocida infection
• Understand the epidemiology of Pasteurella multocida

American Academy of Pediatrics 290


PREP ® Self-Assessment PREPSA 2021
Suggested Readings
• American Academy of Pediatrics. Pasteurella infections. In: Kimberlin DW, Brady MT,
Jackson MA, Long SS, eds. Red Book: 2018-2021 Report of the Committee on Infectious
Diseases. 31st ed. Elk Grove Village, IL: American Academy of Pediatrics; 2018:606-
607 .
• American Academy of Pediatrics. Bite wounds. In: Kimberlin DW, Brady MT, Jackson
MA, Long SS, eds. Red Book: 2018-2021 Report of the Committee on Infectious
Diseases. 31st ed. Elk Grove Village, IL: American Academy of Pediatrics; 2018:189-
195.
• Bula-Rudas FJ, Olcott JL. Human and animal bites. Pediatr Rev. 2018;39(10):490-500.
doi:10.1542/pir.2017-0212.
• Herendeen NE, Szilagyi PG. Animal and human bites. In: McInerny TK, Adam HM,
Campbell DE, DeWitt TG, Foy JM, Kamat DM, eds. American Academy of Pediatrics
Textbook of Pediatric Care. 2nd ed. Elk Grove Village, IL: American Academy of
Pediatrics; 2017:1716-1718. Pediatric Care Online .

American Academy of Pediatrics 291


PREP ® Self-Assessment PREPSA 2021
Question 78
A 7-year-old boy is having a follow-up visit for attention-deficit/hyperactivity disorder (ADHD).
His parents ask about a newly publicized device for the treatment of hyperactive and impulsive
symptoms in children with ADHD. They are excited about the possibility of an effective
nonpharmacologic treatment and would like to know about the benefits and harms of this
intervention.

Of the following, the study design MOST likely to provide the desired information is a
A. case-control study
B. cohort study
C. cross-sectional study
D. randomized controlled trial

American Academy of Pediatrics 292


PREP ® Self-Assessment PREPSA 2021
Correct Answer: D
A randomized controlled trial (RCT) would best provide information about the effectiveness of
the attention-deficit/hyperactivity disorder treatment being considered by the parents in the
vignette. Randomized controlled trials are considered the “gold standard” study design for an
intervention; a well-designed and well-managed study can provide a strong level of evidence.
Often RCTs are used to compare the effectiveness of a new treatment with a currently accepted
treatment. By randomly assigning study participants to the experimental group or control group,
confounders should be balanced such that the 2 groups should be equivalent at baseline. Thus,
any differences at the end of the study (eg, positive and negative outcomes such as benefits and
harms) should be the result of differences in treatment.

A case control study can be used with small sample sizes and is particularly useful for studying
rare conditions. Risk factors or exposures are compared between cases (study subjects with the
condition) and controls (study subjects without the condition). This study design can look at risk
factors for rare conditions but cannot be used to determine incidence or prevalence.
A cohort study follows a group of people over time, and is therefore useful for studying events
that happen in sequence. A cohort study can examine current exposure and future outcome
(prospective cohort study) or past exposure and current outcome (retrospective cohort study). By
following a group of patients with the condition over time, a prospective cohort study can
provide information about a condition’s natural history and prognosis.

A cross-sectional study can be used to study potential associations between exposures and
outcomes at a point in time. Because these factors are measured at the same time, a cross-
sectional study cannot be used to determine cause. A cross-sectional study can also provide
information about the accuracy of diagnostic tests (eg, sensitivity, specificity) by comparing how
a new test performs relative to a reference or gold standard test in identifying those with the
target condition.

PREP Pearls
• A randomized controlled trial is the study design most likely to yield valid information
about the benefits and/or harms of an intervention.
• A cohort study is the study design most likely to yield valid information about the
prognosis of a condition.
• A cross-sectional study is the study design most likely to yield valid information about
the accuracy of a diagnostic test.

ABP Content Specifications(s)


• Identify the study design most likely to yield valid information about the accuracy of a
diagnostic test
• Identify the study design most likely to yield valid information about the prognosis of a
condition
• Identify the study design most likely to yield valid information about the benefits and/or
harms of an intervention

American Academy of Pediatrics 293


PREP ® Self-Assessment PREPSA 2021
Suggested Readings
• Hernandez RG, Rowe PC. Research and statistics: cohort studies. Pediatr
Rev. 2009;30(9):364-365. doi: 10.1542/pir.30-9-364.
• Johnson SL. Research and statistics: a question of time: cross-sectional versus
longitudinal study designs. Pediatr Rev. 2010;31(6):250-251. doi: 10.1542/pir.31-6-250.
• Palaia A. Research and statistics: study design and data sources. Pediatr Rev.
2013;34(8):371-372. doi: 10.1542/pir.34-8-371.
• Perry-Parrish C, Dodge R. Research and statistics: validity hierarchy for study design and
study type. Pediatr Rev. 2010;31(1):27-29. doi: 10.1542/pir.31-1-27.
• Sibinga EMS, Jennings JM. Research and statistics: strengths and limitations of
randomized, controlled trials. Pediatr Rev. 2010;31(7):296-297. doi: 10.1542/pir.31-7-
296.
• Upadhya K, Rowe P. Research and statistics: case-control studies. Pediatr
Rev. 2010;31(2):70-71. doi: 10.1542/pir.31-2-70.

American Academy of Pediatrics 294


PREP ® Self-Assessment PREPSA 2021
Question 79
A 2-year-old previously healthy boy is being evaluated for a 3-day history of cough and
congestion. The boy developed a fever up to 38.3°C 2 days ago for which he has been receiving
acetaminophen as needed; his last dose was yesterday at bedtime. This morning, his congestion
became thick and green. He has had emesis associated with cough, but no diarrhea. He has
continued to drink adequately but is not eating well.

On physical examination, the boy’s vital signs include a temperature of 37.8°C, a heart rate of
120 beats/min, a respiratory rate of 20 breaths/min, and an oxygen saturation of 97% on room air
via pulse oximetry. He is quiet and cooperative. He has no drainage from his eyes, his tympanic
membranes are gray with clear fluid, and he has thick nasal discharge bilaterally. His heart has a
regular rate and rhythm; his lungs are clear, with no increased work of breathing.

Of the following, the MOST likely diagnosis is


A. acute bacterial sinusitis
B. acute otitis media
C. bacterial lower respiratory infection
D. viral upper respiratory infection

American Academy of Pediatrics 295


PREP ® Self-Assessment PREPSA 2021
Correct Answer: D
Viral upper respiratory tract infections (URI) are among the most common reasons for visits to a
pediatric office. Children younger than 2 years who attend daycare may have as many as 14
URIs in a year. Older children and adults may have up to 6 URIs per year. Understanding the
natural course of the illness is important to help distinguish a viral URI from a secondary
bacterial infection, which is less common.

Typically, a viral URI begins with a sore throat, followed by congestion or rhinorrhea and a
cough. Fever is common, especially in the first 3 days of the illness. The symptoms usually peak
between the third and fifth day of illness. Symptoms may linger for up to 2 weeks, especially the
cough, which is the last symptom to improve.

Several viruses may cause a URI, with enterovirus and rhinovirus being the most common. Three
viruses often peak each winter season in the United States: respiratory syncytial virus first,
followed by influenza and then human metapneumovirus. Parainfluenza viruses also cause URIs
and are associated with croup. Adenovirus is also common and frequently presents with the
additional symptoms of conjunctivitis, gastroenteritis, or both.

Bacterial sinusitis is present in 5% to 10% of cases of URI; it is distinguished from a simple viral
URI by at least 10 days of symptoms without improvement, a new or worsening fever (> 39°C),
or worsening cough after initial improvement. A thorough history should be obtained to help
distinguish between bacterial sinusitis and a new viral URI. Acute otitis media is more frequently
seen in children with viral URI, affecting up to 36% of those younger than 12 months and 15%
of those between 24 and 36 months of age. On otoscopic examination, affected children have
bulging tympanic membranes with opaque fluid and loss of landmarks. The diagnosis of
bacterial lower respiratory tract infection (in children, most commonly community-acquired
pneumonia) can be made clinically. The child usually has persistent fevers; may appear ill, be
vomiting, or both; have decreased oxygen saturation levels; and have focal crackles or
asymmetric breath sounds on auscultation.

PREP Pearls
• Viruses are the most common cause of upper respiratory tract infections.
• Viral upper respiratory tract infections typically improve within 14 days.
• Antibiotics are not indicated for viral upper respiratory tract infections.

ABP Content Specifications(s)


• Identify the etiology of an upper respiratory tract infection
• Understand the natural history of an upper respiratory tract infection

Suggested Readings
• Centers for Disease Control and Infection. Nonspecific upper respiratory tract
infection. https://www.cdc.gov/getsmart/community/materials-references/print-
materials/hcp/adult-tract-infection.pdf.
• Weintraub B. Upper respiratory tract infections. Pediatr Rev. 2015;36(12):554-556.
doi: 10.1542/pir.36-12-554.

American Academy of Pediatrics 296


PREP ® Self-Assessment PREPSA 2021
Question 80
A 2-day-old male newborn is transferred to the neonatal intensive care unit after developing
petechiae. He was born via spontaneous vaginal delivery at term after an uncomplicated
pregnancy. His mother experienced postpartum hemorrhage, and a laboratory evaluation at the
time revealed that she had a platelet count of 15 × 103/µL (15 × 109/L). One month ago, she had
a complete blood cell count with normal findings.

Laboratory data for the newborn are shown:


Laboratory Test Result
White blood cell count 12,500/µL (12.5 × 109/L)
Hemoglobin 18.2 g/dL (182 g/L)
Platelet count 12 × 103/µL (12 × 109/L)

Of the following, the MOST appropriate next step in treatment is to administer


A. anti-D monoclonal antibody
B. intravenous immune globulin
C. maternal platelets
D. methylprednisolone

American Academy of Pediatrics 297


PREP ® Self-Assessment PREPSA 2021
Correct Answer: B
The infant in the vignette has severe thrombocytopenia. Perinatal thrombocytopenia can be
productive (eg, a genetic bone marrow failure syndrome such as congenital amegakaryocytic
thrombocytopenia), consumptive (eg, disseminated intravascular coagulation), or destructive (ie,
maternal alloantibodies to the neonate’s platelets). Given the mother’s new onset of severe
thrombocytopenia, the most likely cause of the neonate’s thrombocytopenia is a maternal
immune thrombocytopenic purpura (ITP). Maternal ITP results from the passive transplacental
transmission of a maternally derived antibody to a surface antigen found on both maternal and
neonatal platelets, resulting in thrombocytopenia in both the mother and the neonate. This can be
contrasted with neonatal alloimmune thrombocytopenia (NAIT), which occurs when there is
passive transplacental transmission of a maternally derived alloantibody to an antigen found on
both paternal and neonatal platelets. Neonatal alloimmune thrombocytopenia will result in
thrombocytopenia in the neonate, but not in the mother. Both maternal ITP and NAIT are self-
limiting, requiring only temporary support. Transmission of the alloantibody in both cases ceases
at birth, and the antibody titer drops as it binds to neonatal platelets. Maternal ITP can be
temporized through the administration of intravenous immune globulin.

Anti-D is an antibody to the Rhesus (Rh) antigen on red blood cells. It should be administered to
all Rh-negative women during pregnancy. When there is an Rh mismatch between the maternal
and fetal red blood cells such that the fetal red blood cells express the Rh antigen but maternal
red blood cells do not, there is a risk of alloantigen sensitization in the mother, increasing the risk
for hemolytic disease of the newborn in subsequent pregnancies. The purpose of administering
anti-D to the mother after delivery is to clear any fetal Rh+ red blood cells from the maternal
circulation, thereby minimizing the risk of alloantigen sensitization.

Maternal platelets are an effective therapy for severe thrombocytopenia due to NAIT (see
above). As the alloantibody in NAIT is specific for a paternal and neonatal platelet antigen but
not for a maternal antigen, maternal platelets would have a normal lifespan if administered to the
neonate. They would not be an effective therapy for maternal ITP, because the alloantibody
binds to antigens on maternal platelets, thereby eliminating them from circulation when
administered to the neonate.

Methylprednisolone, or any other immunosuppressant agent, would not be an effective therapy


for maternal ITP or NAIT because these are not autoimmune disorders in the neonate, but rather
are caused by passively transmitted maternal antibody.

PREP Pearls
• Maternal immune thrombocytopenic purpura results from the passive transplacental
transmission of a maternally derived antibody to a surface antigen found on both
maternal and neonatal platelets, resulting in thrombocytopenia in both the mother and the
neonate.
• Maternal immune thrombocytopenic purpura can be temporized through the
administration of intravenous immune globulin.
• Methylprednisolone, or any other immunosuppressant agent, would not be an effective
therapy for maternal immune thrombocytopenic purpura or neonatal alloimmune

American Academy of Pediatrics 298


PREP ® Self-Assessment PREPSA 2021
thrombocytopenia, because these are not autoimmune disorders in the neonate, but rather
caused by passively transmitted maternal antibody.

MOCA-Peds Objective
• Evaluate and manage a patient with the new onset of thrombocytopenia.

ABP Content Specifications(s)


• Recognize the clinical and laboratory features of an infant whose mother has ITP, and
manage appropriately

Suggested Readings
• Baer VL, Lambert DK, Henry E, Christensen RD. Severe thrombocytopenia in the
NICU. Pediatrics. 2009;124(6):e1095-1100. doi:10.1542/peds.2009-0582.
• Gunnink SF, Vlug R, Fijnvandraat K, van der Bom JG, Stanworth SJ, Lopriore E.
Neonatal thrombocytopenia: etiology, management and outcome. Expert Rev Hematol.
2014;7(3):387-395. doi:10.1586/17474086.2014.902301.
• Kamphuis MM, Paridaans NP, Porcelijn L, Lopriore E, Oepkes D. Incidence and
consequences of neonatal alloimmune thrombocytopenia: a systematic
review. Pediatrics. 2014;133(4):715-721. doi:10.1542/peds.2013-3320.
• Winkelhorst D, Oepkes D, Lopriore E. Fetal and neonatal alloimmune thrombocytopenia:
evidence based antenatal and postnatal management strategies. Expert Rev
Hematol. 2017;10(8):729-737. doi:10.1080/17474086.2017.1346471.

American Academy of Pediatrics 299


PREP ® Self-Assessment PREPSA 2021
Question 81
A mother calls the pediatrician to report that her 5-year-old daughter revealed last night that her
12-year-old brother has been coming into her room at night and asking her to touch his
“privates.” When questioned by his mother, the boy denies that this has happened. The mother
plans to sleep in the same room as her daughter until an evaluation is completed.

Of the following, in addition to scheduling an appointment for this child, the MOST appropriate
next step is to
A. advise the mother that the brother should be immediately removed from the home to stay
with relatives
B. reassure the mother that she has taken all necessary steps to ensure the safety of her
children
C. report the situation to local child protective services
D. report the situation to local police

American Academy of Pediatrics 300


PREP ® Self-Assessment PREPSA 2021
Correct Answer: C
As a mandated reporter, the pedatrician in the vignette is required to report suspected child abuse
to local child protective services (CPS). Removing the brother from the home may be necessary
once more information is gathered by investigators, but is not the best next step. In some
situations, removal of a child perpetrator from the family may be detrimental to the child and the
family unit. Although the mother has taken steps to ensure her daughter’s safety by sleeping in
her room, a report and thorough investigation is necessary. Law enforcement may be involved in
some physical or sexual abuse investigations, but reporting to CPS is the first step.

Each state has slightly different child abuse reporting laws. The vast majority require that
mandated reporters (eg, medical and mental health professionals, teachers, child care providers,
social workers, law enforcement personnel) report known cases of child abuse and cases where
the individual has a “reasonable suspicion” that abuse has occurred. A summary of state child
abuse reporting laws is available online at www.childwelfare.gov (Child Welfare Information
Gateway).

When a child presents with an injury or physical examination finding that is concerning for
abuse, a complete history should be obtained and physical examination performed. When sexual
abuse is disclosed or alleged, it is usually not necessary for the clinician to hear directly from the
child about the abuse; in fact, it is typically best to defer questioning to the trained forensic
interviewer. Parents should be notified when a report is made to CPS, although the optimal
timing, location, and method of disclosure is dependent on the situation.

PREP Pearls
• As mandated reporters, pedatricians are required to report known or suspected child
abuse to local child protective services.
• When sexual abuse is disclosed or alleged, it is usually not necessary for the clinician to
hear directly from the child about the abuse; in fact, it is typically best to defer
questioning to the trained forensic interviewer.

MOCA-Peds Objective
• Respond appropriately to a child’s disclosure of sexual abuse.

ABP Content Specifications(s)


• Provide apporpriate guidance and support to a family during an investigation of child
abuse or neglect
• Understand the physician’s duty and ethical obligation to report suspected child abuse or
neglect

American Academy of Pediatrics 301


PREP ® Self-Assessment PREPSA 2021
Suggested Readings
• Asnes AG, Leventhal JM. Managing child abuse: general principles. Pediatr
Rev. 2010:31(2):47-55. doi: 10.1542/pir.31-2-47.
• Chiesa A, Goldson E. Child sexual abuse. Pediatr Rev. 2017:38(3):105-118.
doi: 10.1542/pir.2016-0113.
• Jenny C, Crawford-Jakubiak JE; Committee on Child Abuse and Neglect; American
Academy of Pediatrics. The evaluation of children in the primary care setting when
sexual abuse is suspected. Pediatrics. 2013;132(2):e558-e567. doi: 10.1542/peds.2013-
1741.
• US Department of Health & Human Services. Child Welfare Information
Gateway. https://www.childwelfare.gov.

American Academy of Pediatrics 302


PREP ® Self-Assessment PREPSA 2021
Question 82
A 16-year-old adolescent is being evaluated for concerns about 6 months of worsening fatigue,
dizziness, nausea, and abdominal pain. The girl describes daily generalized abdominal pain with
nausea, which is worse with dairy intake. She has had intermittent nonbilious nonbloody emesis.
She reports no diarrhea, constipation, hematochezia, oily-appearing stools, or significant weight
loss. She has a history of well-controlled asthma and normal menses. Physical examination
findings demonstrate a blood pressure of 122/58 mm Hg, heart rate of 91 beats/min, weight of 46
kg (11th percentile), height of 154 cm (8th percentile), and body mass index of 19 kg/m2 (33rd
percentile for age). She is very pale. The remainder of her findings are within normal limits.

Laboratory data are as follows:


White blood cell 5,110/µL (5.11 × 109/L)
Hemoglobin 4.4 g/dL (44 g/L)
Hematocrit 17.5%
Mean corpuscular volume 64 fL
Platelets 842 × 103/µL (842 × 109/L)
Erythrocyte sedimentation rate <1 mm/h
Alanine aminotransferase 26 U/L
Aspartate aminotransferase 24 U/L
Albumin 3.8 g/dL (38 g/L)

Upper endoscopy is performed (Item Q82). Biopsy specimens obtained from the small intestine
demonstrate severe villous atrophy with increased intraepithelial lymphocytes.

American Academy of Pediatrics 303


PREP ® Self-Assessment PREPSA 2021

Item Q82: Small intestine findings. Courtesy of J Sullivan

Of the following, the test MOST likely to confirm this adolescent’s diagnosis is
A. fecal elastase 1
B. stool polymerase chain reaction for detection of Shigella
C. sweat chloride
D. tissue transglutaminase antibody

American Academy of Pediatrics 304


PREP ® Self-Assessment PREPSA 2021
Correct Answer: D
The adolescent in the vignette has celiac disease, and tissue transglutaminase antibody (tTG-IgA)
levels are recommended as initial testing. Tissue transglutaminase antibody is highly sensitive
(90%-100%) and specific (95%-100%) for celiac disease in individuals with normal total IgA
levels, therefore, initial testing for celiac disease should include both tTG-IgA and total IgA
levels. Celiac “panels,” which may include other serologies, such as tTG-IgG antibody,
antiendomysial antibody, antigliadin antibody, and deamidated gliadin peptide antibody, are not
recommended for initial diagnosis, but could be useful in individuals with IgA deficiency or
indeterminate testing results. Diagnosis should be confirmed with upper gastrointestinal tract
endoscopy and biopsies. Histopathologic findings of celiac disease include villous atrophy with
intraepithelial lymphocytosis.

Typically, celiac disease is associated with gastrointestinal symptoms (most commonly diarrhea,
bloating, flatulence) and failure to thrive. It is important to recognize that individuals with celiac
disease may also exhibit other symptoms including constipation and transaminitis as well as
extraintestinal symptoms (anemia, dermatitis herpetiformis, dental enamel hypoplasia, recurrent
aphthous ulcers, arthritis/arthralgia, and headache). Finally, individuals with celiac disease may
be asymptomatic.

Evaluation for celiac disease should be performed for first-degree relatives of individuals with
celiac disease and in individuals with autoimmune thyroid disease, autoimmune liver disease,
trisomy 21, type 1 diabetes mellitus, Williams syndrome, Turner syndrome, and IgA deficiency.
In all individuals, except those with IgA deficiency, both tTG-IgA and total IgA levels should be
obtained. Individuals who are IgA deficient should be evaluated with tTG-IgG, endomysial
antibody-IgG, or deamidated gliadin peptide antibody-IgG. Diagnosis should be confirmed with
upper gastrointestinal tract endoscopy and biopsies.

Individuals with celiac disease should adhere to a strict gluten-free diet, avoiding gluten-
containing grains including wheat, barley, and rye; nutritional counseling and education is
critical, because gluten may be present in food sources, which may be less obvious to the child
and family (eg, broths/soup bases, soy sauce, and over-the-counter medicines).

Fecal elastase 1 is a test to help diagnose pancreatic insufficiency. Although pancreatic


insufficiency could cause abdominal pain, it typically presents with steatorrhea (malodorous, oily
stools) and would not cause villous atrophy. Shigella species infect the large intestine and
generally present with bloody diarrhea, therefore the duodenal histologic findings and clinical
history of the adolescent in the vignette make this unlikely. Sweat chloride testing would be
helpful for diagnosing cystic fibrosis, but would not be diagnostic for celiac disease.

PREP Pearls
• Celiac testing should be considered in individuals with gastrointestinal symptoms
(including abdominal pain, diarrhea, and bloating) and extraintestinal symptoms
(including anemia, poor growth, and dermatitis herpetiformis).
• Initial screening for celiac disease should include both tissue transglutaminase antibody
and total IgA testing.

American Academy of Pediatrics 305


PREP ® Self-Assessment PREPSA 2021

ABP Content Specifications(s)


• Recognize the clinical features associated with celiac disease
• Plan appropriate dietary management for a patient with celiac disease
• Plan the appropriate diagnostic evaluation for celiac disease

Suggested Readings
• Ediger TR, Hill ID. Celiac disease. Pediatr Rev. 2014;35(10):409-416.
doi: 10.1542/pir.35-10-409.
• Hill ID, Fasano A, Guandalini S, et al. NASPGHAN clinical report on the diagnosis and
treatment of gluten-related disorders. J Pediatr Gastroenterol Nutr. 2016;63:156-165.
doi: 10.1097/MPG.0000000000001216.
• Safta AM, Kerner JA. Gluten-sensitive enteropathy (celiac sprue). In: McInerny TK,
Adam HM, Campbell DE, DeWitt TG, Foy JM, Kamat DM, eds. American Academy of
Pediatrics Textbook of Pediatric Care. 2nd ed. Itasca, IL: American Academy of
Pediatrics; 2016;chap 259:2097-2103. Pediatric Care Online.

American Academy of Pediatrics 306


PREP ® Self-Assessment PREPSA 2021
Question 83
A neonate born at term is being evaluated in the newborn nursery. She was born overnight via
vaginal delivery to a 27-year-old gravida 1, para 0 woman with a history of asthma. The
mother’s prenatal records report group B Streptococcus–negative status and a positive VDRL
test (1:2) result with positive treponemal test. The neonate has unremarkable physical
examination findings. She is breastfeeding well.

Of the following, the BEST next diagnostic test for this neonate is
A. direct fluorescent antibody
B. Treponema pallidum enzyme immunoassay
C. rapid plasma reagin
D. treponemal polymerase chain reaction

American Academy of Pediatrics 307


PREP ® Self-Assessment PREPSA 2021
Correct Answer: C
For the neonate in the vignette, born to a mother with confirmed syphilis, a nontreponemal test
such as rapid plasma reagin (RPR) should be performed to obtain titers on the neonate.
Treponemal tests, such as Treponema pallidum enzyme immunoassay (TP-EIA) or T
pallidum chemiluminescent assay (TP-CIA), are used to confirm infection with T pallidum.
However, treponemal tests are not used in the diagnostic evaluation of neonates born to mothers
with syphilis infection because passive maternal transfer of antibodies would lead to positive
results. Direct fluorescent antibody tests are no longer available in the United States. Though
treponemal polymerase chain reaction testing is available, it is not used clinically.

Rates of congenital syphilis, caused by the spirochete T pallidum, are rising. Detection and
treatment during pregnancy reduces maternal morbidity and prevents neonatal transmission.
Women should be screened for syphilis early in pregnancy. Those who test negative should
undergo repeat syphilis screening at the time of delivery to identify infection later in pregnancy.

Conventional screening for syphilis may be performed with nontreponemal serologic tests such
as the VDRL or RPR tests. These tests detect the presence of antibodies against cardiolipin-
cholesterol-lecithin complex. Both RPR and VDRL may be falsely positive in the setting of viral
infection, lymphoma, tuberculosis, malaria, endocarditis, connective tissue disease, or substance
abuse. Both RPR and VDRL may be falsely negative early in cases of congenital syphilis, early
primary syphilis, and late congenital syphilis.

Some laboratories have adopted a “reverse sequence screening” approach; pregnant women are
initially screened with a treponemal specific test such as TP-EIA or TP-CIA. Women with a
positive treponemal specific test are subsequently tested using a nontreponemal serologic test
such as RPR or VDRL to obtain titers. If test results are discordant, that is, the treponemal test
result is positive but the nontreponemal test result is negative, a different treponemal test should
be performed. The reverse sequence screening approach may have higher false-positive rates in
communities with a low prevalence of syphilis. An algorithm for both conventional and reverse
sequence testing is presented in Item C83.

With adequate maternal treatment of primary or secondary syphilis infection, the risk of
congenital syphilis can be markedly reduced. Either RPR or VDRL titers may be followed
during pregnancy to confirm adequate response to treatment. Ideally, this testing should be
performed at the same laboratory. A 4-fold decrease in titers reflects adequate treatment; a 4-fold
increase in titers suggests new infection or relapse.

American Academy of Pediatrics 308


PREP ® Self-Assessment PREPSA 2021
PREP Pearls
• There are 2 approaches to screening pregnant women for syphilis:
o Conventional screening: Screen with rapid plasma reagin (RPR) or VDRL tests,
which are nontreponemal serologic tests. Positive results are confirmed with a
treponemal test such as Treponema pallidum enzyme immunoassay (TP-EIA).
o Reverse screening: Screen with treponemal specific tests such as TP-EIA. If
positive, RPR or VDRL is sent to obtain titers.
• A 4-fold decrease in RPR or VDRL titers suggests an appropriate response to treatment; a
4-fold increase in titers indicates inadequate treatment or new infection.
• For a neonate born to a mother with confirmed syphilis, a nontreponemal test such as the
RPR test should be performed to obtain titers on the neonate.

ABP Content Specifications(s)


• Plan the management of a neonate whose mother has abnormal prenatal laboratory
findings

Suggested Readings
• American Academy of Pediatrics. Summary of infectious disease. Syphilis. In: Kimberlin
DW, Brady MT, Jackson MA, Long SS eds. Red Book: 2018-2021 Report of the
Committee on Infectious Diseases. Elk Grove Village, IL: American Academy of
Pediatrics; 2018:773-788.
• Butterfield R. Syphilis. Pediatr Rev. 2014;35(5):212-213. doi: 10.1542/pir.35-5-212.
• Kojaoghlanian T. The newborn at risk for infection. In: McInerny TK, Adam HM,
Campbell DE, DeWitt TG, Foy JM, Kamat DM, eds. American Academy of Pediatrics
Textbook of Pediatric Care. 2nd ed. Itasca, IL: American Academy of Pediatrics;
2016;chap 102:899-908. Pediatric Care Online.

American Academy of Pediatrics 309


PREP ® Self-Assessment PREPSA 2021
Question 84
An 8-year-old boy is brought to the office for evaluation of nighttime bedwetting. He has never
been dry at night and has urinary accidents 5 nights per week. The boy’s parents report that he
has been completely continent during the day since 3 years of age. He has no history of urinary
frequency, urgency, or hesitancy. He has a normal daily bowel movement and has not had fecal
soiling. He is described as a deep sleeper. He is embarrassed to sleep at his cousin’s house. His
father also had nighttime bedwetting until he was 9 years old. The boy’s parents decreased his
fluid intake before bedtime and tried bedwetting alarms, but with no improvement in the boy’s
frequency of nighttime bedwetting.

He is at the 50th percentile for weight and height. He has no skin lesions on his back. His
abdomen is soft with no organomegaly. His neurological examination findings are normal.
A urinalysis performed in the office has normal results. A 48-hour voiding diary shows that the
boy voids 4 to 6 times per day with a normal maximum urine volume of 300 mL.

If pharmacological therapy is pursued, the BEST initial agent is


A. desmopressin
B. imipramine
C. oxybutynin
D. polyethylene glycol

American Academy of Pediatrics 310


PREP ® Self-Assessment PREPSA 2021
Correct Answer: A
The boy in this vignette has nighttime incontinence, a family history of bedwetting, an absence
of urinary symptoms or constipation, normal physical examination findings, and a normal
urinary bladder capacity, all of which favor a diagnosis of primary monosymptomatic nocturnal
enuresis. A trial of decreasing fluids at bedtime and use of bedwetting alarms has been
unsuccessful. The first-line pharmacological treatment option for this child is desmopressin.

Urinary incontinence is defined as involuntary leakage of urine, which can be either continuous
(caused by anatomical abnormalities such as ectopic ureter) or intermittent (a functional
disturbance). The leakage of urine can occur during daytime while awake (daytime incontinence)
or mostly at night during sleep (nocturnal enuresis [NE]). Nocturnal enuresis is classified as
primary (never been dry at night) and secondary (dry period of 6 months or longer).

Nocturnal enuresis can be further classified into primary monosymptomatic nocturnal enuresis
(PMNE) and nonmonosymptomatic nocturnal enuresis (NMNE). Primary monosymptomatic
nocturnal enuresis is defined as enuresis without symptoms of lower urinary tract or bladder
dysfunction. Nonmonosymptomatic nocturnal enuresis is defined as enuresis with symptoms of
lower urinary tract or bladder dysfunction, as seen in children with lower urinary tract infections,
anatomic abnormalities, or neurogenic bladder.

Nocturnal enuresis is a common condition, with a prevalence of 10% to 15% in children by 6


years of age. The majority of children outgrow the condition by the teenage years; 1% or 2%
may continue to have NE during adulthood. Nocturnal enuresis is more common in boys and in
children with a parent or parents who had similar symptoms. A detailed history is required to
accurately characterize the type of NE and should include questions about the following: number
of nighttime bedwetting episodes per week, perceived nocturnal volume, child’s bedtime routine
and fluid consumption, whether the child has had a dry period of 6 months or longer, daytime
incontinence, frequency, urgency, urinary retention, and other features of voiding dysfunction. A
thorough history regarding stooling habits, fecal incontinence, and constipation is also required.
A history of difficulty in being aroused from sleep, snoring, and sleep apnea suggests the reason
for NE. It is also imperative to inquire regarding the psychosocial stressors in children with NE,
such as the birth of a new sibling, parents’ divorce, the death of a relative, or school difficulties.

A complete physical examination that includes the genitalia and perianal areas is essential in the
evaluation of NE. In a child with PMNE, the physical examination findings are usually normal.
In a child with NMNE, a complete genital examination may reveal continuous leaking into the
perineum, suggestive of an ectopic ureter, labial adhesions, or meatal stenosis. Fecal soiling and
anal fissures and tears suggest severe constipation as a potential contributor to the NE. A
neurological examination should focus on perineal sensation, sacral dimples, tufts of hair, and
gluteal clefts. Children with a history of snoring should also be examined for tonsillar
enlargement.

Laboratory evaluation in children with NE includes a complete urinalysis to evaluate for an


underlying renal disease. A 24-hour voiding diary helps to establish polyuria and also provides
an estimate of the bladder capacity using the maximum urine volume in a single void. In children

American Academy of Pediatrics 311


PREP ® Self-Assessment PREPSA 2021
older than 2 years, the expected bladder capacity in milliliters is equal to (age in years + 2) × 30.
The child in the vignette has a normal estimated bladder capacity of 300 mL.

Treatment of all children with NE includes limiting the fluid intake before bedtime, emptying the
bladder before going to sleep, voiding again at night (before the parents go to bed), and
establishing a consistent bedtime routine. In children with PMNE, bedwetting alarms are one of
the most effective treatment options and have a high long-term cure rate. Desmopressin is the
first-line pharmacological treatment of choice. Desmopressin acts by reducing the urine volume
at night but has a relapse rate higher than that associated with bedwetting alarms. Oxybutynin, an
anticholinergic, is not recommended as first-line treatment for children with PMNE. It is used in
children with a small-capacity bladder or a hyperactive bladder when first-line medication
treatment with desmopressin has failed. The child in the vignette has a normal bladder capacity,
so oxybutynin is not indicated. Imipramine, a tricyclic antidepressant, is not considered as a first-
line agent owing to its associated high enuresis relapse rate and associated risk of QT
prolongation in children.

Treatment of NMNE includes addressing any lower urinary tract dysfunction, constipation, and
underlying comorbid conditions. If enuresis continues even after lower urinary tract dysfunction
and constipation are corrected, medical or behavioral therapy should be started. The child in the
vignette has no features of bowel dysfunction, and so treatment with polyethylene glycol is not
indicated.

PREP Pearls
• Primary monosymptomatic nocturnal enuresis is defined as enuresis without symptoms of
lower urinary tract abnormalities or bladder dysfunction.
• Treatment of nocturnal enuresis includes limiting the fluid intake before bedtime,
emptying the bladder before going to sleep, and establishing a consistent bedtime routine.
• For primary monosymptomatic nocturnal enuresis, bedwetting alarms are among the
most effective treatment options, and desmopressin is the first-line pharmacological
therapy.

ABP Content Specifications(s)


• Plan the appropriate evaluation of enuresis of various types, including that occurring after
continence has been achieved
• Recognize the clinical and laboratory findings associated with daytime and nocturnal
urinary incontinence in male and female patients
• Identify the possible renal causes of nocturnal incontinence
• Recognize the clinical and laboratory findings associated with voiding dysfunction
• Plan the appropriate management of enuresis of various types
• Plan the appropriate diagnostic evaluation and management of incontinence

American Academy of Pediatrics 312


PREP ® Self-Assessment PREPSA 2021
Suggested Readings
• Bayne AP, Skoog SJ. Nocturnal enuresis: an approach to assessment and
treatment. Pediatr Rev. 2014;35(8);327-334. doi:10.1542/pir.35-8-327.
• Iorember FM. Enuresis. In: McInerny TK, Adam HM, Campbell DE, DeWitt TG, Foy
JM, Kamat DM, eds. American Academy of Pediatrics Textbook of Pediatric Care. 2nd
ed. Elk Grove Village, IL: American Academy of Pediatrics; 2017:2005-2010. Pediatric
Care Online.
• Roth EB, Austin PF. Evaluation and treatment of nonmonosymptomatic enuresis. Pediatr
Rev. 2014;35(10);430-436. doi:10.1542/pir.35-10-430.

American Academy of Pediatrics 313


PREP ® Self-Assessment PREPSA 2021
Question 85
A 13-year-old adolescent boy is seen after being hit in the left orbital area with a handball. He
immediately complained of left eye pain and nausea and was brought to the emergency
department for evaluation. On physical examination, there is mild left periorbital edema and
ecchymosis but no bony tenderness. His pupils are equal, round, and reactive to light, and
extraocular movements are intact. The conjunctivae are clear, and fluorescein examination
reveals no corneal or scleral abrasions. Tonometry reveals normal intraocular pressure; visual
acuity is 20/20 bilaterally. Slit-lamp inspection reveals a hyphema that occupies less than 25% of
the anterior chamber (Item Q85).

Item Q85: Findings for the boy described in the vignette.


Reprinted with permission from Lavrich, JB; Heersink, S. RedEye/Pink Eye. In: McInerny TK,
Adam HM, Campbell DE, DeWitt TG, Foy JM, Kamat DM, eds. American Academy of
Pediatrics Textbook of Pediatric Care. 2nd ed. Itasca, IL: American Academy of Pediatrics;
2016.

Of the following, the BEST next management step for this boy is
A. administration of 100% oxygen
B. administration of topical tissue plasminogen activator
C. placement of an eye shield
D. placement of the bed in the Trendelenburg position

American Academy of Pediatrics 314


PREP ® Self-Assessment PREPSA 2021
Correct Answer: C
The adolescent in the vignette has sustained a traumatic hyphema. A hyphema is a collection of
blood in the anterior chamber of the eye that occurs spontaneously or as a result of blunt or
penetrating eye trauma. Blunt trauma to the eye results in distortion of normal ocular anatomy,
and subsequent shearing of blood vessels in the iris and ciliary body leads to the development of
a hyphema. This type of injury should be considered any time a child is struck in the eye with a
baseball, handball, or similar object. Penetrating trauma causes direct injury to the iris and
resultant bleeding. The initial step in the management of a hyphema from any mechanism is
placement of an eye shield to protect the eye from further injury.

The anterior chamber is surrounded by the cornea (anteriorly), iris, and lens (posteriorly), and the
angle of the eye. The diagnosis of a hyphema is made clinically with visualization of blood in the
anterior chamber (Item C85A). Small hyphemas are best viewed with the patient sitting upright
and inspection of the eye from the lateral aspect. Hyphemas are graded based on the percentage
of the anterior chamber they occupy, with higher-grade hyphemas carrying a worse long-term
prognosis. Additional physical examination findings may include photophobia, subconjunctival
hemorrhage (Item C85B), elevated intraocular pressure, decreased visual acuity, corneal
abrasion (Item C85C and Item C85D), or a ruptured globe. Depending on the mechanism of
injury, damage to surrounding bony structures may be suspected as well. Item C85E lists
additional traumatic eye injuries and key physical examination findings.

Item C85A: Hyphema.


Reprinted with permission from Lavrich, JB; Heersink, S. RedEye/Pink Eye. In: McInerny TK,
Adam HM, Campbell DE, DeWitt TG, Foy JM, Kamat DM, eds. American Academy of
Pediatrics Textbook of Pediatric Care. 2nd ed. Itasca, IL: American Academy of Pediatrics;
2016.

American Academy of Pediatrics 315


PREP ® Self-Assessment PREPSA 2021

Item C85B: Subconjunctival Hemorrhage.


Reprinted with permission from Lavrich, JB; Heersink, S. RedEye/Pink Eye. In: McInerny TK,
Adam HM, Campbell DE, DeWitt TG, Foy JM, Kamat DM, eds. American Academy of
Pediatrics Textbook of Pediatric Care. 2nd ed. Itasca, IL: American Academy of Pediatrics;
2016.

Item C85C: Corneal abrasion.


Reprinted with permission from Lavrich, JB; Heersink, S. RedEye/Pink Eye. In: McInerny TK,
Adam HM, Campbell DE, DeWitt TG, Foy JM, Kamat DM, eds. American Academy of
Pediatrics Textbook of Pediatric Care. 2nd ed. Itasca, IL: American Academy of Pediatrics;
2016

American Academy of Pediatrics 316


PREP ® Self-Assessment PREPSA 2021

Item C85D: Corneal Abrasion.Courtesy of D Fein

Hyphema management includes placement of an eye shield, bed rest, eye rest, topical
cycloplegics and corticosteroids and appropriate analgesia. The head of the bed should be
elevated to 30 degrees to allow the hyphema to settle at the bottom of the anterior chamber
without obscuring the visual axis while preserving blood flow to the eye. Nausea should be
managed aggressively because vomiting will cause a rapid increase in intraocular pressure and
can exacerbate the hyphema. Most hyphemas can safely be managed on an outpatient basis;
however, hyphemas associated with increased intraocular pressure or high-grade hyphemas
warrant admission to the hospital. In addition, children with underlying bleeding disorders, sickle
cell disease, and those taking anticoagulation medications are at increased risk for complications
American Academy of Pediatrics 317
PREP ® Self-Assessment PREPSA 2021
and warrant admission to the hospital for closer monitoring and to minimize the risk of
rebleeding. In rare cases, surgery may be indicated.

There is no role for administration of 100% oxygen or topical tissue plasminogen activator in the
management of traumatic hyphema. The Trendelenburg position, placement of the patient supine
with the foot of the bed elevated 15 to 30 degrees, is the opposite of the position in which a
patient with a hyphema should be placed.

PREP Pearls
• Hyphema should be considered any time a child is struck in the eye with a baseball,
handball, or similar object.
• Management of hyphema includes an eye shield, bed rest in a reverse Trendelenburg
position at 30 degrees, topical cycloplegics and corticosteroids, and aggressive pain and
nausea control.
• Children with underlying bleeding disorders, sickle cell disease, and those taking
anticoagulation medications are at increased risk for complications of hyphema.

ABP Content Specifications(s)


• Plan the appropriate clinical evaluation of trauma to the eye, including hyphema
• Recognize the clinical findings associated with blow-out fracture of the orbit

Suggested Readings
• Rao RC, Bohra LI, Roarty JD. Ocular trauma. In: McInerny TK, Adam HM, Campbell
DE, DeWitt TG, Foy JM, Kamat DM, eds. American Academy of Pediatrics Textbook of
Pediatric Care. 2nd ed. Itasca, IL: American Academy of Pediatrics; 2016; chap
300. Pediatric Care Online.
• Walton W, Von Hagen S, Grigorian R, Zarbin M. Management of traumatic
hyphema. Surv Ophthalmol. 2002;47(4):297-334. doi: 10.1016/s0039-6257(02)00317-x.

American Academy of Pediatrics 318


PREP ® Self-Assessment PREPSA 2021
Question 86
A healthy 2-month-old male infant, born at full term, is seen for a health supervision visit. The
family recently moved from another state. His mother reports persistent drainage of clear fluid
from his umbilicus despite several cauterization procedures with silver nitrate by his previous
pediatrician. The infant is otherwise doing well and has remained afebrile. He has had normal
intake and output. His vital signs and growth parameters are normal. His physical examination
findings are normal except for a 3-mm-diameter umbilical granuloma with serous drainage.
There is no erythema or tenderness around the umbilicus.

Of the following, the BEST next step in management is to


A. ligate the umbilical stump with absorbable sutures
B. perform ultrasonography of the umbilicus
C. prescribe oral cephalexin
D. provide parental reassurance

American Academy of Pediatrics 319


PREP ® Self-Assessment PREPSA 2021
Correct Answer: B
The infant in the vignette should undergo umbilical ultrasonography to assess for persistent
patent umbilical structures. Ligation of the granuloma is inappropriate because of its size and
shape. Antibiotics are not necessary because the child has no evidence of infection. Reassurance
is inadequate because the drainage has persisted despite repeated attempts at cauterization.
In utero, the umbilical cord is the sole source of nutrition for the developing fetus. After birth,
the healthy neonate no longer requires this structure and the cord is usually cut, leaving a small
stump. In the United States, delivering physicians usually practice delayed cord clamping to
enhance neonatal iron stores and decrease the risk of intraventricular hemorrhage and necrotizing
enterocolitis.

Care for the umbilical stump involves practicing good hand hygiene and keeping the stump dry.
Applying alcohol is no longer recommended; this practice can delay cord separation by keeping
the cord moist and it does not effectively reduce bacterial colonization. In developing countries,
or where delivery of the infant may be less hygienic, the application of antiseptics such as
chlorhexidine may be used to reduce infection rates.

The umbilical stump usually falls off after 1 to 2 weeks, but may take up to 3 weeks. Before the
umbilicus is completely healed and dried, caregivers should limit bathing to sponge baths only. If
the stump is soiled with fecal matter, simple soap and water is adequate for cleaning. Causes of
delayed cord separation include prematurity, antimicrobial use, or immunologic disorders.
Investigation of immunologic function is recommended for neonates with delayed cord
separation who also have other signs of immune system problems.

After cord separation, a small umbilical granuloma may form, which may have serous or
serosanguinous drainage. The granuloma can be treated with silver nitrate cauterization.
Granulomas that are pedunculated can be ligated with absorbable sutures. Persistent granulomas
should be evaluated for other abnormalities.

Persistent umbilical drainage is the most common presentation for urachal anomalies such as
umbilical polyps, urachal cysts, and patent urachal sinuses. Ultrasonography is the preferred
imaging modality for these anomalies. Because of the risk of urachal adenocarcinoma, children
who have a urachal anomaly as well as other symptoms such as infection, pain, or drainage of
fecal matter should undergo surgical excision. Children who are otherwise asymptomatic can be
monitored closely.

When the umbilical cord falls off, the neonate may be found to have an umbilical hernia. Parents
should be reassured that most umbilical hernias will spontaneously close by 6 years of age.
Home remedies such as treatment with tape or coins are not indicated and do not accelerate
closure. Referral to a surgeon is indicated if the hernia is trunk-like in appearance or becomes
incarcerated. Strangulation of an umbilical hernia is rare, and should be treated as an emergency.
Referral should also be considered for an umbilical hernia that has not closed by age 6 years.

American Academy of Pediatrics 320


PREP ® Self-Assessment PREPSA 2021
PREP Pearls
• Ultrasonography is the preferred imaging modality to evaluate infants with persistent
umbilical granulomas or drainage; surgical referral should be considered.
• Dry cord care is recommended for infants delivered under hygienic conditions.
• Most umbilical hernias close spontaneously by age 6 years.

ABP Content Specifications(s)


• Plan appropriate umbilical cord care

Suggested Readings
• Marion RW, Samanich J. Umbilical anomalies. In: McInerny TK, Adam HM, Campbell
DE, DeWitt TG, Foy JM, Kamat DM, eds. American Academy of Pediatrics Textbook of
Pediatric Care. 2nd ed. Itasca, IL: American Academy of Pediatrics; 2016;chap
343:2745-2747. Pediatric Care Online.
• Muniraman H, Sardesai T, Sardesai M. Disorders of the umbilical cord. Pediatr Rev.
2018;39(7):332-341. doi:10.1542/pir.2017-0202.
• Stewart D, Benitz W; Committee on Fetus and Newborn. Umbilical cord care. Pediatrics.
2016;138(3). doi: 10.1542/peds.2016-2149.

American Academy of Pediatrics 321


PREP ® Self-Assessment PREPSA 2021
Question 87
An 11-year-old who was assigned female at birth is seen for a health supervision visit. He has
been diagnosed with gender dysphoria by a psychologist who has followed him for several years.
He asks to be called “Michael” and uses the pronouns “he” and “him.” His mother states that he
displays typical male play behaviors; prefers typical male clothing, including underwear; and has
insisted that he is a boy since about age 3 years. He became very distressed when he recently
began showing signs of typical female puberty. An interdisciplinary gender team has
recommended that he start receiving a gonadotropin-releasing hormone agonist to suppress
puberty. His parents have consented, and he has provided assent for this therapy. They
understand the potential adverse effects, including that on bone health. He is wearing typical
male clothing and has a typical male hair style. His sexual maturity rating is 2 for breast and
pubic hair development. The remainder of his physical examination findings are normal.

Of the following, the team’s action that BEST represents the ethical principle of beneficence is
A. ensuring that the parents have consented to therapy
B. ensuring that the child and family understand potential adverse effects of therapy
C. making the recommendation to initiate therapy
D. obtaining assent of the child before the initiation of therapy

American Academy of Pediatrics 322


PREP ® Self-Assessment PREPSA 2021
Correct Answer: C
The child described in the vignette is a transgender male who is showing worsening gender
dysphoria with the onset of typical female puberty. The interdisciplinary gender team’s action
that best represents the ethical principle of beneficence is making the recommendation to initiate
therapy with a gonadotropin-releasing hormone (GnRH) agonist to suppress puberty.

Beneficence is the ethical principle concerned with benefiting others. Therapy with a GnRH
agonist benefits the child by suppressing pubertal progression, an unwanted outcome from the
child’s perspective, with the goal of improving gender dysphoria. This therapy also facilitates
full gender transition in the future, because the effects of normal female puberty would not need
to be reversed. Although GnRH agonists are not formally approved by the United States Food
and Drug Administration for gender-affirming care at this time, clinical practice guidelines
support their use in the treatment of gender dysphoria.

Nonmaleficence is the ethical principle concerned with minimizing harm. Applied to this
vignette, the effects of GnRH agonist are reversible if the decision to progress through female
puberty is made in the future. Guidelines recommend waiting until the child has reached sexual
maturity rating 2 to initiate GnRH agonist. This timing allows for confirmation of the gender
dysphoria diagnosis (gender dysphoria worsens with the onset of puberty), and prevents
unnecessary treatment in a prepubertal child.

Ensuring that the child and family understand the potential adverse effects of therapy, ensuring
that the parents have consented to therapy, and obtaining the child’s assent before the initiation
of therapy all relate to the ethical principle of autonomy. Autonomy is the principle concerned
with allowing people to make decisions for themselves. These decisions should be informed and
should be made with parents’ consent and the child’s assent.

The fourth ethical principle, justice, is concerned with fairness and equality. This principle
relates to youth undergoing gender-affirming care in the potential inequalities in access to and
insurance coverage of therapy.

PREP Pearls
• Therapy with a gonadotropin-releasing hormone agonist for youth undergoing gender-
affirming care shows beneficence by suppressing pubertal progression with the goal of
improving gender dysphoria.
• Guidelines recommend waiting until youth undergoing gender-affirming care achieve
sexual maturity rating 2 to initiate gonadotropin-releasing hormone agonist. This timing
allows confirmation of the gender dysphoria diagnosis (gender dysphoria worsens with
the onset of puberty), and prevents unnecessary treatment in a prepubertal child.

ABP Content Specifications(s)


• Recognize and apply ethical principles involved in using new technologies for sex/gender
assignment

American Academy of Pediatrics 323


PREP ® Self-Assessment PREPSA 2021
Suggested Readings
• Hembree WC, Cohen-Kettenis PT, Gooren L, et al. Endocrine treatment of gender-
dysphoric/gender-incongruent persons: an Endocrine Society clinical practice guideline
(published corrections appear in J Clin Endocrinol Metab. 2018;103(2):699 and J Clin
Endocrinol Metab. 2018;103(7):2758-2759). J Clin Endocrinol Metab.
2017;102(11):3869-3903. doi:10.1210/jc.2017-01658.
• Jennings J, Jennings J. Trans teen shares her story. Pediatr Rev. 2016;37(3):99-100.
doi:10.1542/pir.2016-002.
• Kimberly LL, Folkers KM, Friesen P, et al. Ethical issues in gender-affirming care for
youth. Pediatrics. 2018;142(6):e20181537. doi:10.1542/peds.2018-1537.
• Lopez X, Stewart S, Jacobson-Dickman E. Approach to children and adolescents with
gender dysphoria. Pediatr Rev. 2016;37(3):89-96. doi:10.1542/pir.2015-0032.
• Shumer DE, Nokoff NJ, Spack NP. Advances in the care of transgender children and
adolescents. Adv Pediatr. 2016;63(1):79-102. doi:10.1016/j.yapd.2016.04.018.

American Academy of Pediatrics 324


PREP ® Self-Assessment PREPSA 2021
Question 88
The mother of a 5-year-old girl is concerned about her daughter’s excessive weight gain. The
family spends a lot of time watching TV and playing computer games. The girl’s classmates are
starting to tease her about her weight, particularly during physical education. The mother has
obesity, and she is very concerned that her daughter seems to be following a similar weight
trajectory. The girl’s last health supervision visit occurred 6 months ago. At that time, her body
mass index was at the 80th percentile; her history and physical examination findings were
otherwise unremarkable. The family has met with a dietician for guidance. The mother seeks
advice on sustainable changes that will encourage her daughter to become more physically
active.

Of the following, the mother can MOST likely achieve this goal by
A. encouraging her daughter to participate in a local sports program
B. increasing her own level of physical activity
C. requesting a referral for physical therapy for her daughter
D. setting time limits on computer use at home

American Academy of Pediatrics 325


PREP ® Self-Assessment PREPSA 2021
Correct Answer: B
Parents are highly influential in determining patterns of physical activity and screen use in young
children. Activity levels in children are increased when parents support and encourage physical
activity; serve as role models for healthy levels of physical activity; and facilitate physical
activity as a family unit. Therefore, of the response choices listed, the mother is most likely to
achieve her goal of sustainably increasing her daughter’s level of physical activity if she
increases her own.

Current recommendations are for children and adolescents 5 to 17 years of age to participate in
60 minutes of moderate to vigorous physical activity every day. Many children participate in
physical activity through involvement in sports. However, younger children typically do not have
the attention span or physical skills to gain significant benefit from organized sports participation
until they are 6 or 7 years old. The 5 year old in the vignette would benefit more from a variety
of opportunities for physically active free play, which will enhance the development of motor,
social, cognitive, and self-regulation skills.

Young children who learn to enjoy physical activity and are given opportunities to develop a
variety of fundamental movement skills (eg, running, throwing, and jumping) at an early age are
more likely to engage in higher levels of activity in adulthood. Physical activity in younger
children should be focused on fun and variety; physically inactive parents may need additional
guidance on how to create opportunities for their children. To enhance children’s levels of
physical activity, providers may counsel families to:
• Provide toys that encourage physical movement
o Balls in a variety of shapes and sizes
o Ride-on toys without motors or batteries
o Rocking toys
o Boxes for climbing and building
• Find opportunities to increase walking as a family
o Walking-oriented family activities (eg, zoo, park, scavenger hunts)
o Geocaching or other activities that combine technology with physical activity
o Walking, in place of other transportation modes, when feasible
• Encourage children to perform or help with physically active chores
o Dog walking
o Yard work
o Housework
• Learn a new activity as a family
o Rock climbing
o Bird watching
o Dancing
o Gardening

The We Can website from the National Heart, Lung, and Blood Institute has additional
information and links for families on how to engage children in healthy levels of activity
(https://www.nhlbi.nih.gov/health/educational/wecan/get-active/getting-active.htm).
The health effects of physical activity and sedentary time were previously considered to be
interrelated and to essentially represent opposite ends of the same spectrum. However, there is
American Academy of Pediatrics 326
PREP ® Self-Assessment PREPSA 2021
increasing recognition that excessive sedentary time and inadequate levels of moderate to
vigorous physical activity exert independent influences on child health. Screen time is the largest
contributor to sedentary time in children and adolescents. High levels of screen time are
associated with changes in weight, caloric intake, and metabolic rate that are independent of
physical activity levels. Decreasing screen time does not automatically result in increased levels
of physical activity: these issues need to be addressed separately by pediatric health care
providers.

It is estimated that 5% to 6% of children manifest findings consistent with developmental


coordination disorder (DCD), which is characterized by poor or immature motor skills that are
independent of other medical causes. Parents will often note that these children seem “clumsy”
compared with their peers. These motor difficulties often lead to low levels of physical activity
throughout the lifespan. Recent findings suggest that early identification and appropriate
treatment of children with DCD results in improved motor proficiency and increased voluntary
engagement in physical activity. Physical therapy with an emphasis on motor skills and
neuromotor task training appears to be of benefit. Various treatment modalities are currently
under study. No motor developmental deficits were noted in the girl in this vignette, therefore
physical therapy would not be expected to result in increased physical activity levels.

PREP Pearls
• Parents should be counseled to serve as role models for healthy levels of physical
activity.
• Decreased screen time does not correlate with increased participation in healthy levels of
physical activity.
• Children with developmental coordination disorder benefit from early referral for
physical therapy.

ABP Content Specifications(s)


• Understand the importance of regular exercise to promote good general health

American Academy of Pediatrics 327


PREP ® Self-Assessment PREPSA 2021
Suggested Readings
• Blank R, Barnett AL, Cairney J, Green D, et al. International clinical practice
recommendations on the definition, diagnosis, assessment, intervention, and psychosocial
aspects of developmental coordination disorder. Dev Med Child Neurol. 2019;61(3):242-
285. doi: 10.1111/dmcn.14132.
• Lobelo F, Muth ND, Hanson S, Nemeth BA, Council on Sports Medicine and Fitness,
Section on Obesity. Physical activity assessment and counseling in pediatric clinical
settings. Pediatrics;145(3): e20193992. doi: 10.1542/peds.2019-3992.
• O'Brien W, Issartel J, Belton S. Relationship between physical activity, screen time and
weight status among young adolescents. Sports (Basel). 2018;6(3):57.
doi: 10.3390/sports6030057.
• Xu H, Wen LM, Rissel C. Associations of parental influences with physical activity and
screen time among young children: a systematic review. J Obes. 2015;2015:546925.
doi: 10.1155/2015/546925.
• Yogman M, Garner A, Hutchinson J, et al. The power of play: a pediatric role in
enhancing development in young children. Pediatrics. 2018;142(3):e20182058.
doi: 10.1542/peds.2018-2058.

American Academy of Pediatrics 328


PREP ® Self-Assessment PREPSA 2021
Question 89
A 12-day-old infant is seen in the emergency department for evaluation of fever. Yesterday she
was noted to be fussy and today, she has a temperature of 39°C and is difficult to console. She
has no other symptoms. She was born at term via vaginal delivery to a 34-year-old woman who
had an uncomplicated pregnancy. Her vital signs include a temperature of 38.6°C, heart rate of
180 beats/min, respiratory rate of 50 breaths/min, and blood pressure of 82/45 mm Hg. On
physical examination, she is irritable and her anterior fontanelle is full. Blood, urine, and
cerebrospinal fluid specimens are collected for microbiologic evaluation. Gram stain of her
cerebrospinal fluid is shown in Item Q89.

Item Q89: Cerebrospinal fluid Gram stain.


Copyright Martha Lepow and reprinted with permission from Reprinted with permission from
Kimberlin DW, et al, eds. Red Book Online. Itasca, IL: American Academy of Pediatrics; 2018

Of the following, the BEST treatment for this infection is


A. ampicillin
B. azithromycin
C. ceftazidime
D. vancomycin

American Academy of Pediatrics 329


PREP ® Self-Assessment PREPSA 2021
Correct Answer: A
Of the choices listed, the best treatment for the neonate in the vignette is ampicillin. The clinical
presentation suggests a diagnosis of meningitis, which is supported by the presence of gram-
positive rods in her cerebrospinal fluid. Listeria monocytogenes is the most likely pathogen given
this laboratory finding. The treatment of choice for Listeria monocytogenes infections is
ampicillin.

Neonates can acquire Listeria infection transplacentally or during the birthing process from a
colonized mother. In older children and adults, Listeria infection is acquired by consumption of
contaminated foods. Listeria monocytogenes can survive in refrigerated and processed foods,
which gives the organism a unique niche in food-borne illnesses. Some high-risk foods include
unpasteurized dairy products (including soft cheeses), raw sprouts, ready-to-eat deli meats, and
hot dogs (unless cooked to 73.8°C).

Individuals at greatest risk for infection include neonates, the elderly, pregnant women, and
individuals with defects in cellular immunity. Clinical manifestations of Listeria infections
include gastroenteritis, bacteremia, and meningitis. Listeria infections in newborns can manifest
as early-onset disease characterized by sepsis and pneumonia or late-onset disease which usually
results in meningitis. Infection during pregnancy can manifest as a flu-like illness; it can lead to
spontaneous abortion, fetal death, or preterm delivery. However, infection in pregnant women
can be asymptomatic as seen in the vignette.

Severe Listeria infection is usually treated with a combination of ampicillin and an


aminoglycoside, such as gentamicin. Macrolides other than erythromycin, such as azithromycin,
have only moderate activity against Listeria. Listeria organisms are resistant to cephalosporins,
thus ceftazidime should not be used to treat this neonate’s infection. Vancomycin has activity
against Listeria monocytogenes, but experience with this drug is limited and treatment failures
have been reported.

PREP Pearls
• The treatment of choice for Listeria monocytogenes infection is ampicillin; for severe
infections an aminoglycoside should be added for synergy.
• Individuals at greatest risk of Listeria infection include neonates, the elderly, pregnant
women, and individuals with defects in cellular immunity.
• Clinical manifestations of Listeria infection may include gastroenteritis, bacteremia, and
meningitis.

ABP Content Specifications(s)


• Understand the epidemiology of Listeria monocytogenes
• Recognize the clinical features associated with Listeria monocytogenes infection
• Plan appropriate management for a patient with Listeria monocytogenes infection

American Academy of Pediatrics 330


PREP ® Self-Assessment PREPSA 2021
Suggested Readings
• American Academy of Pediatrics. Listeria monocytogenes infections. In: Kimberlin DW,
Brady MT, Jackson MA, Long SS, eds. Red Book: 2018 Report of the Committee on
Infectious Diseases. American Academy of Pediatrics; 2018:511-515. Red Book Online.
• Kojaoghlanian T. The newborn at risk for infection. In: McInerny TK, Adam HM,
Campbell DE, DeWitt TG, Foy JM, Kamat DM, eds. American Academy of Pediatrics
Textbook of Pediatric Care. 2nd ed. Itasca, IL: American Academy of Pediatrics;
2016;chap 102:899-908. Pediatric Care Online.
• Lee P, Krilov L. Listeria. Pediatr Rev. 2018;39(3):153-155. doi: 10.1542/pir.2017-0062.

American Academy of Pediatrics 331


PREP ® Self-Assessment PREPSA 2021
Question 90
A 15-year-old basketball player with hypertrophic cardiomyopathy comes for consultation
regarding return to play. Three months ago, he collapsed at a basketball game and was
successfully resuscitated. After additional cardiac consultation, an implantable cardioverter
defibrillator (ICD) was placed. He has not received any shocks since ICD placement. The
cardiologist has informed him and his family that he may consider returning to basketball at this
time, as long as they all understand the potential risks. The patient is highly interested in
returning to basketball, but his parents are hesitant and wish to learn more about all his options.

Of the following, the principle that is MOST applicable to this decision is


A. autonomy
B. beneficence
C. justice
D. nonmaleficence

American Academy of Pediatrics 332


PREP ® Self-Assessment PREPSA 2021
Correct Answer: D
Sports participation in athletes with implantable cardiac defibrillators (ICDs) may be considered
if the athlete has not received any ICD shocks from episodes of ventricular flutter or fibrillation
for at least 3 months. However, sports participation increases the likelihood of both appropriate
and inappropriate shocks, as well as the potential for device-related damage in contact/collision
sports. These issues need to be considered when discussing the risk versus benefit ratio of return
to basketball for the boy in the vignette.

The 4 basic ethical principles of modern medicine are nonmaleficence, beneficence, autonomy,
and justice. Nonmaleficence requires consideration of the potential harms that may accrue to a
patient in a given circumstance. In this case, the chief concerns regarding return to basketball are
the potential for additional shock delivery and risk of ICD damage. Therefore, nonmaleficence is
most applicable. The patient and his family need to be informed of the risks associated with a
return to basketball, and to weigh these against the patient’s desire to return to this activity. For
example, a 2018 study looking at the consequences of sports participation in subjects with ICDs
found that about 27% of subjects reported additional shocks with sports, but no other ICD-
related adverse events. From a medical perspective, this is not an undue risk. However,
considering the discomfort associated with ICD shock delivery, the patient may decide that he
would prefer to avoid activities that increase the risk of additional shocks.

Beneficence is defined as action that is in the patient’s best interest. Beneficence and
nonmaleficence often coexist, and potential benefits need to be considered in light of potential
risks. Although there would be multiple benefits to clearing this patient for basketball
participation, these are not the primary issues under consideration. Therefore, beneficence is not
the most applicable ethical principle being applied in this vignette.

Autonomy recognizes the right of individual patients to make decisions for themselves.
Competent patients with the capacity to understand treatment risks, benefits, and alternatives are
generally accorded the right to autonomy. Capacity is highly variable in the pediatric population
and will depend on the patient’s developmental stage, as well as the specific issue at hand. Most
adolescents are considered to be developing autonomy, and should be given increasing say in
their treatment options as they move toward adulthood. At 15 years of age, the adolescent in the
vignette may have the capacity to fully consider the consequences of this decision. Although
patient autonomy may become a dominant issue if he and his parents disagree on the decision to
return to sport, or if the family disagrees with the provider’s recommendation, it is not the
primary issue in this vignette.

The principle of justice is based on both individual and social factors, and relies on the fair
settlement of competing interests. This principle ensures that a decision for the benefit of 1
patient (or segment of patients) is not at the expense of others. The case in this vignette would
not be expected to have significant medical impact on others, and therefore justice is not the
primary issue in this case.

American Academy of Pediatrics 333


PREP ® Self-Assessment PREPSA 2021
PREP Pearls
• Nonmaleficence (do no harm) and beneficence (action for the patient’s benefit) are
usually complementary in medical ethics decision-making.
• The appropriate delegation of autonomy in decisions to adolescents is dependent on their
stage of development and the particulars of the situation.
• The principle of justice ensures that a decision for the benefit of 1 patient (or segment of
patients) is not at the expense of others.

MOCA-Peds Objective
• Identify contraindications for sports participation.

ABP Content Specifications(s)


• Recognize and apply ethical principles regarding the care of children and adolescents
with disabilities

Suggested Readings
• Cummings C, Mercurio MR. Session 2: Autonomy, Beneficence, and the Rights of
Parents and Children. American Academy of Pediatrics website. https://www.aap.org/en-
us/Documents/Bioethics-
AutonomyBeneficenceAndTheRightsofParentsandChildren.pdf .
• Rowell EE, Fecteau A, Katz AL, Beals D, et al. Ethics. American Pediatric Surgical
Association website. https://www.pedsurglibrary.com/apsa/view/Pediatric-Surgery-
NaT/829247/all/Ethics.
• Saarel EV, Law I, Berul CI, Ackerman MJ, et al. Safety of Sports for Young Patients
With Implantable cardioverter-defibrillators: long-term results of the multinational ICD
Sports Registry. Circ Arrhythm Electrophysiol. 2018;11:e006305.
doi: 10.1161/CIRCEP.118.006305.

American Academy of Pediatrics 334


PREP ® Self-Assessment PREPSA 2021
Question 91
A male neonate born several hours ago is transferred to the special care nursery because of
cyanosis noted by the nurse. His mother received limited prenatal care but had unremarkable
prenatal laboratory results. The neonate is afebrile. He has a heart rate of 160 beats/min, a blood
pressure of 75/45 mm Hg in the right upper extremity, a blood pressure of 78/46 mm Hg in the
right lower extremity, a respiratory rate of 60 breaths/min, and an oxygen saturation of 70% that
increases to 74% after the addition of supplemental oxygen. He is breathing comfortably and has
clear breath sounds. His heart rate is regular with a normal S1 and S2 and a grade 3/6 pansystolic
murmur heard best at the lower left sternal border. His hands and feet are slightly cool, but he is
otherwise warm and well-perfused. His liver is palpated 2 to 3 cm below the costal margin. A
chest radiograph is obtained (Item Q91).

American Academy of Pediatrics 335


PREP ® Self-Assessment PREPSA 2021
Item Q91: Radiograph for the neonate described in the vignette. Courtesy of M McBride

Of the following, the MOST likely diagnosis for this neonate is


A. Ebstein anomaly
B. hypoplastic left heart syndrome
C. truncus arteriosus
D. ventricular septal defect

American Academy of Pediatrics 336


PREP ® Self-Assessment PREPSA 2021
Correct Answer: A
The neonate in the vignette is noted to be cyanotic at several hours of age and is only minimally
responsive to supplemental oxygen. On physical examination, there is no difference between
right upper and right lower extremity blood pressure, which makes a left-sided obstructive lesion
such as hypoplastic left heart syndrome less likely. Additionally, the child has a murmur
consistent with tricuspid regurgitation. A chest radiograph demonstrates significant cardiomegaly
and dark lung fields, consistent with cyanotic congenital heart disease with decreased pulmonary
blood flow. Although the relative amount of pulmonary-to-systemic blood flow in congenital
heart disease depends on the relationship between pulmonary and systemic vascular resistance,
of the choices listed, Ebstein anomaly is the most likely diagnosis. From a clinical perspective, a
chest radiograph with the degree of cardiomegaly and decreased pulmonary blood flow, as
shown in Item C91, should raise the suspicion for Ebstein anomaly. A ventricular septal defect
would show pulmonary blood flow equal to or greater than systemic blood flow, and therefore
the dark lungs depicted in Item C91 do not fit. Depending on how large the ventricular septal
defect is, there may be either no appreciable murmur or a murmur consistent with a typical
ventricular septal defect. Although hypoplastic left heart syndrome and truncus arteriosus in
some settings can have less pulmonary blood flow relative to systemic blood flow, they are more
classically associated with excessive pulmonary blood flow, insufficient systemic blood flow,
and normal oxygen saturations with poor systemic perfusion.

American Academy of Pediatrics 337


PREP ® Self-Assessment PREPSA 2021

The mnemonic of the “five Ts” is often used to remember the causes of cyanotic congenital heart
disease: transposition of the great arteries, tetralogy of Fallot, tricuspid atresia, total anomalous
pulmonary venous connection, and truncus arteriosus. Unfortunately, while this construct is
organized and easy to remember, it is not entirely correct. Patients with cyanotic congenital heart
disease have cyanosis because of complete intracardiac mixing, decreased pulmonary blood
flow, obstructed pulmonary venous return, or decreased mixing in the context of transposition of
the great arteries. Patients with complete intracardiac mixing (single ventricles and truncus
arteriosus) with a balanced pulmonary and systemic circulation will typically have an oxygen
saturation of 75% to 85%. However, some of these patients will have pulmonary overcirculation
with normal oxygen saturations.

American Academy of Pediatrics 338


PREP ® Self-Assessment PREPSA 2021
Patients with pulmonary atresia or critical pulmonary stenosis (which may occur in combination
with other cardiac defects, ie, tetralogy of Fallot) have decreased pulmonary blood flow and,
therefore, right-to-left shunting through intracardiac shunts; thus, they can have profound
cyanosis. Such patients will require prostaglandin infusion to keep the ductus arteriosus open and
maintain adequate pulmonary blood flow. An oxygen saturation of 75% to 85% is expected after
reopening of the ductus arteriosus. Total anomalous pulmonary venous connection occurs in a
variety of ways in which ultimately the pulmonary veins do not return back to the left atrium. In
these lesions, the pulmonary veins are at risk of becoming obstructed, and when they do, this is a
true cardiac surgical emergency. These babies have pulmonary hypertension, hypoxia due to
ventilation:perfuson mismatch, and systemic hypoperfusion due to decreased preload of the left
ventricle. Finally, those with transposition of the great arteries, typically a lesion with increased
pulmonary blood flow, can experience hypoxia if there is inadequate mixing. In this lesion,
pulmonary and systemic circulations are running parallel to each other as opposed to in series, a
situation that results in oxygenated blood’s returning to the lungs and deoxygenated blood’s
returning to the body.

Infants need to have a sufficient intracardiac mixing, preferably at the atrial level for proper
mixing. In patients with transposition of the great arteries who have cyanosis, a prostaglandin
infusion should be initiated. Infants without adequate intracardiac mixing (ie, an atrial septal
defect) will need a balloon atrial septostomy to create an unrestrictive atrial level shunt. Babies
who are prenatally diagnosed with a ductal-dependent lesion can undergo a prostaglandin
infusion initiated in the delivery room. As the ductus will not have had an opportunity to begin to
close, the drip can be started at a low dose. Infants who are not prenatally diagnosed and present
in extremis will likely need the prostaglandin infusion started at a high dose. Side effects tend to
be dose dependent and include apnea, flushing, fever, tachycardia, and vasodilatation.

PREP Pearls
• Cyanotic congenital heart disease occurs as a result of complete intracardiac mixing,
decreased pulmonary blood flow, obstructed pulmonary venous return, or transposition
physiology with inadequate intracardiac mixing.
• Many cyanotic congenital heart disease lesions (not truncus arteriosus or total anomalous
pulmonary venous connection) require the initiation of prostaglandin infusion shortly
after birth.

American Academy of Pediatrics 339


PREP ® Self-Assessment PREPSA 2021
MOCA-Peds Objective
• Recognize genetic causes of congenital heart disease.

ABP Content Specifications(s)


• Recognize complications associated with cyanotic congenital heart disease
• Plan immediate management of a hypoxic episode in a child who has cyanotic congenital
disease
• Recognize the major clinical findings associated with the various types of cyanotic
congenital heart disease

Suggested Readings
• Puri K, Allen HD, Qureshi AM. Congenital heart disease. Pediatr Rev. 2017;38(10):471-
486. doi:10.1542/pir.2017-0032.
• Silberbach M, Hannon D. Presentation of congenital heart disease in the neonate and
young infant. Pediatr Rev. 2007;28(4):123-131. doi:10.1542/pir.28-4-123.
• Sutton NJ, Walsh CA. The newborn with a heart murmur or cyanosis. In: McInerny TK,
Adam HM, Campbell DE, DeWitt TG, Foy JM, Karnat DM, eds. American Academy of
Pediatrics Textbook of Pediatric Care. 2nd ed. Elk Grove Village, IL: American
Academy of Pediatrics; 2017:888-898. Pediatric Care Online.

American Academy of Pediatrics 340


PREP ® Self-Assessment PREPSA 2021
Question 92
A 7-year-old girl with a 3-year history of intermittent cough, wheezing with viral respiratory
infections, and repeated episodes of respiratory distress with low oxygen saturation is being
evaluated. Each time she is ill she has been treated with bronchodilators and oral corticosteroids.
Inhaled corticosteroids have been prescribed as an asthma controller medication, but her parents
have not given the medication consistently. Today the girl’s parents report that she has a 5-day
history of worsening cough and nasal congestion, but no fever or other somatic symptoms. Her
parents have not heard her wheeze, but they do sense that she is having difficulty breathing.
On physical examination, she is afebrile with a room air oxygen saturation of 92% on pulse
oximetry, heart rate of 130 beats/min, and blood pressure of 100/70 mm Hg. Her mucous
membranes are moist and there is good skin turgor. Moderate intercostal and suprasternal
retractions are seen, with generally diminished breath sounds and diffuse end-expiratory
wheezes. No focal adventitious sounds can be heard in the chest. After treatment with albuterol
2.5 mg with wet nebulization, she seems a bit more comfortable, but her oxygen saturation is not
improved, wheezing is not resolved, and she still has retractions. Chest radiography is performed
(Item Q92).

Item Q92: Chest radiographs of the girl in the vignette. Courtesy of M. Guill

Of the following, the MOST appropriate next management step for this girl is
A. endotracheal intubation and admission to the intensive care unit
B. hospital admission for frequent inhaled bronchodilators and systemic corticosteroids
C. outpatient management with antibiotics with next day follow-up
D. referral for urgent bronchoscopy and bronchoalveolar lavage

American Academy of Pediatrics 341


PREP ® Self-Assessment PREPSA 2021
Correct Answer: B
The girl in the vignette has an asthma exacerbation with incomplete response to inhaled
bronchodilators. The most appropriate intervention for this girl is hospital admission for
aggressive bronchodilator treatment and systemic steroids. She merits close observation for
worsening of her clinical status despite intervention. While the findings of hyperinflation,
peribronchial cuffing, and right middle lobe atelectasis seen on her chest radiographs are often
misdiagnosed as pneumonia, this girl has no fever or other somatic symptoms to suggest
infection. Although she does have mild respiratory distress, there is no indication of impending
respiratory failure or need for intubation at this point. Acute bronchoscopy would be indicated if
there were concern for foreign body aspiration, but her age and the event history do not support
this as a cause of her symptoms. Mucus plugging with atelectasis may be a component of severe
asthma and is appropriately addressed with attention to the asthma itself. In his situation, chest
physical therapy may be beneficial.

The girl in the vignette likely has moderate to severe asthma that has not been well managed.
The history of frequent exacerbations with viral infections and need for oral steroids suggests
that she is at risk for severe exacerbation. Lack of wheezing between episodes does not guarantee
normal lung function. It is entirely likely that she has a current baseline of impaired airflow
which would be demonstrated as decreased forced expiratory volume in 1 second (FEV1) on
pulmonary function testing. A normal physical examination and lack of symptoms between
episodes of illness can give a false indication of intermittent asthma and lead to under-
recognition of severity. Daily asthma controller therapy, such as an inhaled corticosteroid, should
be considered for any child needing repeat courses of systemic steroids to manage episodes of
wheezing. The best treatment of persistent asthma is appropriate daily controller therapy plus
recognition of triggers, addressing those that can be ameliorated.

PREP Pearls
• Daily asthma controller therapy, such as an inhaled corticosteroid, should be considered
for any child needing repeat courses of systemic steroids to manage episodes of
wheezing.
• Chest radiography findings of hyperinflation, peribronchial cuffing, and right middle lobe
atelectasis seen in asthma are often misdiagnosed as pneumonia, even when there is no
fever or other somatic symptoms to suggest infection.
• The best treatment of persistent asthma is appropriate daily controller therapy plus
recognition of triggers, addressing those that can be ameliorated.

ABP Content Specifications(s)


• Recognize the characteristics of a child with asthma who is at risk of hospitalization
• Understand the risks and benefits of corticosteroid therapy in the treatment of a patient
who has an acute exacerbation of asthma
• Recognize the development of atelectasis during an acute exacerbation of asthma, and
manage appropriately
• Recognize the signs and symptoms of poorly controlled asthma
• Know the classifications of asthma and their components
• Know the diagnostic criteria for asthma

American Academy of Pediatrics 342


PREP ® Self-Assessment PREPSA 2021
Suggested Readings
• Dinakar C. Asthma. In: McInerny TK, Adam HM, Campbell DE, DeWitt TG, Foy JM,
Kamat DM, eds. American Academy of Pediatrics Textbook of Pediatric Care. 2nd ed.
Itasca, IL: American Academy of Pediatrics; 2016;chap 218:1736-1754. Pediatric Care
Online.
• Link HW. Pediatric asthma in a nutshell. Pediatr Rev. 2014;35:287-297.
doi: 10.1542/pir.35-7-287.
• Okada Y, Nakamura T, Maeda M, et al. Utility of therapeutic strategy based on the
modified pulmonary index score for childhood asthma exacerbation. Allergy Asthma
Proc. 2019;40:111-115. doi: 10.2500/aap.2019.40.4203.
• Patel SJ, Teach SJ. Asthma. Pediatr Rev 2019,40:549-567;
DOI: https://doi.org/10.1542/pir.2018-0282.
• Wood PR, VL Hill. Practical management of asthma. Pediatr Rev. 2009;30:375-384.
doi: 10.1542/pir.30-10-375.

American Academy of Pediatrics 343


PREP ® Self-Assessment PREPSA 2021
Question 93
A 15-year-old adolescent girl is seen for her annual health supervision visit. Her mother is
concerned that she has not started her period and has lost some weight. The patient is a
competitive gymnast and is on a club gymnastics team. She has not had any acne flares,
hirsutism, headaches, changes in her vision, or galactorrhea. She reports no fever, diarrhea, joint
pain, or rashes. She has never been sexually active. There is no family history of menstrual
disorders, thyroid disease, or inflammatory bowel disease. She is alert and cooperative. She is
afebrile and has a heart rate of 44 beats/min and a blood pressure of 92/50 mm Hg. Her weight is
47.2 kg (25th percentile), and her height is 155 cm (10th percentile). Her growth chart is shown
(Item Q93). Her physical examination findings are otherwise normal except for extremities that
are cool to the touch.

American Academy of Pediatrics 344


PREP ® Self-Assessment PREPSA 2021

American Academy of Pediatrics 345


PREP ® Self-Assessment PREPSA 2021
Of the following, the MOST likely diagnosis is
A. Addison disease
B. anorexia nervosa
C. Crohn disease
D. hyperthyroidism

American Academy of Pediatrics 346


PREP ® Self-Assessment PREPSA 2021
Correct Answer: B
The girl in the vignette has a history that is suggestive of anorexia nervosa. She is a gymnast who
has not had menarche and has experienced recent weight loss, bradycardia, and an abrupt arrest
in her growth, as seen on her growth chart. Addison disease has common features of weight loss,
hypotension, skin hyperpigmentation, and abdominal complaints. Crohn disease is associated
with abdominal pain, diarrhea, and weight loss. Growth failure is seen in patients with Crohn
disease, and the height velocity may decrease before the onset of abdominal symptoms. Addison
disease and Crohn disease are less likely diagnoses because this adolescent girl has not had any
recent illnesses (i.e. vomiting or diarrhea), has no abdominal symptoms, and no signs of skin
hyperpigmentation or other skin lesions. Hyperthyroidism is associated with unexplained weight
loss and menstrual irregularities, but is also associated with an increase in height velocity and
palpitations/tachycardia.

Delayed puberty is defined as absence of pubertal development at an age that is 2 to 2.5 standard
deviations above the mean for the general population. The first sign of puberty in girls is breast
development and in boys is testicular enlargement. Evaluation may be necessary in girls with
lack of breast development by age 13 years and in boys with lack of testicular enlargement by
age 14 years. Delayed puberty is divided into 2 categories according to hypothalamic-pituitary-
gonadal (HPG) axis dysfunction. Primary hypogonadism (hypergonadotrophic hypogonadism) is
associated with high levels of luteinizing hormone (LH) and follicle-stimulating hormone (FSH)
because of failure of gonadal sex steroid production despite activation of the HPG axis (primary
gonadal failure). Turner syndrome in female patients (ovarian failure) and Klinefelter syndrome
in male patients (testicular failure) are examples. Secondary hypogonadism (hypogonadotropic
hypogonadism) is associated with a delay of maturation in the HPG axis, with low-normal levels
of LH and FSH because of systemic illness (ie, inflammatory bowel disease, cystic fibrosis,
diabetes mellitus); genetic syndromes (ie, Prader-Willi syndrome, isolated gonadotropin-
releasing hormone deficiency, and Kallmann syndrome); and anatomic causes (ie, a pituitary
tumor or craniopharyngioma). Two additional causes of delayed puberty are constitutional delay
of growth and puberty, which is self-limited, and androgen insensitivity syndrome.

Height velocity is important to monitor in pediatric patients because deviations from normal
growth patterns can be the first sign of an underlying medical condition. Peak height velocity
occurs earlier in girls at sexual maturity rating (SMR) stage 2 to 3 versus boys at SMR stage 3 to
4. Normal peak height velocity occurs over a 2-year to 3-year period and ranges from 5 to 11
cm/year in boys and 6 to 10 cm/year in girls. Growth is 99% complete by age 17 years.

The evaluation for an arrest in height begins with a detailed history and physical examination to
determine if puberty started and then stopped or never started, and obtain information regarding
chronic illness, medical interventions (ie, chemotherapy), nutrition, and intense exercise. Height,
weight, and body mass index should be plotted on appropriate growth curves. The history and
physical examination will guide the choice of diagnostic studies. Laboratory evaluation often
includes a complete blood cell count and differential, thyroid-stimulating hormone, free
thyrotropin, liver function tests, and erythrocyte sedimentation rate.

For the girl described in the vignette, if she had no breast development, a karyotype should be
obtained. If the karyotype shows 45,X chromosomes, this is diagnostic for Turner syndrome. If
American Academy of Pediatrics 347
PREP ® Self-Assessment PREPSA 2021
the karyotype is normal, LH, FSH, and thyroid-stimulating hormone levels and bone age can be
obtained to evaluate for hypergonadotropic and hypogonadotropic hypogonadism, and
constitutional delay of growth and puberty. If the patient has a history of chronic disease, it is
likely functional hypogonadism, which will respond to disease management of the chronic
condition. For a body mass index less than 18 kg/m2, nutritional rehabilitation should be initiated
and the patient screened for an eating disorder. If the adolescent is age 16 years or older and has
not had a period but has had breast development, pelvic ultrasonography is indicated to evaluate
for a structural anomaly.

PREP Pearls
• Deviations in height from normal growth patterns can be the first sign of an underlying
medical condition.
• Female adolescents with weight loss who engage in intense exercise should be screened
for an eating disorder.
• Peak height velocity occurs at sexual maturity rating stage 2 to 3 in girls versus stage 3 to
4 in boys.

MOCA-Peds Objective
• Evaluate a child with hyperpigmentation.

ABP Content Specifications(s)


• Plan the appropriate evaluation of premature arrest of previously normal growth rate in
an adolescent

Suggested Readings
• Dye AM, Nelson GB, Diaz-Thomas A. Delayed puberty. Pediatr Ann. 2018;47(1):e16-
e22. doi:10.3928/19382359-20171215-01.
• Kritzler RK, Long D, Plotnick L. Puberty: normal and abnormal. In: McInerny TK,
Adam HM, Campbell DE, DeWitt TG, Foy JM, Kamat DM, eds. American Academy of
Pediatrics Textbook of Pediatric Care. 2nd ed. Elk Grove Village, IL: American
Academy of Pediatrics; 2017:1540-1544. Pediatric Care Online.
• Raivio T, Miettinen PJ. Constitutional delay of puberty versus congenital
hypogonadotropic hypogonadism: genetics, management and updates. Best Pract Res
Clin Endocrinol Metab. 2019;33(3):101316. doi:10.1016/j.beem.2019.101316.
• Wolf RM, Long D. Pubertal development. Pediatr Rev. 2016;37(7):292-300.
doi:10.1542/pir.2015-0065.

American Academy of Pediatrics 348


PREP ® Self-Assessment PREPSA 2021
Question 94
A 2-year-old girl with mild global developmental delay and refractory epilepsy is placed on a
ketogenic diet for management of her seizures. She experiences improved seizure control and is
followed closely by a pediatric epileptologist and dietician. Her mother brings her for her health
supervision visit and states that overall, the girl is tolerating the diet well with mild constipation
and is taking her multivitamin with calcium and vitamin D supplements daily. Her mother states
that she understands that the diet can have other side effects and asks if the girl should be
monitored for additional complications.

Of the following, the girl is MOST at risk for


A. carnitine deficiency
B. kidney stones
C. osteopenia
D. zinc deficiency

American Academy of Pediatrics 349


PREP ® Self-Assessment PREPSA 2021
Correct Answer: B
Children consuming a ketogenic diet are at risk for a number of complications, including kidney
stones. Although the most common adverse reactions are gastrointestinal (constipation, emesis,
or abdominal pain) and hyperlipidemia, kidney stones can occur in 3% to 7% of children on this
diet. Children who develop kidney stones generally do not require discontinuation of the diet.
The incidence of kidney stones can be reduced with the use of oral citrates, which should be used
empirically in children with additional risk factors. While on a ketogenic diet, a screening
urinary calcium-to-creatinine ratio measurement is recommended every 3 months. Renal
ultrasonography should be performed if signs or symptoms of kidney stones develop.

A ketogenic diet can be effective in the management of medically refractory epilepsy. With
initiation of the diet, half of children will show a greater than 50% reduction in seizure burden
independent of electroencephalography (EEG) findings, epilepsy type, or age. The mechanism of
action is unknown. Several conditions are recognized to benefit from early initiation of the
ketogenic diet, including glucose transporter 1 deficiency syndrome, pyruvate dehydrogenase
deficiency, epilepsy with myoclonic-atonic seizures, Dravet syndrome, infantile spasms,
tuberous sclerosis, and epileptic encephalopathies. Like medications, the ketogenic diet can have
adverse effects and complications because of its restricted nature; it should not be considered a
“natural” treatment.

Multidisciplinary care with a team that includes a neurologist, dietician, pharmacist, and
pediatrician is crucial for children placed on a ketogenic diet. Before initiating the diet, the
family should receive counseling regarding its use and maintenance, and the child should
undergo a comprehensive nutritional assessment and baseline laboratory studies (Item C94). The
diet is generally initiated in the hospital after a period of fasting; however, for some children a
modified form of the diet can be implemented in the outpatient setting.

Carnitine supplementation has been a controversial issue. Long-term use of a ketogenic diet has
been associated with low carnitine levels, which can produce symptoms ranging from fatigue and
muscle weakness to liver and heart problems. Carnitine levels are often monitored while on the
diet, with supplementation provided if levels are low or symptoms develop. Because of its
restricted nature, all children on the ketogenic diet should receive a carbohydrate-free
multivitamin with additional calcium and vitamin D supplementation daily to ensure adequate
nutrition and prevent the development of osteopenia. Zinc supplementation is not recommended
for children on the ketogenic diet, though periodic evaluation for trace mineral deficiencies may
be considered.

American Academy of Pediatrics 350


PREP ® Self-Assessment PREPSA 2021
PREP Pearls
• The ketogenic diet can be effective in the management of children with medically
refractory epilepsy. With initiation of the diet, half of children show a 50% or more
reduction in seizure burden independent of epilepsy type or electroencephalographic
findings.
• Common adverse events of the ketogenic diet include gastrointestinal complaints,
hyperlipidemia, and kidney stones.
• Carbohydrate-free multivitamin, calcium, and vitamin D supplementation is
recommended to prevent vitamin and mineral deficiencies, and the development of
osteopenia.

MOCA-Peds Objective
• Recognize the risk factors for nutritional insufficiency (eg, food insecurity, restricted
diets, etc.).

ABP Content Specifications(s)


• Plan the dietary management for a patient with a neurologic impairment

Suggested Readings
• Kossoff EH, Zupec-Kania BA, Auvin S, et al; Charlie Foundation, Matthew's Friends,
Practice Committee of the Child Neurology Society. Optimal clinical management for
children receiving dietary therapies for epilepsy: updated recommendations of the
International Ketogenic Diet Study Group. Epilepsia Open. 2018;3(2):175-192.
doi: 10.1002/epi4.12225.
• Luat A, Coyle L, Kamat D. The ketogenic diet: a practical guide for
pediatricians. Pediatr Ann. 2016;45(12):e446-e450. doi: 10.3928/19382359-20161109-
01.
• Roddy SM, McBride M. Seizure disorders. In: McInerny TK, Adam HM, Campbell DE,
DeWitt TG, Foy JM, Kamat DM, eds. American Academy of Pediatrics Textbook of
Pediatric Care. 2nd ed. Itasca, IL: American Academy of Pediatrics; 2016;chap
327:2599-2616. Pediatric Care Online.

American Academy of Pediatrics 351


PREP ® Self-Assessment PREPSA 2021
Question 95
A 2-year-old girl is brought to the emergency department by emergency medical services after
being found in the family garage with significantly altered mental status. She was lying in a
puddle of her own urine, with emesis around her mouth. Emergency medical technicians
suctioned her oropharynx and noted copious oral secretions. She has a temperature of 38.5°C, a
heart rate of 75 beats/min, a blood pressure of 95/65 mm Hg, and a respiratory rate of 14
breaths/min with shallow breaths and poor effort. Her pupils are pinpoint bilaterally, and
wheezing is heard throughout both lung fields. There are no signs of head trauma. The skin is
warm to the touch, and no rashes are noted. The remainder of the physical examination findings
are normal.

Of the following, the BEST medication to administer is


A. albuterol
B. atropine
C. naloxone
D. sodium bicarbonate

American Academy of Pediatrics 352


PREP ® Self-Assessment PREPSA 2021
Correct Answer: B
The girl in the vignette demonstrates signs of organophosphate toxicity, likely from exposure to
pesticides in the garage. Atropine is the best medication to administer for organophosphate
toxicity. Organophosphates stimulate both the sympathetic and parasympathetic nervous
systems; however, the typical clinical scenario of organophosphate toxicity involves symptoms
of overstimulation of the parasympathetic system, demonstrated in the girl in the vignette by
urination, emesis, bradycardia, bronchospasm, and miosis (pupillary constriction).

Organophosphates bind to and inactivate acetylcholinesterase in red blood cells, resulting in an


overabundance of acetylcholine. The parasympathetic effects of organophosphate poisoning can
be seen in multiple organ systems, including the heart and exocrine glands, and in smooth
muscles. Excess acetylcholine results in parasympathetic nervous system stimulation of both
nicotinic and muscarinic receptors. At low doses, the muscarinic symptoms predominate,
including the “SLUDGE” symptoms of salivation, lacrimation, urination, diarrhea,
gastrointestinal cramping, and emesis. Patients may also demonstrate miosis, bradycardia and
atrioventricular block, bronchospasm, and bronchorrhea. Overstimulation of nicotinic receptors
found at the neuromuscular junction can lead to fasciculations and myoclonic jerks; in severe
toxicity, it can cause flaccid paralysis. In the central nervous system, this results in headache,
slurred speech, ataxia, seizures, and coma. The most severe cases may result in respiratory
failure as a result of bronchospasm, excessive respiratory secretions, respiratory depression, or
paralysis of the respiratory muscles.

The first-line pharmacologic antidote for organophosphate poisoning is atropine, which


competes with acetylcholine at muscarinic receptors and reverses the effects of organophosphate
toxicity by blocking further parasympathetic activation. Pralidoxime is a cholinesterase
reactivator that can reverse the effects of respiratory and skeletal muscle weakness, and it should
be given to patients with weakness because atropine works only on muscarinic receptors.
Although albuterol may be helpful in the treatment of bronchospasm, it has no effect on the
primary toxicity of organophosphate poisoning. Naloxone is a pure opioid antagonist that acts as
a competitive antagonist at μ, κ, and Σ opioid receptors. Sodium bicarbonate will have no direct
effect on the toxicities seen in organophosphate poisoning.

PREP Pearls
• Organophosphate poisoning results in overstimulation of the parasympathetic nervous
system.
• Atropine is the antidote of choice for organophosphate poisoning.
• Atropine acts only on muscarinic acetylcholine receptors.

ABP Content Specifications(s)


• Recognize the signs and symptoms of organophosphate poisoning, and manage
appropriately

American Academy of Pediatrics 353


PREP ® Self-Assessment PREPSA 2021
Suggested Readings
• Fine JS. Poisoning. In: McInerny TK, Adam HM, Campbell DE, DeWitt TG, Foy JM,
Kamat DM, eds. American Academy of Pediatrics Textbook of Pediatric Care. 2nd ed.
Elk Grove Village, IL: American Academy of Pediatrics; 2017:2924-2949. Pediatric Care
Online .
• Roberts JR, Karr CJ; Council on Environmental Health. Pesticide exposure in
children. Pediatrics. 2012;130(6):e1765-e1788. doi:10.1542/peds.2012-2758.
• Zwiener RJ, Ginsburg CM. Organophosphate and carbamate poisoning in infants and
children (published correction appears in Pediatrics. 1988;81[5]:683). Pediatrics.
1988;81(1):121-126. https://pediatrics.aappublications.org/content/81/1/121.

American Academy of Pediatrics 354


PREP ® Self-Assessment PREPSA 2021
Question 96
A 16-year-old adolescent girl is seen in the clinic for a pregnancy test. The limits of
confidentiality are discussed. She reports that she recently met her older boyfriend on a social
media site. She sneaks out of her house to see him every night; her family does not know about
him. He frequently takes her to parties at which she participates in sexual activities with his
friends when he asks her to. He rewards her with clothes and jewelry. Her last sexual encounter
with his friend was 1 week ago and was unprotected. Physical examination findings are
unremarkable. The result of her pregnancy test is negative. She reports no abdominal or
genitourinary symptoms. Results of testing for sexually transmitted infections are pending.
Concerns for her safety are discussed. The patient requests that her parents not be told.

Of the following, the BEST next step in treatment is to contact


A. child protective services
B. the local law enforcement agency
C. a national trafficking organization hotline
D. the patient’s parents

American Academy of Pediatrics 355


PREP ® Self-Assessment PREPSA 2021
Correct Answer: C
Children and adolescents who experience sexual exploitation, including sex trafficking, human
trafficking, and commercial sexual exploitation (CSE), seek medical attention for a variety of
reasons but seldom self-identify. Unfortunately, with the rise of social media and the help of an
anonymous cyberspace environment, opportunities abound for the sexual exploitation of children
and adolescents by sex offenders. Many survivors perceive these sexually exploitative
relationships as consensual.

When responding to cases of suspected CSE, it is critical to avoid further harm to the patient.
Ideally, a national trafficking organization (such as the National Human Trafficking Resource
Center Hotline at 1-888-373-7888 sponsored by the Polaris Project) should be contacted initially
for assistance. The hotline can assist in identifying sensitive law enforcement agents to ensure
that the patient is treated as a victim of sexual exploitation rather than a criminal guilty of
prostitution. Although it is reasonable to contact the patient’s parents, state or local law
enforcement, state child protective services (CPS) agency, or local child advocacy centers, health
care providers, too, should be cognizant of potential harm that could occur.

Although the health care provider is likely a mandated reporter in most states, providers must use
extreme caution when making a report to CPS without first contacting a national trafficking
organization. Mandated reports to CPS may not actually result in a favorable outcome because of
the variability of services available and the possible lack of understanding by individual CPS
workers as to the unique issues facing victims of CSE. Additionally, although federal law
designates children and adolescents as victims who cannot consent to commercial sexual acts,
many states still view commercial sexual exploitation of children through the lens of prostitution
laws, thus treating minors as criminals rather than victims. Youth who are victims of CSE are
less likely to receive critical services and protection when they enter the juvenile justice system,
and they may suffer additional trauma and ultimately re-enter the world of trafficking upon
release.

It is critical for health care providers to be aware of possible risk factors and indicators for CSE
or trafficking because most victims will not self-identify or disclose this information (Item C96).
When potential indicators are identified, the health care provider may ask more direct questions,
although many barriers to disclosure exist, including fear of the exploiter, shame, hostility, and
intoxication. Thus, it is critical to build rapport with the individual, discuss limits of
confidentiality, and to avoid causing additional trauma.

American Academy of Pediatrics 356


PREP ® Self-Assessment PREPSA 2021

Direction questions that may be helpful to elicit a history of sexual exploitation include:
• Have you ever been asked to exchange sex for money, food, shelter, or other items?
American Academy of Pediatrics 357
PREP ® Self-Assessment PREPSA 2021
• Have you ever been asked by someone to have sex with another person?
• Have you ever had sexual photographs taken of you or posted on the internet?

Research shows that parents underestimate the likelihood that their child or teen will engage in
online contact with strangers; this highlights the importance of parents’ promoting Internet
safety, monitoring online usage, using parental control software, and maintaining awareness of
children’s Internet activities from an early age. Furthermore, health care professionals should
counsel patients and families and work with them to develop individualized family rules and
guidelines surrounding the Internet and social media usage, such as the American Academy of
Pediatrics Family Media Plan (www.healthychildren.org/MediaUsePlan). Providers should also
provide anticipatory guidance to pediatric patients about protecting themselves from
exploitation.

PREP Pearls
• Children and adolescents who have experienced sexual exploitation, including sex
trafficking, human trafficking, and commercial sexual exploitation, seek medical
attention for a variety of reasons but seldom self-identify.
• When responding to cases of suspected commercial sexual exploitation, a national
trafficking organization (such as the National Human Trafficking Resource Center
Hotline at 1-888-373-7888, sponsored by the Polaris Project) should be contacted for
assistance.
• Providers should also provide anticipatory guidance to pediatric patients about protecting
themselves from online exploitation.

MOCA-Peds Objective
• Respond appropriately to a child’s disclosure of sexual abuse.

ABP Content Specifications(s)


• Counsel patients regarding the proper use of the internet and social networking sites
• Understand the potential effects of various media on child and adolescent behavior

American Academy of Pediatrics 358


PREP ® Self-Assessment PREPSA 2021
Suggested Readings
• American Professional Society on the Abuse of Children Task Force. The commercial
sexual exploitation of children: the medical provider’s role in identification, assessment,
and treatment. ASPAC Practice Guidelines. Chicago, IL: American Professional Society
on the Abuse of Children; 2013. Available at: https://www.apsac.org/9235fgnl8.
• Brown AC, Barron CE. Human trafficking. Pediatr Rev. 2018;39(2):102-103.
doi:10.1542/pir.2016-0181.
• Chassiakos YLR, Radesky K, Christakis D, Moreno MA, Cross C; American Academy
of Pediatrics Council on Communications and Media. Children and adolescents and
digital media. Pediatrics. 2016;138(5):e20162593. doi:10.1542/peds.2016-2593.
• Greenbaum J, Bodrick N; Committee on Child Abuse and Neglect; Section on
International Child Health. Global human trafficking and child victimization. Pediatrics.
2017;140(6):e20173138. doi:10.1542/peds.2017-3138.
• Greenbaum J, Crawford-Jakubiak JE; Committee on Child Abuse and Neglect. Child sex
trafficking and commercial sexual exploitation: health care needs of victims. Pediatrics.
2015;135(3):566-574. doi:10.1542/peds.2014-4138.

American Academy of Pediatrics 359


PREP ® Self-Assessment PREPSA 2021
Question 97
A 4-year-old boy is seen for a health supervision visit. He has eczema and has had recurrent ear
infections that required the placement of myringotomy tubes. He attends preschool, knows his
letters, and can draw a triangle and a square. He is interactive during the physical examination.
His height is at the 30th percentile, his weight is at the 50th percentile, and his head
circumference is at the 70th percentile. He can hop on one foot for 2 seconds and sing his
favorite nursery rhyme. His mother reports that his father, paternal uncle, and grandmother have
short stature, a large head, small fingers, and shortening of both arms and thighs. His father has
been relatively healthy except for obstructive sleep apnea. His grandmother had stenosis of the
lumbar spine and hearing loss. His uncle recently had a newborn son with a large head and short
limbs who underwent ventriculoperitoneal shunt insertion because of obstructive hydrocephalus.

Of the following, the inheritance pattern of the disorder MOST likely seen in this family is
A. autosomal dominant with complete penetrance
B. autosomal dominant with incomplete penetrance
C. X-linked dominant
D. X-linked recessive

American Academy of Pediatrics 360


PREP ® Self-Assessment PREPSA 2021
Correct Answer: A
The disorder seen in the family described in the vignette is achondroplasia. Achondroplasia is
characterized by disproportionate short stature with rhizomelic (proximal segment) shortening
and macrocephaly. Facial features include a depressed nasal bridge with retrusion of the midface
and frontal bossing. Achondroplasia is inherited in an autosomal dominant pattern with complete
penetrance: all patients who carry the pathogenic variant will exhibit signs and symptoms of the
disorder. The patient in the vignette is not affected with achondroplasia, because he is of normal
height and does not have macrocephaly or rhizomelic shortening. Affected family members
include the patient’s father, paternal uncle, uncle’s son, and paternal grandmother (Item C97).
Other features seen in achondroplasia include limitation of elbow extension, a trident
configuration of the hands, bowed legs, and exaggerated lumbar lordosis once the child begins to
walk. Individuals with achondroplasia have normal intelligence and life span. Early motor delay
is common owing to hypotonia. Complications of the disorder include a risk of increased
intracranial pressure owing to obstructive hydrocephalus, craniocervical junction compression,
obstructive sleep apnea, conductive hearing loss, persistent kyphosis, and spinal stenosis (in
adults). Owing to risk of these complications, surveillance after diagnosis includes baseline
imaging of the craniocervical junction; ongoing assessment of hearing, development, and
growth; and a low threshold for evaluation of spinal stenosis in adults.

Item C97: Achondroplasia Pedigree.Courtesy of A Sidhu

Hereditary breast and ovarian cancer syndrome owing to pathogenic variants in the BRCA1 gene
is an example of autosomal-dominant inheritance pattern with incomplete penetrance. In this
condition, females with BRCA1 pathogenic variant have a lifetime risk of developing breast
cancer that is 80 percent rather than 100 percent. Hence, the disorder has incomplete penetrance,
American Academy of Pediatrics 361
PREP ® Self-Assessment PREPSA 2021
meaning that not everyone with the disease mutation has signs and symptoms of the condition.
The response choices of X-linked disorders are incorrect; the pedigree shows male-to-male
transmission, which would not be seen in X-linked disorders, whether dominant or recessive.

PREP Pearls
• Achondroplasia is characterized by disproportionate short stature with rhizomelic
shortening, macrocephaly, and facial features of frontal bossing and midface retrusion.
• The inheritance pattern of achondroplasia is autosomal dominant with complete
penetrance.

MOCA-Peds Objective
• Differentiate between normal and abnormal variations in head growth and manage
appropriately.

ABP Content Specifications(s)


• Recognize the inheritance pattern of achondroplasia

Suggested Readings
• Pauli RM, Legare JM. Achondroplasia. In: Adam MP, Ardinger HH, Pagon RA, Wallace
SE, Bean LJH, Stephens K, Amemiya A, eds. GeneReviews. 1998 Oct 12 [updated 2018
May 10] https://pubmed.ncbi.nlm.nih.gov/20301331/.
• Pereira E. Achondroplasia (published correction appears in Pediatr Rev. 2019
Aug;40[8]:438). Pediatr Rev. 2019 Jun;40(6):316-318. doi:10.1542/pir.2018-0009.
• Trotter T, Hall JG; American Academy of Pediatrics Committee on Genetics. Health
supervision for children with achondroplasia (published correction appears in Pediatrics.
2005 Dec;116[6]:1615). Pediatrics. 2005 Sep;116(3):771-783. doi:10.1542/peds.2005-
1440.

American Academy of Pediatrics 362


PREP ® Self-Assessment PREPSA 2021
Question 98
A previously healthy, 3-year-old boy has complained intermittently of pain in the left ear for the
last 4 weeks. He had rhinorrhea and a cough around the time the pain began, which self-resolved.
He has not had any fevers. He is developing appropriately.

On physical examination, the boy appears well. He has no rhinorrhea or nasal congestion. His
right tympanic membrane appears gray and translucent with good landmarks. His left tympanic
membrane appears gray with clear fluid visible behind it (Item Q98).

Of the following, the BEST next step in management of this boy’s condition is to
A. monitor clinically for 2 months
B. prescribe an antihistamine
C. refer for a hearing evaluation
D. refer for tympanostomy tube placement

American Academy of Pediatrics 363


PREP ® Self-Assessment PREPSA 2021
Correct Answer: A
The boy in the vignette has otitis media with effusion (OME), supported by the findings of clear
fluid behind a normal-appearing tympanic membrane and no fever. This condition likely began
after an upper respiratory tract infection. Otitis media with effusion occurs often in toddlers and
school-age children, especially those with allergic rhinitis or eustachian tube dysfunction. It is
commonly self-limiting, with resolution within 3 months.

Children with OME should be monitored for resolution of the fluid collection for a total of 3
months before further evaluation or treatment is recommended. Although the frequency of visits
is not standardized, periodic re-evaluation is warranted. At each follow-up visit, the appearance
of the tympanic membrane, including position; color and quality of any fluid; and visualization
of landmarks should be documented. The provider should inquire about symptoms such as
difficulty paying attention, delayed speech, or difficulties with balance. Accommodations can be
made for otherwise healthy children with mild transient hearing loss while observing for
resolution of the fluid, such as preferential seating in the classroom and facing the child when
speaking to him or her.

Treatment with antihistamines is not recommended, because this does not improve the long-term
outcome of OME and has associated risks in young children. Neither hearing evaluation nor
referral to otolaryngology for consideration of tympanostomy tube placement is recommended
for children with OME unless the fluid is continuously present for more than 3 months.
Children with chronic medical conditions affecting speech or balance, such as trisomy 21,
autism, or craniofacial abnormalities, are at increased risk of acquiring OME and require special
consideration during treatment of OME. They may require earlier intervention and more frequent
audiology testing.

PREP Pearls
• Otitis media with effusion is typically self-limiting and usually resolves within 3 months.
• Antihistamines are not recommended in the treatment of otitis media with effusion.

ABP Content Specifications(s)


• Recognize potential physical, behavioral, and developmental complications associated
with otitis media with effusion
• Recognize conditions (including allergic rhinitis, adenoidal hypertrophy, eustachian tube
abnormalities) associated with otitis media with effusion
• Understand the natural history of otitis media with effusion in patients of various ages
• Plan the appropriate initial and follow-up management of otitis media with effusion in
patients of various ages, including when complications occur

American Academy of Pediatrics 364


PREP ® Self-Assessment PREPSA 2021
Suggested Readings
• American Academy of Family Physicians, American Academy of Otolaryngology-Head
and Neck Surgery and American Academy of Pediatrics Subcommittee on Otitis Media
With Effusion Pediatrics. Otitis media With effusion. Pediatr Rev. 2004;113(5):1412-
1429.
doi:10.1542/peds.113.5.1412. https://pediatrics.aappublications.org/content/113/5/1412.
• Rosa-Olivares J, Porro A, Rodriguez-Varela M, Riefkohl G, Niroomand-Rad I. Otitis
media: to treat, to refer, to do nothing: a review for the practitioner. Pediatr Rev.
2015;36(11):480-488. doi:10.1542/pir.36-11-480.
• Tan TQ. Otitis media and otitis externa. In: McInerny TK, Adam HM, Campbell DE,
DeWitt TG, Foy JM, Kamat DM, eds. American Academy of Pediatrics Textbook of
Pediatric Care. 2nd ed. Elk Grove Village, IL: American Academy of Pediatrics;
2016:2452-2457. Pediatric Care Online.

American Academy of Pediatrics 365


PREP ® Self-Assessment PREPSA 2021
Question 99
A 6-year-old boy is being evaluated for concerns about his behaviors. He has an autism spectrum
disorder and is receiving special education services and in-home applied behavioral analysis
therapy. Despite using strategies learned during therapy, his mother reports difficulty managing
his behaviors. She is concerned about his safety when they are out in public, as he often darts
away from her. In stores, he pulls items down from the shelves. At school, he has difficulty
staying seated and has left the classroom without permission on several occasions. He is not
aggressive toward others and does not engage in self-injurious behaviors. Review of systems is
otherwise negative. The boy speaks using short complete sentences and makes some eye contact.
With encouragement, he cooperates with the physical examination, findings of which are within
normal limits.

Of the following, the BEST next management step for this boy is to
A. make a referral to occupational therapy
B. prescribe an atypical antipsychotic medication
C. request a more restrictive educational environment
D. screen for a coexisting mental health condition

American Academy of Pediatrics 366


PREP ® Self-Assessment PREPSA 2021
Correct Answer: D
The boy in the vignette has problematic behaviors of hyperactivity and elopement. His symptoms
are concerning for attention-deficit/hyperactivity disorder (ADHD), which commonly co-occurs
with autism spectrum disorder (ASD). As a child with ASD, this boy is at high risk for
coexisting medical and mental health conditions. The health care provider should be alert to this
possibility and periodically screen for these conditions, especially when parents have behavioral
concerns.

Autism spectrum disorder is a neurodevelopmental disorder with a prevalence of 1 in 59


children. The Diagnostic and Statistical Manual of Mental Disorders Fifth Edition (DSM-5)
criteria for the diagnosis of ASD include 1) social communication and interaction deficits, and 2)
restricted, repetitive patterns of behaviors, interests, or activities (See Hyman et al). Autism
spectrum disorder is highly heritable and has a biological basis. A genetic syndrome or
difference (eg, mutation, microdeletion, microduplication) can be identified in 10% to 20% of
cases of autism. Twin and sibling studies highlight the important role of genetic factors in ASD.
Neuroimaging, neuropathologic, and electrophysiologic studies have reported various brain
abnormalities, asymmetries, and differences in people with ASD.

The American Academy of Pediatrics recommends screening all children for autism at ages 18
and 24 months with an autism-specific standardized screening tool, such as the Modified
Checklist for Autism in Toddlers Revised with Follow-Up (MCHAT-R/F). A positive screening
result identifies the child as being at risk for autism and in need of a clinical diagnostic
evaluation. This screening is recommended because evidence shows that ASD can be identified
in toddlers and that providing early intervention services to these children improves outcomes.

For the child diagnosed with ASD, the pediatrician should conduct a medical workup that
includes genetic testing (eg, chromosomal microarray analysis, fragile X analysis) and/or referral
to genetics. If a specific genetic condition is suspected, targeted testing can be ordered
(eg, MECP2 testing for Rett syndrome for a girl with acquired microcephaly, stereotypic hand
movements, and developmental regression). The pediatrician should also ensure that the child
with ASD is referred for and/or receiving appropriate therapy and services. Those younger than 3
years should be referred to Early Intervention, whereas those aged 3 years and older should be
referred to their local school district (special education).

Behavioral, developmental, and educational interventions are the mainstay of treatment for ASD.
These include:
• Applied behavior analysis (ABA)
• Developmental relationship-focused interventions (DIR) (eg, Developmental Individual-
Difference, Relationship (DIR)/Floortime)
• Naturalistic developmental behavioral interventions (eg, Early Start Denver Model)
• Parent management training
• Classroom-based models (eg, Treatment and Education of Autistic and Related
Communication Handicapped Children [TEACCH])

Speech and language therapy and occupational therapy can also address the developmental needs
of children with ASD.
American Academy of Pediatrics 367
PREP ® Self-Assessment PREPSA 2021
The primary care physician should be aware of the common coexisting medical and mental
health conditions and problematic behaviors that may be seen in children and adults with ASD
(Item C99).

When concerning behaviors are present, the primary care physician should first evaluate for a
possible medical problem (eg, dental problem, constipation, otitis media), because pain or
discomfort could be the cause of behavioral change in children with ASD. Coexisting mental
health disorders should be treated, when appropriate, with behavioral interventions, but may
require the addition of psychopharmacologic agents.

Occupational therapy can help address fine motor–adaptive skills, sensory problems, and feeding
problems, but would be less useful for the problematic behaviors described in the vignette.
Children should be educated in the least restrictive environment possible. A more restrictive
educational environment would not address the concerning behaviors that are occurring at home

American Academy of Pediatrics 368


PREP ® Self-Assessment PREPSA 2021
and in public. The atypical antipsychotics risperidone and aripiprazole are approved by the US
Food and Drug Administration for the treatment of irritability in children with ASD and could be
considered for the child in the vignette; however, before beginning treatment, the best next
management step is to screen for a coexisting mental health condition. Identifying a coexisting
mental health condition and then targeting treatment of that condition would be a more effective
approach for this child with ASD.

PREP Pearls
• Children with autism are at increased risk for seizures (particularly those with intellectual
disability), gastrointestinal complaints, feeding disorders, obesity, pica, sleep disturbance,
and motor disorders.
• Common coexisting mental health conditions in children with autism spectrum disorder
(ASD) include attention-deficit/hyperactivity disorder (in ~50% of children with ASD),
anxiety (in 40%-66% of school-aged children and adults with ASD), and depressive
disorders (in 12%-33% of children and adults with ASD).
• When concerning behaviors are present in a child with ASD, the primary care physician
should first evaluate for a possible medical problem (eg, dental problem, constipation,
otitis media), because pain or discomfort could be the cause of behavioral change.

ABP Content Specifications(s)


• Plan age-appropriate screening evaluation for autism spectrum disorders
• Plan appropriate management for autism spectrum disorders
• Understand the diagnostic criteria for autism spectrum disorders
• Understand the biologic basis of autistic behavior

Suggested Readings
• Caronna EB, Costello EM, Augustyn M. Autism spectrum disorders. In: Augustyn M,
Zuckerman B, eds. Zuckerman Parker Handbook of Developmental and Behavioral
Pediatrics for Primary Care. 4th ed. Philadelphia, PA: Wolters Kluwer; 2019:140-146.
• Harrington JW, Allen K. The clinician’s guide to autism. P?ediatr Rev. 2014;35(2):62-
77. doi: 10.1542/pir.35-2-62.
• Hyman SL, Levy SE, Myers SM, Council on Children with Disabilities. Identification,
evaluation and management of children with autism spectrum disorder. P?ediatrics.
2020;145(1):e20193447. doi: 10.1542/peds.2019-3447.
• Kryszak E, Mulick JA, Butter EM. Autism spectrum disorder. In: McInerny TK, Adam
HM, Campbell DE, DeWitt TG, Foy JM, Kamat DM, eds. American Academy of
Pediatrics Textbook of Pediatric Care. 2nd ed. Itasca, IL: American Academy of
Pediatrics; 2016;chap 221:1777-1785. Pediatric Care Online.
• Myers SM, Challman TD. Autism spectrum disorder. In: Voigt RG, Macias MM, Myers
SM, Tapia CD, eds. Developmental and Behavioral Pediatrics. 2nd ed. Itasca, IL:
American Academy of Pediatrics; 2018:407-475.

American Academy of Pediatrics 369


PREP ® Self-Assessment PREPSA 2021
Question 100
A 6-year-old girl is seen by her pediatrician after failing a school vision screening. A subsequent
evaluation by an ophthalmologist reveals no vision in the right eye and bilateral optic disk
atrophy. She is referred to the emergency department. She has no significant medical history.
She appears well and in no distress. Her height is at the first percentile, and her weight is at the
seventh percentile. A growth chart provided by her mother shows that she was at the 15th
percentile for height 1 year ago. She has a temperature of 37.1°C, a heart rate of 92 beats/min, a
blood pressure of 108/78 mm Hg, a respiratory rate of 22 breaths/min, and an oxygen saturation
of 98% on room air. Other than a complete lack of discernible vision in her right eye, the
examination findings are unremarkable. Findings from emergent magnetic resonance imaging
are shown (Item Q100 ).

Item Q100: Magnetic resonance imaging for the patient described in the vignette.
Courtesy of J Fish

Of the following, the MOST likely outcome after initial treatment is


A. panhypopituitarism requiring lifelong hormone replacement
B. progression to herniation and respiratory failure
C. progressive loss of vision, gradually resulting in blindness
D. recurrent, metastatic disease refractory to therapy

American Academy of Pediatrics 370


PREP ® Self-Assessment PREPSA 2021
Correct Answer: A
The child in the vignette has experienced optic nerve atrophy due to tumor compression of the
optic nerves, and she has fallen from the 15th percentile to the first percentile on her growth
curve because of growth hormone deficiency resulting from pituitary destruction. This
presentation and the magnetic resonance image shown in the vignette (Item C100A) are most
consistent with a pituitary craniopharyngioma. Craniopharyngiomas are derived from pituitary
embryonic tissue, and they account for just under 10% of central nervous system tumors in
children. These tumors tend to be localized. The morbidity associated with them is due to
damage to the pituitary gland and the optic chiasm incurred as the tumor grows, and secondary to
treatment. As such, children with craniopharyngiomas frequently experience central
hypothyroidism, hypocortisolism, hypogonadism, growth hormone deficiency, and diabetes
insipidus, all of which require hormone replacement. Injury to the hypothalamus from treatment
increases the risk of hypothalamic hyperphagia. Given the position of the pituitary gland, large
pituitary tumors can compress the optic chiasm, resulting in bilateral vision loss. The treatment
of craniopharyngiomas is surgical resection, with radiation to those that are incompletely
resected. Overall 5-year survival is more than 80%. Children with hypopituitarism at diagnosis
tend to remain deficient in pituitary hormones, requiring lifelong hormone replacement.
Hypopituitarism frequently develops after treatment as well.

Item C100A: Magnetic resonance imaging consistent with a pituitary craniopharyngioma.


Courtesy of J Fish

If treated, craniopharyngiomas do not lead to herniation and respiratory failure. This outcome is
more closely associated with brainstem gliomas. Left untreated, the optic nerve compression due
to craniopharyngiomas can lead to complete vision loss. Once treated, however, the vision loss
tends to stabilize. It may not improve, and the child in the vignette may end up with permanent
vision deficits, but it also is unlikely to worsen. Craniopharyngiomas treated with complete
resection or partial resection and radiation tend to not recur, and they very rarely
metastasize. Item C100B shows the common locations of different types of brain tumors.

American Academy of Pediatrics 371


PREP ® Self-Assessment PREPSA 2021

Item C100B: Anatomic location of pediatric brain tumor subtypes


Reprinted with permission from Crawford J. Childhood brain tumors. Pediatr Rev.
2013;34(2):63-78.

PREP Pearls
• Children with craniopharyngiomas frequently experience central hypothyroidism,
hypocortisolism, hypogonadism, growth hormone deficiency, and diabetes insipidus, all
of which require hormone replacement.
• Craniopharyngiomas treated with complete resection or partial resection and radiation
tend to not recur, and they very rarely metastasize.
• Large pituitary tumors can compress the optic chiasm, resulting in bilateral vision loss.

MOCA-Peds Objective
• Recognize and plan initial evaluation of a child with a pituitary disorder.

ABP Content Specifications(s)


• Recognize the clinical findings associated with a brain tumor, including
craniopharyngioma

American Academy of Pediatrics 372


PREP ® Self-Assessment PREPSA 2021
Suggested Readings
• Crawford J. Childhood brain tumors. Pediatr Rev. 2013;34(2):63-78. doi: 10.1542/pir.34-
2-63.
• Müller HL, Merchant TE, Warmuth-Metz M, Martinez-Barbera JP, Puget S.
Craniopharyngioma. Nat Rev Dis Primers. 2019;5(1):75. doi: 10.1038/s41572-019-0125-
9.
• Rey-Casserly C, Diver T. Late effects of pediatric brain tumors. Curr Opin
Pediatr. 2019;31(6):789-796. doi:10.1097/MOP.0000000000000837.
• Rush S, Cohen BM. Brain tumors. In: McInerny TK, Adam HM, Campbell DE, DeWitt
TG, Foy JM, Kamat DM, eds. American Academy of Pediatrics Textbook of Pediatric
Care. 2nd ed. Elk Grove Village, IL: American Academy of Pediatrics; 2017:1792-
1796. Pediatric Care Online .

American Academy of Pediatrics 373


PREP ® Self-Assessment PREPSA 2021
Question 101
A 6-month-old male infant developed a pruritic rash 2 weeks ago. He has been otherwise well
and is taking no medications. There are numerous erythematous papules located on his trunk and
extremities, including the hands (Item Q101A) and feet (Item Q101B).

Item Q101A: Papules on the right hand of the infant described in the vignette. Courtesy of D
Krowchuk

Item Q101B: Papules on the left foot of the infant described in the vignette.
Courtesy of D Krowchuk

American Academy of Pediatrics 374


PREP ® Self-Assessment PREPSA 2021
Of the following, the MOST appropriate treatment is
A. cephalexin administered orally
B. hydrocortisone administered topically
C. hydroxyzine administered orally
D. permethrin administered topically

American Academy of Pediatrics 375


PREP ® Self-Assessment PREPSA 2021
Correct Answer: D
The infant described in the vignette has a generalized eruption composed of erythematous
papules. Notably, the eruption involves the hands and feet, including the palms and soles. These
findings suggest a diagnosis of scabies and, accordingly, the infant should be treated with
permethrin 5% cream topically. Hydrocortisone and hydroxyzine could be used adjunctively to
relieve pruritus, and cephalexin might be used if there was evidence of secondary bacterial
infection. However, none of these agents would eradicate the infestation. Although atopic
dermatitis may produce a generalized eruption in infants and, as a result, be confused with
scabies, the presence of large papules and involvement of the palms and soles is uncommon.

Scabies, from the Latin scabere, to scratch, results from infestation with the mite Sarcoptes
scabiei. Spread is primarily by direct contact with an infested individual (especially in settings of
environmental crowding), although fomites may be responsible. Two to three weeks after
infestation, pruritus and rash develop. Lesions are erythematous papules, nodules (Item C101A),
and burrows (Item C101B) located in the interdigital spaces, wrist flexures, axillae, and waist. In
girls, the areolae may be involved; in boys, papules and nodules may affect the penis and
scrotum (Item C101C). In infants, the eruption may be generalized and vesiculopustules may be
observed on the palms and soles. A variant of scabies, called “crusted” (or “Norwegian”)
scabies, occurs in immunocompromised or debilitated patients and presents as hyperkeratotic
plaques that may mimic eczema or psoriasis.

American Academy of Pediatrics 376


PREP ® Self-Assessment PREPSA 2021

Item C101A: Large papule and a papulovesicle on the dorsum of the foot of an infant who has
scabies. Courtesy of D Krowchuk

American Academy of Pediatrics 377


PREP ® Self-Assessment PREPSA 2021

Item C101B: Burrow (arrow) located on the heel of an infant who has scabies.
Courtesy of D Krowchuk

American Academy of Pediatrics 378


PREP ® Self-Assessment PREPSA 2021

Item C101C: Papules and nodules on the penis of a boy who has scabies.
Courtesy of D Krowchuk

Scabies is treated with permethrin cream 5% applied topically for 8 to 14 hours (ie, overnight). It
is approved for use in those 2 months of age or older but is recommended by many for the
treatment of scabies in younger infants. The cream is applied to the entire skin surface from the
neck to the toes. In infants and, possibly, in young children and the elderly, the head (including
the face) should be treated. Permethrin is not completely ovicidal and, for this reason, a second
treatment is recommended 7 to 14 days after the initial application. The symptoms and signs of
scabies represent a hypersensitivity reaction to the mite and its products; therefore, 2 to 4 weeks
may be required for the pruritus and rash to subside. Household or other close contacts may be
infested but not yet symptomatic and, as a result, ideally should receive a single application of
permethrin at the time the index case is first treated. Bed linens and clothing should be laundered
in hot water and dried at a high temperature, although some advise a simpler strategy of placing
items in a dryer at 60°C for 10 min. Items that cannot be treated in this manner may be stored in
a sealed plastic bag for a minimum of 3 days.

American Academy of Pediatrics 379


PREP ® Self-Assessment PREPSA 2021
An alternative to permethrin is oral ivermectin. It is approved for use in adults with
uncomplicated scabies (a single dose of 200 µg/kg repeated in 1 to 2 weeks) and crusted scabies
(a single daily dose on days 1, 2, 8, 9, and 15). Ivermectin is not approved for the treatment of
scabies in children and is not recommended for use in those who are younger than 5 years or who
weigh less than 15 kg. Other topical therapies for scabies exist, but their role in treatment is
limited. These include crotamiton (poor efficacy), lindane (poor efficacy, potential
neurotoxicity), and benzyl benzoate (contact dermatitis, neurologic toxicity if ingested).

PREP Pearls
• In children and adolescents, the lesions of scabies typically are located in flexural areas
(eg, between the digits, wrist flexors). However, in infants, the eruption is generalized
and the palms and soles often are affected.
• In boys, papules and nodules often are present on the penis and scrotum.
• Topical permethrin is the treatment of choice for scabies in children.
• Asymptomatic household contacts should be treated at the time the index case is first
treated.

MOCA-Peds Objective
• Know the indications and appropriate testing for parasitic disorders.

ABP Content Specifications(s)


• Plan the appropriate management of scabies
• Recognize the clinical manifestations of scabies

Suggested Readings
• Golant AK, Levitt JO. Scabies: a review of diagnosis and management based on mite
biology. Pediatr Rev. 2012;33(1):e1-e12. doi:10.1542/pir.33-1-e1.
• Hill TA, Cohen B. Scabies in babies. Pediatr Dermatol. 2017;34(6):690-694.
doi:10.1111/pde.13255.
• Mancini AJ, Krowchuk DP, eds. Pediatric Dermatology: A Quick Reference Guide. 3rd
ed. Elk Grove Village, IL: American Academy of Pediatrics; 2016.

American Academy of Pediatrics 380


PREP ® Self-Assessment PREPSA 2021
Question 102
A 17-year-old adolescent boy is seen for evaluation of lower back pain that has been present for
several months. He has no history of trauma or back injury. He reports that his back is stiff and
painful when he wakes in the morning and that he has intermittent bilateral hip pain. His
symptoms improve over the course of the day. His medical history is significant for plantar
fasciitis and anterior uveitis. He appears well and has age-appropriate vital signs. He has
tenderness over his sacroiliac joints and limited back flexion and extension. The remainder of his
physical examination, including joint and skin examinations, has normal findings.

Of the following, the laboratory result MOST commonly associated with the adolescent’s
condition is the presence of
A. anti–double-stranded DNA antibody
B. antinuclear antibody
C. human leukocyte antigen-B27
D. rheumatoid factor

American Academy of Pediatrics 381


PREP ® Self-Assessment PREPSA 2021
Correct Answer: C
The boy in the vignette has signs and symptoms consistent with enthesitis-related arthritis
(ERA), which is most commonly associated with the presence of human leukocyte antigen
(HLA)-B27. Juvenile ankylosing spondylitis is a type of ERA, with diagnostic criteria requiring
radiologic evidence of bilateral sacroiliitis

Although not all children with arthritis satisfy criteria for any specific category (and some meet
criteria for more than 1), the International League of Associations for Rheumatology
recommends the following categories of juvenile idiopathic arthritis:
• Systemic arthritis
• Polyarthritis (rheumatoid factor + and -)
• Oligoarthritis
• ERA
• Psoriatic arthritis
• Undifferentiated

Approximately 10% to 20% of children with juvenile idiopathic arthritis have ERA, which is
defined as:
• Arthritis with enthesitis or
• Arthritis and at least 2 of the following:
o Presence of or history of sacroiliac joint tenderness and/or inflammatory
lumbosacral pain
o HLA-B27 antigen
o Onset of arthritis in a boy older than 6 years
o Anterior uveitis
o Positive family history of ankylosing spondylitis, ERA, sacroiliitis with
inflammatory bowel disease, reactive arthritis, or acute anterior uveitis in a first-
degree relative

Enthesitis-related arthritis occurs more often in males, and the mean age at diagnosis is 12 years.
Enthesitis occurs most commonly at the patella or Achilles tendon insertion sites, and arthritis
and axial symptoms are typically worse in the morning and responsive to heat and nonsteroidal
anti-inflammatory medications. Between 60% and 90% of children with ERA have HLA-B27
antigens. Rheumatoid factor is usually negative, and anti–double-stranded DNA antibodies and
antinuclear antibodies are uncommon in patients with ERA.

American Academy of Pediatrics 382


PREP ® Self-Assessment PREPSA 2021

PREP Pearls
• Enthesitis-related arthritis is a subtype of juvenile idiopathic arthritis defined as arthritis
with enthesitis or arthritis and at least 2 of the following: presence or history of sacroiliac
joint tenderness and/or inflammatory lumbosacral pain; human leukocyte antigen-B27
antigen; onset of arthritis in a boy older than 6 years; anterior uveitis; and positive family
history of ankylosing spondylitis, enthesitis-related arthritis, sacroiliitis with
inflammatory bowel disease, reactive arthritis, or acute anterior uveitis.
• The majority of children with enthesitis-related arthritis have human leukocyte antigen-
B27 antigens.
• In patients with enthesitis-related arthritis, rheumatoid factor is usually negative, and
anti–double-stranded DNA antibodies and antinuclear antibodies are uncommon.

ABP Content Specifications(s)


• Recognize the clinical and laboratory findings associated with ankylosing spondylitis

Suggested Readings
• Martini A, Ravelli A, Avcin T, et al; fPediatric Rheumatology International Trials
Organization (PRINTO). Toward new classification criteria for juvenile idiopathic
arthritis: first steps, Pediatric Rheumatology International Trials Organization
International Consensus. J Rheumatol. 2019;46(2):190-97. doi: 10.3899/jrheum.180168.
• Petty RE, Southwood TR, Manners P, et al; International League of Associations for
Rheumatology classification of juvenile idiopathic arthritis; second revision, Edmonton,
2001. J Rheumatol. 2004;31(2):390-392. PMID: 14760812.
• Rosenthal A, Janow G. Enthesitis-related juvenile idiopathic arthritis. Pediatr
Rev. 2019;40(5): 256-258. doi: 10.1542/pir.2017-0177.
• Siegel DM, Gewanter HL, Sahai S. Rheumatologic diseases. In: McInerny TK, Adam
HM, Campbell DE, DeWitt TG, Foy JM, Kamat DM. Textbook of Pediatric Care. 2nd
ed. Elk Grove Village, IL: American Academy of Pediatrics; 2018:2578-2591. Pediatric
Care Online.

American Academy of Pediatrics 383


PREP ® Self-Assessment PREPSA 2021

Question 103
A 6-year-old girl with a 1-day history of fever, limp, and pain in the right lower extremity is
brought to the emergency department. Two weeks ago, she had a mild upper respiratory tract
infection. There has been no recent trauma. She is alert and interactive and has a temperature of
39.5°C. She is lying with her right lower extremity abducted and externally rotated. Examination
of the musculoskeletal system reveals discomfort with internal and external rotation of the right
hip. The remainder of the physical examination findings are normal. Laboratory findings are
notable for a white blood cell count of 18,000/μL (18 × 109/L) with 70% segmented neutrophils
and an elevated C-reactive protein level. Plain radiography of the right hip has normal findings,
and a blood culture is obtained.

Of the following, the BEST next step in management is to


A. administer ibuprofen
B. administer intravenous clindamycin
C. perform ultrasonography of the right hip
D. schedule orthopedic consultation for the next day

American Academy of Pediatrics 384


PREP ® Self-Assessment PREPSA 2021
Correct Answer: C
For the child described in the vignette, the clinical presentation of acute onset of fever, limp, and
limited range of motion of the right hip associated with leukocytosis and high C-reactive protein
(CRP) raises concern regarding septic arthritis of the hip. The best next step in management is to
obtain an ultrasonogram of the hip to help in excluding the diagnosis of septic arthritis. Detection
of a hip effusion requires a diagnostic aspiration of the joint space and analysis of the synovial
fluid. If the diagnosis of septic arthritis is suspected or confirmed, prompt surgical drainage and
irrigation of the joint space are recommended. Delay in surgical drainage can result in serious
complications (eg, aseptic necrosis of the femoral head) because increased intra-articular
pressure resulting from hip joint space infection compromises vascular flow. Other management
options after hip aspiration include obtaining a blood culture, initiating empiric antibiotic
therapy, and instituting pain management.

Septic arthritis in childhood most frequently results from hematogenous spread of bacteria,
although infection can also occur owing to local spread from a contiguous infection, as well as
because of traumatic or surgical infection.The bacterial etiology of septic arthritis varies with
age. In neonates, Staphylococcus aureus, group B Streptococcus, and gram-negative enteric
bacilli are the usual pathogens. Beyond the neonatal age group, S aureus is the most common
infecting organism in all age groups. Kingella kingae is a common cause of septic arthritis in
children younger than 5 years. Other important pathogens include group A streptococci and
Streptococcus pneumoniae. In unimmunized children, Haemophilus influenzae type b must be
considered. Neisseria gonorrhoeae must be considered in neonates and sexually active
adolescents. In addition to S aureus, Salmonella spp frequently cause osteomyelitis and septic
arthritis in patients with sickle cell anemia. Immunocompromised hosts are at high risk of
developing gram-negative bacterial infections. In chronic septic arthritis, mycobacteria and fungi
should be considered.

Septic arthritis is usually accompanied by fever, malaise, poor appetite, and irritability. In the
infant or neonate, there may be fever, poor feeding, lethargy, and pseudoparalysis of the
extremity. In the older child, the signs are more localized. Approximately 75% of cases of septic
arthritis involve the joints of the lower extremities. Limp or refusal to walk is the most frequent
presentation. The knee is the most commonly involved site, followed by the hip and ankle,
although the elbow and shoulder also may be affected. Examination of the infected joint will
reveal local erythema, warmth, swelling, pain, and decreased range of motion. Joint dislocation
may be observed. Smaller distal joints are less likely to be affected than are larger proximal
joints. Over 90% of children with septic arthritis have monoarticular joint infections.
Polyarticular infections may occur with certain pathogens (eg, N gonorrhoeae, Neisseria
meningitidis, Salmonella spp, and, rarely, S aureus).

Clinical suspicion of septic arthritis should lead to the examination of the joint fluid. Synovial
fluid aspirated should be sent for Gram staining, aerobic and anaerobic bacterial cultures, and
white blood cell (WBC) count with differential. Synovial fluid in septic arthritis is typically
turbid or grossly purulent. A synovial fluid WBC count of greater than 50,000 cells/mm3, with a
predominance of polymorphonuclear leukocytes is strongly suggestive of septic arthritis even if
the joint fluid culture finding is negative. The yield of organisms from joint fluid culture is about

American Academy of Pediatrics 385


PREP ® Self-Assessment PREPSA 2021
50% to 60%. Enhanced culture techniques or polymerase chain reaction of joint fluid may be
necessary to detect fastidious organisms such as K kingae.

The results of plain radiography may be normal, or they may show periarticular soft-tissue
swelling and widening of the joint space owing to a large joint effusion. Ultrasonography of the
hips is the modality of choice to detect joint effusion in a case of suspected septic arthritis of the
hip joint. Magnetic resonance imaging is highly sensitive for detection of inflammatory joint
fluid and is superior to computed tomography in delineation of adjacent bone and soft-tissue
structures.

The WBC count may be elevated, with a predominance of polymorphonuclear leukocytes. Blood
cultures are positive in 40% of patients with septic arthritis. Markers of systemic inflammation
including erythrocyte sedimentation rate (ESR) or CRP have been used as an adjunct to culture
for the diagnosis of bone and joint infections and for monitoring response to treatment. Levels of
CRP or ESR are elevated in most patients with septic arthritis. The ESR usually rises 3 to 5 days
after initiation of therapy and then slowly returns to normal within about 4 weeks. In contrast,
CRP peaks at day 2 of therapy and can normalize within 1 week in uncomplicated cases.

The initial empiric antimicrobial therapy in cases of septic arthritis beyond the neonatal period
should include an antistaphylococcal agent, either a β-lactamase–resistant penicillin (eg,
nafcillin, oxacillin), a first-generation cephalosporin (eg, cefazolin), or clindamycin or
vancomycin. In children aged 5 years or younger, in addition to S aureus, empiric coverage for K
kingae with ampicillin-sulbactam or second-generation or third-generation cephalosporin must
be considered. Empiric coverage for H influenzae type b with ceftriaxone is also warranted for
unimmunized patients. In sexually active adolescents, empiric coverage should include agents
active against N gonorrhoeae (such as ceftriaxone).

There is considerable experience in the sequential parenteral-oral regimen for antibiotic therapy
for osteomyelitis and septic arthritis. Oral antibiotic therapy can be instituted only when a
patient's clinical condition has improved (eg, resolution of fever, decrease in pain, increase in
mobility), when CRP levels are decreased, and when medication compliance and close
monitoring can be ensured. For oral therapy with β-lactam antibiotics, a dosage two to three
times that used for mild infections should be used to achieve high synovial fluid:serum ratios.
Besides being convenient for patients and families, oral therapy decreases the risk of
complications of long-term intravenous therapy. Erythrocyte sedimentation rate and CRP are
useful in monitoring response to therapy. The appropriate duration of therapy for septic arthritis
is still controversial and depends on the infecting pathogen, the joint involved, and the host.
However, it is advised to treat for at least 3 to 4 weeks in uncomplicated cases. Longer duration
of therapy (eg, 4 weeks) may be necessary for septic arthritis of the hip or shoulder; it may also
be necessary if the septic arthritis is caused by S aureus or gram-negative bacteria. Infection of a
small to medium joint such as the knee that is caused by S pneumoniae, H
influenzae, or Neisseria spp requires treatment of about 2 or 3 weeks.

In septic arthritis of the hip (or shoulder) joint, open surgical drainage should be performed
immediately; waiting until the next day for orthopedic consultation is not appropriate. In the
treatment of septic arthritis of joints other than the hip, open surgical drainage is not necessary
American Academy of Pediatrics 386
PREP ® Self-Assessment PREPSA 2021
and therapy can be individualized. Sequelae of septic arthritis in children are not uncommon and
include cartilage damage, stiffness of joint with poor mobility, abnormal bone growth if the
epiphysis is involved, unstable joint, and joint dislocation.

PREP Pearls
• Staphylococcus aureus is the most common cause of joint infections in all age groups;
microbial invasion of the synovial space typically results from hematogenous seeding.
• Early diagnosis via needle aspiration of the affected joint and prompt initiation of
appropriate antimicrobial therapy, in conjunction with drainage of the affected joint, are
critical to avoid destruction of the articular cartilage and prevent disability.
• Empiric antibiotic regimens should always include adequate antistaphylococcal coverage.
• Ultrasonography is the initial step in management of suspected septic arthritis of the hip.

ABP Content Specifications(s)


• Differentiate the clinical findings of pyogenic arthritis from those of toxic synovitis and
arthralgia
• Plan the appropriate diagnostic evaluation of synovitis
• Understand the natural history of pyogenic arthritis

Suggested Readings
• Brown DW, Sheffer BW. Pediatric septic arthritis: an update. Orthop Clin North Am.
2019;50(4):461-470. doi:10.1016/j.ocl.2019.05.003.
• Herman MJ, Martinek M. The limping child. Pediatr Rev. 2015;36(5):184-195.
doi:10.1542/pir.36-5-184.
• Wolf M. Knee pain in children, part II: limp and life-threatening conditions, hip
pathology and effusion. Pediatr Rev. 2016;37(2):72-76. doi:10.1542/pir.2015-0041.
• Yee-Guardino S, Goldfarb J. Septic arthritis. In: McInerny TK, Adam HM, Campbell
DE, DeWitt TG, Foy JM, Kamat DM, eds. American Academy of Pediatrics Textbook of
Pediatric Care. 2nd ed. Elk Grove Village, IL: American Academy of Pediatric;
2017:2617-2619. Pediatric Care Online.

American Academy of Pediatrics 387


PREP ® Self-Assessment PREPSA 2021
Question 104
A young couple is having a routine prenatal visit. The pregnancy has been uncomplicated and
they are looking forward to the arrival of their child. They ask how they can ensure optimal
bonding with their child in the neonatal period.

Of the following, the BEST recommendation for this family is to


A. have a grandparent stay with the family for the first month after the birth
B. have the neonate room in after allowing the mother to rest
C. initiate skin-to-skin contact with the neonate immediately after birth
D. set a regular, predictable schedule for feeding and sleeping

American Academy of Pediatrics 388


PREP ® Self-Assessment PREPSA 2021
Correct Answer: C
Placing the neonate skin-to-skin with her parent immediately after birth promotes parental
bonding with the child, and among the response choices, is the best recommendation for this
family. Rooming in, with newborn care provided in the mother’s room, also supports attachment
but does not need to wait until after the mother has rested. Although having a grandparent stay
with the family can provide valuable support and a predictable schedule for feeding and sleeping
helps the infant self-regulate, these are not directly and immediately beneficial to parent-infant
attachment and bonding.

Attachment between parent and infant begins in utero when amniotic fluid exposes the fetus to
the mother’s body odor and voice (via transmitted sounds). A newborn responds preferentially to
the mother’s voice and will turn toward and be soothed by her voice. This response provides
positive feedback to the mother and helps deepen her bond and her child’s attachment.

Development of parent-infant attachment can be more challenging when parents are separated
from their newborn or when there is significant stress. This may occur when the newborn is ill or
born premature and requires care in the neonatal intensive care unit (NICU). Emotional stress
may also be higher after a multiple birth, particularly when 1 or more of the infants require care
in the NICU. When there is loss during a multiple-gestation pregnancy or in the newborn period,
parents may have difficulty bonding with the surviving infant(s). Neonatal care providers can
support parent bonding and infant attachment by ensuring that family-centered, developmentally
supportive care is provided, particularly in these difficult circumstances.

PREP Pearls
• Placing the neonate skin-to-skin with a parent immediately after birth promotes parental
bonding with the child.
• Attachment between parent and neonate begins in utero when amniotic fluid exposes the
fetus to the mother’s body odor and voice (via transmitted sounds).
• Development of parent-infant attachment can be more challenging when parents are
separated from their newborn or when there is significant stress, such as that associated
with prematurity, illness, or multiple births.

ABP Content Specifications(s)


• Understand the various factors that influence parent-infant attachment

Suggested Readings
• Frazier KF, Scharf RJ. Parent-infant attachment. Pediatr Rev. 2015;36(1):41-42.
doi: 10.1542/pir.36-1-41.
• Westrup B. Family-centered developmentally supportive care. NeoReviews.
2014;15(8):e325-335. doi: 10.1542/neo.15-8-e325.

American Academy of Pediatrics 389


PREP ® Self-Assessment PREPSA 2021
Question 105
A previously healthy, 4-year-old girl is brought to the office by her mother for concerns about
prolonged diarrhea of 13 days’ duration. For the first 4 to 5 days of her illness, she had several
episodes of nonbilious, nonbloody vomiting and nonbloody diarrhea per day. Her mother kept
her hydrated by giving her multiple cups of apple juice, sports drinks, and ginger ale daily. Her
vomiting resolved, but her diarrhea has continued so her mother continued her hydration
regimen. The girl has remained afebrile throughout this illness and has not had other symptoms.
Her vital signs are normal, weight is appropriate and stable, and physical examination findings
are normal. Stool studies for bacterial culture, ova and parasites, and rotavirus are negative.

Of the following, the BEST advice to give this girl’s mother is to


A. admit her to the hospital
B. prescribe an antimotility agent
C. resume age-appropriate diet
D. refer her to a gastroenterologist

American Academy of Pediatrics 390


PREP ® Self-Assessment PREPSA 2021
Correct Answer: C
The child in the vignette has continued diarrhea because of her current hydration regimen of high
osmolality fluids. She should therefore resume a normal age-appropriate diet. She does not
require hospital admission because she is well hydrated. Antimotility agents are not
recommended for acute diarrhea. A gastroenterologist referral is unnecessary because her
primary care physician should be able to manage her current acute condition.

Acute diarrhea in children is most often caused by viral infection. Dehydration in this setting can
occur quickly and early or be gradual in onset. Early age-appropriate feeding can significantly
abate stool volume and duration of diarrhea, minimizing the degree of dehydration. Most
children will experience no dehydration at all; they should continue an age-appropriate, healthy
diet consisting of all food groups including dairy while avoiding foods high in fat and simple
sugars, such as juices. Excess juice intake may prolong diarrhea; the high carbohydrate load
exceeds the intestine’s absorption ability, leading to an osmotic diarrhea.

Children with mild to moderate dehydration in the setting of acute diarrhea should be given an
oral rehydration solution. This is the mainstay of therapy, because these solutions have been
designed to contain the appropriate osmolality and sodium to prevent further dehydration and
hypo- or hypernatremia. Item C105A compares the composition of inappropriate common
beverages (sports drinks) sometimes considered by parents for rehydration with that of oral
rehydration solutions currently recommended. Item C105B offers guidelines for how much and
how frequently oral rehydration solution should be given based on weight and degree of
dehydration.

American Academy of Pediatrics 391


PREP ® Self-Assessment PREPSA 2021

Children who have severe dehydration or who have hypo- or hypernatremic dehydration should
be given intravenous rehydration in the hospital setting.

PREP Pearls
• Most children with acute diarrheal illnesses will not experience dehydration and should
continue an age-appropriate healthy diet consisting of all food groups, including dairy;
foods high in fat and simple sugars, such as juices, should be avoided.
• Oral rehydration solution, with the appropriate osmolality and sodium content, is the
mainstay of therapy for children who have mild or moderate dehydration from an acute
diarrheal illness.
• Administration of appropriate oral rehydration solutions to children with acute diarrhea
may prevent severe dehydration and hypo- or hypernatremia that otherwise would require
intravenous rehydration.

ABP Content Specifications(s)


• Understand the role of oral rehydration solutions in the treatment of acute diarrheal
dehydration
• Understand the differences between and the rationale for the composition of oral
rehydration solutions

American Academy of Pediatrics 392


PREP ® Self-Assessment PREPSA 2021
Suggested Readings
• American Academy of Pediatrics Committee on Nutrition. Oral therapy for acute
diarrhea. In: Kleinman RE, Greer FR, eds. Pediatric Nutrition. 8th ed. Itasca, IL:
American Academy of Pediatrics; 2019:815-828.
• CaJacob NJ, Cohen MB. Update on diarrhea. Pediatr Rev. 2016;37(8):313-322.
doi: 10.1542/pir.2015-0099.
• Heyman MB, Abrams SA; Section on Gastroenterology, Hepatology, and Nutrition, and
Committee on Nutrition. Fruit juice in infants, children, and adolescents: current
recommendations. Pediatrics. 2017;139(6):e20170967. doi: 10.1542/peds.2017-0967.
• Powers KS. Dehydration: isonatremic, hyponatremic, and hypernatremic recognition and
management. Pediatr Rev. 2015;36(7):274-285. doi: 10.1542/pir.36-7-274.
• Shane AL, Mody RK, Crump JA, et al. 2017 Infectious diseases society of america
clinical practice guidelines for the diagnosis and management of infectious diarrhea. Clin
Inf Dis. 2017;65(12):1963-1973. doi: 10.1093/cid/cix959.

American Academy of Pediatrics 393


PREP ® Self-Assessment PREPSA 2021
Question 106
A 15-year-old, previously healthy adolescent is brought to the emergency department directly
from football practice; he complained of left upper quadrant abdominal pain after being tackled.
There was no loss of consciousness; he does not have a headache and has not vomited. He has
left shoulder pain that is worse with inspiration. Vital signs include a temperature of 37.2°C,
heart rate of 100 beats/min, respiratory rate of 20 breaths/min, blood pressure of 106/65 mm Hg,
and room air oxygen saturation on pulse oximetry of 100%. He appears to be in pain but is alert
and oriented. His examination findings are significant for left upper quadrant tenderness and
slight abdominal distention. His skin is warm and well-perfused; he has good range of motion of
all extremities. Two large-bore intravenous lines are placed, and a chest radiograph has normal
findings.

Of the following, the BEST abdominal imaging modality to confirm the diagnosis is
A. computed tomography
B. magnetic resonance imaging
C. radiography
D. ultrasonography

American Academy of Pediatrics 394


PREP ® Self-Assessment PREPSA 2021
Correct Answer: A
The adolescent in the vignette has sustained a splenic injury. Despite this potentially severe
injury, he is currently hemodynamically stable. Computed tomography with intravenous contrast
is the imaging modality of choice to confirm specific intra-abdominal injuries in children and
adolescents with blunt abdominal trauma who are hemodynamically stable.

The spleen resides in the left upper quadrant and is protected by the 9th through 11th ribs. It
functions as an organ of the hematopoietic system by filtering red blood cells and playing a role
in both humoral and cell-mediated immunity. The spleen is vulnerable to injury during blunt
abdominal trauma, particularly in children, because the chest wall is more pliable than in adults.
In younger children there may be significant injury to intra-abdominal organs without any
external signs of trauma. Certain conditions that cause enlargement of the spleen (eg, infectious
mononucleosis) may put an individual at higher risk for splenic injury. The diagnosis of splenic
injury should be considered after blunt abdominal trauma if there is left upper quadrant pain,
tenderness, or bruising. Left shoulder pain may result from irritation of the left hemidiaphragm
by blood (Kehr sign).

Evaluation of a child who has sustained trauma involves a primary survey, which dictates a
sequential evaluation of the airway, breathing, circulation, and disability (Glascow Coma Scale,
pupil size, reactivity and symmetry, and blood glucose level). Any life-threatening injuries
identified should be treated immediately (Item C106). At the end of the primary survey, the
child should be exposed (while respecting his/her dignity and minimizing heat loss) for a
thorough secondary survey; this may include the use of evaluation adjuncts, such as a focused
assessment sonography in trauma (FAST) examination, chest and pelvic radiographs, laboratory
analyses, and electrocardiography.

American Academy of Pediatrics 395


PREP ® Self-Assessment PREPSA 2021

In hemodynamically stable children with blunt abdominal trauma, FAST examination is of


limited use. A FAST examination that includes sonographic views of Morrison pouch, the
perisplenic recess, and the retrovesicular area can detect free intraperitoneal fluid; however, this
imaging modality is not used to diagnose specific injuries. Detection of free fluid on a FAST
examination in a hemodynamically stable patient indicates the need for further evaluation. The
absence of intraperitoneal free fluid on FAST examination in a hemodynamically stable patient
should not be used as the only decision point on which to determine the need for further
evaluation. The presence of intra-abdominal free fluid on FAST examination after blunt
abdominal trauma in a child who is not hemodynamically stable may obviate the need for further
American Academy of Pediatrics 396
PREP ® Self-Assessment PREPSA 2021
imaging and result in the patient being taken directly to the operating room to identify the source
of the bleeding.

Most splenic injuries in hemodynamically stable children are managed with supportive care and
do not require operative intervention. Hemodynamically unstable patients with a splenic injury
will require either surgical repair or a splenectomy. If splenectomy is required, it is important to
evaluate the child’s immunization status and to consider prophylactic antibiotics.
There are no current indications for the use of magnetic resonance imaging for the initial
evaluation of blunt abdominal trauma. Abdominal radiography is unlikely to yield any useful
information in the setting of trauma; however, radiography of the pelvic bones may be indicated.

PREP Pearls
• Computed tomography with intravenous contrast is the imaging modality of choice for
confirmation of splenic injury in the setting of blunt abdominal trauma.
• A hemodynamically unstable patient with blunt abdominal trauma and a focused
assessment sonography in trauma examination that shows free intraperitoneal fluid
should be considered for direct transfer to the operating room without further imaging.
• The primary survey of a child who has sustained trauma should be done in a stepwise
fashion, beginning with evaluation and management of the airway, breathing, circulation,
and disability (Glascow Coma Scale, pupillary reaction, size and symmetry, blood
glucose level).

ABP Content Specifications(s)


• Plan the appropriate initial evaluation in a patient with a suspected ruptured spleen

Suggested Readings
• American College of Surgeons. Abdominal and pelvic trauma . In: ATLS Advanced
Trauma Life Support Student Course Manual. 10th ed. Chicago, IL: American College of
Surgeons; 2018:82-101.
• Fein DM, Fagan MJ. Overall approach to trauma in the emergency department. Pediatr
Rev. 2018;39(10):479-489. doi: 10.1542/pir.2017-0246.
• Stengel D, Leisterer J, Ferrada P, Ekkernkamp A, Mutze S, Hoenning A. Point-of-care
ultrasonography for diagnosing thoracoabdominal injuries in patients with blunt
trauma. Cochrane Database Syst Rev. 2018;12:CD012669.
doi: 10.1002/14651858.CD012669.pub2.

American Academy of Pediatrics 397


PREP ® Self-Assessment PREPSA 2021
Question 107
A 2-month-old infant is brought to the clinic for evaluation of jaundice noted last week by her
family members. She has been otherwise well with normal growth and development. She is
breastfed exclusively. She has multiple stools daily; her parents describe the color as green and
brown. The infant was born at term; the pregnancy and delivery were uncomplicated. She had
mild jaundice for the first week after birth, which resolved without phototherapy. Her newborn
screening results were normal.

Her physical examination reveals a weight of 4.4 kg (25th percentile for age), length of 55 cm
(25th percentile for age), and weight-length ratio at the 40th percentile for age. She has scleral
icterus. Her abdomen is soft and nondistended, with a liver edge palpable 2 to 3 cm below the
right sternal costal margin. There is no splenomegaly. The remainder of her physical
examination findings are normal. Item Q107 shows the appearance of her stool in the diaper.

American Academy of Pediatrics 398


PREP ® Self-Assessment PREPSA 2021

Item Q107: Stool.Courtesy of J Sullivan


American Academy of Pediatrics 399
PREP ® Self-Assessment PREPSA 2021

Laboratory data are as follows:


Laboratory Test Result
Alanine transaminase 83 U/L
Aspartate transaminase 147 U/L
γ-glutamyltransferase 1,662 U/L
Total bilirubin 9.7 mg/dL (166 μmol/L)
Direct bilirubin 6.5 mg/dL (111 μmol/L)
White blood cell count 12,200/µL (12.2 × 109/L)
Hematocrit 36.4%
Platelet count 530 × 103/µL (530 × 109/L)

Urinalysis, urine culture, and urine cytomegalovirus testing results are pending.

Of the following, the best NEXT test to perform is


A. abdominal ultrasonography
B. liver biopsy
C. morning cortisol level
D. sweat chloride

American Academy of Pediatrics 400


PREP ® Self-Assessment PREPSA 2021
Correct Answer: A
The infant in the vignette has cholestasis as evidenced by the presence of conjugated
hyperbilirubinemia. This is likely pathologic and requires prompt diagnosis and treatment. In
addition to the testing already performed, stool color should be assessed for identification of
acholic stools. Examples of acholic stool appearance can be seen in Item C107. The initial
evaluation for the cause of cholestasis in this infant should include obtaining α1-antitrypsin
phenotype and level, thyroid function tests, abdominal ultrasonography, and reviewing the
newborn screen results.

American Academy of Pediatrics 401


PREP ® Self-Assessment PREPSA 2021

American Academy of Pediatrics 402


PREP ® Self-Assessment PREPSA 2021

Item C107: Acholic stools. If the stool color resembles the specimen numbered 1–3 (white, clay-
colored, or light yellowish), biliary atresia should be considered.
Reprinted with permission from Chen SM, Chang MH, Du JC, et al. Screening for Biliary
Atresia by Infant Stool Color Card in Taiwan. Pediatrics. 2006;117(4):1148.

The most common causes of cholestasis in the first few months after birth include biliary atresia
(BA), idiopathic neonatal cholestasis, parenteral nutrition–associated cholestasis (in infants
receiving prolonged parenteral nutrition), obstruction caused by common bile duct gallstone or
choledochal cyst, cystic fibrosis, and metabolic/endocrine disorders including tyrosinemia,
panhypopituitarism, and Alagille syndrome.

In infants who appear to have jaundice after age 2 to 3 weeks, particularly in the presence of
acholic stools, serum total and direct (conjugated) bilirubin levels should be measured. A direct
bilirubin level greater than 1.0 mg/dL (17.1 μmol/L) is abnormal; further evaluation is required
expeditiously and urgent referral should be made to a pediatric gastroenterologist. Initial
evaluation for the cholestatic neonate or infant includes testing that can quickly: 1) determine the
severity of liver disease and 2) inform the care team of the results. If no etiology is determined
on initial evaluation, further evaluation for specific diseases, including BA, should be
performed.

Biliary atresia, a progressive, fibro-obliterative neonatal liver disease, needs to be considered in


an urgent fashion because a delayed diagnosis (after 3 months of age) can result in poor
outcomes and the need for liver transplantation. It is imperative to quickly identify and refer
neonates and infants with suspected biliary atresia to both a pediatric gastroenterologist and
pediatric surgeon.

Evaluation for BA includes:


• liver function tests (including markers of liver synthetic function, such as albumin,
glucose, prothrombin time, and ammonia)
• γ-glutamyltransferase (which is often very elevated in BA)
• liver biopsy (looking for evidence of fibrosis, bile duct plugging, and bile ductular
proliferation).

The diagnosis of BA is confirmed with intraoperative cholangiography performed by a pediatric


surgeon. A Kasai hepatoportoenterostomy (HPE) is required to reestablish bile flow. If the Kasai
HPE is performed before age 60 days, approximately two-thirds of patients will have improved
biliary flow and not need urgent liver transplantation; most of these children will still develop
cirrhosis and portal hypertension, and many still need a liver transplant later in life. If the Kasai
HPE is performed after age 90 days, outcomes are significantly worse and most of these infants
will need a liver transplant before age 2 years.

For the infant in this vignette, a liver biopsy is not the best next test because the initial tier of
testing has not yet been performed. Testing for panhypopituitarism and cystic fibrosis should not

American Academy of Pediatrics 403


PREP ® Self-Assessment PREPSA 2021
be performed unless the initial testing tier is inconclusive. In addition, panhypopituitarism is
generally associated with normal levels of γ-glutamyltransferase.

PREP Pearls
• The presence of jaundice and acholic stools in a neonate/young infant should prompt
urgent assessment.
• Evaluation of cholestasis, evidenced by an elevated conjugated (direct) bilirubin level of
>1.0 mg/dL (17.1 µmol/L), should include abdominal ultrasonography and urgent
consultation with a pediatric gastroenterologist and pediatric surgeon.
• Biliary atresia is the most common cause of neonatal cholestasis; initial management with
a Kasai hepatoportoenterostomy is most efficacious in infants younger than 60 days.

ABP Content Specifications(s)


• Plan the appropriate diagnostic evaluation for a patient in whom biliary atresia is
suspected
• Recognize the clinical features associated with a choledochal cyst
• Recognize the clinical features associated with biliary atresia, and manage appropriately
• Plan the initial management of obstructive jaundice

Suggested Readings
• Brumbaugh D, Mack C. Conjugated hyperbilirubinemia in children. Pediatr Rev.
2012;33(7)291-302. doi: 10.1542/pir.33-7-291.
• Fawaz R, Baumann U, Ekong U, et al. Guideline for the evaluation of cholestatic
jaundice in infants: joint recommendations of the North American Society for Pediatric
Gastroenterology, Hepatology, and Nutrition and the European Society for Pediatric
Gastroenterology, Hepatology and Nutrition. J Pediatr Gastroenterol
Nutr. 2017;64(1):154-168. doi: 10.1097/MPG.0000000000001334.
• Pan D, Rivas Y. Jaundice: newborn to age 2 months. Pediatr Rev. 2017;38(11):499-510.
doi: 10.1542/pir.2015-0132.
• Pan DH, Rivas Y. Jaundice. In: McInerny TK, Adam HM, Campbell DE, DeWitt TG,
Foy JM, Kamat DM, eds. American Academy of Pediatrics Textbook of Pediatric Care.
2nd ed. Itasca, IL: American Academy of Pediatrics; 2016;chap 170:1473-1479. Pediatric
Care Online.

American Academy of Pediatrics 404


PREP ® Self-Assessment PREPSA 2021
Question 108
A 4-month-old infant is seen in the office as a new patient. She has a seizure disorder that is
controlled by medication. She is doing well, and her mother has no concerns regarding her
eating, growth, or development. Her physical examination findings are unremarkable, except for
multiple hypopigmented macules (Item Q108). Her father has similar hypopigmented lesions on
the skin, a benign brain tumor, and renal cysts.

Item Q108: Findings for the infant described in the vignette.


Reprinted with permission from Mancini AJ, Krowchuk DP, eds. Pediatric Dermatology. A
Quick Reference Guide. 3rd ed. Elk Grove Village, IL: American Academy of Pediatrics; 2016
American Academy of Pediatrics 405
PREP ® Self-Assessment PREPSA 2021

Of the following, the MOST appropriate study for this infant is


A. echocardiography to evaluate for rhabdomyomas
B. echocardiography to evaluate for ventricular septal defect(s)
C. electrocardiography to evaluate for atrioventricular block
D. electrocardiography to evaluate for prolonged QT interval

American Academy of Pediatrics 406


PREP ® Self-Assessment PREPSA 2021
Correct Answer: A
The patient in the vignette has a seizure disorder and hypopigmented lesions, as well as a family
history suggestive of tuberous sclerosis. Cardiac rhabdomyomas are commonly seen in infants
with tuberous sclerosis, and therefore an echocardiogram to evaluate for these lesions is
warranted. Tuberous sclerosis is not associated with ventricular septal defects, atrioventricular
block, or prolonged QT interval.

Tuberous sclerosis complex (TSC) is an autosomal-dominant, neurocutaneous, multisystem


disorder that results in localized cellular overgrowth that, in turn, leads to benign tumors
(hamartomas) in multiple organs, most commonly the brain, skin, kidney, and eye. It has two
gene mutations, TSC1 (chromosome 9q34) and TSC2 (chromosome 16p3). Manifestations of the
disease can be variable. The diagnostic criteria established by the TSC Consensus conference is
described: a definite diagnosis requires two major features or one major feature with more than
two minor features or the presence of a TSC1 or TSC2 mutation; a possible diagnosis requires
either one major feature or more than two minor features (see Staley BA, Vail EA, Thiele EA).
Tuberous sclerosis complex can be variable in degree of disease and timing of onset of different
criteria. A 2017 study by Davis and colleagues demonstrated the variability of features at initial
presentation (Item C108 ). Cardiac rhabdomyomas are typically benign and regress with time,
although they may cause arrhythmias and obstruction to blood flow.

American Academy of Pediatrics 407


PREP ® Self-Assessment PREPSA 2021

PREP Pearls
• Tuberous sclerosis complex is an autosomal-dominant, neurocutaneous, multisystem
disorder that results in hamartomas most often in the skin, brain, kidney, and eye.
• Cardiac rhabdomyomas are common early in life and are typically benign and regress
with time, although they may cause arrhythmias and obstruction to blood flow.

ABP Content Specifications(s)


• Recognize cardiac conditions associated with tuberous sclerosis

American Academy of Pediatrics 408


PREP ® Self-Assessment PREPSA 2021
Suggested Readings
• Davis PE, Filip-Dhima R, Sideridis G, et al; Tuberous Sclerosis Complex Autism Center
of Excellence Research Network. Presentation and diagnosis of tuberous sclerosis
complex in infants. Pediatrics. 2017;140(6):e20164040. doi:10.1542/peds.2016-4040.
• DiMario FJ Jr, Sahin M, Ebrahimi-Fakhari D. Tuberous sclerosis complex. Pediatr Clin
North Am. 2015;62(3):633-648. doi:10.1016/j.pcl.2015.03.005.
• Dove DE, Smith ML. Neurocutaneous syndromes. In: McInerny TK, Adam HM,
Campbell DE, DeWitt TG, Foy JM, Kamat DM, eds. American Academy of Pediatrics
Textbook of Pediatric Care. 2nd ed. Elk Grove Village, IL: American Academy of
Pediatrics; 2017:2379-2395. Pediatric Care Online .
• Staley BA, Vail EA, Thiele EA. Tuberous sclerosis complex: diagnostic challenges,
presenting symptoms, and commonly missed signs. Pediatrics. 2011;127(1):e117-e125.
doi:10.1542/peds.2010-0192.

American Academy of Pediatrics 409


PREP ® Self-Assessment PREPSA 2021
Question 109
A female infant born at 26 weeks’ gestational age required tracheal intubation and ventilator
management for 6 weeks. She then underwent extubation and was placed on nasal continuous
positive airway pressure (CPAP) and eventually weaned to nasal cannula oxygen at 0.1 L/min.
Since her discharge from the hospital at 4 months of age, the girl has had poor growth and
repeated episodes of respiratory illness with noisy breathing, stertor, and coarse wheezing that
does not respond to inhaled bronchodilators. She is currently 9 months old, and her growth
trajectory has continued to decline since hospital discharge despite high-calorie infant formula
and additional nutritional support.

Of the following, the MOST accurate statement regarding this girl’s symptoms is that
A. she will likely require tracheostomy and long-term ventilator management
B. the noisy breathing and stertor are likely to improve by 18 to 24 months of age
C. use of inhaled corticosteroids will improve her wheezing
D. use of inhaled ipratropium may be beneficial

American Academy of Pediatrics 410


PREP ® Self-Assessment PREPSA 2021
Correct Answer: A
The infant in the vignette has critical tracheomalacia impairing ventilation, resulting in chronic
respiratory failure, which is leading to growth failure. Since she was successfully weaned from
ventilator support, the infant has had progressive respiratory symptoms unresponsive to
bronchodilator treatment, which are negatively affecting her health. She will likely require
tracheostomy and long-term ventilation to support her respiratory status and promote a good
long-term outcome.

Tracheomalacia should be considered in infants who have recurrent wheezing that does not seem
to be reactive. Most infants with tracheomalacia will improve by 18 to 24 months of age and do
not have respiratory compromise requiring ventilator support. However, the infant in the vignette
has severe symptoms that are unlikely to improve in the typical time frame. Premature infants
and those who have had tracheal intubation are the patients most at risk for tracheomalacia.
Wheezing unresponsive to bronchodilators is not likely to benefit from inhaled corticosteroids or
muscarinic agents (ie, ipratropium).

Laryngomalacia is associated with noisy breathing and stertor (low-pitched inspiratory sound) or
stridor, but not wheezing. Laryngomalacia is rarely responsible for recurrent respiratory illnesses
and is likely to improve by the second year after birth. Critical laryngomalacia may be associated
with poor feeding, oxygen desaturation, and poor growth. However, laryngomalacia of that
severity would require surgical laryngoplasty rather than observation for 1 to 2 years.

Both laryngomalacia and tracheomalacia represent dynamic airway obstruction. Laryngomalacia


is characterized by inspiratory stridor/stertor, the hallmark of an extrathoracic airway
obstruction. With increased negative intrathoracic pressure, air moving over the narrowed
epiglottis and larynx generates a high-pitched inspiratory noise. Tracheomalacia is characterized
by expiratory wheezing that is coarse and has a monophonic quality, compared with the
polyphonic or musical quality of small airway wheezing related to asthma or bronchiolitis.
Intrathoracic sources of airway obstruction are associated with expiratory wheezing, because
increased intrathoracic pressure during expiration causes collapse of small airways (asthma,
bronchiolitis) or inadequately supported large airways (tracheomalacia).

PREP Pearls
• Tracheomalacia is common in premature infants.
• Most congenital tracheomalacia will improve with time and maturity.
• Critical tracheomalacia may require long-term intubation and mechanical ventilation.

ABP Content Specifications(s)


• Understand the various etiologies of tracheomalacia
• Recognize the clinical findings associated with tracheomalacia and laryngomalacia

American Academy of Pediatrics 411


PREP ® Self-Assessment PREPSA 2021
Suggested Readings
• Conrad C, Cornfield DN. Airway obstruction. In: McInerny TK, Adam HM, Campbell
DE, DeWitt TG, Foy JM, Kamat DM, eds. American Academy of Pediatrics Textbook of
Pediatric Care. 2nd ed. Itasca, IL: American Academy of Pediatrics; 2016;chap
348:2777-2785. Pediatric Care Online.
• Fraga JC, Jennings RW, Kim PC. Pediatric tracheomalacia. Semin Pediatr
Surg. 2016;25:156-164. doi: 10.1053/j.sempedsurg.2016.02.008.
• Hysinger EB, Panitch HB. Paediatric tracheomalacia. Paedtr Respir Rev. 2016;17:9-15.
doi: 10.1016/j.prrv.2015.03.002.

American Academy of Pediatrics 412


PREP ® Self-Assessment PREPSA 2021
Question 110
A 6-month-old girl with Down syndrome and a complete atrioventricular septal defect is
admitted for surgical repair of the heart defect. On postoperative day 5, she develops a fever. Her
vital signs include a temperature of 39.6°C, heart rate of 158 beats/min, respiratory rate of 53
breaths/min, and blood pressure of 85/41 mm Hg. She is receiving mechanical ventilation. Her
breath sounds are coarse. There is a II/VI systolic murmur, and the liver edge is palpable 3 cm
below the costal margin.

Laboratory tests show the following:


Laboratory Test Result
White blood cells 14,500/µL (14.5 × 109/L)
Hemoglobin 9.0 g/dL (90 g/L)
Platelet count 270 × 103/µL (270 × 109/L)
Differential count 72% segmented neutrophils, 12% lymphocytes, 16% monocytes
C-reactive protein 5.9 mg/L (56 nmol/L)
Blood culture Enterococcus gallinarum
Urinalysis Slightly cloudy, protein 1+, nitrite negative, leukocyte negative

Of the following, the diagnostic study MOST indicated in this patient is


A. abdominal ultrasonography
B. chest radiography
C. echocardiography
D. urine culture

American Academy of Pediatrics 413


PREP ® Self-Assessment PREPSA 2021
Correct Answer: C
The diagnostic study most indicated for the infant in the vignette is echocardiography. She has
congenital heart disease that has recently been surgically repaired and during the postoperative
course developed enterococcal bacteremia. Echocardiography is indicated to assess for
endocarditis.

Endocarditis should be considered in the setting of enterococcal bacteremia in children with


congenital heart disease or children with prolonged bacteremia after appropriate treatment and
source control (ie, vascular device removal). After staphylococci and streptococci, enterococci
are the third most common cause of endocarditis.

Enterococci are part of the normal flora of the human intestinal tract. They can cause a wide
array of human infections, including those of the urinary tract, abdominal cavity, and
bloodstream. Less commonly, enterococcal infections can lead to endocarditis and meningitis.
The most common infection caused by enterococci are urinary tract infections (UTIs). Risk
factors for enterococcal UTIs include instrumentation of the urinary tract, anatomic
abnormalities, and prior antimicrobial use. In children, enterococcal bacteremia is largely a
nosocomial infection seen in immunocompromised patients or critically ill children with
intravascular catheters. Risk factors for enterococcal meningitis include prematurity and
neurosurgical procedures.

Abdominal ultrasonography could be considered if there were concerns for an abdominal process
as the source of the bacteremia. However, the physical examination findings do not suggest an
abdominal process. Chest radiography could be considered if there were concerns for a
ventilator-associated pneumonia, but Enterococcus species are unlikely to cause pneumonia.
Urinary tract infection is unlikely in the absence of pyuria. Therefore, urine culture is not
needed.

PREP Pearls
• Endocarditis should be considered in the setting of enterococcal bacteremia in children
with congenital heart disease or children with prolonged bacteremia after appropriate
treatment and source control (ie, vascular device removal).
• Invasive enterococcal infections are largely nosocomial in origin.
• Enterococci can cause a wide array of human infections including those of the urinary
tract, abdominal cavity, and bloodstream and, less commonly, present as endocarditis and
meningitis.

MOCA-Peds Objective
• Recognize genetic causes of congenital heart disease.

ABP Content Specifications(s)


• Recognize the clinical syndromes associated with enterococcal infections

American Academy of Pediatrics 414


PREP ® Self-Assessment PREPSA 2021
Suggested Readings
• American Academy of Pediatrics. Non-group A or B streptococcal and enterococcal
infections. In: Kimberlin DW, Brady MT, Jackson MA, Long SS, eds. Red Book: 2018
Report of the Committee on Infectious Diseases. Itasca, IL: American Academy of
Pediatrics; 2018:768-772. Red Book Online.
• Butler K. Enterococcal infection in children. Semin Pediatr Infect Dis. 2006;17(3):128-
139. doi: 10.1053/j.spid.2006.06.006.

American Academy of Pediatrics 415


PREP ® Self-Assessment PREPSA 2021
Question 111
A 4-month-old female infant with a 5-day history of diarrhea is brought to the emergency
department. She has been refusing to drink formula for 2 days. She has no history of vomiting,
fever, or abdominal distension. She is sleepy but has a high-pitched cry on stimulation. She has a
heart rate of 160 beats/min, a respiratory rate of 22 breaths/min, and a blood pressure of 70/38
mm Hg. Her skin feels doughy. Her mucous membranes are dry. The rest of her physical
examination findings are unremarkable.

Laboratory results are as follows:


Laboratory Test Result
Sodium 162 mEq/L (162 mmol/L)
Potassium 3.1 mEq/L (3.1 mmol/L)
Chloride 115 mEq/L (115 mmol/L)
Bicarbonate 18 mEq/L (18 mmol/L)
Blood urea
28 mg/dL (10.0 mmol/L)
nitrogen
Creatinine 0.5 mg/dL (44 µmol/L)
Glucose 90 mg/dL (5.0 mmol/L)
340 mOsm/kg (340 mmol/kg) (reference range, 275-295 mOsm/kg [275-295
Osmolality
mmol/kg])

Of the following, the MOST likely complication of rapid correction of this infant’s condition is
A. central pontine myelinolysis
B. cerebral edema
C. dural sinus thrombosis
D. subdural hemorrhage

American Academy of Pediatrics 416


PREP ® Self-Assessment PREPSA 2021

Correct Answer: B
The infant in this vignette has hypernatremic dehydration caused by gastroenteritis. A rapid
correction of the high serum sodium can result in shifting of water from the extracellular fluid
(ECF) compartment to the intracellular fluid (ICF) compartment, leading to cerebral edema.
Hypernatremia is defined as a serum sodium of greater than 150 mEq/L. Hypernatremic
dehydration reflects water loss in excess of solute loss, resulting in a serum sodium of greater
than 150 mEq/L. Hypernatremic dehydration is commonly seen in infants with acute diarrhea
(eg, rotavirus), resulting from inadequate replacement of water losses because of poor feeding.
Infants with hypernatremia can be somnolescent, and they may have a high-pitched cry on
stimulation. Their skin characteristically has a doughy or velvety feel.

Hypernatremia (hyperosmolality) causes fluid to shift from the ICF to the ECF. The ICF
depletion in the brain cells results in their shrinkage. This can lead to rupture of the bridging
veins, in turn resulting in subarachnoid, subdural, and intraparenchymal hemorrhage. Dural sinus
thrombosis can also occur. As the ECF is relatively preserved in the initial stages of
hypernatremic dehydration, the child may falsely appear well, and thus the parent may delay
seeking medical attention.

Infants with hypernatremic dehydration need close monitoring and a slow correction of the
serum sodium and the free water deficit. Initially, the serum sodium may need to be monitored
every 4 to 6 hours. Infants with serum sodium greater than 160 mEq/L should undergo sodium
correction over a period of 48 to 72 hours. The target drop in the serum sodium is less than 0.5
mEq/L per hour or 10 to 12 mEq/L per day. Prolonged hypernatremia (hyperosmolality) leads to
accumulation of osmotically active substances (idiogenic osmoles) within the cells of the brain
that help counter the increased osmolality of plasma and protect the brain from fluid shifts. With
rapid correction of hypernatremia, idiogenic osmoles continue to occupy intracellular brain
compartments and create a fluid shift from the ECF to the ICF, leading to cerebral edema and
seizure.

Central pontine myelinolysis can result from a rapid correction of hyponatremia due to a shift of
water from the brain cells (ICF) to the ECF. Subdural hemorrhage and dural sinus thrombosis are
seen with development of hypernatremia, but not as a complication of its rapid correction.

PREP Pearls
• Hypernatremic dehydration, commonly seen in infants with acute diarrhea, reflects
excess water loss as compared with solute loss.
• Infants with hypernatremic dehydration may have a high-pitched cry on stimulation and a
doughy feel to the skin on examination.
• Hypernatremic dehydration should be corrected slowly to prevent cerebral edema.

ABP Content Specifications(s)


• Identify symptoms associated with hypernatremia, including those associated with rapid
rehydration

American Academy of Pediatrics 417


PREP ® Self-Assessment PREPSA 2021
Suggested Readings
• Jain A. Body fluid composition. Pediatr Rev. 2015;36(4):141-150. doi:10.1542/pir.36-4-
141.
• Mahajan P, Felt JR. Fluids, electrolytes, and acid-base composition. In: McInerny TK,
Adam HM, Campbell DE, DeWitt TG, Foy JM, Kamat DM, eds. American Academy of
Pediatrics Textbook of Pediatric Care. 2nd ed. Elk Grove Village, IL: American
Academy of Pediatrics; 2017:419-432. Pediatric Care Online .
• Powers KS. Dehydration: isonatremic, hyponatremic, and hypernatremic recognition and
management. Pediatr Rev. 2015;36(7):274-283. doi:10.1542/pir.36-7-274.

American Academy of Pediatrics 418


PREP ® Self-Assessment PREPSA 2021
Question 112
A 14-year-old basketball player is being evaluated for a recurrent left ankle injury. Several days
ago he sustained his third inversion injury to this ankle over the past several months. These
injuries result in mild swelling over the lateral ankle for several days, which he treats with a
compression wrap, ice, and several doses of ibuprofen. He was able to return to full basketball
participation within 7 to 10 days after each injury. However, with the current injury, the ankle
feels unstable and he tapes it before practice. The patient is otherwise healthy.

On physical examination, the adolescent’s left ankle is slightly swollen and minimally tender
inferior to the lateral malleolus. He has full and pain-free strength and range of motion in both
ankles. There is increased laxity with anterior drawer testing on the left compared with the right.
His gait is normal and he moves comfortably throughout the evaluation. The adolescent has
some difficulty with single leg hop and balance testing on the left leg compared with the right.

Of the following, the BEST next step in management for this patient is
A. anteroposterior and lateral ankle radiography
B. magnetic resonance imaging of the left ankle
C. referral to a physical therapist
D. use of an ankle-stabilizing brace

American Academy of Pediatrics 419


PREP ® Self-Assessment PREPSA 2021
Correct Answer: C
Ankle sprains are the most common sports-related injury in children and adolescents. Full
recovery after ankle sprains includes restoration of function and normalization of strength and
balance. If these are not achieved before return to full activity, the risk of recurrent sprain is
significantly increased. The single leg hop is a good method for assessing strength in the office
setting. If this is impaired in the injured leg compared with the uninjured leg, the child would
benefit from instruction on therapeutic exercise.

A sprain is an injury to a ligament. Severity of injury is classified into 3 grades:


• Grade 1: Stretching of the fibers of the ligament
• Grade 2: Partial tear of the ligament
• Grade 3: Full tear of the ligament

Most ankle sprains occur after an inversion injury in which the foot “rolls in” and the lateral
structures of the ankle joint are “stretched.” In the skeletally mature adolescent, the most
common injury after this mechanism is a sprain of the anterior talofibular ligament (Item
C112A). This typically presents with maximal pain and tenderness just anterior to the lateral
malleolus and swelling over the lateral aspect of the ankle. Ankle laxity on physical examination
suggests that this adolescent has sustained a grade 2 injury.

Item C112A: Anatomy of lateral ankle ligaments.

American Academy of Pediatrics 420


PREP ® Self-Assessment PREPSA 2021
Reproduced with permission from Griffin LY (ed): Essentials of Musculoskeletal Care, ed. 3.
Rosemont, IL. American Academy of Orthopaedic Surgeons. 2005.

The Ottawa ankle rules are applied to determine the appropriate use of radiography after acute
ankle injury. These rules state that radiography should be performed when there is pain in the
malleolar region of the ankle and either:
• tenderness along the posterior edge, or tip, of the distal 6 cm of the tibia or fibula, OR
• inability to bear weight immediately after injury and in the office.

The boy in the vignette does not exhibit any of the aforementioned findings, and the location of
his tenderness just distal to the fibula most strongly suggests a ligamentous injury. Therefore,
radiography is not indicated. Physical therapy will be most likely to speed his recovery.

Treatment of ankle sprains includes the following measures.


1. Elimination of swelling: Ongoing swelling impairs joint function. An elastic compression
wrap should be applied and families instructed on the use of ice until the swelling is
eliminated.
a. Ice application instructions: Apply to the injured area for up to 10 to 15 minutes
per session. This may be repeated up to every hour, but is most important after
activity.
2. Protection of injury site: The goal is to normalize gait as much as possible; this will vary
from patient to patient, but may include the use of crutches, a brace, or a walking boot.
a. Immobilization may prolong ultimate recovery time, and should be kept to a
minimum.
3. Restore range of motion, strength, and balance: Many children will recover more quickly
with physical therapy, which can be performed as part of a home program. The following
activities should be added to the regimen in order:
a. Begin range of motion exercises as soon as tolerated
i. Draw alphabet with the great toe
b. Start strengthening when range of motion is normalized
i. Peroneal strengthening (Item C112B)
ii. Calf strengthening (Item C112C)
iii. Balance training (Item C112D)
4. Graduated return to sport activity
a. May begin after range of motion, strength, and balance have been normalized, and
walking with a normal gait.
i. Single leg hop is good test for strength
ii. Single leg stance with eyes closed is a good test for balance
b. Progress through the following steps in sequence when able to perform each
activity pain-free:
i. Jogging/running in straight line
ii. Sprinting in straight line
iii. Lateral maneuvers
iv. Cutting/pivoting
v. Sport-specific skills

American Academy of Pediatrics 421


PREP ® Self-Assessment PREPSA 2021

Item C112B: Ankle strengthening with resistance bands.


Reprinted with permission from Anderson S. Acute foot and ankle injuries. In: Harris SS,
Anderson SJ, eds. Care of the Young Athlete. 2nd ed. Itasca, IL: American Academy of
Pediatrics; 2010;449

American Academy of Pediatrics 422


PREP ® Self-Assessment PREPSA 2021

Item C112C: Calf strengthening.


Reprinted with permission from Anderson S. Acute foot and ankle injuries. In: Harris SS,
Anderson SJ, eds. Care of the Young Athlete. 2nd ed. Itasca, IL: American Academy of
Pediatrics; 2010;450

American Academy of Pediatrics 423


PREP ® Self-Assessment PREPSA 2021

Item C112D: Balance training on balance board.


Reprinted with permission from Anderson S. Acute foot and ankle injuries. In: Harris SS,
Anderson SJ, eds. Care of the Young Athlete. 2nd ed. Itasca, IL: American Academy of
Pediatrics; 2010;450.

When sports activities are resumed before completion of a full rehabilitation program, risk of
recurrent injury is increased, particularly in ankles with lax ligaments. Ankle braces can help
prevent recurrent injury, but are not a substitute for adequate rehabilitation.

Magnetic resonance imaging is recommended if there is concern for a ligamentous injury that
may warrant surgery, to detect osteochondral lesions in cases of recurrent ankle pain and
swelling, or if pain or swelling does not improve with rehabilitation.

American Academy of Pediatrics 424


PREP ® Self-Assessment PREPSA 2021

PREP Pearls
• Full recovery of an ankle sprain includes restoration of normal range of motion, strength,
and balance.
• Risk of recurrent injury is markedly increased with premature return to activity after
ankle sprain.
• The single leg hop is a good method for assessing strength and the single leg stance with
eyes closed is a good test for balance in the office setting.

MOCA-Peds Objective
• Identify contraindications for sports participation.

ABP Content Specifications(s)


• Recognize the importance of adequate rehabilitation of current injury in the prevention of
future injury among athletes

Suggested Readings
• Anderson S. Acute foot and ankle injuries. In: Harris SS, Anderson SJ, eds. Care of the
Young Athlete. 2nd ed. Itasca, IL: American Academy of Pediatrics; 2010;443-456.
• Kannikeswaran N, Suresh S. Sports musculoskeletal injuries. In: McInerny TK, Adam
HM, Campbell DE, DeWitt TG, Foy JM, Kamat DM, eds. American Academy of
Pediatrics Textbook of Pediatric Care. 2nd ed. Itasca, IL: American Academy of
Pediatrics; 2016;chap 334:2675-2685. Pediatric Care Online.

American Academy of Pediatrics 425


PREP ® Self-Assessment PREPSA 2021
Question 113
A 5-year-old boy is seen in the office for concerns of pubic hair growth and penile enlargement.
During a recent bath, his mother noticed pubic hair and thought his penis looked too large. He
has no significant medical history. His newborn screen was normal, including a 17-
hydroxyprogesterone level. There is no family history of precocious puberty. His vital signs are
normal for age. His height and weight are at the 95th and 75th percentile, respectively. He has
mild acne over his forehead and a small amount of axillary hair. He has a pubertal-sized penis,
and his sexual maturity rating is 3 for pubic hair. His testes are 2 mL in volume bilaterally. The
remainder of his physical examination findings are unremarkable.

Laboratory data are shown:


Laboratory Test Resulta
Luteinizing hormone < 0.1 mIU/mL (reference range, 0.02-0.3 mIU/mL)
Follicle-stimulating hormone 0.6 mIU/mL (reference range, 0.26-3 mIU/mL)
210 ng/dL (7.3 nmol/L) (reference range,
Testosterone
< 2.5-10 ng/dL [0.1-0.4 nmol/L])
Dehydroepiandrosterone 1,600 μg/dL (43.2 µmol/L) (reference range, 13-83 μg/dL [0.4-
sulfate 2.2 µmol/L])
a
Reference ranges are for individuals younger than 10 years.

The result of bone age radiography is 8 years.

Of the following, the MOST likely source of this boy’s precocious puberty is
A. exogenous steroids
B. his adrenal gland
C. his central nervous system
D. his gonads

American Academy of Pediatrics 426


PREP ® Self-Assessment PREPSA 2021
Correct Answer: B
The boy described in the vignette has precocious puberty, exhibiting signs of puberty before age
9 years. These signs include penile enlargement, pubic hair growth, tall stature, and an advanced
bone age. His testicular volume of 2 mL is prepubertal (< 4 mL), indicating a peripheral,
noncentral source of his precocious puberty. His undetectable level of luteinizing hormone (LH)
level confirms a noncentral source.

The boy’s markedly elevated dehydroepiandrosterone sulfate (DHEA-S) level makes his adrenal
gland the most likely source of his precocious puberty. This level is consistent with an androgen-
secreting adrenal tumor. Dehydroepiandrosterone sulfate is made by the adrenal gland and is a
marker of adrenal androgen production. Adrenal androgens can be metabolized to testosterone,
the source of his elevated testosterone level.

Distinguishing central from peripheral sources of precocious puberty is the first step in
narrowing the differential diagnosis. In central precocious puberty, the hypothalamic-pituitary-
gonadal axis is activated. An LH level of 0.3 mIU/mL or higher is consistent with activation of
this axis. For boys, testicular volume of 4 mL or greater or length of 2.5 cm or greater signals the
onset of central puberty due to LH stimulation. Thus, testicular volume or size in boys is an
important physical examination finding in differentiating central from peripheral precocious
puberty. Although central precocious puberty is often idiopathic in girls, boys with central
precocious puberty are more likely to have central nervous system pathology.

Potential sources of peripheral precocious puberty include the adrenal gland, gonads, a human
chorionic gonadotropin (hCG)–secreting tumor (in males), exogenous exposure to sex steroid,
and severe hypothyroidism. Low LH and follicle-stimulating hormone (FSH) levels are
consistent with peripheral precocious puberty. Etiologies of precocious puberty from the adrenal
gland include congenital adrenal hyperplasia and androgen-secreting adrenal tumors. Gonadal
sources of precocious puberty include sex-steroid–secreting gonadal tumors, McCune-Albright
syndrome (constitutive activation of gonadotropin receptors), and in males, familial male-limited
precocious puberty (constitutive activation of the LH receptor). Human chorionic gonadotropin–
secreting tumors cause precocious puberty in males because hCG stimulates the LH receptor on
the testes. Severe hypothyroidism can cause precocious puberty owing to high levels of thyroid-
stimulating hormone cross-reacting with the gonadotropin receptors. Item C113 lists the
differential diagnosis of precocious puberty.

American Academy of Pediatrics 427


PREP ® Self-Assessment PREPSA 2021

American Academy of Pediatrics 428


PREP ® Self-Assessment PREPSA 2021
The traditional definition of precocious puberty is signs of puberty before the age of 8 years in
girls and 9 years in boys. Important historical information in the evaluation of precocious
puberty includes onset and progression of pubertal signs, potential exposure to exogenous sex
steroids, family history, and any symptoms relating to the central nervous system. In addition to
pubertal signs and staging on examination, neurological, abdominal (for palpable tumor), and
skin examinations are also important. Café-au-lait spots may be suggestive of McCune-Albright
syndrome or neurofibromatosis type 1, both of which can be associated with precocious puberty.

An initial evaluation for precocious puberty includes measurement of LH, FSH, thyroid-
stimulating hormone, estradiol in girls, testosterone in boys, and a bone age radiograph.
Measurement of LH levels after stimulation with gonadotropin-releasing hormone agonist may
be necessary to detect central precocious puberty. In boys, a quantitative hCG level is indicated
to evaluate for an hCG-secreting tumor. Pelvic ultrasonography in girls is helpful in determining
ovarian volume and if there is concern regarding an ovarian tumor. In girls with signs of excess
androgen exposure and in boys with prepubertal-sized testes, measurement of 17-
hydroxyprogesterone (for congenital adrenal hyperplasia due to 21-hydroxylase deficiency),
DHEA-S (marker of adrenal androgen production), and testosterone (in girls) is indicated.
Testicular ultrasonography is indicated if a testicular tumor is suspected. In cases of central
precocious puberty, brain magnetic resonance imaging should be considered.

For the boy in the vignette, his central nervous system is not the source of his precocious puberty
because of his prepubertal-sized testes (< 4 mL in volume) and undetectable LH level.
Exogenous steroids are not the source of his precocious puberty, because exogenous steroids
would not cause the markedly elevated DHEA-S level. Without the elevated DHEA-S level,
however, his low LH level, prepubertal-sized testes, and elevated testosterone level could be
consistent with exogenous testosterone exposure. Similarly, his gonads are not the source of his
precocious puberty, given their prepubertal size and his markedly elevated DHEA-S level.

PREP Pearls
• Testicular volume or size in boys is an important physical examination finding in
differentiating central from peripheral precocious puberty.
• Although central precocious puberty is often idiopathic in girls, boys with central
precocious puberty are significantly more likely to have central nervous system
pathology.
• Dehydroepiandrosterone sulfate (DHEA-S) is made by the adrenal gland and is a marker
of adrenal androgen production.

ABP Content Specifications(s)


• Plan an appropriate diagnostic evaluation to differentiate the various causes of precocious
puberty, including that associated with an adrenal etiology
• Recognize the clinical features associated with precocious puberty, including that caused
by tumors

American Academy of Pediatrics 429


PREP ® Self-Assessment PREPSA 2021
Suggested Readings
• Kritzler RK, Long D, Plotnick L. Precocious puberty. In: McInerny TK, Adam HM,
Campbell DE, DeWitt TG, Foy JM, Kamat DM, eds. American Academy of Pediatrics
Textbook of Pediatric Care. 2nd ed. Elk Grove Village, IL: American Academy of
Pediatrics; 2017:1540-1544. Pediatric Care Online.
• Long D. Precocious puberty. Pediatr Rev. 2015;36(7):319-321. doi:10.1542/pir.36-7-319.
• Muir A. Precocious puberty. Pediatr Rev. 2006;27(10):373-381. doi:10.1542/pir.27-10-
373.

American Academy of Pediatrics 430


PREP ® Self-Assessment PREPSA 2021
Question 114
A 25-hour-old neonate with abdominal distention is being evaluated. His 34-year-old mother has
gestational diabetes controlled by diet; her hemoglobin A1c level is 6.6%. Echogenic bowel was
noted on prenatal ultrasonography. He was delivered vaginally with an Apgar score of 9 at 1 and
5 minutes. He has breastfed 6 times with a good latch. He has not yet had a bowel movement. On
physical examination, the neonate appears comfortable. He has a prominent white forelock,
barrel-shaped chest, moderate abdominal distention, and decreased bowel sounds (Item Q114A).
Abdominal radiography is performed (Item Q114B).

American Academy of Pediatrics 431


PREP ® Self-Assessment PREPSA 2021

Item Q114B: Abdominal radiograph for the neonate described in the vignette.
Courtesy of M LaTuga

Of the following, the MOST likely diagnosis for this neonate is


A. Hirschsprung disease
B. inadequate oral intake
C. necrotizing enterocolitis
D. small left colon syndrome

American Academy of Pediatrics 432


PREP ® Self-Assessment PREPSA 2021
Correct Answer: A
The neonate’s clinical presentation of delayed passage of meconium and dilated bowel on
abdominal radiography, in combination with a white forelock suggests the diagnosis of
Hirschsprung disease. Waardenburg type 4, or Waardenburg-Shah syndrome, is known to be
associated with Hirschsprung disease.

Hirschsprung disease is caused by arrested migration of neuroblast cells to the rectum, resulting
in ineffective relaxation and increased muscle tone. In most cases, ganglion cells are absent only
in the rectosigmoid area. Long-segment aganglionosis occurs in approximately 10% of affected
children. Hirschsprung disease is diagnosed on suction rectal biopsy. A water-soluble contrast
enema may be helpful by demonstrating a transition zone between normal colon and the narrow
colon without ganglion cells. Patients with Hirschsprung disease are at risk for enterocolitis
(which can progress to toxic megacolon), in which bacterial translocation occurs in areas of
bowel dilation, causing a systemic inflammatory response.

Waardenburg syndrome (WS) is inherited in an autosomal dominant pattern. Neonates may have
a white forelock, wide nasal bridge, sensorineural deafness, heterochromia iridis, and lateral
displacement of the inner canthi of the eyes. Germline mutations in the PAX3 gene are
responsible for WS. There are 4 clinical subtypes; type 4 (Waardenburg-Shah syndrome) is
always associated with Hirschsprung disease (Item C114).

A neonate with necrotizing enterocolitis would have significant tenderness on abdominal


palpation, which was not noted in the neonate in the vignette. Although associated with neonates
born to mothers with diabetes, small left colon syndrome typically presents with signs and
American Academy of Pediatrics 433
PREP ® Self-Assessment PREPSA 2021
symptoms of small bowel obstruction. A neonate with inadequate intake would demonstrate
feeding difficulties, unlike the neonate in the vignette.

PREP Pearls
• Hirschsprung disease typically presents in the neonatal period with delayed passage of
meconium, abdominal distention, and feeding intolerance.
• Waardenburg syndrome is characterized by a prominent white forelock, wide nasal
bridge, sensorineural deafness, heterochromia iridis, and in some types, lateral
displacement of the inner canthi.
• Waardenburg syndrome type 4 (Waardenburg-Shah syndrome) is always associated with
Hirschsprung disease.

ABP Content Specifications(s)


• Recognize the clinical and laboratory features associated with intestinal obstruction in a
newborn infant, and manage appropriately
• Recognize the clinical and laboratory features associated with necrotizing enterocolitis in
a newborn infant

Suggested Readings
• Joyce JC. Hypopigmented lesions. In: Kliegman R, St Geme JW, Blum NJ, Shah SS,
Tasker RC, Wilson K, eds. Nelson Textbook of Pediatrics. Philadelphia, PA: Elsevier;
2020:3477-3480.e1.
• Levitt M, Garza JM, Pena A, Lawal T. Colorectal disorders. In: McInerny TK, Adam
HM, Campbell DE, DeWitt TG, Foy JM, Kamat DM, eds. American Academy of
Pediatrics Textbook of Pediatric Care. 2nd ed. Itasca, IL: American Academy of
Pediatrics; 2016;chap 232:1970-1880. Pediatric Care Online.
• Maqbool A, Fiorino KN, Liacouras CA. Motility disorders and Hirschsprung disease. In:
Kliegman R, St Geme JW, Blum NJ, Shah SS, Tasker RC, Wilson K, eds. Nelson
Textbook of Pediatrics. Philadelphia, PA: Elsevier; 2020:1955-1965.e1.

American Academy of Pediatrics 434


PREP ® Self-Assessment PREPSA 2021

Question 115
A 4-year-old boy is brought to the clinic for evaluation of a neck mass. Two weeks ago, he had a
low-grade fever, rhinorrhea, and cough that have resolved. Since then, his mother noticed a
painless lump on the center of his neck. His vital signs and growth parameters are within normal
limits. His tympanic membranes, nares, and oropharynx are normal. There is a 3-cm midline,
painless, fluctuant, mobile mass (Item Q115) on his anterior neck without overlying erythema or
discharge. The mass elevates with tongue protrusion and swallowing.

Item Q115: Midline anterior neck mass as described for the boy in this vignette.
Reprinted with permission from Doernbecher Children’s Hospital, Portland, Oregon

Of the following, the BEST next step in treatment is


A. empiric antibiotics
B. fine needle aspiration
C. thyroid-stimulating hormone analysis
D. ultrasonography

American Academy of Pediatrics 435


PREP ® Self-Assessment PREPSA 2021

Correct Answer: D
The boy in this vignette has a thyroglossal duct cyst (TGDC), the most common midline neck
mass. He should be evaluated via ultrasonography, which is the best imaging modality. Although
complete surgical excision is the treatment of choice for TGDCs, it is important to first obtain
ultrasonographic imaging to determine the presence of normal thyroid gland tissue, and the
cyst’s position and relationship to the hyoid bone. If thyroid ectopia is found on ultrasonography,
additional imaging may be required in addition to thyroid function analysis. Although fine needle
aspiration may exclude other diagnoses, ultrasonography would still be required before surgical
excision could proceed to determine anatomical relationships. Empiric antibiotics are
recommended for infected TGDCs, which are indicated clinically by an edematous, painful
midline mass with wall thickening and loss of hypoechoic appearance on ultrasonography.

The differential diagnosis of TGDC should include dermoid cysts, branchial cleft anomalies, and
lymphangiomas (cystic hygromas). Dermoid cysts are benign neoplasms that are commonly
found in the newborn period, typically more superficial and mobile than TGDCs, and slow-
growing; in addition, they may present with recurrent infections. Branchial cleft anomalies
include cysts, tracts, and fistulas that commonly present in late childhood with infection.
Thyroglossal duct cysts are not associated with sinus tracts or fistulas. Lymphangiomas are
mobile, spongy, painless lesions more frequently located in the posterior triangle of the neck and
are often initially present in the newborn period.

To ensure complete removal of TGDCs and avoid recurrence, surgery involves removing not
only the cyst, but also the middle portion of the hyoid bone and any tissue protruding upward to
the foramen cecum, known as the Sistrunk procedure. Although less than 1% of TGDCs are
associated with malignancy, the resected specimen should be sent for pathologic testing.

PREP Pearls
• The differential diagnosis of thyroglossal duct cyst should include dermoid cysts,
branchial cleft anomalies, and lymphangiomas (cystic hygromas).
• Complete surgical excision is the treatment of choice for thyroglossal duct cysts.
• Prior to surgical excision, ultrasonographic imaging is necessary to determine the
presence of normal thyroid gland tissue, the cyst’s position, and the cyst’s relationship to
the hyoid bone.

ABP Content Specifications(s)


• Recognize the clinical features associated with branchial cleft anomalies, and plan
appropriate evaluation and management
• Recognize the clinical features associated with a thyroglossal duct cyst
• Formulate a differential diagnosis of a neck mass

American Academy of Pediatrics 436


PREP ® Self-Assessment PREPSA 2021
Suggested Readings
• Geddes G, Butterly MM, Patel SM, Marra S. Pediatric neck masses. Pediatr Rev.
2013;34(3):115-124. doi:10.1542/pir.34-3-115.
• Herendeen NE, Szilagyi PG. Cystic and solid masses of the face and neck. In: McInerny
TK, Adam HM, Campbell DE, DeWitt TG, Foy JM, Kamat DM, eds. American
Academy of Pediatrics Textbook of Pediatric Care. 2nd ed. Elk Grove Village, IL:
American Academy of Pediatrics; 2017:1933-1935. Pediatric Care Online .
• Meier JD, Grimmer JF. Evaluation and management of neck masses in children. Am Fam
Physician. 2014;89(5):353-358. https://www.aafp.org/afp/2014/0301/p353.html.
• Quintanilla-Dieck L, Penn EB Jr. Congenital neck masses. Clin
Perinatol. 2018;45(4):769-785. doi:10.1016/j.clp.2018.07.012.

American Academy of Pediatrics 437


PREP ® Self-Assessment PREPSA 2021
Question 116
A 3-year-old boy is brought to the office by his mother for a health supervision visit. His mother
is planning to enroll him in preschool, but is uncertain which educational setting would be best.
The boy speaks in 3-word sentences/phrases and follows 3-step commands. He can balance on
each foot for 3 seconds. He can draw lines and stack up to 3 blocks. The boy knows his sex and
age. He can match letters. He enjoys playing with other children and has an easy temperament.
He is toilet trained. The boy has no significant medical history and his growth and physical
examination findings are within normal limits.

Of the following, the MOST appropriate recommendation is a


A. general education program
B. general education program with specialized academic instruction
C. general education program with physical therapy
D. special education program

American Academy of Pediatrics 438


PREP ® Self-Assessment PREPSA 2021
Correct Answer: A
The 3-year-old boy in the vignette has delayed fine motor skills but is age appropriate in all other
domains of development. He does not meet the eligibility criteria for a special education program
or services under any of the 13 disability categories (Item C116). Therefore, this boy would be
placed in a general education program. He does not need specialized academic instruction for a
learning disability or physical therapy for gross motor delay.

The Individuals with Disabilities Education Act (IDEA) is a federal law that provides for early
Intervention (EI) and special education services for children with learning or physical
disabilities. Early intervention programs serve children from birth to 3 years of age with
developmental delays (physical, cognitive, communication, social/emotional, adaptive) or a
condition with high probability of developmental delay. Early Intervention programs are family-
centered and provide parents with knowledge and resources (eg, developmental therapies, family
training) to improve their child’s development. In the United States, EI programs serve about 2%
of children from birth to age 3 years, whereas special education services serve about 13% of
people 3 to 21 years old.

Once a child is determined to be eligible under any of the 13 disability categories, an


Individualized Education Program is developed to provide services and accommodations to meet
the child’s special educational needs. Children who need intensive services may be placed in a
special classroom; however, services should be delivered in the least restrictive environment,
with the child supported in a “mainstream” standard educational setting with typical peers
whenever possible. Including children with disabilities in a regular classroom promotes empathy,
understanding, and appreciation of diversity in the classroom students. With educational
inclusion, the student with a disability may achieve greater social, communication, and academic
gains. Although the general education teacher may find it challenging to adequately meet the
educational needs of the included students, ultimately educational placement should be
determined based on the needs of the individual student.

PREP Pearls
• Early intervention programs serve children from birth to 3 years of age with
developmental delays (physical, cognitive, communication, social/emotional, adaptive) or
a condition with high probability of developmental delay.
• The 13 disability categories for special education services are autism, deaf-blindness,
deafness, emotional disturbance, hearing impairment, intellectual disability, multiple
disabilities, orthopedic impairment, other health impairment, specific learning disability,
speech or language impairment, traumatic brain injury, and visual impairment (including
blindness).
• Special education services should be delivered in the least restrictive environment, with
the child supported in a “mainstream” standard educational setting with typical peers
whenever possible.

American Academy of Pediatrics 439


PREP ® Self-Assessment PREPSA 2021
ABP Content Specifications(s)
• Understand the general goals of early intervention programs for children of various ages
who have learning disabilities
• Understand the educational criteria required for placement in special classrooms and the
factors affecting those decisions
• Understand the advantages and disadvantages of educational inclusion for patients of
various ages who have learning or physical disabilities

Suggested Readings
• Augustyn M, Wolf M. Learning disability. In: Augustyn M, Zuckerman B,
eds. Zuckerman Parker Handbook of Developmental and Behavioral Pediatrics for
Primary Care. 4th ed. Philadelphia, PA: Wolters Kluwer; 2019:317-321.
• Fogler JM, Barbaresi WJ. Learning disabilities. In: Voigt RG, Macias MM, Myers SM,
Tapia CD, eds. Developmental and Behavioral Pediatrics. 2nd ed. Itasca, IL: American
Academy of Pediatrics; 2018:367-381.
• Frankowski BL. Learning difficulty. In: McInerny TK, Adam HM, Campbell DE, DeWitt
TG, Foy JM, Kamat DM, eds. American Academy of Pediatrics Textbook of Pediatric
Care. 2nd ed. Itasca, IL: American Academy of Pediatrics; 2016;chap 172:1484-
1489. Pediatric Care Online.
• Rose L, Herzig LD, Hussey-Gardner B. Early intervention and the role of
pediatricians. Pediatr Rev. 2014;35(1):e1-e10. doi: 10.1542/pir.35-1-e1.

American Academy of Pediatrics 440


PREP ® Self-Assessment PREPSA 2021
Question 117
A 14-year-old adolescent boy is being evaluated for persistent symptoms after he sustained a
concussion while playing football. He was tackled during a game, hitting his head against the
ground with loss of consciousness for an estimated 5 seconds. He immediately complained of
headache and vertigo and was removed from play. Head computed tomography performed at that
time was normal.

Over the next 3 months, he complained of persistent dull headaches, intermittent vertigo,
insomnia, and moodiness. He had trouble focusing and his grades were falling, which he
attributed to feeling constantly in a fog.

Before his concussion, the boy had episodic migraine headaches, which occurred infrequently
and responded to ibuprofen. There is a family history of depression, and the patient had seen a
counselor a few years ago for anxiety symptoms.

At this visit, he is wearing sunglasses. He is alert and answering questions appropriately but
appears disengaged. Notable neurologic findings include difficulty with recall of 3 objects after 5
minutes, performance of serial 7s, and tandem gait testing.

Of the following, the element of his history that PREDISPOSES him to this presentation is
A. family history of depression
B. first-time concussion
C. history of anxiety
D. immediate postinjury symptoms

American Academy of Pediatrics 441


PREP ® Self-Assessment PREPSA 2021
Correct Answer: C
The adolescent in the vignette has postconcussive syndrome, a neurocognitive disorder affecting
approximately 10% to 15% of those with concussion. Symptom onset is within 4 weeks after a
concussion and lasts a minimum of 3 months. Risk factors for the development of
postconcussive syndrome include:
• pre-existing psychiatric disorders such as anxiety, depression, or poor coping skills
• multiple prior concussions
• history of migraine headaches
• learning disorder
• factors related to the concussion itself (severe initial symptom burden or delayed
symptom onset, loss of consciousness for >1 minute, vestibular symptoms)

This adolescent’s personal history of anxiety is a risk factor for later development of
postconcussive syndrome. A family history of depression is not considered a risk factor. First-
time concussion and a relatively mild to moderate initial symptom burden would decrease his
risk for postconcussive syndrome.

Postconcussive syndrome is a heterogeneous disorder much debated in the literature with varied
definitions and criteria. Common categories of symptoms include:
• Physical:
o Posttraumatic headache is the most common symptom; the headache quality
varies (eg, tension type or migraine). Treatment is tailored to headache type, and
typically includes multiple modalities including abortive medication with
nonsteroidal anti-inflammatory drugs, preventive medications, biofeedback, or
cognitive behavioral therapy.
o Vestibular symptoms may include dizziness, vertigo, poor coordination, or
balance. Physical therapy can help address these symptoms.
• Psychological:
o Postconcussion irritability, emotional lability, anxiety, or depression are common
and can persist, in particular in patients with preexisting psychiatric
comorbidities.
• Cognitive:
o Affected children may report feeling foggy, difficulty concentrating, or memory
issues.
o Symptoms can be delineated on neuropsychology assessments.
o School accommodations (eg, longer time to complete work, work without screens,
extra time for homework or tests, opportunity for rest, or shorter school day) and
working academically just below symptom threshold can help children continue
their school activities (return to learn).
• Sleep disturbance:
o Insomnia or hypersomnia.
o Sleep hygiene and a return to a regular daily schedule can minimize the
contribution of sleep disturbances to symptoms.

American Academy of Pediatrics 442


PREP ® Self-Assessment PREPSA 2021
The previous recommendations, of complete brain rest (physical and cognitive) until symptoms
resolve, were demonstrated to increase the risk of depressive symptoms arising from social
isolation and prolonged inactivity. The currently recommended practice of subthreshold light
physical activity with graduated intensity allows for safe reintroduction of physical activity
without exacerbation of postconcussive symptoms (return to play). Return to contact activity
requires complete resolution of symptoms both at rest and with physical exertion and return to
baseline academic performance.

Concussion and postconcussive management require a multidisciplinary care team and patient
and parent education. The Centers for Disease Control and Prevention have published updated
guidelines on the diagnosis and management of concussion in conjunction with a physician and
patient portal website with resources and evidence-based information, which can be accessed
at https://www.cdc.gov/headsup/index.html.

PREP Pearls
• Postconcussive syndrome is a heterogeneous disorder presenting with persistent physical,
psychosocial/emotional, cognitive, and/or sleep symptoms in children after a concussion.
• Risk factors for postconcussive syndrome include severe initial symptom burden or
delayed onset; loss of consciousness for more than 1 minute at the time of injury;
vestibular symptoms; and preinjury history of psychiatric disorders, migraine headache,
learning disorder, or repeated concussions.
• For postconcussive syndrome, a supportive, multidisciplinary approach to specific
symptom management is recommended. Physical and cognitive activity with
accommodations as needed performed at a subsymptom threshold, is the current
recommendation.

MOCA-Peds Objective
• Identify contraindications for sports participation.

ABP Content Specifications(s)


• Understand the long-term neurologic and behavior consequences of head trauma

Suggested Readings
• Centers for Disease Control and Prevention. Heads
up. https://www.cdc.gov/headsup/index.html.
• Gelineau-Morel, Zinkus T, Le Pichon J. Pediatric head trauma: a review and
update. Pediatr Rev. 2019;40(9):468-481. doi: 10.1542/pir.2018-0257.
• Halstead M, Walter K, Moffatt K; Council on Sports Medicine and Fitness. Sports-
related concussion in children and adolescents. Pediatrics. 2018;142(6):e20183074;
doi: 10.1542/peds.2018-3074.
• Lumba-Brown A., Yeates K, Sarmiento K, et al. Centers for Disease Control and
Prevention guideline on the diagnosis and management of mild traumatic brain injury
among children. JAMA Pediatr. 2018;172(11):e182853.
doi: 10.1001/jamapediatrics.2018.2853.

American Academy of Pediatrics 443


PREP ® Self-Assessment PREPSA 2021
Question 118
A 4-year-old boy is seen in the emergency department 30 min after his mother saw him swallow
something shiny. Since then, the boy has been complaining of abdominal pain but is not
drooling. His heart rate is 110 beats/min, his respiratory rate is 20 breaths/min, and his oxygen
saturation as measured via pulse oximetry is 98% in room air. He is alert and interactive. His
breath sounds are normal, with no wheezing or stridor. A chest radiograph is obtained (Item
Q118).

Item Q118: Radiograph for the boy described in the vignette.Courtesy of J. Reed

American Academy of Pediatrics 444


PREP ® Self-Assessment PREPSA 2021
Of the following, the BEST next step in caring for this boy is
A. endoscopic removal of object
B. reassurance to his mother
C. repeat radiography in 6 hours
D. upper gastrointestinal series

American Academy of Pediatrics 445


PREP ® Self-Assessment PREPSA 2021

Correct Answer: A
The boy in the vignette has swallowed a button battery that is lodged in the esophagus and
should be removed promptly, ideally within 2 hours of ingestion. Because of the risk of severe
injury caused by the battery, follow-up imaging and reassurance are not appropriate next steps
for the boy in the vignette.

Children often swallow coins and batteries, which can become lodged in the esophagus.
Symptoms of an object lodged in the esophagus include throat pain, abdominal pain, excessive
drooling, and nausea. Coins and button batteries appear similar on radiography, but a button
battery will show an outer rim often referred to as a "halo effect." When a button battery
becomes lodged in the esophagus, the battery’s electrical circuit is completed; this leads to
erosive electrical burns unless the battery is removed quickly via endoscopy. Endoscopy allows
the clinician to visualize the extent and location of the injury. Other evidence-based interventions
that can help prevent injury include giving the child honey (if older than 12 months of age) or
sucralfate suspension shortly after ingestion. After battery removal, if there is no perforation,
rinsing with a gentle acid such as sterile acetic acid can help neutralize residual alkali from the
battery. Careful observation in the hospital is important if esophageal injury is observed during
endoscopy because complications—including fistulas, erosion into blood vessels, or esophageal
perforation—can occur 48 hours or longer after the removal of the battery.

If the button battery has already passed into the stomach at the time of presentation, management
depends on whether the child is symptomatic. In a symptomatic child, even with mild symptoms,
the battery should be removed. Asymptomatic children without any underlying conditions, who
swallowed a battery smaller than 12 mm, and who have no additional foreign bodies such as a
second battery or magnet, can be closely observed at home by a reliable caregiver. Imaging
should be repeated in 10 to 14 days to confirm that the battery has passed in the stool.

Coins are much more commonly ingested. The most concerning complication of coin ingestion is
aspiration into the trachea. The location of the coin in the trachea versus the esophagus can be
determined via radiography. Typically, a coin in the trachea is positioned perpendicularly on an
anteroposterior radiograph, whereas the more common “face-on” view is seen when the coin is
in the esophagus. In any child with a coin suspected to be in the trachea or who is symptomatic,
the coin should be removed under direct visualization. Most coins lodged in the distal esophagus
will pass into the stomach without intervention, whereas only about 15% of coins in the proximal
esophagus will pass to the stomach within 24 hours. Children who are asymptomatic can be
observed and undergo repeat imaging after 12 to 24 hours. Once the coin is in the stomach or
more distal, it is likely to pass without complications. Additional radiography can be performed
if the coin is not observed in the stool by 4 weeks.

American Academy of Pediatrics 446


PREP ® Self-Assessment PREPSA 2021
PREP Pearls
• A button battery lodged in the esophagus should be removed promptly, ideally within 2
hours of ingestion.
• Most coins lodged in the distal esophagus will pass into the stomach without
intervention; only about 15% of coins in the proximal esophagus will pass to the stomach
within 24 hours.
• On an anteroposterior radiograph, a coin in the trachea will typically be positioned
perpendicularly, whereas a coin in the esophagus will appear “face on.” A button battery
in the esophagus will have the radiographic appearance of a halo.

MOCA-Peds Objective
• Manage foreign body ingestion.

ABP Content Specifications(s)


• Plan the management of a patient who has ingested a coin
• Plan the management of a patient who has ingested a button battery

Suggested Readings
• Clute J, Frey T, Reed J. Visual diagnosis: button versus battery: foreign body ingestions
in pediatrics. Pediatr Rev. 2019;40(11):e39-e41; doi:10.1542/pir.2017-0263.
• Green S. Ingested and aspirated foreign bodies. Pediatr Rev. 2015;36(10):430-437.
doi:10.1542/pir.36-10-430.
• National Capital Poison Center. National Capital Poison Center button battery ingestion
triage and treatment guideline. https://www.poison.org/battery/guideline.
• Skae CC, Parikh SR. Foreign bodies of the ear, nose, airway, and esophagus. In:
McInerny TK, Adam HM, Campbell DE, DeWitt TG, Foy JM, Kamat DM,
eds. American Academy of Pediatrics Textbook of Pediatric Care. 2nd ed. Elk Grove
Village, IL: American Academy of Pediatrics; 2016:2021-2026. Pediatric Care Online.

American Academy of Pediatrics 447


PREP ® Self-Assessment PREPSA 2021
Question 119
Four patients in the emergency department are being evaluated for bruising or bleeding.
In room 1, a 3-year-old girl has a 4-cm, raised, purple hematoma in the middle of her forehead.
She also has purple discoloration under her eyes and bruises on her elbows and shins. Her
parents report that yesterday she was riding a wheeled toy in the house and fell down a flight of 6
stairs. She landed on ceramic tile on all 4 extremities and hit her head. Her laboratory data are
shown:
Laboratory Test Result
White blood cell count 8,400/µL (8.4 × 109/L)
Hemoglobin 12.1 g/dL (121 g/L)
Platelet count 352 × 103/µL (325 × 109/L)
Prothrombin time 12.9 s
Partial thromboplastin time 33 s

In room 2, a 5-year-old boy has epistaxis that has lasted for 30 minutes despite continuous
pressure applied by his mother. She reports that he experiences epistaxis approximately every
other day. She also reports that his gums often bleed when he brushes his teeth and that he
always seems to be bruised. His laboratory data are shown:
Laboratory Test Result
White blood cell count 9,540/µL (9.5 × 109/L)
Hemoglobin 12.6 g/dL (126 g/L)
Platelet count 248 × 103/µL (248 × 109/L)
Prothrombin time 12.7 s
Partial thromboplastin time 30.1 s

In room 3, a 12-year-old boy has had bruising on his shins and upper arms for the last couple of
weeks. He has a seizure disorder that has been well controlled on levetiracetam for the last year.
He states that he has been feeling well and has not had any seizures or trauma. His laboratory
data are shown:
Laboratory Test Result
White blood cell count 5,420/µL (5.4 × 109/L)
Hemoglobin 11.3 g/dL (113 g/L)
Platelet count 21 × 103/µL (21 × 109/L)
Prothrombin time 11.5 s
Partial thromboplastin time 29.2 s

In room 4, a 4-year-old girl has a “rash” and bruises on her extremities that started 2 days ago.
She had been previously healthy with normal growth and development. She has petechiae on her
face, upper chest, arms, and legs, as well as ecchymoses of various sizes on her arms and legs.
Her laboratory data are shown:

American Academy of Pediatrics 448


PREP ® Self-Assessment PREPSA 2021
Laboratory Test Result
White blood cell count 7,520/µL (7.5 × 109/L)
Hemoglobin 12.1 g/dL (121 g/L)
Platelet count 2.0 × 103/µL (2.0 × 109/L)
Prothrombin time 13 s
Partial thromboplastin time 31.2 s

Of these 4 patients, the child MOST likely to have a heritable bleeding diathesis is in
A. room 1
B. room 2
C. room 3
D. room 4

American Academy of Pediatrics 449


PREP ® Self-Assessment PREPSA 2021
Correct Answer: B
The ability to achieve hemostasis is dependent on having a normal number of functional
platelets, as well as the ability to form fibrin. Patients with coagulation disorders have defects in
one of the following:
• Platelet number
• Platelet function
• The ability to form fibrin
• von Willebrand disease (an abnormal fibrin/platelet interaction)

Examples of the laboratory findings associated with various coagulation disorders are shown
in Item C119. Patients with heritable coagulation disorders will typically have a history of
frequent bleeding. The child in room 2 in the vignette has had frequent epistaxis, gum bleeding
with tooth brushing, and atypical bruising. Of the 4 children presented in the vignette, this
history is the most consistent with a heritable coagulation disorder. Values for his prothrombin
time (PT), partial thromboplastin time (PTT), and platelet number are all normal, suggesting that
he could have either a platelet function deficit, dysfibrinogenemia, or type I von Willebrand
disease.

American Academy of Pediatrics 450


PREP ® Self-Assessment PREPSA 2021

American Academy of Pediatrics 451


PREP ® Self-Assessment PREPSA 2021
The child in room 1 in the vignette has been previously healthy and has a history of trauma that
would explain her bruising, as well as normal values for PT, PTT, and platelet number. In the
absence of any other bleeding history, this child has no evidence of a coagulation disorder. The
child in room 3 has abnormal bruising, is receiving anticonvulsant therapy, and exhibits isolated
thrombocytopenia. The most likely diagnosis is a drug-induced thrombocytopenia with resultant
ecchymoses. The child in room 4 has been previously well and presents with the sudden onset of
petechiae, ecchymoses, and very severe thrombocytopenia with normal values for PT, PTT,
hemoglobin, and white blood cell count. This presentation is most consistent with immune
thrombocytopenia purpura.

PREP Pearls
• Heritable coagulation disorders typically present with a personal or family history of
abnormal bleeding.
• Normal values for prothrombin time, partial thromboplastin time, and platelet number do
not rule out a heritable coagulation disorder (eg, platelet function disorder or
dysfibrinogenemia).

MOCA-Peds Objective
• Evaluate and manage a patient with the new onset of thrombocytopenia.

ABP Content Specifications(s)


• Recognize clinical findings associated with qualitative platelet disorders
• Understand the role of medications in the development of thrombocytopenia
• Recognize the significance of thrombocytopenia in neonates and older children and
manage appropriately
• Distinguish clinical findings associated with thrombocytopenia from those caused by
normal bruising
• Identify the etiology of bruising in a child with a normal or slightly increased platelet
count

Suggested Readings
• Callaghan MU, Rajpurkar M. Coagulation disorders. In: McInerny TK, Adam HM,
Campbell DE, DeWitt TG, Foy JM, Kamat DM, eds. American Academy of Pediatrics
Textbook of Pediatric Care. 2nd ed. Elk Grove Village, IL: American Academy of
Pediatrics; 2017:1858-1867. Pediatric Care Online .
• Cooper S, Takemoto C. Von Willebrand disease. Pediatr Rev. 2014;35(3):136-137.
doi:10.1542/pir.35-3-136.
• Rodriguez V, Warad D. Pediatric coagulation disorders. Pediatr Rev. 2016;37(7):279-
291. doi:10.1542/pir.2015-0062.
• Zimmerman B, Valentino LA. Hemophilia: in review. Pediatr Rev. 2013;34(7):289-294.
doi:10.1542/pir.34-7-289.

American Academy of Pediatrics 452


PREP ® Self-Assessment PREPSA 2021
Question 120
A 17-year-old patient is seen for a health supervision visit. The patient seems withdrawn during
the visit and does not contribute much to the conversation. During the confidential psychosocial
interview, the patient discloses that 6 weeks ago she went to a party during a freshman
orientation program and was given some punch that she thinks was altered. She knows that she
was sexually assaulted but cannot recall what happened or who harmed her. She has not
disclosed this information to anyone and fears having to go back to campus. She declines to
contact the school and let them know what has occurred. She reports no abdominal pain, vaginal
discharge, or dysuria. Her last menstrual period was 1 week ago.

The pediatrician explains that the patient will be screened for sexually transmitted infections and
that as a mandated reporter, the physician is required to file a report. The pediatrician also shares
concerns about the patient’s mental health after this traumatic event.

Of the following, the BEST next step in management is to


A. encourage the patient to discuss the incident with her mother and recommend therapy to
address the traumatic event
B. encourage the patient to report the incident to officials at the college and recommend that
she not attend the college where the traumatic event occurred
C. provide the patient with resources if she decides she wants to go to therapy before she
starts college in the fall
D. recommend that the patient undergo a sexual assault medical examination to collect
evidence in case she decides she wants to report the incident to law enforcement in the
future

American Academy of Pediatrics 453


PREP ® Self-Assessment PREPSA 2021
Correct Answer: A
The girl in the vignette was sexually assaulted 6 weeks before disclosing the event to her
pediatrician; she is showing signs of being withdrawn and now fears going to college.
Encouraging her to disclose the event to her mother is the first step in ensuring she has a support
system and an advocate to assist with scheduling her for therapy, as well as to make sure she is
compliant with both medical and mental health follow up. A discussion about reporting the event
to the college may be warranted if the adolescent consents. However, encouraging her to not
attend the college where the assault occurred does not address the trauma, which can have
adverse effects on mental, physical, emotional, and social functioning. Only providing resources
for therapeutic services does not ensure the adolescent will follow through. Too much time has
passed to perform a sexual forensic examination.

Sexual assault is a public health problem and a violent crime. It is any nonconsensual sexual act
and includes sexual contact with or without penetration, as well as touching of breasts or the
anogenital area. In the United States, adolescents and young adults between 12 and 34 years of
age have the highest rates of being sexually assaulted. Compared with adult women, adolescent
girls are less likely to seek medical services or press charges against perpetrators after they have
been assaulted. Adolescents are also more likely to have used alcohol or drugs before being
assaulted. Sexual assault can lead to medical consequences of unintended pregnancy, sexually
transmitted infections, and mental health disorders such as depression, posttraumatic stress
disorder, and suicidal ideation and attempts.

The 2019 Association of American Universities Climate Control Survey, which surveyed more
than 181,000 students from 33 colleges and universities, revealed that 23% of female
undergraduate respondents reported having been sexually assaulted. Most sexually violent
crimes on college and university campuses have the highest rates among students who are
female, transgender, nonbinary, and genderqueer. The majority of the time, the perpetrator is
someone the victim knows.

During routine health supervision visits, it is important for primary care providers to inquire if
their adolescent patients have been victims of sexual assault. It should be a standard part of the
confidential history that is conducted with the parent out of the room. If an adolescent does
disclose a sexual assault, it is important to be supportive and nonjudgmental. Medical providers
must be aware of reporting laws for sexual assault for their state(s) of practice, and the
adolescent should be informed if a report will be filed. The pediatrician should obtain a detailed
history and, if the event occurred less than 72 hours before the time of disclosure, should
encourage the adolescent to consent to undergo a medical and sexual forensic examination. A
forensic examination is usually performed in an emergency department or by a sexual assault
team to try to collect DNA samples from the perpetrator. If the adolescent declines a forensic
examination, a medical examination would still be warranted and should include a pregnancy test
in women, screening for sexually transmitted infections, drug screening, and evaluation for
injuries. Female patients should be given emergency contraception within 120 hours of an assault
and human immunodeficiency virus prophylaxis within 72 hours of an assault. The Centers for
Disease Control and Prevention also recommends empirical treatment for Chlamydia infection,
gonorrhea, and trichomoniasis. If the assault is reported more than 72 hours but less than 7 days
after the event, collection of samples is dependent on protocols followed by local authorities. It is
American Academy of Pediatrics 454
PREP ® Self-Assessment PREPSA 2021
difficult to collect evidence a week after the event. The pediatrician should inquire whether the
adolescent has safety concerns about the perpetrator—in particular, whether the adolescent has
been threatened or is being trafficked.

From a mental health perspective, after an assault it is important to assess whether the adolescent
is having any suicidal thoughts. Victims of assault often blame themselves and have low self-
esteem. It is important to encourage adolescent victims to share what happened with a trusted
parent, caregiver, or teacher so that they are supported, but also so that they have someone to
monitor how they are coping, ensure compliance with follow up for repeat screening for sexually
transmitted infections, and watch for mental health sequelae. It is also imperative to provide the
adolescent with resources for counseling services.

PREP Pearls
• Pediatricians should routinely ask about sexual assault and dating violence during the
confidential interview with adolescent patients.
• Pediatricians should become familiar with the reporting laws regarding sexual assault in
adolescents, which vary by state.
• Encouraging adolescents to disclose sexual assault events to their parents is the first step
in ensuring they have a support system and an advocate to assist with scheduling them for
therapy, as well as to make sure they are compliant with both medical and mental health
follow up.

MOCA-Peds Objective
• Respond appropriately to a child’s disclosure of sexual abuse.

ABP Content Specifications(s)


• Recognize the medical and emotional needs of an adolescent victim of sexual assault, and
manage appropriately

Suggested Readings
• ACOG committee opinion no. 777: sexual assault. Obstet Gynecol. 2019;133(4):e296-
e302. doi:10.1097/AOG.0000000000003178.
• Crawford-Jakubiak JE, Alderman EM, Leventhal JM; Committee on Child Abuse and
Neglect; Committee on Adolescence. Care of the adolescent after an acute sexual assault
(published correction appears in Pediatrics. 2017;139[3]:e20164243). Pediatrics.
2017;139(3):e1-e16. doi:10.1542/peds.2016-4243.

American Academy of Pediatrics 455


PREP ® Self-Assessment PREPSA 2021
Question 121
A pediatrician notes that the number of patients with asthma seen in the clinic appears to be
increasing. The pediatrician wants to evaluate the relationship between asthma symptoms and
medication use in children with asthma.

Of the following, the question that BEST frames an evidence-based–medicine approach to this
clinical situation is
A. Among children aged 12 to 36 months with asthma, does the use of a controller
medication decrease asthma symptoms when compared with no controller use?
B. Among children aged 12 to 36 months with asthma, does the use of inhaled
corticosteroids decrease growth velocity?
C. Among children aged 12 to 36 months with asthma, does the use of inhaled
corticosteroids plus bronchodilators compared with the use of bronchodilators alone
reduce the frequency of emergency department visits?
D. Among children aged 12 to 36 months with moderate persistent asthma, does
montelukast result in fewer asthma exacerbations?

American Academy of Pediatrics 456


PREP ® Self-Assessment PREPSA 2021
Correct Answer: C
The term “evidence-based medicine” (EBM) is used to describe a formal and deliberate approach
to a clinical question that includes the use of high-quality data to inform clinical decisions for
individual patients. The practice of EBM integrates clinical expertise with the best available
evidence from a systematic review of the medical literature. Physicians must be skilled in the
interpretation of research studies to assess which data are applicable, accurate, and of high
quality. Implicit in the use of EBM is the need to formulate an appropriate clinical question. A
question that is formatted incorrectly will ultimately yield incomplete or inaccurate answers, or
even no meaningful answer.

PICO is an abbreviation that incorporates the necessary elements of a well-formulated question


that can be used for evidence-based clinical decision making. Each component of the PICO
question is necessary to ensure that there is enough specificity to yield data from a literature
search and study appraisal that will inform the clinical care of the patient. The four elements of a
PICO question are patient problem or population, intervention, comparison group, and outcome
measure. Of the choices listed, only response C incorporates all of the elements of a PICO
question:

Among children aged 12 to 36 months with asthma (P), does the use of inhaled corticosteroids
plus bronchodilators (I) compared with the use of bronchodilators alone (C) reduce the
frequency of emergency department visits (O)?

Response A lacks specificity of an outcome measure; “asthma symptoms” is too generic a term
to yield meaningful literature search results. Responses B and D have no comparison group.

PREP Pearls
• The practice of evidence-based medicine involves the use of high-quality clinical data to
inform decisions about the care of individual patients.
• A formal, structured clinical question that contains PICO elements—a patient population,
an intervention, a comparison group, and an outcome—is required when practicing
evidence-based medicine.

MOCA-Peds Objective
• Understand the concept of evidence-based practice.

ABP Content Specifications(s)


• Given the need for specific clinical information, identify a clear, structured, searchable
clinical question

Suggested Readings
• Moyer V, Neuspiel DR. PICO questions: what are they and why bother? AAP Grand
Rounds. 2014;31(5):50. doi:10.1542/gr.31-5-50.
• Onady GM, Raslich MA. Evidence-based medicine: asking the answerable question
(question templates as tools). Pediatr Rev. 2003;24(8):265-268. doi:10.1542/pir.24-8-
265.

American Academy of Pediatrics 457


PREP ® Self-Assessment PREPSA 2021
Question 122
A 16-year-old adolescent is brought to the emergency department by ambulance after he was
found wandering outside at 2 am. He is accompanied by a friend who reports that the patient
ingested an over-the-counter cough and cold medication that night in order to get high. The
patient is uncooperative and agitated. He is not oriented to place or situation. He has a heart rate
of 140 beats/min, blood pressure of 125/90 mm Hg, respiratory rate of 12 breaths/min, and
oxygen saturation of 100% in room air. He is diaphoretic, and his pupils are dilated.

Of the following, the ingredient that is MOST likely responsible for the adolescent’s signs and
symptoms is
A. acetaminophen
B. chlorpheniramine
C. dextromethorphan
D. guaifenesin

American Academy of Pediatrics 458


PREP ® Self-Assessment PREPSA 2021
Correct Answer: C
The boy in the vignette has symptoms consistent with dextromethorphan ingestion.
Dextromethorphan is an over-the-counter cough suppressant that is commonly abused. It is the
D-isomer of a codeine analog, levorphanol, and also has N-methyl-D-aspartate-receptor
antagonist properties. When taken in excess, it can cause euphoria, hallucinations, a feeling of
dissociation, tachycardia, mydriasis, and diaphoresis. Treatment of dextromethorphan overdose
is supportive. Because many cough and cold preparations that contain dextromethorphan also
contain other ingredients with potential toxicities (eg, acetaminophen, antihistamines,
pseudoephedrine), it is important to attempt to determine the exact medication ingested and to
monitor for additional symptoms.

Early symptoms of an acetaminophen overdose are mild and nonspecific, including nausea,
vomiting, diaphoresis, and lethargy. Liver function abnormalities peak between 72 and 96 hours
after ingestion and may be accompanied by hepatic encephalopathy. Chlorpheniramine is an
antihistamine with H1-receptor antagonist properties. Overdoses cause features of
anticholinergic toxicity, including agitation, tachycardia, hypertension, elevated temperature, dry
and flushed skin, and decreased pupillary responsiveness. Guaifenesin is an expectorant.
Overdose may cause nausea or vomiting.

PREP Pearls
• Dextromethorphan is an over-the-counter cough suppressant that, when taken in excess,
can cause euphoria, hallucinations, and a feeling of dissociation as well as tachycardia,
mydriasis, and diaphoresis.
• Chlorpheniramine is an over-the-counter antihistamine. Overdoses cause features of
anticholinergic toxicity, including agitation, tachycardia, hypertension, elevated
temperature, dry and flushed skin, and decreased pupillary responsiveness.

ABP Content Specifications(s)


• Recognize the risk of abuse of over-the-counter cough and cold preparations

Suggested Readings
• Boyer EW. Dextromethorphan abuse. Pediatr Emerg Care. 2004;20(12):858-863.
doi: 10.1097/01.pec.0000148039.14588.d0.
• Fine JS. Poisoning. In: McInerny TK, Adam HM, Campbell DE, DeWitt TG, Foy JM,
Kamat DM. Textbook of Pediatric Care. 2nd ed. Elk Grove Village, IL: American
Academy of Pediatrics; 2018:2924-2949. Pediatric Care Online.
• Wang GS, Hoyte C. Common substances of abuse. Pediatr Rev. 2018;39(8):403-414.
doi: 10.1542/pir.2017-0267.

American Academy of Pediatrics 459


PREP ® Self-Assessment PREPSA 2021
Question 123
A neonate is born at term via precipitous vaginal delivery to a 22-year-old woman who recently
emigrated from El Salvador and did not receive any prenatal care. A rapid human
immunodeficiency virus test and rapid plasma reagin test performed on the mother soon after
delivery have negative results. The neonate weighs 2.2 kg and has microcephaly. He is alert and
has a good cry, normal tone, and normal activity. Red reflex is decreased in the right eye and
absent in the left eye. He has mild hepatosplenomegaly, petechiae, and a grade 2/6 systolic
ejection murmur on auscultation of the left upper chest. The remainder of his physical
examination findings are unremarkable. Laboratory data are notable for thrombocytopenia. A
computed tomographic scan of the head reveals calcifications. Audiologic evaluation reveals
absent auditory brainstem responses in both ears. Skeletal survey shows the presence of
transverse lucent bands in both proximal humeral metaphyses. Evaluation by an ophthalmologist
reveals bilateral cataracts.

Of the following, the MOST likely etiology for the findings in this neonate is
A. cytomegalovirus
B. rubella
C. syphilis
D. toxoplasma

American Academy of Pediatrics 460


PREP ® Self-Assessment PREPSA 2021
Correct Answer: B
The neonate described in the vignette has clinical manifestations of microcephaly, cataracts,
sensorineural hearing impairment, radiolucent osseous lesions, and possible cardiac disease, all
of which are highly suggestive of congenital rubella syndrome (CRS). Rubella virus infection of
the mother during pregnancy can lead to CRS, miscarriage, or fetal demise.

The risk of CRS is highest among neonates born to mothers in low-income and middle-income
countries because these women are more likely to be nonimmune or partially immune to rubella
owing to their having received no or incomplete immunizations. In contrast, the widespread use
of rubella vaccine (as part of the routine childhood MMR immunization series) resulted in the
elimination of rubella and CRS in the United States in 2004; imported cases accounted for the
majority of the 94 cases of rubella and nine cases of CRS reported during the post elimination
period of 2004 to 2014. In 2015, the Pan American Health Organization declared the elimination
of rubella and CRS from the region of the Americas.

Humans are the only source of rubella infection. In the United States, approximately 8% to 10%
of women of reproductive age lack immunity to rubella, making them susceptible to acquisition
of primary infection during pregnancy. Mother-to-baby transmission of rubella can occur during
pregnancy through transplacental spread of maternal virus. The risk of CRS with birth defects
can occur throughout pregnancy after maternal infection. However, the risk of fetal infection is
inversely related to the time in gestation when infection occurred. Congenital defects occur in up
to 85% of newborns with primary maternal infection during the first 12 weeks of gestation;
maternal infection during weeks 13 to 16 weeks of gestation and at the end of the second
trimester can lead to congenital defects in approximately 50% and 25% of newborns,
respectively. Mild forms of CRS may be asymptomatic at birth.

The most common congenital anomalies seen in CRS are ocular (eg, cataracts [Item C123A]),
retinitis, microphthalmos, glaucoma), cardiac (eg, patent ductus arteriosus, peripheral pulmonary
artery stenosis), auditory (eg, sensorineural hearing loss), and neurologic (eg, microcephaly,
meningoencephalitis, developmental delay, mental retardation, autism). Other manifestations of
CRS include radiolucent bone lesions (Item C123B), intrauterine growth restriction, intracranial
calcifications, jaundice, hepatosplenomegaly, petechial/purpuric rash due to thrombocytopenia,
and “blueberry muffin” lesions (Item C123C) due to extramedullary hematopoiesis.

American Academy of Pediatrics 461


PREP ® Self-Assessment PREPSA 2021

Item C123A: Bilateral cataracts in an infant with congenital rubella syndrome.


Courtesy of the Centers for Disease Control and Prevention

Item C123B: Radiolucent bone lesions in the metaphysis.


Reprinted with permission from Kimberlin DW, et al. eds. Red Book Online. American Academy
of Pediatrics. 2018.

American Academy of Pediatrics 462


PREP ® Self-Assessment PREPSA 2021

Item C123C: Blueberry muffin skin lesions.


Courtesy of Immunization Action Coalition and the Centers for Disease Control and Prevention.

The diagnosis of CRS is confirmed by detection of rubella-specific serum IgM antibody (usually
within the first 6 months after birth) or detection of virus in culture or polymerase chain reaction
(ie, throat, nose, urine). Documentation of stable or rubella-specific rising IgG titers over the first
7 to 11 months after birth is also suggestive of congenital infection. Rubella-specific serologic
testing must be performed at a reference laboratory, because false-positive and false-negative test
results are frequently encountered. Ophthalmologic evaluation may detect cataracts or
retinopathy, whereas computed tomographic scan of the brain may detect periventricular
calcifications. However, retinitis and intracranial calcifications may also be noted in other
congenital infections such as toxoplasmosis, cytomegalovirus, and zika virus. All neonates with
suspected CRS must have a hearing evaluation.

The treatment of CRS is supportive; no antiviral therapy is available. Infants with CRS must
have close follow-up with audiologic, ophthalmologic, and neurodevelopmental services. Infants
with CRS often shed the virus in saliva or urine for up to 1 year after birth. Contact isolation for
at least 1 year after birth is recommended for infants with proven or suspected CRS. Strict hand
hygiene is crucial for reducing transmission of virus to susceptible pregnant contacts.

The differential diagnosis of CRS must include other important congenital and perinatal
infections, such as cytomegalovirus, toxoplasmosis, syphilis, herpes simplex virus, lymphocytic
American Academy of Pediatrics 463
PREP ® Self-Assessment PREPSA 2021
choriomeningitis virus, and zika virus. Although each of the congenital infections share similar
clinical findings, there are unique clinical features and sequelae distinguishing one from the
other. This child is showing classic findings of CRS, making a diagnosis of congenital
cytomegalovirus less likely. The clinical manifestations of congenital syphilis include snuffles,
mucocutaneous lesions, osteochondritis, pseudoparalysis, hepatosplenomegaly, and
lymphadenopathy. Congenital toxoplasmosis is characterized by the triad of hydrocephalus,
chorioretinitis, and cerebral calcifications.

PREP Pearls
• Rubella virus infection of the mother during pregnancy can lead to congenital rubella
syndrome, miscarriage, or fetal demise.
• The risk of congenital rubella syndrome is highest among neonates born to mothers
immigrating to the United States from low-income and middle-income countries because
they have received no or incomplete immunizations.
• The most common manifestations of congenital rubella syndrome include microcephaly,
cataracts, sensorineural deafness, peripheral pulmonic stenosis, and radiolucent bone
lesions.

MOCA-Peds Objective
• Know the indications and appropriate testing for parasitic disorders.

ABP Content Specifications(s)


• Recognize the clinical features associated with congenital and postnatally acquired
rubella virus infection
• Understand the epidemiology of the rubella virus

Suggested Readings
• American Academy of Pediatrics. Rubella. In: Kimberlin DW, Brady MT, Jackson MA,
Long SS, eds. Red Book: 2018-2021 Report of the Committee on Infectious
Diseases. 31st ed. Elk Grove Village, IL: American Academy of Pediatrics; 2018:705-
710. Red Book Online.
• Centers for Disease Control and Prevention. Three cases of congenital rubella syndrome
in the postelimination era—Maryland, Alabama, and Illinois, 2012. MMWR Morb Mortal
Wkly Rep. 2013;62(12):226-229.
• Kojaoghlanian T. The newborn at risk for infection. In: McInerny TK, Adam HM,
Campbell DE, DeWitt TG, Foy JM, Kamat DM, eds. American Academy of Pediatrics
Textbook of Pediatric Care. 2nd ed. Elk Grove Village, IL: American Academy of
Pediatrics; 2017:899-908. Pediatric Care Online.
• McLean HQ, Fiebelkorn AP, Temte JL, Wallace GS; Centers for Disease Control and
Prevention. Prevention of measles, rubella, congenital rubella, and mumps, 2013
summary: recommendations of the Advisory Committee on Immunization Practices
(ACIP). MMWR Recomm Rep. 2013;62(RR-4):1-34.

American Academy of Pediatrics 464


PREP ® Self-Assessment PREPSA 2021
Question 124
A male neonate is evaluated in the nursery. He is stable on room air. He has midface hypoplasia,
up-slanting palpebral fissures, epicanthal folds, single transverse palmar crease, and wide gaps
between the first and second toes. An echocardiogram shows an atrioventricular canal defect. A
karyotype for the neonate shows trisomy 21 with an unbalanced translocation of chromosomes
14 and 21. On the basis of this result, karyotypes for both parents are recommended.

Of the following, the karyotype that carries the HIGHEST risk of recurrence of this syndrome for
these parents in subsequent pregnancies is
A. 45,XY,t(14;21) in the father
B. 45,XX,t(14;21) in the mother
C. 46,XY,t(14;21) in the newborn
D. 47,XY,+21 in the newborn

American Academy of Pediatrics 465


PREP ® Self-Assessment PREPSA 2021
Correct Answer: B
The neonate in the vignette has Down syndrome. Down syndrome is caused by an extra
chromosome 21. Karyotype is an important tool for confirming the diagnosis and determining
accurate recurrence risk (RR) for parents. The majority (approximately 95%) of cases with Down
syndrome are caused by a freestanding extra chromosome 21, depicted as 47,XY,+21 in the
answer choice for a male neonate. A freestanding extra chromosome 21 arises because of meiotic
nondisjunction, which in 90% of cases is maternal in origin. The risk of parents’ having another
child with Down syndrome with this karyotype would be 1% above the age-adjusted risk of the
mother in subsequent pregnancy.

In approximately 3% to 4% of cases with Down syndrome, the extra 21 arises because of an


unbalanced translocation between chromosome 21 and another acrocentric chromosome (13, 14,
15, 21, or 22) (two normal copies of chromosome 21 and one extra copy translocated onto
another chromosome). This would be denoted as 46,XY,t(14:21) for a male neonate, as in the
response above, in whom the entire chromosome 21 is translocated to chromosome 14. In
translocation cases, it is important to obtain the parents’ karyotype to determine whether either
parent has a balanced translocation, because this changes the RR (in a balanced translocation
there are two copies of chromosome 21, one freestanding and one translocated to another
chromosome). In approximately 25% to 30% of patients with Down syndrome who have an
unbalanced translocation, a parent will be a carrier of a balanced translocation. If neither parent
carries a balanced translocation, the RR is the same as that of freestanding chromosome 21. If
either parent carries a balanced translocation, the RR would be based on the sex of the parent.
The RR would be 10% to 15% if the mother is the carrier (mother’s karyotype: 45,XX,t[14;21])
versus 2% to 5% if the father is the carrier (father’s karyotype: 45,XY,t[14;21]). These
percentages are lower than would be expected on the basis of Mendelian genetics, owing to the
likely increased risk of spontaneous miscarriage in any fetus carrying a chromosomal
abnormality. If either parent is a carrier for translocation between chromosome 21 and another
21, the RR would be 100% with every liveborn child.

PREP Pearls
• Down syndrome is caused by an extra chromosome 21.
• Recurrence risk is higher if the mother is a carrier of a balanced translocation and lowest
if the child has a freestanding chromosome 21.

ABP Content Specifications(s)


• Understand the risk factors of having another child with trisomy 21 when the mother is a
balanced translocation carrier
• Understand the risk factors associated with subsequent pregnancies when an infant is
born with a translocation chromosome abnormality

American Academy of Pediatrics 466


PREP ® Self-Assessment PREPSA 2021
Suggested Readings
• Bull MJ. Down syndrome: managing the child and family. In: McInerny TK, Adam HM,
Campbell DE, DeWitt TG, Foy JM, Kamat DM, eds. American Academy of Pediatrics
Textbook of Pediatric Care. 2nd ed. Elk Grove Village, IL: American Academy of
Pediatrics; 2017:1976-1983. Pediatric Care Online .
• Bull MJ; Committee on Genetics. Health supervision for children with Down syndrome
(published correction appears in Pediatrics. 2011 Dec;128[6]:1212). Pediatrics.
2011;128(2):393-406. doi:10.1542/peds.2011-1605.
• Sheets KB, Crissman BG, Feist CD, et al. Practice guidelines for communicating a
prenatal or postnatal diagnosis of Down syndrome: recommendations of the national
society of genetic counselors. J Genet Couns. 2011;20(5):432-441. doi:10.1007/s10897-
011-9375-8.

American Academy of Pediatrics 467


PREP ® Self-Assessment PREPSA 2021
Question 125
An 8-year-old boy is brought to the office with concerns about a mass in his groin. He was
diagnosed with a buttock abscess 3 days ago that was successfully incised and drained. He has
been otherwise well, afebrile, and with no pain. His vital signs are normal. The boy’s physical
examination reveals a healing incision on his left buttock and a 2-cm subcutaneous, nontender,
nonerythematous, mobile, well-circumscribed nodule in his left groin.

Of the following, the BEST next step in management is to


A. admit him for intravenous antibiotics
B. obtain a complete blood cell count
C. perform ultrasonography of the lesion
D. provide parental reassurance

American Academy of Pediatrics 468


PREP ® Self-Assessment PREPSA 2021
Correct Answer: D
The child in the vignette has reactive lymphadenopathy from his recent abscess, so his parents
may be reassured and counseled on the self-resolving nature of his mass. Intravenous antibiotics,
a complete blood cell count, and ultrasonography are unnecessary because he exhibits no signs
of systemic infection or illness and the lesion has no malignant features such as firmness or
immobility.

Inguinal masses in children are most commonly the result of inguinal hernias, hydroceles, and
lymph nodes. Inguinal hernias and hydroceles are more prevalent in infants, whereas enlarged
lymph nodes and other less common causes are more prevalent in children and adolescents.
Inguinal hernias and hydroceles result from an abnormal persistent communication between the
abdomen and groin, known as a processus vaginalis in males and canal of Nuck in females. Item
C125 is a visual representation of the development of hernias and hydroceles in males; the
process is similar for females. Right-sided inguinal hernias are more common because of later
closure of the right-sided communication. Patients with a left-sided inguinal hernia are more
likely to also have a right-sided hernia.

Item C125: Development of hernias and hydroceles in males.


Reprinted with permission from Palmer LS. Scrotal swelling and pain. In: McInerny TK, Adam
HM, Campbell DE, DeWitt TG, Foy JM, Kamat DM, eds. American Academy of Pediatrics
Textbook of Pediatric Care. 2nd ed. Itasca, IL: American Academy of Pediatrics; 2016

An inguinal hernia or hydrocele usually presents as a painless inguinal bulge that worsens with
increased intra-abdominal pressure. Hernia incarceration is less common as an initial
presentation; this may be painless at first, but becomes more painful with time and may lead to
strangulation. Physical examination to assess for a hernia or hydrocele begins with inspection of
both sides of the child’s scrotum and inguinal region when the child is calm as well as when the
child is able to increase his intra-abdominal pressure, for example, when crying or jumping.
Palpation should begin at the superior aspect of the inguinal canal. In males, the silk-stocking
sign indicative of a hernia may be elicited by rubbing the spermatic cord contents from side to
side. Ultrasonography is usually not necessary for the diagnosis of hernia or hydrocele, but can
be used in males to verify the presence of a normal testicle. Surgical repair of a hydrocele is
usually performed after age 1 year. Repair for most inguinal hernias is nonurgent, but is
emergent when incarcerated.

American Academy of Pediatrics 469


PREP ® Self-Assessment PREPSA 2021

Inguinal lymph node enlargement is another common cause of inguinal masses. These nodes
drain from the lower extremities, buttocks, genitalia, and the abdominal wall below the
umbilicus. Normal inguinal lymph node size is up to 1.5 cm. Those larger than 2 cm are
indicative of a more serious cause, such as urinary tract or perineal infection in younger children,
and venereal disease or malignancy in adolescents. Patients who have acute lymph node
enlargement suspicious for infection (eg, tender to palpation, fluctuant, erythematous), but
without another source of infection, may be given an antibiotic trial to cover Staphylococcus
aureus, group A Streptococcus, and possibly Bartonella. If the enlargement does not resolve or
is chronic, a complete blood cell count, serologies for specific infectious organisms,
inflammatory markers, and markers for cell turnover may be recommended. If any results are
concerning, or the cause of lymph node enlargement is still elusive, patients should be referred
for specialist consultation for further evaluation.

Less common causes of inguinal masses include tumors and ectopic gonadal tissue.
Ultrasonography and biopsy may be useful in these cases.

PREP Pearls
• Reactive lymphadenopathy is one of the most common causes of inguinal masses in older
children and adolescents, and requires limited evaluation.
• Inguinal hernia and hydrocele are a more common cause of inguinal masses in younger
age groups.
• Patients with inguinal masses of uncertain etiology should undergo more extensive
evaluation with possible specialist referral.

MOCA-Peds Objective
• Evaluate and manage a child with an inguinal mass.

ABP Content Specifications(s)


• Plan the appropriate diagnostic evaluation of an inguinal mass in patients of various ages
• Understand the causes of an inguinal mass in patients of various ages

Suggested Readings
• Palmer LS. Hernias and hydroceles. Pediatr Rev. 2013;34(10):457-464.
doi: 10.1542/pir.34-10-457.
• Palmer LS. Scrotal swelling and pain. In: McInerny TK, Adam HM, Campbell DE,
DeWitt TG, Foy JM, Kamat DM, eds. American Academy of Pediatrics Textbook of
Pediatric Care. 2nd ed. Itasca, IL: American Academy of Pediatrics; 2016;chap
190:1571-1577. Pediatric Care Online.
• Sahai S. Lymphadenopathy. Pediatr Rev. 2013;34(5):216-227. doi: 10.1542/pir.34-5-216.

American Academy of Pediatrics 470


PREP ® Self-Assessment PREPSA 2021
Question 126
A 4-year-old boy is seen in the emergency department for 6 hours of progressive restlessness,
abnormal movements of the extremities, blurred vision, and drooling. His parents report that the
boy was in good health until he felt something sharp touch his right great toe as he put on his
shoes. His parents examined his toe at the time but were unable to see any marks. Over the
course of the next hour, he complained of a “funny feeling” in his right leg. Shortly thereafter,
the other symptoms commenced.

On physical examination, the boy’s temperature is 38.2°C, heart rate is 140 beats/min,
respiratory rate is 22 breaths/min, blood pressure is 110/60 mm Hg, and his room air oxygen
saturation is 100%. He is alert and oriented and able to answer questions appropriately. The boy
appears uncomfortable and is intermittently writhing on the stretcher. His pupils are equal,
round, and reactive to light; however, he has abnormal extraocular movements. His mucous
membranes are moist and he has minimal drooling. On auscultation, his heart has a normal
rhythm with no murmurs, scattered expiratory wheezes can be heard; and his abdominal
examination findings are normal. On skin examination, no marks are found and there is no
bruising. Palpation of his right great toe produces intense pain.

Of the following, the creature MOST likely to have caused this child’s symptoms is
A. Centruroides sculpturatus (Arizona bark scorpion)
B. Crotalus atrox (Western diamondback snake)
C. Latrodectus mactans (brown recluse spider)
D. Vespula squamosa (southern yellowjacket wasp)

American Academy of Pediatrics 471


PREP ® Self-Assessment PREPSA 2021
Correct Answer: A
The child in the vignette is experiencing symptoms of a scorpion sting. Scorpion stings are
extremely painful and can also cause local numbness and swelling without significant erythema
at the site of the sting or elsewhere on the body. Palpation or percussion of the affected area
causes an intense pain. In addition to significant local pain, scorpion stings have the potential to
cause severe systemic symptoms; the venom contains neurotoxins that result in both sympathetic
(eg, tachycardia, hyperthermia) and parasympathetic (eg, drooling, wheezing) symptoms.
Scorpion envenomation can also cause various neuromuscular symptoms, including restlessness,
muscular fasciculations, ataxia, and cranial nerve abnormalities such as abnormal eye
movements. Presence of cranial nerve and/or somatic skeletal muscle dysfunction are evidence
of a severe envenomation. Other severe sequelae of scorpion envenomation include myocarditis,
pancreatitis, and respiratory failure.

Scorpion envenomations are a significant issue worldwide, with over 1 million stings reported
annually. Though there is less mortality from scorpion stings than from bites of certain snakes
and spiders, there remains significant pediatric morbidity and a risk of death, especially in young
children. The only species of scorpion indigenous to the United States whose venom is potent
enough to cause severe symptoms in humans is the bark scorpion (Centruroides sculpturatus).
The primary habitat for this scorpion is the southwestern United States; however, the smuggling
of exotic insects has increased worldwide to the point where one may see scorpion stings in all
geographic areas.

Similar to spiders, scorpions are a part of the Arthropoda phylum in the class Arachnida.
However, instead of envenomation from a bite, scorpions use a stinger known as a telson, located
in their tail. When hunting, the scorpion will grasp its prey in its pincers, swing its tail over its
body, and sting. Scorpions are not inherently aggressive; they only sting humans in self-defense;
however, given the scorpion’s propensity for hiding in sleeping bags and clothing, recipients of a
scorpion sting are often unsuspecting and unaware that they have been “aggressive” toward the
creature.

Management of scorpion stings commences with the evaluation and management of the airway,
breathing, and circulation. Symptomatic treatment is paramount and ranges from analgesia to the
use of benzodiazepines for neuromuscular excitation to intubation for respiratory failure. The
child’s tetanus vaccination status should be assessed; postexposure prophylaxis should be
provided if necessary. Antivenom for Centruroides sculpturatus is available and should be
considered for use in severe cases.

Envenomation from a diamondback snake can also cause severe systemic neurologic symptoms;
however, typically, significant erythema and edema are seen at the site of the bite which may
progress to ecchymosis and hemorrhagic blebs. Systemic symptoms caused by the bite of a
brown recluse spider are rare, though the local manifestations can be severe and ultimately
require procedures to facilitate appropriate healing. The sting of a yellowjacket is painful and is
typically associated with local edema and erythema. Systemic symptoms are rare, and when
present, are typically consistent with anaphylaxis.

American Academy of Pediatrics 472


PREP ® Self-Assessment PREPSA 2021
PREP Pearls
• Scorpion stings present with a mixture of sympathetic, parasympathetic, and
neuromuscular symptoms.
• Supportive care is the mainstay of management for scorpion stings.
• Scorpion antivenom can be considered for use in severe cases.
• Systemic symptoms caused by the bite of a brown recluse spider are rare, but the local
manifestations can be severe.

ABP Content Specifications(s)


• Plan the appropriate management of a scorpion sting

Suggested Readings
• Banerji S, Bronstein AC. Envenomations. In: McInerny TK, Adam HM, Campbell DE,
DeWitt TG, Foy JM, Kamat DM, eds. American Academy of Pediatrics Textbook of
Pediatric Care. 2nd ed. Itasca, IL: American Academy of Pediatrics; 2016;chap
358. Pediatric Care Online.
• Isbister GK, Bawaskar HS. Scorpion envenomation. N Engl J Med. 2014;371(5):457-463.
doi: 10.1056/NEJMra1401108.
• Skolnik AB, Ewald MB. Pediatric scorpion envenomation in the United States:
morbidity, mortality, and therapeutic innovations. Pediatr Emerg Care. 2013;29(1):98-
103; doi: 10.1097/PEC.0b013e31827b5733.

American Academy of Pediatrics 473


PREP ® Self-Assessment PREPSA 2021
Question 127
A previously healthy, 4-year-old girl is seen in the office for evaluation of a 4-day history of
shortness of breath and hoarseness. Her family noticed a lump in her neck 3 weeks ago, and it
has continued to grow. She has also had a dry cough for 1 week, which her family attributed to a
viral illness. She is moderately distressed with shortness of breath. There is a 3-cm × 3-cm,
nontender, supraclavicular mass on the left side of her neck, and her neck veins are prominent.
Her face is mildly plethoric, and she has audible stridor. She has diminished breath sounds on the
left side, but the right side is clear with no abnormal sounds. The remainder of her physical
examination findings are unremarkable. A chest radiograph (Item Q127A) and computed
tomographic scan of the chest (Item Q127B) are obtained.

Item Q127A: Chest radiograph of the girl in the vignette. Courtesy of J. Kane

American Academy of Pediatrics 474


PREP ® Self-Assessment PREPSA 2021

Item Q127B: Computed tomogram of the girl in the vignette. Courtesy of J. Kane

Of the following, the mechanism that BEST explains the physical examination findings is
A. compression of the superior vena cava
B. left mainstem bronchus obstruction
C. pulmonary embolism
D. tumor lysis syndrome

American Academy of Pediatrics 475


PREP ® Self-Assessment PREPSA 2021
Correct Answer: A
The patient in the vignette is displaying physical signs of superior vena cava (SVC) syndrome
caused by compression of the SVC by an anterior mediastinal mass. The radiograph and
computed tomographic scan provided (Item C127A, Item C127B) demonstrate a large anterior
mediastinal mass compressing the SVC, as well as tracheal compression and deviation. A second
radiograph from a different patient (Item C127C) shows a large right-sided mediastinal mass
obscuring the right heart border with widening of the mediastinum.

Item C127A: Chest radiograph of the girl in the vignette. Courtesy of J. Kane

American Academy of Pediatrics 476


PREP ® Self-Assessment PREPSA 2021

Item C127B: Computed tomogram of the girl in the vignette. Courtesy of J. Kane

American Academy of Pediatrics 477


PREP ® Self-Assessment PREPSA 2021

Item C127C: Chest radiograph of a patient with anterior mediastinal mass. Courtesy of J. Kane

Superior vena cava syndrome results from compression of vascular structures in the
mediastinum, causing obstruction of the venous drainage of the SVC. Mediastinal tumors are the
most common primary causes of SVC syndrome; however, thrombi associated with long-term
indwelling central venous catheters are also a known risk factor.

The SVC is formed by the confluence of the left and right brachiocephalic veins in the middle
one-third of the mediastinum. Venous drainage to the right atrium is facilitated by low pressures
in the right heart. Additional structures that interface with the SVC include the right mainstem
bronchus, numerous lymph node chains, and the aorta. In the presence of large mediastinal
tumors, severe lymphadenopathy, or dilation of the ascending aorta, the SVC is compressed,
resulting in diminished venous return to the right side of the heart. In severe cases, complete
occlusion can result in complete absence of venous return. The severity of SVC syndrome
depends on the degree of SVC occlusion and the degree of collateral circulation to promote
venous drainage.

American Academy of Pediatrics 478


PREP ® Self-Assessment PREPSA 2021

Clinical features of SVC syndrome relate to the lack of adequate venous return to the right side
of the heart with upstream stasis of blood flow. Because the SVC receives blood from both the
left and right brachiocephalic veins, patients will exhibit edema of the head and neck, distended
neck veins, facial plethora, and cyanosis of the face. Patients may also develop cough, shortness
of breath, and orthopnea, all of which suggest upper airway obstruction. Symptoms are generally
worse when patients are placed in the supine position; in patients with significant vascular or
airway compromise, prone positioning may help relieve some of the compression of the SVC
and can be life saving.

The management of SVC syndrome is targeted at relief of symptoms and alleviating the primary
process. Pediatric patients with rapid-onset obstruction of the SVC caused by mediastinal tumors
are at risk of experiencing life-threatening complications. Initial stabilization and resuscitation
depends on the severity of symptoms and type of primary mediastinal tumor. Supportive care,
including elevation of the head of the bed, supplemental oxygen, and judicious use of diuretics,
may be required. Treatment for tumor reduction should be coordinated by the pediatric
intensivist and oncologist and may require corticosteroids, chemotherapy, or radiation therapy. In
severe cases, cardiopulmonary bypass or extracorporeal membrane oxygenation may be needed
until SVC compression can be relieved.

Although left mainstem bronchus obstruction may occur from the mass lesion shown, this alone
would not result in all of the symptoms described. Neither pulmonary embolism nor tumor lysis
syndrome would produce the symptoms described.

PREP Pearls
• Anterior mediastinal masses may cause obstruction of venous drainage of the superior
vena cava, which is a medical emergency.
• Facial plethora, cyanosis, and neck vein distension can all be signs of superior vena cava
syndrome.
• Initial steps in the management of superior vena cava syndrome include targeted therapy
aimed at relief of superior vena cava obstruction while limiting supine positioning.

ABP Content Specifications(s)


• Recognize the clinical findings associated with superior vena cava syndrome

Suggested Readings
• Issa PY, Brihi ER, Janin Y, Slim MS. Superior vena cava syndrome in childhood: report
of ten cases and review of the literature. Pediatrics. 1983;71(3):337-
341. https://pediatrics.aappublications.org/content/71/3/337/tab-article-info.
• Yuen S, Linney MJ, John P, Berry K. Superior vena cava obstruction in an 8-month old
infant. Pediatr Emerg Care. 2003;19(2):91-94. doi:10.1097/00006565-200304000-
00007.

American Academy of Pediatrics 479


PREP ® Self-Assessment PREPSA 2021
Question 128
A 2-year-old girl, not yet toilet trained, is being evaluated for diarrhea. She began having loose
stools 4 months ago, and now has 4 to 6 nonbloody stools daily. Her first-morning stool tends to
be somewhat formed. As the day progresses, her stools become looser, with mucus and whole
pieces of food present. She has frequent diaper rashes that resolve after a single application of
over-the-counter diaper cream. The girl is gaining weight well and meeting her developmental
milestones. Her mother is lactose-intolerant and has been keeping lactose out of the girl’s diet;
the girl has no other diet restrictions. She drinks 24 oz of juice and 20 oz of lactose-free whole
milk daily. Her favorite foods are pancakes and syrup, peanut butter and jelly sandwiches, and
spaghetti and meatballs. Her medical and surgical history are unremarkable.

Her weight is 12.6 kg (50th percentile), length is 87 cm (40th percentile), and body mass index is
17 kg/m2 (67th percentile). She is well nourished and interactive. Her abdomen is soft and
nontender, with mild distention. She has no diaper rash. The remainder of the physical
examination findings are within normal limits.

Of the following, the dietary intervention MOST likely to improve the girl’s symptoms is
reduced consumption of
A. cow milk protein
B. fat
C. fructose
D. gluten

American Academy of Pediatrics 480


PREP ® Self-Assessment PREPSA 2021
Correct Answer: C
The child in the vignette has functional diarrhea, also known as chronic nonspecific diarrhea or
toddler’s diarrhea, which can be exacerbated by excessive fructose intake resulting from juice or
sugar-sweetened beverage consumption. Thus, limiting fructose intake can improve symptoms in
children with functional diarrhea. Low-fat and low-fiber diets may also exacerbate functional
diarrhea.

Parents often have concerns about the frequency of defecation, and helping them to understand
normal stool patterns is important. At 4 weeks of age, average stool frequency is 3 per day, and
generally higher in breastfed infants. Stool frequency slows during early childhood, and by age 4
years, the average frequency is once daily. Nearly all children (97%) between ages 1 and 3 years
pass stools from once every other day to up to 3 times daily, with 11% passing stools 3 times
daily. Up to 25% of 1- to 3-year-old children may pass stools at night during sleep.

Functional diarrhea is defined as passage of more than 3 stools daily for more than 4 weeks, with
onset of symptoms between 6 and 60 months of age. It is related to a self-limited alteration in
intestinal motility that results in a shortened intestinal transit time. This diagnosis requires that
there should be no evidence of a malabsorptive syndrome, failure to thrive, blood in the stool, or
severe diaper dermatitis that cannot be managed with diaper creams or ointments. Children with
functional diarrhea commonly have stools with mucus and/or undigested food and generally have
more liquid stool as the day progresses. Treatment involves reassurance and dietary interventions
including limiting fructose intake (particularly sugar sweetened beverages and low-fiber foods)
and increasing intake of fiber and fat. Functional diarrhea generally resolves by age 5 to 6 years.

Cow milk protein restriction is helpful for children with cow milk protein intolerance/food
protein–induced proctocolitis. Typical symptoms include onset of symptoms during infancy and
hematochezia, which are not consistent with the clinical presentation of the child in the vignette.
Lower-fat diets may exacerbate functional diarrhea, therefore limiting fat would not improve this
child’s symptoms. Reducing gluten would not be expected to improve functional diarrhea in this
age group.

PREP Pearls
• Functional diarrhea is a benign and self-limiting process in young children.
• Functional diarrhea occurs in the absence of evidence of a malabsorptive syndrome,
failure to thrive, hematochezia, and severe resistant diaper rash.
• Helpful dietary interventions for functional diarrhea include limiting fructose and
increasing fiber and fat intake.

ABP Content Specifications(s)


• Recognize the clinical and laboratory features associated with milk-protein intolerance,
and manage appropriately
• Recognize the clinical features associated with chronic nonspecific diarrhea, and the
prognosis for affected patients
• Identify possible causes of chronic nonspecific diarrhea

American Academy of Pediatrics 481


PREP ® Self-Assessment PREPSA 2021
Suggested Readings
• Benninga MC, Nurko S, Fauer C, Hyman PE, Roberts Ian St. James, Schecter NL.
Childhood functional gastrointestinal disorders: neonate/toddler. Gastroenterology.
2016;150:1443-1455. doi: 10.1053/j.gastro.2016.02.016.
• CaJacob NJ, Cohen MB. Update on diarrhea. Pediatr Rev. 2016;37(8):313-322.
doi: 10.1542/pir.2015-0099.
• Ulshen MH. Diarrhea and steatorrhea. In: McInerny TK, Adam HM, Campbell DE,
DeWitt TG, Foy JM, Kamat DM, eds. American Academy of Pediatrics Textbook of
Pediatric Care. 2nd ed. Itasca, IL: American Academy of Pediatrics; 2016;chap
138:1267-
1281. https://pediatriccare.solutions.aap.org/chapter.aspx?sectionId=108723097&bookId
=1626&resultClick=1.

American Academy of Pediatrics 482


PREP ® Self-Assessment PREPSA 2021
Question 129
An 8-year-old boy with asthma is brought to the clinic for routine follow-up and a health
supervision visit. The boy has had three exacerbations of asthma since his last visit 6 months
ago. His mother has read on the Internet about an herbal supplement that can reduce
exacerbations in children with asthma. She would like to start giving this herbal supplement to
her son. She notes that a number of people have posted comments indicating that their own
children had fewer asthma attacks when they took this supplement.

Of the following, the BEST way to assess the efficacy of this herbal supplement in children with
asthma is to perform a
A. cohort study
B. cross-sectional study
C. randomized controlled trial
D. retrospective chart review

American Academy of Pediatrics 483


PREP ® Self-Assessment PREPSA 2021
Correct Answer: C
Designing clinical research requires a thoughtful approach to addressing the specific clinical
question. Each specific type of research study design has advantages and disadvantages. The best
way to assess the efficacy of the herbal supplement described in the vignette in children with
asthma is to perform a randomized controlled trial.

A randomized controlled trial is considered the reference standard of research design. In these
studies, participants receive random assignment to either an experimental or a control group. As
the study is performed, the only expected difference between the two groups is the outcome
variable being studied. By randomly assigning patients to different treatment options, a
randomized controlled trial limits population and investigator bias. To provide greater avoidance
of bias, interventions can also be masked for both participant and researcher. Randomized
controlled trials are expensive and require extensive time and effort to recruit and follow patients
prospectively over time. Patients lost to follow-up often pose a challenge in randomized
controlled trials.

A cohort study is a prospective observational study that involves select members of a population
who share a common condition or exposure and follows them over time. Cohort studies are
performed to determine whether an initial exposure or risk factor is associated with the outcome
measure. Randomization is not done in a cohort study; this may result in significant differences
in the baseline characteristics within the cohorts. Because a cohort study follows patients over
time, outcome measures cannot be obtained immediately and in some instances can take years to
accomplish. For instance, a cohort study might be used to assess whether exposure to lead early
in life affects intellectual performance in school-aged children. A cohort of children exposed to
lead and another of children not so exposed would be followed over time and assessed for
intellectual performance during the study period.

A cross-sectional study evaluates the relationship between variables in a defined population at a


single point in time (or limited short period). These studies are used to define the frequency of a
condition in a population at a set point in time and are often used to assess the burden of a
disease or health needs of a population. Cross-sectional studies are quick and easy to conduct.
These studies focusing on a specific problem at a specific point in time can measure only
prevalence and not incidence of a disease or condition. Risk factors and outcomes are measured
simultaneously, and causation cannot be determined. A cross-sectional design might be used to
assess the prevalence of asthma among children aged 6 to 10 years in a sample of US elementary
schoolchildren.

Retrospective studies use data that have previously been collected (such as information recorded
in a medical record). In a retrospective study, the data usually were not originally collected as
part of a research study. Retrospective chart review is a method of collecting data and can be
used in a number of different study designs, including case-control studies. Retrospective studies
are generally considered inferior to prospective studies but are inexpensive and easy to perform.
Retrospective data are often used for preliminary studies or for the study of rare conditions. The
results of retrospective studies frequently serve as the impetus for subsequent prospective
studies.

American Academy of Pediatrics 484


PREP ® Self-Assessment PREPSA 2021

PREP Pearls
• The specific research question determines the type of study design that should be used.
• Randomized controlled trials are considered the reference standard for prospective
analysis of a specific treatment or intervention.

ABP Content Specifications(s)


• Understand the validity hierarchy for study design and study type
• Understand the uses and limitations of controlled clinical trials
• Understand the uses and limitations of randomized clinical trials

Suggested Readings
• Bothwell LE, Greene JA, Podolsky SH, Jones DS. Assessing the gold standard—lessons
from the history of RCTs. N Engl J Med. 2016;374(22):2175-2181.
doi:10.1056/NEJMms1604593.
• Robbins BW. Evidence-based medicine. In: McInerny TK, Adam HM, Campbell DE,
DeWitt TG, Foy JM, Kamat DM, eds. American Academy of Pediatrics Textbook of
Pediatric Care. 2nd ed. Elk Grove Village, IL: American Academy of Pediatrics;
2017:29-32. Pediatric Care Online.
• Sibbald B, Roland M. Understanding controlled trials. Why are randomised controlled
trials important? BMJ. 1998;316(7126):201. doi:10.1136/bmj.316.7126.201.

American Academy of Pediatrics 485


PREP ® Self-Assessment PREPSA 2021
Question 130
During a health supervision visit, a 14-year-old adolescent boy with known primary hypertension
is found to have a blood pressure of 146/90 mm Hg. The cuff size was confirmed as appropriate.
His body mass index is at the 85th percentile. His history and physical examination findings are
otherwise unremarkable, and previous evaluations for end-organ damage have been negative.
Bilateral radial and femoral pulses are normal and without delay. The adolescent has been
participating in high school football and weight training without complaint.

Of the following, the BEST recommendation for this adolescent regarding football participation
is to
A. continue aerobic and low-intensity conditioning only
B. continue current participation without restriction
C. continue noncontact conditioning only
D. discontinue all sports and conditioning activity

American Academy of Pediatrics 486


PREP ® Self-Assessment PREPSA 2021

Correct Answer: A
The prevalence of hypertension among overweight and obese adolescents ranges from 3.8% to
24.8%; it is the most common cardiovascular condition seen in competitive athletes. The
adolescent in this vignette has a blood pressure reading consistent with stage 2 hypertension. He
merits restriction from competitive sports activities but he should continue aerobic and low-
intensity conditioning only. Although participation in exercise is generally associated with a
decrease in blood pressure over time, the acute blood pressure response during exercise varies
depending on the type of activity (Item C130).

American Academy of Pediatrics 487


PREP ® Self-Assessment PREPSA 2021

Item C130: Classification of sports by cardiovascular demand.


Adapted and reprinted with permission from Mitchell JH, Haskell W, Snell P, Van Camp SP.
38th Bethesda Conference. J Am Coll Cardiol. 2005;45(8):1364 –1367.

American Academy of Pediatrics 488


PREP ® Self-Assessment PREPSA 2021
• Dynamic activity, which includes traditional “aerobic activity” such as running, results in
an acute increase in systolic blood pressure, but a decrease in diastolic blood pressure and
total peripheral resistance.
• Static activity, such as weight training, results in an acute increase in systolic and
diastolic blood pressures, without significant change in total peripheral resistance.

There is a theoretical concern about potential harm from these additional blood pressure
elevations in young hypertensive athletes, particularly regarding the potential for cerebrovascular
accidents. However, there is little evidence to support these concerns; current recommendations,
outlined here, are primarily based on consensus and expert opinion.

The AAP’s 2017 “Clinical Practice Guideline for Screening and Management of High Blood
Pressure in Children and Adolescents” makes the following “key action statements” regarding
those diagnosed with elevated blood pressure or hypertension and athletic participation:
o Clinicians should recommend moderate to vigorous physical activity at least 3 to
5 days per week (30-60 minutes per session) to help reduce blood pressure (grade
C, weak recommendation).
o Children and adolescents with hypertension may participate in competitive sports
once hypertensive target organ effects (including left ventricular hypertrophy) and
risk have been assessed (grade C, moderate recommendation).
• It is important to remember that athletic conditioning itself may lead to left ventricular
hypertrophy; distinguishing between pathologic and physiologic changes may be
difficult.
• Children and adolescents with stage 2 hypertension should receive treatment to lower
blood pressure to below stage 2 thresholds before participating in competitive sports
(grade C, weak recommendation).

The American Heart Association, American College of Cardiology, and the AAP’s Council on
Sports Medicine and Fitness more specifically recommend that athletes with stage 2
hypertension should be restricted from high static activity (Item C130) until blood pressure is
under control. Although football is considered a “moderate static” activity, weight training is
considered “high static” and this boy should not participate until his blood pressure is below his
stage 2 threshold (ie, <140/90 mm Hg).

Whether a conditioning activity is contact vs noncontact is not pertinent when considering


activity restrictions related to high blood pressure.

PREP Pearls
• Children and adolescents with elevated blood pressure (previously known as
prehypertension) or stage 1 hypertension can participate in unrestricted athletic activity as
long as there is no left ventricular hypertrophy or other end-organ damage.
• Children and adolescents with diagnosed hypertension should achieve blood pressure
control below stage 2 levels before clearance to play a competitive sport.
• Static activity, such as weight training, results in an acute increase in systolic and
diastolic blood pressures, without significant change in total peripheral resistance until
blood pressure readings are below stage 2 thresholds.
American Academy of Pediatrics 489
PREP ® Self-Assessment PREPSA 2021

MOCA-Peds Objective
• Identify contraindications for sports participation.

ABP Content Specifications(s)


• Identify which sports are appropriate for athletes with various conditions that may limit
sports participation

Suggested Readings
• Black HR, Sica D, Ferdinand K, White WB; American Heart Association
Electrocardiography and Arrhythmias Committee of Council on Clinical Cardiology,
Council on Cardiovascular Disease in Young, Council on Cardiovascular and Stroke
Nursing, Council on Functional Genomics and Translational Biology, and American
College of Cardiology. Eligibility and disqualification recommendations for competitive
athletes with cardiovascular abnormalities: Task Force 6: Hypertension: a scientific
statement from the American Heart Association and the American College of
Cardiology. Circulation. 2015;132(22):e298-302. doi: 10.1161/CIR.0000000000000242.
• Flynn JT, Kaelber DC, Baker-Smith CM, et al; Subcommittee on Screening and
Management of High Blood Pressure in Children. Clinical practice guideline for
screening and management of high blood pressure in children and
adolescents. Pediatrics. 2017;140(3):e20171904. doi: 10.1542/peds.2017-3035.
• Miller SM, Peterson AR. The sports preparticipation evaluation. Pediatr
Rev. 2019;40(3):108-128. doi: 10.1542/pir.2016-0216.
• Weaver DJ. Pediatric hypertension: review of updated guidelines. Pediatr
Rev. 2019;40(7):354-358. doi: 10.1542/pir.2018-0014.

American Academy of Pediatrics 490


PREP ® Self-Assessment PREPSA 2021
Question 131
A previously healthy 15-year-old adolescent boy who has experienced body aches and dark urine
for 24 hours is brought to the emergency department. He has diffuse pain over his thigh, back,
and arm muscles. He has voided only a small amount of red-colored urine since the previous
night. He has no history of fever, vomiting, diarrhea, or sore throat. He started football season 2
days before presentation and participated in a push-up challenge. He has a heart rate of 110
beats/min, a respiratory rate of 16 breaths/min, and a blood pressure of 100/70 mm Hg. He has
diffuse tenderness of his proximal muscles. The rest of the physical examination findings are
unremarkable. An electrocardiogram shows peaked T waves.

Intravenous normal saline 20 mL/kg is started. Laboratory data are shown:


Laboratory Test Result
Blood
Sodium 136 mEq/L (136 mmol/L)
Potassium 7.4 mEq/L (7.4 mmol/L)
Chloride 100 mEq/L (100 mmol/L)
Bicarbonate 16 mEq/L (16 mmol/L)
Blood urea nitrogen 40 mg/dL (14.3 mmol/L)
Creatinine 2.2 mg/dL (194 µmol/L)
Calcium 8.0 mg/dL (2.00 mmol/L)
Phosphorus 6.0 mg/dL (1.94 mmol/L)
Creatine kinase 76,000 U/L
Glucose 100 mg/dL (5.0 mmol/L)
Urine
pH 6.0
Specific gravity 1.020
Blood 3+
Protein Negative
Red blood cells < 2/HPF
White blood cells < 5/HPF

Of the following, the BEST next step in management is intravenous administration of


A. calcium gluconate
B. furosemide
C. insulin
D. sodium bicarbonate

American Academy of Pediatrics 491


PREP ® Self-Assessment PREPSA 2021
Correct Answer: A
The adolescent boy in this vignette, with a history of intense muscle activity, muscle pain, and
elevated creatinine kinase, has rhabdomyolysis. He has myoglobinuria, suggested by dark red
urine; positive test result for blood on urinalysis; and absence of red blood cells on urine
microscopy. He also has associated acute kidney injury, severe hyperkalemia, metabolic
acidosis, and hyperphosphatemia. With hyperkalemia causing a peaked T wave on
electrocardiogram, intravenous calcium gluconate should be urgently administered to this child
to stabilize the cardiac membrane.

Hyperkalemia is defined as a serum potassium level higher than the upper limit of normal value.
Hyperkalemia can be a result of increased potassium intake, transcellular shift of potassium from
the intracellular to the extracellular compartment, and decreased renal excretion. The causes of
hyperkalemia are shown in Item C131A.

American Academy of Pediatrics 492


PREP ® Self-Assessment PREPSA 2021

American Academy of Pediatrics 493


PREP ® Self-Assessment PREPSA 2021
Mild hyperkalemia (5.5 mEq/L-6 mEq/L) and moderate hyperkalemia (6 mEq/L-7 mEq/L) may
be asymptomatic. Severe hyperkalemia (more than 7 mEq/L) can be life threatening and can
present with muscle weakness or paralysis and syncope and palpitations due to cardiac
conduction abnormalities. The electrocardiographic findings in hyperkalemia include tall peaked
T waves, prolonged PR interval, absent or disappearing P wave, widening QRS complex,
ventricular fibrillation, and asystole.

The treatment of hyperkalemia involves stabilization of cardiac conduction, rapidly shifting


potassium from the extracellular to the intracellular compartment, and increasing excretion of
potassium. In severe hyperkalemia or with presence of electrocardiographic abnormalities, the
immediate first step is to administer a calcium gluconate infusion. Calcium infusion does not
decrease the serum potassium value but stabilizes the cardiac membrane and prevents the risk of
developing cardiac arrhythmias.

The serum potassium value can be decreased temporarily by rapidly shifting the potassium from
the extracellular compartment to the intracellular. The combination of insulin and dextrose
infusion is the preferred method because of its rapid onset of action. Inhaled β-adrenergic agonist
(albuterol) is an alternative for children who do not have secure intravenous access. However,
the effect of albuterol is short lived and should be avoided in children with preexisting cardiac
arrhythmia. Intravenous sodium bicarbonate also moves potassium intracellularly; however, its
benefit in hyperkalemia is controversial, and it is used only in combination with other therapies.
The therapies that remove potassium from the body are use of loop and thiazide diuretics; use of
cation exchange resins, which bind potassium in the gastrointestinal tract (sodium polystyrene
sulfonate); and dialysis. The common medications used in the treatment of hyperkalemia are
shown in Item C131B.

The adolescent in this vignette has severe hyperkalemia and peaked T wave and, thus, needs
immediate infusion of calcium gluconate. Insulin, furosemide, and sodium bicarbonate are
treatment options to reduce the serum potassium levels; however, their administration should be
considered only after the calcium gluconate infusion.

American Academy of Pediatrics 494


PREP ® Self-Assessment PREPSA 2021
PREP Pearls
• Severe hyperkalemia can present with muscle weakness, paralysis, and syncope and
palpitations due to cardiac conduction abnormalities.
• Electrocardiographic findings in hyperkalemia include tall peaked T waves, prolonged
PR interval, absent P wave, widening QRS complex, ventricular fibrillation, and asystole.
• In severe hyperkalemia or with presence of electrocardiographic abnormalities, the
immediate first step is to administer a calcium gluconate infusion to stabilize the cardiac
membrane.

ABP Content Specifications(s)


• Recognize the clinical and laboratory features associated with hyperkalemia, and manage
appropriately, including during an emergency situation
• Recognize the clinical and laboratory features associated with hypokalemia, and manage
appropriately, including during an emergency situation

Suggested Readings
• Goldstein SL, Adrogue HE. Acute kidney injury. In: McInerny TK, Adam HM, Campbell
DE, DeWitt TG, Foy JM, Kamat DM, eds. American Academy of Pediatrics Textbook of
Pediatric Care. 2nd ed. Elk Grove Village, IL: American Academy of Pediatrics;
2017:2895-2898. Pediatric Care Online .
• Mahajan P, Felt JR. Fluids, electrolytes, and acid-base composition. In: McInerny TK,
Adam HM, Campbell DE, DeWitt TG, Foy JM, Kamat DM, eds. American Academy of
Pediatrics Textbook of Pediatric Care. 2nd ed. Elk Grove Village, IL: American
Academy of Pediatrics; 2017:419-432. Pediatric Care Online .

American Academy of Pediatrics 495


PREP ® Self-Assessment PREPSA 2021
Question 132
A newborn is being evaluated in the nursery. He was born this morning via vaginal delivery to a
32-year-old mother who did not receive prenatal care until 28 weeks of gestation. Apgar scores
were 8 and 9 at 1 and 5 minutes, respectively. He did well at birth and has been rooming in with
his mother. Physical examination reveals a newborn whose growth parameters are appropriate
for gestational age and has otherwise unremarkable examination findings. His mother’s medical
records are reviewed.

Maternal laboratory data are as follows:


Laboratory Test Patient Result
Group B Streptococcus culture Negative
HIV enzyme-linked immunosorbent assay Negative
Rapid plasma reagin Nonreactive
Hepatitis B surface antigen Negative
Hepatitis B core antibody Positive
Hepatitis B surface antibody Positive
Rubella Nonimmune

Of the following, the mother’s hepatitis B status is MOST likely


A. actively infected
B. immune due to infection
C. immune due to vaccination
D. susceptible to infection

American Academy of Pediatrics 496


PREP ® Self-Assessment PREPSA 2021
Correct Answer: B
Various assays are available for the diagnosis of hepatitis B virus infection, including antigen
and antibody tests and molecular assays. The laboratory data shown in the vignette demonstrate
that the mother’s hepatitis B virus status is immune because of infection. Hepatitis B core
antibodies develop only in the presence of infection; core antigen is not contained in the vaccine.
The absence of hepatitis B surface antigen and presence of hepatitis B surface antibodies indicate
a resolved infection (Item C132).

Hepatitis B surface antigen is present in individuals with acute or chronic infection, and in
vaccinated individuals up to 3 weeks after vaccination. The presence of hepatitis B e antigen
identifies individuals who are at higher risk of transmitting infection.

American Academy of Pediatrics 497


PREP ® Self-Assessment PREPSA 2021
Antibodies against hepatitis B surface antigen are found in individuals with resolved hepatitis B
virus infection or those with immunity resulting from immunization. Individuals who have
developed antibodies against hepatitis B e antigen are less likely to transmit infection. Antibodies
against hepatitis B core antigen can be found in individuals with acute, chronic, or resolved
hepatitis B virus infection. Hepatitis B quantitative polymerase chain reaction tests are used to
monitor chronic infection and response to therapy.

PREP Pearls
• Hepatitis B surface antigen is present in individuals with acute or chronic infection and
vaccinated individuals for 3 weeks after receiving the hepatitis B vaccination.
• Hepatitis B core antibodies develop only in the presence of infection; core antigen is not
contained in the vaccine.
• The presence of hepatitis B e antigen identifies individuals who are at higher risk of
transmitting infection.

ABP Content Specifications(s)


• Plan the diagnostic evaluation of suspected hepatitis B virus infection

Suggested Readings
• American Academy of Pediatrics. Hepatitis B. In: Kimberlin DW, Brady MT, Jackson
MA, Long SS, eds. Red Book: 2018 Report of the Committee on Infectious Diseases.
Itasca, IL: American Academy of Pediatrics; 2018:401-428.
• Davis A, Rosenthal P. Hepatitis B in children. Pediatr Rev. 2008;29(4):111-120.
doi: 10.1542/pir.29-4-111.
• Hardikar W, Schwarz KB. Hepatitis. In: McInerny TK, Adam HM, Campbell DE,
DeWitt TG, Foy JM, Kamat DM, eds. American Academy of Pediatrics Textbook of
Pediatric Care. 2nd ed. Itasca, IL: American Academy of Pediatrics; 2016; chap
265. Pediatric Care Online.

American Academy of Pediatrics 498


PREP ® Self-Assessment PREPSA 2021
Question 133
A 16-year-old adolescent boy is being seen for his annual health supervision visit. His mother is
concerned that he has recently quit playing on both his school and club soccer teams. He has
always been an A student but is failing two classes at school, and he has started having outbursts
at home and speaks rudely to his parents and younger sister. The mother expresses her frustration
and inquires whether her son needs a psychiatric evaluation.

During the confidential interview, he reports that his mother is exaggerating and that he can get
his grades up in the classes he is failing. He quit playing soccer because it no longer interests
him. He discloses that over the last 6 months he has smoked cigarettes once or twice per week,
smoked marijiuana daily to help him sleep at night, and has tried ecstasy once. He reports no
other drug use and does not drink alcohol. He does not feel that he has a problem and says he
knows he can stop using any of these substances at any time. When asked if it would be all right
to discuss his substance use with his mother, he becomes agitated and does not want anything he
discussed in private to be disclosed.

Of the following, the BEST next step in treatment is to


A. break his confidentiality and discuss the substance use concerns with his mother because
it is unlikely, he will be able to stop using on his own
B. create a safety contract stipulating that he will cut his marijuana use down to three times
per week and follow up with him in 1 month
C. order a urine drug screen, without informing the patient, to evaluate whether he is using
other substances aside from what he disclosed
D. provide him with resource information about 24-hour hotlines and drug rehabilitation
programs should he decide he needs assistance in cutting down on his drug use

American Academy of Pediatrics 499


PREP ® Self-Assessment PREPSA 2021
Correct Answer: A
The boy in the vignette does not admit to having a substance abuse problem and does not want
information about his substance use disclosed to his mother. He is using marijuana daily, which
is likely involved in his changing grades and disrespectful behavior. It is unlikely that he will
create a safety contract or seek treatment on his own, because he does not acknowledge that he
has a problem. A urine drug test is not a part of routine drug screening, and if it is done without
consent, it jeopardizes the patient-physician relationship. The best plan to ensure that this
adolescent remains safe and receives appropriate services would be to break confidentiality and
involve his mother in the plan of care.

The American Academy of Pediatrics recommends screening adolescents for substance abuse at
routine health supervision visits starting at age 12 years. The primary care provider (PCP) is
often the first to identify a substance abuse problem and is instrumental in managing the disorder
and providing resources if higher levels of care are needed for treatment. The PCP also has to
determine when confidentiality about an adolescent’s care may need to be broken. The
substances most commonly used by adolescents are alcohol, tobacco, and marijuana. Inhalants,
prescription drugs, cocaine, ecstasy, and heroin are also drugs of abuse in adolescents, although
far less common. More complexities have arisen with the use of e-cigarettes/vaping and the
legalization of marijuana for medicinal purposes, recreational purposes, or both in at least 33
states in the United States.

Multiple factors contribute to adolescent substance use and abuse, including genetic, biological,
and social influences. From a biological standpoint, the prefrontal cortex, which is associated
with executive functioning, is one of the last portions of the brain to mature; it does not fully
develop until approximately age 25 years. The adolescent who is using marijuana or alcohol is
activating and reinforcing his or her incentive and reward pathways to respond to the substance
of use. Social and genetic factors may be protective against substance use when adolescents
connect to school, sports teams, and family in a positive way, having a sense of community.
Parental modeling of responsible behavior about drug and alcohol use sets clear expectations
about undesirable substance use. On the converse side, poor choices made by parents or peers
can influence impressionable adolescents and potentiate substance abuse.

Substance use should be suspected in an adolescent who has a recent change in school
performance or drops out; becomes more aggressive or violent; is disrespectful and
argumentative with family members; and shows signs of depression or expresses suicidal
ideation. There are numerous validated screening tools to assess substance use in adolescents,
available both online and in paper format. The Screening to Brief Intervention tool screens for
use of alcohol, tobacco, marijuana, and other illicit drugs. It differentiates between no use, no
substance use disorder (SUD), and moderate to severe SUD on the basis of criteria
from Diagnostic and Statistical Manual of Mental Disorders, 5th edition. Another commonly
used assessment tool is the CRAFFT questions (https://crafft.org), which allow a quick screening
to assess for substance use. A score of 2 or higher on the CRAFFT questions constitutes a
positive result and requires further assessment for SUD.

Brief interventions can be done in the PCP’s office via motivational interviewing to evaluate
whether an adolescent is interested in making a change and to help the adolescent identify
American Academy of Pediatrics 500
PREP ® Self-Assessment PREPSA 2021
reasons for change or identify consequences from use. For adolescents who are identified by
screening tools as being at lower risk with their substance use, a safety plan should be
established so they are not put at risk by, for instance, a driver who may be under the influence.
Adolescents identified as having severe SUD should be referred for formal rehabilitation
treatment (inpatient versus outpatient). Primary care providers must be knowledgeable about
resources available in their communities, including drug and alcohol centers and mental health
services. Substance use is included as a part of adolescent confidentiality; however,
confidentiality may need to be broken if the teen is at high risk of injuring self or others, is
unwilling to make changes, or refuses to disclose to his or her parents. If the adolescent is 18
years or older, recommendations can be made; however, information cannot be disclosed to
parents without consent. High risk of experiencing injury includes driving under the influence,
making hospital visits for drug-related problems, using intravenous drugs, and combining
sedatives (eg, taking alcohol and benzodiazepines). It is best to encourage the teen to disclose his
or her substance abuse problems to a parent or caregiver to establish support throughout the
treatment process, as relapse may occur.

PREP Pearls
• Pediatric patients should be screened for substance abuse at routine health supervision
visits starting at age 12 years.
• The most commonly abused drugs used by adolescents are tobacco, marijuana, and
alcohol.
• It is crucial to involve family members in the treatment of adolescents with moderate to
severe substance use disorder, as relapse is likely to occur.
• In situations of substance abuse in which patient well-being is at risk, it is appropriate to
break confidentiality to allow care to occur.

MOCA-Peds Objective
• Characterize alcohol use in an adolescent.

ABP Content Specifications(s)


• Understand the clinical circumstances in which further evaluation for substance use/abuse
is indicated
• Understand the importance of a screening interview for substance use/abuse in
adolescents
• Recognize the important history to obtain when evaluating a patient for possible
substance use/abuse

American Academy of Pediatrics 501


PREP ® Self-Assessment PREPSA 2021
Suggested Readings
• Levy S, Weitzman ER. Approaches to adolescent and young adult substance use. In:
Neinstein LS, Katzman DK, Callahan ST, Gordon CM, Joffe A, Rickert VI,
eds. Neinstein’s Adolescent and Young Adult Health Care: A Practical Guide. 6th ed.
Philadelphia, PA: Wolters Kluwer; 2016:567-576.
• Levy S, Williams JF; Committee on Substance Use and Prevention. Substance use
screening, brief intervention, and referral to treatment. Pediatrics.
2016;138(1);e20161210. doi:10.1542/peds.2016-1210.
• Nackers KA, Kokotailo P, Levy SJ. Substance abuse: general principles. Pediatr Rev.
2015; 36(12):535-544. doi:10.1542/pir.36-12-535.

American Academy of Pediatrics 502


PREP ® Self-Assessment PREPSA 2021
Question 134
A 10-year-old girl with type 1 diabetes is seen for decreased appetite and fatigue over the past
month. She eats three meals per day, but she consumes small portions because of nausea. She has
not had abdominal pain, vomiting, diarrhea, constipation, or recent illness. Her diabetes is treated
with rapid-acting insulin via an insulin pump. She has had a 2.3-kg weight loss since her last
health supervision visit 4 months ago. Her vital signs and the remainder of her physical
examination findings are normal. Her most recent hemoglobin A1c level was 7.9% 2 months
ago. Today, her hemoglobin level is 10 g/dL (100 g/L) with a mean corpuscular volume of 72 fL.

Of the following, the MOST likely underlying diagnosis causing this girl’s symptoms is
A. adrenal insufficiency
B. celiac disease
C. hypothyroidism
D. inflammatory bowel disease

American Academy of Pediatrics 503


PREP ® Self-Assessment PREPSA 2021
Correct Answer: B
The girl described in the vignette has type 1 diabetes and symptoms consistent with celiac
disease. These symptoms include decreased appetite, fatigue, nausea, weight loss, and microcytic
anemia. Celiac disease occurs with increased frequency in those with type 1 diabetes. Occurring
in up to 16% of those with type 1 diabetes, celiac disease is the second most common associated
autoimmune condition after autoimmune thyroid disease. Guidelines recommend screening
children with type 1 diabetes for celiac disease around the time of their diabetes diagnosis, with
repeat screening within 2 years of diagnosis, then after 5 years. More frequent screening is
recommended for those with symptoms of celiac disease. Common symptoms in those with type
1 diabetes include poor growth and increased hypoglycemia. A tissue transglutaminase IgA
antibody level is the recommended screening test.

Autoimmune thyroid disease, usually hypothyroidism, is the autoimmune condition most


commonly associated with type 1 diabetes. Autoimmune thyroid disease occurs in up to 30% of
those with type 1 diabetes. Guidelines recommend testing those with type 1 diabetes for thyroid
peroxidase and antithyroglobulin antibodies around the time of their diabetes diagnosis.
Guidelines also recommend measuring thyroid-stimulating hormone level around the time of
diagnosis after glycemic control is established, then every 1 to 2 years. More frequent
measurement is indicated for symptoms or signs of thyroid dysfunction, such as an enlarged
thyroid gland, abnormal linear growth, or unexplained glycemic variability. The signs and
symptoms of the girl in the vignette are more consistent with celiac disease than with
hypothyroidism. Weight loss is not a common symptom of hypothyroidism.

Adrenal insufficiency also occurs with increased frequency in those with type 1 diabetes.
However, the overall incidence, especially in the pediatric population, remains low. There are no
routine screening recommendations for adrenal insufficiency in those with type 1 diabetes. For
the girl in the vignette, her fatigue, nausea, and weight loss can be consistent with adrenal
insufficiency. However, celiac disease is more common, and her microcytic anemia is more
consistent with celiac disease.

The girl’s lack of abdominal pain and diarrhea makes inflammatory bowel disease less likely.
Celiac disease is also more common than inflammatory bowel disease in those with type 1
diabetes.

PREP Pearls
• Autoimmune thyroid disease, usually hypothyroidism, is the autoimmune condition most
commonly associated with type 1 diabetes.
• Celiac disease is the autoimmune condition second most commonly associated with type
1 diabetes.
• Guidelines recommend routine screening for thyroid disease and celiac disease in patients
with type 1 diabetes at the time of diagnosis of diabetes and periodically thereafter.

American Academy of Pediatrics 504


PREP ® Self-Assessment PREPSA 2021
ABP Content Specifications(s)
• Plan the appropriate management of type 1 diabetes to effectively achieve good control
and to avoid long-term complications

Suggested Readings
• American Diabetes Association. 13. Children and adolescents: standards of medical care
in diabetes—2020. Diabetes Care. 2020;43(Suppl 1):S163-S182. doi:10.2337/dc20-
S013.
• Ediger TR, Hill ID. Celiac disease. Pediatr Rev. 2014;35(10):409-415.
doi:10.1542/pir.35-10-409.
• Gregory JM, Moore DJ, Simmons JH. Type 1 diabetes mellitus. Pediatr
Rev. 2013;34(5):203-215. doi:10.1542/pir.34-5-203.
• Javed A, Schwenk WF, Tebben P. Diabetes mellitus. In: McInerny TK, Adam HM,
Campbell DE, DeWitt TG, Foy JM, Kamat DM, eds. American Academy of Pediatrics
Textbook of Pediatric Care. 2nd ed. Elk Grove Village, IL: American Academy of
Pediatrics; 2017:1952-1961. Pediatric Care Online.

American Academy of Pediatrics 505


PREP ® Self-Assessment PREPSA 2021
Question 135
A 6-week-old male infant is brought to the clinic for evaluation of fussiness. His mother reports
that for the past week, he has been crying excessively. He usually cries in the evening for 3 hours
and is difficult to console. She has tried swaddling and comforting her infant without success. He
has been afebrile, breastfeeding well, and voiding and stooling normally. His physical
examination yields normal findings. The mother asks which treatment may help her infant.

Of the following, the BEST recommendation is


A. antacid therapy
B. elimination diet for the mother
C. probiotics
D. simethicone

American Academy of Pediatrics 506


PREP ® Self-Assessment PREPSA 2021
Correct Answer: C
Infant colic has been defined as crying more than 3 hours per day, 3 days per week, and 3 weeks
at a time. Colic is common, occurring in up to one-quarter of all infants. It peaks around 6 weeks
of age and typically resolves by 4 months of age. Periods of crying typically occur in the late
afternoon or evening. In between periods of crying, colicky infants feed and act normally. The
growth trajectory and examination findings of infants with colic are typically normal. No
laboratory or imaging evaluation is needed for the diagnosis, but a detailed history and physical
examination should rule out other causes of excessive crying, such as constipation, feeding
disorders, gastroesophageal reflux disease, infection, or occult fracture.

Colic can be disruptive and very troubling to parents. As such, many interventions have been
tested in an attempt to find an easy, scalable, effective way to quiet excessive infant crying.
These efforts—including maternal elimination diets, simethicone, antacid therapy, gripe water,
soy-based formulas, and infant massage—have been largely unsuccessful in eliminating colic.
The prevailing advice to parents of colicky infants consists of reassurance that it is a self-limited
problem and is best handled by swaddling the infant, using a pacifier, and maintaining a quiet,
soothing environment. Although the first-line management of colic is parental support, this does
not always ease symptoms or parental stress.

Probiotics are not suggested for routine management of colic in breastfed infants, but a recent
randomized control trial and a systematic review demonstrated that treatment with Lactobacillus
reuteri DSM 17938 was associated with decreased crying time in breastfed infants. However, the
possible benefit of L reuteri must be weighed against the risks—including cost, lack of insurance
coverage, uncertain safety, and lack of evaluation by the US Food and Drug Administration—
especially because colic naturally improves over time. Nevertheless, for breastfed infants who do
not respond to standard measures and whose parents prefer to use probiotics, L reuteri may be
considered after discussion of the uncertainties, risks, and benefits of its use.

PREP Pearls
• Colic is common, is self-limited, and can be diagnosed with a careful history and physical
examination.
• First-line management of colic consists of parental support and infant soothing measures.
• For breastfed infants who do not respond to standard measures and whose parents prefer
to use probiotics, Lactobacillus reuteri may be considered after discussion of the
uncertainties, risks, and benefits of its use.

ABP Content Specifications(s)


• Plan the appropriate evaluation of colic
• Differentiate normal variations in feeding patterns from those that reflect poor parenting

American Academy of Pediatrics 507


PREP ® Self-Assessment PREPSA 2021
Suggested Readings
• Baum RA. Colic. In: McInerny TK, Adam HM, Campbell DE, DeWitt TG, Foy JM,
Kamat DM, eds. American Academy of Pediatrics Textbook of Pediatric Care. 2nd ed.
Elk Grove Village, IL: American Academy of Pediatrics; 2017:1868-1870. Pediatric Care
Online .
• Cohen GM, Albertini LW. Colic. Pediatr Rev. 2012;33(7):332-333. doi:10.1542/pir.33-7-
332.
• Perry R, Hunt K, Ernst E. Nutritional supplements and other complementary medicines
for infantile colic: a systematic review. Pediatrics. 2011;127(4):720-733.
doi:http://dx.doi.org/10.1542/peds.2010-2098.
• Sung V, D’Amico F, Cabana MD, et al. Lactobacillus reuteri to treat infant colic: a meta-
analysis. Pediatrics. 2018;141(1):e20171811. doi:10.1542/peds.2017-1811.

American Academy of Pediatrics 508


PREP ® Self-Assessment PREPSA 2021
Question 136
A practice seeks to decrease obesity rates as a way to improve population health and is looking
to identify risk factors that may be associated with obesity. After discussion, the team
hypothesizes that children who play video games for more than 2 hours a day have higher body
mass index (BMI) percentiles. The staff develops a survey to document BMI percentiles and
number of hours spent playing video games every day. This survey is distributed to each child
during the annual health supervision visit over the course of 1 month. The results are as follows:
Video game No. of Patients with BMI <95th No. of Patients with BMI >95th
Total
use Percentile for Age Percentile for Age
<2 h/day 160 40 200
>2 h/day 50 50 100
Total 210 90 300

Of the following, the study DESIGN used by the practice was


A. case control
B. cross-sectional
C. randomized controlled
D. retrospective cohort

American Academy of Pediatrics 509


PREP ® Self-Assessment PREPSA 2021
Correct Answer: B
The office practice in the vignette was studying the association between hours spent playing
video games every day and obesity. Because all measurements (obtained through a single
survey) were made at the same time and not followed over time, this is a cross-sectional study.
Cross-sectional studies are a type of observational study that can be used to measure associations
and can assess prevalence of an outcome or disease. Cross-sectional studies can generally be
completed quickly and are relatively inexpensive; as such, they are a good choice for
investigation before conducting a larger cohort study or randomized controlled trial (RCT).
Limitations of cross-sectional studies include an inability to establish causal relationships and
difficulty in studying rare, less prevalent diseases or outcomes.

A case-control study is a type of observational study used to assess associations between


exposures (risk) and outcomes. This type of study design is most commonly retrospective and
involves using 2 groups: those with the disease/outcome (cases) and those who do not have the
disease/outcome (controls). Case-control studies determine an odds ratio and can be used to
evaluate rare outcomes or diseases; they are generally inexpensive and not very time consuming.
Similar to a cross-sectional study, a case-control study measures associations but not causality.
An RCT is an experimental study that can determine causation between a treatment and a desired
outcome. Typically RCTs randomize subjects into 1 of 2 groups: those receiving treatment
(experimental group) and those receiving no treatment/placebo (control group). They are
generally more time consuming and expensive.

A cohort study is an observational study that assesses associations between exposures (risk) and
outcomes by analyzing 2 groups: those with the exposure/risk and those without the
exposure/risk. A cohort study can be prospective or retrospective. Cohort studies calculate
relative risk (incidence in exposed/incidence in unexposed). Limitations of cohort studies include
selection bias and the cost and time involved if studying rare diseases/outcomes.
Item C136 shows the strengths and weaknesses of different observational and research study
designs.

PREP Pearls
• Cross-sectional studies are observational studies that can determine the prevalence of a
particular outcome or disease.
• Case-control studies are observational studies that can be used to determine the odds ratio
of developing a particular outcome by analyzing groups of subjects with and without
particular exposures.
• Cohort studies are observational studies that calculate relative risk from an exposure
(incidence in exposed/incidence in unexposed).
• Randomized controlled trials are experimental studies that can determine causation
between a treatment and a desired outcome.

ABP Content Specifications(s)


• Understand the uses and limitations of descriptive epidemiologic studies

American Academy of Pediatrics 510


PREP ® Self-Assessment PREPSA 2021
Suggested Readings
• Centers for Disease Control and Prevention. Principles of epidemiology in public health
practice, third edition: an introduction to applied epidemiology and
biostatistics. https://www.cdc.gov/csels/dsepd/ss1978/lesson1/section7.html.
• Mann CJ. Observational research methods: research design II—cohort, cross sectional,
and case control series. Emerg Med J. 2003;20:54-60. doi: 10.1136/emj.20.1.54.
• Perry-Parrish C, Dodge R. Validity hierarchy for study design and study type. Pediatr
Rev. 2010;31(1):27-29. doi: 10.1542/pir.31-1-27.

American Academy of Pediatrics 511


PREP ® Self-Assessment PREPSA 2021
Question 137
A 2-month-old male infant is seen for a health supervision visit. His mother reports that he has
noisy breathing, particularly when he is sleeping at night. He is breastfed and nurses well; he has
only occasional regurgitations after feeding. On physical examination, his room air oxygen
saturation is 100%, weight is at the 60th percentile, and length is at the 80th percentile. The
infant breathes quietly at rest, but has inspiratory stridor when he is agitated or crying, which
resolves when he calms. When placed supine on the examination table, he has intermittent stridor
associated with intercostal retractions. When placed prone, the stridor resolves.

Of the following, the BEST next management step for this infant is
A. change from breast milk to an elemental formula
B. immediate referral to an otolaryngologist for evaluation
C. reassurance that symptoms will resolve by age 18 to 24 months
D. use of a home apnea monitor for 6 months

American Academy of Pediatrics 512


PREP ® Self-Assessment PREPSA 2021
Correct Answer: C
The infant in the vignette has laryngomalacia, which will likely resolve without intervention by
age 12 to 18 months. Evidence of good feeding and growth, normal oxygen saturation, and lack
of respiratory compromise are encouraging findings for a benign course. Reassurance is the only
intervention indicated at this point. Because the stridor is often dynamic, it may not be as
obvious at an office visit as it may seem to the parents during the night at home. Inviting parents
to bring audio/video recordings of the breathing pattern to an office visit is a way to make sure
they feel heard and understood.

There is nothing in the vignette to suggest milk intolerance, therefore changing to elemental
formula would not be helpful in this case. Referral to an otolaryngologist may be requested by
parents who want to feel absolutely confident that no intervention is needed. However, in the
presence of good growth and normal oxygen saturation, the primary care physician can reassure
the parents that this is not necessary. Use of an apnea monitor or oximeter may increase parental
anxiety and divert attention from the infant to the monitor.

Stridor can be inspiratory or biphasic. Inspiratory stridor is associated with variable extrathoracic
obstruction (such as laryngomalacia). Biphasic (inspiratory and expiratory) stridor is associated
with a fixed obstruction, usually of the upper trachea, such as a tracheal stenosis or extrinsic
compression from a mass or vascular structure.

Some causes of chronic stridor do require additional evaluation and intervention.


Laryngomalacia varies in severity; although usually not concerning, it can be critical and present
with poor growth, respiratory distress, and low oxygen saturations, particularly with sleep. In
such cases, a referral to otolaryngology for confirmation and consideration of laryngoplasty is
appropriate. Other causes of stridor in an infant include vocal cord paralysis (unilateral or
bilateral), hemangioma or other mass lesion of the extrathoracic airway, and tracheal stenosis.
Infants who have had endotracheal intubation, particularly those who had repeated
instrumentation of the airway or prolonged intubation, are at risk for secondary damage to the
airway, which can present as inspiratory or biphasic stridor.

The most common cause of acute stridor in children is viral croup, which is associated with an
insidious onset of fever, coryza, and other upper respiratory symptoms; a high-pitched, barking
cough; and inspiratory stridor. Most episodes of croup can be managed expectantly and
conservatively, but in some children the inspiratory stridor and respiratory distress warrant
treatment with dexamethasone or prednisolone. Other causes of acute stridor include
retropharyngeal abscess and bacterial epiglottitis (a medical emergency) which is characterized
by high fever, severe respiratory distress, and inability to handle oral secretions.

PREP Pearls
• The most common cause of stridor in healthy infants is laryngomalacia.
• The most common cause of acute stridor in children is viral croup.
• Infants with laryngomalacia with good feeding and growth, normal oxygen saturation,
and lack of respiratory compromise need no intervention other than reassurance.

American Academy of Pediatrics 513


PREP ® Self-Assessment PREPSA 2021
MOCA-Peds Objective
• Recognize and manage paradoxical vocal cord dysfunction.

ABP Content Specifications(s)


• Plan the appropriate clinical and diagnostic evaluation of laryngeal and vocal cord
disorders
• Plan the appropriate clinical and diagnostic evaluation of stridor in patients of various
ages
• Plan the appropriate management for stridor of various etiologies

Suggested Readings
• Escobar ML, Needleman J. Stridor. Pediatr Rev. 2015;36:135-137. doi: 10.1542/pir.36-3-
135.
• Ida JB, Thompson DM. Pediatric stridor. Otolaryngol Clin North Am. 2014;47:795-819.
doi: 10.1016/j.otc.2014.06.005.
• Pfleger A, Eber E. Assessment and causes of stridor. Paedir Resp Rev. 2016;18:64-72.
doi: 10.1016/j.prrv.2015.10.003.
• Vicencio AG, Bent JP. Stridor. In: McInerny TK, Adam HM, Campbell DE, DeWitt TG,
Foy JM, Kamat DM, eds. American Academy of Pediatrics Textbook of Pediatric
Care. 2nd ed. Itasca, IL: American Academy of Pediatrics; 2016;chap 197:1615-
1619. Pediatric Care Online.

American Academy of Pediatrics 514


PREP ® Self-Assessment PREPSA 2021
Question 138
A girl comes to the clinic with her mother for a health supervision visit. She sits quietly reading a
book, which she can do with just a little help from her mother. The girl’s participation in age-
appropriate activities and her development are reviewed. She has joined a soccer team this year
and really enjoys playing competitive sports. Math is her favorite class; they are learning
fractions. She names several friends and says she enjoys playing with both sexes but does not
have one single best friend.

Of the following, the girl’s age is MOST likely


A. 4 years
B. 6 years
C. 8 years
D. 10 years

American Academy of Pediatrics 515


PREP ® Self-Assessment PREPSA 2021
Correct Answer: C
On the basis of the social and cognitive skills described, the girl in the vignette is most likely 8
years old. Children’s social and cognitive skills grow substantially from 6 to 12 years of age.
This 8-year-old is developing relationships with her peers, often of both sexes, but may not have
a single best friend yet. At this age, it is appropriate to participate in competitive sports, as well
as other group activities such as scouting. Efficiency and confidence in reading and school
subjects, as well as independence, are growing.

Six-year-olds begin to develop an awareness of their peers and make meaningful relationships
with peers and adults. Other skills include following simple directions, counting to 100,
demonstrating basic writing skills, and demonstrating emerging reading skills such as
recognizing words on sight or reading simple books. Participation in sports begins, but the focus
is on participation rather than competition.

For a 10-year-old, problem-solving skills and developing self-confidence are important.


Increasing importance of peer relationships, usually of the same sex, and sharing ideas with their
peers become more apparent. Participation in after-school activities such as sports, music, or
other organizations increases.

By 12 years of age, an awareness and engagement within the community, outside the peer group,
develops. The ability to show compassion, empathy, and resilience strengthens. More complex
and abstract topics are learned in school.

PREP Pearls
• Peer relationships become increasingly influential from 6 to 12 years of age.
• Participation in sports and other activities begins around 6 years of age and increases
throughout grade school.
• Between the ages of 6 and 12 years, a child’s focus moves from oneself to peers and,
finally, to the community.

ABP Content Specifications(s)


• Evaluate the cognitive and behavioral developmental progress/status of a child at 6 to 12
years of age

Suggested Readings
• American Academy of Pediatrics; Hagan JF, Shaw JS, Duncan PM, eds. Middle
childhood: 5 and 6 year visits. In: Bright Futures: Guidelines for Health Supervision of
Infants, Children, and Adolescents, Pocket Guide. Itasca, IL: American Academy of
Pediatrics;
2017. https://brightfutures.aap.org/Bright%20Futures%20Documents/BF4_MiddleChildh
oodVisits.pdf.
• Scharf RJ, Scharf GJ, Stroustrup A. Developmental milestones. Pediatr
Rev. 2016;37(1):25-37. doi:10.1542/pir.2014-0103.

American Academy of Pediatrics 516


PREP ® Self-Assessment PREPSA 2021
Question 139
A 17-year-old adolescent girl is seen for a health supervision visit. Her medical history is
significant for a diagnosis of acute T-lymphoblastic leukemia at 12 years of age. She was treated
with chemotherapy and 1,800-cGy cranial irradiation. She completed all treatment 2 years ago
and remains in complete remission.

The importance of healthy eating, particularly given the treatment she received, is discussed.

Of the following, the BEST description of how she compares with her peers is that she has
A. a higher basal metabolic rate
B. a higher protein intake requirement
C. an increased risk of experiencing anorexia nervosa
D. an increased risk of experiencing metabolic syndrome

American Academy of Pediatrics 517


PREP ® Self-Assessment PREPSA 2021
Correct Answer: D
Whereas the caloric needs of children being treated for cancer are increased over their basal
state, those of survivors are very different. Survivors of childhood cancer—especially those who
received hypothalamic radiation therapy (including cranial radiation therapy), as did the girl in
the vignette—are at increased risk of experiencing metabolic syndrome. This disease state is
characterized by
• Overweight
• Hypertension
• Hyperlipidemia
• Hyperglycemia (diabetes)

The treatment of childhood cancer has made remarkable progress during the last 60 years, such
that more than 80% of children diagnosed with cancer today will be cured. As a consequence,
there are now more than 500,000 survivors of childhood cancer alive in the United States. A
significant majority of survivors of childhood cancer have a chronic health condition as a
consequence of disease or the therapy they received. Given the high burden of both morbidity
and mortality among survivors of childhood cancer, the Children’s Oncology Group has
developed guidelines for lifelong late-effects screening for this population (available
at www.survivorshipguidelines.org).

Children actively being treated for cancer have higher caloric needs, but survivors of childhood
cancer have neither a basal metabolic rate nor a need for dietary protein higher than that of
otherwise healthy children. Although survivors of childhood cancer are at higher risk than is the
general population of experiencing psychiatric disorders, they are not at higher risk of
experiencing anorexia nervosa.

PREP Pearls
• Survivors of childhood cancer, especially those who received cranial radiation therapy,
are at increased risk of developing metabolic syndrome.
• A significant majority of survivors of childhood cancer have a chronic health condition as
a consequence of the disease or therapy they received.
• Metabolic syndrome is a disease state characterized by overweight, hypertension,
hyperlipidemia, and hyperglycemia.

ABP Content Specifications(s)


• Recognize the specific nutritional problems in a child with a malignancy

American Academy of Pediatrics 518


PREP ® Self-Assessment PREPSA 2021
Suggested Readings
• Bhatia S. Long-term complications of therapeutic exposures in childhood: lessons learned
from childhood cancer survivors. Pediatrics. 2012;130(6):1141-1143.
doi:10.1542/peds.2012-2884.
• Lee L, Sanders RA. Metabolic syndrome. Pediatr Rev. 2012;33(10):459-466.
doi:10.1542/pir.33-10-459.
• Wang HC, Gahagan S. Obesity and metabolic syndrome. In: McInerny TK, Adam HM,
Campbell DE, DeWitt TG, Foy JM, Kamat DM, eds. American Academy of Pediatrics
Textbook of Pediatric Care. 2nd ed. Elk Grove Village, IL: American Academy of
Pediatrics; 2017:2396-2405. Pediatric Care Online .
• Wittcopp C, Conroy R. Metabolic syndrome in children and adolescents. Pediatr Rev.
2016;37(5):193-202. doi:10.1542/pir.2014-0095.

American Academy of Pediatrics 519


PREP ® Self-Assessment PREPSA 2021
Question 140
A 35-week-gestation female neonate is delivered at home precipitously. Her mother had routine
prenatal care. The father drives the mother and neonate to the nearest hospital, with the mother
holding the neonate in a wet receiving blanket. On arrival at the emergency department, the
neonate has a temperature of 35.6°C, heart rate of 92 beats/min, respiratory rate of 15
breaths/min, blood pressure of 55/37 mm Hg, and oxygen saturation on pulse oximetry of 87% in
room air. Her extremities are cool.

Of the following, the MOST likely mechanism for the neonate’s heat loss is
A. conductive
B. convective
C. evaporative
D. radiant

American Academy of Pediatrics 520


PREP ® Self-Assessment PREPSA 2021
Correct Answer: C
Thermoregulation is crucial for neonates. They are born wet and have a large surface area-to-
volume ratio, increasing their risk of heat loss. They have a limited ability to maintain a normal
temperature between 36.5°C and 37.5°C. Hypothermia in neonates has been associated with a
number of adverse outcomes including hypoglycemia, apnea, and bradycardia. There are 4
primary mechanisms for heat loss in a neonate: evaporative, convective, conductive, and radiant.

The neonate in the vignette is losing heat by evaporation. The wet blanket wrapped around the
neonate allows continuous evaporative heat loss. Radiant heat loss refers to heat loss to the
surrounding air. Because the neonate is covered with a blanket, there is likely minimal radiant
heat loss. Conductive heat loss describes thermal energy transferred to an adjacent object.
Because the neonate is being held by her mother, conductive heat loss should be minimal.
Convective heat loss is loss of heat because of moving air around the neonate. If this neonate
were exposed to air conditioning or air drafts, the convective heat loss could be more significant.

For term neonates, immediate skin-to-skin care is an effective means of preventing hypothermia.
Neonates should be placed prone on their mother’s abdomen with a dry blanket over them to
prevent evaporative and convective heat loss. Premature neonates in high-resource settings
should be covered with a plastic covering or placed on a thermal mattress to prevent heat loss.

PREP Pearls
• There are 4 primary mechanisms for heat loss in a neonate: evaporative, convective,
conductive, and radiant.
• Evaporative heat loss is significant in neonates in part because of their high surface area-
to-volume ratio.
• Immediate skin-to-skin care after delivery is an effective means of preventing
hypothermia in term neonates.

ABP Content Specifications(s)


• Recognize the signs and symptoms of cold stress in a newborn infant, and manage
appropriately
• Recognize the signs and symptoms of heat loss in a newborn infant, and manage
appropriately

Suggested Readings
• Geis GM, Wood KS. Care of the sick or premature infant before transport. In: McInerny
TK, Adam HM, Campbell DE, DeWitt TG, Foy JM, Kamat DM, eds. American Academy
of Pediatrics Textbook of Pediatric Care. 2nd ed. Itasca, IL: American Academy of
Pediatrics; 2016;chap 111:1011-1017. Pediatric Care Online.
• Goyal NK. The newborn infant. In: Kliegman R, St Geme JW, Blum NJ, Shah SS, Tasker
RC, Wilson Karen, eds. Nelson Textbook of Pediatrics. Philadelphia, PA: Elsevier;
2020:867-876.e1.
• Warren JB, Phillipi CA. Care of the well newborn. Pediatr Rev. 2012;33(1):4-18.
doi: 10.1542/pir.33-1-4.

American Academy of Pediatrics 521


PREP ® Self-Assessment PREPSA 2021
Question 141
A 3-year-old boy is seen in the office for evaluation of swelling over his body of 5 days’
duration. His parents report that the swelling started over his eyelids (Item Q141) and
progressed to his lower limbs. He has no history of recent illness, increase in appetite, decrease
in urine output, or change in urine color. He is at the 95th percentile for weight and 50th
percentile for height. He has a heart rate of 100 beats/min, a respiratory rate of 18 breaths/min,
and a blood pressure of 105/62 mm Hg. His lungs are clear. There is mild distension of his
abdomen but no shifting dullness upon palpation. He has 2+ pitting edema of his lower
extremities. His genital examination has normal findings.

Item Q141: Orbital swelling as described for the patient in the vignette. Courtesy of M Rimsza.

American Academy of Pediatrics 522


PREP ® Self-Assessment PREPSA 2021
Laboratory data are shown:
Laboratory Test Result
Blood
Sodium 132 mEq/L (132 mmol/L)
Potassium 4.1 mEq/L (4.1 mmol/L)
Chloride 100 mEq/L (100 mmol/L)
Bicarbonate 22 mEq/L (22 mmol/L)
Blood urea nitrogen 18 mg/dL (6.4 mmol/L)
Creatinine 0.3 mg/dL (27 µmol/L)
Calcium 6.9 mg/dL (1.73 mmol/L)
Albumin 1.5 g/dL (15 g/L)
Urine
pH 6.0
Specific gravity 1.030
Protein 4+
Blood Negative
Red blood cells < 2/HPF
White blood cells < 5/HPF

Of the following, the MOST appropriate treatment for this child is


A. intravenous calcium gluconate
B. intravenous furosemide
C. oral enalapril
D. oral prednisolone

American Academy of Pediatrics 523


PREP ® Self-Assessment PREPSA 2021
Correct Answer: D
The boy in this vignette has edema, hypoalbuminemia, and proteinuria, all of which favor a
diagnosis of nephrotic syndrome (NS). The most appropriate treatment of NS in this child is oral
prednisolone.

Nephrotic syndrome is characterized by proteinuria (urine protein:creatinine ratio > 2 mg/mg or


urine protein > 40 mg/m2/hour), hypoalbuminemia (< 2.5 g/dL), edema, and hyperlipidemia. The
peak age of presentation is between 2 years and 10 years. Minimal change disease is the most
common histology in children with idiopathic NS.

Prednisone or prednisolone is the first-line treatment for idiopathic NS. The recommended
starting dose for an initial episode is 60 mg/m2/day (2 mg/kg/day) in two divided doses for 6
weeks, followed by 40 mg/m2 (1.5 mg/kg/day) as a single dose every other day for 6 weeks.
Nephrotic syndrome is known to have relapses and remission. The treatment of relapse includes
prednisone at 60 mg/m2/day (2 mg/kg/day) in two divided doses until the urine protein is
negative or at a trace amount for 3 days, followed by 40 mg/m2 (1.5 mg/kg/day) as a single dose
every other day for 4 weeks.

The single best predictor of long-term prognosis for NS is the initial response to prednisone
therapy. Children who experience a relapse on tapering doses of prednisone or within 2 weeks of
stopping prednisone therapy are categorized as having steroid-dependent NS. Children who do
not respond to an initial course of high-dose prednisone are defined as having steroid-resistant
NS. Children with steroid-resistant NS require a renal biopsy to evaluate renal pathology for
focal segmental glomerulosclerosis, which has a poor prognosis compared with minimal change
disease. Alternative therapies in steroid-dependent and steroid-resistant NS include
mycophenolate mofetil, cyclophosphamide, cyclosporine, tacrolimus, and rituximab.
Angiotensin-converting enzyme inhibitors (enalapril) and angiotensin II receptor blockers
(losartan) are used as supplemental therapy to reduce persistent proteinuria in patients with
steroid-resistant NS and are not indicated for the child in this vignette, who is experiencing an
initial episode of NS.

Children with NS may exhibit mild to severe edema. Furosemide alone or in combination with
intravenous albumin (used to restore low intravascular volume) may be required in children with
generalized anasarca, fluid overload, pleural effusion, ascites, and scrotal edema. Because the
child in the vignette has only periorbital swelling and 2+ pitting edema of the extremities,
intravenous furosemide is not indicated.

The total calcium levels are low in NS because of low serum albumin, but the ionized calcium
levels are normal. The total serum calcium falls by 0.8 mg/dL for every 1 g/dL fall in serum
albumin, as shown by the following formula:
Corrected calcium = Measured calcium (mg/dL) + [0.8 × {4-serum albumin (g/dL)}]
The corrected calcium for the child in this vignette is 8.9 mg/dL and, thus, intravenous calcium
gluconate is not required.

American Academy of Pediatrics 524


PREP ® Self-Assessment PREPSA 2021
PREP Pearls
• Idiopathic nephrotic syndrome is characterized by edema, proteinuria, hypoalbuminemia,
and hyperlipidemia.
• Minimal change disease is the most common histology in children with idiopathic
nephrotic syndrome.
• Prednisone or prednisolone is the first-line treatment for idiopathic nephrotic syndrome.

ABP Content Specifications(s)


• Plan the appropriate initial management of the first episode of minimal-change nephrotic
syndrome
• Recognize the clinical and laboratory findings associated with minimal-change nephrotic
syndrome
• Understand the natural history of minimal-change nephrotic syndrome

Suggested Readings
• Andolino TP, Reid-Adam J. Nephrotic syndrome. Pediatr Rev. 2015;36(3):117-125.
doi:10.1542/pir.36-3-117.
• Varade WS. Nephrotic syndrome. In: McInerny TK, Adam HM, Campbell DE, DeWitt
TG, Foy JM, Kamat DM, eds. American Academy of Pediatrics Textbook of Pediatric
Care. 2nd ed. Elk Grove Village, IL: American Academy of Pediatrics; 2017:2368-
2374. Pediatric Care Online.

American Academy of Pediatrics 525


PREP ® Self-Assessment PREPSA 2021
Question 142
A 17-year-old adolescent is seen for a health supervision visit. She is planning a trip to Florida
for spring break and was advised by a friend to start using a tanning bed weekly to prepare for
the trip. She asks about the best way to decrease the likelihood that she will experience a sunburn
in Florida.

Of the following, the MOST appropriate advice is that she should


A. apply a lotion containing aloe after sun exposure
B. apply a sunblock containing SPF-30 or higher before sun exposure
C. tan in a tanning bed before the trip
D. use a self-tanning spray before the trip

American Academy of Pediatrics 526


PREP ® Self-Assessment PREPSA 2021
Correct Answer: B
Strategies to decrease the likelihood of sunburn include avoidance of sun exposure, use of
physical blocks (eg, SPF-containing clothing), and application of a sunblock containing SPF-30
or higher before sun exposure. Lotions containing aloe may feel good after sun exposure, but do
not decrease the likelihood of sunburn or other skin damage. Tanning bed exposure increases the
risk of skin cancer and does not prevent sunburn; in fact, individuals who use a tanning bed
before vacation have been shown to use fewer sun-protection precautions during vacation,
leading to an even higher overall risk. Self-tanning sprays do not decrease sunburns or sun-
related skin damage.

Skin cancer is the most commonly diagnosed cancer in the United States. Ultraviolet radiation
(UVR) from sunlight and tanning beds has been shown to cause skin cancer; the strength of
association is mediated by the individual’s skin type, age, amount and chronicity of sun
exposure, and use of sun-protection measures. Intermittent, intense UVR exposure is thought to
contribute to melanoma pathogenesis and cumulative UVR exposure to the development of
squamous cell carcinoma.

The American Academy of Pediatrics recommends that pediatricians incorporate advice about
UVR into health supervision visits, including recommendations against tanning beds and
adherence to UVR-protective measures including:
• Avoid sunburns
• Wear protective clothing and hats
• Seek shade
• Use extra caution near snow, water, and sand which can reflect UVR
• Apply sunscreen
• Wear sunglasses

PREP Pearls
• Ultraviolet radiation from sunlight and tanning beds causes skin cancer.
• Sun-protective recommendations include avoid sunburns; wear protective clothing and
hats; seek shade; use extra caution near snow, water, and sand which can reflect
ultraviolet radiation; apply sunscreen; wear sunglasses.

ABP Content Specifications(s)


• Understand the clinical findings and risks associated with sun damage to the skin

Suggested Readings
• Council on Environmental Health, Section on Dermatology, Balk SJ. Ultraviolet
radiation: a hazard to children and adolescents. Pediatrics. 2011;127(3):588-597.
doi: 10.1542/peds.2010-3501.
• Long M. Sun exposure. Pediatr Rev. 2017;38(9):446-447. doi: 10.1542/pir.2016-0233.
• US Department of Health and Human Services. Skin cancer reports and
publications. https://www.hhs.gov/surgeongeneral/reports-and-publications/skin-
cancer/index.html.

American Academy of Pediatrics 527


PREP ® Self-Assessment PREPSA 2021
Question 143
A 4-month-old infant with a 3-day history of rhinorrhea, cough, and increased work of breathing
is brought to the emergency department. Her medical history is remarkable for premature birth at
31 weeks’ gestation and one episode of reactive airway disease at 2 months of age that required
hospitalization. She appears ill. She has a temperature of 37.7°C, a heart rate of 158 beats/min, a
respiratory rate of 56 breaths/min, a blood pressure of 76/48 mm Hg, and an oxygen saturation of
91% on room air. Chest examination is remarkable for subcostal and intercostal retractions and
bilateral diffuse expiratory wheezes. The remainder of her physical examination findings are
normal. Laboratory data are shown:

Laboratory Test Result


White blood cell count 12,000/µL (12.0 × 109/L)
Lymphocytes 65%
Polymorphonuclear leukocytes 25%
Monocytes 10%

Direct fluorescent antibody assay of nasopharyngeal swab is negative for respiratory syncytial
virus. A chest radiograph shows peribronchial cuffing, air trapping, and patchy areas of
atelectasis.

Of the following, the MOST likely cause of this child’s illness is


A. adenovirus
B. bocavirus
C. human metapneumovirus
D. influenza virus

American Academy of Pediatrics 528


PREP ® Self-Assessment PREPSA 2021
Correct Answer: C
The ill-appearing infant described in the vignette has a history of prematurity, an antecedent
upper respiratory tract infection, tachypnea, retractions, and wheezing with radiographic
evidence of peribronchial cuffing, as well as atelectasis that is consistent with the diagnosis of
bronchiolitis caused by a community respiratory virus. Bronchiolitis is a common inflammatory
disease of the bronchioles in infants and young children younger than 2 years. Although
bronchiolitis is caused by many community respiratory viruses, respiratory syncytial virus (RSV)
and human metapneumovirus, both members of the Paramyxoviridae family, are leading causes
and can result in severe disease in preterm infants and immunocompromised hosts. For the infant
described, who has a negative polymerase chain reaction for RSV, human metapneumovirus is
the most likely diagnosis. Lower respiratory tract infection due to adenovirus results in
pneumonia and, rarely, pertussis-like syndrome, croup, and bronchiolitis. Influenza infection in
infants is characterized by a nonspecific febrile illness, including a sepsis-like picture and, rarely,
pertussis-like syndrome, croup, bronchiolitis, or pneumonia. Measles, coronavirus, and
rhinovirus can also cause bronchiolitis. Human bocavirus can cause acute respiratory infections,
but often as a copathogen with other community respiratory viruses.

Human metapneumovirus infection is an important cause of acute respiratory tract infections in


individuals of all ages. Humans represent the sole source of infection. Transmission of human
metapneumovirus occurs through direct contact and exposure to contaminated respiratory tract
secretions. Annual infections typically occur during late winter and early spring in temperate
regions. The majority of children are infected with human metapneumovirus by age 5 years and
recurrent infection is common throughout the lifespan.

Human metapneumovirus is detected in 6% to 12% of children with acute respiratory infections


in the inpatient or outpatient setting. Co-infection with RSV and other community respiratory
viruses may be noted. The incubation period of human metapneumovirus is about 3 to 5 days.
Shedding of the virus in respiratory tract secretions can last up to 2 weeks in infants, but virus
shedding may be prolonged from weeks to months in immunosuppressed hosts.
Bronchiolitis is a common manifestation of human metapneumovirus, followed by pneumonia,
exacerbations of asthma, croup, upper respiratory tract infections, and otitis media. Preterm
infants (< 32 weeks), immunosuppressed hosts, and individuals with chronic lung disease are at
risk of experiencing severe infection and mortality. Similar to influenza, human
metapneumovirus infection may be associated with invasive bacterial infection, such
as Streptococcus pneumoniae.

The laboratory diagnosis of human metapneumovirus infection is often made via reverse-
transcriptase multiplex polymerase chain reaction assays that can rapidly detect community
respiratory viruses. Rapid antigen detection methods such as immunofluorescent assays
involving monoclonal antibodies can also detect the virus in nasopharyngeal specimens, but the
utility of these assays is limited by variable sensitivity. Other diagnostic techniques, such as viral
isolation via culture and acute and convalescent serology, are rarely used except for research
purposes or in outbreaks.

Treatment for human metapneumovirus infection is supportive. Antimicrobial therapy is not


indicated for uncomplicated human metapneumovirus infection unless there is evidence of
American Academy of Pediatrics 529
PREP ® Self-Assessment PREPSA 2021
concurrent bacterial infection complicating the clinical course of hospitalized patients. Contact
precautions (in addition to standard precautions) are recommended for hospitalized patients with
human metapneumovirus infection for the duration of illness.

PREP Pearls
• Human metapneumovirus is a common etiology of upper and lower respiratory tract
infections in children and adults.
• Antimicrobial therapy is not indicated for uncomplicated human metapneumovirus
infection unless there is evidence of concurrent bacterial infection complicating the
clinical course of hospitalized patients.

ABP Content Specifications(s)


• Recognize the clinical features associated with human metapneumovirus infection
• Understand the epidemiology of human metapneumovirus infection

Suggested Readings
• American Academy of Pediatrics. Human metapneumovirus. In: Kimberlin DW, Brady
MT, Jackson MA, Long SS, eds. Red Book: 2018-2021 Report of the Committee on
Infectious Diseases. 31st ed. Elk Grove Village, IL: American Academy of Pediatrics;
2018:561-563. Red Book Online.
• Ralston SL, Lieberthal AS, Meissner HC. Clinical practice guideline: the diagnosis,
management and prevention of bronchiolitis. Pediatrics. 2014:134(5):e1474-e1502.
doi:10.1542/peds.2015-2862.
• Vinci A, Lee PJ, Krilov LR. Human metapneumovirus infection. Pediatr
Rev. 2018;39(12):623-624. doi:10.1542/pir.2017-0213.

American Academy of Pediatrics 530


PREP ® Self-Assessment PREPSA 2021
Question 144
A 2-week-old female infant is seen during rounds in the neonatal intensive care unit. The infant
was delivered at 32 weeks’ gestation because of maternal preeclampsia; she was small for
gestational age with a birthweight of 1,200 g. Her neonatal course was complicated by mild
respiratory distress requiring nasal continuous positive airway pressure for the first 4 days after
birth and unconjugated hyperbilirubinemia for which she received 3 days of phototherapy in the
first week after birth. She tolerated enteral feeding well, and was receiving fortified breast milk.
However, her mother’s milk supply has decreased and the infant was recently switched to a cow
milk–based premature formula. She is growing well and takes a combination of oral and
nasogastric feedings without difficulty. Her parents ask about using a cow milk–based formula
designed for term infants.

Of the following, the condition this neonate would be MOST at risk for, with the use of this
suggested formula, is
A. acrodermatitis enteropathica
B. cow milk protein allergy
C. hypervitaminosis A
D. osteopenia of prematurity

American Academy of Pediatrics 531


PREP ® Self-Assessment PREPSA 2021
Correct Answer: D
This preterm small-for-gestational age (SGA) neonate in the vignette, at corrected gestational
age of 36 weeks, is being fed a fortified premature formula containing higher levels of protein,
sodium, potassium, calcium, phosphorus, and iron compared with term formula. Changing to a
standard term formula could increase her risk of developing osteopenia of prematurity.

Compared with term infants, premature infants have decreased accretion of calcium and
phosphorus and therefore require increased calcium and phosphorus intake. Unfortified human
milk and standard term formulas do not meet the increased calcium and phosphorus requirements
of the preterm infant. Calcium and phosphorus deficiency can lead to osteopenia of prematurity
(also known as neonatal metabolic bone disease). Use of fortified human milk or a fortified
premature formula improves bone mineralization and therefore decreases the risk of developing
osteopenia of prematurity. Preterm formula should be used, in most cases, until a corrected
gestational age of 40 weeks.

The risk for acrodermatitis enteropathica (a congenital disorder of zinc metabolism) would not
be increased by using term infant formula in this neonate. Because the neonate is already on a
cow milk protein–containing formula, the risk for cow milk protein allergy would not increase
by changing to standard (cow milk protein-containing) term formula. Hypervitaminosis A occurs
because of excessive vitamin A intake, which would not occur in an infant taking standard term
formula alone.

PREP Pearls
• Preterm neonates have increased calcium and phosphorus requirements compared with
those delivered at term.
• Use of fortified breast milk or enriched preterm formula with increased calcium and
phosphorus can decrease the likelihood of the development of osteopenia of prematurity.

MOCA-Peds Objective
• Recognize the risk factors for nutritional insufficiency (eg, food insecurity, restricted
diets, etc.).

ABP Content Specifications(s)


• Understand the dietary mineral requirements of patients of various ages, including those
born prematurely, and the circumstances in which those requirements may change
• Know the content of various infant formulas and milk sources, the indications for their
use, and possible side effects

Suggested Readings
• Abrams A; Committee on Nutrition. Calcium and vitamin D requirements of enterally fed
preterm infants. Pediatrics. 2013;131(5):e1676-e1683. doi: 10.1542/peds.2013-0420.
• Lapillonne A, O’Connor DL, Wang D, Rigo J. Nutritional recommendations for the late-
preterm infant the preterm infant after hospital discharge. J Pediatr. 2013;162(3
suppl):S90-S100. doi: 10.1016/j.jpeds.2012.11.058.
• Moreira Al, Jacob R, Lavender L, Escaname E. Metabolic bone disease of
prematurity. NeoReviews. 2015;16(11):e631-e641. doi: 10.1542/neo.16-11-e631.
American Academy of Pediatrics 532
PREP ® Self-Assessment PREPSA 2021
Question 145
An 18-month-old girl is brought to the emergency department for evaluation of a left arm injury.
Twenty minutes before arrival, she was walking down the street holding her father’s hand when
she slipped. She never fell to the ground because her father tightened his grip on her wrist;
however, since then she has been crying and not moving her left arm. On physical examination,
she cries when approached and is holding her left arm to her side with the elbow slightly flexed.
The parents are informed that their daughter’s condition can be resolved with a quick maneuver.

Of the following, the intervention MOST likely to be successful is


A. hyperflexion
B. hyperextension
C. hyperpronation
D. hypersupination

American Academy of Pediatrics 533


PREP ® Self-Assessment PREPSA 2021
Correct Answer: C
The child in the vignette has sustained a subluxation of the radial head, colloquially known as a
“nursemaid’s elbow.” The typical mechanism for a radial head subluxation is axial traction on an
extended elbow while the forearm is pronated. This typically occurs as a parent grabs the child’s
arm to prevent him/her from falling or when a child pulls away from a parent holding the arm.
These mechanisms account for the majority of left arm radial head subluxation cases in the
young child, because that is the hand that a right-handed parent preferentially holds.

Radial head subluxation typically occurs in children between the ages of 6 months and 5 years. It
occurs when part of the annular ligament slides over the proximal portion of the radial head
(Item C145A), becomes displaced into the radiohumeral joint, and is then entrapped between the
humeral capitellum and the radius. This causes pain and a reluctance to move the affected elbow.
The child tends to hold the affected arm with the shoulder adducted, the elbow extended or
partially flexed, and the forearm pronated. The child is usually calm when held by the parent and
undisturbed, but becomes agitated with attempts at elbow manipulation. Physical examination
findings of swelling, ecchymosis, bony tenderness, or visible deformities are inconsistent with a
diagnosis of radial head subluxation and should prompt evaluation for an underlying fracture.

Item C145A: Nursemaid’s Elbow.


Reprinted with permission from Strahlman RS. Fractures and dislocations. In: McInerny TK,
Adam HM, Campbell DE, DeWitt TG, Foy JM, Kamat DM, eds. American Academy of
Pediatrics Textbook of Pediatric Care. 2nd ed. Elk Grove Village, IL: American Academy of
Pediatrics; 2016.

If the history provided by the caregiver is consistent with that of the classic mechanism of
sustaining radial head subluxation, the diagnosis can be made clinically and there is no need for
imaging before an attempted reduction. If the mechanism of injury is unclear, or if there was

American Academy of Pediatrics 534


PREP ® Self-Assessment PREPSA 2021
direct trauma, plain radiography of the elbow can be performed to evaluate for an underlying
fracture. Radiographic findings will typically be normal with radial head subluxation.

Radial head subluxation can be reduced by 1 of 2 mechanisms. In the hyperpronation method


(Item C145B), the provider stabilizes the child’s forearm by grasping the affected elbow, then
hyperpronates the forearm with the other hand. In the supination/flexion method (Item C145C),
the elbow is similarly stabilized while gentle traction is placed on the forearm. The provider then
fully supinates the forearm and flexes the elbow in 1 fluid movement. A click may be palpated at
the elbow with a successful reduction using either method. There is weak evidence to suggest
that the hyperpronation method has a higher first-attempt success rate. Either method of radial
head subluxation reduction is painful and children may be reluctant to immediately use the
reduced joint. However, most children will begin to use their affected arm within 10 minutes
after reduction. Once radial head subluxation is reduced, there is no need for a splint, activity
restriction, or clinical follow-up.

Item C145B: Radial head subluxation by the hyperpronation method.


Reprinted with permission from Meckler GD, Spiro DM. Technical tip: Radial head subluxation.
Pediatr Rev. 2008;29(7):e42–e43

Item C145C: Radial head subluxation by the Supination/Flexion method.


Reprinted with permission from Meckler GD, Spiro DM. Technical tip: Radial head
subluxation. Pediatr Rev. 2008;29(7):e42–e43

If the reduction is initially unsuccessful, the provider can reattempt with the other mechanism.
After a second unsuccessful reduction, plain radiography should be performed to ensure there is
no underlying fracture. If the radiographs are normal and the subluxation cannot be reduced, the
child’s arm should be placed in a sling and instructed to follow up with the primary care provider
in several days. If movement is still decreased at follow-up, the child should be referred to a
pediatric orthopedic surgeon.

PREP Pearls
• Radial head subluxation is a clinical diagnosis; radiography is not warranted if the
mechanism of injury is consistent with the diagnosis and there are no findings of
swelling, ecchymosis, bony tenderness, or deformity.
• Reduction of radial head subluxation can be achieved via either hyperpronation of the
forearm or supination of the forearm and flexion of the elbow.

ABP Content Specifications(s)


• Recognize the clinical findings associated with subluxation of the radial head, and
manage appropriately

American Academy of Pediatrics 535


PREP ® Self-Assessment PREPSA 2021
Suggested Readings
• Browner EA. Nursemaid's elbow (annular ligament displacement). Pediatr
Rev. 2013;34(8):366-367. doi: 10.1542/pir.34-8-366.
• Krul M, van der Wouden JC, Kruithof EJ, van Suijlekom-Smit LW, Koes BW.
Manipulative interventions for reducing pulled elbow in young children. Cochrane
Database Syst Rev. 2017;7(7):CD007759. doi: 10.1002/14651858.CD007759.pub4.
• Meckler GD, Spiro DM. Technical tip: radial head subluxation. Pediatr
Rev. 2008;29(7):e42-e43. doi: 10.1542/pir.29-7-e42.
• Strahlman RS. Fractures and dislocations. In: McInerny TK, Adam HM, Campbell DE,
DeWitt TG, Foy JM, Kamat DM, eds. American Academy of Pediatrics Textbook of
Pediatric Care. 2nd ed. Elk Grove Village, IL: American Academy of Pediatrics;
2016:1952-1962. Pediatric Care Online.

American Academy of Pediatrics 536


PREP ® Self-Assessment PREPSA 2021
Question 146
A 2-hour-old male neonate is being seen in the well-child nursery for feeding difficulties. He was
born via normal spontaneous vaginal delivery at 40 weeks’ gestation to a primigravida mother
who has been healthy and had an uncomplicated pregnancy. His mother is reporting difficulty
with latching and notes that his tongue seems to be anchored to the bottom of his mouth.

Of the following, the BEST management approach for this infant-mother dyad is to
A. offer lactation support
B. perform a frenulotomy
C. provide a breast pump
D. provide donor breast milk

American Academy of Pediatrics 537


PREP ® Self-Assessment PREPSA 2021
Correct Answer: A
The infant-mother dyad in the vignette should be offered lactation support to enforce good
breastfeeding habits as early as possible. Performing a frenulotomy is inadvisable at this time
because other noninvasive techniques have not been attempted. Providing a breast pump or
donor milk are also inappropriate; the infant should be given the opportunity to breastfeed
directly from his mother’s breast as often as possible.

The terms “ankyloglossia” and “tongue-tie” most commonly refer to a short anterior lingual
frenulum that limits tongue movement. Prevalence rates range from 4% to 10%, depending on
the definition used.

Very little evidence exists to inform patients and medical providers as to whether or not
ankyloglossia in healthy children should be corrected at any age. This is especially true in older
children, for whom speech problems or social problems such as licking ice cream must be
considered. Most available studies are focused on breastfeeding outcomes, but the quality of
these studies are at best mediocre. Any decision to perform a frenulotomy in a healthy neonate or
infant should be made in conjunction with the patient’s parents, primary care physician, lactation
consultant, and other specialists. The procedure should be performed by a trained provider;
simple frenulotomies are preferred over laser therapy or more complex interventions.

Release of posterior tongue-tie or lip tie in healthy infants have also become popular. Evidence
regarding benefits are even more lacking. Open and nonjudgmental communication between the
parent(s) and provider is key in all such discussions.

PREP Pearls
• For feeding problems in healthy breastfeeding infant-mother dyads, lactation support
should be offered before any other intervention.
• Evidence for procedural correction of shortened oral frenulum in healthy neonates,
infants, and children is lacking; the decision to perform such a procedure should be made
with open communication among the patient’s parents, primary care physician, and
specialists.

ABP Content Specifications(s)


• Plan the most appropriate management of a short lingual frenulum

American Academy of Pediatrics 538


PREP ® Self-Assessment PREPSA 2021
Suggested Readings
• Bunik M. The pediatrician’s role in encouraging exclusive breastfeeding. Pediatr Rev.
2017;38(8):353-368. doi: 10.1542/pir.2016-0109.
• Chinnadurai S, Francis DO, Epstein RA, Morad A, Kohanim S, McPheeters M.
Treatment of ankyloglossia for reasons other than breastfeeding: a systematic
review. Pediatrics. 2015;135(6):e1467-e1474. doi: 10.1542/peds.2015-0660.
• Francis DO, Krishnaswami S, McPheeters M. Treatment of ankyloglossia and
breastfeeding outcomes: a systematic review. Pediatrics. 2015;135(6):1458-e1466.
doi: 10.1542/peds.2015-0658.
• Younger Meek J, Hatcher AJ; Section on Breastfeeding. The breastfeeding-friendly
pediatric office practice. Pediatrics. 2017;139(5):e20170647. doi: 10.1542/peds.2017-
0647.

American Academy of Pediatrics 539


PREP ® Self-Assessment PREPSA 2021
Question 147
A 7-year-old girl is seen for a health supervision visit. Her mother is concerned that she is the
smallest child in her class. The girl’s medical history is significant for placement of ear tubes at
age 2 years because of multiple episodes of acute otitis media. She takes no medication. She is
generally doing well in the second grade but requires extra help in math. A comprehensive
review of systems is otherwise unremarkable. Her adjusted midparental height is 164 cm (50th
percentile for an adult female), and pubertal timing for both parents was normal. Vital signs are
normal for age. Her growth chart is shown in Item Q147. Her body mass index is 15.4
kg/m2 (50th percentile). The physical examination findings are unremarkable. A bone age
radiograph is read as concordant with her chronological age.

American Academy of Pediatrics 540


PREP ® Self-Assessment PREPSA 2021

American Academy of Pediatrics 541


PREP ® Self-Assessment PREPSA 2021
Of the following, the test MOST likely to lead to this girl’s diagnosis is
A. insulin-like growth factor-1
B. a karyotype
C. thyroid-stimulating hormone
D. tissue transglutaminase IgA antibody

American Academy of Pediatrics 542


PREP ® Self-Assessment PREPSA 2021
Correct Answer: B
The girl described in the vignette has Turner syndrome. The test most likely to lead to this girl’s
diagnosis is a karyotype. Turner syndrome results from a missing or structurally abnormal sex
chromosome in a phenotypic female. About one-half of girls with Turner syndrome have a 45,X
karyotype. The other half have a karyotype with a portion of an X chromosome missing or
rearranged, or a karyotype that contains mosaicism (eg, 45,X/46,XX). Turner syndrome is
relatively common, occurring in one in 2,500 live female births.

Features of Turner syndrome displayed by the girl in the vignette include her short stature with
declining growth velocity, recurrent otitis media, and difficulty with math. Other common
features include ptosis; epicanthal folds; low-set, prominent ears; high-arched palate; neck
webbing; low posterior hairline; broad chest with wide-spaced nipples; multiple nevi; congenital
heart disease (eg, coarctation of the aorta); renal anomalies (eg, horseshoe kidney); skeletal
anomalies (scoliosis, shortened 4th metacarpals, high upper-to-lower segment ratio); and primary
ovarian failure. However, these features may not be present or may not be recognized. Short
stature is the most common manifestation of Turner syndrome and may be the only presenting
feature. Turner syndrome should be considered in all girls with short stature.

Not only is this girl’s height curve consistent with Turner syndrome, but also her normal body
mass index (50th percentile) and concordant bone age. Her normal body mass index makes a
nutritional disorder, gastrointestinal disorder, or chronic systemic disease less likely. Her
concordant bone age makes these disorders as well as an endocrine disorder unlikely. A growth
curve with declining height velocity and normal body mass index, along with a concordant bone
age, is most consistent with a genetic syndrome or disorder.

Declining growth velocity can be consistent with an endocrine disorder such as growth hormone
deficiency or hypothyroidism. Bone age, however, would be delayed in these endocrine
disorders. For nutritional disorders, gastrointestinal disorders, and other chronic systemic
disease, weight is usually affected before height or more than height (so that body mass index is
relatively low), and bone age would be expected to be delayed.

An insulin-like growth factor-1 level to evaluate for growth hormone deficiency would be
normal for the girl in the vignette. Girls with Turner syndrome have an increased incidence of
autoimmune hypothyroidism, but hypothyroidism would not explain the growth pattern for the
girl in the vignette. Thus, a thyroid-stimulating hormone level test would not be the most likely
test to reveal the diagnosis. Similarly, girls with Turner syndrome have a higher incidence of
celiac disease. However, celiac disease would not explain the growth pattern for the girl in the
vignette. Thus, a tissue transglutaminase IgA antibody test would not be the test most likely to
reveal the diagnosis.

Available guidelines provide screening recommendations at the time of diagnosis and throughout
life for those with Turner syndrome.

American Academy of Pediatrics 543


PREP ® Self-Assessment PREPSA 2021
PREP Pearls
• Short stature is the most common manifestation of Turner syndrome and may be the only
presenting feature.
• Normal versus abnormal height velocity, body mass index, and bone age are important
factors to consider in narrowing the differential diagnosis of short stature.

MOCA-Peds Objective
• Recognize the genetic syndromes that may present with abnormal growth.

ABP Content Specifications(s)


• Identify the most common causes of short stature
• Plan the evaluation of a child with short stature or whose height percentiles have
decreased

Suggested Readings
• Braun LR, Marino R. Disorders of growth and stature. Pediatr Rev. 2017;38(7):293-304.
doi:10.1542/pir.2016-0178.
• Gravholt CH, Andersen NH, Conway GS, et al; International Turner Syndrome
Consensus Group. Clinical practice guidelines for the care of girls and women with
Turner syndrome: proceedings from the 2016 Cincinnati International Turner Syndrome
Meeting. Eur J Endocrinol. 2017;177(3):G1-G70. doi:10.1530/EJE-17-0430.
• Kaplowitz P. Short stature. In: McInerny TK, Adam HM, Campbell DE, DeWitt TG, Foy
JM, Kamat DM, eds. American Academy of Pediatrics Textbook of Pediatric Care. 2nd
ed. Elk Grove Village, IL: American Academy of Pediatrics; 2017:1585-1588. Pediatric
Care Online.
• Milbrandt T, Thomas E. Turner syndrome. Pediatr Rev. 2013;34(9):420-421.
doi:10.1542/pir.34-9-420.
• Rose SR, Vogiatzi MG, Copeland KC. A general pediatric approach to evaluating a short
child. Pediatr Rev. 2005;26(11):410-420. doi:10.1542/pir.26-11-410.

American Academy of Pediatrics 544


PREP ® Self-Assessment PREPSA 2021
Question 148
The mother of a girl who is 6 years and 10 months old expresses concerns about her daughter’s
poor school performance. The girl started kindergarten in the fall. She was hesitant to engage in
class activities and was teased by her classmates for her small size. In comparison with her
classmates, she required additional guidance and support from her teachers to understand class
routines and class lessons. After a few weeks, she was moved to a transitional kindergarten class,
where the girl adjusted well and made several friends. Her teacher recently informed the girl’s
mother that she is performing at a level appropriate for the class. Her mother is concerned about
why her daughter was not successful in kindergarten and would like additional guidance.
The girl has no significant medical history. She walked at age 12 months, spoke single words at
age 11 months, and spoke in 2-word phrases/sentences at age 2 years. Her height and weight are
at the 10th percentile. During the visit, the girl initially stays close to her mother and is quiet, but
eventually engages in a brief conversation about the toy unicorn she has brought. Physical
examination findings are within normal limits; vision and hearing screening results are normal.

Of the following, the MOST appropriate next step is to recommend


A. cognitive behavioral therapy
B. genetic testing
C. psychoeducational evaluation
D. reassessment in the fall

American Academy of Pediatrics 545


PREP ® Self-Assessment PREPSA 2021
Correct Answer: C
The girl in the vignette is performing at the level of her classmates who are more than a year
younger. Although her slow-to-warm temperament and small size could have affected her social
adjustment, these factors do not explain this girl’s difficulty understanding class lessons or
routines. Her history is suspicious for a possible learning disability or mild intellectual disability.
A psychoeducational assessment is the most appropriate next step to determine the reason for
this child’s failure to succeed in an age-appropriate classroom setting and to determine her
special educational needs.

Difficulties in school may arise from child-specific factors, environmental factors, and social
factors. A child may have problems with his/her physical health (eg, acute or chronic illness,
vision or hearing impairment, pain, medication side effect) and/or mental health (eg, anxiety,
depression). A child may have a learning disability, attention-deficit/hyperactivity disorder, or
developmental disorder (eg, autism) that interferes with learning ability and school performance.
Sleep problems and substance use can negatively affect alertness and cognition. A child’s
temperament may affect his/her participation in school activities; a slow-to-warm child may be
reluctant to engage, whereas a difficult child may be easily frustrated. Environmental factors that
may affect learning include the child’s fit with teachers, teacher use of positive reinforcement,
and school expectations for academic achievement. Family dysfunction, parental discord or
divorce, family mental or physical health problems, violence (eg, abuse, bullying), and housing
or food insecurity can also contribute to a child’s academic underachievement.

When a child is performing poorly in school, various factors should be considered and any issues
relevant for the child identified and addressed. The primary care physician should evaluate for
health problems including sensory impairments (ie, vision or hearing impairment), screening for
developmental and mental health conditions, and seek information about social factors that could
be contributing to school failure. Parents should be guided to submit a written request to the
child’s school for a psychoeducational evaluation. This evaluation should assess the child’s
intelligence, achievement, and social-emotional functioning via review of academic
history/records; classroom observation; psychological evaluation; and student, parent, and
teacher interviews. This psychoeducational evaluation can help identify the reasons for a child’s
academic underachievement and determine eligibility for special education services to provide
for the child’s educational needs.

If the girl in the vignette were identified as having anxiety, she could benefit from cognitive
behavioral therapy. This could help with her engagement in class, but would not address her
difficulties with understanding what is being taught. Genetic testing would be indicated if the girl
were identified through evaluation to have mild intellectual disability. Genetic testing typically is
not indicated for a learning disability unless there is intellectual disability, dysmorphic findings
on physical examination, or family or clinical history suggestive of a particular condition.
Because this girl’s history is concerning for possible learning disability or mild intellectual
disability, waiting until the fall to assess her educational performance would delay identification
of the reason for her learning difficulties and her access to needed educational supports and
services.

American Academy of Pediatrics 546


PREP ® Self-Assessment PREPSA 2021
PREP Pearls
• Difficulties in school may arise from child-specific factors (eg, problems with physical
and/or mental health, learning disability, developmental disorder, temperament),
environmental factors (eg, fit with teachers, school expectations), and social factors (eg,
family dysfunction, family mental or physical health problems, violence).
• When a child is performing poorly in school, the primary care physician should evaluate
for health problems including sensory impairments, screen for developmental and mental
health conditions, and seek information about social factors that could be contributing to
school failure.
• When a child is performing poorly in school, parents should be guided to submit a written
request to the child’s school for a psychoeducational evaluation.

MOCA-Peds Objective
• Recognize the effect of parental chronic illness or disability on child health.

ABP Content Specifications(s)


• Plan the appropriate diagnostic evaluation of poor school performance
• Understand the various etiologies of school-related difficulties
• Recognize that factors such as temperament, family environment, illness, medications,
and mental disorders contribute to academic underachievement

Suggested Readings
• Augustyn M, Wolf M. Learning disability. In: Augustyn M, Zuckerman B,
eds. Zuckerman Parker Handbook of Developmental and Behavioral Pediatrics for
Primary Care. 4th ed. Philadelphia, PA: Wolters Kluwer; 2019:317-321.
• Byrd RS. School failure: assessment, intervention, and prevention in primary pediatric
care. Pediatr Rev. 2005;26(7):233-243. doi: 10.1542/pir.26-7-233.
• Fogler JM, Barbaresi WJ. Learning Disabilities. In: Voigt RG, Macias MM, Myers SM,
Tapia CD, eds. Developmental and Behavioral Pediatrics. 2nd ed. Itasca, IL: American
Academy of Pediatrics; 2018:367-381.

American Academy of Pediatrics 547


PREP ® Self-Assessment PREPSA 2021
Question 149
A female newborn is evaluated in the neonatal intensive care unit. The pregnancy was
complicated by intrauterine growth retardation. Prenatal ultrasonography showed a single kidney
on the right side and vertebral segmentation defects throughout the thoracic and lumbar spine. A
few minutes after birth, the neonate experienced respiratory distress that necessitated intubation.
She has anal atresia and no right thumb. A nasogastric tube curls up in the esophagus, and
radiographs suggest a tracheoesophageal fistula and show the vertebral defects.

Of the following, the additional birth defect(s) MOST likely found in this neonate is (are)
A. cleft palate
B. iris and retinal colobomas
C. small ears and congenital hearing loss
D. ventricular and atrial septal defects

American Academy of Pediatrics 548


PREP ® Self-Assessment PREPSA 2021
Correct Answer: D
The neonate in the vignette has clinical features consistent with VATER/VACTERL association.
Genetic association is defined as the nonrandom occurrence of two or more congenital
malformations that occur together more often than can be explained by chance. The term
“VATER association” was coined in 1972, encompassing vertebral defects (V), anal atresia (A),
tracheoesophageal fistula (TE), and renal defects (R). Over the next 2 years, researchers
proposed expanding the abbreviation from VATER to VACTERL, adding cardiac defects (C)
and limb defects (L). At least three of the above-mentioned congenital malformations must be
present to arrive at a diagnosis of VACTERL association. The neonate in the vignette has
vertebral defects, anal atresia, tracheoesophageal fistula, renal aplasia (single right kidney), and
limb defect (absent thumb). Cardiac malformation, most commonly ventricular and atrial septal
defects, can be seen in VACTERL association.

VACTERL association is a diagnosis of exclusion. Common disorders in the differential


diagnosis include Alagille syndrome (vertebral, cardiac, and renal anomalies; bile duct paucity
and cholestasis; and eye anomalies such as posterior embryotoxon), CHARGE syndrome
(coloboma, heart defects, atresia choanae [choanal atresia], growth retardation, genital
abnormalities, and ear abnormalities), 22q11.2 deletion syndrome (see description below),
Townes-Brocks syndrome (see description below), and Fanconi anemia (in which virtually all
features of VACTERL association can occur but with hematologic and pigmentation
abnormalities that differentiate it from VACTERL). The diagnostic approach in suspected
patients is to rule out syndromic etiologies via thorough examination, imaging to detect other
associated anomalies, and genetic analysis.

The underlying genetic etiology of an association is not known. CHARGE, which was described
as an association in the past, constituted colobomas (C), heart defects (H), atresia choanae (A),
retardation (R) either physical or mental, genital anomalies (G), and ear anomalies (E).
CHARGE is now designated as a syndrome; approximately 50% of cases are known to be caused
by pathogenic variants in the CHD7 gene.

Cleft palate would be an incorrect answer for the child in the vignette, because craniofacial
anomalies are not part of VACTERL association. Cleft palate can be seen in many syndromes,
the most common being velocardiofacial syndrome, also called DiGeorge, or 22q11.2 deletion
syndrome. The 22q11.2 deletion syndrome is characterized by distinctive facial features (high
and broad nasal bridge, hooded eyelids, hypertelorism, and asymmetric crying facies), congenital
heart disease (most commonly, conotruncal malformations such as tetralogy of Fallot), immune
deficiency, abnormalities of the palate (most commonly, velopharyngeal incompetence,
submucosal cleft palate, bifid uvula, cleft palate), hypocalcemia, and renal anomalies. Iris and
retinal colobomas are seen in CHARGE syndrome and not in VACTERL association, as
described previously. Small ears and congenital hearing loss would be incorrect response
choices, because hearing deficit is not a feature of VACTERL. It can be seen in Townes-Brocks
syndrome, which is characterized by imperforate anus, thumb anomalies, renal anomalies, and
cardiac anomalies, in addition to dysplastic ears and hearing loss. Tracheoesophageal fistula is
not a feature of Townes-Brocks syndrome.

American Academy of Pediatrics 549


PREP ® Self-Assessment PREPSA 2021
PREP Pearls
• VACTERL association, formerly known as VATER association, stands for vertebral
defects (V), anal atresia (A), cardiac defects (C), tracheoesophageal fistula (TE), renal
defects (R), and limb defects (L).
• Diagnosis of VACTERL is established when at least three of these features are present.
• VACTERL association is a diagnosis of exclusion.

ABP Content Specifications(s)


• Recognize the clinical features associated with genetic associations (eg, VACTERL,
VATER)

Suggested Readings
• Shaw-Smith C. Oesophageal atresia, tracheo-esophageal fistula, and the VACTERL
association: review of genetics and epidemiology. J Med Genet. 2006;43(7):545-554.
doi:10.1136/jmg.2005.038158.
• Solomon BD. VACTERL/VATER Association. Orphanet J Rare Dis. 2011;6:56.
doi:10.1186/1750-1172-6-56.

American Academy of Pediatrics 550


PREP ® Self-Assessment PREPSA 2021
Question 150
A 15-year-old female soccer player is being evaluated for a 1-year history of progressive left
groin pain. She denies trauma or change in activity before the onset of symptoms. The pain is
primarily anterior, but at times radiates into the lateral hip as well. Symptoms worsen with
physical activity, particularly running and jumping. She has not been able to participate in soccer
training for the past week because of the pain. She denies swelling, change in bowel or bladder
function, or any distal neurovascular complaints and is otherwise healthy, with normal growth
and development.

On physical examination, the adolescent is in no acute distress. Her gait is antalgic; abdominal
and genitourinary examinations are normal; lumbar spine has free range of motion; and her lower
extremity neurovascular examination findings are normal. There is mild tenderness on
examination of the left anterior hip and groin, but no swelling or inguinal lymphadenopathy is
appreciated. Flexion and internal rotation of the left hip are painful and limited compared with
the right. Examination of hip internal rotation is shown in Item Q150 .

Item Q150: Hip examination: internal rotation. Courtesy of M LaBotz

Of the following, the BEST next management step for this adolescent is to
A. obtain a complete blood cell count, C-reactive protein level, and erythrocyte
sedimentation rate
B. perform magnetic resonance imaging of the left hip
C. perform standing radiography of the pelvis and left hip
D. prescribe a nonsteroidal anti-inflammatory medication and physical therapy

American Academy of Pediatrics 551


PREP ® Self-Assessment PREPSA 2021
Correct Answer: C
The history and physical examination findings for the adolescent in this vignette suggest
interarticular hip pathology. Weight-bearing radiography is the best initial step to assess the bony
anatomy of the hip. If radiography is not diagnostic, advanced imaging should be considered.

Atraumatic groin pain in children and adolescents most commonly occurs because of
femoroacetabular joint pathology. The differential diagnosis is broad and age- and activity-
dependent. For this adolescent athlete with longstanding, progressive hip pain, potential
diagnoses include femoroacetabular impingement (FAI), acetabular labrum tear, femoral neck
stress fracture, and hip flexor tendinitis and/or bursitis. The restricted range of motion noted on
her physical examination suggests that there is some form of mechanical restriction within the
joint, which makes FAI or labral tear most likely. Children and adolescents with FAI or labral
tears often describe the pain as “deep” anterior groin pain, or with the “C” sign as shown in Item
C150A. Children and adolescents with labral tears may report episodes of the hip” locking up”
or “getting stuck.”

American Academy of Pediatrics 552


PREP ® Self-Assessment PREPSA 2021

Item C150A: “C” sign commonly used by patients to describe pain in femoroacetabular
impingement. Reprinted with permission from Sheppard ED, Read CR, Wills BW, Estes AR.
Femoroacetabular Impingement in Pediatric Patients. Pediatr Rev. 2019,40(3):131.

Femoroacetabular impingement occurs when there is abnormal contact between the femur and
the acetabulum, which is classified as a combination of cam and pincer mechanisms. Cam
impingement (Item C150B-B and Item C150B-C) results from a bony prominence of the
femoral head and neck that “bumps up” against the acetabular rim and labrum, particularly with
hip flexion. In young athletes, this results from repetitive stress across the proximal femoral
growth plate, and is most commonly seen in athletes and others who participate in vigorous
activities with repeated hip flexion.

American Academy of Pediatrics 553


PREP ® Self-Assessment PREPSA 2021

Item C150B: A. Normal hip anatomy. B. CAM deformity. C. Engagement of the CAM lesion. D.
Pincer acetabular lesion. E. Pincer lesion impinging on the femoral neck. Reprinted with
permission from Sheppard ED, Read CR, Wills BW, Estes AR. Femoroacetabular Impingement
in Pediatric Patients. Pediatr Rev. 2019,40(3):130.

Pincer impingement (Item C150B-D and Item C150B-E) results from an “overcoverage” of the
femoral head in the acetabular socket, which creates a mechanical block with hip movement and
increases the risk of damage to the labrum. These patients often report a longstanding lack of
bilateral hip flexibility. The “overcovered” hip is essentially the opposite end of the spectrum
from hip dysplasia, while the “undercovered” hip is relatively unstable. Although hip dysplasia is
generally detected during infancy or the neonatal period, cases of borderline hip dysplasia (BHD)
can present in adolescents and young adults with similar symptoms as FAI. Borderline hip
dysplasia is identified on radiography. Joint instability in BHD leads to compensatory
hypertrophy of the cartilaginous labrum. Patients with cam or pincer impingement of the hip, or
with BHD, are at increased risk for labral tears and accelerated degenerative change.

Laboratory evaluation would be indicated to assess for a possible infectious or rheumatologic


process in the hip. This would be appropriate if the girl’s symptoms included systemic or
multijoint complaints, or if advanced imaging indicates an interarticular effusion without other
pathology. However, laboratory assessment is not the best next step in this case.
Magnetic resonance imaging can assist in the diagnosis of a stress fracture or muscle/tendon
pathology, but the sensitivity for pathology of the cartilaginous labrum is poor. When FAI or a
labral tear is suspected, magnetic resonance arthrography is preferred for better delineation of
cartilage anatomy and possible tear, and would be the best choice for this patient if initial
radiography is negative.
American Academy of Pediatrics 554
PREP ® Self-Assessment PREPSA 2021
The degree of restriction of internal rotation in this hip strongly suggests interarticular pathology.
Diagnostic evaluation is needed before physical therapy or other treatment options are
considered. Physical therapy is the initial treatment for most cases of FAI. Children who fail to
improve with physical therapy or those with large cam lesions, BHD, or labral tears on imaging
studies should be referred to an orthopedic surgeon for possible surgery. A brief course of
nonsteroidal anti-inflammatory medication may provide symptom relief, and may increase
comfort when physical therapy is initiated, or while awaiting additional evaluation. However,
these medications should not be used over the long term and are not a substitute for appropriate
rehabilitation or other treatment measures.

PREP Pearls
• Groin pain suggests interarticular pathology and warrants radiographic evaluation before
considering treatment options.
• The acetabular labrum is not well-visualized with magnetic resonance imaging and
requires arthrography to rule out possible tears.
• Cam and pincer impingement and borderline hip dysplasia may lead to early
osteoarthritis of the hip.

MOCA-Peds Objective
• Recognize common causes of joint pain in children and adolescents.

ABP Content Specifications(s)


• Recognize the clinical findings associated with developmental dysplasia/subluxation of
the hip
• Plan the appropriate diagnostic evaluation of developmental dysplasia/subluxation of the
hip in patients of various ages

Suggested Readings
• Pennock AT, Bomar JD, Johnson KP, Randich K, Upasani VV. Nonoperative
management of femoroacetabular impingement: a prospective study. Am J Sports
Med. 2018:46(14):3415-3422. doi: 10.1177/0363546518804805.
• Sheppard ED, Read CR, Wills BW, Estes AR. Femoroacetabular impingement in
pediatric patients. Pediatr Rev. 2019,40(3):129-137. doi: 10.1542/pir.2017-0137.

American Academy of Pediatrics 555


PREP ® Self-Assessment PREPSA 2021
Question 151
A 38-month-old boy is brought to clinic in July to establish care. He emigrated from El Salvador
3 months ago. He has no significant medical history, allergies, or medications. He and his mother
were detained at the border in a family shelter while awaiting reunification with family members.
He underwent basic medical screening, including tuberculosis screening, which had negative
results, and received vaccinations. His mother brings his vaccine record from El Salvador,
indicating that he received a combination vaccine of hepatitis B, DTaP, and inactivated polio at
2, 4, and 6 months of age. Two months ago at the border shelter, he received Haemophilus
influenzae type B, DTaP, MMR, pneumococcal conjugate, and varicella vaccines.

In addition to hepatitis A, the most appropriate vaccine(s) for this boy at this time is (are)
A. DTaP
B. Haemophilus influenzae type B and pneumococcal conjugate
C. Haemophilus influenzae type B, pneumococcal conjugate, and DTaP
D. none

American Academy of Pediatrics 556


PREP ® Self-Assessment PREPSA 2021
Correct Answer: D
The 38 month-old boy in the vignette is up-to-date on his vaccines and, with the exception of
hepatitis A vaccine, is not currently due for any more. He was on the catch-up immunization
schedule because he was more than 1 month behind on his vaccines when he immigrated to the
United States, having only received his 2-month, 4-month, and 6-month vaccines. At the border
family shelter, he received his 12-month vaccines (MMR and varicella) and his 15-month
vaccines (DTaP, pneumococcal conjugate, and Haemophilus influenzae type B). He is also due
to receive the influenza vaccine, which usually expires at the end of June and typically is not
available until the fall. Aside from the annual influenza vaccine and the second hepatitis A
vaccine in 6 months, he will not be due for additional vaccines (DTaP, inactivated polio vaccine,
MMR, varicella) until he turns 4 years old (updated catch-up schedules can be found at
the Centers for Disease Control and Prevention. Recommended child and adolescent
immunization schedule for ages 18 years or younger.)

PREP Pearls
• Children who are more than 1 month behind on vaccinations will need catch-up
vaccinations per the Centers for Disease Control and Prevention immunization schedule.
• Hepatitis A vaccine is typically given at 1 year of age, with a second dose 6 months after.

MOCA-Peds Objective
• Evaluate an internationally adopted child for infectious diseases and appropriate
immunizations.

ABP Content Specifications(s)


• Plan an immunization schedule for a child or adolescent who begins receiving
immunizations late or whose immunizations are delayed

Suggested Readings
• Centers for Disease Control and Prevention. Recommended child and adolescent
immunization schedule for ages 18 years or younger, United States,
2020. https://www.cdc.gov/vaccines/schedules/hcp/imz/child-adolescent.html.
• Humiston SG, Atkinson WL, Rand C, Szilagyi PG. Immunizations. In: McInerny TK,
Adam HM, Campbell DE, DeWitt TG, Foy JM, Kamat DM, eds. American Academy of
Pediatrics Textbook of Pediatric Care. 2nd ed. Elk Grove Village, IL: American
Academy of Pediatrics; 2017:160-167. Pediatric Care Online .
• Linton JM, Green A; Council on Community Pediatrics. Providing care for children in
immigrant families. Pediatrics. 2019;144(3):e20192077. doi:10.1542/peds.2019-2077.

American Academy of Pediatrics 557


PREP ® Self-Assessment PREPSA 2021
Question 152
A 6-week-old male infant born at 32 weeks’ gestational age has been hospitalized in the neonatal
intensive care unit since birth. He has multiple congenital anomalies including nonshunted
hydrocephalus and agenesis of the corpus callosum. He has had 4 episodes of fever. With the
first episode, results of a full evaluation for a serious bacterial infection, including cerebrospinal
fluid evaluation, were negative. With later episodes, blood cultures, urine cultures, and
respiratory viral panels have been negative.

His vital signs today include a temperature of 38.5°C, heart rate of 145 beats/min, respiratory
rate of 30 breaths/min, and blood pressure of 75/44 mm Hg. Physical examination reveals a well-
appearing infant with macrocephaly; a soft anterior fontanelle; a nasogastric tube; a II/VI systolic
murmur audible throughout the precordium, axillae, and back; a soft but protuberant abdomen
with no masses or enlargement of the liver or spleen; and warm extremities with capillary refill
of less than 3 seconds.

Of the following, the BEST next step in the evaluation of this infant includes
A. blood culture
B. lumbar puncture
C. observation
D. procalcitonin

American Academy of Pediatrics 558


PREP ® Self-Assessment PREPSA 2021
Correct Answer: C
For the infant in the vignette, the next best step in evaluation is observation. He has had 4
previous evaluations for serious bacterial infections, which did not reveal a bacterial source of
fever. In addition, he is clinically stable and well-appearing. The historical information and his
current appearance, in the context of known central nervous system abnormalities, should raise
concern for fever of central origin.

The hypothalamus is the brain’s thermoregulatory center. The posterior portion of the
hypothalamus controls heat conservation whereas the anterior portion controls heat dissipation.
Thus, lesions in the posterior hypothalamus are associated with hypothermia and conversely,
lesions in the anterior hypothalamus are associated with hyperthermia.

Shapiro syndrome is a rare disorder characterized by episodic hypothermia, hyperhidrosis, and


agenesis of the corpus callosum. Reverse Shapiro syndrome, characterized by episodic
hyperthermia in the setting of agenesis of the corpus callosum, is a rare cause of fever of
unknown origin. Though poorly understood, the absence of the corpus callosum in both Shapiro
and reverse Shapiro syndrome is believed to lead to hypothalamic dysfunction and thus altered
thermoregulation.

A blood culture and lumbar puncture should be considered in young febrile infants, depending
on their age and risk factors, to assess for an invasive bacterial infection. Procalcitonin is an
inflammatory marker that is more specific for an invasive bacterial infection than is the
peripheral white blood cell count.

PREP Pearls
• The hypothalamus is the brain’s thermoregulatory center.
• Lesions of the hypothalamus can lead to either hypothermia or hyperthermia.
• Fever of central origin is a rare cause of fever of unknown origin.

MOCA-Peds Objective
• Evaluate and manage a child with unexplained persistent fever.

ABP Content Specifications(s)


• Understand the effects of the immature/abnormal hypothalamic thermoregulatory system
on the development of fever in infants and children who have diseases of the central
nervous system

Suggested Readings
• Adam HM. Physiology and management of fever. In: McInerny TK, Adam HM,
Campbell DE, DeWitt TG, Foy JM, Kamat DM, eds. American Academy of Pediatrics
Textbook of Pediatric Care. 2nd ed. Itasca, IL: American Academy of Pediatrics;
2016;chap 53:375-378. Pediatric Care Online.
• Zawadka M, Szmuda M, Mazurkiewicz-Beldzinska M. Thermoregulation disorders of
central origin- how to diagnose and treat. Anaesthesiol Intensive Ther. 2017;49(3):227-
234. doi: 10.5603/AIT.2017.0042.

American Academy of Pediatrics 559


PREP ® Self-Assessment PREPSA 2021
Question 153
A screening program conducted at a high school is screening student athletes with
electrocardiography to identify undiagnosed heart disease. Of 1,000 students, two students are
found to have a prolonged QT interval.

Of the following, the parameter of long QT syndrome that can BEST be calculated by these data
is the
A. incidence
B. odds ratio
C. prevalence
D. relative risk

American Academy of Pediatrics 560


PREP ® Self-Assessment PREPSA 2021
Correct Answer: C
The data that are given include the number of affected cases in a specific population at a given
point in time—in other words, the prevalence. Incidence is the number of new cases that develop
over a specific period. The relative risk is an estimate of the extent of the association between an
exposure to a risk factor and a disease. It can be thought of as the likelihood of developing a
certain disease after exposure to a certain risk factor, or as the incidence of disease in the
exposed group divided by the incidence of disease in the unexposed group. The odds ratio, on the
other hand, is the probability of exposure in those with the disease compared with the probability
of exposure in those without the disease. Refer to Item C153 for an example of a 2 × 2 table. A 2
× 2 table can help elucidate the associations between an exposure and an outcome. The relative
risk can be displayed mathematically as RR = (a/ a + b)/(c/c + d). The odds ratio can be
demonstrated mathematically as OR = (a/a + b)/(b/a + b) = a/b.

Pretest probability and posttest probability are other epidemiologic terms that are important to
understand in the context of disease states. Pretest probability is the probability of disease before
a diagnostic test is ordered. Before a history is taken or a physical examination is conducted, the
pretest probability of a disease is the same as the prevalence of a disease. Posttest probability is
the likelihood of disease after the result of a diagnostic test is known.

PREP Pearls
• Prevalence is the number of affected cases in a specific population at a given point in
time.
• Incidence is the number of new cases that develop over a specific period.
• Pretest probability is the probability of disease before a diagnostic test is ordered; posttest
probability is the likelihood of disease after the result of a diagnostic test is known.

ABP Content Specifications(s)


• Understand pre-test and post-test probability
• Understand prevalence and incidence

American Academy of Pediatrics 561


PREP ® Self-Assessment PREPSA 2021
Suggested Readings
• Connor KA. Quantifying associations: understanding relative risks and odds
ratios. Pediatr Rev. 2012;33(10):473-474. doi:10.1542/pir.33-10-473.
• Kuhn JE, Greenfield ML, Wojtys EM. A statistics primer: prevalence, incidence, relative
risks, and odds ratios: some epidemiologic concepts in the sports medicine literature. Am
J Sports Med. 1997;25(3):414-416.
• Parikh R, Parikh S, Arun E, Thomas R. Likelihood ratios: clinical application in day-to-
day practice. Indian J Ophthalmol. 2009;57(3):217-221. doi:10.4103/0301-4738.49397.
• Spronk I, Korevaar JC, Poos R, et al. Calculating incidence rates and prevalence
proportions: not as simple as it seems. BMC Public Health. 2019;19(1):512.
doi:10.1186/s12889-019-6820-3.

American Academy of Pediatrics 562


PREP ® Self-Assessment PREPSA 2021
Question 154
A 1-year-old girl with high fevers is brought to the emergency department. She developed fever
4 days ago, was seen at an urgent care center 2 days ago, and was diagnosed with a viral illness.
Her parents were instructed to give her acetaminophen as needed for the fever. Since that time,
she has remained febrile. This morning, her parents found that her temperature was 39.1°C
rectally. They administered a dose of acetaminophen and brought her to the emergency
department. She had previously been well with normal growth and development. Her
immunizations are up-to-date.

She has a temperature of 37.9°C, a heart rate of 102 beats/min, a blood pressure of 98/62 mm
Hg, a respiratory rate of 22 breaths/min, and an oxygen saturation of 98% on room air. She is
awake, alert, and smiling in her mother’s arms. Her physical examination reveals small, freely
movable lymph nodes in the bilateral anterior cervical chains, as well as a right tympanic
membrane as shown in Item Q154. The remainder of her physical examination findings are
unremarkable.

Item Q154: Right tympanic membrane of the child in the vignette.


Courtesy of D McCormick and reprinted with permisson from Siegel RM, Bien JP. Acute otitis
media in children: a continuing story. Pediatr Rev. 2004;25(6):189.

American Academy of Pediatrics 563


PREP ® Self-Assessment PREPSA 2021
A nasal rapid viral panel is positive for parainfluenza. Laboratory data are shown:
Laboratory Test Result
White blood cell count 3,100/µL (3.1 × 109/L)
Hemoglobin 11.9 g/dL (119 g/L)
Platelet count 412 × 103/µL (412 × 109/L)
Neutrophil count 120/µL (0.12 × 109/L)
Lymphocyte count 2,800/µL (2.8 × 109/L)

Of the following, the MOST appropriate next step in treatment is to


A. consult an otolaryngologist for tympanocentesis to culture the middle ear fluid
B. obtain a blood culture and administer parenteral cefepime
C. prescribe oral amoxicillin/clavulanic acid for 10 days and discharge home
D. provide reassurance and discharge home with instructions to continue acetaminophen
therapy

American Academy of Pediatrics 564


PREP ® Self-Assessment PREPSA 2021
Correct Answer: B
The child in the vignette has a fever and very severe neutropenia. Neutropenia is categorized as
mild (1,000-1,500 neutrophils/µL), moderate (500-1,000 neutrophils/µL), severe (200-500
neutrophils/µL) and very severe (<200 neutrophils/µL). Owing to defective innate immunity,
children with neutropenia are at higher risk of developing invasive bacterial infections. As such,
all febrile episodes should be carefully considered and broad-spectrum antibiotics administered if
there is any concern regarding a clinically relevant immune deficit. As there are no prior absolute
neutrophil counts available for the child in the vignette to help define whether her neutropenia is
heritable or acquired, caution should be used, a blood culture obtained, and parenteral broad-
spectrum antibiotics administered.

The immune system can be divided into innate and adaptive components. These are defined by
their specificity: the innate immune system is nonspecific, whereas the adaptive immune system
is highly specific. The innate immune system includes physical barriers such as hair, skin, and
mucosa, as well as cellular components that include neutrophils and macrophages. The adaptive
immune system is composed of B and T cells responsible for humoral and cellular immunity.
Neutropenia is a deficit of the innate immune system, and it puts patients at risk of developing
invasive bacterial and fungal infections. Neutropenia can be heritable or acquired. Both
autoimmune neutropenia and viral suppression of neutrophil production are common causes of
acquired neutropenia, and both are not usually associated with a markedly increased risk of
developing invasive infection. Children who have repeated invasive infections and neutropenia
should undergo a thorough evaluation to determine the etiology (Item C154 ).

American Academy of Pediatrics 565


PREP ® Self-Assessment PREPSA 2021

Most episodes of otitis media can be treated empirically without tympanocentesis unless the
infection is refractory to antibiotic treatment. Given that the child in the vignette has not yet been
treated with antibiotics, tympanocentesis would be premature. Treatment with oral
amoxicillin/clavulanic acid would not provide adequately broad coverage for febrile neutropenia.
As the etiology of the neutropenia is not clear, reassurance and discharge without antibiotics
would be inappropriate in this setting of febrile neutropenia.

PREP Pearls
• Neutropenia is categorized as mild (1,000-1,500 neutrophils/µL), moderate (500-1,000
neutrophils/µL), severe (200-500 neutrophils/µL), and very severe (<200
neutrophils/µL).
• Neutropenia is a deficit of the innate immune system, and it puts patients at risk of
developing invasive bacterial and fungal infections.
• Children who have repeated invasive infections and neutropenia should undergo a
thorough evaluation to determine the etiology of the neutropenia.
• A child presenting with fever and neutropenia requires careful evaluation, blood cultures,
and treatment with broad-spectrum antibiotics unless the neutropenia is known to be of
benign nature.

American Academy of Pediatrics 566


PREP ® Self-Assessment PREPSA 2021
ABP Content Specifications(s)
• Understand the infection risks associated with neutropenia

Suggested Readings
• Chinn IK, Orange JS. Immunodeficiency disorders. Pediatr Rev. 2019;40(5):229-242.
doi:10.1542/pir.2017-0308.
• Newburger PE. Autoimmune and other acquired neutropenias. Hematology Am Soc
Hematol Educ Program. 2016;2016(1):38-42. doi:10.1182/asheducation-2016.1.38.
• Walkovich K, Boxer LA. How to approach neutropenia in childhood. Pediatr Rev.
2013;34(4):173-184. doi:10.1542/pir.34-4-173.

American Academy of Pediatrics 567


PREP ® Self-Assessment PREPSA 2021
Question 155
A 17-year-old adolescent girl is seen in the office for evaluation of a thin, malodorous vaginal
discharge for the past 10 days. She has intermittent pruritus and has experienced dysuria for the
past 2 days. She reports no fever, abdominal pain, or back pain. A vaginal douche and over-the-
counter miconazole vaginal cream have not improved her symptoms. She is sexually active and
had coitarche at age 16 years. She has had two male partners and has been dating her 18-year-old
boyfriend for the past 2 months. She engages in vaginal sex only. Her last sexual encounter was
2 weeks ago and without a condom. A contraceptive implant was placed in her left arm 4 months
ago for birth control. Screening for sexually transmitted infections (Chlamydia trachomatis,
Neisseria gonorrhœae, and human immunodeficiency virus) 6 months ago had negative results.
A pelvic examination shows normal external genitalia with no lesions. Speculum examination is
significant for yellow, frothy, malodorous discharge, with a few scattered petechiae on the
cervix. On bimanual examination, there is no cervical motion or adnexal or uterine tenderness.

Of the following, the organism that is the MOST likely cause of this patient’s symptoms is
A. Candida albicans
B. Chlamydia trachomatis
C. Gardnerella vaginalis
D. Trichomonas vaginalis

American Academy of Pediatrics 568


PREP ® Self-Assessment PREPSA 2021
Correct Answer: D
The girl in the vignette has trichomoniasis, which is caused by the protozoan
parasite Trichomonas vaginalis. Most infected individuals with trichomoniasis are asymptomatic
(70%-85%). Women who are symptomatic may have a yellowish-green vaginal discharge
characteristically described as being frothy. Other presenting symptoms may include dysuria,
vaginal itching, and odor. On speculum examination, the cervix may resemble a strawberry
owing to inflammation and petechiae, but this is a rare physical examination finding (in less than
5% of infected women). Ten percent of men with infection may exhibit dysuria or penile
discharge. Trichomoniasis may put a woman at risk of experiencing preterm labor and increased
risk of acquiring other sexually transmitted infections.

The diagnosis of trichomoniasis in a symptomatic female is commonly made from a wet mount
examination of collected vaginal discharge. The protozoa are often mobile on microscopic
examination (Item C155). The sensitivity for microscopic examination is 50% to 70%. Culture
of vaginal fluid, semen, and urine is considered the reference standard; however, culture is rarely
done because it requires special culture and transport media, and the results take up to 1 week.
Other means of diagnosis include nucleic acid amplification tests (NAATs). These tests have
better sensitivity and specificity (95%-100%) than culture. The US Food and Drug
Administration has approved NAATs for use in specimens from females. Testing for
trichomoniasis with NAATs can be used for urine samples and urethral swabs in men, if the
laboratory has been validated per clinical laboratory improvement amendments. More often, men
are treated presumptively if their female partners are found to have trichomoniasis.

The recommended first-line treatment regimen for trichomoniasis includes metronidazole 2 g


orally or tinidazole 2 g orally as a single dose. The alternative regimen is metronidazole 500 mg
orally twice daily for 7 days, if the initial treatment was not effective and there is no reinfection.
Metronidazole intravaginal gel is available, but its efficacy is poor. It is important that partners
be treated to prevent reinfection.

The American Academy of Pediatrics, the Centers for Disease Control and Prevention, and the
US Preventive Services Task Force recommends annual screening for chlamydia and gonorrhea
in adolescents who are sexually active and younger than 25 years. However, routine screening
for trichomoniasis in asymptomatic females is not recommended unless the female is in a high-
risk setting such as a sexually transmitted disease clinic or correctional facility; has multiple
sexual partners; or exchanges sex for money.

The girl in this vignette has undergone screening within the past 6 months for gonorrhea and
chlamydia, the results of which were negative for both. If she were to be symptomatic
with Chlamydia trachomatis infection, she may have exhibited mucopurulent endocervical
discharge, a friable cervix, or both. Gardnerella vaginalis is an anaerobe that is a common
etiology in bacterial vaginosis. Bacterial vaginosis is associated with a thin whitish-grey vaginal
discharge that often has a fishy (amine) odor. A candida vulvovaginitis is usually associated with
a thick, white, odorless vaginal discharge with itching, burning, or irritation. It is likely the girl’s
symptoms would have improved after a course of intravaginal miconazole.

American Academy of Pediatrics 569


PREP ® Self-Assessment PREPSA 2021
PREP Pearls
• The majority of adolescents and young adults with trichomoniasis are asymptomatic.
• A female is usually identified as having trichomoniasis owing to vaginal discharge.
• Screening for Trichomonas vaginalis is not recommended as part of routine screening for
sexually transmitted infections in adolescents and young adults.

MOCA-Peds Objective
• Screen an adolescent for sexually transmitted diseases and manage appropriately.

ABP Content Specifications(s)


• Plan the appropriate diagnostic evaluation when Trichomonas vaginalis infestation is
suspected
• Recognize the clinical features associated with Trichomonas vaginalis infestation, and
manage appropriately, including management of sexual partners

Suggested Readings
• Custodio H. Protozoan parasites. Pediatr Rev. 2016;37(2):59-69. doi:10.1542/pir.2015-
0006.
• Joffe A. Sexually transmitted infections. In: McInerny TK, Adam HM, Campbell DE,
DeWitt TG, Foy JM, Kamat DM, eds. American Academy of Pediatrics Textbook of
Pediatric Care. 2nd ed. Elk Grove Village, IL: American Academy of Pediatrics;
2017:2628-2651. Pediatric Care Online .
• Marcell AV, Burstein GR; Committee on Adolescence. Sexual and reproductive health
care services in the pediatric setting. Pediatrics. 2017;140(5);e20172858.
doi:10.1542/peds.2017-2858.
• Perry MA, Allison BA. Gonorrheal diseases. Pediatr Rev. 2018;39(8):427-429.
doi:10.1542/pir.2017-0120.

American Academy of Pediatrics 570


PREP ® Self-Assessment PREPSA 2021
Question 156
A 3-year-old child is admitted to the pediatric intensive care unit with acute respiratory failure
requiring intubation and mechanical ventilation. Central venous access is desired, and placement
of a right-sided subclavian catheter is attempted multiple times; however, definitive access is not
obtained. A right-sided internal jugular venous catheter is then placed, and a central venous
pressure waveform is transduced before the line is sutured in place. Soon afterward, the patient
experiences worsening respiratory status, requiring higher peak airway pressures and higher
fraction of inspired oxygen to prevent desaturations. A chest radiograph is obtained (Item
Q156).

Item Q156: Chest radiograph for the patient described in this vignette. Courtesy of J Kane

Of the following, the MOST appropriate intervention is to


A. obtain a computed tomographic scan of the chest
B. perform needle decompression of a left-sided pneumothorax
C. place a thoracostomy tube in the right side of the chest
D. remove the right-sided internal jugular central venous catheter

American Academy of Pediatrics 571


PREP ® Self-Assessment PREPSA 2021

Correct Answer: C
The patient in the vignette has developed a right-sided hemothorax as a result of failed central
venous catheter placement in the subclavian vein. The image (Item C156) reveals a large right-
sided pleural effusion, which, given the scenario of a failed subclavian catheter attempt, makes a
hemothorax the most likely underlying etiology. Of the responses listed, the most appropriate
intervention is to place a thoracostomy tube in the right side of the chest to evacuate the
hemothorax. Hemothorax occurs when blood collects in the pleural space and may result from
direct injury to the lung parenchyma, trauma to the chest wall, or injury to intrathoracic vascular
structures.

Item C156: Large right-sided pleural effusion. Courtesy of J Kane

The pleural space is a virtual space that, in adults, can accommodate approximately 15 mL of
fluid. An accumulation of fluid in the pleural space results in a pleural effusion and, if
significantly large, can impair respiratory function. Transudates are formed when there is a
disturbance in the balance between the production of pleural fluid and its absorption, often with
increased hydrostatic or decreased oncotic pressure (examples include congestive heart failure,
nephrotic syndrome, and cirrhosis with ascites), whereas exudates generally form because of
increased capillary permeability (examples include infection, inflammation, malignancy, and
chylothorax). Criteria to define an exudative pleural fluid include a ratio of pleural protein to

American Academy of Pediatrics 572


PREP ® Self-Assessment PREPSA 2021
serum protein of greater than 0.5, a ratio of pleural lactate dehydrogenase (LDH) to serum LDH
greater than 0.6, and LDH of pleural fluid greater than 66% of the upper limit of serum LDH.

The majority of pleural effusions in children result from bacterial pneumonia and are
parapneumonic. These can transition from being a simple exudative effusion with normal pH and
glucose to being a more fibrinopurulent effusion with pus formation and polymorphonuclear
leukocyte infiltration and fibrin deposition, resulting in a decrease in glucose and pH and an
increase in LDH. With time, the fibrinopurulent effusion may form loculations or promote
fibroblast growth in the parenchymal and visceral pleura, resulting in a fibrin peel. Other causes
of pleural effusion include neoplasm, tracking of fluid from abdominal pathology (pancreatitis),
chest trauma, congestive heart failure, and capillary leak syndrome or systemic inflammatory
response. Iatrogenic causes of pleural effusions include chylothorax after cardiac surgery as well
as malposition or failed intravascular catheter placement, as in the patient in this vignette.

For simple effusions, conservative therapy with diuretics and colloids (including albumin) may
promote absorption of pleural fluid. However, more complicated effusions, including
hemothorax or purulent effusions, often require formal drainage with a thoracostomy tube. A
hemothorax often requires a large-bore chest tube to limit the likelihood of thrombus formation
in the tube, which would obstruct drainage.

Although computed tomography of the chest can help elucidate the underlying cause of a pleural
effusion and also delineate the size of the effusion, in the vignette the most likely cause of the
hemothorax is failed central venous access; as such, there is no indication for exposing the
patient to the radiation associated with computed tomography of the chest. There is no left-sided
pneumothorax, so needle decompression would not be indicated. Finally, because the right-sided
central venous catheter reveals a central venous pressure waveform by transduction, there is no
indication for removal of the catheter.

PREP Pearls
• Pleural effusions are commonly associated with infectious pneumonia.
• Hemothorax is a known complication of central venous access, particularly of the
subclavian vessels.
• Large pleural effusions may impair respiratory function and require drainage with a
thoracostomy tube or therapeutic pleurocentesis.

ABP Content Specifications(s)


• Recognize the clinical findings associated with a hemothorax or flail chest

Suggested Readings
• Efrati O, Barak A. Pleural effusions in the pediatric population. Pediatr
Rev. 2002;23(12):417-426. doi:10.1542/pir.23-12-417.
• Sahn SA. Diagnosis and management of parapneumonic effusions and empyema. Clin
Infect Dis. 2007;45(11):1480-1486. doi:10.1086/522996.

American Academy of Pediatrics 573


PREP ® Self-Assessment PREPSA 2021
Question 157
A 15-year-old adolescent who recently emigrated to the United States from Tanzania is being
evaluated. He has been adopted by an uncle with HIV disease. Upon arrival in the United States,
the adolescent had a negative serum test result for HIV antibodies and a tuberculin skin test,
which had an 11-mm induration 60 hours after placement. He currently has no complaints and
normal physical examination findings.

Of the following, the BEST next management step for this patient is
A. a 2-view chest radiograph
B. gastric aspirate for acid-fast organisms
C. repeat tuberculin skin test in 2 months
D. treatment with a 4-drug regimen for tuberculosis

American Academy of Pediatrics 574


PREP ® Self-Assessment PREPSA 2021
Correct Answer: A
The boy in the vignette has a positive result on the tuberculin skin test (TST), defined as an
induration of 10 mm or more in a child who recently emigrated from an area where tuberculosis
is endemic. The most appropriate next step is a 2-view chest radiograph. Given his lack of any
symptoms of disease, if the radiography result is negative, the boy should be treated for latent
tuberculosis with 1 of several appropriate regimens. If radiographic findings suggest active
tuberculosis, it would then be appropriate to obtain either sputum or gastric aspirate samples for
acid-fast staining and culture. While awaiting culture results, treatment for active tuberculosis is
indicated if there are symptoms of tuberculosis or an abnormal chest radiograph. Repeating the
TST in 2 months would not be the appropriate next step in management for the boy in the
vignette.

Latent tuberculosis is most often treated with daily isoniazid for 9 months; several alternative
regimens are outlined in Item C157A. Active tuberculosis disease is usually treated with a 4-
drug regimen of isoniazid, rifampin, pyrazinamide, and ethambutol for 2 months, followed by 4
months of isoniazid and rifampin. Alternative regimens may be needed in situations of drug
intolerance or resistant tuberculosis. Second- and third-line antibiotics include aminoglycosides,
quinolones, and other drugs used in combination (Item C157B).

American Academy of Pediatrics 575


PREP ® Self-Assessment PREPSA 2021

American Academy of Pediatrics 576


PREP ® Self-Assessment PREPSA 2021

American Academy of Pediatrics 577


PREP ® Self-Assessment PREPSA 2021

American Academy of Pediatrics 578


PREP ® Self-Assessment PREPSA 2021
Interpretation of the TST differs by the age of the subject and the risk for active disease. Item
C157C outlines the criteria for a positive test result in various populations. An alternative to the
TST is the interferon γ release assay (IGRA) which measures the response to in-vitro exposure of
T cells to Mycobacterium tuberculosis. The IGRA is preferred in patients with prior exposure to
the Bacille Calmette-Guerin vaccine (BCG) because it does not recognize the antigens contained
in BCG. It is also the preferred test in patients who are not likely to return for interpretation of a
TST. Neither the TST nor the IGRA differentiates between latent and active disease, but both
provide a screening method that measures immunologic sensitization from exposure to the
organism indicating the need for further investigation. Active tuberculosis disease is confirmed
with a culture of expectorated organisms in sputum or gastric aspirate or from bronchoalveolar
lavage fluid.

PREP Pearls
• Interpretation of the tuberculin skin test varies with age and risk for active disease.
• Both the tuberculin skin test and the interferon γ release assay can detect exposure and
sensitization to Mycobacterium tuberculosis, but cannot diagnose active disease.
• Diagnosis of active tuberculosis disease is based on symptoms and chest radiography
findings in the presence of a positive screening test for sensitization (tuberculin skin test
or interferon γ release assay); diagnosis should be confirmed based on culture
of Mycobacterium tuberculosis from sputum, gastric aspirate, or bronchoalveolar lavage
fluid.

ABP Content Specifications(s)


• Interpret the results of a tuberculin skin test, differentiating among positive, false-
positive, and false-negative reactions
• Plan appropriate management for a patient with Mycobacterium tuberculosis infection in
various circumstances
• Identify patients at risk of latent tuberculosis and manage appropriately

Suggested Readings
• American Academy of Pediatrics. Tuberculosis. In: Kimberlin DW, Brady MT, Jackson
MA, Long SS, eds. Red Book: 2018-2021 Report of the Committee on Infectious
Diseases. Itasca, IL: American Academy of Pediatrics; 2018:829-853. Red Book Online.
• Carvalho I, Goletti D, Manga S, et al. Managing latent tuberculosis infection and
tuberculosis in children. Pulmonology. 2018;24:106-114.
doi: 10.1016/j.rppnen.2017.10.007.
• Holmberg PJ, Temesgen Z, Banerjee R. Tuberculosis in children. Pediatr
Rev. 2019;40:168-176. doi: 10.1542/pir.2018-0093.
• Lewinsohn DM, Leonard MK, LoBue PA, et al. Official American Thoracic
Society/Infectious Disease Society of America/Centers for Disease Control and
Prevention practice guidelines: diagnosis of tuberculosis in adults and children. Clin
Infect Dis. 2017;64(2):111-115. doi: 10.1093/cid/ciw778.
• Nahid P, Dorman SE, Alipanah N, et al. Official American Thoracic Society/Centers for
Disease Control and Prevention/Infectious Disease Society of America clinical practice
guidelines: treatment of drug susceptible tuberculosis. Clin Infect Dis. 2016;63:e147-
e195. doi: 10.1093/cid/ciw376.
American Academy of Pediatrics 579
PREP ® Self-Assessment PREPSA 2021
Question 158
A 12-year-old girl is being evaluated for tooth pain that started a few days ago, but worsened
yesterday. She has left upper molar pain with localized gingival swelling; the pain is making it
difficult to eat. Her vital signs include a temperature of 38.3°C, a heart rate of 105 beats/min, and
a respiratory rate of 17 breaths/min. Her mouth has an area of redness, swelling, and fluctuance
along the maxillary left molar, which is tender to the touch.

Of the following, the MOST likely cause of this infection is


A. Actinomyces israelii
B. Pseudomonas aeruginosa
C. Staphylococcus aureus
D. Streptococcus pyogenes

American Academy of Pediatrics 580


PREP ® Self-Assessment PREPSA 2021
Correct Answer: A
Dental infections are associated with anaerobic bacteria. Of the response choices, Actinomyces
israelii is the only anaerobic bacteria.

Routine oral hygiene is important throughout childhood and adolescence. Brushing twice daily
with a fluoridated toothpaste and an electric toothbrush and flossing daily should be encouraged
at health supervision visits. Twice-yearly fluoride treatments can help protect dental enamel;
these treatments can be applied in a general pediatric office or during routine dental visits. A
healthy diet that avoids excessive fats and simple carbohydrates, especially sugary beverages,
reduces plaque accumulation and improves dental health. During the early adolescent years,
additional counseling should include recommendation for avoidance of all forms of nicotine and
information to promote understanding that oral piercings with metal jewelry can injure teeth and
gums. These injuries and overall poor dental health increase the risk of local and systemic
infection.

Pseudomonas aeruginosa, Staphylococcus aureus, and Streptococcus pyogenes are less likely
causes of dental infection in the girl in the vignette.

PREP Pearls
• Anaerobic bacteria are the cause of most dental infections.
• Good routine dental hygiene should be reviewed at health supervision visits.

ABP Content Specifications(s)


• Understand the association between an anaerobic infection and dental and periodontal
disease

Suggested Readings
• Brook I. The role of anaerobic bacteria in bacteremia. Anaerobe. 2010;16(3):183-189.
doi:10.1016/j.anaerobe.2009.12.001.
• Silk H, Kwok A. Addressing adolescent oral health: a review. Pediatr
Rev. 2017;38(2):61-68. doi:10.1542/pir.2016-0134.
• Slayton RL. Oral health. In: McInerny TK, Adam HM, Campbell DE, DeWitt TG, Foy
JM, Kamat DM, eds. American Academy of Pediatrics Textbook of Pediatric Care. 2nd
ed. Elk Grove Village, IL: American Academy of Pediatrics; 2016:281-290. Pediatric
Care Online.

American Academy of Pediatrics 581


PREP ® Self-Assessment PREPSA 2021
Question 159
A 6-year-old girl is undergoing evaluation of a “bald spot” on her scalp that was first noted 3
weeks ago. Since then, the area has grown somewhat smaller. The girl has been in good health
and takes no medications. Her temperature is 37°C, and her other vital signs are normal. The
girl’s physical examination findings are unremarkable, with the exception of a round patch of
nearly complete hair loss on the parietal scalp. The scalp appears normal without erythema or
scaling (Item Q159 ).

Item Q159: A patch of nearly complete alopecia. Courtesy of D Krowchuk

Of the following, the MOST appropriate management of this girl’s condition is


A. griseofulvin orally
B. mupirocin topically
C. no intervention
D. referral for cognitive behavioral therapy

American Academy of Pediatrics 582


PREP ® Self-Assessment PREPSA 2021

Correct Answer: C
The girl in the vignette has a round patch of nearly complete hair loss and a normal-appearing
scalp. These findings suggest a diagnosis of alopecia areata. Because the patch is small and
appears to be improving, no therapy is necessary. In the absence of scale, evidence of
inflammation (erythema or pustule formation), or “black-dot” hairs (the remnants of broken hairs
within follicles), tinea capitis is unlikely and griseofulvin therapy is not indicated (Item C159A).
Traction on hairs, often the result of tight braiding, may produce folliculitis (Item C159B) and
ultimately alopecia. In severe cases in which concern exists regarding bacterial infection, a
topical or oral antibiotic may be prescribed. Cognitive-behavioral therapy may be used for those
who have hair-pulling disorder (trichotillomania). The physical findings in this condition differ
from those of alopecia areata: a patch of incomplete alopecia within which hairs of differing
lengths may be seen (Item C159C).

American Academy of Pediatrics 583


PREP ® Self-Assessment PREPSA 2021
Item C159A: In the most common form of tinea capitis, there are one or more patches of hair
loss within which one may see scale, “black-dot” hairs (yellow arrows), or pustules (red
arrows). Reprinted with permission from Mancini AJ, Krowchuk DP, eds. Pediatric
Dermatology. A Quick Reference Guide. 3rd ed. Elk Grove Village, IL: American Academy of
Pediatrics; 2016.

Item C159B: Traction on hairs may result in erythematous follicular papules or pustules
(arrows). Continued traction may result in alopecia. Courtesy of D Krowchuk

American Academy of Pediatrics 584


PREP ® Self-Assessment PREPSA 2021

Item C159C: In hair-pulling disorder an area of incomplete hair loss is seen within which hairs
of differing lengths are present. In this patient, 2 areas of hemorrhage (arrows) are present at
sites where hairs were pulled. Courtesy of D Krowchuk

Alopecia areata is an autoimmune disease in which T-lymphocytes target specific autoantigens


expressed by hair follicles. Genetic susceptibility (approximately 15% of patients have an
affected first-degree relative) and environmental insults (physical or emotional stress, hormones,
infection) contribute to the disease process. Associated autoimmune diseases, particularly
thyroiditis, occur rarely in affected children (routine testing of thyroid function in children who
have alopecia areata is not recommended). The prevalence of alopecia areata is estimated to be
0.2% and the lifetime risk is 1% to 2%. Two-thirds of patients have onset of disease before the
age of 16 years.

The typical presentation is the sudden appearance of one or a few round or oval, well-defined
patches of hair loss; the scalp appears normal. At the periphery of patches of alopecia, one may
observe short hairs that are broader distally than proximally (“exclamation-point” hairs). Some
patients develop numerous areas of hair loss or a circumferential loss of hair involving the
temporal, parietal, and occipital scalp (ie, the ophiasis pattern). In a minority of individuals
(approximately 10%) the disease progresses to loss of all or nearly all scalp hair (alopecia totalis)
or body hair (alopecia universalis) (Item C159D). Nail pitting occurs in about 20% of affected
patients (Item C159E).
American Academy of Pediatrics 585
PREP ® Self-Assessment PREPSA 2021

Item C159D: Alopecia totalis: there is loss of nearly all scalp hair. Courtesy of D Krowchuk

American Academy of Pediatrics 586


PREP ® Self-Assessment PREPSA 2021

Item C159E: Multiple small nail pits may be observed in those who have alopecia areata.
Reprinted with permission from Mancini AJ, Krowchuk DP, eds. Pediatric Dermatology. A
Quick Reference Guide. 3rd ed. Elk Grove Village, IL: American Academy of Pediatrics; 2016.

The prognosis of alopecia areata is variable. Among those who have a few small patches of hair
loss, most will regrow hair within 1 year. In such cases, observation may be appropriate. The
prognosis is more guarded for those who have extensive hair loss, the ophiasis pattern, a
coexisting autoimmune disorder, or a family history of alopecia areata.

Treating alopecia areata is challenging, and those who have significant disease are best treated
by a dermatologist. First-line therapy typically includes a topical corticosteroid, often in
conjunction with topical minoxidil. The topical corticosteroid selected usually is an ultrapotent
agent (group I [eg, clobetasol propionate]), or for children <10 years of age, a potent agent
(group II [eg, fluocinonide]). To prevent steroid-induced atrophy, application is in a “pulsed”
fashion (applied consecutively for 4-5 days, followed by 2-3 days without treatment). Other

American Academy of Pediatrics 587


PREP ® Self-Assessment PREPSA 2021
options include intralesional or oral corticosteroids, methotrexate, excimer laser, anthralin
topically, topical immunotherapy, or oral or topical Janus kinase inhibitors. Areas of hair loss
can be masked by hairstyle changes, hats, or headbands. For those with extensive involvement, a
wig may be helpful. Locks of Love (www.locksoflove.org) provides hairpieces to financially
disadvantaged children younger than 21 years. The National Alopecia Areata Foundation
(www.naaf.org) provides information and support for patients and families.

PREP Pearls
• Alopecia areata presents as one or more round patches of hair loss; the scalp appears
normal without scaling, black-dot hairs, or inflammation.
• For children who have one or a few patches of alopecia areata, the chance of spontaneous
regrowth of hair is very good. The prognosis is poorer for those who have extensive hair
loss, the ophiasis pattern, a coexisting autoimmune disorder, or a family history of
alopecia areata.
• The management of alopecia areata is challenging; most often, initial therapy employs a
potent topical corticosteroid and topical minoxidil.

ABP Content Specifications(s)


• Recognize the clinical findings associated with telogen effluvium, and manage
appropriately
• Recognize the clinical findings associated with alopecia areata and manage appropriately
• Recognize the clinical findings associated with trichotillomania, and manage
appropriately

Suggested Readings
• Mancini AJ, Krowchuk DP, eds. Pediatric Dermatology. A Quick Reference Guide. 3rd
ed. Elk Grove Villlage, IL; American Academy of Pediatrics: 2016.
• Strazzulla LC, Wang EHC, Avila L, et al. Alopecia areata: an appraisal of new treatment
approaches and overview of current therapies. J Am Acad Dermatol. 2018;78(1):15-24.
doi:10.1016/j.jaad.2017.04.1142.
• Strazzulla LC, Wang EHC, Avila L, et al. Alopecia areata: disease characteristics,
clinical evaluation, and new perspectives on pathogenesis. J Am Acad
Dermatol. 2018;78(1):1-12. doi:10.1016/j.jaad.2017.04.1141.

American Academy of Pediatrics 588


PREP ® Self-Assessment PREPSA 2021
Question 160
An 8-year-old girl is seen in the office for concerns about her academic performance. Her
teachers note that she tends to daydream and is struggling to stay focused on her work; at times
they find it difficult to get her attention. Her parents report that sometimes they must ask her a
question 2 or 3 times before receiving a response. She is an otherwise healthy child and when
engaged in a task she is able to complete it. The girl reports that she finds herself “losing time”
periodically throughout the day which sometimes makes it hard to stay focused on her activity.
In the office, the girl is sitting quietly next to her parents. She intermittently stares, then rapidly
returns to the conversation. Her general physical examination findings are unremarkable.
Neurologic findings are normal, though during the examination, she has an episode of staring,
during which she is not responsive to direction for a few seconds.

Of the following, the BEST next management step for this girl is
A. counseling
B. electroencephalography
C. magnetic resonance imaging of the brain
D. neuropsychological evaluation

American Academy of Pediatrics 589


PREP ® Self-Assessment PREPSA 2021

Correct Answer: B
The clinical presentation of the girl in the vignette is most consistent with absence seizures.
Electroencephalography (EEG) is the best next step in the diagnostic workup for this girl’s
staring episodes. Childhood absence epilepsy is a common childhood epilepsy syndrome
presenting between the ages of 4 and 10 years. An absence seizure is a brief lapse in
consciousness, clinically appearing as a staring episode, lasting seconds, and occurring up to
hundreds of times in a day. Automatisms can be present and may include eye blinking, eyelid
fluttering, lip smacking, or hand or face twitching. No postictal state occurs. One-third of
patients develop generalized tonic-clonic seizures as part of the absence epilepsy, which may
affect antiepileptic medication choice. Because the clinical presentation of absence seizures can
be subtle, they can be mistaken for inattention or daydreaming; their frequency is often
underestimated.

The diagnosis of absence seizures is confirmed with EEG, which will demonstrate interictal 3-Hz
generalized spike and wave discharges. Seizures can be provoked in the office setting and during
EEG recording with hyperventilation maneuvers. Recommended antiepileptic medication
choices include ethosuximide, valproic acid, and lamotrigine. Ethosuximide is considered the
first-line treatment option for absence seizures, because of its favorable side effect profile and
efficacy; however, it does not provide adequate treatment for children who also have generalized
tonic-clonic seizures. The prognosis for children with absence seizures is favorable, with most
children outgrowing their epilepsy in adolescence.

Findings on magnetic resonance imaging of the brain are generally normal in children with
childhood absence epilepsy. Counseling may be beneficial in children with epilepsy who have
comorbid anxiety or mood disorders, which are more common in the patient population with
epilepsy. Because children with absence epilepsy have a higher rate of attention-
deficit/hyperactivity disorder as well as behavioral and cognitive disorders than the general
population, neuropsychological evaluation may be helpful to evaluate for these common
comorbidities, but would not be the best next step in this child’s evaluation.

PREP Pearls
• An absence seizure is a brief lapse in consciousness, clinically appearing as a staring
episode, which can have associated automatisms or generalized tonic-clonic seizures.
• Ethosuximide is the first-line antiepileptic medication of choice for absence seizures.
Alternative options include valproic acid or lamotrigine.
• Absence seizures are commonly mistaken for inattention or daydreaming. The presence
of interictal 3-Hz spikes and slow-wave generalized discharges on
electroencephalography can confirm the diagnosis.

ABP Content Specifications(s)


• Recognize the clinical findings associated with absence seizures, and manage
appropriately

American Academy of Pediatrics 590


PREP ® Self-Assessment PREPSA 2021
Suggested Readings
• Glauser TA, Cnaan A, Shinnar S, et al. Ethosuximide, valproic acid and lamotrigine in
childhood absence epilepsy. N Engl J Med. 2010;362:790-799.
doi: 10.1056/NEJMoa0902014.
• Park JT, Shahid AM, Jammoul A. Common pediatric epilepsy syndromes. Pediatr
Ann. 2015;44(2):e30-e35. doi: 10.3928/00904481-20150203-09.
• Roddy SM, McBride M. Seizure disorders. In: McInerny TK, Adam HM, Campbell DE,
DeWitt TG, Foy JM, Kamat DM, eds. American Academy of Pediatrics Textbook of
Pediatric Care. 2nd ed. Itasca, IL: American Academy of Pediatrics; 2016;chap
327:2599-2616. Pediatric Care Online.
• Sidhu R, Velayudam K, Barnes G. Pediatric seizures. Pediatr Rev. 2013;34(8):333-342.
doi: 10.1542/pir.34-8-333.

American Academy of Pediatrics 591


PREP ® Self-Assessment PREPSA 2021
Question 161
A 3-day-old neonate with trisomy 21 and a muscular ventricular septal defect is having an initial
health supervision visit. She has been feeding well. Her current weight is 5% below birthweight.
Her hemoglobin on discharge was 14 g/dL (140 g/L). Physical examination reveals a temperature
of 37.1°C, heart rate of 135 beats/min, respiratory rate of 24 breaths/min, and blood pressure of
72/45 mm Hg. She has the typical facies of trisomy 21, mild generalized hypotonia, +2 dorsalis
pedis pulses, and bluish discoloration of the hands and feet.

Of the following, the MOST likely reason for this neonate’s cyanosis is
A. acrocyanosis
B. congenital heart disease
C. methemoglobinemia
D. polycythemia

American Academy of Pediatrics 592


PREP ® Self-Assessment PREPSA 2021
Correct Answer: A
Cyanosis may be classified as peripheral or central depending on its location on physical
examination. In a healthy term neonate, acrocyanosis (peripheral cyanosis of the hands and feet)
is a benign condition seen in cool environments, after feeding, or after bathing. It is thought to be
caused by immature vascular control. Acrocyanosis is more pronounced in neonates with trisomy
21.

A muscular ventricular septal defect would not present with hypoxemia in the neonatal period.
This neonate has no risk factors for methemoglobinemia. Although polycythemia can be
associated with acrocyanosis, a hemoglobin level of 14 g/dL (140 g/dL) is not considered
polycythemic.

Neonates with central cyanosis will have a bluish discoloration of the lips or tongue associated
with hypoxemia. The differential diagnosis for central cyanosis in a neonate includes respiratory,
cardiac, and neurologic conditions. Cyanotic congenital heart disease such as tetralogy of Fallot,
tricuspid atresia, transposition of the great arteries, truncus arteriosus, total anomalous
pulmonary venous return, and hypoplastic left heart syndrome can cause central cyanosis.
Neonates with lung disease such as transient tachypnea of the newborn, meconium aspiration
syndrome, pulmonary hypertension, and congenital diaphragmatic hernia may have central
cyanosis. Upper airway obstruction caused by choanal atresia or Pierre-Robin sequence may also
present with central cyanosis. Severe neurologic disease impairing respiration such as seizures,
neuromuscular disorders, or intracranial hemorrhage may result in central cyanosis.

It should be noted that cyanosis may be difficult to detect visually. Cyanosis is visible when 3
g/dL (30 g/L) of deoxygenated hemoglobin is present. Therefore, neonates with polycythemia
with a higher hemoglobin concentration may appear cyanotic even though their oxygen content
is normal; patients with anemia may not be visibly cyanotic.

PREP Pearls
• In a healthy term neonate, acrocyanosis (peripheral cyanosis of the hands and feet) is a
benign condition seen in cool environments, after feeding, or after bathing.
• With central cyanosis, the lips or tongue may appear bluish in association with
hypoxemia.
• The differential diagnosis for central cyanosis in a neonate includes respiratory, cardiac,
and neurologic conditions.

ABP Content Specifications(s)


• Recognize that peripheral cyanosis is common in healthy newborn infants

American Academy of Pediatrics 593


PREP ® Self-Assessment PREPSA 2021
Suggested Readings
• Enlow E, Greenberg JM. Clinical manifestations of diseases in the newborn period. In:
Kliegman R, St Geme JW, Blum NJ, Shah SS, Tasker RC, Wilson K, eds. Nelson
Textbook of Pediatrics. Philadelphia, PA: Elsevier; 2020:910-913.e4.
• Fleck DE, Hoeltzel MF. Hand and foot color change: diagnosis and management. Pediatr
Rev. 2017;38:511-519. doi: 10.1542/pir.2016-0234.
• Tingelstad J. Consultation with the specialist: nonrespiratory cyanosis. Pediatr
Rev. 1999;20:350-352. doi: 10.1542/pir.20-10-350.
• Ushay HM. Cyanosis. In: McInerny TK, Adam HM, Campbell DE, DeWitt TG, Foy JM,
Kamat DM, eds. American Academy of Pediatrics Textbook of Pediatric Care. 2nd ed.
Itasca, IL: American Academy of Pediatrics; 2016;chap 136:1252-1257. Pediatric Care
Online.

American Academy of Pediatrics 594


PREP ® Self-Assessment PREPSA 2021
Question 162
A 6-week-old infant girl is seen in the office for evaluation of a 4-day history of cough,
posttussive emesis, and fussiness. The mother reports that the infant became apneic and limp for
a brief period after a coughing episode at home. Her grandmother has had a chronic cough for
more than 1 month. The infant has a temperature of 37.2°C, a heart rate of 148 beats/min, a
respiratory rate of 37 breaths/min, a blood pressure of 82/44 mm Hg, and an oxygen saturation of
96% on room air. Findings of a respiratory examination are notable for bilateral coarse breath
sounds. Several episodes of transient apnea are observed. Findings from the rest of the physical
examination are unremarkable. Rapid viral antigen testing via direct fluorescent antibody is
negative for respiratory syncytial virus.

Of the following, the BEST next diagnostic step in the evaluation of this infant’s illness is
A. chest radiography
B. complete blood cell count
C. nasopharyngeal swab for polymerase chain reaction
D. serology

American Academy of Pediatrics 595


PREP ® Self-Assessment PREPSA 2021
Correct Answer: C
The infant described in this vignette has a clinical picture suggestive of pertussis (ie, whooping
cough). Pertussis is a highly contagious acute respiratory tract infection caused by Bordetella
pertussis, a fastidious gram-negative coccobacillus. The diagnosis of pertussis may be
challenging in infants younger than 6 months. Pertussis in young infants can have an atypical
presentation during the early stages of illness characterized by gagging, gasping, apnea, or
bradycardia without a “whoop.”

Nucleic acid amplification tests using polymerase chain reaction assay from nasopharyngeal
(NP) wash, aspirate, or Dacron swab is the diagnostic modality of choice for diagnosis of
pertussis. Compared to culture, polymerase chain reaction assays are highly sensitive and
specific, and results are rapidly available. Commercially available serologic assay using IgG
antibody to pertussis toxin (PT) may be used for presumptive diagnosis of pertussis. Elevated
IgG antibody to PT is detected approximately 2 to 8 weeks after a coughing illness and is
indicative of recent pertussis disease (only in the absence of recent pertussis vaccination). A
single positive IgG level to PT (> 100 IU/mL) is indicative of diagnosis. Complete blood count
may show marked leukocytosis (15,000 to 100,000 cells/µL) owing to an increase in absolute
lymphocytes in approximately 75% of unimmunized children. A chest radiograph may show
perihilar infiltrate or interstitial edema with or without atelectasis. Consolidation is noted in
pertussis cases complicated by secondary bacterial infection.

Pertussis cases declined in the United States after the introduction of whole-cell pertussis (DTP)
vaccine in the 1940s. A gradual increase in pertussis cases was noted during the 1980s. By 2017,
pertussis cases peaked at more than 18,000. Acellular pertussis (DTaP) vaccine replaced DTP for
all doses by 1997. Immunity to pertussis wanes over time after receipt of acellular pertussis
vaccine. The epidemiology of pertussis has changed in the United States owing to many
contributing factors, including the exclusive use of acellular pertussis vaccines, waning
immunity, changes to the microorganism, vaccine refusal, and undetected cases. Adults are
reservoirs of the infection. Children younger than 1 year have the highest reported rate of
pertussis; infants younger than 6 months are at risk of experiencing severe disease and death.
The incubation period of pertussis is 7 to 10 days but may be prolonged up to 21 days. Clinical
illness is categorized into three stages, each lasting approximately 2 weeks; the total duration of
illness ranges from 6 to 10 weeks. In the initial catarrhal stage, patients exhibit symptoms of
cough and rhinorrhea. This is followed by the paroxysmal stage, consisting of repeated coughing
spells during the same breath followed by a “whoop” with associated emesis. Fever is typically
low grade or absent. The illness ends with the convalescent stage, during which symptoms wane
over weeks to months.

Pertussis must be included in the differential diagnosis of patients who have perioral cyanosis
during coughing spells, posttussive emesis, and prolonged coughing illness.

Young infants are at risk of experiencing complications such as pneumonia, pulmonary


hypertension, hypoxia, seizures, and encephalopathy. The disease is fatal in approximately 1% of
infants younger than 2 months and 0.5% in affected infants aged 2 to 11 months. Complications
of pertussis in adults include syncope, rib fractures, pneumonia, and weight loss. The systemic

American Academy of Pediatrics 596


PREP ® Self-Assessment PREPSA 2021
manifestations and laboratory findings of marked leukocytosis and lymphocytosis noted in
infants with pertussis may be related to pertussis toxin, a key virulence factor of B pertussis.
Other Bordetella species include Bordetella parapertussis, which can result in a less severe
pertussis-like illness. A number of other infections due to community respiratory viruses can
mimic pertussis, among them adenovirus, respiratory syncytial virus, Mycoplasma pneumoniae,
and Chlamydia trachomatis.

The treatment and postexposure prophylaxis antimicrobial agent of choice for pertussis is a 5-
day course of azithromycin, regardless of age. Trimethoprim-sulfamethoxazole can be used as an
alternative agent in patients older than 2 months who are not able to tolerate azithromycin or are
infected with a macrolide-resistant strain (Item C162). In addition to standard precautions,
droplet isolation for at least 5 days after initiation of azithromycin therapy is recommended for
hospitalized patients with pertussis.

Immunization remains the key strategy to prevent pertussis. In 2012, universal maternal
immunization with TdaP during each pregnancy, preferably at 27 to 36 weeks of gestation, was
recommended to prevent pertussis-associated deaths in young infants. Early diagnosis of
pertussis in conjunction with initiation of appropriate antimicrobial therapy may be associated
with lower case fatality in infants.

PREP Pearls
• Pertussis in young infants can have an atypical presentation characterized by gagging,
gasping, apnea, or bradycardia without the characteristic “whoop.”
• Children younger than 1 year have the highest reported rate of pertussis; infants younger
than 6 months are at risk of experiencing severe disease and death.
• Immunization is the key strategy to prevent pertussis.

American Academy of Pediatrics 597


PREP ® Self-Assessment PREPSA 2021
ABP Content Specifications(s)
• Recognize the clinical features associated with pertussis in children of various ages
• Understand the epidemiology of Bordetella pertussis
• Plan the appropriate diagnostic evaluation of a patient in whom pertussis is suspected
• Plan the appropriate management of pertussis in its various stages, including treatment
for contacts of infected patients

Suggested Readings
• American Academy of Pediatrics. Pertussis (whooping cough). In: Kimberlin DW, Brady
MT, Jackson MA, Long SS, eds. Red Book: 2018-2021 Report of the Committee on
Infectious Diseases. 31st ed. Elk Grove Village, IL: American Academy of Pediatrics;
2018:620-634. Red Book Online.
• Daniels HL, Sabella C. Bordetella pertussis (pertussis). Pediatr Rev. 2018;39(5):247-257.
doi:10.1542/pir.2017-0229.
• Sabella C. Pertussis (whooping cough). In: McInerny TK, Adam HM, Campbell DE,
DeWitt TG, Foy JM, Kamat DM, eds. American Academy of Pediatrics Textbook of
Pediatric Care. 2nd ed. Elk Grove Village, IL: American Academy of Pediatrics;
2017:2493-2497. Pediatric Care Online .
• Tiwari T, Murphy TV, Moran J; National Immunization Program, CDC. Recommended
antimicrobial agents for the treatment and postexposure prophylaxis of pertussis: 2005
CDC guidelines. MMWR Recomm Rep. 2005;54(RR-14):1-16.

American Academy of Pediatrics 598


PREP ® Self-Assessment PREPSA 2021
Question 163
A 4-year-old boy is seen in the clinic for evaluation of cough and eye redness. He has had nasal
congestion, mild cough, and bilateral itchy, red eyes for the past week. He has been afebrile and
otherwise well, with a normal appetite and energy. He has normal vital signs. He has mild
bilateral periorbital edema and erythema. His bulbar and palpebral conjunctiva are erythematous
but without discharge. His tympanic membranes are normal, and he has normal findings from
cardiac and respiratory examinations.

Of the following, the MOST appropriate treatment for the boy’s ocular symptoms is
A. erythromycin eye ointment
B. ketotifen eye drops
C. oral cephalexin
D. polymyxin B/trimethoprim eye drops

American Academy of Pediatrics 599


PREP ® Self-Assessment PREPSA 2021
Correct Answer: B
The boy in the vignette has symptoms of an allergic conjunctivitis. Of the response choices, the
most appropriate treatment for this condition is ketotifen eye drops, an H-1 antihistamine and
mast cell stabilizer. Topical antibiotics (eg, erythromycin eye ointment, polymyxin
B/trimethoprim eye drops) and oral antibiotics like cephalexin are inappropriate.
Conjunctivitis is a common and usually benign condition. Acute conjunctivitis can be bacterial,
viral, allergic, or nonallergic/noninfectious (Item C163).

American Academy of Pediatrics 600


PREP ® Self-Assessment PREPSA 2021

American Academy of Pediatrics 601


PREP ® Self-Assessment PREPSA 2021
The boy in the vignette has nasal congestion, mild cough, and bilateral itchy, red eyes. Although
his symptoms could be consistent with either a viral or allergic conjunctivitis, the presence of
pruritus and the absence of physical examination findings consistent with upper respiratory tract
infection make allergic conjunctivitis more likely. In addition to removal of the allergen,
treatment for allergic conjunctivitis can be with supportive measures (eg, eye lubricant, cool
compresses), a short course of vasoconstrictor/antihistamine combination drops (eg, naphazoline
and pheniramine), antihistamine drops (eg, olopatadine, ketotifen), or mast cell-stabilizing drops
(eg, cromolyn sodium).

PREP Pearls
• Patients with bacterial conjunctivitis typically have thick, purulent discharge and
symptoms may be unilateral; viral conjunctivitis typically causes bilateral injection with
watery or mucousy discharge and a sensation of “grittiness,” and patients have associated
viral symptoms; allergic conjunctivitis is usually bilateral with watery discharge and
notable pruritus.
• Allergic conjunctivitis can be treated with removal of the allergen, supportive care with
topical eye lubricants and cool compresses, short courses of antihistamine-vasoconstrictor
combination eye drops, antihistamine eye drops, or mast cell-stabilizing eye drops.

ABP Content Specifications(s)


• Differentiate the clinical findings associated with infectious conjunctivitis from those of
allergic conjunctivitis

Suggested Readings
• American Academy of Ophthalmology. Conjunctivitis. Preferred Practice Pattern -
2018. https://www.aao.org/preferred-practice-pattern/conjunctivitis-ppp-2018.
• Richards A, Guzman-Cottrill JA. Conjunctivitis. Pediatr Rev. 2010;31(5):196-208.
doi: 10.1542/pir.31-5-196.
• Sheikh A, Hurwitz B, van Schayck CP, McLean S, Nurmatov U. Antibiotics versus
placebo for acute bacterial conjunctivitis. Cochrane Database Syst
Rev. 2012;(9):CD001211. doi: 10.1002/14651858.CD001211.pub3.

American Academy of Pediatrics 602


PREP ® Self-Assessment PREPSA 2021
Question 164
A patient with sepsis is receiving two antibiotics in the pediatric intensive care unit. The
antibiotic plasma concentrations are tracked over time (Item Q164).

Item Q164: Antibiotic plasma concentrations over time. Courtesy of J Kane

Of the following, the BEST explanation for the difference in the plasma concentration curves for
the two drugs is that the patient has
A. become hypermetabolic because of exogenous catecholamine infusions
B. decreased glomerular filtration and drug B has renal metabolism
C. increased hepatic blood flow and drug B has hepatic metabolism
D. started renal replacement therapy and drug B has renal metabolism

American Academy of Pediatrics 603


PREP ® Self-Assessment PREPSA 2021

Correct Answer: B
When prescribing medications, clinicians must take into consideration the effect of hepatic or
renal impairment on drug metabolism and clearance. Similarly, as some medications can induce
or decrease the enzymatic activity that affects the metabolism of other drugs, it is important that
clinicians have an understanding of specific drug interactions that may result in higher or lower
metabolism and clearance.

Item C164 shows two situations related to medication clearance and elimination half-life. The
vertical axis shows medication plasma concentration and the horizontal axis represents time.
Drug A (the bottom blue curve) represents a steady state, in which the plasma concentration is
maintained over time. The drug B curve (the top red curve) shows drug accumulation and an
increased half-life owing to ineffective clearance mechanisms. The difference in the plasma
concentration curves for the two drugs shown in Item C164 is caused by a reduced clearance and
increased half-life of drug B compared with drug A. Of the responses listed, the best response is
decreased glomerular filtration and drug B has renal metabolism. Each of the other responses
would result in decreased drug half-life and increased clearance.

Item C164: Antibiotic plasma concentrations over time. Courtesy of J Kane

American Academy of Pediatrics 604


PREP ® Self-Assessment PREPSA 2021
Factors that decrease medications’ peak plasma concentration include poor gastrointestinal
absorption of enteral medications or increased volume of distribution. In contrast, factors that
will result in higher peak plasma concentration include decreased protein binding and decreased
clearance mechanisms. Elimination half-life is defined as the time it takes for the plasma
concentration of the drug or the total amount of drug in the body to be reduced by 50%. Factors
that increase a medication’s half-life include decreased renal or hepatic clearance (depending on
the pharmacodynamics of the drug) or a decrease in overall metabolic rate. Conversely, both the
use of renal replacement therapy or dialysis and enzymatic induction of hepatic metabolism will
decrease a medication’s half-life.

Patients with a hypermetabolic state will metabolize medications at a faster rate and thus reduce
the half-life and increase clearance. Increased hepatic blood flow with a medication that
undergoes hepatic metabolism will result in higher clearance and lower half-life. Finally, renal
replacement therapy will significantly increase clearance and reduce half-life of medications
with renal metabolism.

PREP Pearls
• Factors that will increase a medication’s half-life include decreased renal or hepatic
clearance or a decrease in overall metabolic rate.
• Both renal replacement therapy or dialysis and enzymatic induction of hepatic
metabolism will decrease a medication’s half-life.
• Elimination half-life is defined as the time it takes for the plasma concentration of the
drug or the total amount of drug in the body to be reduced by 50%.

ABP Content Specifications(s)


• Understand the circumstances that require adjustment of renally excreted antibiotic doses
in patients of various ages

Suggested Readings
• Funk RS, Brown JT, Abdel-Rahman SMl. Pediatric pharmacokinetics: human
development and drug disposition. Pediatr Clin North Am. 2012;59(5):1001-1016.
doi:10.1016/j.pcl.2012.07.003.
• Loebstein R, Koren G. Clinical pharmacology and therapeutic drug monitoring in
neonates and children. Pediatr Rev. 1998;19(12):423-428. doi:10.1542/pir.19-12-423.

American Academy of Pediatrics 605


PREP ® Self-Assessment PREPSA 2021
Question 165
Three adolescents are brought to the emergency department 2 days after attending a political
rally at which a small explosion occurred near them. None of the girls sustained any physical
injuries, so they did not seek immediate medical care. Later that evening, all three began to have
nonbloody nonbilious vomiting, diarrhea, mild headache, and fatigue, which has persisted. In the
emergency department, all three adolescents appear well and have normal vital signs. On
physical examination, each patient has mild diffuse abdominal tenderness and one appears to be
mildly dehydrated. The remainder of their physical examination findings are normal. Intravenous
lines are placed and a bolus of normal saline is administered to each adolescent. All three have a
decreased absolute lymphocyte count, but laboratory findings are otherwise normal.

Of the following, the MOST likely cause of the adolescents’ symptoms is exposure to
A. anthrax
B. phosgene gas
C. radiation
D. VX gas

American Academy of Pediatrics 606


PREP ® Self-Assessment PREPSA 2021
Correct Answer: C
The adolescents in the vignette are suffering from radiation poisoning after exposure from a
“dirty bomb” at the political rally they attended. The constellation of symptoms after whole-body
exposure to ionizing radiation is known as acute radiation syndrome (ARS). The degree of
symptoms from ARS is dependent on the total-body dose of radiation.

Acute radiation syndrome has four phases:


• prodromal: the initial manifestation of symptoms; lasts 2 to 3 days after exposure
• latent: abeyance of symptoms for several days to a month (timing is dependent on total-
body dose)
• overt illness: the recurrence of symptoms; lasts for weeks to months
• recovery or death

Acute radiation syndrome manifestations may affect the gastrointestinal system, hematopoietic
system, the central nervous system, or a combination of these. The gastrointestinal syndrome
includes nausea, vomiting, diarrhea, and abdominal pain. Symptoms begin at an exposure of 1
Grey (Gy); the severity is directly related to the total body dose. Time to onset of symptoms is
inversely related to the total-body exposure. Severe gastrointestinal complications, including
bowel necrosis and perforation, can occur with significantly higher doses of radiation.
The hematopoietic syndrome manifests at exposures of 1 Gy or higher. Affected individuals
develop lymphopenia, neutropenia, thrombocytopenia, and ultimately anemia. Lymphopenia can
occur as early as 6 hours after a large total-body dose of radiation. Absolute lymphocyte counts
after radiation exposure follow known curves; thus, the absolute lymphocyte count at a specific
interval can be used to approximate both the radiation dose and the prognosis. An absolute
lymphocyte count of less than 300/µL 48 hours after radiation exposure portends a poor
prognosis.

Central nervous system syndrome (or cerebrovascular syndrome) occurs with radiation doses
exceeding 3 to 4 Gy; symptoms include lethargy and headache. More severe symptoms, such as
cerebral edema and seizures, occur with doses greater than 10 Gy.

The cutaneous syndrome involves erythema, pruritus, pain, and blistering; there is potential for
necrosis with high doses. Cutaneous symptoms usually do not occur during the prodromal phase.

Radiation is categorized as either nonionizing or ionizing. Nonionizing radiation includes very-


low-frequency waves, radio waves, microwaves, infrared waves, visible light waves, and
ultraviolet waves. Nonionizing radiation does not have enough energy to cause acute radiation
injury. Ionizing radiation includes both nonparticulate radiation (x-rays and γ-rays) and
particulate radiation (α-rays, β-rays and neutrons). Ionizing radiation interacts with other atoms
and molecules and causes cellular damage at the molecular level. There are background levels of
ionizing radiation in nature that include cosmic radiation, solar radiation, and radon. Air travel
increases ionizing radiation exposure. Additional sources of ionizing radiation include nuclear
power and research facilities, medical imaging procedures, and therapeutic nuclear medicine.

As with all emergencies, care for the patient with radiation poisoning begins with assessment of
the airway, breathing, and circulation; however, special care must be taken to ensure the safety of
American Academy of Pediatrics 607
PREP ® Self-Assessment PREPSA 2021
providers. Patients not requiring immediate intervention should undergo decontamination before
treatment. When providing care to victims of radiation poisoning, health care workers should
don a gown, a mask, a hair cover, shoe covers, and a double layer of gloves. The multiple layers
of gloves allow for frequent changing of the outer layer while minimizing the risk of skin
exposure. Management of radiation poisoning is mostly limited to supportive care and treatment
of concomitant injuries, including thermal burns.

Anthrax presents as a cutaneous, gastrointestinal, or inhalational syndrome. Cutaneous anthrax


typically affects people who work with animals and handle animal products; it manifests as a
localized ulcer. Gastrointestinal anthrax is transmitted through ingestion of contaminated meat.
Inhalational anthrax is the most lethal form of anthrax; it can be used as a bioterrorism agent.
Inhalational anthrax infection presents with acute onset of fever and respiratory symptoms and
can be rapidly fatal.

Phosgene gas is a chemical classified as a pulmonary agent (as is chlorine gas); it primarily
affects the upper and lower respiratory tracts. VX gas is a highly toxic nerve agent that causes
anticholinergic symptoms immediately on exposure.

PREP Pearls
• Acute radiation syndrome can include gastrointestinal, neurologic, cutaneous, or
hematologic signs and symptoms or a combination thereof.
• Absolute lymphocyte counts decrease following known patterns after radiation exposure
and can be used to deduce approximate timing and doses.
• Ionizing radiation is present in nature; however, medical imaging, medical therapies, and
certain occupations can increase the risk of exposure.

ABP Content Specifications(s)


• Recognize the clinical presentation of radiation exposure and the risk factors (including
medical imaging) for such exposure

Suggested Readings
• Christensen DM, Iddins CJ, Sugarman SL. Ionizing radiation injuries and
illnesses. Emerg Med Clin North Am. 2014;32(1):245-265.
doi:10.1016/j.emc.2013.10.002.
• Levin TL. Pediatric imaging: radiation exposure and how we image. Pediatr
Rev. 2018;39(1):50-52. doi:10.1542/pir.2017-0181.
• Rella JG. Radiation. In: Nelson LS, Howland MA, Lewin NW, Smith SA, Goldfrank LR,
Hoffman RS, eds. Goldfrank’s Toxicologic Emergencies. 11th ed. New York, NY:
McGraw-Hill; 2019:1762-1773.
• Seeyave DM, Brown KM. Environmental, emergencies, radiological emergencies, bites
& stings. In: Bachur RG, Shaw KN, eds. Fleisher & Ludwig’s Textbook of Pediatric
Emergency Medicine. 7th ed. Philadelphia, PA: LWW; 2015:718-760.
• US Department of Health and Human Services. Radiation emergency medical
management. https://www.remm.nlm.gov/index.html#.

American Academy of Pediatrics 608


PREP ® Self-Assessment PREPSA 2021
Question 166
A 2-year-old boy is brought to the emergency department for evaluation of rectal bleeding. He
has a 1-month history of intermittent rectal prolapse, which occurs every 2 to 3 days and is easily
reducible at home, with no rectal bleeding noted. His first episode of prolapse was easily reduced
by a physician in the emergency department. He is an otherwise healthy boy with normal growth,
who is not yet toilet trained. He has 1 to 2 stools daily which his parents describe as soft blobs.
He has never had a history of infrequent or hard stools. Because of his history of prolapse, the
boy has been taking polyethylene glycol 3350 daily for the last month. Physical examination
reveals a hemodynamically stable child. Digital rectal examination finds dark red blood and a
palpable rectal mass; the remainder of his examination findings are normal. Laboratory
evaluation reveals a normal hemoglobin level for age.

Of the following, the test MOST likely to confirm the etiology of this boy’s condition is
A. colonic manometry
B. colonoscopy
C. Meckel scan
D. sweat chloride

American Academy of Pediatrics 609


PREP ® Self-Assessment PREPSA 2021
Correct Answer: B
The child in the vignette has recurrent rectal prolapse without a history of constipation or cystic
fibrosis. Given the finding of a palpable rectal mass, the most likely cause of his recurrent rectal
prolapse and rectal bleeding would be a rectal polyp. This could be confirmed on colonoscopy.

Rectal prolapse occurs most commonly in children younger than 4 years because of normal
anatomic variants in this age group. Children older than 4 years are more likely to have a
pathologic condition contributing to prolapse. Constipation is the most common cause of rectal
prolapse in children, as a result of increased intra-abdominal pressure caused by prolonged
sitting and straining during attempts to pass stools. Chronic severe cough, because of cystic
fibrosis or other conditions, may also cause rectal prolapse related to increased intra-abdominal
pressure. Rectal polyps can act as a lead point inducing intussusception, thus resulting in rectal
prolapse. Hirschsprung disease, anorectal anomalies, ulcerative colitis, and infectious colitis can
also cause rectal prolapse.

In the acute setting, if the rectal prolapse does not resolve spontaneously, manual reduction
should occur. If this is unsuccessful, an urgent surgical consultation is needed. After the prolapse
resolves, management should be directed toward treating the underlying condition. In cases of
constipation, the most common cause of rectal prolapse, laxative therapy should begin, as well as
limiting toilet sitting to 5 to 10 minutes, using a child insert on the toilet seat, and using a 2-step
stool to raise the feet to promote complete stool evacuation.

Colonic manometry may be useful to evaluate for Hirschsprung disease or other causes of
intractable constipation. However, this procedure would not be indicated for the child in the
vignette because he does not have a clinical history that supports constipation. A Meckel scan is
not indicated at this time, because the palpable rectal mass makes a polyp the more likely cause
of this child’s bleeding. Sweat chloride testing is an essential tool to diagnose cystic fibrosis.
However, without pulmonary or gastrointestinal signs or symptoms of cystic fibrosis and a
palpable rectal mass, sweat chloride testing would not be the next best test for this patient.

PREP Pearls
• Constipation is the most common cause of rectal prolapse in children.
• Conditions including cystic fibrosis, anorectal anomalies, Hirschsprung disease, rectal
polyp, and colitis should be considered when evaluating rectal prolapse.
• The treatment for acute rectal prolapse is aimed at reducing the prolapse; the underlying
cause should then be sought and treated.

ABP Content Specifications(s)


• Identify the clinical conditions other than cystic fibrosis that are associated with rectal
prolapse

American Academy of Pediatrics 610


PREP ® Self-Assessment PREPSA 2021
Suggested Readings
• Cares K, El-Baba M. Rectal prolapse in children: significance and management. Curr
Gastroenterol Rep. 2016;18(5):22. doi: 10.1007/s11894-016-0496-y.
• Colombo JM, Wassom MC, Rosen JM. Constipation and encopresis in
childhood. Pediatr Rev. 2015;36(9):392-402. doi: 10.1542/pir.36-9-392.
• Levitt M, Garza JM, Pena A, Lawal T. Colorectal disorders. In: McInerny TK, Adam
HM, Campbell DE, DeWitt TG, Foy JM, Kamat DM, eds. American Academy of
Pediatrics Textbook of Pediatric Care. 2nd ed. Itasca, IL: American Academy of
Pediatrics; 2016;chap 232:1870-1880. Pediatric Care Online.

American Academy of Pediatrics 611


PREP ® Self-Assessment PREPSA 2021
Question 167
A neonate is being evaluated 8 hours after birth for decreased perfusion. He was born at 38
weeks' gestation. His mother is a 33-year-old woman with a history of poorly controlled
gestational diabetes. Her laboratory testing is significant for group B Streptococcus–negative
status and hemoglobin A1c of 9.3% at 36 weeks’ gestation. The neonate has breastfed twice. His
vital signs show a heart rate of 205 beats/min, respiratory rate of 35 breaths/min, blood pressure
of 45/23 mm Hg, and temperature of 37.2°C. On physical examination, he appears plethoric, has
a 1/6 systolic murmur at the left lower sternal border, and +1 peripheral pulses.
Echocardiography reveals asymmetric septal hypertrophy.

Of the following, the pharmacologic therapy MOST appropriate for this neonate is
A. adenosine
B. digoxin
C. dopamine
D. propranolol

American Academy of Pediatrics 612


PREP ® Self-Assessment PREPSA 2021
Correct Answer: D
Neonates born to mothers with poorly controlled diabetes are at risk for a number of cardiac
complications. In utero, because of maternal hyperglycemia, the fetus produces extra insulin. The
elevated glucose and insulin levels interrupt normal cardiac development, increasing the risk for
congenital heart disease. Cardiomegaly may be seen in up to 30% of neonates born to mothers
with diabetes. Far fewer develop heart failure (5%-10%). These neonates are also at risk for
asymmetric hypertrophy of the intraventricular septum. The pathogenesis is thought to be related
to glycogen loading after hyperglycemia and hyperinsulinemia. Functionally, these neonates
have decreased cardiac output, as seen with the neonate in this vignette; they may require a
normal saline bolus to maintain adequate preload. Some will need pharmacologic treatment with
a β-blocker such as propranolol. Asymmetric septal hypertrophy resolves over time.

Adenosine would be used to treat supraventricular tachycardia; this diagnosis is unlikely in a


neonate with a heart rate of 205 beats/min. Digoxin is not the best treatment for asymmetric
septal hypertrophy. Inotropes such as dopamine would worsen this neonate’s functional
obstruction and should be avoided.

Most neonates who are born to mothers with poorly controlled diabetes develop hypoglycemia.
The clinical presentation should be classified as asymptomatic or symptomatic (eg, jitteriness,
hypotonia, or irritability). Symptomatic neonates and asymptomatic neonates with persistently
low glucose values may require intravenous dextrose infusion to maintain euglycemia. This
hypoglycemia is self-limited and typically resolves within 2 to 3 days after birth. Neonates born
to mothers with poorly controlled diabetes also have an increased rate of polycythemia and
respiratory distress syndrome. In addition, they may be large for gestational age, leading to
higher risk of brachial plexus injury.

PREP Pearls
• Neonates born to mothers with poorly controlled diabetes are at risk for cardiomegaly,
asymmetric septal hypertrophy, and heart failure.
• Neonates with asymmetric septal hypertrophy may develop dynamic outlet obstruction
with decreased cardiac output.

ABP Content Specifications(s)


• Recognize the clinical and laboratory features in an infant of a diabetic mother, and
manage appropriately

Suggested Readings
• Nafday SM. Abnormalities of fetal growth. In: McInerny TK, Adam HM, Campbell DE,
DeWitt TG, Foy JM, Kamat DM, eds. American Academy of Pediatrics Textbook of
Pediatric Care. 2nd ed. Itasca, IL: American Academy of Pediatrics; 2016;chap 98:847-
857. Pediatric Care Online.
• Sheanon NM, Muglia LJ. The endocrine system. In: Kliegman R, St Geme JW, Blum NJ,
Shah SS, Tasker RC, Wilson K, eds. Nelson Textbook of Pediatrics. Philadelphia, PA:
Elsevier; 2020:982-985.e1.

American Academy of Pediatrics 613


PREP ® Self-Assessment PREPSA 2021
Question 168
A 1-day-old neonate is admitted to the neonatal intensive care unit for persistent hypoglycemia.
He was born at 40 weeks’ gestation by cesarean delivery because of failure of labor progression.
The pregnancy was otherwise uncomplicated. His birth weight was 3400 g. Jitteriness noted at 8
hours after birth prompted measurement of a plasma glucose level, which was 28 mg/dL (1.6
mmol/L). Despite treatment with oral feedings of standard infant formula, the neonate was
unable to maintain a plasma glucose level above 50 mg/dL (2.8 mmol/L). He is requiring a
glucose infusion rate of 8 mg/kg/min to maintain normal glucose levels. He is afebrile and has
normal vital signs for age. Physical examination findings are normal except for the genital
examination. His right testicle is not palpable, and his left testicle is palpable high in the scrotum.
The stretched phallic length is 1.6 cm.

Of the following, the MOST likely etiology of this neonate’s hypoglycemia is


A. a fatty acid oxidation disorder
B. a glycogen storage disease
C. hyperinsulinism
D. hypopituitarism

American Academy of Pediatrics 614


PREP ® Self-Assessment PREPSA 2021
Correct Answer: D
The neonate described in the vignette has hypopituitarism. His small penis size and undescended
testicle with the second testicle found high in the scrotum are manifestations of hypogonadism
due to gonadotropin deficiency. Hypogonadism associated with persistent hypoglycemia is
consistent with hypopituitarism. His hypoglycemia is due to other anterior pituitary hormone
deficiencies (adrenocorticotropic hormone, growth hormone). Failure of labor progression is
another feature of hypopituitarism.

Hypoglycemia is a common presenting symptom of congenital hypopituitarism. Other symptoms


include postdates gestation, breech presentation, poor feeding, low weight, hypotonia, and
cholestatic jaundice. Septo-optic dysplasia (optic nerve hypoplasia, absent septum pellucidum) is
a common cause of congenital hypopituitarism. The optic nerve hypoplasia can cause nystagmus
and may be associated with midline defects. If the posterior pituitary hormone vasopressin is
deficient, diabetes insipidus results.

Males with hypopituitarism are often born with a small penis, cryptorchidism, or both.
Micropenis is defined as a normally formed penis with a stretched length that is more than 2.5
standard deviations below the mean for age. Stretched phallic length is measured from the base
of the symphysis pubis to the tip of the glans with gentle tension applied. A stretched phallic
length of less than 2 to 2.5 cm for a term neonate meets criteria for micropenis. The neonate in
the vignette, with a stretched phallic length of 1.6 cm, meets this definition.

Micropenis occurs owing to a deficiency of fetal testosterone later in gestation, when


testosterone-dependent phallic growth and testicular descent occurs. Testosterone, with local
conversion to dihydrotestosterone, is necessary for differentiation of the male genitalia during
the first trimester. Fetal testosterone production during the first trimester is controlled by
placental human chorionic gonadotropin acting via the fetal testicular luteinizing hormone
receptor. Therefore, when there is a fetal gonadotropin (luteinizing hormone) deficiency,
differentiation of the male genitalia during the first trimester remains normal. Later in gestation,
lack of fetal gonadotropins results in decreased testosterone production and poor growth of the
male genitalia. Thus, micropenis without other genital ambiguity is often due to gonadotropin
deficiency. Gonadotropin deficiency may be isolated (eg, Kallmann syndrome) or associated
with other pituitary hormone deficiencies. Inadequate testosterone production can also occur near
the end of gestation owing to primary testicular dysfunction (as in Klinefelter syndrome),
resulting in micropenis.

Neonates with micropenis should undergo evaluation in the immediate newborn period to detect
and treat potentially life-threatening conditions. If genital ambiguity is present, the infant should
be evaluated for disorders of sex development and monitored to prevent any associated adrenal
crisis. Infants with micropenis who have no other genital ambiguity should be monitored for
hypoglycemia and evaluated for other pituitary hormone deficiencies.

A fatty acid oxidation disorder, glycogen storage disease, and hyperinsulinism are less likely
etiologies of hypoglycemia for the neonate in the vignette. Fatty acid oxidation disorders and
glycogen storage disease are not associated with micropenis. Glycogen storage disease is usually

American Academy of Pediatrics 615


PREP ® Self-Assessment PREPSA 2021
associated with hepatomegaly. Neonates with hyperinsulinism are often large for gestational age
and require higher glucose infusion rates (>10 mg/kg/min).

PREP Pearls
• Hypoglycemia is a common presenting symptom of congenital hypopituitarism.
• Gonadotropin deficiency is high on the differential diagnosis of micropenis and can be
isolated or associated with other pituitary hormone deficiencies.
• Neonates with micropenis should undergo evaluation in the immediate newborn period to
detect and treat hypoglycemia, adrenal crisis, and other consequences of hypopituitarism.

MOCA-Peds Objective
• Recognize and plan initial evaluation of a child with a pituitary disorder.

ABP Content Specifications(s)


• Understand the clinical diagnosis of micropenis

Suggested Readings
• Hatipoglu N, Kurtoglu S. Micropenis: etiology, diagnosis and treatment approaches. J
Clin Res Pediatr Endocrinol. 2013;5(4):217-223. doi:10.4274/Jcrpe.1135.
• Jack J, Young SB. Endocrinology. In: Hughes HK, Kahl LK, eds. The Harriet Lane
Handbook. 21st ed. Philadelphia, PA: Elsevier; 2018:255-289.
• Pierce M, Madison L. Evaluation and initial management of hypopituitarism (published
correction appears in Pediatr Rev. 2016;37[10]:441). Pediatr Rev. 2016;37(9):370-376.
doi:10.1542/pir.2015-0081.
• Suresh S, Santhanam I. Hypoglycemia. In: McInerny TK, Adam HM, Campbell DE,
DeWitt TG, Foy JM, Kamat DM, eds. American Academy of Pediatrics Textbook of
Pediatric Care. 2nd ed. Elk Grove Village, IL: American Academy of Pediatrics;
2017:2881-2887. Pediatric Care Online.
• Thompson-Branch A, Havranek T. Neonatal hypoglycemia. Pediatr Rev.
2017;38(4):147-157. doi:10.1542/pir.2016-0063.

American Academy of Pediatrics 616


PREP ® Self-Assessment PREPSA 2021
Question 169
A study is conducted in a tertiary care center to examine the relationship between morbidity in
children with cystic fibrosis and annual household income. Results showed that children of
families with incomes below the poverty line were 5 times as likely to have cystic fibrosis–
related morbidities compared with children of families whose incomes were above the poverty
line. An attempt was then made to increase access to health care for children living in households
with incomes below the poverty line. A social worker was assigned to each family and provided
gas vouchers and mileage and taxi reimbursement to the parents of patients whose incomes were
below the poverty line.

Of the following, the type of bias that MOST accurately describes what was corrected in the
study is
A. confounding bias
B. interviewer bias
C. recall bias
D. selection bias

American Academy of Pediatrics 617


PREP ® Self-Assessment PREPSA 2021
Correct Answer: A
Bias is a type of error in a research study that has the potential to affect outcome. The type of
bias that was corrected in the study described in the vignette is called “confounding bias.”
Confounding bias occurs when extraneous factors or variables are present in the study that can
distort the association between exposure and outcome. The confounding bias in the study
described in the vignette was access to health care, which is shown to be more limited in children
living in households with incomes below the poverty line versus above it. Decreased health care
access leads to low compliance for acute-care and routine-care visits in patients with chronic
medical conditions such as cystic fibrosis. Therefore, the bias has the potential to affect the
results of the study conducted by the tertiary care center by influencing morbidity. Confounding
can be addressed in a study design by using techniques such as restriction, matching, and
randomization. Based on the results of the study described in the vignette, the confounding bias
was identified and then addressed by providing participants with resources to help mitigate the
cost of transportation to the care center and thereby increasing access to medical care.

Interviewer bias occurs when the information for the study is obtained or interpreted via an
interview or a chart review. This is more likely when the disease classification is known to the
interviewer. Interviewer bias is commonly addressed by using a standard protocol for interview,
blinding the interviewer to the participants’ exposure status, or by using interviewers who are not
involved in the disease or test determination. Recall bias occurs when exposure is incorrectly
reported for the outcome. Participants are asked to recall past events that can be influenced by
subsequent events or experiences. An example of recall bias is the link between autism spectrum
disorder and administration of MMR vaccine. After publication of an article in the
journal Lancet in 1998 (Wakefield et al, later retracted), parents of children with autism spectrum
disorder were more likely to recall that symptoms occurred soon after administration of MMR
vaccine in contrast to reports before the publication. Recall bias can be corrected by performing a
prospective study in which validated scales for obtaining data are used, and by comparing the
medical record with patient-reported subjective data. Selection bias occurs when the study
population is initially being determined. Bias can occur if the participants differ from the
population of interest and appropriate randomization is not performed. An example of selection
bias would be if the case or control participants are selected from members of a hospitalized
population, because they would have different exposures than the general population and,
therefore, conclusions from the study could not be generalized. Selection bias can be corrected
by randomization, performing a prospective study, and using strict criteria to select participants.

PREP Pearls
• Bias is a type of error that has a potential impact on the outcome of a research study.
• It is important for a researcher to identify and correct for any potential bias that can occur
during planning, implementation, and data analysis of a study.

ABP Content Specifications(s)


• Understand confounding and how to control for it in a study
• Understand bias and how it might distort the estimate of the association between
exposure and outcome

American Academy of Pediatrics 618


PREP ® Self-Assessment PREPSA 2021
Suggested Readings
• Jennings JM, Sibinga E. Research and statistics: understanding and identifying bias in
research studies. Pediatr Rev. 2010;31(4):161-162. doi:10.1542/pir.31-4-161.
• Stone JC, Glass K, Clark J, Munn Z, Tugwell P, Doi SAR. A unified framework for bias
assessment in clinical research. Int J Evid Based Healthc. 2019 May 10.
doi:10.1097/XEB.0000000000000165.

American Academy of Pediatrics 619


PREP ® Self-Assessment PREPSA 2021
Question 170
A 5-year-old boy is seen in the clinic for tick removal. He was hiking in the woods in Colorado
with his parents today when they noticed a tick partially embedded in his calf. They brought him
promptly to the clinic.

Of the following, the BEST next step in management is to


A. apply an extinguished hot match to the tick and prescribe prophylactic amoxicillin
B. apply isopropyl alcohol to the tick and prescribe prophylactic doxycycline
C. grasp the tick close to the skin and firmly pull up; prescribe prophylactic doxycycline
D. grasp the tick close to the skin and firmly pull up; provide no additional therapy

American Academy of Pediatrics 620


PREP ® Self-Assessment PREPSA 2021
Correct Answer: D
Ticks should be removed immediately by using fine-tipped forceps or tweezers to pull steadily
upwards from their point of attachment as close to the skin as possible. Care must be taken to
avoid twisting or crushing the tick, because this may break the arachnid, leaving the head
embedded in the skin. If this occurs, the head will be spontaneously expelled over time (as with
any foreign body). Several methods of tick removal have been shown to be ineffective.
Petroleum jelly, nail polish, isopropyl alcohol, and other occlusive substances have been
proposed to induce detachment by suffocating the tick. These methods are not successful,
because ticks have a very low respiratory rate and it is difficult to create a completely occlusive
barrier. Ticks should not be burnt with a match. This method of tick removal carries a high risk
of injury to the child, especially in uncooperative children, and has not been shown to induce
detachment. The risk of contracting Lyme disease is low if a tick is attached for less than 24 to
36 hours; ticks, therefore, should be removed promptly.

According to the Red Book: 2018 -2020 Report of the Committee on Infectious Diseases, “In
areas of high endemicity (the coastal northeast), where 30% to 50% of Ixodes scapularis ticks
harbor Borrelia burgdorferi, the risk of Lyme disease is no higher than 3%, even after high-risk
deer tick bites. The risk is extremely low after brief attachment (eg, a flat, nonengorged deer tick
is found) and is higher after engorgement, especially if a nymphal deer tick has been attached for
36 hours or more. Testing of the tick for spirochete infection has a poor predictive value and is
not recommended. In areas of high risk, a single prophylactic 200-mg dose (4 mg/kg for patients
weighing less than 45 kg) of doxycycline can reduce the risk of acquiring Lyme disease after the
bite of an I scapularis tick. Benefits of prophylaxis likely outweigh risks when the tick is
engorged (or has been attached for at least 36 hours based on exposure history) and prophylaxis
can be started within 72 hours of tick removal. Amoxicillin prophylaxis has been insufficiently
studied, but it would likely require a full course because of its short half-life, which in turn
would increase the likelihood of toxicity.” The Centers for Disease Control and Prevention has
revised the recommendations regarding the use of doxycycline in children younger than 8 years
and indicates that a short course (≤ 21 days) may be used in this age group, given the unlikely
risk of dental staining or enamel hypoplasia. Readers are advised to consult the Centers for
Disease Control and Prevention and Red Book updates for any changes to these
recommendations.

PREP Pearls
• Ticks should be removed immediately by using fine-tipped forceps or tweezers to pull
steadily upwards from the point of attachment as close to the skin as possible.
• In areas of high risk for Lyme disease, a single prophylactic dose of doxycycline can
reduce the risk of acquiring the disease after the bite of an Ixodes scapularis tick.
• Benefits of prophylaxis likely outweigh risks when the tick has been attached for at least
36 hours and prophylaxis can be started within 72 hours of tick removal.
• The Centers for Disease Control and Prevention has recently changed its
recommendations to allow the use of doxycycline prophylaxis in children younger than 8
years.

American Academy of Pediatrics 621


PREP ® Self-Assessment PREPSA 2021
ABP Content Specifications(s)
• Advise parents regarding the appropriate method to remove an attached tick

Suggested Readings
• American Academy of Pediatrics. Prevention of mosquitoborne and tickborne infections:
tick inspection and removal. In: Kimberlin DW, Brady MT, Jackson MA, Long SS,
eds. Red Book: 2018-2021 Report of the Committee on Infectious Diseases. 31st ed.
Itasca, IL: American Academy of Pediatrics; 2018:200. Red Book Online.
• Needham GR. Evaluation of five popular methods for tick removal. Pediatrics.
1985;75(6):997-1002. https://pediatrics.aappublications.org/content/75/6/997.
• Read JS. Tickborne diseases in children in the United States. Pediatr Rev.
2019;40(8):381-397. doi: 10.1542/pir.2018-0304 .
• Wormser GP, Dattwyler RJ, Shapiro ED, et al. The clinical assessment, treatment, and
prevention of lyme disease, human granulocytic anaplasmosis, and babesiosis: clinical
practice guidelines by the Infectious Diseases Society of America. Clin Infect
Dis. 2006;43(9):1089-1134. doi:10.1086/508667.

American Academy of Pediatrics 622


PREP ® Self-Assessment PREPSA 2021
Question 171
An 8-year-old is seen in the pediatric clinic for nausea and vomiting. While at school today, he
vomited 3 times. The emesis was yellow and did not contain blood. He has had abdominal
cramping and 1 watery stool. Earlier in the day he attended a breakfast at school where he
consumed yogurt, granola, and pancakes. Several other classmates who attended the breakfast
have similar symptoms. His vital signs are a temperature of 37.2°C, heart rate of 85 beats/min,
respiratory rate of 16 breaths/min, and blood pressure of 98/60 mm Hg. Physical examination
reveals increased bowel sounds and mild abdominal tenderness.

Of the following, the pathogen MOST likely to be responsible for his illness is
A. Campylobacter species
B. Listeria monocytogenes
C. Staphylococcus aureus
D. Yersinia enterocolitica

American Academy of Pediatrics 623


PREP ® Self-Assessment PREPSA 2021
Correct Answer: C
The pathogen most likely to be responsible for the illness in the boy in the vignette
is Staphylococcus aureus. Ingestion of S aureus enterotoxin usually manifests in an abrupt onset
of nausea and vomiting lasting 1 to 2 days. Diarrhea can occur. The typical incubation period for
staphylococcal foodborne illness is 2 to 4 hours. The short incubation period distinguishes this
illness from many other foodborne organisms.

Foodborne outbreaks of Staphylococcus aureus have occurred in restaurants, hotels, and cruises.
Foods typically become contaminated by a food handler with S aureus colonization/infection of
the hands or nasopharynx. Implicated foods include meats (including poultry) and milk- or egg-
based products.

The typical incubation period for Campylobacter infection is 2 to 5 days. Infection


with Campylobacter species typically manifests as moderate to severe abdominal pain, bloody
diarrhea, and fever. Invasive infections, including bacteremia, are uncommon. Immunoreactive
complications, such as Guillain-Barré syndrome and reactive arthritis, have been associated
with Campylobacter infection. The principal reservoir for Campylobacter is domestic and wild
birds. Risk factors associated with Campylobacter species infections include exposure to
foodborne outbreaks in restaurants, hotels, and cruises; consumption of unpasteurized dairy
products or undercooked poultry; swimming in or drinking untreated fresh water; travel to a
resource-limited country; and visiting a farm or petting zoo.

The incubation period for Listeria gastroenteritis is 24 hours. Listeria infections can manifest as
gastroenteritis, bacteremia, or meningitis. Severe manifestations occur at the extremes of age, in
pregnant women and their infants, and in individuals with impaired cell-mediated
immunity. Listeria organisms have the ability to grow well at refrigerator temperatures, and thus
have been involved in foodborne outbreaks involving refrigerated items. Foods implicated
include deli meats, unpasteurized milk, soft cheeses, and fresh and frozen fruits and vegetables.

The typical incubation period for Yersinia infections is 4 to 6 days. The clinical manifestations
are similar to those of Campylobacter infections. Yersinia enterocolitica infections can mimic
appendicitis. Diarrhea can be prolonged, sometimes lasting for more than 2 weeks. Invasive
infections include bacteremia, osteomyelitis, and meningitis. The main reservoir for Yersinia is
swine. Foods associated with Y enterocolitica infections include unpasteurized dairy products
and undercooked pork (especially chitterlings).

American Academy of Pediatrics 624


PREP ® Self-Assessment PREPSA 2021
PREP Pearls
• The short incubation period of 2 to 4 hours for staphylococcal foodborne illness
distinguishes it from many other foodborne infections.
• Risk factors associated with food and waterborne infection outbreaks include consuming
food in restaurants, hotels, and on cruises; consumption of unpasteurized dairy products
or undercooked poultry; swimming in or drinking untreated fresh water; travel to a
resource-limited country; and visiting a farm or petting zoo.
• Infection with staphylococcal enterotoxins typically manifests as nausea and vomiting,
whereas Campylobacter and Yersinia species infections typically manifest as fever,
moderate to severe abdominal pain, and bloody diarrhea.

ABP Content Specifications(s)


• Know the toxic substances that may contaminate food sources (eg, mercury, Escherichia
coli)
• Know the contaminants potentially found in drinking water (eg, mercury, Escherichia
coli)
• Recognize which pathogens can be transmitted by contaminated recreational water, while
providing guidance about prevention of such infections

Suggested Readings
• American Academy of Pediatrics. Prevention of illnesses associated with recreational
water use. In: Kimberlin DW, Brady MT, Jackson MA, Long SS, eds. Red Book: 2018-
2021 Report of the Committee on Infectious Diseases. Itasca, IL: American Academy of
Pediatrics; 2018:201-204. Red Book Online .
• Shane A, Mody R, Crump J, et al. 2017 Infectious DIseases Society of America clinical
practice guidelines for the diagnosis and management of infectious diarrhea. Clin Infect
Dis. 2017;65(12):e45-80.

American Academy of Pediatrics 625


PREP ® Self-Assessment PREPSA 2021
Question 172
A 15-year-old adolescent boy is being evaluated for left knee pain of 4 months’ duration. The
pain began while playing on 2 different basketball teams, but he denies a specific injury. The
pain is primarily anterior and medial to the patella, and worsens during and after activity. Several
weeks ago he developed intermittent left knee swelling and reports that the knee will
occasionally “get stuck” in a given position. He has no other joint complaints and is otherwise
healthy. Findings on physical examination of his left knee include a mild effusion, mild
tenderness over the anterior medial femoral condyle, and pain with flexion greater than 90
degrees and with resisted extension. The knee is stable to valgus and varus stress testing;
Lachman and posterior drawer test results are negative. The adolescent’s gait is antalgic. He feels
pain with attempts to squat and jump.

Of the following, the BEST next management step for this boy is to
A. obtain a complete blood cell count and C-reactive protein level
B. prescribe crutches and order radiography of the knee
C. provide a compression wrap and prescribe physical therapy
D. refer to rheumatology for aspiration and synovial fluid analysis

American Academy of Pediatrics 626


PREP ® Self-Assessment PREPSA 2021
Correct Answer: B
The adolescent in the vignette should undergo imaging of his knee and use crutches for
ambulation. Although knee pain is common in young athletes, development of an effusion and
mechanical symptoms is suggestive of significant internal derangement and warrants additional
evaluation. The differential diagnosis for this scenario includes osteochondritis dissecans (OCD),
symptomatic discoid meniscus, meniscal tear, and plica syndrome. Of these, OCD has the
highest risk for permanent damage to the knee, and needs to be ruled out before making a
treatment plan.

Osteochondritis dissecans is a derangement of the articular cartilage and subchondral bone that
typically presents between 13 and 21 years of age. It most commonly involves the lateral aspect
of the medial femoral condyle. Although a direct cause is often not discernible, OCD can result
from various insults that lead to loss of blood flow and subsequent necrosis of the subchondral
bone. The overlying articular cartilage softens and is initially intact. However, with disease
progression, a cleavage plane develops, with separation of the injured tissue from the underlying
bone. In more advanced cases, the cartilage-containing fragment detaches and becomes a loose
body in the joint, which may cause the joint to lock, as described by the boy in this vignette. The
early presentation of OCD may be similar to patellofemoral syndrome. Initial symptoms usually
include vague anterior knee pain that worsens with activity. However, the presence of an
effusion and joint locking should prompt the provider to obtain imaging studies that include
anteroposterior, lateral, patellar, and tunnel view radiographs. A typical OCD lesion is shown
in Item C172A. Early OCD lesions are often radiographically occult. In cases with a high index
of suspicion for OCD, magnetic resonance imaging is warranted regardless of the radiography
result; some providers may elect to forego radiography.

American Academy of Pediatrics 627


PREP ® Self-Assessment PREPSA 2021
Item C172A: Radiograph and MRI of osteochondritis dissecans of the medial femoral condyle.
Courtesy of Atanda A, Reddy D, Rice JA, Terry MA. Injuries and chronic conditions of the knee
in young athletes. Pediatr Rev. 2009;30(11):424.

Magnetic resonance imaging is important for the detection and staging of OCD lesions.
Treatment relies on staging as described in Item C172B. Osteochondritis dissecans stages 1 and
2 are considered stable lesions and can be treated conservatively (Item C172C). Osteochondritis
dissecans stages 3 to 5 are considered unstable and should be sent for orthopedic evaluation.

American Academy of Pediatrics 628


PREP ® Self-Assessment PREPSA 2021

American Academy of Pediatrics 629


PREP ® Self-Assessment PREPSA 2021

Discoid menisci are anatomic variants that can cause knee swelling and locking. These are far
more common in the lateral meniscus, and are bilateral in about 20% of patients. Studies report a
prevalence of discoid menisci in the United States of 3% to 5%; rates are 2 to 3 times higher in
Japan and Korea. Although these are commonly asymptomatic incidental findings, discoid
menisci are prone to atraumatic tears. Other meniscal tears in children and adolescents are
typically associated with a traumatic event. Magnetic resonance imaging is the best imaging
modality to assess for discoid menisci or meniscal tears.

Plicae are embryologic remnants that create folds of synovial tissue surrounding the knee. Most
remain asymptomatic, but synovial plica syndrome (SPS) may occur when the medial
peripatellar plica thickens and is irritated when impinged between the patellar tendon and medial
femoral condyle. Most cases of SPS result in relatively mild peripatellar clicking or catching, and
swelling is usually minimal, but more severe cases can produce symptoms very similar to those
described by the boy in the vignette. In SPS, the thickened fold is tender on physical examination
and knee flexion/extension is painful. Physical therapy is the cornerstone of treatment for SPS;
however, until a diagnosis is made, providing a compression wrap and prescribing physical
therapy would not be appropriate.

Knee pain with effusion can also result from an infectious or inflammatory process. These are
typically acute in onset and may have systemic or multijoint involvement. Any child or
adolescent with atraumatic knee swelling accompanied by systemic symptoms should undergo
testing, which may include complete blood cell count, C-reactive protein, erythrocyte
sedimentation rate, antibody testing for Lyme disease, and joint aspiration with synovial fluid
analysis. However, in this case, the presence of mechanical symptoms are more suggestive of
orthopedic pathology, which should be ruled out before evaluating for systemic causes.

PREP Pearls
• Imaging studies are warranted for pediatric patients with a knee effusion.
• Osteochondritis dissecans must be ruled out in a child with joint swelling and painful
mechanical symptoms (eg, locking, catching).

MOCA-Peds Objective
• Evaluate and manage the adolescent with syncope.

ABP Content Specifications(s)


• Recognize the clinical findings associated with sports-related patellofemoral dysfunction,
and manage appropriately
• Recognize the clinical findings associated with sports-related internal derangement of the
knee
• Identify risk factors associated with sports-related patellofemoral dysfunction
• Understand when orthopedic consultation is required for a sports-related knee injury

American Academy of Pediatrics 630


PREP ® Self-Assessment PREPSA 2021
Suggested Readings
• Kannikeswaran N, Suresh S. Sports musculoskeletal injuries. In: McInerny TK, Adam
HM, Campbell DE, DeWitt TG, Foy JM, Kamat DM, eds. American Academy of
Pediatrics Textbook of Pediatric Care. 2nd ed. Elk Grove Village, IL: American
Academy of Pediatrics; 2016;chap 334:2675-2685. Pediatric Care Online.
• Wolf M. Knee pain in children, part I: evaluation. Pediatr Rev. 2016;37(1):18-24.
doi: 10.1542/pir.2015-0040.
• Wolf M. Knee pain in children, part II: limb- and life-threatening conditions, hip
pathology, and effusion. Pediatr Rev. 2016;37(2):72-77. doi: 10.1542/pir.2015-0041.
• Wolf M. Knee pain in children, part III: stress injuries, benign bone tumors, growing
pains. Pediatr Rev. 2016;37(3):114-119. doi: 10.1542/pir.2015-0042.

American Academy of Pediatrics 631


PREP ® Self-Assessment PREPSA 2021
Question 173
A 9-year-old boy is brought to the emergency department after experiencing a syncopal episode
while playing soccer. He was running down the field when he passed out. He had regained
consciousness by the time his coach ran to his side. He is otherwise healthy, and his vital signs
are normal. He is awake, alert, and following commands. His physical examination findings are
unremarkable. His electrocardiogram is shown in Item Q173.

Item Q173: Electrocardiogram for the patient described in the vignette.


Courtesy of M McBride

Of the following, the MOST likely diagnosis for this patient is


A. atrioventricular block
B. prolonged QTc
C. right bundle branch block
D. sick sinus syndrome

American Academy of Pediatrics 632


PREP ® Self-Assessment PREPSA 2021
Correct Answer: B
The patient in the vignette has syncope with exertion, which raises concern about a cardiac
etiology. His electrocardiogram demonstrates sinus rhythm with a prolonged QTc interval. There
is no evidence of a right bundle branch block, sick sinus syndrome, or an atrioventricular block.
The QTc interval is measured from the start of the QRS complex to the end of the T wave. The
QT interval must be corrected to the heart rate; this is done using Bazett’s formula, according to
which the measured QT interval is divided by the square root of the preceding R-R interval. The
normal QTc interval is less than 440 ms but is variable according to age and sex. Long QT
syndrome can be acquired (via electrolyte abnormalities, drugs, or underlying medical disorders)
or inherited.

Those with acquired or inherited long QT syndrome can initially demonstrate syncope, seizures,
palpitations, or sudden death. They are at risk of developing torsades de pointes (translated as
“twisting of the points”), a polymorphic ventricular tachycardia. Many patients will experience
symptoms after an adrenergic arousal such as experienced with exercise or hearing an abrupt
noise. It is advisable to refer patients with long QT syndrome to a pediatric cardiologist, who can
recommend appropriate treatment and monitoring. These patients should seek genetic
counseling, and their family members should undergo screening.

PREP Pearls
• The QT interval—the start of the QRS complex to the end of the T wave—must be
corrected to the heart rate by dividing by the square root of the preceding R-R interval.
• Those with acquired or inherited prolonged QT syndrome are at risk of experiencing
palpitations, syncope, seizures, or sudden death.

ABP Content Specifications(s)


• Understand the clinical significance of a prolonged corrected QT interval

Suggested Readings
• Ackerman MJ. The long QT syndrome. Pediatr Rev. 1998;19(7):232-238.
doi:10.1542/pir.19-7-232.
• Friedman MJ, Mull CC, Sharieff GQ, Tsarouhas N. Prolonged QT syndrome in children:
an uncommon but potentially fatal entity. J Emerg Med. 2003;24(2):173-179.
doi:10.1016/s0736-4679(02)00721-7.
• Ramaswamy P. Syncope. In: McInerny TK, Adam HM, Campbell DE, DeWitt TG, Foy
JM, Kamat DM, eds. American Academy of Pediatrics Textbook of Pediatric Care. 2nd
ed. Elk Grove Village, IL: American Academy of Pediatrics; 2017:1636-1640. Pediatric
Care Online .

American Academy of Pediatrics 633


PREP ® Self-Assessment PREPSA 2021
Question 174
A 14-year-old adolescent girl with no significant medical history is brought to the pediatrician’s
office by her mother for evaluation of a breast mass. Four days before the start of her period, she
noticed a dull ache and a lump in her right breast. The breast pain resolved during her period, but
the mass is still present. She has not experienced any redness or swelling of her breast and has
not noticed any nipple discharge. She had menarche at age 12 years, and her periods have been
regular. She has never been sexually active. Her paternal grandmother has a history of breast
cancer at age 65 years.

Her vitals signs are normal for her age. There is a 2 × 2-cm, rubbery, mobile nodule in the upper
outer quadrant of the right breast. There is no erythema, induration, or tenderness on palpation.
There is no nipple or skin retraction. There is no cervical, axillary, or inguinal lymphadenopathy.
The remainder of the physical examination findings are normal.

Of the following, the MOST appropriate next step in management is to


A. observe the mass for the next 2 menstrual cycles
B. order mammography
C. order ultrasonography of the right breast
D. schedule an excisional biopsy

American Academy of Pediatrics 634


PREP ® Self-Assessment PREPSA 2021

Correct Answer: A
The girl in the vignette has a classic history for a fibroadenoma, and the preferred management
would be to observe the mass over her next two menstrual cycles. If there is concern about the
size or consistency of the mass, an ultrasonogram would be the imaging study of choice.
Ultrasonography would differentiate among fibroadenomas, cystic lesions, and abscesses.
Mammography is rarely done in adolescents, and an excisional biopsy would be indicated only if
an abnormality is detected on ultrasonogram.

Thelarche is usually the first sign of pubertal development in most girls; it occurs between 8 and
13 years of age, with the average age being 10.3 years. The majority of breast masses in
adolescent females are benign, with the most common being a fibroadenoma. Fibroadenomas
occur in response to estrogen stimulation, and they may become larger during menstrual cycles.
Benign fibroadenomas range between 2 and 3 cm and are most commonly located in the upper
outer quadrants of the breast. Giant fibroadenomas are greater than 5 cm. These masses are often
mobile and well circumscribed, have a rubbery feel, and are nontender. It is recommended that a
mass presumed to be a fibroadenoma be observed for a minimum of two menstrual cycles before
any type of evaluation is performed.

Fibroadenomas account for 68% of breast masses in adolescent girls. Other benign breast masses
include fibrocystic changes (15%) and breast abscesses (3%). Additional benign breast masses
include juvenile papillomas and fluid-filled cysts. Malignant masses such as phyllodes tumors
and breast cancer are extremely rare in adolescent girls. Breast cancer would be of more concern
in a girl with a history of radiation therapy to the chest area or known malignancy that may
metastasize.

The evaluation of a breast mass begins with the history and physical examination. The particular
details that must be evaluated are the length of time the mass has been present; any change in its
size (especially with menstruation); nipple discharge; and any history of breast surgeries, chest
wall procedures, or radiation that may have occurred. The physical examination should note the
size of the mass, any redness or tenderness (suggestive of an abscess), the nature of the mass
(firm, rubbery, mobile, fixed), and the appearance of the overlying skin.

PREP Pearls
• The most common breast mass in an adolescent female is a benign fibroadenoma.
• A fibroadenoma can be observed for a minimum of two menstrual cycles before any type
of evaluation is performed, as the mass may resolve itself.
• Ultrasonography is the imaging modality of choice for initial evaluation of a breast mass.

MOCA-Peds Objective
• Recognize and manage breast disorders in adolescents.

American Academy of Pediatrics 635


PREP ® Self-Assessment PREPSA 2021
ABP Content Specifications(s)
• Understand the significance of a breast mass in an adolescent girl as it relates to puberty

Suggested Readings
• DeSilva NK. Breast development and disorders in the adolescent female. Best Pract Res
Clin Obstet Gynæcol. 2018;48:40-50. doi:10.1016/j.bpobgyn.2017.08.009.
• Khaja A, DeSilva NK. The female adolescent breast: disorders of development. Curr
Opin Obstet Gynecol. 2019;31(5):293-297. doi:10.1097/GCO.0000000000000564.
• Wolf RM, Long D. Pubertal development. Pediatr Rev. 2016;37(7);292-300.
doi:10.1542/pir.2015-0065.

American Academy of Pediatrics 636


PREP ® Self-Assessment PREPSA 2021
Question 175
A 2-day-old neonate is nearly ready to be discharged. He was born at 39 weeks’ gestation
without any complications during pregnancy or delivery. He is feeding well and has voided and
passed stools. His vital signs, including oxygen saturation, have been normal since birth. Despite
several attempts, he failed the otoacoustic emission test. Before discharge, repeat otoacoustic
emission testing is arranged.

Of the following, based on current care guidelines, the MOST appropriate timeframe for the
completion of this testing is within
A. 1 month
B. 3 months
C. 6 months
D. 9 months

American Academy of Pediatrics 637


PREP ® Self-Assessment PREPSA 2021
Correct Answer: B
Hearing loss should be identified in all infants by age 3 months, according to the Universal
Hearing Screening Program guidelines. Children with hearing loss often coo and babble like
other infants initially, but then lose these skills after 6 to 9 months of age. They do not continue
to develop speech, such as saying “mama” or “dada” at the typical age of 12 to 15 months.
Because of their hearing loss, these infants often do not look at their caregivers when spoken to
and are less likely to make eye contact. Gross motor function can also be delayed, such as not
sitting or walking as expected. Based on this and other evidence, the Universal Hearing
Screening Program recommends that interventions for infants with hearing loss should begin by
6 months of age, and the child should be established in a medical home.

Two screening tests for hearing are commonly used in the newborn nursery. Automated auditory
brainstem response testing measures the auditory nerve and brainstem response to sounds by
using electrodes. Otoacoustic emission (OAE) testing measures the sound emitted from the
cochlea in response to a projected sound. Results of OAE testing are reported as present or
absent. In addition to hearing loss, an absent OAE result can be caused by occlusion of the
external auditory canal or fluid in the middle ear. If the results of a hearing screening are
abnormal, a full automated auditory brainstem response test should be completed within 3
months, as recommended for the infant in the vignette, to assess the severity of hearing loss; this
often requires sedation in infants.

Hearing loss can be caused by problems in the external auditory canal, middle ear, or inner ear.
Hearing loss resulting from a condition of the external auditory canal or middle ear is defined as
“conductive hearing loss.” Hearing loss resulting from a condition involving or proximal to the
cochlea is termed “sensorineural hearing loss.” At times, both auditory regions may contribute,
and the resulting condition is defined as mixed hearing loss.

Obstruction of the outer ear with cerumen or a foreign body, or malformations of the external ear
such as stenosis or atresia (often seen with craniofacial abnormalities), can interfere with
hearing. The middle ear consists of the tympanic membrane, middle ear bones or ossicles
(malleus, incus, stapes), and the oval window. Examples of middle ear conditions that may result
in conductive hearing loss include effusions (such as in acute otitis media), because the middle
ear bones require air to transmit sound; a large perforation in the tympanic membrane; or
abnormally shaped or ossified ossicles. Effusions are a common cause of conductive hearing
loss, especially after episodes of acute otitis media; 40% of children have an effusion persisting 1
month after treatment, 20% at 2 months, and 10% at 3 months. Most often, the hearing loss is
transient; however, if the effusion persists for 3 months, or if there are symptoms of hearing loss,
formal hearing testing and treatment of the effusion should be considered.

Sensorineural hearing loss has numerous causes, including infections, medications, genetic
conditions, trauma, and other medical conditions. Common infections include bacterial
meningitis as well as congenital infections such as toxoplasmosis, varicella, cytomegalovirus,
syphilis, and herpes simplex virus. Cytomegalovirus is the most common cause of acquired
sensorineural hearing loss, with 50% of symptomatic infants and 10% to 15% of asymptomatic
infants experiencing hearing loss. Ototoxic medications include aminoglycosides, loop diuretics,
and cisplatin. Genetic causes include Alport syndrome, neurofibromatosis type 2, and Stickler
American Academy of Pediatrics 638
PREP ® Self-Assessment PREPSA 2021
syndrome. Trauma, especially of the temporal bone, can lead to hearing loss. In addition, infants
who are at increased risk of experiencing sensorineural hearing loss include those who have
elevated bilirubin levels requiring an exchange transfusion, are receiving ventilator support for
more than 10 days, have Apgar scores of less than 6 at 10 minutes after birth, a birthweight of
less than 1,500 g, or have a family history of congenital or early hearing loss.

PREP Pearls
• Infants with hearing loss should be identified by 3 months of age.
• Cytomegalovirus is the most common cause of acquired sensorineural hearing loss.
• Hearing loss is associated with language, social, and motor developmental delays;
intervention should begin by 6 months of age.

ABP Content Specifications(s)


• Recognize age-related clinical findings associated with hearing loss of various etiologies
• Understand the natural history and etiologies of conductive hearing loss
• Understand the etiologies (eg, infectious, genetic, traumatic) of sensorineural hearing loss
• Recognize conditions that contribute to hearing loss/impairment in patients of various
ages, and the effects of that hearing loss on language development and learning

Suggested Readings
• American Academy of Pediatrics. Newborn hearing screening FAQs.
Healthychildren.org website. Healthychildren.org.
• Eddins AC. Hearing. In: Tanski S, Garfunkel LC, Duncan PM, Weitzman M,
eds. Performing Preventive Services: A Bright Futures Handbook. Elk Grove Village, IL:
American Academy of Pediatrics; 2010:129-126.
• Grindle C. Pediatric hearing loss. Pediatr Rev. 2014;35(11):456-464. doi:10.1542/pir.35-
11-456.
• Tharpe AM, Sladen DP, Ann Rothpletz A. Hearing loss. In: McInerny TK, Adam HM,
Campbell DE, DeWitt TG, Foy JM, Kamat DM, eds. American Academy of Pediatrics
Textbook of Pediatric Care. 2nd ed. Elk Grove Village, IL: American Academy of
Pediatrics; 2017:1408-1411. Pediatric Care Online .

American Academy of Pediatrics 639


PREP ® Self-Assessment PREPSA 2021
Question 176
An otherwise healthy, 4-month-old male infant with a hemangioma is scheduled to start
propranolol treatment. Possible complications of treatment are discussed with his parents.

Of the following, the MOST likely complication is


A. apnea
B. hypertension
C. hypoglycemia
D. tachycardia

American Academy of Pediatrics 640


PREP ® Self-Assessment PREPSA 2021
Correct Answer: C
The infant in the vignette is about to start receiving propranolol for treatment of a hemangioma.
Of the response choices listed, only hypoglycemia is a known adverse effect of β-blocking drugs.
β-Blockers work on β receptors to inhibit the binding of the neurotransmitters norepinephrine
and epinephrine. There are three different kinds of β receptors: β1 receptors are on the heart and
kidneys; β2 receptors are on the lungs, gastrointestinal tract, liver, uterus, vascular smooth
muscle, and skeletal muscle; and β3 receptors are in fat cells.

Types of β-blocking drugs differ according to the β receptors that they block. Some are
nonselective (eg, propranolol) and block both β1 and β2 receptors, whereas others are selective
(eg, metoprolol), blocking only β1. Mechanism of action and adverse effects of β-blockers are
based on the locations of these receptors and the type of β-blocker administered (nonselective
versus selective). Adverse effects could include bradycardia, conduction disturbances (eg,
atrioventricular block), hypotension, bronchospasm, and hypoglycemia.

Before initiating β-blocker therapy, the clinician should obtain a thorough cardiopulmonary
history and ask the child’s parents or caregivers about poor feeding, exercise intolerance,
dyspnea, tachypnea, diaphoresis, wheezing, heart murmur, and family history of heart block or
arrhythmia. Additionally, the clinician should perform a thorough cardiopulmonary examination,
paying specific attention to heart rate, blood pressure, and auscultation. Some have advocated for
routine pretreatment electrocardiography, although the consensus statement by Drolet et al
recommends obtaining an electrocardiogram only for those with a lower-than-normal heart rate
for age; a family history of congenital heart conditions, arrhythmias, or maternal connective
tissue disease; or a personal history of arrhythmia or an arrhythmia noted during physical
examination. This same publication notes the following as contraindications to propranolol
(specifically): cardiogenic shock, sinus bradycardia, hypotension, greater-than-first-degree heart
block, heart failure (although carvedilol is often used in this setting), bronchial asthma, or
hypersensitivity to the medication.

There are no specific guidelines for monitoring after β-blocker administration. Some centers will
admit young infants to an inpatient setting, whereas older children are more routinely monitored
in an outpatient setting. Some centers may choose to monitor patients for 1 to 4 hours on an
outpatient basis. Studies failed to demonstrate asymptomatic hypoglycemia and, therefore,
caregivers are instructed to give β-blockers during daytime hours with a feeding shortly after
administration. β-Blockers may need to be withheld during intercurrent illnesses, and parents and
caregivers should be educated about the signs and symptoms of hypoglycemia.

PREP Pearls
• β-Blocking agents block binding of the neurotransmitters norepinephrine and epinephrine
to β1, β2, and/or β3 receptors. Selective β-blockers act preferentially on β1 receptors.
• Adverse effects of ß-blocking drugs include hypotension, bradycardia, conduction delays,
bronchospasm, and hypoglycemia.
• A thorough cardiopulmonary history and examination should be performed before
initiation of ß-blocker therapy, and parents and caregivers should be educated about the
signs and symptoms of hypoglycemia.

American Academy of Pediatrics 641


PREP ® Self-Assessment PREPSA 2021
ABP Content Specifications(s)
• Recognize the adverse effects associated with beta-blocking drugs

Suggested Readings
• Arneja JS, Benson A, Gilardino MS. Hemangiomas. In: McInerny TK, Adam HM,
Campbell DE, DeWitt TG, Foy JM, Kamat DM, eds. American Academy of Pediatrics
Textbook of Pediatric Care. 2nd ed. Elk Grove Village, IL: American Academy of
Pediatrics; 2017:2110-2116. Pediatric Care Online .
• Baselga E, Debowska-Baginska B, Przewratil P, et al. Efficacy of propranolol between 6
and 12 months of age in high-risk infantile hemangioma. Pediatrics.
2018;142(3):e20173866. doi:10.1542/peds.2017-3866.
• Droitcourt C, Kerbrat S, Rault C, et al. Safety of oral propranolol for infantile
hemangioma. Pediatrics. 2018;141(6):e20173783. doi:10.1542/peds.2017-3783.
• Drolet BA, Frommelt PC, Chamlin SL, et al. Initiation and use of propranolol for
infantile hemangioma: report of a consensus conference. Pediatrics. 2013;131(1):128-
140. doi:10.1542/peds.2012-1691.
• Krowchuk DP, Frieden IJ, Mancini AJ, et al; Subcommittee on the Management of
Infantile Hemangiomas. Clinical practice guideline for the management of infantile
hemangiomas. Pediatrics. 2019;143(1):e20183475. doi:10.1542/peds.2018-3475.

American Academy of Pediatrics 642


PREP ® Self-Assessment PREPSA 2021
Question 177
An 11-month-old female infant is brought to the emergency department for evaluation of pallor
and jaundice. She and her family immigrated to the United States from Pakistan 2 months ago.
She has a temperature of 37.6°C, a heart rate of 122 beats/min, a respiratory rate of 22
breaths/min, a blood pressure of 90/68 mm Hg, and an oxygen saturation of 97% on room air.
Her conjunctivae are pale and slightly icteric. Her lungs are clear to auscultation bilaterally, and
her heart examination reveals a normal rhythm with tachycardia. The spleen is palpable 2.5 cm
below the costal margin.

Laboratory data are shown:


Laboratory Test Result
White blood cell count 10.4 × 103/µL (10.4 × 109/L)
Hemoglobin 5.1 g/dL (51 g/L)
Platelet count 462 × 103/µL (462 × 109/L)
Neutrophils 48%
Lymphocytes 46%
Monocytes 6%
Mean corpuscular volume 51 fL
Hemoglobin A 0%
Hemoglobin A2 0%
Hemoglobin F 100%

Of the following, the condition MOST likely to complicate this patient’s disease course is
A. acute lymphoblastic leukemia
B. iron deficiency
C. iron overload
D. vitamin B12 deficiency

American Academy of Pediatrics 643


PREP ® Self-Assessment PREPSA 2021
Correct Answer: C
The girl in the vignette exhibits a severe microcytic anemia with a complete absence of
hemoglobin A on electrophoresis. This presentation is diagnostic of β thalassemia major.
Thalassemia and thalassemia trait have an increased incidence among Middle Eastern and
southeast Asian populations. Children with β thalassemia major are dependent on red cell
transfusions, and the only cures are an allogeneic hematopoietic stem cell transplant or gene
therapy. The chronic red cell transfusions needed to sustain these children deliver significant
parenteral iron. There is no mechanism for the body to eliminate excess iron, and only a small
amount of iron is lost through sloughing of the intestinal mucosa, or through menstrual bleeding
in postmenarcheal girls. Iron absorption is tightly regulated through the intestine. As a
consequence, children with β thalassemia major (transfusion-dependent thalassemia) will
inevitably develop iron overload. The iron overload can be managed with aggressive chelation,
but it remains a significant driver of morbidity and mortality in this population.

Hemoglobin consists of two α globin chains, two β globin chains and a heme molecule centered
around an iron atom (Item C177A). Microcytic anemias result from the underproduction of
hemoglobin, resulting in smaller erythrocytes during the hematopoietic process. A deficiency in
any component of hemoglobin will result in a microcytic anemia, including iron deficiency and a
defective production of the globin protein (Item C177B). α thalassemia occurs when there are
defective α globin genes (of which there are 4). A single defective α gene results in a silent
carrier, two defective genes in α thalassemia trait, three in hemoglobin H disease, and four in
hydrops fetalis (Item C177C). As there are only two β globin genes, a single defective gene
results in β thalassemia trait and two defective genes in β thalassemia major.

American Academy of Pediatrics 644


PREP ® Self-Assessment PREPSA 2021

American Academy of Pediatrics 645


PREP ® Self-Assessment PREPSA 2021

American Academy of Pediatrics 646


PREP ® Self-Assessment PREPSA 2021

Children with β thalassemia major are not at higher risk of experiencing acute lymphoblastic
leukemia, and they do not develop either iron deficiency or vitamin B12 deficiency.

PREP Pearls
• Microcytic anemias result from the underproduction of hemoglobin, resulting in smaller
erythrocytes during the hematopoietic process. A deficiency in any component of
hemoglobin will result in a microcytic anemia, including iron deficiency and a defective
production of the α or β globin subchains.
• A single defective α globin gene results in a silent carrier, two defective genes in α
thalassemia trait, three in hemoglobin H disease, and four in hydrops fetalis. As there are
only two β globin genes, a single defective gene results in β thalassemia trait and two
defective genes in β thalassemia major.
• Children with β thalassemia major (transfusion dependent thalassemia) will inevitably
develop iron overload.

American Academy of Pediatrics 647


PREP ® Self-Assessment PREPSA 2021

ABP Content Specifications(s)


• Plan the appropriate diagnostic evaluation of suspected thalassemia
• Recognize the clinical and laboratory findings associated with thalassemia major

Suggested Readings
• DeLoughery TG. Microcytic anemia. N Engl J Med. 2014;371(14):1324-1331.
doi:10.1056/NEJMra1215361.
• Kelly N. Thalassemia. Pediatr Rev. 2012;33(9):434-435;. doi:10.1542/pir.33-9-434.
• Kett JC. Anemia in infancy. Pediatr Rev. 2012;33(4):186-187. doi:10.1542/pir.33-4-186.
• McFarren AK, Levy AS. Anemia and pallor. In: McInerny TK, Adam HM, Campbell
DE, DeWitt TG, Foy JM, Kamat DM, eds. American Academy of Pediatrics Textbook of
Pediatric Care. 2nd ed. Elk Grove Village, IL: American Academy of Pediatrics;
2017:1199-1208. Pediatric Care Online .
• Richardson M. Microcytic anemia. Pediatr Rev. 2007;28(1):5-14. doi:10.1542/pir.28-1-5.

American Academy of Pediatrics 648


PREP ® Self-Assessment PREPSA 2021
Question 178
An 18-month-old, previously healthy girl is seen for a health supervision visit. She was born at
term following an uncomplicated pregnancy and delivery. Six months ago, she had been meeting
her milestones: she had a well-developed pincer grasp, was taking independent steps, spoke 1
word in addition to "mama" and "dada", and was beginning to point. Today, her mother reports
that she has no new words, is no longer saying "mama" and "dada", and is no longer pointing out
things she wants. At times she seems clumsy when trying to pick up or play with toys.

Her weight is 11 kg (46th percentile), length is 80 cm (40th percentile), and head circumference
is 44 cm (3rd percentile). Her head circumference growth curve is shown in Item Q178. Physical
examination reveals an awake and alert girl with a paucity of eye contact, who has difficulty
engaging in play and no understandable words. Her cranial nerve findings are normal. Motor
examination reveals mild hypotonia in all 4 extremities, full strength; normal deep tendon
reflexes; and normal sensory, coordination, and gait findings. The remainder of her examination
findings are normal.

Item Q178: Head Circumference Chart. Courtesy of J Goldstein

American Academy of Pediatrics 649


PREP ® Self-Assessment PREPSA 2021
Of the following, this girl’s MOST likely diagnosis is
A. adrenoleukodystrophy
B. Landau-Kleffner syndrome
C. Rett syndrome
D. Zellweger syndrome

American Academy of Pediatrics 650


PREP ® Self-Assessment PREPSA 2021
Correct Answer: C
The clinical presentation of the child in the vignette is consistent with Rett syndrome, a genetic
neurodevelopmental disorder occurring almost exclusively in girls. Following a period of normal
development, children with Rett syndrome experience a developmental regression with loss of
speech, stereotyped hand motions with lack of purposeful movements, an autistic phenotype, and
gait abnormalities (Item C178A). The developmental regression is preceded by a plateau of head
growth resulting in acquired microcephaly. It is important to measure head circumference at each
health supervision visit through age 3 years (Item C178B); poor growth noted on serial head
circumference measurements may be the first evidence of this condition. The associated features
may include seizures, respiratory and cardiac abnormalities, autonomic dysfunction, and sleep
disturbances.

American Academy of Pediatrics 651


PREP ® Self-Assessment PREPSA 2021

American Academy of Pediatrics 652


PREP ® Self-Assessment PREPSA 2021
The diagnosis of Rett syndrome is made clinically, and can be confirmed in a majority of
patients, with genetic testing demonstrating mutations in MECP2 on the X chromosome.
Atypical Rett syndrome can also be seen with mutations
in CDKL5 or FOXG1. MECP2 mutations are usually sporadic de novo mutations. Very
rarely, MECP2 mutations are seen in boys who have severe neonatal encephalopathy and/or a
Rett syndrome phenotype.

Treatment is supportive, with a multidisciplinary care team emphasizing optimization of quality


of life for the child and family. Clinical trials are underway to explore potential treatments.
X-linked adrenoleukodystrophy is a genetic peroxisomal disorder affecting the central nervous
system myelin because of lack of oxidation of very-long-chain fatty acids. Affected boys become
symptomatic between 4 and 8 years of age. Initial symptoms may mimic an attention-
deficit/hyperactivity disorder phenotype or there may be vision complaints. This is followed by a
global developmental regression with accompanying adrenocortical dysfunction. Magnetic
resonance imaging of the brain demonstrates a classic bilateral posterior-predominant
inflammatory demyelination affecting the parieto-occipital lobes. Hematopoietic stem cell
transplantation, if performed early, can arrest disease progression.

Landau-Kleffner syndrome (acquired epileptiform aphasia) is an epilepsy syndrome of unknown


etiology. The condition presents in early childhood with language regression secondary to
frequent epileptiform discharges in the parieto-occipital regions, activated by sleep. Treatment
includes speech therapy in conjunction with antiepileptic medications, corticosteroids, or
epilepsy surgery.

Zellweger syndrome (also known as cerebrohepatorenal syndrome) is an autosomal recessive


peroxisomal disorder presenting in infancy with hypotonia and poor feeding, accompanied by
distinctively dysmorphic facial features. Infants often have seizures, cortical malformations, and
renal and liver disease. Treatment is symptomatic.

PREP Pearls
• Rett syndrome is an X-linked disorder, predominantly affecting girls, characterized by a
period of normal development followed by motor, language, and cognitive regression,
loss of purposeful hand movements replaced with hand wringing, and an autistic-like
phenotype.
• Serial head circumference measurements can demonstrate decelerating head growth
during the time of normal development and be an early clue in patients with Rett
syndrome.
• Treatment of Rett syndrome is currently supportive.

MOCA-Peds Objective
• Differentiate between normal and abnormal variations in head growth and manage
appropriately.

ABP Content Specifications(s)


• Recognize the clinical findings associated with a degenerative disorder of the central
nervous system
American Academy of Pediatrics 653
PREP ® Self-Assessment PREPSA 2021

Suggested Readings
• Glaze D, Neul J, Kaufmann W, et al. Double-blind, randomized, placebo controlled study
of trofinetide in pediatric Rett syndrome. Neurology. 2019;92(16):e1912-e1925.
doi: 10.1212/WNL.0000000000007316.
• Kaur S, Christodoulou J. MECP2 disorders. GeneReviews.
2001. https://www.ncbi.nlm.nih.gov/books/NBK1497/.
• Moeschler JB, Shevell M; Committee on Genetics. Comprehensive evaluation of the
child with intellectual disability or global developmental delays. Pediatrics.
2014;134(3):e903-e918. doi: 10.1542/peds.2014-1839.

American Academy of Pediatrics 654


PREP ® Self-Assessment PREPSA 2021
Question 179
A 4-year-old boy with symptoms of a viral respiratory infection for 3 days is being evaluated for
sudden onset of fever to 39oC, respiratory distress with barking cough, and inspiratory/expiratory
monophasic wheezing. His medical history is unremarkable. On physical examination, he
appears ill and uncomfortable, with intercostal and suprasternal retractions.

Of the following, the pathogen MOST likely associated with this boy’s condition is
A. adenovirus
B. Pseudomonas aeruginosa
C. respiratory syncytial virus
D. Staphylococcus aureus

American Academy of Pediatrics 655


PREP ® Self-Assessment PREPSA 2021
Correct Answer: D
The boy in the vignette has had a viral respiratory illness, with a sudden worsening of symptoms,
including respiratory distress and fever, indicative of a bacterial tracheitis as a
superinfection. Staphylococcus aureus is the most common organism implicated in bacterial
tracheitis. Adenovirus or respiratory syncytial virus might be the underlying cause of his initial
respiratory symptoms, though parainfluenza and influenza viruses are also common precursor
infections. Pseudomonas aeruginosa commonly colonizes the airway in children with
tracheostomy or other conditions in which secretions form pools in the upper airway, but it is
rarely implicated in acute, uncomplicated respiratory illnesses.

Bacterial tracheitis can be rapidly progressing and life threatening. Up to 80% of patients with
bacterial tracheitis require intubation, thus the first priority in patient management is careful
evaluation and stabilization of the airway. Once the airway is stable, airway radiography and
direct visualization of the upper airway in a controlled setting will help to confirm the diagnosis.
Dense purulent secretions and tracheal pseudomembranes may be present and cultures of the
secretions should be performed. In addition to Staphylococcus aureus, Haemophilus
influenzae, Moraxella catarrhalis, Streptococcus pyogenes, and α-hemolytic streptococci may be
implicated in bacterial tracheitis. The initial antibiotic choice should be fairly broad to cover all
of these organisms, and then narrowed to treat the specific organism identified.

Differentiation of bacterial tracheitis from acute epiglottitis and viral croup will depend on age,
presentation, and severity of illness. Item C179 outlines differences among the 3 acute
respiratory illnesses.

American Academy of Pediatrics 656


PREP ® Self-Assessment PREPSA 2021

American Academy of Pediatrics 657


PREP ® Self-Assessment PREPSA 2021
PREP Pearls
• Bacterial tracheitis is a potentially life-threatening emergency.
• Most acute bacterial tracheitis in previously healthy children occurs as a superinfection
secondary to a viral respiratory illness, most often with parainfluenza, respiratory
syncytial virus,or influenza.
• Staphylococcus aureus is the most common bacteria responsible for tracheitis.

ABP Content Specifications(s)


• Identify the pathogens most likely associated with tracheitis
• Recognize the clinical findings, including disease course, associated with tracheitis
• Plan the appropriate management of tracheitis

Suggested Readings
• Blot M, Bonniaud-Blot P, Farrolt N, et al. Update on childhood and adult infectious
tracheitis. Med Mal Infect. 2017;47:443-452. doi: 10.1016/j.medmal.2017.06.006.
• Casazza G, Graham ME, Nelson D, Chaulk D, Sandweiss D, Meier J. Pediatric bacterial
tracheitis--a variable entity: case series with literature review. Otolaryngol Head Neck
Surg. 2019;160:546-549. doi: 10.1177/0194599818808774.
• Conrad C, Cornfield DN. Airway obstruction. In: McInerny TK, Adam HM, Campbell
DE, DeWitt TG, Foy JM, Kamat DM, eds. American Academy of Pediatrics Textbook of
Pediatric Care. 2nd ed. Itasca, IL: American Academy of Pediatrics; 2016;chap
348:2777-2785. Pediatric Care Online.
• Kuo CY, Parikh SR. Bacterial tracheitis. Pediatr Rev. 2014;35:497-499.
doi: 10.1542/pir.35-11-497.
• Tebruegge M, Pantazidou A, Yau C, et al. Bacterial tracheitis: tremendously rare but
truly important—a systematic review. J Pediatr Infect Dis. 2009;4:199. doi: 10.3233/JPI-
2009-0179.

American Academy of Pediatrics 658


PREP ® Self-Assessment PREPSA 2021
Question 180
A 1-month-old female infant is being seen for a health supervision visit. She was born at 38
weeks' gestation via cesarean delivery because of breech presentation. Her physical examination
is significant for a positive Ortolani maneuver on the right. Ultrasonography of the hip confirms
the suspected diagnosis.

Of the following, the MOST likely mechanism for her condition is


A. association
B. deformation sequence
C. disruption sequence
D. malformation syndrome

American Academy of Pediatrics 659


PREP ® Self-Assessment PREPSA 2021
Correct Answer: B
The neonate in the vignette has developmental dysplasia of the right hip (DDH). Developmental
dysplasia of the right hip results from abnormal alignment and movement of the head of the
femur within the acetabulum in utero. When the femoral head is not correctly approximated
within the acetabulum, the acetabulum does not develop properly. Deformation sequence is the
best explanation for the clinical presentation of DDH. A deformity caused by a deformation
sequence results from abnormal mechanical or structural external forces, rather than an intrinsic
abnormality of the fetus.

In the case of disruption sequence, a normally developing fetus experiences an event or exposure
that alters the normal course of development; this may have a vascular, infectious, mechanical,
or metabolic cause. Amniotic band syndrome is an example of a disruption sequence.
Malformation sequence is abnormal tissue development leading to congenital anomalies, such as
atrioventricular canal defect in trisomy 21. An association like VACTERL (vertebral anomalies,
cardiac defect, tracheoesophageal fistula, renal anomalies and limb anomalies) is an example of a
collection of anomalies commonly seen together without an identified genetic mutation.
Developmental dysplasia of the hip affects 1 in 1,000 live births in the United States. Risk
factors for DDH include female sex, breech presentation, positive family history, and incorrect
swaddling techniques (Item C180A). Swaddling should allow for hip flexion and abduction, as
well as knee flexion (Item C180B and Item C180C). Surveillance for DDH should include
ongoing assessment at each health supervision visit of hip mobility (range of motion) and hip
stability (Ortolani and Barlow maneuvers) through age 12 weeks. Assessment for limitation of
hip abduction should continue until the child is ambulating with a normal age-appropriate gait.
Care should be taken during the Barlow maneuver to provide support to the hip joint and avoid
posterior displacement.

American Academy of Pediatrics 660


PREP ® Self-Assessment PREPSA 2021

Item C180A: Traditional swaddling with the legs together and extended is associated with an
increased risk of hip dislocation.
Reprinted with permission from Price CT, Schwend RM. Improper swaddling a risk factor for
developmental dysplasia of hip. AAP News 2011;32 (9)

American Academy of Pediatrics 661


PREP ® Self-Assessment PREPSA 2021

Item C180B: Carrying children in the straddle position is associated with a very low rate of hip
dislocation Reprinted with permission from Price CT, Schwend RM. Improper swaddling a risk
factor for developmental dysplasia of hip. AAP News 2011;32 (9).

American Academy of Pediatrics 662


PREP ® Self-Assessment PREPSA 2021

Item C180C: Proper swaddling techniques allow for ample room for hip and knee flexion with
free leg movement.
Reprinted with permission from Price CT, Schwend RM. Improper swaddling a risk factor for
developmental dysplasia of hip. AAP News 2011;32 (9).

PREP Pearls
• Developmental dysplasia of the hip results from a deformation sequence: improper
alignment of the femoral head with the acetabulum impairs proper development of the
hip joint.
• Risk factors for developmental dysplasia include female sex, breech presentation,
positive family history, and incorrect swaddling techniques.
• Appropriate swaddling allows for hip flexion and abduction as well as knee flexion.

ABP Content Specifications(s)


• Recognize the anatomic effects of amniotic bands
• Understand how positional deformations and/or malformations develop in a fetus

Suggested Readings
• Nemeth BA, Narotam V. Developmental dysplasia of the hip. Pediatr Rev. 2012;33:553.
doi: 10.1542/pir.33-12-553.
• Shaw BA, Segal LS; Section on Orthopaedics. Evaluation and referral for developmental
dysplasia of the hip in infants. Pediatrics. 2016;138:e1-e11. doi: 10.1542/peds.2016-
3107.
• Slavotinek AM. Dysmorphology. In: Kliegman RM, St Geme JW, Blum NJ, Shah SS,
Tasker RC, Wilson KM. Nelson Textbook of Pediatrics. Philadelphia, PA: Elsevier;
2020:986-996.
American Academy of Pediatrics 663
PREP ® Self-Assessment PREPSA 2021
Question 181
An 11-year-old boy is brought to the emergency department for evaluation of nausea, vomiting,
abdominal cramps, and watery diarrhea that began abruptly 3 days ago. One week before the
onset, he helped his grandmother prepare a meal that included pork intestines (“chitlins”). Other
family members have also become ill with a similar gastrointestinal illness. Medical history is
significant for β thalassemia major, for which he has received monthly blood transfusions since
infancy and is treated with an iron-chelating agent. His temperature is 38°C. He has dry mucous
membranes and sunken eyes. His capillary refill time is 3 seconds. Laboratory data are shown:

Laboratory Test Result


White blood cell count 33,200/µL (33.2 × 109/L)
Neutrophils 82%
Bands 10%
Lymphocytes 7%
Monocytes 1%
Sodium 130 mEq/L (130 mmol/L)

Of the following, the MOST likely cause of his illness is


A. Bacillus cereus
B. Salmonella enterica
C. Shigella sonnei
D. Yersinia enterocolitica

American Academy of Pediatrics 664


PREP ® Self-Assessment PREPSA 2021
Correct Answer: D
The clinical presentation of the boy described in this vignette, with acute onset of gastroenteritis
after recent consumption of pork intestines (called “chitterlings” or “chitlins”), is suggestive
of Yersinia enterocolitica gastroenteritis. Yersinia enteritis may be complicated by bacteremia,
especially in infants and older children who are at high risk of developing invasive infection,
such as underlying conditions with excessive iron storage (eg, receipt of iron chelation therapy,
sickle cell disease, β thalassemia, receipt of chronic transfusion therapy), and
immunocompromised hosts.

Yersinia species are gram-negative bacilli with three primary human pathogens (Y
enterocolitica, Yersinia pseudotuberculosis, Yersinia pestis). Transmission
of Y enterocolitica occurs mainly through the fecal-oral route, often via ingestion of
contaminated meat products (especially pork), inadequately pasteurized milk products,
contaminated water, and contaminated vegetables. Person-to-person spread of Yersinia enteritis
has occurred from handling raw pork intestines (ie, chitterlings) without good hand hygiene
during traditional winter holiday celebrations. Other less frequent modes of transmission include
direct or indirect contact with animals (eg, pigs, dogs, cats, rats).
According to the Centers for Disease Control and Prevention Foodborne Diseases Active
Surveillance Network (FoodNet), the average annual incidence of Yersinia foodborne infection
during 2006 to 2009 FoodNet surveillance was 0.5 per 100,000 individuals. Children older than
5 years have the highest infection rates, with 24.08 cases per 100,000 individuals in 2009, which
accounted for 47% of infections.

Yersinia enterocolitica can cause a variety of age-specific clinical syndromes ranging from
acute, uncomplicated enteritis to terminal ileitis, pseudoappendicitis syndrome, bacteremia,
necrotizing enterocolitis, and sepsis. Following an incubation period of 4 to 6 days (range, 1-14
days), infected people typically have fever, blood or mucus-tinged diarrhea, and abdominal pain.
The diarrhea can persist up to 2 weeks. Extraintestinal infection is unusual and includes
musculoskeletal infections, peritonitis, pneumonia, meningitis, endocarditis, and focal abscesses
of the liver and spleen. In older children and adults, especially those with human leukocyte
antigen-B27, Y enterocolitica is also implicated in postinfectious sequelae, including erythema
nodosum, reactive arthritis, and proliferative glomerulonephritis.

Yersinia species can be isolated from routine stool cultures, blood, and other tissue sites.
Nonculture-based methods (eg, gene-based multiple polymerase chain reaction platforms) have
been developed for rapid diagnosis of Yersinia spp and other enteric bacterial, viral, and parasitic
pathogens. In severe infections or infections complicated by bacteremia or sepsis caused
by Yersinia spp, it is crucial to isolate the organism by culture to obtain the antimicrobial
susceptibility data needed to guide antibiotic therapy.

Antimicrobial therapy (with parenteral third-generation cephalosporin such as ceftriaxone) is


recommended for all immunocompromised patients, neonates and infants, and all patients with
invasive Y enterocolitica infection (bacteremia, sepsis, extraintestinal infection). Cerebrospinal
fluid studies are indicated in neonates. Antimicrobial treatment may also be considered in
immunocompetent patients with Yersinia enterocolitis, pseudo-appendicitis syndrome, or
mesenteric adenitis, given the decrease in duration of shedding of the organism after antibiotic
American Academy of Pediatrics 665
PREP ® Self-Assessment PREPSA 2021
treatment, although the clinical benefit is unclear. Besides third-generation cephalosporins, Y
enterocolitica is susceptible to trimethoprim-sulfamethoxazole, fluoroquinolones,
aminoglycosides, and doxycycline. Symptomatic treatment with correction of dehydration and
replacement of fluids and electrolytes, without the need for specific antimicrobial therapy, may
suffice for otherwise healthy children who have Yersinia enteritis.

Many pathogens cause foodborne enteric


infections: Campylobacter, Cryptosporidium, Salmonella, shiga toxin-producing Escherichia
coli, Shigella, Vibrio, and Yersinia. The consumption of chitterlings has not been linked to other
pathogens such as Bacillus cereus, nontyphoidal Salmonella, and Shigella enteritis. Therefore, Y
enterocolitica is the preferred response for the adolescent described in the vignette. Bacillus
cereus is an important cause of toxin-mediated foodborne illness in the United States and can
cause both an emetic and a diarrheal syndrome. The emetic syndrome can occur after ingestion
of contaminated fried rice containing preformed enteric toxin. The diarrheal syndrome caused
by B cereus is a severe illness and results in watery diarrhea, vomiting, and abdominal pain.
Nontyphoidal Salmonella is an important cause of foodborne enteritis, but the incubation period
usually ranges from 6 to 48 hours, and poultry, beef, eggs, and dairy products are the major food
vehicles of transmission. Shigella sonnei infection is primarily transmitted from person to person
via the fecal-oral route. The incubation period of Shigella enteritis is typically 1 to 3 days. The
illness is characterized by watery or mucoid stools and/or dysentery, abdominal pain, and
tenesmus.

PREP Pearls
• Transmission of Yersinia enterocolitica occurs mainly through the fecal-oral route, often
via ingestion of contaminated pork products or unpasteurized or inadequately pasteurized
milk products, contaminated water, and contaminated vegetables.
• Yersinia enteritis may be complicated by bacteremia, especially in infants, older children
with excessive iron storage (eg, deferoxamine therapy, sickle cell disease, β thalassemia),
and immunocompromised hosts.
• Antimicrobial therapy is recommended for all immunocompromised patients, neonates
and infants, and all patients with invasive Yersinia infection.

ABP Content Specifications(s)


• Plan appropriate management for a patient with Yersinia enterocolitica infection
• Recognize the clinical features associated with Yersinia enterocolitica infection

American Academy of Pediatrics 666


PREP ® Self-Assessment PREPSA 2021
Suggested Readings
• American Academy of Pediatrics. Yersinia enterocolitica and Yersinia
pseudotuberculosis infections. In: Kimberlin DW, Brady MT, Jackson MA, Long SS,
eds. Red Book: 2018-2021 Report of the Committee on Infectious Diseases. 31st ed. Elk
Grove Village, IL: American Academy of Pediatrics; 2018:891-894.
• Centers for Disease Control and Prevention (CDC). Yersinia enterocolitica gastroenteritis
among infants exposed to chitterlings—Chicago, Illinois, 2002. MMWR Morb Mortal
Wkly Rep. 2003;52(40):956-958.
• Committee on Infectious Diseases; Committee on Nutrition; American Academy of
Pediatrics. Consumption of raw and unpasteurized milk and milk products by pregnant
women and children. Pediatrics. 2014;133(1):175-179. doi:10.1542/peds.2013-3502.
• Marder EP, Griffin PM, Cieslak PR, et al. Preliminary incidence and trends of infections
with pathogens transmitted commonly through food—Foodborne Diseases Active
Surveillance Network, 10 U.S. Sites, 2006-2017. MMWR Morb Mortal Wkly
Rep. 2018;67(11):324-328. doi:10.15585/mmwr.mm6711a3.

American Academy of Pediatrics 667


PREP ® Self-Assessment PREPSA 2021
Question 182
A 15-year-old adolescent girl with a rash and difficulty breathing is seen in the pediatrician’s
office for evaluation. She has no known allergies. For long-term control of her asthma, she
recently started using an inhaled medication newly approved by the US Food and Drug
Administration. The medication has no specific black-box warnings and is approved for use in
patients older than 12 years. She has urticaria throughout her body and diffuse wheezing. She has
a temperature of 36.5°C, a heart rate of 130 beats/min, a respiratory rate of 28 breaths/min, and a
blood pressure of 100/40 mm Hg. Her breathing is labored with audible stridor and wheezes and
suprasternal retraction. The remainder of the physical examination findings are unremarkable.
She receives intramuscular epinephrine and is transferred to the intensive care unit. A possible
adverse drug reaction is suspected.

Of the following, the MOST appropriate reporting entity is


A. the Food and Drug Administration MedWatch program
B. The Joint Commission
C. the local department of health
D. the principal investigator of the original clinical trial

American Academy of Pediatrics 668


PREP ® Self-Assessment PREPSA 2021
Correct Answer: A
The patient in the vignette is experiencing an adverse drug reaction (ADR) to a medication
approved by the US Food and Drug Administration (FDA) that was prescribed according to the
labeling guidance. As such, the best answer is to report this to the FDA MedWatch program.
Adverse drug reactions are common in children, and data suggest the incidence of pediatric
hospitalization as a result of an ADR ranges from 4% to 22%. Children may be at higher risk of
experiencing ADRs than are adults because of age-related differences in physiology, differences
in patterns of disease, and smaller size and body surface area.

The US Department of Health and Human Services administers a voluntary program for
reporting adverse events called MedWatch. The MedWatch program provides a process by
which clinicians can voluntarily report serious adverse events associated with the use of an FDA-
regulated product. Specifically, the Department of Health and Human Services encourages
MedWatch submissions regarding safety issues related to FDA-approved medications, biologics,
medical devices, dietary supplements, or cosmetics. As a result of voluntary reporting, the FDA
has issued safety warnings, made changes to FDA-approved medication labeling, required
pharmaceutical companies to perform postmarketing studies to obtain additional safety data, and
even withdrawn prescription and over-the-counter medications from the consumer market. With
respect to ADRs specifically, the FDA MedWatch program encourages physicians to report
events that are fatal, life-threatening, or permanently disabling; require prolonged
hospitalization; cause congenital anomaly; or require an intervention to prevent permanent
impairment or damage.

For MedWatch reports involving product design, manufacturing quality, or counterfeit products,
the FDA can work with manufacturers to issue a recall or to request modification of product
design or manufacturing processes, and the agency may request improvements in instructions or
warnings for use. For safety issues with medications, MedWatch reporting may result in new
labeling, including “boxed” warnings (about drug-drug and food-drug interactions), specific
contraindications, dose adjustments in specific populations, precautions, or information about
adverse reactions.

Neither the Joint Commission nor local departments of public health have a designated role in
the regulation of FDA-approved medications. Additionally, because the medication is already
FDA-approved, there is no reason to inform the principal investigator of the clinical trial that led
to approval of the medication.

PREP Pearls
• The US Food and Drug Administration encourages MedWatch submissions regarding
safety issues related to medications, biologics, medical devices, dietary supplements, or
cosmetics.
• Online MedWatch forms are available, and physicians should complete them when they
suspect serious adverse reactions.
• Pediatric patients may be more prone to adverse drug reactions than are adult patients.

American Academy of Pediatrics 669


PREP ® Self-Assessment PREPSA 2021
ABP Content Specifications(s)
• Understand the circumstances for and process of reporting adverse drug reactions to the
Food and Drug Administration

Suggested Readings
• Clavenna A, Bonati M. Adverse drug reactions in childhood: a review of prospective
studies and safety alerts. Arch Dis Child. 2009;94(9):724-728.
doi:10.1136/adc.2008.154377.
• Le J, Nguyen T, Law AV, Hodding J. Adverse drug reactions among children over a 10-
year period. Pediatrics. 2006;118(2):555-562. doi:10.1542/peds.2005-2429.
• US Food and Drug Administration. MedWatch: The FDA Safety Information and
Adverse Event Reporting Program. https://www.fda.gov/safety/medwatch-fda-safety-
information-and-adverse-event-reporting-program.

American Academy of Pediatrics 670


PREP ® Self-Assessment PREPSA 2021
Question 183
A 10-year-old boy with cystic fibrosis is seen in the emergency department with concerns about
having felt tired for 1 day. He was playing basketball with his cousin in hot weather 2 days ago.
He has no recent history of fever, vomiting, diarrhea, cough, or breathing difficulty. His
medications include inhaled tobramycin, pancreatic enzyme supplements, and multivitamins. He
has a heart rate of 110 beats/min, a respiratory rate of 14 breaths/min, and a blood pressure of
100/60 mm Hg. His mucous membranes are dry, and his chest is clear to auscultation. The rest of
the examination findings are unremarkable. Laboratory studies are ordered.

Of the following, the MOST likely laboratory results in this child are

A.
Sodium Potassium Chloride Bicarbonate
(mEq/L) (mEq/L) (mEq/L) (mEq/L)

130 3.0 92 28

B.
Sodium Potassium Chloride Bicarbonate
(mEq/L) (mEq/L) (mEq/L) (mEq/L)

150 4.0 110 16

C.
Sodium Potassium Chloride Bicarbonate
(mEq/L) (mEq/L) (mEq/L) (mEq/L)

130 3.0 110 16

D.
Sodium Potassium Chloride Bicarbonate
(mEq/L) (mEq/L) (mEq/L) (mEq/L)

140 4.0 102 24

American Academy of Pediatrics 671


PREP ® Self-Assessment PREPSA 2021
Correct Answer: A
The child in this vignette with cystic fibrosis (CF) has hypochloremia, hyponatremic
dehydration, metabolic alkalosis, and hypokalemia because of excessive loss of electrolytes with
sweating. The defect in the cystic fibrosis transmembrane regulator gene leads to increased
sodium chloride losses from the sweat gland. The chloride depletion leads to extracellular
volume contraction and metabolic alkalosis. Hypokalemia occurs as a result of losses with
increased strenuous activity and heat.

Infants with CF are at a higher risk of experiencing hypochloremic hyponatremic dehydration


than are older children and adults. Routine supplementation with sodium chloride is
recommended in infants, and the dose must be increased with fever, vomiting, diarrhea, or
exposure to high temperature. Older children and adults with CF in these high-risk settings
should increase their consumption of salty foods to prevent dehydration.

The differential diagnosis of hypochloremia, hyponatremia, and hypokalemia with alkalosis


includes electrolyte losses from the gastrointestinal tract (vomiting, chloride losing diarrhea),
kidney (Bartter syndrome, furosemide therapy), and sweat glands (cystic fibrosis).
Hyperchloremia, hypernatremia, and acidosis (response choice B) are commonly seen in infants
with hypernatremic dehydration secondary to acute diarrheal illness. Hyperchloremia, normal
anion gap acidosis, and hypokalemia (response choice C) can be seen in children with renal
tubular acidosis and diarrhea. Normal electrolyte values (response choice D) are unlikely for the
child with CF in this vignette, because he has dehydration caused by strenuous activity.

PREP Pearls
• Children with cystic fibrosis are at risk of experiencing hypochloremia, hyponatremic
dehydration, hypokalemia, and metabolic alkalosis because of loss of electrolytes with
sweating.
• Sodium chloride supplementation prevents the risk of electrolyte disturbances in the
setting of fever, vomiting, diarrhea, strenuous activity, or exposure to heat.

ABP Content Specifications(s)


• Recognize the association of hypochloremic/hyponatremic dehydration in patients who
have cystic fibrosis

Suggested Readings
• Paranjape SM, Mogayzel PJ Jr. Cystic fibrosis. Pediatr Rev. 2014;35(5);194-205.
doi:10.1542/pir.35-5-194.
• Willey-Courand DB, Marshall BC. Cystic fibrosis. In: McInerny TK, Adam HM,
Campbell DE, DeWitt TG, Foy JM, Kamat DM, eds. American Academy of Pediatrics
Textbook of Pediatric Care. 2nd ed. Elk Grove Village, IL: American Academy of
Pediatrics; 2017:1936-1947. Pediatric Care Online .

American Academy of Pediatrics 672


PREP ® Self-Assessment PREPSA 2021
Question 184
A 16-year-old boy adolescent boy is brought to the office by his father for behavioral concerns.
His son had been doing well in school until this past year. His father feels that his son’s
motivation to do well academically has dropped, and he recently learned that his son has been
skipping classes. The adolescent has been keeping to himself and spending time in his room
napping or playing video games. He does not want to go when his father suggests family outings.
They argue constantly. The adolescent had been doing well with treatment for attention-
deficit/hyperactivity disorder with a combination of behavioral therapy, school support, and
stimulant medication. Both the father and son report that recently the son has been having more
difficulty with distractibility and inattention. When interviewed separately, the adolescent states
that his father is too controlling and will not let him go out with his friends. He states that he
plays video games because he is “bored” and there is nothing else to do. On physical
examination, the boy is noted to have gained 10 pounds since his last visit 2 months ago.

Of the following, the MOST accurate statement regarding this boy’s behavior is that
A. adjusting the medication dose will resolve the behavioral concerns
B. interpersonal psychotherapy is an appropriate treatment option
C. the behaviors are consistent with normal adolescent mood swings
D. treatment should include prescription of a mood stabilizer

American Academy of Pediatrics 673


PREP ® Self-Assessment PREPSA 2021
Correct Answer: B
The boy in the vignette is demonstrating behavior consistent with depression. Interpersonal
psychotherapy is an evidence-based treatment for depression in adolescents and is an appropriate
treatment option for this boy, who is at high risk for depression because of his previous diagnosis
of attention-deficit/hyperactivity disorder (ADHD).

Although adjusting the dose of stimulant medication may help with inattention from
incompletely controlled ADHD, this adolescent’s increased distractibility is more likely because
of the coexisting depression. Given the persistence of his depressive symptoms, their impact on
academic, emotional, and social functioning, and the presence of somatic symptoms, this boy is
experiencing more than normal adolescent mood swings. The primary medications used to treat
significant depression in adolescents are selective serotonin reuptake inhibitors, not mood
stabilizers.

The lifetime prevalence of depression is approximately 13% (~8% in adolescent boys and 18%
in adolescent girls). The diagnosis of depression involves at least 2 weeks of depressed or
irritable mood and/or loss of pleasure or interest (ie, anhedonia). Accompanying signs and
symptoms can include changes in appetite/weight, sleep patterns, activity level, energy level, and
concentration. The depressed individual may have feelings of poor self-worth, guilt, or suicidal
ideation. Children may display irritability, disruptive behaviors, social withdrawal, school
problems, or somatic complaints. Adolescents may engage in substance use or other risk-taking
behaviors. Although typical adolescents may be “moody” or have mood swings, the emotions of
the adolescent with depression are more intense, persistent, and accompanied by other depressive
symptoms (eg, anhedonia and changes in weight, energy, sleep) and significant impairment in
functioning (eg, academics, social).

When evaluating for depression, the pediatric clinician should assess for suicidal ideation and
risk factors for suicide (eg, previous attempt, family history of suicide). Immediate mental health
specialty evaluation should be performed for those determined to be unsafe, and a suicide
emergency plan and safety plan should be developed in partnership with the child and their
family. Children who are deemed appropriate for outpatient treatment should be linked to
resources and actively monitored with frequent visits. The provider should also ensure that
coexisting conditions such as anxiety disorders, ADHD, and substance use disorders are
identified and addressed.

Importantly, the provider should provide psychoeducation to the child and family about
depression and options for treatment. Two evidence-based psychotherapies for depression in
adolescents are interpersonal psychotherapy and cognitive behavioral therapy. Interpersonal
psychotherapy addresses relationship and communication problems that contribute to or result
from depression. Cognitive behavioral therapy focuses on reframing negative thoughts and
changing behaviors to improve mood. These therapies are usually provided over 8 to 20 weeks
and may be sufficient to treat mild-to-moderate depression. Moderate-to-severe depression
typically requires the addition of a selective serotonin reuptake inhibitor, the first-line medication
for treatment of depression in children. The US Food and Drug Administration has approved
fluoxetine for treating depression in children and adolescents and escitalopram for treating
depression in adolescents.
American Academy of Pediatrics 674
PREP ® Self-Assessment PREPSA 2021

PREP Pearls
• Although typical adolescents may be “moody” or have mood swings, the emotions of the
adolescent with depression are more intense, persistent, and accompanied by other
depressive symptoms (eg, anhedonia and changes in weight, energy, and sleep) and
significant impairment in functioning (eg, academics, social).
• Evidence-based psychotherapies for depression in adolescents include 1) interpersonal
psychotherapy, which addresses relationship and communication problems that
contribute to or result from depression and 2) cognitive behavioral therapy, which
focuses on reframing negative thoughts and changing behaviors to improve mood.
• The US Food and Drug Administration has approved fluoxetine for treating depression in
children and adolescents and escitalopram for treating depression in adolescents.

ABP Content Specifications(s)


• Recognize co-morbidities commonly associated with depressive disorders in children and
adolescents
• Distinguish the findings associated with normal mood swings in an adolescent from those
of a depressive disorder
• Recognize the clinical findings associated with depressive disorders in children and
adolescents, and manage appropriately

Suggested Readings
• Bogdanovic N, Durham MP. Depression. In: Augustyn M, Zuckerman B, eds. Zuckerman
Parker Handbook of Developmental and Behavioral Pediatrics for Primary Care. 4th ed.
Philadelphia, PA: Wolters Kluwer; 2019:203-206.
• Cheung AH, Zuckerbrot RA, Jensen PS, Laraque D, Stein REK; GLAD-PC Steering
Group. Guidelines for adolescent depression in primary care (GLAD-PC): part II—
treatment and ongoing management. Pediatrics.2018;141(3):e20174082.
doi: 10.1542/peds.2017-4082.
• Maslow GR, Dunlap K, Chung RJ. Depression and suicide in children and
adolescents. Pediatr Rev. 2015;36(7):299-310. doi: 10.1542/pir.36-7-299.
• Schlozman SC, Jellinek MS. Pediatric history: assessing functioning and mental health.
In: McInerny TK, Adam HM, Campbell DE, DeWitt TG, Foy JM, Kamat DM,
eds. American Academy of Pediatrics Textbook of Pediatric Care. 2nd ed. Itasca, IL:
American Academy of Pediatrics; 2016;chap 15:99-103. Pediatric Care Online.
• Zuckerbrot RA, Cheung A, Jensen PS, Stein REK, Laraque D; GLAD-PC Steering
Group. Guidelines for Adolescent Depression in Primary Care (GLAD-PC): part I.
practice preparation, identification, assessment, and initial management. Pediatrics.
2018;141(3):e20174081. doi: 10.1542/peds.2017-4081.

American Academy of Pediatrics 675


PREP ® Self-Assessment PREPSA 2021
Question 185
A healthy, 15-year-old adolescent by is brought to the emergency department with concerns of
acute intoxication. His parents found him several hours ago sitting alone in a bathroom filled
with a pungent odor surrounded by drug paraphernalia. He was unable to respond coherently. On
physical examination, his temperature is 37.5°F, heart rate is 110 beats/min, respiratory rate is 22
breaths/min, blood pressure is 130/85 mm Hg, and his oxygen saturation on pulse oximetry is
98% in room air. He is moving slowly and is slightly unkempt. He answers some questions
appropriately but others nonsensically. He reports having auditory hallucinations. His physical
examination findings are otherwise normal.

Of the following, the MOST likely cause of this adolescent’s symptoms is


A. cocaine
B. heroin
C. lysergamide
D. marijuana

American Academy of Pediatrics 676


PREP ® Self-Assessment PREPSA 2021
Correct Answer: D
The adolescent’s signs and symptoms described in the vignette result from marijuana use. Acute
marijuana intoxication may lead to hypertension, tachycardia, sedation, and psychosis. Acute
cocaine toxicity generally causes significant agitation and anxiety, along with cardiac
abnormalities such as arrhythmias. Heroin intoxication usually leads to hypoventilation, mild
hypotension, and miosis. Lysergamide has no odor and may cause significant euphoria.

Plants and fungi are ubiquitous, with most being harmless. Select plants and fungi are cultivated
because they produce chemicals that activate the reward pathways in the central nervous system.
Concentrating and/or adding functional groups to these chemicals can increase their potency and
likelihood of abuse. Item C185A lists plant or fungus-derived illicit substances and their
physiologic effects.

Marijuana, which comes from the cannabis plant, is the most common illicit substance abused by
adolescents. Street names include “weed,” “pot,” “Mary Jane,” and “reefer.” Increasing local
legalization of its recreational use has led to a decreased perception of risk. Indeed, many
adolescents actively seek it out to alleviate feelings of anxiety and depression. Marijuana is
composed of numerous cannabinoids, with THC (tetrahydrocannabinol) being the most potent.
Cannabinoids activate the cannabinoid receptors in the brain. It can be inhaled or ingested.
Inhalation results in a more rapid onset of action, but ingestion can prolong its effects. Urine
screening may detect THC metabolites for as long as 30 days after exposure, depending on the
chronicity of exposure. Urine screening cannot, however, detect synthetic cannabinoids such as
“K2” or “spice,” which may cause more severe symptoms than THC. Symptoms can be treated
with short-acting benzodiazepines and/or antipsychotics. Chronic use can lead to poor cognitive
performance and cannabis hyperemesis syndrome.

The rates of accidental marijuana ingestion among young children have increased following its
decriminalization in many places. Symptoms in young children may be severe because of their
decreased body mass relative to potency, and may even result in a coma. Pediatricians should
counsel parents on the potential adverse effects of marijuana in young children and adolescents.
The American Academy of Pediatrics Committee on Substance Use and Prevention has offered
talking points for addressing marijuana exposure in young children, adolescents, and parents
(https://pediatrics.aappublications.org/content/139/3/e20164069).

American Academy of Pediatrics 677


PREP ® Self-Assessment PREPSA 2021
Cocaine is extracted from the Erythroxylum coca plant. Street names include “C,” “blow,”
“candy,” and “snow.” It can be inhaled, ingested, or injected. Symptoms result from cocaine’s
ability to block the reuptake of epinephrine, norepinephrine, serotonin, and dopamine. Urine
screening tests can detect cocaine for up to 4 days after use. Treatment is supportive. β-
adrenergic antagonists are contraindicated because of the theoretical risk of uninhibited α-
adrenergic stimulation. If intubation is required, succinylcholine is contraindicated because it
may lead to hyperkalemia, rhabdomyolysis, paralysis, and/or prolonged cocaine toxicity. For
hyperthermia, rapid cooling measures are especially critical. Benzodiazepines and/or
antipsychotics may also be used. Long-term cocaine use may lead to stroke, malnourishment,
and seizures.

Heroin is a semisynthetic opioid derived from the poppy plant. Street names include “dope,”
“white horse,” “brown sugar,” and “skag.” It is usually injected or inhaled, resulting in activation
of opioid receptors. Heroin may be detected in urine up to 4 days after exposure. Treatment of
toxicity includes naloxone administration along with supportive measures.

Lysergamide is an odorless product synthesized from a substance made by the Claviceps


purpurea fungus. Street names include “acid,” “Blue Heaven,” “Yellow Sunshine,” and “cubes.”
It is usually ingested and activates serotonin receptors. Those taking serotonin reuptake
inhibitors with lysergamide may therefore experience serotonin toxicity. Lysergamide
metabolites may be detected in urine for up to 5 days after exposure. Treatment is mainly
supportive and may include benzodiazepines. Physical restraints should be avoided, because
their use may result in hyperthermia and rhabdomyolysis. Chronic lysergamide use may lead to
persistent perception disorders.

Individuals who have experienced intoxication from intentional illicit substance use should be
referred to a mental health provider, with close monitoring in their medical home. Children with
unintentional toxic exposure to illicit substances may need to be referred to child protective
services. Neonates who are known to have been exposed to illicit drugs during the last 5 months
of pregnancy may have their meconium tested for metabolites of cocaine, marijuana,
amphetamines, and opiates.

Plants not used to produce illicit substances may also have toxic effects. Item C185B lists
common, clinically relevant toxic plants and their antidote (if available).

American Academy of Pediatrics 678


PREP ® Self-Assessment PREPSA 2021

American Academy of Pediatrics 679


PREP ® Self-Assessment PREPSA 2021

PREP Pearls
• Acute marijuana intoxication may lead to hypertension, tachycardia, sedation, and
psychosis.
• Although treatment of toxicity from plant-derived substances is mostly supportive,
antidotes to select toxins can be life-saving.

ABP Content Specifications(s)


• Recognize the signs and symptoms following the ingestion of a potentially toxic plant,
and manage appropriately

Suggested Readings
• Fine JS. Poisoning. In: McInerny TK, Adam HM, Campbell DE, DeWitt TG, Foy JM,
Kamat DM, eds. American Academy of Pediatrics Textbook of Pediatric Care. 2nd ed.
Itasca, IL: American Academy of Pediatrics; 2016;chap 369:2924-2949. Pediatric Care
Online.
• Ryan SA, Ammerman SD; Committee on Substance Use and Prevention. Counseling
parents and teens about marijuana use in the era of legalization of marijuana. Pediatrics.
2017;139(3):e20164069. doi: 10.1542/peds.2016-4069.
• Wang GS, Hoyte C. Common substances of abuse. Pediatr Rev. 2018;39(8):403-414.
doi: 10.1542/pir.2017-0267.

American Academy of Pediatrics 680


PREP ® Self-Assessment PREPSA 2021
Question 186
A 14-year-old adolescent boy is brought to the emergency department for evaluation of difficult
and painful swallowing. His symptoms began 5 hours ago after he ate ramen noodles and
chicken sausage. He noticed a feeling that something “was stuck” in his chest; he has been
retching and vomiting since then. He has a 2-year history of dysphagia but has never experienced
symptoms to this degree. He has a history of multiple environmental allergies, for which he
receives immunotherapy, and a history of anaphylaxis to eggs. On physical examination, the boy
is uncomfortable appearing and frequently retching and spitting clear liquid into the emesis basin
near his bed. His examination findings are otherwise normal. Chest radiograph is normal.

Of the following, the BEST next management step for this adolescent is
A. abdominal ultrasonography
B. endoscopy
C. epinephrine injection
D. esophagography

American Academy of Pediatrics 681


PREP ® Self-Assessment PREPSA 2021
Correct Answer: B
The adolescent in the vignette has acute esophageal obstruction, evidenced by chest pain,
retching, and vomiting. He is unable to tolerate oral secretions based on the frequent spitting of
clear liquid observed during the physical examination. Acute esophageal obstruction necessitates
endoscopic management to remove the foreign body (in this case, likely food, given his history
and negative chest radiography findings). During this boy’s endoscopic evaluation, the food
bolus was removed; findings concerning for eosinophilic esophagitis, such as linear ridging and
white plaques, were seen (Item C186).

American Academy of Pediatrics 682


PREP ® Self-Assessment PREPSA 2021
Eosinophilic esophagitis (EoE) is an immune-mediated inflammatory condition of the
esophagus, most commonly associated with non-IgE allergic response to food allergens
(including milk, egg, wheat, soy, nuts/peanuts, and fish/shellfish). Eosinophilic esophagitis often
presents with dysphagia and/or chronic reflux-like symptoms in older children. It is not
uncommon for older children to have chronic dysphagia to which they have acclimated by
chewing and eating slowly, cutting foods (meats in particular) into very small bites, and drinking
significant amounts of liquids while eating. A sensation of “food sticking” in the chest while
eating may be reported. The initial presentation may be an acute esophageal obstruction, as seen
in the vignette. Up to 88% of children with food impaction requiring therapeutic endoscopy have
EoE. Younger children with EoE often have more generalized symptoms, including vomiting,
feeding difficulties, and even failure to thrive. A history of atopy is common in children with
EoE.

Histologically EoE is defined as 15 or more eosinophils present in an esophageal biopsy sample,


after excluding other esophageal pathologies (generally gastroesophageal reflux disease
[GERD]). To decrease the risk that GERD will be misdiagnosed as EoE, children with suspected
EoE should be treated with a proton pump inhibitor for a minimum of 6 to 8 weeks before
diagnostic endoscopy. Following removal of the food impaction, the adolescent in the vignette
underwent proton pump inhibitor therapy for 12 weeks and then underwent repeat endoscopy,
which confirmed the suspected diagnosis of EoE.

Management of EoE can include dietary and/or medical therapies and should involve a
multidisciplinary team including a gastroenterologist, allergist, and dietitian. Dietary
modification includes removal of the foods most commonly associated with EoE. Unfortunately,
traditional food allergy testing does not always correlate with the offending food antigen in EoE.
Thus, empiric 3-, 4-, or 6-food elimination diets, or guided food elimination diets in conjunction
with an allergist, may be prescribed. Endoscopic surveillance is used to ensure histologic
remission, because clinical improvement in symptoms does not always correlate with histologic
remission. It is important to note that even the 3-food elimination diet (milk-, egg-, and wheat-
free diet) may be difficult to follow, and it is critical to involve a dietitian. Medical management
of EoE may include the use of swallowed topical steroids (fluticasone or oral viscous
budesonide) and proton pump inhibitors which may be used alone or in conjunction with dietary
modification. Rarely, children may not respond to elimination diets and/or medical therapies; in
such cases, an exclusive amino acid–based elemental formula may be necessary to induce
remission (administered orally, via nasogastric, or gastrostomy tube).

Abdominal ultrasonography would not be useful for the adolescent in this vignette. Because this
adolescent is not experiencing symptoms of anaphylaxis, epinephrine injection is not indicated.
Esophagography may be useful when there is a possible or partial esophageal obstruction
(vomiting, difficulty swallowing), but when the obstruction is complete (when the patient is
unable to swallow oral secretions), a contrast study may pose additional aspiration risk and could
coat the foreign body and esophageal mucosa, compromising the necessary endoscopy.

American Academy of Pediatrics 683


PREP ® Self-Assessment PREPSA 2021
PREP Pearls
• Esophageal obstruction presents clinically with chest pain, dysphagia, and the inability to
swallow fluids or oral secretions.
• Emergent consultation with a pediatric gastroenterologist and a therapeutic endoscopy are
indicated for esophageal obstruction.
• Older children with esophageal food impaction are likely to have eosinophilic
esophagitis.

MOCA-Peds Objective
• Manage foreign body ingestion.

ABP Content Specifications(s)


• Recognize the clinical features associated with eosinophilic or allergic esophagitis

Suggested Readings
• Adamiak T, Plati KF. Pediatric esophageal disorders: diagnosis and treatment of reflux
and eosinophilic esophagitis. Pediatr Rev. 2018;39(8):392-402. doi: 10.1542/pir.2017-
0266.
• Le-Carlson M, Kerner JA. Gastrointestinal allergy. In: McInerny TK, Adam HM,
Campbell DE, DeWitt TG, Foy JM, Kamat DM, eds. American Academy of Pediatrics
Textbook of Pediatric Care. 2nd ed. Itasca, IL: American Academy of Pediatrics; 2016;
chap 256:2076-2080. Pediatric Care Online.

American Academy of Pediatrics 684


PREP ® Self-Assessment PREPSA 2021
Question 187
The local sheriff arrives at the emergency department with an agitated, aggressive, disheveled
17-year-old adolescent girl in handcuffs. The police were called to a party at a fraternity house
and found her fighting with another girl in the front yard. The police tried to coax her into the
police car voluntarily, but they had to restrain her after she bit one police officer and punched
another in the face. In the emergency department, she is combative. She is shouting and cursing
at the police and hospital staff. On physical examination, she has a fever, hypertension,
tachycardia, tachypnea, and diaphoresis. Her pupils are dilated. She has left periorbital and lower
lip swelling and superficial abrasions on the dorsa of both hands. She has no cardiac murmur, her
lungs are clear to auscultation, and her abdomen is soft and nontender.

Of the following, the MOST likely substance causing the adolescent’s symptoms is
A. amphetamine
B. benzodiazepine
C. lysergic acid diethylamide
D. psilocybin

American Academy of Pediatrics 685


PREP ® Self-Assessment PREPSA 2021
Correct Answer: A
The adolescent described in the vignette is displaying symptoms of a sympathomimetic
toxidrome that are the hallmarks of an amphetamine intoxication. Excessive sympathetic
stimulation causes tachycardia, hypertension, hyperthermia, mydriasis, and diaphoresis.
Agitation, psychosis, or violent behavior can lead to injuries with significant morbidity or
mortality. Severe amphetamine intoxication can result in seizures, cardiac ischemia,
rhabdomyolysis, renal failure, electrolyte disturbances, and even death.

Amphetamine toxicity is generally a clinical diagnosis. Amphetamines are included on most


urine drug screening panels; however, because tests can have both false-negative and false-
positive results (pseudoephedrine can cause a false-positive result for amphetamines), reliance on
urine drug testing for a diagnosis of amphetamine toxicity is not recommended. Evaluation of a
child suspected to have acute amphetamine intoxication should include electrocardiography as
well as serum testing for acetaminophen, salicylates, and alcohol, all of which are common
coingestants.

Assessment and management of acute amphetamine intoxication in adolescents can be


challenging given adolescents’ propensity for violence. Safety of healthcare workers should be
considered before approaching the agitated patient. Although verbal de-escalation may
occasionally be effective, sedatives such as benzodiazepines are considered first-line therapy.
High doses may be necessary to achieve the desired effect. Benzodiazepines also play a key role
in the treatment of amphetamine-induced hypertension, hyperthermia, and seizures. Careful
attention should be paid to the increased risk of rhabdomyolysis if physical restraints are required
for patient and provider safety. As with all emergencies, evaluation begins with assessment of
the airway, breathing, and circulation. Intravenous fluids should be administered, because
amphetamine-intoxicated patients are at risk of experiencing dehydration. Psychosocial
evaluation and support are key aspects of treatment for adolescents with amphetamine
intoxication. Procurement of illicit substances often places adolescents in unsafe situations and
often exposes them to other harmful drugs. Amphetamine abusers may become tolerant,
necessitating higher doses for similar effects; addiction can also occur.

Benzodiazepines are used as anxiolytics and are commonly abused. Benzodiazepine overdose
causes bradycardia, hypotension, miosis, and respiratory depression. Lysergic acid diethylamide
(LSD), a common hallucinogen, results in a spectrum of neuropsychiatric symptoms, including
euphoria, synesthesia, and hyperacuity relating to the perception of colors and other objects.
Psilocybin is the hallucinogen in “magic mushrooms” and produces effects similar to LSD.

PREP Pearls
• Amphetamine intoxication presents with a sympathomimetic response and agitation.
• Benzodiazepines and supportive care are the mainstays of treatment for amphetamine
intoxication.

American Academy of Pediatrics 686


PREP ® Self-Assessment PREPSA 2021
ABP Content Specifications(s)
• Recognize the major behavioral consequences of amphetamine use/abuse
• Identify the major physiologic consequences associated with amphetamine use/abuse,
including those associated with the various means of administration
• Recognize the clinical findings associated with an acute amphetamine intoxication, and
manage appropriately

Suggested Readings
• Morgan JP. Amphetamine and methamphetamine during the 1990s. Pediatr
Rev. 1992;13;330. doi:10.1542/pir.13-9-330.
• Spyres MB, Jang DH. Amphetamines. In: Hoffman RS, Nelson LS, Goldfrank LR,
Howland MA, Lewin NA, Smith SW, eds. Goldfrank’s Toxicologic Emergencies. 11th
ed. New York, NY: McGraw-Hill; 2019:1099-1010.

American Academy of Pediatrics 687


PREP ® Self-Assessment PREPSA 2021
Question 188
A 16-year-old basketball player is being evaluated for persistent dizziness. One week ago she
collided with another player during a game. She continued to play, but within minutes,
developed a significant headache. She was diagnosed with a concussion in the emergency
department, and was advised to rest for several days and follow up with her primary care
physician. Currently, she is feeling markedly better, other than persistent dizziness and a sense of
being “off balance,” particularly with positional changes. Her physical examination findings are
notable for provocation of dizziness with head movement and position changes. Neurologic
findings are otherwise nonfocal. The remainder of her examination findings are unremarkable,
including full and pain-free range of motion of the neck.

Of the following, the BEST next management step for this girl would be
A. computerized neuropsychological testing
B. continued rest until symptoms resolve
C. magnetic resonance imaging of the brain
D. referral for vestibular rehabilitation

American Academy of Pediatrics 688


PREP ® Self-Assessment PREPSA 2021
Correct Answer: D
The symptoms reported by the adolescent in the vignette strongly suggest ongoing impairment of
her vestibular system. She would benefit from a referral for vestibular rehabilitation. Traditional
concussion treatment recommendations have focused on physical and cognitive rest while
passively waiting for spontaneous resolution of symptoms. However, more recent evidence
supports identifying those symptom subsets that are amenable to dedicated treatment and can
help avoid prolonged periods away from school and sports.

Patients with concussion symptoms involving the vestibulo-ocular system appear to be at


markedly higher risk for prolonged postconcussion symptoms. The vestibulo-ocular system helps
to maintain gaze stability with daily activity, and is critical for effective function in classroom
situations where students are constantly moving their heads to take notes, read/watch screens,
and engage with teachers and classmates. Common symptoms of vestibulo-ocular deficits
include dizziness, headache, “fogginess,” difficulty with reading, eye pain, and blurred or double
vision. In particular, patients reporting dizziness after an injury appear to be at 6 times greater
risk for prolonged recovery (>3 weeks) compared with other postconcussion symptoms.

A brief vestibulo-oculomotor screen has been validated for use in postconcussion patients, and
can be readily performed in 3 to 4 minutes during an office visit. This includes assessment of
oculomotor smooth pursuits, vertical and horizontal saccades, convergence, vestibular-ocular
reflex, and visual motion sensitivity. Descriptions of these examinations and an examination
template can be found in Mucha A, Collins MW, Elbin RJ, et al. (Appendices 1 and 2):

Identifying patients with ongoing vestibulo-ocular difficulties and referring them for
rehabilitation appears to enhance their recovery, but the optimal timing for initiating this process
is not yet known. If patients are experiencing rapid and ongoing recovery, it is reasonable to
monitor symptoms expectantly. However, for patients who have reached a plateau in their
recovery, or have slowly resolving symptoms that are significantly delaying a return to academic
activity, vestibular rehabilitation is warranted. An increasing number of physical therapists are
achieving competency in vestibular rehabilitation and incorporating this into comprehensive
concussion treatment protocols.

Although computerized neuropsychological testing) may be helpful for “return to play” decision-
making in young athletes, it is not diagnostic for concussion, and does not inform treatment
choice in patients who are still symptomatic. The tests are most useful when there is a preinjury
baseline result for comparison with results obtained after complete resolution of concussion-
related symptoms.

Prolonged periods of physical rest after a concussion have been shown to actually prolong
symptom recovery. Current guidelines, for most cases, recommend several days of relative rest
after concussion, followed by subsequent progressive return to levels of physical activity that do
not aggravate patients’ symptoms and do not place them at increased risk for additional head
injury. Progress can be self-monitored or under the guidance of an athletic trainer or physical
therapist. For families who prefer self-monitored activity, the following adaptation of the Buffalo
Concussion Treadmill test may be helpful. This is often best performed on a treadmill or
stationary bicycle, and requires a mechanism for monitoring heart rate (now found on many
American Academy of Pediatrics 689
PREP ® Self-Assessment PREPSA 2021
watches/fitness monitors, or embedded within cardiovascular training equipment at many fitness
facilities):
1. Have patient rate current symptoms at rest on a 1 to 10 Likert scale. The test is not
recommended for patients scoring more than 7 on this scale.
2. Start at the intensity of a brisk walk and slightly increase the intensity every minute
(resistance on a bike and incline on a treadmill).
3. Continue with graduated increases in intensity until symptoms increase significantly (ie,
an increase of 3 points on the Likert scale) or new symptoms develop. Patients should
note heart rate at the time that either of these occur.
4. Calculate 80% of the heart rate at the onset of increased or new symptoms. This becomes
the ceiling heart rate for any training/conditioning activity.
5. This test may be repeated every 5 to 7 days to assess for improvements in exercise
capacity.

It is important to recognize that even though physical activity is now encouraged earlier during
concussion recovery, criteria for return to contact sport remain unchanged and require: 1)
resolution of concussion-related symptoms at rest; 2) no provocation of symptoms with physical
exertion; and 3) return to baseline levels of cognitive function/academic performance.
Concussion is a clinical diagnosis; standard neuroimaging (eg, computed tomography and
magnetic resonance imaging) is only warranted to rule out other intracranial pathology.
Significant worsening of neurologic symptoms or focal neurologic findings on examination
would merit imaging in a postconcussion patient, but is not indicated for the adolescent in this
vignette.

PREP Pearls
• Dizziness in a patient after a concussion is predictive of prolonged recovery.
• Patients with persistent vestibular symptoms after a concussion should be identified and
referred for rehabilitation.
• A brief vestibulo-ocular assessment can be readily performed in the provider’s office.

MOCA-Peds Objective
• Identify contraindications for sports participation.

ABP Content Specifications(s)


• Plan the appropriate management of a sports-related ear injury

American Academy of Pediatrics 690


PREP ® Self-Assessment PREPSA 2021
Suggested Readings
• Corwin DJ, Wiebe DJ, Zonfrillo MR, et al. Vestibular deficits following youth
concussion. J Pediatr. 2015;166(5):1221-1225. doi: 10.1016/j.jpeds.2015.01.039.
• Halstead ME, Walter KD, Moffatt K; Council on Sports Medicine and Fitness. Sport-
related concussion in children and adolescents. Pediatrics. 2018;142(6):e2018307.
doi: 10.1542/peds.2018-3074.
• Kapadia M, Scheid A, Fine E, Zoffness R. Review of the management of pediatric post-
concussion syndrome-a multi-disciplinary, individualized approach. Curr Rev
Musculoskelet Med. 2019;12(1):57-66. doi: 10.1007/s12178-019-09533-x.
• Mucha A, Collins MW, Elbin RJ, et al. A brief Vestibular/Ocular Motor Screening
(VOMS) assessment to evaluate concussions: preliminary findings. Am J Sports
Med. 2014;42(10):2479-2486. doi: 10.1177/0363546514543775.

American Academy of Pediatrics 691


PREP ® Self-Assessment PREPSA 2021
Question 189
An 11-year-old boy is seen in the pediatric clinic for evaluation of fever. He has had daily fever
to at least 39°C for the last 10 days. Other symptoms include malaise, decreased appetite,
abdominal pain, and back pain. He has not had any rash, sick contacts, or contact with pets.
Three weeks ago, he travelled to Mexico and consumed cheese from his family's dairy farm. He
did not have contact with any farm animals. His vital signs include a temperature of 39.1°C,
heart rate of 95 beats/min, respiratory rate of 18 breaths/min, and blood pressure of 98/57 mm
Hg. Physical examination reveals hepatosplenomegaly but no lymphadenopathy.

Of the following, the organism MOST likely to be responsible for the boy’s illness is
A. Bartonella henselae
B. Brucella melitensis
C. Francisella tularensis
D. Salmonella typhi

American Academy of Pediatrics 692


PREP ® Self-Assessment PREPSA 2021
Correct Answer: B
The organism most likely to be responsible for the boy’s illness is Brucella
melitensis. Brucellosis is an important consideration in the differential diagnosis of fever of
unknown origin, particularly if recent history suggests a potential exposure to the organism. The
boy in the vignette had recently consumed cheese from his family’s dairy farm. Consumption of
unpasteurized dairy products is the top risk factor for brucellosis in children in the United States.

Brucellosis is a zoonotic illness transmitted by contact with tissue or fluid of infected animals,
including cattle, goats, sheep, and swine. It can be acquired by direct contact, inhalation, or
ingestion. The symptoms of brucellosis are nonspecific and can include fever, malaise, myalgias,
arthralgias, abdominal pain, and headache. Physical examination may reveal lymphadenopathy,
hepatosplenomegaly, and arthritis. The serum agglutination test is the most commonly used
diagnostic test and is the preferred serologic assay. Brucella can grow on cultures of specimens
such as blood and tissues.

Bartonella henselae is another important organism to consider when evaluating a child with a
fever of unknown origin. This zoonotic infection is typically acquired after contact with cats.
However, other animals, including dogs, can be infected. Transmission can also occur via
contact with infected fleas. The most common manifestation of B henselae infection is
lymphadenopathy/lymphadenitis. Disseminated Bartonella infections present as fever of
unknown origin; abdominal ultrasonography may reveal hepatosplenic lesions in children with
disseminated infection.

Francisella tularensis is the organism responsible for tularemia. There are several clinical forms
of illness, depending on the means by which the organism is acquired: ulceroglandular,
glandular, oculoglandular, oropharyngeal, pneumonic, and typhoidal. Disease after a tick bite can
manifest as an eschar at the bite site with local adenopathy. If inhaled, pulmonic disease can
develop. Typhoidal tularemia manifests as high fever and hepatosplenomegaly. Infection is most
commonly acquired via tick bites, but can also occur with other arthropod bites, direct contact
with infected animals, inhalation of contaminated aerosols, or ingestion of contaminated food.

Salmonella typhi infection should be considered when evaluating fever in an individual who has
traveled abroad. Symptoms can include abdominal pain and either diarrhea or constipation.
Findings on physical examination can include hepatosplenomegaly and rose spots. Unlike
other Salmonella infections, typhoid fever is not a zoonotic infection. It is spread from person to
person, typically from improper hygiene by an infected food handler. In the United States, most
cases occur after travel to endemic countries, especially those in Asia.

PREP Pearls
• Brucellosis is a zoonotic illness transmitted by contact with tissue or fluid of infected
animals, including cattle, goats, sheep, and swine.
• Consumption of unpasteurized dairy products is the top risk factor for brucellosis in
children in the United States.
• The symptoms of brucellosis are nonspecific and can include fever, malaise, myalgias,
arthralgias, abdominal pain, and headache. Physical examination may reveal
lymphadenopathy, hepatosplenomegaly, and arthritis.
American Academy of Pediatrics 693
PREP ® Self-Assessment PREPSA 2021

MOCA-Peds Objective
• Evaluate and manage a child with unexplained persistent fever.

ABP Content Specifications(s)


• Understand the importance of considering brucellosis in the differential diagnosis of
fever of unknown origin

Suggested Readings
• American Academy of Pediatrics. Brucellosis. In: Kimberlin DW, Brady MT, Jackson
MA, Long SS, eds. Red Book: 2018 Report of the Committee on Infectious Diseases.
Itasca, IL: American Academy of Pediatrics; 2018:255-257. Red Book Online.
• Antoon J, Potisek N, Lohr J. Pediatric fever of unknown origin. Pediatr
Rev. 2015;36(9):380-391. doi: 10.1542/pir.36-9-380.
• van der Jagt EW. Fever of unknown origin. In: McInerny TK, Adam HM, Campbell DE,
DeWitt TG, Foy JM, Kamat DM, eds. American Academy of Pediatrics Textbook of
Pediatric Care. 2nd ed. Itasca, IL: American Academy of Pediatrics; 2016;chap
153:1361-1365. Pediatric Care Online.

American Academy of Pediatrics 694


PREP ® Self-Assessment PREPSA 2021
Question 190
A 9-month-old infant is brought to the office for evaluation of a white bump on his penis. He
was circumcised at 2 days of age and has no other significant medical history. He has a 3-mm,
firm, white nodule on the dorsal side of his penis, just proximal to the corona. A remnant of
foreskin extends approximately 5 mm beyond the edge of the corona on the dorsal side and is
adherent to the glans in this area. The remainder of the findings from his genitourinary
examination are normal.

Of the following, the MOST appropriate next step in management is


A. application of triamcinolone ointment to the glans
B. daily manual retraction of the foreskin
C. reassurance and routine hygiene recommendations
D. referral to a urologist for circumcision revision

American Academy of Pediatrics 695


PREP ® Self-Assessment PREPSA 2021
Correct Answer: C
The infant in the vignette has smegma and a small adhesion of the foreskin. Both findings are
benign and reassurance with routine hygiene recommendations is the most appropriate next step
in management.

At birth the foreskin or prepuce is adhered to the glans penis. In uncircumcised infants, the
epithelial layers of the foreskin and glans begin to separate during the first few years after birth.
The age at which the foreskin is completely retractable is variable; approximately 1% of
uncircumcised 7th grade boys still have a physiologic phimosis. Pathologic phimosis is caused
by scarring of the foreskin and increases the risk of other complications including paraphimosis,
recurrent urinary tract infections, and balanoposthitis. Topical corticosteroids and gentle
retraction are generally sufficient in most cases of pathologic phimosis, with surgical
intervention offered only rarely. Remnants of foreskin can readhere to the glans after
circumcision. This tissue typically separates from the glans over time (as it would if the child had
not been circumcised) and so treatment is unnecessary unless the adhesion is obstructing the
urethral opening.

Smegma is a collection of epithelial cells and sebaceous secretions. It can accumulate under the
foreskin of uncircumcised boys or under remnants of foreskin in those who were circumcised.
Triamcinolone would be a first-line treatment for a pathologic phimosis, but is not necessary for
the small adhesion that is not obstructing the urethral opening seen in the infant in the vignette.
Forceful retraction of the foreskin is painful and can cause bleeding and local inflammation that
may lead to even stronger adherence of the foreskin to the glans. Circumcision revision is not
indicated for this patient.

PREP Pearls
• Physiologic phimosis (ie, the natural adherence of the foreskin to the glans penis)
resolves over time, with the majority of foreskins being fully retractable before the onset
of puberty.
• Pathologic phimosis is caused by scarring of the foreskin and increases the risk of other
complications including paraphimosis, recurrent urinary tract infections, and
balanoposthitis. Topical corticosteroids and gentle retraction are first-line treatment for
pathologic phimosis.
• Smegma is a collection of epithelial cells and sebaceous secretions that accumulate under
the foreskin of uncircumcised boys or under remnants of foreskin in those who were
circumcised.

MOCA-Peds Objective
• Counsel parents regarding circumcision.

ABP Content Specifications(s)


• Recognize the significance of smegma accumulation beneath an infant’s prepuce

American Academy of Pediatrics 696


PREP ® Self-Assessment PREPSA 2021
Suggested Readings
• American Academy of Pediatrics Task Force on Circumcision. Male
circumcision. Pediatrics. 2012:130(3):e756-e785. doi: 10.1542/peds.2012-1990.
• Lawless MR. The foreskin. Pediatr Rev. 2006;27(12):477-488. doi: 10.1542/pir.27-12-
477.
• Yang C, Liu X, Wei GH. Foreskin development in 10,421 Chinese boys aged 0-18
years. World J Pediatr. 2009;5(4):312-315. doi: 10.1007/s12519-009-0060-z.

American Academy of Pediatrics 697


PREP ® Self-Assessment PREPSA 2021
Question 191
A 17-year-old adolescent boy is seen in the clinic with a several-year history of epigastric pain
that has significantly worsened over the last 6 months. He was prescribed proton pump inhibitor
therapy 2 months ago. Despite good medication adherence, he has continued to have abdominal
pain. He has tried over-the-counter antacids with some mild and transient improvement in
symptoms. He denies vomiting, regurgitation, heartburn, and weight loss. He moved from Nepal
2 years ago. The adolescent is otherwise healthy and has no known drug allergies. His physical
examination is notable for a weight of 53.7 kg (11th percentile for age), height of 165 cm (10th
percentile for age), and body mass index of 19.7 kg/m2 (30th percentile for age). His abdomen is
soft, with epigastric tenderness. There is no hepatosplenomegaly. The remainder of his
examination findings are within normal limits.

Recent laboratory studies show the following:


Laboratory Test Result
Hemoglobin 15.5 g/dL (155 g/L)
Alanine aminotransferase 27 U/L
Aspartate aminotransferase 32 U/L
Albumin 4.7 g/dL

Endoscopy reveals nodularity in the gastric antrum and a duodenal ulcer (Item Q191). Biopsies
demonstrate chronic gastritis and erosive duodenitis, both with the presence of gram-negative,
spiral-shaped bacilli. Eradication therapy is recommended to address his suspected condition.

American Academy of Pediatrics 698


PREP ® Self-Assessment PREPSA 2021

Item Q191: Duodenal ulcer. Courtesy of J Sullivan

Of the following, the BEST indication for this treatment is


A. chronic gastritis
B. duodenal ulcer
C. epigastric pain
D. regurgitation

American Academy of Pediatrics 699


PREP ® Self-Assessment PREPSA 2021
Correct Answer: B
The adolescent in the vignette has Helicobacter pylori and a duodenal ulcer, therefore
eradication treatment for H pylori is recommended.

Helicobacter pylori, a gram-negative, spiral-shaped bacillus, infects more than half of the world
population, and many of the affected individuals are asymptomatic. Most of those affected are
adults, because the prevalence of H pylori infection increases with age. The overall prevalence
of H pylori infection in North America is 37%; in Africa the prevalence is 70%. Adults are more
likely than children to develop complications of H pylori infection, including peptic ulcer disease
and gastric cancer. Thus, recommendations for the diagnosis and treatment of H pylori infection
differ in children compared with adults.

Eradication treatment is recommended for all children with H pylori–associated gastric and/or
duodenal ulcer disease by the 2016 European Society for Pediatric Gastroenterology,
Hepatology, and Nutrition (ESPGHAN) and the North American Society for Pediatric
Gastroenterology, Hepatology, and Nutrition (NASPGHAN) guidelines for the management of H
pylori in children. Both the ulceration and symptoms will respond to treatment of H
pylori infection. Because the presence of ulceration is a key factor considered in the management
plan, making the diagnosis of H pylori infection via endoscopy is the gold standard. Noninvasive
testing for H pylori infection (stool antigen testing, urease breath testing) is not indicated for
diagnosis of H pylori, but is used to assess treatment response. Endoscopy should be offered as a
means to diagnose the cause of the child’s symptoms (abdominal pain) and not solely to
diagnose H pylori infection.

Children with chronic abdominal pain (including chronic gastritis or epigastric pain) or
regurgitation and histologically proven H pylori infection without a gastric or duodenal ulcer do
not always experience symptom improvement with H pylori treatment. Thus, eradication therapy
in such cases should be a shared decision between the gastroenterologist and the family.
Eradication treatment of H pylori infection includes a proton pump inhibitor and antibiotics for
14 days, as detailed in Item C191A and Item C191B. Treatment effect should be assessed with a
urease breath test or H pylori stool antigen test 4 weeks after completion.

American Academy of Pediatrics 700


PREP ® Self-Assessment PREPSA 2021

Item C191A: Susceptibility-based treatment for Helicobacter pylori. (Adapted. from Jones NL,
Koletzko S, Goodman K, et al; ESPGHAN, NASPGHAN. Joint ESPGHAN/NASPGHAN
guidelines for the management of Helicobacter pylori in children and adolescents (update 2016).
J Pediatr Gastroenterol Nutr. 2017;64(6):991–1003.)
Reprinted with permission from Sierra D, Wood M, Kolli S, Felipiez LM. Pediatric Gastritis,
Gastropathy, and peptic ulcer disease. Pediatr Rev 2018;29;546.

American Academy of Pediatrics 701


PREP ® Self-Assessment PREPSA 2021
Item C191B: Weight-based doses for Helicobacter pylori treatment.
Reprinted with permission from Sierra D, Wood M, Kolli S, Felipiez LM. Pediatric Gastritis,
Gastropathy, and peptic ulcer disease. Pediatr Rev 2018;29;547.

PREP Pearls
• Eradication treatment for Helicobacter pylori infection is recommended for H pylori–
associated gastric and/or duodenal ulcers. Treatment for H pylori infection in the absence
of ulcer disease may not successfully resolve the child’s symptoms.
• Endoscopy with testing for Helicobacter pylori is the gold standard for diagnosis of H
pylori infection in children.
• Noninvasive testing (eg, stool Helicobacter pylori antigen or urease breath testing) is
indicated for monitoring treatment success; these tests should not be used to make an
initial H pylori diagnosis.

ABP Content Specifications(s)


• Plan the diagnostic evaluation of Helicobacter pylori infection
• Identify risk factors for Helicobacter pylori infection
• Recognize the major clinical features associated with Helicobacter pylori infection
• Plan appropriate management for a patient with Helicobacter pylori infection

Suggested Readings
• Jones NL, Koletzko S, Goodman K, et al, Joint ESPGHAN/NASGPHAN guidelines for
the management of Helicobacter pylori in children and adolescents (update 2016). J
Pediatr Gastroenterol Nutr. 2017;64(6):991-1003.
doi: 10.1097/MPG.0000000000001594.
• Loizides AM, Orellana KA, Thompson JF. Abdominal pain. In: McInerny TK, Adam
HM, Campbell DE, DeWitt TG, Foy JM, Kamat DM, eds. American Academy of
Pediatrics Textbook of Pediatric Care. 2nd ed. Itasca, IL: American Academy of
Pediatrics; 2016;chap 125:1181-1187. Pediatric Care Online.
• Sierra D, Wood M, Kolli S, Felipiez LM. Pediatric gastritis, gastropathy, and peptic ulcer
disease. Pediatr Rev. 2018;29;542-549. doi: 10.1542/pir.2017-0234.

American Academy of Pediatrics 702


PREP ® Self-Assessment PREPSA 2021
Question 192
A 15-year-old adolescent girl is brought to the office by her mother for evaluation of irregular
periods. She had menarche at age 12 years, and her periods were regular for the first 1.5 to 2
years. Over the past 12 months, she has had a period every 1 to 3 months. Her last menstrual
period was 4 months ago. Since her health supervision visit 1 year ago, she has gained 9 kg,
which she attributes to a poor diet. She has had intermittent flares of acne but no headaches,
visual changes, abnormal hair growth, or nipple discharge. There is no family history of
menstrual disorders. During the confidential psychosocial interview, she reports that she has
never been sexually active.

She has a temperature of 37°C, a heart rate of 80 beats/min, a blood pressure of 120/80 mm Hg,
and a respiratory rate of 18 breaths/min. She has a weight of 68 kg (>90th percentile), height of
164 cm (50th percentile), and body mass index of 25.7 kg/m2 (92nd percentile). She has
scattered closed comedones on her forehead and cheeks and mild hair growth on her upper lip
and lower abdomen. The remainder of her physical examination findings are unremarkable.

Of the following, the test(s) MOST likely to help confirm the diagnosis is (are)
A. follicle-stimulating hormone and luteinizing hormone
B. free and total testosterone
C. thyroid-stimulating hormone and free thyroxine
D. urine human chorionic gonadotropin

American Academy of Pediatrics 703


PREP ® Self-Assessment PREPSA 2021
Correct Answer: B
The adolescent girl in the vignette has ovulatory dysfunction and signs of hyperandrogenemia
(acne and hirsutism), and she is overweight, with a 9-kg weight gain over the past year. This
group of signs and symptoms places polycystic ovary syndrome (PCOS) high on the differential
diagnosis. Of the response choices listed, tests of free and total testosterone levels are the most
sensitive in making the diagnosis of hyperandrogenism and PCOS.

Polycystic ovary syndrome is characterized by ovulatory dysfunction, hyperandrogenism, and


polycystic ovaries. It affects 6% to 10% of reproductive-aged women. It is the most common
cause of hyperandrogenic anovulation and infertility in young women. Clinical symptoms
include acne, hirsutism, obesity, and menstrual irregularities such as amenorrhea (primary and
secondary), oligomenorrhea, and abnormal uterine bleeding. Women with PCOS are often at risk
of experiencing insulin resistance, diabetes mellitus, and cardiovascular disease such as
hypertension and dyslipidemia.

The exact etiology of PCOS is unknown, and the diagnosis can be confusing, as there are three
sets of diagnostic criteria established by different expert groups (Item C192). Hyperandrogenism
can be based on a combination of clinical features (acne or hirsutism) or biochemical evaluation
(elevated androgen levels). Hirsutism is scored via the Ferriman-Gallwey Hirsutism Scoring
System. A score of 8 or higher is considered abnormal hair growth. The most sensitive test for
the assessment of hyperandrogenemia in women is a free testosterone level. This can also be
determined from calculations by using the total testosterone and sex hormone–binding globulin
levels. Ovulatory dysfunction is defined as having a menstrual cycle less than every 21 days or
more than every 45 days. Polycystic ovarian morphology on ultrasonogram is 12 or more
follicles in one or both ovaries (2-9 mm in diameter) and an ovarian volume greater than 10 mL.
In adolescent girls, the diagnosis of PCOS is often made on the basis of anovulatory cycles and
evidence of hyperandrogenemia.

American Academy of Pediatrics 704


PREP ® Self-Assessment PREPSA 2021

Polycystic ovarian syndrome is often a diagnosis of exclusion when a female exhibits signs of
hyperandrogenism or irregular periods. The workup may include laboratory screenings for late-
onset congenital adrenal hyperplasia, thyroid disease, hyperprolactinemia, an androgen-secreting
tumor, and primary ovarian failure.

Follicle-stimulating hormone and luteinizing hormone, thyroid-stimulating hormone and free


thyroxine, and urine human chorionic gonadotropin can all be included in the workup to
facilitate evaluation for hyperandrogenemia and menstrual abnormalities, but the most sensitive
test to yield the diagnosis of PCOS is free and total testosterone.

PREP Pearls
• Polycystic ovary syndrome is the most common cause of chronic anovulation and
infertility in young women.
• The diagnosis of polycystic ovary syndrome in adolescent girls is often based on
evidence of hyperandrogenemia and ovulatory dysfunction.
• The most sensitive test for diagnosis in women is a free and total testosterone level.

ABP Content Specifications(s)


• Recognize the clinical findings associated with secondary amenorrhea of various
etiologies, and manage appropriately
• Identify the clinical findings associated with polycystic ovary syndrome
• Recognize the clinical findings associated with primary amenorrhea of various etiologies,
and manage appropriately

American Academy of Pediatrics 705


PREP ® Self-Assessment PREPSA 2021

Suggested Readings
• ACOG practice bulletin no. 194: polycystic ovarian syndrome. Obstet
Gynecol. 2018;131(6):e157-e171. doi:10.1097/AOG.0000000000002656.
• Ibáñez L, Oberfield SE, Witchel S, et al. An International Consortium update:
pathophysiology, diagnosis, and treatment of polycystic ovarian syndrome in
adolescence. Horm Res Paediatr. 2017;88(6):371-395. doi:10.1159/000479371.
• Klein GW, Gangat M. Hirsutism, hypertrichosis, and precocious sexual hair
development. In: McInerny TK, Adam HM, Campbell DE, DeWitt TG, Foy JM, Kamat
DM, eds. American Academy of Pediatrics Textbook of Pediatric Care. 2nd ed. Elk
Grove Village, IL: American Academy of Pediatrics; 2017:1444-1451. Pediatric Care
Online .
• McCartney CR, Marshall JC. Polycystic ovary syndrome. N Engl J Med.2016;375(1):54-
64. doi:10.1056/NEJMcp1514916.
• Rosenfeld RL. The diagnosis of polycystic ovary syndrome in adolescents. Pediatrics.
2015;136(6):1154-1165. doi:10.1542/peds.2015-1430.

American Academy of Pediatrics 706


PREP ® Self-Assessment PREPSA 2021
Question 193
A term male newborn is evaluated in the neonatal intensive care unit. The pregnancy was
complicated by reduced fetal movement and increased amniotic fluid. He was intubated
immediately after birth because of respiratory distress and hypotonia. He has required continued
respiratory support. He has a tented upper lip, hypotonia, and bilateral clubfeet. His mother has
droopy eyelids and difficulty releasing her grip after shaking hands. She reports a personal
history of having a cardiac pacemaker and bilateral cataracts.

Of the following, the MOST likely diagnosis in this neonate is


A. Duchenne muscular dystrophy
B. Huntington disease
C. myotonic dystrophy type 1
D. Prader-Willi syndrome

American Academy of Pediatrics 707


PREP ® Self-Assessment PREPSA 2021
Correct Answer: C
The neonate in the vignette has type 1 myotonic dystrophy (DM1), also referred to as Steinert
disease. Clinical features are varied and can range from mild to severe. Three types exist: mild,
classic, and congenital. Patients with mild DM1 may have clinical features that include cataracts
and mild myotonia; their life span is normal. Some patients with mild DM1 can have diabetes
mellitus. Affected individuals’ condition can go undiagnosed owing to the mild nature of their
phenotype. Classic DM1 presents with muscle weakness and wasting, myotonia, cardiac
conduction abnormalities, and cataracts. Affected individuals usually exhibit symptoms in their
20s, although facial features such as droopy eyelids, weak smile, and thin face can be seen in
childhood. Muscle weakness is distal in location, and this leads to foot drop and gait instability.
Fine motor tasks become progressively difficult to perform over time. Hand grip myotonia, as
described in the mother in the vignette, can affect activities of daily living. Cardiac conduction
abnormalities leading to sudden cardiac death are a common cause of mortality in classic DM1.
Endocrine abnormalities of all hormone-secreting organs are known to be associated with this
condition.

At the severe end of the spectrum, congenital DM1, as seen for the patient in the vignette,
presents at birth. Prenatal history in congenital DM1 includes decreased fetal movement and
increased amniotic fluid owing to the fetus’ inability to suck and swallow normally. After
delivery, neonates demonstrate hypotonia and respiratory distress that requires support. Facial
muscle weakness leads to the classic feature of tented upper lip. Infants that survive the initial
period of respiratory insufficiency show gradual improvement in motor function in early
childhood. Weakness and myotonia become more evident in the second decade of life, similar to
classic DM1.

Type 1 myotonic dystrophy is caused by expansion of CTG trinucleotide repeats in the


noncoding region of the gene DMPK. The CTG repeat ranges for the three types have a
considerable amount of overlap. Hence, the repeat size is not a predictor of disease severity. A
general rule is that longer repeats correlate with increased severity and earlier age of onset. The
repeats can be unstable and expand in length during meiosis, leading to a higher repeat size in the
child transmitted from a parent with a lower repeat size. This is called genetic anticipation, in
which each successive generation experiences an increase in disease severity and an earlier age
of disease onset. Typically, a neonate with congenital DM1 inherits the expanded CTG allele
from the mother as described for the patient in the vignette, although transmission from the
father has also been reported. Type 1 myotonic dystrophy is inherited in an autosomal-dominant
manner. Males and females are equally affected. Recurrence risk is 50% with every child.
Diagnosis of DM1 is made via CTG repeat analysis of the DMPK gene. Treatment is largely
supportive, with care given to increased frequency of cardiac conduction abnormalities and
endocrine defects.

Duchenne muscular dystrophy is an X-linked recessive disorder. The presentation is not in the
neonatal period. Affected males exhibit muscle weakness and elevated creatine kinase level in
early childhood, and they experience delay in achieving motor milestones. Carrier females can
present with cardiomyopathy and mild muscle weakness but not grip myotonia or cataracts.
Huntington disease is characterized by motor, cognitive, and psychiatric difficulties; it typically
manifests around age 35 to 45 years. The neonatal period is normal, and grip myotonia and
American Academy of Pediatrics 708
PREP ® Self-Assessment PREPSA 2021
cataracts are not seen. Neonates with Prader-Willi syndrome have neonatal hypotonia. However,
the mother’s history of droopy eyelids, grip myotonia, cardiac pacemaker, and bilateral cataracts
makes Prader-Willi syndrome unlikely.

PREP Pearls
• Myotonic dystrophy type 1 is an autosomal dominant disorder caused by CTG
trinucleotide repeat expansion of the gene DMPK.
• Congenital myotonic dystrophy type 1 presents in the neonatal period with hypotonia and
respiratory compromise.
• Genetic anticipation is seen in myotonic dystrophy type 1 with increased disease severity
and earlier age of onset from one generation to the next.

ABP Content Specifications(s)


• Understand the impact of genetic anticipation on the presentation of genetic disorders

Suggested Readings
• Bird TD. Myotonic dystrophy type 1. In: Adam MP, Ardinger HH, Pagon RA, et al,
eds. GeneReviews. 1999 Sep 17 [updated 2019 Oct 3]. Seattle, WA: University of
Washington, Seattle; 1993-2020.
• Hassib N. Index of suspicion. Pediatr Rev. 2003;24(9):311-316. doi:10.1542/pir.24-9-
311.

American Academy of Pediatrics 709


PREP ® Self-Assessment PREPSA 2021
Question 194
A 17-year-old adolescent girl is being evaluated for a vaginal discharge. She has had increased
white discharge during the past week without pain or itching. She has had three sexual partners
in the past year and uses condoms most of the time. On physical examination, she has a thin
white vaginal discharge with a fishlike odor. A wet mount of the discharge is significant for clue
cells. Results of testing for chlamydia and gonorrhea and a urine pregnancy test are negative.

Of the following, the BEST treatment for this patient’s condition is


A. azithromycin
B. doxycycline
C. metronidazole
D. reassurance

American Academy of Pediatrics 710


PREP ® Self-Assessment PREPSA 2021
Correct Answer: C
The adolescent in the vignette has bacterial vaginosis, which is best treated with metronidazole.
Doxycycline is used to treat chlamydia; azithromycin (with ceftriaxone) is used to treat Neisseria
gonorrhœae. The girl in the vignette is symptomatic, so reassurance is not sufficient.

Bacterial vaginosis occurs when the common vaginal flora of lactobacillus organisms are
replaced by anaerobic bacteria (most frequently Gardnerella vaginalis). It most commonly
occurs in sexually active postpubescent girls. Symptoms include a thin white or gray vaginal
discharge with a fishy odor. The discharge may increase after intercourse or with menses.
Bacterial vaginosis typically is not associated with dysuria or abdominal pain, which are more
commonly present with sexually transmitted infections. Testing for sexually transmitted
infections should be performed as part of the evaluation of sexually active patients with bacterial
vaginosis.

In prepubescent girls, bacterial vaginosis can be associated with a vaginal foreign body, infection
with Escherichia coli or shigella, or sexually transmitted infections. In this age group, a complete
history should be obtained and physical examination performed to evaluate for sexual abuse.
First-line treatment for bacterial vaginosis is oral or intravaginal metronidazole. Patients should
be cautioned against consuming alcohol while taking this medication. Oral or intravaginal
clindamycin can be used as an alternative. Even with appropriate treatment, 30% of patients will
experience recurrence within 3 months.

PREP Pearls
• Bacterial vaginosis most frequently is caused by overgrowth of Gardnerella vaginalis.
• Bacterial vaginosis is treated with oral or intravaginal metronidazole, and it often recurs
after treatment.
• Prepubsecent girls with bacterial vaginosis should be evaluated for evidence of sexual
abuse.

ABP Content Specifications(s)


• Recognize the etiology of a vaginal discharge in patients of various ages and manage
appropriately

Suggested Readings
• American Academy of Pediatrics. Bacterial vaginosis. Healthychildren.org.
• American Academy of Pediatrics. Bacterial vaginosis. In: Kimberlin DW, Brady MT,
Jackson MA, Long SS, eds. Red Book: 2018-2021 Report of the Committee on Infectious
Diseases. 31st ed. Itasca, IL: American Academy of Pediatrics; 2018:239-242. Red Book
Online .
• Joffe A. Sexually transmitted infections. In: McInerny TK, Adam HM, Campbell DE,
DeWitt TG, Foy JM, Karnat DM, eds. American Academy of Pediatrics Textbook of
Pediatric Care. 2nd ed. Elk Grove Village, IL: American Academy of Pediatrics;
2017:2628-2651. Pediatric Care Online.
• O'Brien G. Bacterial vaginosis. Pediatr Rev. 2008;29(6)209-211. doi:10.1542/pir.29-6-
209.

American Academy of Pediatrics 711


PREP ® Self-Assessment PREPSA 2021
Question 195
A 10-year-old boy is seen for a health supervision visit. The boy and his family have no concerns
about his health. His medical and surgical history are notable for recurrent otitis media. Bilateral
tympanostomy tubes were placed 5 years ago and fell out after about 1 year. The appearance of
his tympanic membrane is noted in Item Q195. The findings of his hearing screening are
normal.

Item Q195: Appearance of the tympanic membrane of the patient described in the vignette.
Courtesy of J McClay

Of the following, the MOST likely diagnosis for this patient is


A. cholesteatoma
B. chronic otitis media with effusion
C. myringosclerosis
D. tympanic membrane perforation

American Academy of Pediatrics 712


PREP ® Self-Assessment PREPSA 2021
Correct Answer: C
The boy in this vignette has myringosclerosis in the setting of recurrent otitis media after
tympanostomy tube placement and extrusion. Myringosclerosis is characterized clinically by
calcium and phosphate crystal deposits within the tympanic membrane that appear as whitish
plaques on otoscopy and may demonstrate decreased mobility on pneumatic otoscopy.
According to a meta-analysis, 32% of patients developed myringosclerosis after tympanostomy
tube extrusion. Myringosclerosis is typically asymptomatic and rarely requires treatment, as
hearing loss that involves the ossicles is uncommon. Pediatricians should avoid using the term
“scarring of the tympanic membrane” with parents, as it may cause angst; histologically, scarring
(collagen deposition) does not occur with myringosclerosis.

Diseases of the middle ear, occurring either before or after an episode of acute otitis media,
include eustachian tube dysfunction, middle ear effusion, and chronic serous otitis media.
Chronic otitis media with effusion typically occurs after an episode of acute otitis media and may
persist for weeks to months. However, unlike acute otitis media, signs of infection, such as fever
and otalgia, are absent. When symptoms are present, they manifest most commonly as hearing
loss, ear fullness, and balance problems. Otoscopic findings of middle ear disease include a
retracted tympanic membrane, effusion with or without air fluid levels behind the tympanic
membrane, opacification of the tympanic membrane, and impaired tympanic membrane mobility
when positive pressure is applied via pneumatic otoscopy. Complications of prolonged middle
ear disease include development of retraction pockets, hearing loss, tympanosclerosis, and
cholesteatoma.

Cholesteatoma is a benign skin tumor typically located in the middle ear and mastoid space.
Clinical suspicion of a cholesteatoma should arise when otoscopic examination of the tympanic
membrane reveals a vague, irregular white mass that may resemble tympanosclerosis and may
protrude from the tympanic membrane with or without recurrent otorrhea. Congenital
cholesteatomas may occur but are less common than acquired cholesteatomas resulting from a
history of middle ear disease.

Risk factors for the development of recurrent or chronic middle ear disease include a family
history of otitis media, formula feeding (instead of breastfeeding), prolonged bottle use,
adenoidal hypertrophy, secondhand smoke exposure, day care attendance, and low
socioeconomic status. Allergic rhinitis may be associated with middle ear disease (as
demonstrated in observational studies). Gastroesophageal reflux disease has not been proven to
cause middle ear disease. The tympanic membrane shown in Item C195 does not show a
perforation.

American Academy of Pediatrics 713


PREP ® Self-Assessment PREPSA 2021

Item C195: Tympanic membrane with myringosclerosis and no evidence of perforation.


Courtesy of J McClay

PREP Pearls
• Myringosclerosis is characterized clinically by asymptomatic calcium and phosphate
crystal deposits within the tympanic membrane that appear as whitish plaques on
otoscopy.
• Myringosclerosis is typically asymptomatic and rarely requires treatment, because
hearing loss that involves the ossicles is uncommon.
• Complications of prolonged middle ear disease include development of retraction
pockets, hearing loss, tympanosclerosis, and cholesteatoma.

ABP Content Specifications(s)


• Recognize conditions associated with recurrent otitis media
• Plan the appropriate management of recurrent otitis media, including follow-up
evaluation and when an exacerbation has occurred

American Academy of Pediatrics 714


PREP ® Self-Assessment PREPSA 2021
Suggested Readings
• Kay DJ, Nelson M, Rosenfeld RM. Meta-analysis of tympanostomy tube
sequelae. Otolaryngol Head Neck Surg. 201;124(4):374-380.
doi:10.1067/mhn.2001.113941.
• Nguyen CV, Parikh SR. In brief: Cholesteatoma. Pediatr Rev. 2008;29(9):330-331.
doi:10.1542/pir.29-9-330.
• Rosa-Olivares J, Porro A, Rodriguez-Varela M, Riefkohl G, Niroomand-Rad I. Otitis
media: to treat, to refer, to do nothing—a review for the practitioner. Pediatr
Rev. 2015;36(11):480-486. doi:10.1542/pir.36-11-480.
• Tan TQ. Otitis media and otitis externa. In: McInerny TK, Adam HM, Campbell DE,
DeWitt TG, Foy JM, Kamat DM, eds. American Academy of Pediatrics Textbook of
Pediatric Care. 2nd ed. Elk Grove Village, IL: American Academy of Pediatrics;
2017:2452-2457. Pediatric Care Online .

American Academy of Pediatrics 715


PREP ® Self-Assessment PREPSA 2021
Question 196
A 7-year-old boy is seen in the pediatric neurology clinic at the local children’s hospital for
evaluation. He has a rapidly progressive neurodegenerative disease that is causing loss of
coordination, interaction, cognition, and speech. The neurologist decides to enroll him in a
research study to determine the etiology of his undiagnosed condition. Appropriate counseling is
performed, and the parents consent to the research study on behalf of their son. The consent
includes performing whole-genome analysis on a peripheral blood sample and discussion of his
case at relevant regional and national meetings. The researcher updates the family with possible
etiologies after discussion of his case at a national meeting.

Of the following, the principle of research involving children that is BEST outlined in the
vignette is
A. children should never be harmed by their participation in research
B. ethical research benefits children
C. research involving children must be just and equitable
D. respecting the dignity of children is core to ethical research

American Academy of Pediatrics 716


PREP ® Self-Assessment PREPSA 2021
Correct Answer: B
Research involving children carries a concern of maintaining human dignity and ensuring that
the rights and welfare of the child are being respected. This would include all research in which a
child is taking part, either directly or indirectly, across the range of research context, including
evaluation, monitoring, and implementation. For a researcher, it is important to consider ethical
issues at the planning and implementation stages of the study, as well as during the
postdissemination stage of research. Children are considered a vulnerable population owing to
their limited autonomy and reliance on parents or guardians for consent to research participation.
The institutional review board is a committee that reviews the methods proposed for research to
ensure that they are ethical in the context of the study. The institutional review board has
strategies in place to help overcome the issue of limited autonomy. One of these is the
requirement of direct benefit and documentation of assent. Assent, in a research study context,
considers the intellectual capacity and prospect of direct benefit for the participant. Other
mechanisms employed by the institutional review board are to include pediatric expertise in the
committee, ensure minimal-risk outcomes during review of study protocols, and include training
requirements for personnel involved in the study.

The case described in the vignette is demonstrating the ethical principle of research benefiting
children. The neurologist is conducting a research study on an undiagnosed neurodegenerative
disease. The family is counseled about and provides consent for the child’s participation in the
study, which includes performing whole-genome sequencing analysis. The neurologist then
discusses the patient’s case at regional and national meetings. After discussion, he or she updates
the family with possible etiologies to aid in the patient’s diagnosis. This is all indicative of the
benefit that the patient and any other child with a similar neurodegenerative condition might
receive from this research study. Because neurodegenerative conditions in the pediatric
population are rare and require multidisciplinary care, discussion of these cases at regional and
national meetings benefits children with similar cases. The researcher could have chosen to
obtain consent only for the whole-genome sequencing analysis and not discuss the case with or
report findings back to the family.

There is no apparent direct harm, inequity, or lack of dignity for the patient in the vignette.
Inequity would arise if the study was to exclude children of certain ethnic/cultural or
geographical backgrounds, which would not be relevant to this vignette. The patient is not being
harmed by being included in the research study.

PREP Pearls
• Research involving children should adhere to certain ethical principles.
• The institutional review board, a research oversight committee, has measures in place to
overcome the issue of limited autonomy and ensure minimal risk while respecting the
dignity of the children involved in a research study.

ABP Content Specifications(s)


• Recognize and apply ethical principles regarding research involving children

American Academy of Pediatrics 717


PREP ® Self-Assessment PREPSA 2021
Suggested Readings
• Graham A, Powell M, Taylor N, Anderson D, Fitzgerald R. Ethical Research Involving
Children. Florence, Italy: UNICEF Office of Research-Innocenti; 2013.
• Rose CD. Ethical conduct of research in children: pediatricians and their IRB (part 1 of
2). Pediatrics. 2017 May;139(5). pii:e20163648. doi:10.1542/peds.2016-3648.
• Rose CD. Ethical conduct of research in children: pediatricians and their IRB (part 2 of
2). Pediatrics. 2017;139(6). pii:e20163650. doi:10.1542/peds.2016-3650.

American Academy of Pediatrics 718


PREP ® Self-Assessment PREPSA 2021
Question 197
A 3-month-old infant is being evaluated for a hoarse cry of 2 weeks’ duration. His mother is
concerned about noisy breathing and progressive difficulty feeding. On physical examination, his
vital signs are normal and he has an oxygen saturation of 98% on pulse oximetry. He has mild
inspiratory stridor with increased work of breathing and intercostal retractions. His breath sounds
are clear. Cardiovascular examination findings are normal. No hepatosplenomegaly is noted.

Of the following, the finding that would be MOST helpful in making a diagnosis is
A. eczema in flexural creases and scalp
B. family history of asthma and allergies
C. hemangioma of the scalp and lower lip
D. history of prenatal tobacco smoke exposure

American Academy of Pediatrics 719


PREP ® Self-Assessment PREPSA 2021
Correct Answer: C
The infant in the vignette has symptoms of upper airway compromise, including inspiratory
stridor and hoarse cry. First consideration must be given to an anatomic lesion affecting the
upper airway. A physical examination finding of a hemangioma of the scalp and lip would
suggest the possibility of other hemangiomas, including one affecting the airway. Eczema,
family history of asthma and allergies, and prenatal tobacco smoke might all make this child
susceptible to asthma or airway reactivity, but his lung fields are clear and the presence of stridor
suggests an extrathoracic source of obstruction. In contrast, an expiratory wheeze would be
characteristic of small airway obstruction. Hemangiomas affecting the trachea and lower airway
are very uncommon, but must be considered when there is airway compromise in the context of
cutaneous hemangiomas. Onset of symptoms related to hemangiomas is most often seen at 8 to
12 weeks of age, when the growth of the vascular malformation is at its peak.

Other vascular malformations that might present with respiratory symptoms include branching
anomalies of the aorta and pulmonary arteries leading to vascular rings and slings. These often
present with wheezing or cough, and sometimes with feeding difficulty. The wheezing may be
monophonic, indicating a single point of obstruction, as opposed to the polyphonic or musical
wheezes associated with diffuse small airway obstruction. A right aortic arch with aberrant left
subclavian artery and left ductus venosus is the most common of these anomalies, comprising
30% to 65% of all central vascular anomalies. Less common are a double aortic arch (30%-45%)
or aberrant innominate artery (3%-20%) compressing the trachea. Aberrant subclavian arteries
and pulmonary artery slings make up less than 10% of central vascular anomalies. Central
vascular anomalies are often associated with other malformations, particularly cardiac, and with
genetic disorders such as Down syndrome or DiGeorge syndrome. Magnetic resonance imaging
has become the standard method for evaluation of these vascular anomalies, replacing barium
studies, computed tomography, and even cardiac catheterization.

PREP Pearls
• A head or facial hemangioma in the presence of respiratory symptoms suggests the
possibility of a hemangioma in the airway.
• Vascular rings and slings may present with wheezing or cough and dysphagia.
• Magnetic resonance imaging is the modality of choice for the evaluation of central
vascular anomalies.

ABP Content Specifications(s)


• Plan the appropriate clinical and laboratory evaluation of vascular anomalies that affect
the airway

American Academy of Pediatrics 720


PREP ® Self-Assessment PREPSA 2021
Suggested Readings
• Conrad C, Cornfield DN. Airway obstruction. In: McInerny TK, Adam HM, Campbell
DE, DeWitt TG, Foy JM, Kamat DM, eds. American Academy of Pediatrics Textbook of
Pediatric Care. 2nd ed. Itasca, IL: American Academy of Pediatrics; 2016;chap
348:2777-2785. Pediatric Care Online.
• Fogel MA, Pawlowski TW, Harris MA, et al. Comparison and usefulness of cardiac
magnetic resonance vs computed tomography in infants six months of age or younger
with aortic arch anomalies without deep sedation or anesthesia. Am J
Cardiol. 2011;108(1):120-125. doi: 10.1016/j.amjcard.2011.03.008.
• Krowchuk DP, Frieden IJ, Mancini AJ, et al; Subcommittee on the Management of
Infantile Hemangiomas. Clinical practice guideline for the management of infantile
hemangiomas. Pediatrics. 2019;143(1):e20183475. doi: 10.1542/peds.2018-3475.
• Licari A, MancaE, Rispoli GA, et al. Congenital vascular rings: a clinical challenge for
the pediatrician. Pediatr Pulmonol. 2015;50(5):511-524. doi: 10.1002/ppul.23152.

American Academy of Pediatrics 721


PREP ® Self-Assessment PREPSA 2021
Question 198
A 16-year-old adolescent girl is seen in the office for evaluation of bruising on her knees and
ankles. She has no significant bleeding history and has had regular monthly menses lasting 6
days each. She has not been taking any medications, vitamins, or supplements. She reports that
she plays on the school soccer team and is frequently kicked. Her physical examination findings
are remarkable only for nonpalpable bruises of various sizes and ages on her ankles and around
her knees. She is reassured that her bruising is likely related to her soccer participation. A
complete blood cell count reveals the following:

Laboratory Test Result


White blood cell count 4,500/µL (4.5 × 109/L)
Hemoglobin 9.7 g/dL (97 g/L)
Platelet count 128 × 103/µL (128 × 109/L)
Absolute neutrophil count 1,040/µL (1.0 × 109/L)
Mean corpuscular volume 110 fL
Reticulocytes 0.2%

On review of these results, the clinician orders the following additional laboratory tests:
Laboratory Test Result
Ferritin 96 ng/mL (216 pmol/L)
502 pg/mL (370 pmol/L)
Vitamin B12
(reference range, 200-900 pg/mL [148-664 pmol/L])
11 ng/mL (25 nmol/L)
Folate
(reference range, 2-20 ng/mL [5-45 nmol/L])

Of the following, the test MOST likely to confirm the diagnosis is


A. anti-intrinsic factor antibody
B. bone marrow biopsy
C. flow cytometry for lymphoblasts
D. serum polymerase chain reaction for parvovirus

American Academy of Pediatrics 722


PREP ® Self-Assessment PREPSA 2021

Correct Answer: B
The girl in the vignette has mild neutropenia (1,000-1,500 neutrophils/µL), uncompensated
anemia (reticulocytopenia), thrombocytopenia (100,000-150,000 platelets/µL), and
macrocytosis. The differential diagnosis for anemia with reticulocytopenia can be classified by
the presence or absence of other cytopenias. Anemia with reticulocytopenia associated with other
cytopenias (complex anemia) raises concern about bone marrow failure or leukemia (Item
C198A). The differential diagnosis for a macrocytic anemia in children is limited to nutritional
deficiencies (vitamin B12 and folate), bone marrow failure/myelodysplasia, certain drug
exposures (eg, chemotherapeutics), or hypothyroidism. The normal vitamin B12 and folate levels
exclude a nutritional deficiency, and there is no history of chemotherapy exposure or symptoms
of hypothyroidism. Thus, this presentation of macrocytosis adds to the concern regarding bone
marrow failure (eg, aplastic anemia) (Item C198B). The diagnostic test of choice to assess the
cellularity of the bone marrow is a bone marrow biopsy. The diagnostic criteria for severe
aplastic anemia include at least two severe cytopenias (absolute neutrophil count < 500/µL,
platelet count < 20,000/µL, or reticulocyte count < 60,000/µL) and bone marrow cellularity of
less than 25%. Any time a child exhibits multiple cytopenias, a bone marrow evaluation should
be considered.

American Academy of Pediatrics 723


PREP ® Self-Assessment PREPSA 2021

The normal vitamin B12 level excludes a deficiency of intrinsic factor (pernicious anemia).
Leukemia can present with pancytopenia, but the anemia is typically normocytic and there are no
leukemic blasts reported on the differential in the vignette. Thus, flow cytometry would not be
the diagnostic test of choice. Parvovirus can cause reticulocytopenia, but not typically
macrocytosis, and typically not pancytopenia, just an isolated anemia.

PREP Pearls
• The differential diagnosis for a macrocytic anemia in children is limited to nutritional
deficiencies (vitamin B12 and folate), bone marrow failure/myelodysplasia, certain drug
exposures (eg, chemotherapeutics), or hypothyroidism.
• Any time a child exhibits multiple cytopenias, a bone marrow evaluation should be
considered.

MOCA-Peds Objective
• Evaluate and manage a patient with the new onset of thrombocytopenia.

ABP Content Specifications(s)


• Recognize the causes of macrocytic anemia

American Academy of Pediatrics 724


PREP ® Self-Assessment PREPSA 2021
Suggested Readings
• McFarren AK, Levy AS. Anemia and pallor. In: McInerny TK, Adam HM, Campbell
DE, DeWitt TG, Foy JM, Kamat DM, eds. American Academy of Pediatrics Textbook of
Pediatric Care. 2nd ed. Elk Grove Village, IL: American Academy of Pediatrics;
2017:1199-1208. Pediatric Care Online .
• Noronha SA. Aplastic and hypoplastic anemias. Pediatr Rev. 2018;39(12):601-611.
doi:10.1542/pir.2017-0250.
• Sharma R, Nalepa G. Evaluation and management of chronic pancytopenia. Pediatr Rev.
2016;37(3):101-111. doi:10.1542/pir.2014-0087.

American Academy of Pediatrics 725


PREP ® Self-Assessment PREPSA 2021
Question 199
An 8-year-old boy is brought to the emergency department with acute-onset leg weakness. He
had been getting ready for dinner, when he called out to his mother because of difficulty
ambulating, which progressed over an hour to an inability to move his legs. He reports that for
the past week he had experienced some intermittent numbness and tingling in his legs which he
attributed to sitting in a cross-legged position. He has had constipation for the last week that his
mother attributed to a poor diet. He denies trauma, changes in vision, or prior neurologic deficits.

On physical examination, he appears alert and appropriately distressed about his inability to
move his legs. His cranial nerve examination findings are normal. Upper extremity strength,
reflexes, coordination, and sensation are normal. Lower extremity strength is 1/5 bilaterally with
0/4 deep tendon reflexes at the patella and ankles, with upgoing toes to plantar stimulation
bilaterally. He reports no sensation to pinprick, temperature, and vibration testing in the lower
extremities. He feels a change in perceived sensation around the midthoracic region. Rectal
examination reveals diminished tone. He is unable to ambulate.

Of the following, the BEST next step in this boy’s diagnostic workup is
A. computed tomography of the spine
B. electromyography
C. magnetic resonance imaging of the spine
D. plain radiography of the spine

American Academy of Pediatrics 726


PREP ® Self-Assessment PREPSA 2021
Correct Answer: C
The boy in the vignette is exhibiting signs and symptoms of acute myelopathy, which is a
neurologic emergency. Spine magnetic resonance imaging (MRI) with and without contrast is
urgently needed to evaluate for the presence of a compressive lesion potentially requiring acute
surgical intervention and to assess for intrinsic changes in the spinal cord, such as can be seen in
acute transverse myelitis (ATM).

Acute transverse myelitis is an immune-mediated inflammatory disease of the spinal cord that
can occur as an isolated postinfectious event or as a condition on the continuum of
neuroinflammatory disorders that include neuromyelitis optica (NMO), acute disseminated
encephalomyelitis, myelin-oligodendrocyte glycoprotein antibody-related disorders, and multiple
sclerosis (MS). Clinically, myelopathy symptoms evolve over hours to days and can include
bilateral leg and arm weakness, paresthesias or numbness, and bowel or bladder dysfunction.
Physical examination shows flaccid paralysis of the affected limbs with hyporeflexia and a
sensory level correlating with the location of spinal cord demyelination. The sensory level can
distinguish ATM from Guillain-Barré syndrome, which can present similarly in the acute setting.

In children with ATM, spine MRI shows inflammation involving both the gray and white matter;
this can be either patchy or continuous, commonly extending over 3 segments. Brain MRI should
also be performed. Brain lesions, if present, may suggest an increased risk for
neuroinflammatory diseases such as MS or NMO.

Treatment with high-dose corticosteroids should be initiated without delay in cases of suspected
ATM even in the absence of confirmatory imaging findings, because MRI can be normal early in
the disease course. Affected children should be monitored in an intensive care unit setting with
close monitoring of respiratory function, because symptoms can worsen over the first several
days. Supportive management, with attention to bowel and bladder care, early involvement of
therapy services, deep vein thrombosis prophylaxis, and pain control, is critical. The prognosis in
cases of ATM is variable; about 50% of children make a complete recovery within 2 years.

Although computed tomography and radiography of the spine are helpful in evaluating the
vertebral bodies, they would not sufficiently visualize the spinal cord. Electromyography may
show nonspecific changes within the first month of presentation in transverse myelitis, but is not
useful in making the initial diagnosis for a patient with myelopathy.

PREP Pearls
• Acute myelopathies are a neurologic emergency necessitating urgent evaluation, spine
neuroimaging, and prompt treatment initiation.
• Acute transverse myelitis is a diagnosis of exclusion that can be made as part of a
continuum of neuroinflammatory disorders or as an isolated condition, often following an
infection.
• Spine magnetic resonance imaging is the initial imaging of choice to evaluate a patient
suspected to have acute myelopathy. Treatment with high-dose corticosteroids should be
initiated without delay in cases of transverse myelitis, even in the absence of
confirmatory imaging findings because magnetic resonance imaging scans can be normal
early in the disease course.
American Academy of Pediatrics 727
PREP ® Self-Assessment PREPSA 2021
MOCA-Peds Objective
• Recognize the child with spinal cord compression from a mass.

ABP Content Specifications(s)


• Recognize the clinical manifestations of transverse myelitis
• Plan the appropriate diagnostic evaluation of suspected transverse myelitis

Suggested Readings
• Absoud M, Greenburg B, Lim M, Lotze T, Thomas T, Deiva K. Pediatric transverse
myelitis. Neurology. 2016;87(9 suppl 2):S46-S52.
doi: 10.1212/WNL.0000000000002820.
• Scott T, Frohman E, De Seze J, Gronseth G, Weinshenker B. Evidence-based guideline:
Clinical evaluation and treatment of transverse myelitis. Neurology. 2011;77(24):2128-
2134. doi: 10.1212/WNL.0b013e31823dc535.

American Academy of Pediatrics 728


PREP ® Self-Assessment PREPSA 2021
Question 200
A previously healthy, 4-year-old girl with a 2-day history of fever, malaise, and mouth sores is
seen in the emergency department in July. The parents report that some of the girl’s classmates
have had a similar illness in the past few weeks. She has a temperature of 38°C, a heart rate of
131 beats/min, a respiratory rate of 24 breaths/min, and an oxygen saturation of 92% on room
air. An examination of the throat reveals pharyngeal inflammation with lesions on the palate
(Item Q200). The remainder of the physical examination findings are normal.

Item Q200: Throat of the girl described in the vignette.


Reprinted with permission from Larry Frenkel and Kimberlin DW, et al, eds. Red Book Online.
Itasca, IL: American Academy of Pediatrics; 2019

Of the following, the BEST management for this child is


A. administration of acyclovir
B. administration of nystatin
C. administration of prednisone
D. reassurance and close observation

American Academy of Pediatrics 729


PREP ® Self-Assessment PREPSA 2021
Correct Answer: D
The child described in the vignette has a fever, pharyngitis with discrete ulcerative lesions on the
palate, and a history of sick contacts at school with a similar illness. These symptoms are highly
suggestive of herpangina caused by enterovirus.

The nonpolio enteroviruses and human parechoviruses are small non-enveloped ribonucleic acid
viruses in the Picornaviridae family. They are ubiquitous, with more than 130 identified types
(eg, coxsackie, echoviruses), and they commonly cause childhood illnesses. Transmission occurs
via fecal-oral contact. Enterovirus infections typically occur during the summer and early fall,
and infants and young children are at highest risk. The incubation period usually ranges from 3
to 6 days. The disease severity correlates to age and host immunity. Although enteroviruses
commonly cause a mild illness with complete recovery, severe illness and death may occur in
neonates, infants, and patients who are immunocompromised.

Most enterovirus infections in children are asymptomatic. The clinical spectrum of infections
caused by enteroviruses is varied and includes herpangina, hand-foot-and-mouth disease
(HFMD), nonspecific viral exanthems, gastrointestinal or respiratory infections, and aseptic
meningitis in children, especially in neonates and young infants. Severe neurologic syndromes
associated with enterovirus and human parechoviruses include aseptic meningitis, encephalitis,
and acute flaccid paralysis. Neonatal enteroviral disease can present with severe sepsis
associated with hepatitis, coagulopathy, and myocarditis indistinguishable from disseminated
herpes simplex virus disease and bacterial sepsis. Patients with antibody deficiency syndromes
can have persistent central nervous system infection caused by enterovirus infection. Other
clinical syndromes caused by enteroviruses include acute hemorrhagic conjunctivitis,
pleurodynia, hepatitis, and myopericarditis.

Herpangina and HFMD disease are related diseases most commonly caused by group A
coxsackie serotypes. The typical clinical presentation of herpangina is a distinctive enanthem
characterized by painful, discrete yellow or white ulcers on the soft palate, uvula, tonsils, and
posterior oropharyngeal area, often associated with abrupt onset of high fever, headache,
malaise, and sore throat. Poor feeding and dehydration may occur in young children. The oral
mucosal lesions are initially noted as punctate macules that evolve over one day to 3-mm to 4-
mm vesicles followed by ulcers with red rims. The duration of fever is 2 to 4 days, but the oral
lesions may persist up to 1 week. In contrast, HFMD is a distinctive clinical syndrome most
frequently caused by coxsackie A serotypes (A1-6, 8, 10, 16, and 22) and characterized by initial
development of oral vesicular lesions on the buccal mucosa and tongue (similar to those noted in
herpangina) followed by appearance of cutaneous lesions localized to the hands and feet (and
buttocks). Fever in HFMD is usually low-grade compared to herpangina. Hand-foot-and-mouth
disease is a self-limiting illness with resolution by 1 week. In addition to coxsackie A, HFMD
can also be caused by enterovirus 71, which may be associated with encephalitis and
cardiopulmonary failure.

The differential diagnosis of herpangina and HFMD include primary herpetic gingivostomatitis
(caused by herpes simplex virus type 1), aphthous stomatitis, and other conditions (eg, Kawasaki
disease, Stevens Johnson syndrome). Primary herpetic gingivostomatitis is characterized by
erythematous gingiva and clusters of small ulcerative lesions on the gingiva and oral mucous
American Academy of Pediatrics 730
PREP ® Self-Assessment PREPSA 2021
membranes. Perioral vesicular lesions may be noted. In some cases, the clinical course can be
complicated by severe pain and dehydration due to poor feeding, especially in young children.
Treatment is often supportive, but acyclovir may be considered in hospitalized children with
primary herpetic gingivostomatitis within 72 hours of symptom onset. The clinical presentation
of the child described in the vignette is not consistent with oral candidiasis (thrush) or aphthous
stomatitis, so nystatin would not be indicated. Prednisone and other immunosuppressive and
immunomodulating agents have been used for recurrent aphthous stomatitis, a severe ulcerative
disease of unknown etiology affecting the oral mucosa.

In immunocompetent children, treatment is supportive for common enterovirus illness such as


herpangina in the child described in the vignette. No antiviral agents have been approved for the
treatment of enterovirus infections. In immunocompromised patients with chronic enterovirus
meningoencephalitis, treatment with intravenous immune globulin is recommended. Contact
precautions are recommended for hospitalized infants and children with enterovirus infections
for the duration of illness. Vaccines against virulent strain enterovirus 71 are under development.

PREP Pearls
• Enterovirus infections typically occur during the summer and early fall, and infants and
young children are at highest risk; transmission occurs via fecal-oral contact.
• Herpangina and hand-foot-and-mouth disease are related diseases most commonly caused
by group A coxsackie serotypes.
• Herpangina is characterized by painful, discrete yellow or white ulcers on the soft palate,
uvula, tonsils, and posterior oropharyngeal area, often associated with abrupt onset of
high fever, headache, and sore throat.

ABP Content Specifications(s)


• Differentiate clinically among hand-foot-mouth disease, herpangina, acute herpetic
gingivostomatitis, aphthous ulcerations, and benign lesions of the oral cavity

Suggested Readings
• American Academy of Pediatrics. Enterovirus (nonpoliovirus) infections (group A and B
cocksackieviruses, echoviruses, numbered enteroviruses). In: Kimberlin DW, Brady MT,
Jackson MA, Long SS, eds. Red Book: 2018-2021 Report of the Committee on Infectious
Diseases. 31st ed. Itasca, IL: American Academy of Pediatrics; 2018:331-333. Red Book
Online.
• Cunningham D, Rongkavilit C, Udhayashankar K, Niescierenko M. Enterovirus and
evolving infections. In: McInerny TK, Adam HM, Campbell DE, DeWitt TG, Foy JM,
Kamat DM, eds. American Academy of Pediatrics Textbook of Pediatric Care. 2nd ed.
Elk Grove Village, IL: American Academy of Pediatrics; 2017:2001-2004. Pediatric Care
Online .
• Krol DM, Keels MA. Oral conditions. Pediatr Rev. 2007;28(1):15-22.
doi:10.1542/pir.28-1-15.
• Noor A, Krilov LR. Enterovirus infections. Pediatr Rev. 2016;37(12):505-515.
doi:10.1542/pir.2016-0103.

American Academy of Pediatrics 731


PREP ® Self-Assessment PREPSA 2021
Question 201
A 37-year-old primigravid woman with a history of obesity and chronic hypertension is having a
prenatal clinic visit. At 36 weeks’ gestation, she had an abnormal finding on a nonstress test and
was asked to return for a biophysical profile and Doppler studies. She asks about the purpose of
the additional tests.

Of the following, the BEST response is that these tests are meant to
A. assess fetal well-being
B. diagnose renal dysfunction
C. evaluate growth
D. measure cardiac function

American Academy of Pediatrics 732


PREP ® Self-Assessment PREPSA 2021
Correct Answer: A
Obstetricians use a number of tools to assess fetal well-being. Estimated fetal weight consistent
with gestational age, absence of anomalies on ultrasonography, and adequate amniotic fluid
volume suggest stable fetal growth and development. Obstetricians may also use a nonstress test
(NST). A normal NST includes 2 episodes of heart rate greater than 150 beats/min during a 20-
minute test, a sign of normal activity and heart rate variation. The NST has a low rate of false-
negative results and high rate of false-positive results, which helps ensure that most
abnormalities will be identified. An abnormal NST finding should be followed by a biophysical
profile (BPP). A BPP measures 5 items: heart rate, fetal breathing, fetal movement, fetal tone,
and amniotic fluid volume. A BPP score of less than 4 needs immediate evaluation. An
intermediate score of 6 is concerning and should be repeated in 24 hours. A BPP of 8 to 10 is
reassuring.

The most common reason for fetal distress is uteroplacental insufficiency, a mismatch between
nutrients delivered by the placental and fetal requirements. Chronic hypertension is one factor
associated with uteroplacental insufficiency.

Fetal growth can be assessed over time by measuring estimated fetal weight. Fetal
echocardiography is the best test to assess cardiac function. Fetal renal dysfunction may be
estimated by monitoring amniotic fluid levels and evaluating for the presence of hydronephrosis.

PREP Pearls
• A nonstress test measures fetal heart accelerations during a 20-minute period to assess
fetal well-being.
• An abnormal nonstress test result should be followed by a biophysical profile. Both the
nonstress test and biophysical profile have low false-negative rates.
• The most common cause of fetal distress is uteroplacental insufficiency.

ABP Content Specifications(s)


• Understand the appropriate use of stress and non-stress tests during fetal assessment
• Know the factors used by obstetricians to evaluate fetal well-being

Suggested Readings
• Oelberg DG. Consultation with the specialist: prenatal growth: the sum of maternal,
placental, and fetal contributions. Pediatr Rev. 2006;27:224-229. doi: 10.1542/pir.27-6-
224.
• Pschirrer ER, Little GA. Perinatal preventive care: fetal assessment. In: McInerny TK,
Adam HM, Campbell DE, DeWitt TG, Foy JM, Kamat DM, eds. American Academy of
Pediatrics Textbook of Pediatric Care. 2nd ed. Itasca, IL: American Academy of
Pediatrics; 2016;chap 80:667-692. Pediatric Care Online.
• Suhrie KR, Tabbah SM. The fetus. In: Kliegman RM, St Geme JW, Blum NJ, Shah SS,
Tasker RC, Wilson KM, eds. Nelson Textbook of Pediatrics. 21st ed. Philadelphia, PA:
Elsevier; 2019:880-892.

American Academy of Pediatrics 733


PREP ® Self-Assessment PREPSA 2021
Question 202
A 7-year-old boy is brought to the office by his parents who are worried about his learning. His
teacher is also concerned, and the boy was recently evaluated by his school. His parents are
anxious about whether he has a learning disability as it “runs in the family.” They have brought a
copy of his psychoeducational evaluation and would like to understand the results.

His aptitude test scores are:


• Verbal IQ = 68
• Performance IQ = 65
His academic achievement scores are:
• Reading = 62
• Math = 66
• Writing = 60
His adaptive test scores are:
• Conceptual = 60
• Social = 68
• Practical = 64

Of the following, the MOST likely diagnosis is


A. a learning disability in math
B. a learning disability in reading and writing
C. a learning disability in reading, writing, and math
D. mild intellectual disability

American Academy of Pediatrics 734


PREP ® Self-Assessment PREPSA 2021
Correct Answer: D
Results from standardized aptitude (intelligence), achievement, and adaptive tests are typically
reported as standard scores with a mean of 100 and standard deviation (SD) of 15. The normal
range is considered to be within 1 SD (85-115) of the mean. Intellectual disability is present
when both intelligence and adaptive functioning are more than 2 SD below normal (scores <70).
Intellectual disability can be mild (scores 2-3 SD below the mean), moderate (scores 3-4 SD
below the mean), severe (scores 4-5 SD below the mean), or profound (scores >5 SD below the
mean). This boy’s scores are all 2 to 3 SD below normal (55-70), consistent with a diagnosis of
mild intellectual disability.

Standardized IQ tests are used to measure cognition (aptitude). Commonly used IQ tests include
the Wechsler intelligence scales (with specific scales for preschoolers, children, and adults),
Kaufman Assessment Battery for Children, and Differential Ability Scales. Both verbal and
nonverbal abilities are assessed. IQ tests can include measures of working memory, processing
speed, knowledge, problem-solving, and visuospatial skills.

Standardized achievement tests measure performance in reading, math, written language, and
oral language compared with same-age peers. Commonly used achievement tests include the
Kaufman Test of Educational Achievement, Wechsler Individual Achievement Test, Wide
Range Achievement Test, and Woodcock-Johnson Tests of Achievement.

Standardized tests of intelligence and achievement are typically included in psychoeducational


evaluations because these are helpful in understanding a child’s learning profile and determining
eligibility for special education services. For the typical child, scores are usually within 1 SD
(±15) of the mean (100) for each subtest and between subtests. Scores are also usually within 1
SD between measures of intelligence and the corresponding areas of achievement (ie, verbal IQ
and reading/writing achievement; nonverbal IQ and math achievement). School districts may
define learning disability as 1) a discrepancy of at least 1 to 2 SD between IQ scores and
achievement scores, 2) low achievement in a student with at least low average intelligence, or 3)
a student’s failure to respond to evidence-based educational interventions.

The boy in the vignette has no significant discrepancies between his IQ scores and achievement
scores in any domain, nor does he have at least low average intelligence, therefore, he does not
meet criteria for a learning disability. He does, however, meet criteria for mild intellectual
disability and would qualify for an Individualized Education Program to support his learning.

PREP Pearls
• Results from standardized aptitude (intelligence), achievement, and adaptive tests are
typically reported as standard scores with a mean of 100 and standard deviation of 15.
The normal range is considered to be within 1 standard deviation (85-115) of the mean.
• Intellectual disability is present when both intelligence and adaptive functioning are more
than 2 standard deviations below normal (scores <70).
• School districts may define a learning disability by 1) a discrepancy of at least 1 to 2
standard deviations between IQ scores and achievement scores, 2) low achievement in a
student with at least low average intelligence, or 3) a student’s failure to respond to
evidence-based educational interventions.
American Academy of Pediatrics 735
PREP ® Self-Assessment PREPSA 2021

ABP Content Specifications(s)


• Plan the appropriate diagnostic evaluation of achievement and intelligence
• Interpret the results of intelligence quotient tests, with emphasis on understanding the
normal ranges

Suggested Readings
• Braaten EB, Norman D. Intelligence (IQ) testing. Pediatr Rev. 2006;27(11):403-408.
doi: 10.1542/pir.27-11-403.
• Kral MC. Interpreting Psychoeducational Testing Reports, Individualized Family Service
Plans (IFSP), and Individualized Education Program (IEP) plans. In: Augustyn M,
Zuckerman B, eds. Zuckerman Parker Handbook of Developmental and Behavioral
Pediatrics for Primary Care. 4th ed. Philadelphia, PA: Wolters Kluwer; 2019:477-493.
• Krueger C, Loe IM. Neuropsychological testing: what clinicians need to know. In:
Augustyn M, Zuckerman B, eds. Zuckerman Parker Handbook of Developmental and
Behavioral Pediatrics for Primary Care. 4th ed. Philadelphia, PA: Wolters Kluwer;
2019:92-95.
• Phelps RA, Cohen WI. Intellectual disability. In: McInerny TK, Adam HM, Campbell
DE, DeWitt TG, Foy JM, Kamat DM, eds. American Academy of Pediatrics Textbook of
Pediatric Care. 2nd ed. Itasca, IL: American Academy of Pediatrics; 2016;chap
278:2208-2216. Pediatric Care Online.

American Academy of Pediatrics 736


PREP ® Self-Assessment PREPSA 2021
Question 203
A 7-year-old boy with cola-colored urine and decreased urine output of 1 day’s duration is
brought to the emergency department. He has no history of fever, rash, diarrhea, vomiting,
frequency, or burning with urination. He had a sore throat 2 weeks ago. He has a heart rate of
108 beats/min, a respiratory rate is 20 breaths/min, and a blood pressure of 130/90 mm Hg. He
has swelling over his face and mild pitting edema of both lower extremities. His chest is clear to
auscultation, and the rest of the examination findings are unremarkable.

Laboratory data are shown:


Laboratory Test Result
Blood
Sodium 132 mEq/L (132 mmol/L)
Potassium 5.0 mEq/L (5.0 mmol/L)
Chloride 100 mEq/L (100 mmol/L)
Bicarbonate 22 mEq/L (22 mmol/L)
Blood urea nitrogen 28 mg/dL (10.0 mmol/L)
Creatinine 1.8 mg/dL (159 µmol/L)
Albumin 2.5 g/dL (25 g/L)
16 mg/dL (0.16 g/L)
Complement C3
(reference range, 85-142 mg/dL [0.85-1.42 g/L])
22 mg/dL (0.22 g/L)
Complement C4
(reference range, 12-41 mg/dL [0.12-0.41 g/L])
Antistreptolysin O 800 IU
Urine
Blood 3+
Protein 3+
Red blood cells 50-100/HPF
White blood cells 5-10/HPF

Of the following, the MOST appropriate treatment for this child is


A. intravenous furosemide
B. intravenous normal saline
C. oral enalapril
D. oral prednisone

American Academy of Pediatrics 737


PREP ® Self-Assessment PREPSA 2021
Correct Answer: A
The boy in this vignette—with a history of prior sore throat, cola-colored urine, hematuria,
proteinuria, hypertension, azotemia, elevated level of antistreptolysin O, and a low complement 3
level—has acute poststreptococcal glomerulonephritis (PSGN). The presence of edema and
hypertension reveal that he has fluid retention. The most appropriate treatment is to administer
intravenous furosemide.

Poststreptococcal glomerulonephritis is the most common form of acute glomerulonephritis


(AGN) in children. Children with PSGN characteristically have a prior history of sore throat of 1
to 2 weeks’ duration or a skin infection of 2 to 6 weeks’ duration before exhibiting renal
involvement. The presentation of PSGN can range from presence of subclinical features of
microscopic hematuria, mild hypertension, and no edema to a full presentation of AGN,
including gross hematuria, significant hypertension, and acute kidney injury. Hypertension can
be acute, owing to salt and fluid retention; it sometimes can be severe, resulting in cerebral
symptoms that include headache, visual disturbances, and, rarely, hypertensive encephalopathy.
Fluid retention also leads to generalized edema, and in severe cases it causes pulmonary edema
and congestive heart failure.

The laboratory tests supporting a preceding streptococcal infection are an elevated titer of
antistreptolysin O and other streptococcal antibodies (anti-deoxyribonuclease B,
antihyaluronidase). Serum complement 3 (C3) is the most important test to differentiate PSGN
from other forms of AGN, as shown in Item C203. In PSGN, the serum complement C3 value is
low and the serum complement C4 value is usually normal. The other relevant tests for the
diagnosis of AGN include electrolytes, renal function, and complete blood count. Children with
PSGN may have hyponatremia, hyperkalemia, metabolic acidosis, and elevated levels of blood
urea nitrogen and serum creatinine, depending on the degree of acute kidney injury. Urinalysis
with microscopy will show hematuria, proteinuria, and presence of red blood cell casts. Renal
biopsy is indicated in cases with rapidly progressive AGN or with atypical PSGN presentation.

The management of PSGN consists of providing supportive care, treating the sequelae of the
disease (edema, hypertension, hyperkalemia, acute kidney injury), and preventing complications
(encephalopathy, pulmonary edema, congestive heart failure). The initial management should
include restriction of fluid and sodium intake. Hypertension and edema, resulting from sodium
and fluid retention, are best treated using a potent diuretic such as furosemide, as for the boy in
the vignette. If hypertension is persistent, calcium channel blockers are the preferred therapeutic
agent. Severe hypertension may require intravenous infusion of nicardipine, sodium
nitroprusside, or labetalol. Hyperkalemia and acute kidney injury are managed symptomatically,
and they rarely require dialysis.

American Academy of Pediatrics 738


PREP ® Self-Assessment PREPSA 2021

American Academy of Pediatrics 739


PREP ® Self-Assessment PREPSA 2021
The child with PSGN in the vignette has oliguria, which results from decreased glomerular
filtration rate and fluid retention. He has no features of dehydration, and so the oliguria will not
improve with intravenous normal saline. On the contrary, excessive intravenous normal saline
will worsen fluid overload and hypertension. There is no role for prednisone in the management
of PSGN, and it can worsen hypertension. Enalapril, an angiotensin-converting enzyme inhibitor,
can control blood pressure; however, it is not recommended in this child owing to its potential to
worsen renal function and hyperkalemia.

Poststreptococcal glomerulonephritis is a self-limiting condition with a good prognosis. Fluid


overload usually resolves in 10 days; gross hematuria in 1 to 3 weeks; and hypertension, renal
dysfunction, and proteinuria in 3 to 4 weeks. Serum complement levels return to normal values
in 8 to 12 weeks. Microscopic hematuria can take more than 1 year to resolve completely.
Rarely, children with severe renal involvement may have residual renal damage or end-stage
renal disease.

PREP Pearls
• Poststreptococcal glomerulonephritis can present with subclinical features or clinically
apparent acute glomerulonephritis.
• Poststreptococcal glomerulonephritis is characterized by a low serum complement C3
value and a normal serum complement C4 value.
• Hypertension and edema in children with poststreptococcal glomerulonephritis, resulting
from sodium and fluid retention, are best treated with a potent diuretic such as
furosemide.

ABP Content Specifications(s)
• Differentiate acute post-streptococcal glomerulonephritis from other forms of
glomerulonephritis
• Recognize complications associated with post-streptococcal glomerulonephritis

Suggested Readings
• VanDeVoorde RG III. Acute poststreptococcal glomerulonephritis: the most common
acute glomerulonephritis. Pediatr Rev. 2015;36(1):3-12. doi:10.1542/pir.36-1-3.
• Varade WS. Nephritis. In: McInerny TK, Adam HM, Campbell DE, DeWitt TG, Foy JM,
Kamat DM, eds. American Academy of Pediatrics Textbook of Pediatric Care. 2nd ed.
Elk Grove Village, IL: American Academy of Pediatrics; 2017:2358-2367. Pediatric Care
Online .

American Academy of Pediatrics 740


PREP ® Self-Assessment PREPSA 2021
Question 204
A 15-year-old adolescent girl with respiratory distress is brought to the emergency department.
Her parents state that she started to have signs of a mild respiratory tract infection 2 weeks ago,
but her condition suddenly worsened today. She appears mottled and in moderate respiratory
distress. She has a temperature of 37.5°C, a heart rate of 155 beats/min, a respiratory rate of 20
breaths/min, a blood pressure of 105/65 mm Hg, and an oxygen saturation of 90% on room air.
She is awake but somnolent. Her lung examination reveals diffuse rales and rhonchi but no
wheezes. Auscultation of the heart reveals a quiet precordium with a grade 3/6 systolic murmur
and a grade 2/6 diastolic murmur. Her abdomen is soft, and the liver edge is palpable 5 to 6 cm
below the right costal margin. Her skin is mottled and cool with barely palpable distal pulses.
Capillary refill time is 5 sec. There is 2+ edema in both legs up to the knee. The remainder of the
examination is unrevealing. Oxygen is started, vascular access is established, and blood is drawn
for laboratory evaluation.

Of the following, the MOST appropriate medication infusion to start is


A. milrinone
B. norepinephrine
C. prostaglandin
D. vasopressin

American Academy of Pediatrics 741


PREP ® Self-Assessment PREPSA 2021
Correct Answer: A
The patient in the vignette has signs of acute heart failure and cardiogenic shock. On the basis of
the viral prodrome described, the most likely diagnosis is acute infectious or postinfectious
myocarditis and heart failure. Given the severity of the girl’s symptoms, medical cardiac support
is immediately warranted, and the best medication listed is milrinone.

Milrinone is a phosphodiesterase 3 inhibitor that increases cardiac contractility and decreases


pulmonary and systemic vascular resistance, thus improving overall cardiac output.
Phosphodiesterase 3 inhibitors prevent the breakdown of cyclic adenosine monophosphate,
which ultimately results in increases in calcium availability to the myocyte sarcomere. The
increased calcium availability results in increased cardiac inotropy, and the increased calcium
reuptake into the sarcoplasmic reticulum also leads to enhanced myocardial relaxation and
improved diastolic function. Milrinone increases systolic contractility, improves diastolic
relaxation (lusitropy) and also increases both pulmonary and peripheral vasodilation. These
effects, in total, result in increased cardiac output via improvement of ventricular performance
and a reduction in afterload. Patients with cardiogenic shock and hypotension may not tolerate
milrinone owing to the reduction in systemic vascular resistance (SVR), which would worsen
blood pressure and in those cases, epinephrine would be the medication of choice.

Norepinephrine is an intrinsic catecholamine derived from tyrosine. Norepinephrine acts as an ɑ


and a β agonist with major receptor affinity to the ɑ1 and β1 receptors. At low doses, the
β1 effects may be more pronounced and potentially increase cardiac output. However, in higher
doses, the ɑ1 effect predominates, resulting in peripheral vasoconstriction and a dose-dependent
increase in SVR and cardiac afterload. At physiologic doses, the increase in SVR tends to
predominate; this makes this medication the drug of choice in clinical situations in which
increase in SVR is primarily desired (such as septic shock). However, this medication would not
be indicated in patients experiencing cardiogenic shock, as in the patient in the vignette, because
the increase in afterload would be detrimental and may even precipitate cardiac arrest.

Vasopressin (arginine vasopressin) is a direct vasoconstrictor without inotropic and chronotropic


properties and acts on the V1, V2, and V3 receptors. The response is dose dependent and has been
shown to improve blood pressure in patients with septic shock. Vasopressin can potentially
decrease stroke volume and may result in decreased cardiac output secondary to an increased
SVR and elevated afterload. Although shown to be beneficial in septic shock, this medication
would not be indicated in cardiogenic shock and, because of the increase in afterload, may also
precipitate cardiac arrest.

Prostaglandin is used primarily to maintain a patent ductus arteriosus in neonates with ductal-
dependent congenital cardiac disease and has no role in the treatment of the patient in the
vignette.

American Academy of Pediatrics 742


PREP ® Self-Assessment PREPSA 2021
PREP Pearls
• Cardiogenic shock presents with poor systemic perfusion, hepatomegaly, and pulmonary
congestion.
• Unopposed vasopressors that increase systemic vascular resistance without improving
cardiac performance are contraindicated in cardiogenic shock.
• Medical therapy involving inotropes may be required to reverse negative consequences of
cardiogenic shock and prevent deterioration and arrest.

MOCA-Peds Objective
• Recognize and plan the initial management of cardiogenic shock.

ABP Content Specifications(s)


• Plan an appropriate diagnostic evaluation of cardiogenic shock
• Plan appropriate management of cardiogenic shock in children of various ages
• Recognize findings associated with cardiogenic shock in children of various ages

Suggested Readings
• Cornell T, Arutyunyan T, Custer JR. Shock. In: McInerny TK, Adam HM, Campbell DE,
DeWitt TG, Foy JM, Kamat DM, eds. American Academy of Pediatrics Textbook of
Pediatric Care. 2nd ed. Elk Grove Village, IL: American Academy of Pediatrics;
2017:2976-2983. Pediatric Care Online .
• McKiernan CA, Lieberman SA. Circulatory shock in children: an overview. Pediatr
Rev. 2005;26(12):451-460. doi:10.1542/pir.26-12-451.
• Yager P, Noviski N. Shock. Pediatr Rev. 2010;31(8):311-318; doi:10.1542/pir.31-8-311.

American Academy of Pediatrics 743


PREP ® Self-Assessment PREPSA 2021
Question 205
A healthy, 5-year-old girl is brought to the office to discuss concerns of unusual behavior. Her
parents report that over the past several months, their daughter has been masturbating, touching
her mother’s breasts, and asking her father if he can kiss her “vagi-gi.” Her parents report that
her teacher has observed her trying to engage in sexual activity with male classmates. They note
that the behaviors seemed to start after a family friend visited them during a holiday weekend.
Her temperament has otherwise been normal. Their marriage has been healthy and they have
tried to raise their daughter in an emotionally safe and supportive environment. They have never
exposed her to age-inappropriate media. Her vital signs, growth parameters, and physical
examination findings are normal.

Of the following, her behavior may be BEST attributed to


A. age-appropriate behavior
B. gender dysphoria
C. posttraumatic stress disorder
D. sexual abuse

American Academy of Pediatrics 744


PREP ® Self-Assessment PREPSA 2021
Correct Answer: D
The behavior of the child in the vignette is best attributed to sexual abuse, primarily because of
the emergence of hypersexualized behavior after the visit from the family friend. In addition,
although masturbation can be normal at this age, asking to engage in sexual acts is not age-
appropriate. Her behavior does not reflect gender dysphoria, self-identifying as the opposite
gender. Posttraumatic stress disorder also inadequately explains her behavior because she does
not exhibit negative emotions or distress.

After experiencing sexual abuse, children often do not verbally disclose the encounter for many
reasons including guilt, coercion, or a lack of understanding of what is considered consensual or
inappropriate contact. Rather, they may exhibit hypersexualized behavior or seemingly unrelated
behavior changes such as sleep disturbance or worsening academic performance. Sexual abuse
should, therefore, be considered in the differential diagnosis when families express concerns for
these behavior changes. Children may only disclose a small portion of their history at first, to
determine the reaction of the interviewer. They often disclose their history piecemeal over a long
period, sometimes years, often only after the perpetrator is no longer in contact with them.

When eliciting a history of sexual abuse, providers should use nonjudgmental, culturally
sensitive, open-ended, developmentally appropriate questions, especially with young children
who have more concrete thinking. Providers should also use the families’ preferred verbiage for
genitalia instead of medicalized terms. The full history should be documented using direct quotes
from both the provider and patient; this is important from a legal standpoint. Repeated
interviewing should be avoided. Because most primary care pediatricians do not have training in
forensic interviewing, families should be referred to child abuse specialists. Patients who do not
disclose should not be interviewed further. Interviewing the parent and child separately may be
appropriate in some cases.

Physical examination should be preceded by a discussion about how and by whom the anogenital
areas may be touched. Such a discussion can serve not only as a model to the family for healthy
communication, but also to begin or continue disclosure about inappropriate sexual contact.
Chaperones should always be present. Item C205A lists criteria for emergent evaluation.
Providers may use confidential photography to document findings; they should be sensitive to
patients who were photographed as part of their abuse. If there is no injury or need for forensic
evidence collection, the examination should remain noninvasive. The best physical examination
positions for suspected sexual abuse include the frog leg, prone knee-chest, and lateral fetal
positions, but providers should be sensitive to any particular positions used during the abuse.
Physical examination findings are usually normal; families should be counseled that normal
findings should not raise concerns about the validity of the history given by the child. Similarly,
any concerns regarding the determination of female virginity based on physical examination
findings should be addressed sensitively. If indicated, invasive examinations of prepubertal
female children should be performed only by a specialist and under general anesthesia; any
contact with the hymen before it is estrogenized can be painful. Item C205B lists physical
examination findings that are more indicative of abuse or trauma, may be confused for abuse, or
are normal variants.

American Academy of Pediatrics 745


PREP ® Self-Assessment PREPSA 2021

American Academy of Pediatrics 746


PREP ® Self-Assessment PREPSA 2021

American Academy of Pediatrics 747


PREP ® Self-Assessment PREPSA 2021
In many cases of sexual abuse, forensic evidence cannot be collected because of the interval or
mode of abuse. If indicated, specialized “rape kits” with intructions on their use are available
from law enforcement. If needed, Sexual Assault Nurse Examiner programs may be called upon
because these examiners have extensive training in appropriate evidence collection. In cases
involving prepubertal children, the clothing or surrounding objects may yield more evidence than
body fluids from the child.

Laboratory evaluation may include testing for sexually transmitted diseases (STDs), pregnancy,
and substance use. This should be performed on a case-by-case basis taking into account timing,
method of abuse, signs and symptoms, and patient preference. In prepubertal children, testing of
all sites for all STDs is not routinely recommended because of the low yield. The suggested
reading includes the best methods of testing, intervals for follow-up testing, and
recommendations for medical treatment of any STDs.

PREP Pearls
• Sexual abuse should be considered for any age-inappropriate sexual behavior as well as
seemingly unrelated behavioral changes such as sleep disturbance or worsening academic
performance.
• Physical examination findings of children who are victims of sexual abuse are often
normal; some normal findings, or those caused by other conditions, may be confused for
sexual abuse.

MOCA-Peds Objective
• Respond appropriately to a child’s disclosure of sexual abuse.

ABP Content Specifications(s)


• Recognize that most children examined for sexual abuse will have normal examination
findings
• Understand the normal process for a child to disclose sexual abuse and the barriers to
such disclosure
• Recognize which patients require emergent evaluation and physical examination for
sexual abuse or assault
• Recognize the history, signs, and symptoms of sexual abuse
• Plan appropriate collection of forensic evidence and laboratory evaluation during a sexual
abuse investigation
• Understand and utilize effective strategies and approaches to interviewing suspected
victims of sexual abuse or assault
• Recognize abnormal anogenital findings and injuries associated with sexual abuse and
differentiate them from those associated with accidental trauma and other conditions
confused with trauma

American Academy of Pediatrics 748


PREP ® Self-Assessment PREPSA 2021
Suggested Readings
• Chiesa A, Goldson E. Child sexual abuse. Pediatr Rev. 2017;38(3):105-118.
doi: 10.1542/pir.2016-0113.
• Crawford-Jakubiak JE, Alderman EM, Leventhal JM; Committee on Child Abuse and
Neglect, Committee on Adolescence. Care of the adolescent after an acute sexual
assault. Pediatrics. 2017;139(3):e20164243. doi: 10.1542/peds.2016-4243.
• Jenny C, Crawford-Jakubiak JE; Committee on Child Abuse and Neglect. The evaluation
of children in the primary care setting when sexual abuse is suspected. Pediatrics.
2013;132(2): e558-e567. doi: 10.1542/peds.2013-1741.
• Leventhal JM, Asnes AG. Sexual abuse of children. In: McInerny TK, Adam HM,
Campbell DE, DeWitt TG, Foy JM, Kamat DM, eds. American Academy of Pediatrics
Textbook of Pediatric Care. 2nd ed. Itasca, IL: American Academy of Pediatrics;
2016;chap 329:2620-2627. Pediatric Care Online.

American Academy of Pediatrics 749


PREP ® Self-Assessment PREPSA 2021
Question 206
The parents of a 9-month-old infant seek advice about a “spot” that appeared on their daughter’s
upper back several weeks ago. They have observed that when the lesion is rubbed (as happens
when they are drying the skin after bathing her), it may become red and somewhat swollen. The
infant is well in all other respects. The physical examination findings are notable only for a tan-
pink, 1.5-cm, round, thin plaque on the upper back (Item Q206A). After the lesion is rubbed, it
becomes erythematous and more elevated (Item Q206B).

Item Q206A: Lesion as described for the infant in the vignette. Note the lesion’s textural change.
Courtesy of D Krowchuk

American Academy of Pediatrics 750


PREP ® Self-Assessment PREPSA 2021

Item Q206B: Appearance of the lesion after it was rubbed. Courtesy of D Krowchuk

Of the following, the MOST likely diagnosis is a


A. café-au-lait macule
B. connective tissue nevus
C. mastocytoma
D. melanocytic nevus

American Academy of Pediatrics 751


PREP ® Self-Assessment PREPSA 2021
Correct Answer: C
The girl in the vignette has developed a lesion that has an orange-peel (peau d’orange)
appearance, a finding that indicates the presence of a dermal cellular infiltrate. After the lesion is
rubbed or stroked, there is a temporary increase in erythema, often accompanied by swelling (ie,
Darier sign). These changes are the result of mast cell degranulation and the release of mediators
that cause increased blood flow (producing erythema) and vascular fluid leak (producing edema).
The appearance of the lesion and the presence of Darier sign indicate a diagnosis of
mastocytosis, a group of disorders characterized by the accumulation of mast cells in the skin
and, occasionally, other organs. A café-au-lait macule, connective tissue nevus, and melanocytic
nevus would all lack an orange-peel appearance and would not exhibit the Darier sign.

Mastocytosis may be present at birth or develop any time into middle age. Approximately one-
half of patients have the onset in the first 2 years after birth, and an additional 10% develop the
disorder between 2 and 15 years of age. In infants and children, mastocytosis typically is limited
to the skin, is not associated with hematologic disorders (eg, mast cell leukemia), and tends to
resolve spontaneously by adolescence. Cutaneous mastocytosis is a spectrum of disease that
includes mastocytomas, urticaria pigmentosa, diffuse cutaneous mastocytosis, and telangiectasia
macularis eruptiva perstans. Mastocytomas and urticaria pigmentosa are the most common forms
encountered in pediatrics. Both are characterized by lesions that have an orange-peel appearance
and exhibit a Darier sign.

• Mastocytomas (as exhibited by the girl in the vignette) may be solitary or few in number.
They appear as yellow to orange-brown papules or plaques (Item C206A). Lesions range
in size from a few millimeters to several centimeters and may be found anywhere on the
body. Often there is a history of pruritus or recurrent blistering, the result of mast cell
mediator release. Blistering may lead to confusion with disorders such as recurrent herpes
simplex virus infection or bullous impetigo.
• Urticaria pigmentosa is the most common form of cutaneous mastocytosis. It is
characterized by multiple tan to brown macules or thin plaques ranging in size from a few
millimeters to several centimeters (Item C206B). As is the case with mastocytomas, the
lesions of urticaria pigmentosa may cause pruritus, and blistering may occur. If there is
sufficient mediator release, patients may experience flushing, hypotension, abdominal
discomfort, diarrhea, or respiratory distress.

American Academy of Pediatrics 752


PREP ® Self-Assessment PREPSA 2021

Item C206A: A solitary mastocytoma: an orange-brown plaque with an orange-peel


appearance. Courtesy of D Krowchuk

American Academy of Pediatrics 753


PREP ® Self-Assessment PREPSA 2021

Item C206B: Urticaria pigmentosa is characterized by hyperpigmented macules that have an


orange-peel appearance. This patient also manifests dermatographism (arrows), a linear wheal,
and flare at the site of skin stroking.
Courtesy of D Krowchuk

The diagnosis of mastocytosis usually is made clinically. If uncertainty exists, skin biopsy will
demonstrate an accumulation of mast cells in the dermis that may be confirmed by
American Academy of Pediatrics 754
PREP ® Self-Assessment PREPSA 2021
immunohistochemical staining. For the majority of pediatric patients, no further testing is
required. Some advocate measurement of serum tryptase concentration, a mast cell–derived
protease that correlates with disease extent. This may be most useful if systemic involvement is
suspected. If urticaria pigmentosa is very widespread, screening with a complete blood count and
blood chemistries often is recommended.

Most patients who have mastocytosis are asymptomatic and require no intervention. For those
experiencing pruritus, a second-generation (eg, loratadine, cetirizine) or third-generation
(desloratadine, fexofenadine, levocetirizine) H1 antihistamine may be prescribed. Blistering may
be prevented by the application of a potent topical corticosteroid. Patients who have flushing or
diarrhea may benefit from cromolyn or an H2 antagonist (eg, cimetidine, ranitidine). A
premeasured epinephrine pen is indicated for those who experience hypotensive episodes.
Parents should be advised that aspirin, nonsteroidal anti-inflammatory agents, and certain
anesthetic agents may cause mast cell degranulation. Information and support are available
at www.mastokids.org.

PREP Pearls
• Cutaneous mastocytosis is characterized by macules or papules that have an orange-
brown color. Stroking a lesion causes erythema and swelling (ie, Darier sign).
• Most patients who have cutaneous mastocytosis require no treatment, and the lesions
resolve spontaneously.

MOCA-Peds Objective
• Evaluate a child with hyperpigmentation.

ABP Content Specifications(s)


• Recognize the clinical features of the various forms of mastocytosis and manage
appropriately

Suggested Readings
• Carter MC, Metcalfe DD, Komarow HD. Mastocytosis. Immunol Allergy Clin North Am.
2014;34(1):81-96. doi:10.1016/j.iac.2013.09.001.
• Mancini AJ, Krowchuk DP, eds. Pediatric Dermatology: A Quick Reference Guide. 3rd
ed. Elk Grove Village, IL; American Academy of Pediatrics: 2016.

American Academy of Pediatrics 755


PREP ® Self-Assessment PREPSA 2021
Question 207
An 8-week-old male infant with tachypnea and poor oral intake is brought to the emergency
department. He was born at full term and has no relevant medical history. His parents report that
he has been eating less and breathing fast, seems tired, and becomes sweaty with feedings. He
has a heart rate of 170 to 180 beats/min, a blood pressure of 75/45 mm Hg, a respiratory rate of
60 breaths/min with mild subcostal retractions, and oxygen saturation of 95% by pulse oximetry.
He is irritable but consolable. Diffuse crackles are heard on auscultation. He has a gallop rhythm
and a grade 2/6 blowing systolic murmur at the apex. His abdomen is soft and nontender. His
liver edge is palpable 3 cm below the right costal margin. He is cool distally and warm centrally.
A chest radiograph (Item Q207A) and an electrocardiogram (Item Q207B) are obtained.

Item Q207A: Chest radiograph for the infant described in the vignette.
Reprinted with permission from Harper B. Index of suspicion. Pediatr Rev. 2017;38(7):338.

American Academy of Pediatrics 756


PREP ® Self-Assessment PREPSA 2021

Item Q207B: Electrocardiogram for the infant described in the vignette.


Reprinted with permission from Harper B. Index of suspicion. Pediatr Rev. 2017;38(7):340

Of the following, the infant’s MOST likely diagnosis is


A. anomalous coronary artery from the pulmonary artery
B. aortopulmonary window
C. dilated cardiomyopathy
D. tricuspid atresia

American Academy of Pediatrics 757


PREP ® Self-Assessment PREPSA 2021
Correct Answer: A
The baby in the vignette has evidence of heart failure with tachypnea, poor feeding, sweating
with feedings, tachycardia, irritability, a gallop, cool distal extremities, and hepatomegaly. Any
of the choices listed could present in heart failure, whether systolic dysfunction resulting from
cardiomyopathy or pulmonary overcirculation resulting from an aortopulmonary window.
However, the presence of the deep Q waves in the inferior leads on the electrocardiogram (Item
C207A) makes this presentation most consistent with an anomalous left coronary artery from the
pulmonary artery (ALCAPA). As its name implies, ALCAPA refers to the left coronary artery’s
arising from the pulmonary artery instead of from the aorta. In most children, the pulmonary
vascular resistance drops early in infancy, and in children with ALCAPA, this makes it easier for
blood to bypass the aberrant coronary artery and go out to the pulmonary vasculature, resulting
in ischemic injury to the myocardium and, ultimately, heart failure. The ischemia results in the Q
waves seen on electrocardiogram, which distinguishes ALCAPA, rather than the other potential
causes of heart failure listed in the response choices, as the diagnosis in this case.

Item C207A: Chest radiograph for the infant described in the vignette.
Reprinted with permission from Harper B. Index of suspicion. Pediatr Rev. 2017;38(7):338

Pediatric cardiologists often use the term “heart failure” in the context of many different
physiologies, which can be confusing. Systolic dysfunction, as seen in myocarditis, and
pulmonary overcirculation, as seen in a large left-to-right shunt, result in signs and symptoms of
heart failure. Children will exhibit shortness of breath, tachypnea, tachycardia, and sweating with
feedings or exercise intolerance (depending on age). They may have gastrointestinal complaints
of early satiety, nausea, and vomiting. They may complain of palpitations or chest pain. On
physical examination, hepatomegaly, a heart murmur, a cardiac gallop, or a combination of these
may be present.

The etiology of heart failure is age dependent. Children of any age can experience infectious
myocarditis or cardiomyopathy. However, many of the congenital heart diseases, such as
hypoplastic left heart syndrome, critical coarctation of the aorta, or critical aortic stenosis, are
more likely to present in a neonate or in early infancy. See Item C207B for a table showing
causes of heart failure in different age groups.

American Academy of Pediatrics 758


PREP ® Self-Assessment PREPSA 2021

American Academy of Pediatrics 759


PREP ® Self-Assessment PREPSA 2021
PREP Pearls
• Pediatric patients with heart failure can present with tachypnea, shortness of breath, and
exercise intolerance, including feeding intolerance or sweating with feedings.
• The etiology of heart failure in pediatrics can be age related: cardiomyopathy and
myocarditis can occur at any age, whereas some congenital heart diseases and inborn
errors of metabolism are more likely in infancy.
• Anomalous coronary artery from the pulmonary artery (ALCAPA) can present with heart
failure. Prominent Q waves on electrocardiogram are a characteristic feature, given the
ischemia from decreased left coronary artery blood flow as the pulmonary vascular
resistance drops.

MOCA-Peds Objective
• Recognize genetic causes of congenital heart disease.

ABP Content Specifications(s)


• Identify the causes of congestive heart failure in children of various ages
• Recognize the clinical findings associated with congestive heart failure in children of
various ages

Suggested Readings
• Flores S, Checchia PA. Recognition of impending systemic failure. Pediatr
Rev. 2017;38(11):520-529. doi:10.1542/pir.2016-0102.
• Harper B. Case 6: acute-onset respiratory failure in a 4-month-old girl. Pediatr
Rev. 2017;38(7):338-339. doi:10.1542/pir.2016-0093.
• Madriago E, Silberbach M. Heart failure in infants and children (published correction
appears in Pediatr Rev. 2010 Apr;31[4]:158). Pediatr Rev. 2010;31(1):4-12.
doi:10.1542/pir.31-1-4.
• Price JF. Congestive heart failure in children. Pediatr Rev. 2019;40(2):60-70.
doi:10.1542/pir.2016-0168.
• Rusconi PG, Harmon W, Wilkinson JD, Lipshultz SE. Heart failure. In: McInerny TK,
Adam HM, Campbell DE, DeWitt TG, Foy JM, Kamat DM, eds. American Academy of
Pediatrics Textbook of Pediatric Care. 2nd ed. Elk Grove Village, IL: American
Academy of Pediatrics; 2017:2868-2877. Pediatric Care Online .

American Academy of Pediatrics 760


PREP ® Self-Assessment PREPSA 2021
Question 208
A 13-year-old adolescent girl is brought to the clinic for chronic abdominal pain. She has been
seen in the clinic with this condition multiple times over the last several years. She has had
extensive evaluation, including consultation with a pediatric gastroenterologist and resultant
testing. She has been told by multiple providers that the most likely explanation for her
symptoms is functional abdominal pain. She has been encouraged to try cognitive behavioral
therapy; her parents have refused this option because they believe there is “something wrong”
with her physically. Now her parents are requesting a prescription for opioid for her pain and
further testing. A careful review of her history, physical examination findings, and laboratory
data confirms that all findings and results are within normal limits. She is denied a prescription
for opioids.

Of the following, the ethical principle MOST likely used in the health-care allocation decision
for this adolescent is
A. autonomy
B. beneficence
C. justice
D. nonmaleficence

American Academy of Pediatrics 761


PREP ® Self-Assessment PREPSA 2021
Correct Answer: D
Nonmaleficence, or primum non nocere (first, do no harm), is the ethical principle used in the
decision to not prescribe opioids for this adolescent. In this vignette, the health-care provider has
refused the parents’ request for an opioid prescription, because opioid treatment of her chronic
abdominal pain is medically unnecessary and carries the risk of significant adverse health
outcomes, including addiction and risk for overdose.

Opioid use increased significantly after the development of oral prescription opioid pain
medications in the 1990s. These medications were thought to be safe and effective for pain
management and many opioid prescriptions were provided for various conditions. This is
believed to have led to the current opioid epidemic in the United States. Adolescents are
particularly at risk for opioid addiction, because highly stimulating actions (and therefore high
neurologic rewards) drive their desire for repeated stimulation (and repeated neurologic rewards)
in a more intense way than in adults or younger children. In addition, the prefrontal cortex
(which drives impulse control), is not fully mature until the third decade, thus leaving the
adolescent more vulnerable to this addictive drive for stimulation. Because of the physiologic
risks for addiction, health-care providers need to be even more cautious when prescribing highly
addictive medications such as opioids for adolescents. More recent knowledge and understanding
of the risks has led to a decline in the rate of opioid prescription, but overdose death rates
continue to rise.

The other basic principles of medical ethics (Item C208) include beneficence, autonomy, and
justice. Beneficence complements nonmaleficence and it describes acts that benefit or promote
the well-being of the patient. A health-care provider discussing the risks and benefits of dietary
therapy or medications known to treat functional abdominal pain would be an example of
beneficence. Autonomy is the principle that allows parents/guardians to be fully informed and
make independent medical decisions for a child, acknowledging religious and personal beliefs.
Justice is the ethical principle that all people should be treated equally. Beneficence, autonomy,
and justice were not applied in this vignette to decide whether or not to prescribe opioids for
chronic abdominal pain.

American Academy of Pediatrics 762


PREP ® Self-Assessment PREPSA 2021

PREP Pearls
• Nonmaleficence, or primum non nocere (first, do no harm) is the ethical principle applied
when a provider avoids causing harm to a patient.
• Beneficence is the ethical principle applied when a provider acts to benefit or promote the
well-being of a patient.
• Autonomy is the principle that allows parents/guardians to be fully informed and
independently make medical decisions for a child, acknowledging religious and personal
beliefs.
• Justice is the ethical principle requiring that all people be treated equally.

ABP Content Specifications(s)


• Recognize and apply ethical principles involved in managed care issues
• Recognize and apply ethical principles regarding the just allocation of health care
resources

Suggested Readings
• Levy S. Youth and the opioid epidemic. Pediatrics. 2019;143(2):e20182752.
doi: 10.1542/peds.2018-2752.
• Ransom H, Olsson J. Allocation of health care resources: principles for decision-
making. Pediatr Rev. 2019;38:320-329. doi: 10.1542/pir.2016-0012.

American Academy of Pediatrics 763


PREP ® Self-Assessment PREPSA 2021
Question 209
A 9-year-old boy is seen in the office for evaluation of right eye swelling of 3 weeks’ duration.
He reports a painless bump on his right upper eyelid. He reports no fever, eye redness, discharge,
or vision changes. He has a 7-mm, firm, nonmobile nodule on his right upper eyelid without
overlying erythema or discharge (Item Q209 ). His conjunctiva are clear, and extraocular
movements are intact. The red reflex is present and symmetrical bilaterally.

Item Q209: Eyelid of the boy described in the vignette. Courtesy of the American Academy of
Ophthalmology (www.aao.org)

Of the following, the NEXT best step in management is


A. referral to an ophthalmologist
B. use of topical antibiotics
C. use of topical glucocorticoids
D. warm compresses

American Academy of Pediatrics 764


PREP ® Self-Assessment PREPSA 2021
Correct Answer: D
The boy in this vignette has a chalazion of the right upper eyelid. A chalazion results from an
obstruction and inflammation of the meibomian glands of the upper or lower eyelid, causing a
lipogranuloma. Clinically, it presents as a firm, painless, nonmobile, rubbery nodule in the lower
or upper eyelid, which varies in size between 3 and 10 mm. The skin overlying a chalazion has
minimal, if any, inflammatory changes. Larger lesions tend to be chronic and slow-growing,
lasting for several weeks to months. The primary treatment for a chalazion is application of
warm compresses to the eyelid multiple times per day. Instructions should include the avoidance
of microwaving, boiling, or placing compresses in the oven to heat, given the association of these
measures with burns.

Because chalazia are not caused by an infectious process, neither topical nor oral antibiotics are
indicated unless there is a secondary infection present, such as blepharitis or cellulitis. A
secondary infection will typically present with the onset of pain, erythema, edema, tenderness,
and possibly even fever. Most chalazia resolve spontaneously with the use of warm compresses.
Referral to an ophthalmologist should be considered for lesions that fail to resolve after 4 to 6
weeks of conservative management or for recurrent lesions. Excision may be necessary if the
chalazion distorts vision or leads to a cosmetic issue. Glucocorticoid injections, not topical
therapy, may be a treatment initiated by an ophthalmologist for lesions that fail to resolve
spontaneously.

In contrast to a chalazion, an external hordeolum (sty) represents an acute infectious process of


the glands of the eyelid (the meibomian glands or the glands of Zeis or Moll) that are associated
with hair follicles. An internal hordeolum results from an infection of the meibomian gland, the
large sebaceous gland with an opening at the lid margin. The typical infectious agent
is Staphylococcus aureus. Clinically, a sty presents as a tender, edematous, and erythematous
abscess of the eyelid. Internal hordeola tend to have more diffuse edema just under the
conjunctival side of the eyelid. Most hordeola resolve without intervention over several days.
The primary management for a hordeolum consists of warm compresses to help relieve the
obstruction and promote drainage. Use of eye makeup should be avoided. A topical antibiotic
ointment may be applied if significant blepharitis is present, but there is little evidence that
topical antibiotics result in rapid healing. Surgical excision and drainage by an ophthalmologist
may be required for lesions that fail to decrease in size within 1 to 2 weeks.

PREP Pearls
• A chalazion is a painless lipogranuloma that tends to be chronic in nature, lasting weeks
to months, whereas a hordeolum (sty) is an acute infectious process.
• The primary treatment for a chalazion is application of warm compresses to the eyelid
multiple times per day.

ABP Content Specifications(s)


• Plan the appropriate management of a stye
• Plan the appropriate management of a chalazion
• Differentiate the clinical findings associated with a stye from those of a chalazion

American Academy of Pediatrics 765


PREP ® Self-Assessment PREPSA 2021
Suggested Readings
• Aguilera ZP. Chen PL. Eye pain in children. Pediatr Rev. 2016;37(10):418-425.
doi:10.1542/pir.2015-0096.
• Lederman C, Miller M. Hordeola and chalazia. Pediatr Rev. 1999;20(8):283-284.
doi:10.1542/pir.20-8-283.
• Otlisky SE, Hug D, Plummer LS, Stahl ED, Ariss MM, Lindquist TP. Abnormalities of
the lids. In: Kliegman RM, Stanton BF, St Geme JW III, Schor NF, Behrman RE,
eds. Nelson Textbook of Pediatrics. 20th ed. Philadelphia, PA: Saunders Elsevier;
2016:3033-3034.
• Rosales T. Infections of the eye. In: Berkowitz, CD, ed. Berkowitz’s Pediatrics: A
Primary Care Approach. 5th ed. Elk Grove Village, IL: American Academy of
Pediatrics; 2014:495-500.
• Wald ER. Periorbital and orbital infections. Pediatr Rev. 2004;25(9):312-320.
doi:10.1542/pir.25-9-312.

American Academy of Pediatrics 766


PREP ® Self-Assessment PREPSA 2021
Question 210
A 14-year-old adolescent is seen for a routine health supervision visit. He has no complaints, his
development has been normal, and he participates in multiple sports. At 6 years of age, an
intramedullary nail was placed for a left midshaft femur fracture. Subsequent radiography
initially showed normal growth of the left femur; however, the adolescent did not continue
orthopedic follow-up.

On physical examination, there is significant asymmetry of the adolescent’s iliac crests; the left
hip appears elevated compared with the right. He has full range of motion with normal and
symmetric strength on examination of his back, hips, and knees. Leg length measurements from
the anterior superior iliac spine to the medial malleolus reveal that the left leg is 2 cm longer than
the right.

Of the following, the BEST next management step for this adolescent is to
A. follow-up in 6 months for repeat measurement
B. reassure the family that this finding is common
C. refer him to an orthopedic surgeon
D. refer him to an orthotist for a heel lift

American Academy of Pediatrics 767


PREP ® Self-Assessment PREPSA 2021
Correct Answer: C
Leg length discrepancies (LLDs) are found in up to 90% of the population at skeletal maturity,
and are usually clinically insignificant. However, LLDs of 2 cm or greater, as seen in the
adolescent in the vignette, place the child at higher risk for future difficulties and should be
referred to an orthopedic surgeon for surgical consideration. It would not be appropriate to
simply provide reassurance or to delay referral, pending reassessment of the child after 6 months.
Leg length differences can be either congenital or acquired; therefore, a thorough review of the
medical history is essential. Causes of congenital LLDs include hip dysplasia, hemihypertrophy,
deficiencies (or hemimelias) of the tibia or fibula, congenital femoral deficiency, or other limb
hypoplasias. Some cases of congenital LLDs may not be evident at birth, and only become
apparent after a period of growth. Children with suspected congenital LLDs should be referred
for ongoing care with a pediatric orthopedist.

Acquired LLDs are more common and often idiopathic, but may occur with under- or
overgrowth of a limb because of trauma, surgery, infection, radiation, tumor, or dysplasia. Wilms
tumor is typically associated with ipsilateral hemihypertrophy, therefore further evaluation
should be considered if other signs or symptoms are present, such as an abdominal mass,
elevated blood pressure, or hematuria. Leg length discrepancies may increase with skeletal
growth.

Acquired LLDs are often noted incidentally on examination as asymmetry of waist contour or of
iliac crest heights. Once recognized, leg length measurements should be obtained as a baseline.
Functional leg length is determined not only by the true length of the limb itself, but also by
noting any contracture or angular or rotational asymmetry of the hips, knees, ankles, or feet.
Assessment of leg length begins with anterior and posterior visual inspection of both legs while
standing barefoot. Asymmetry of the sacral dimples and iliac crest heights should be noted. True
leg length is obtained by measuring the distance between the anterior superior iliac spine (ie, the
bony prominence over the anterior aspect of the iliac crest) and the inferior aspect of the medial
malleolus of the ankle. Functional leg length can be measured using either direct or indirect
methods.
• Direct measurement: With the child standing, measure the distance from the umbilicus to
the inferior aspect of the medial malleolus.
• Indirect (“block”) measurement: With the child standing, place blocks (or books) of
known height under the shorter limb until the pelvis is leveled. This method is generally
preferred; accuracy is not affected by soft tissue excess, and this method accounts for any
length differences caused by asymmetry in the feet or ankles.

With either of these techniques, documenting the average of 2 separate measurements is


recommended. The joints of the lower limbs should then be assessed for any asymmetry in
alignment or range of motion.

The average LLD in skeletally mature individuals is ~5 mm. Leg length discrepancies of 1 cm or
less are considered to be within normal limits. Current recommendations for the management of
LLDs are noted in Item C210 .

American Academy of Pediatrics 768


PREP ® Self-Assessment PREPSA 2021

Radiography is generally performed in children who are referred for orthopedic surgery
evaluation. However, various imaging techniques are available (eg, scanography,
teleoradiography, orthoradiography, and computed tomography scout radiography). To minimize
radiation exposure, imaging choices should be determined by the orthopedic surgeon’s
preference.

Although small LLDs are generally asymptomatic, their natural history is unclear. Some authors
report that LLDs of 5 mm or greater correlate with future knee, hip, and low back pathology, and
may contribute to pelvic torsion. Children who participate in activities with significant, repeated
impact of the legs (eg, runners, military) may note symptoms for which LLDs serve as a
contributing factor. For example, runners may be prone to developing iliotibial band syndrome in
the longer leg, and use of a heel lift may be of benefit in these cases; however, prophylactic use
of heel lifts in asymptomatic individuals is generally not recommended.

PREP Pearls
• Leg length discrepancies are common and usually asymptomatic.
• Leg length discrepancies should be monitored for progression during linear growth.
• Leg length discrepancies greater than 2 cm warrant evaluation for possible surgical
intervention.

American Academy of Pediatrics 769


PREP ® Self-Assessment PREPSA 2021
ABP Content Specifications(s)
• Recognize the clinical findings associated with leg length discrepancy

Suggested Readings
• Gordon JE, Davis LE. Leg length discrepancy: the natural history (and what do we really
know). J Pediatr Ortho. 2019;39(suppl):S10-13. doi: 10.1097/BPO.0000000000001396.
• Sabharwal S, Kumar A. Methods for assessing leg length discrepancy. Clin Orthop Relat
Res. 2008;466(12):2910-2922. doi: 10.1007/s11999-008-0524-9.

American Academy of Pediatrics 770


PREP ® Self-Assessment PREPSA 2021
Question 211
A 32-year-old woman with HIV infection gives birth to a full-term boy via vaginal delivery.
Rupture of membranes occurred 5 hours before delivery. The woman was taking antiretroviral
medications before conception and continued the same regimen during this pregnancy. Her viral
load was undetectable at her most recent obstetric visit. During labor, she did not receive
intrapartum HIV prophylaxis.

Of the following, the boy’s risk of acquiring HIV infection is influenced MOST by
A. lack of intrapartum prophylaxis
B. length of rupture of membranes
C. maternal viral load
D. mode of delivery

American Academy of Pediatrics 771


PREP ® Self-Assessment PREPSA 2021
Correct Answer: C
The risk of acquiring HIV infection by the boy in the vignette is influenced most by his mother’s
viral load. Mother-to-child transmission of HIV usually occurs during the process of labor and
delivery. Maternal viral load is a key determinant of the likelihood that an HIV-infected mother
will transmit the virus to her child.

Mother-to-child transmission of HIV infection in the United States has been curtailed through
various measures including routine testing of mothers during pregnancy, antiretroviral
prophylaxis for both mother and child, cesarean delivery before the onset of labor or rupture of
membranes, and complete avoidance of breastfeeding. In general, women infected with HIV
should be treated with antiretroviral drugs during pregnancy to both treat the HIV infection and
minimize the potential for transmission to the child.

Intrapartum prophylaxis is not indicated when a woman’s HIV viral load is undetectable near or
during delivery. Women should receive intrapartum prophylaxis with zidovudine when their
viral load is at least 1,000 copies/mL or unknown near the time of delivery.

Generally, the interval since the rupture of membranes influences the likelihood of HIV
transmission, with the risk rising with each hour since membrane rupture. However, for women
whose viral load is 1,000 copies/mL or less, rupture of membranes is not associated with an
increased risk of transmission and is not an indication for cesarean delivery.

For women whose viral load is undetectable, such as the mother in the vignette, cesarean
delivery is not routinely recommended. In the United States, cesarean delivery at 38 weeks’
gestation, before onset of labor or rupture of membranes, is recommended for HIV-infected
women whose viral load is greater than 1,000 copies/mL or whose viral load is unknown.

All infants born to HIV-infected mothers should receive antiretroviral prophylaxis as soon as
possible after birth, ideally within 6 to 12 hours. Infants born to mothers who were treated with
antiretroviral drugs during pregnancy and had sustained viral suppression (HIV RNA level <50
copies/mL) should receive zidovudine treatment for 4 weeks. In other circumstances, infants
should receive either a 2- or 3-drug antiretroviral regimen based on the perinatal guidelines
available at https://aidsinfo.nih.gov/guidelines/html/3/perinatal/0.

PREP Pearls
• Mother-to-child transmission of HIV in the United States has been curtailed through
various measures including routine testing of mothers during pregnancy, antiretroviral
prophylaxis for HIV-positive mothers and their children, cesarean delivery before the
onset of labor or rupture of membranes, and complete avoidance of breastfeeding.
• Maternal viral load is a key determinant of the likelihood that an HIV-infected mother
will transmit the virus to her child.
• For women with viral loads less than 1,000 copies/mL, intrapartum prophylaxis and
routine cesarean delivery before the onset of labor or rupture of membranes is not
indicated.

American Academy of Pediatrics 772


PREP ® Self-Assessment PREPSA 2021
ABP Content Specifications(s)
• Understand the effect of a mother’s positive test for human immunodeficiency virus
(HIV) on the results of her infant’s HIV test
• Understand the epidemiology of human immunodeficiency virus, including the modes of
transmission and how to minimize transmission risk

Suggested Readings
• American Academy of Pediatrics. Human immunodeficiency virus infection. In:
Kimberlin DW, Brady MT, Jackson MA, Long SS, eds. Red Book: 2018 Report of the
Committee on Infectious Diseases. Itasca, IL: American Academy of Pediatrics;
2018:459-476. Red Book Online.
• Brady MT, Persaud D, Moss W. Human immunodeficiency virus infection and acquired
immunodeficiency syndrome. In: McInerny TK, Adam HM, Campbell DE, DeWitt TG,
Foy JM, Kamat DM, eds. American Academy of Pediatrics Textbook of Pediatric
Care. 2nd ed. Itasca, IL: American Academy of Pediatrics; 2016;chap 268:2153-
2161. Pediatric Care Online.
• Panel on Treatment of Pregnant Women With HIV Infection and Prevention of Perinatal
Transmission. Recommendations for use of antiretroviral drugs in transmission in the
United States. https://aidsinfo.nih.gov/guidelines/html/3/perinatal/0.
• Sibery G. Preventing and managing HIV infection in infants, children, and adolescents in
the United States. Pediatr Rev. 2014;35(7):268-286. doi: 10.1542/pir.35-7-268.

American Academy of Pediatrics 773


PREP ® Self-Assessment PREPSA 2021
Question 212
A 4-year-old girl is seen for worsening of a persistent cough and wheezing that has been present
for the last 3 years. She has been treated with inhaled budesonide and albuterol without
resolution of her symptoms. She has intermittent pain in the right lateral abdomen that does not
interfere with sleep or any activities. On physical examination she is healthy appearing, and her
vital signs and growth parameters are normal. The oxygen saturation on pulse oximetry is 96%
on room air. She has nasal congestion but otherwise normal upper airway examination findings.
There is no increased work of breathing. There are no adventitious sounds in the chest, but breath
sounds are diminished on the right. Cardiac examination findings are normal. The liver is
palpable 5 cm below the right costal margin, but the spleen is not palpable. There is no digital
clubbing. A chest radiograph is obtained (Item Q212).

Item Q212: Chest radiographs of the girl in the vignette. Courtesy of M. Guill.

Of the following, the MOST appropriate next step in making a diagnosis is


A. computed tomography of the chest with intravenous contrast
B. flexible bronchoscopy and bronchoalveolar lavage
C. modified barium swallow with speech therapist
D. thoracoscopic biopsy of right lower lobe

American Academy of Pediatrics 774


PREP ® Self-Assessment PREPSA 2021
Correct Answer: A
The girl in the vignette has abnormal radiographic findings in the right lung; areas of
hyperinflation and cyst formation suggest a congenital pulmonary airway malformation that
needs further anatomic definition. Computed tomography of the chest with intravenous contrast
will provide additional definition of lung structure.

Flexible bronchoscopy will provide evaluation of airway anatomy, but will not help evaluate
parenchymal abnormalities or airway abnormalities beyond the segmental bronchus. A modified
barium swallow would assist in evaluating the integrity of this girl’s swallow and her potential
for aspiration, but neither the history nor the radiographic findings is suggestive of aspiration. A
thoracoscopic biopsy would help evaluate the child’s pulmonary microanatomy, but not with the
overall structural abnormality.

The computed tomography findings in this patient (Item C212A, Item C212B, Item
C212C) confirm a large malformation occupying most of the right chest composed of multiple
cysts with atelectasis of innate normal lung. Item C212C shows a truncated right lower lobe
posterior segment bronchus indicating the source of the lesion. The multicystic lesion in that
segment fills the entire right chest, with the remaining lobes and segments being almost
completely atelectatic.

Item C212A: Computed tomography images of the girl in the vignette. Courtesy of M. Guill.

American Academy of Pediatrics 775


PREP ® Self-Assessment PREPSA 2021

Item C212B: Coronal view showing congenital pulmonary airway malformation.


Courtesy of M Guill

American Academy of Pediatrics 776


PREP ® Self-Assessment PREPSA 2021

Item C212C: 3-dimensional reconstruction of the airway and lung from the computed
tomography for the girl in the vignette. Courtesy of M Guill

Congenital pulmonary airway malformation is part of the spectrum of cystic lesions of the lower
airway, which also includes congenital lobar emphysema. Many lesions are diagnosed with
prenatal ultrasonography; 70% of those resolve or decrease in size before birth, 10% remain
unchanged, and 20% increase in size. Those presenting in the newborn period with respiratory
compromise may require surgical resection. Some have no early symptoms or their symptoms
are mild enough that they are not diagnosed until much later.

The presence of persistent or unusual cough, wheezing not responding to appropriate treatment
for asthma, or recurrent respiratory infections indicates a need for additional investigation. Plain
radiography of the chest is recommended as the first imaging modality. Results may suggest the
need for additional testing. In most cases, computed tomography of the chest is the best
additional imaging modality to further investigate anomalies of the pulmonary parenchyma or
airways identified on chest radiography.

American Academy of Pediatrics 777


PREP ® Self-Assessment PREPSA 2021

PREP Pearls
• Most congenital pulmonary airway malformations are diagnosed prenatally; 70% of those
diminish or resolve before birth.
• Rarely, congenital pulmonary airway malformation may present later in life with chronic
respiratory symptoms.
• Congenital pulmonary airway malformation and congenital lobar emphysema represent a
spectrum of airway anomalies present at birth.

ABP Content Specifications(s)


• Recognize the clinical findings associated with congenital malformations of the lower
airway

Suggested Readings
• Cook J, Chitty LS, Decoppi P. The natural history of prenatally diagnosed congenital
cystic lung lesion: long term follow up of 119 cases. Arch Dis Child. 2017;102(9):798-
803. doi: 10.1136/archdischild-2016-311233.
• McBride W. Congenital lesions of the lung. NeoReviews. 2016;17(5):e263-e270.
doi: 10.1542/neo.17-5-e263.
• Nafday SM, Long C. Respiratory distress and breathing disorders in the newborn. In:
McInerny TK, Adam HM, Campbell DE, DeWitt TG, Foy JM, Kamat DM,
eds. American Academy of Pediatrics Textbook of Pediatric Care. 2nd ed. Itasca, IL:
American Academy of Pediatrics; 2016; chap 100:867-887. Pediatric Care Online.
• O’Sullivan B, Kinane TB. Congenital lung anomalies. In: Light MJ, Blaisdell CJ,
Homnick DN, eds. Pediatric Pulmonology. Itasca, IL. American Academy of Pediatrics;
2011;chap 14: 277-307.

American Academy of Pediatrics 778


PREP ® Self-Assessment PREPSA 2021
Question 213
A 3-year-old boy is brought to the emergency department for evaluation of a headache of 2 days’
duration. The boy cannot localize or qualify the pain, but says that his head hurts “a lot.” For the
past 2 days, he has been slightly confused in the morning, but this seems to be resolved when he
comes home from school. He vomited twice on the day of presentation, but denies abdominal
pain or diarrhea. The parents deny trauma and fever. Two older siblings and both parents have
also been having mild generalized headaches for 2 days. The child’s vital signs are normal; aside
from slightly red lips, his physical examination findings are normal.

Of the following, the blood level MOST likely to be high in this boy is
A. carboxyhemoglobin
B. methemoglobin
C. myoglobin
D. sulfhemoglobin

American Academy of Pediatrics 779


PREP ® Self-Assessment PREPSA 2021
Correct Answer: A
The child and family in the vignette are suffering from carbon monoxide poisoning. Carbon
monoxide, known as the “invisible killer,” is a colorless, odorless, tasteless gas, and its
inhalation can be fatal. Carbon monoxide has a stronger affinity for hemoglobin than does
oxygen (approximately 240 times stronger); if inhaled, it will displace oxygen molecules from
hemoglobin, resulting in hypoxemia. Carbon monoxide also diminishes hemoglobin’s ability to
release oxygen and thereby decreases peripheral oxygen delivery.

Most carbon monoxide poisoning events occur from inhalation of smoke during a fire. Other
sources of carbon monoxide include poorly ventilated areas (eg, parking garages, boathouses),
improperly functioning heating systems, charcoal grills, camping stoves, and hookah use. Most
incidents of carbon monoxide toxicity occur during the wintertime. Because of its inability to be
detected by the human senses and its potential lethality, the US Consumer Product Safety
Commission recommends that a carbon monoxide detector with alarms be installed on each level
of every home; if battery-operated, the batteries should be changed biannually at daylight saving
time.

Young children are more susceptible to carbon monoxide poisoning because they have a higher
minute ventilation than adults and, thus, typically exhibit symptoms sooner than adults even in a
shared exposure. Signs and symptoms of carbon monoxide poisoning are nonspecific and include
headache, malaise, dizziness, nausea, and emesis. Mental status may be altered, ranging from
mild confusion to coma. Chest pain and myocardial injury can also occur. Infants and toddlers
may only be fussy and have decreased oral intake. The physical examination findings are
nonspecific, and the lips may appear cherry-red.

Because of the paucity of specific clinical signs and symptoms, providers must have a high index
of suspicion for carbon monoxide poisoning. The diagnosis of carbon monoxide poisoning is
made via blood gas carbon monoxide oximeter analysis, which shows an elevated serum
carboxyhemoglobin level (carbon monoxide oximetry measures hemoglobin,
carboxyhemoglobin, methemoglobin, oxyhemoglobin, and sulfhemoglobin levels). Importantly,
routine transcutaneous pulse oximetry does not differentiate oxyhemoglobin from
carboxyhemoglobin and thus is not useful for the diagnosis of carbon monoxide poisoning.

As with all emergencies, management of carbon monoxide poisoning begins with evaluation and
stabilization of the airway, breathing, and circulation. Endotracheal intubation may be necessary
for respiratory support. High-flow oxygen via a non-rebreather mask is the mainstay of treatment
of carbon monoxide poisoning. Administration of high-flow oxygen decreases the half-life of
carbon monoxide from approximately 300 min to 90 min. Hyperbaric oxygen therapy should be
considered for carbon monoxide levels greater than 25%, severe acidosis, coma, or other
evidence of end-organ dysfunction.

Methemoglobinemia is typically caused by medications, including topical anesthetics, but can


also occur in infants who are drinking well water (because of dissolved nitrates) or young
children with severe diarrhea and dehydration. Because of alterations in absorption of light at the
wavelengths of oxyhemoglobin and deoxyhemoglobin, pulse oximetry in methemoglobinemia

American Academy of Pediatrics 780


PREP ® Self-Assessment PREPSA 2021
will read 85%. Blood drawn from children with methemoglobinemia will appear dark brown,
like chocolate syrup.

Sulfhemoglobinemia results from the binding of a sulfur atom to hemoglobin.


Sulfhemoglobinemia is typically caused by drugs that contain sulfonamides or by occupational
exposures to sulfur. It causes the blood to have a greenish color.
Myoglobin is found in muscle tissue and is released into the blood when muscles are damaged or
during strenuous exercise. In children with rhabdomyolysis, excessive myoglobin may
accumulate in the kidneys and cause damage.

PREP Pearls
• Symptoms of carbon monoxide poisoning are nonspecific and can include altered mental
status, headache, malaise, nausea, and emesis.
• Carbon monoxide poisoning is diagnosed by using blood gas carbon monoxide oximetry.
• High-flow oxygen administration via non-rebreather mask is the mainstay of treatment
for carbon monoxide poisoning. Severe poisoning may necessitate hyperbaric oxygen
therapy.
• The US Consumer Product Safety Commission recommends that a carbon monoxide
detector with alarms be installed on each level of every home; if battery-operated, the
batteries should be changed biannually at daylight saving time.

ABP Content Specifications(s)


• Recognize the signs and symptoms of carbon monoxide poisoning, and manage
appropriately

Suggested Readings
• Fine JS. Poisoning. In: McInerny TK, Adam HM, Campbell DE, DeWitt TG, Foy JM,
Kamat DM, eds. American Academy of Pediatrics Textbook of Pediatric Care. 2nd ed.
Elk Grove Village, IL: American Academy of Pediatrics; 2017:2924-2949. Pediatric Care
Online.
• Kind T. Carbon monoxide. Pediatr Rev. 2005;26(4):150-151. doi:10.1542/pir.26-4-150.
• Tomaszewski C. Carbon monoxide. In: Nelson LS, Howland MA, Lewin NW, Smith SA,
Goldfrank LR, Hoffman RS, eds. Goldfrank’s Toxicologic Emergencies. 11th ed. New
York. McGraw-Hill; 2019:1663-1675.
• US Consumer Product Safety Commission. Carbon monoxide information
center. https://www.cpsc.gov/Safety-Education/Safety-Education-Centers/Carbon-
Monoxide-Information-Center.

American Academy of Pediatrics 781


PREP ® Self-Assessment PREPSA 2021
Question 214
A newborn is seen in a clinic for his first visit after discharge from the neonatal intensive care
unit. He was born vaginally at term to first-time parents after an uncomplicated pregnancy. At
delivery, he was noted to have clusters of blisters on his right hand, his posterior neck, and his
left upper arm. An infectious evaluation had negative results. Skin biopsy with
immunofluorescence microscopy mapping showed staining of basement membrane antigens in
the floor of the blister, consistent with intraepidermal cleavage seen in epidermolysis bullosa.
During his hospitalization, the neonate developed several oral blisters. His parents have been
educated on skin and oral care for his condition. They have questions about his long-term
prognosis and the likelihood that he will develop other medical problems.

Of the following, the MOST common extracutaneous manifestation of this boy’s condition is
A. blindness
B. hepatic fibrosis
C. malnutrition
D. nephritis

American Academy of Pediatrics 782


PREP ® Self-Assessment PREPSA 2021
Correct Answer: C
The newborn in the vignette has epidermolysis bullosa (EB). This is suggested by his physical
examination findings and confirmed on skin biopsy. Epidermolysis bullosa is a heterogeneous
group of rare disorders (approximately 20 per million live births), all characterized by
mechanical fragility of the epithelial tissue. The severity, age at onset of cutaneous
manifestations, and prevalence of extracutaneous symptoms varies between subtypes.

The most common extracutaneous manifestations are nail, dental, mucous membrane, and eye
abnormalities. Malnutrition is a common complication, occurring because of feeding difficulties;
malabsorption; increased caloric requirements because of chronic inflammation, accelerated skin
cell turnover, and continuous wound healing; and excess losses across the abnormal cutaneous
barrier. Other complications include malnutrition-related anemia, hypoalbuminemia, and growth
failure; infection; and an increased risk of skin cancer. Although patients with EB can have
ocular abnormalities (eg, conjunctival irritation, corneal ulcerations, lacrimal duct obstruction)
blindness is rare. Hepatic fibrosis and nephritis are not common manifestations of EB.

PREP Pearls
• Epidermolysis bullosa is a heterogenous group of disorders characterized by mechanical
fragility of the epidermal tissues.
• Common extracutaneous manifestations of epidermolysis bullosa include nail, dental,
mucous membrane, and eye abnormalities. Malnutrition and infection are common
complications of epidermolysis bullosa, and patients have an increased risk of skin
cancer.

ABP Content Specifications(s)


• Recognize the clinical findings associated with epidermolysis bullosa

Suggested Readings
• Fine JD, Bruckner-Tuderman L, Eady RA, et al. Inherited epidermolysis bullosa; updated
recommendations on diagnosis and classification. J Am Acad Dermatol.
2014:70(6):1103-1126. doi: 10.1016/j.jaad.2014.01.903.
• Fine JD. Inherited epidermolysis bullosa. Orphanet J Rare Dis. 2010;5:12.
doi: 10.1186/1750-1172-5-12.

American Academy of Pediatrics 783


PREP ® Self-Assessment PREPSA 2021
Question 215
A 17-year-old adolescent girl with bipolar disorder and oppositional defiant disorder is brought
to the emergency department from juvenile hall for concern of weight loss over the past 2 weeks.
She is serving a 6-week sentence for stealing her mother’s car and not meeting her probation
requirements. She states that she has been unable to sleep and has tremors and a decreased
appetite. She has had intermittent abdominal pain but no nausea or vomiting. She has lost
approximately 5 kg during the past 2 weeks. She denies intentionally trying to lose weight. She
declines to answer questions regarding her use of tobacco, alcohol, or drugs. Her last menstrual
period was 2 months ago, and she had a contraceptive implant placed 1 month ago. She has a
temperature of 37°C, a heart rate of 120 beats/min, a blood pressure of 108/74 mm Hg, and a
respiratory rate of 20 breaths/min. Her weight is 49 kg (10th percentile), and her height is 164
cm (50th percentile). She appears restless. The remainder of her physical examination findings
are unremarkable.

Of the following, the patient’s MOST likely diagnosis is


A. adrenal insufficiency
B. anorexia nervosa
C. cannabis withdrawal
D. hyperthyroidism

American Academy of Pediatrics 784


PREP ® Self-Assessment PREPSA 2021
Correct Answer: C
The girl in the vignette is showing signs of cannabis withdrawal. Symptoms usually occur within
1 week and can last up to 2 weeks in individuals with heavy or long-term use who abruptly stop
using. Cannabis withdrawal is a clinical diagnosis based on three of seven symptoms:
• Anxiety
• Restlessness
• Irritability/aggression
• Insomnia
• Decreased appetite/weight loss
• Depression
• Somatic complaints (including tremors, sweating, fever, chills, and abdominal pain)

Hyperthyroidism, adrenal insufficiency, and anorexia nervosa would all be included in the
differential diagnosis for weight loss in an adolescent. However, this girl’s recent onset of
symptoms and unwillingness to disclose information about substance use raise the most
suspicion about withdrawal.

The active agents in marijuana are a variety of cannabinoid compounds from the cannabis plant.
The main phytocannabinoids are tetrahydrocannabinol (THC), cannabidiol, and cannabinol,
which bind to the cannabinoid receptors (CB1 and CB2) in the central and peripheral nervous
system. CB1 receptors are primarily in the brain; CB2 receptors are in the peripheral tissues and
immune cells. There are also synthetic cannabinoids with chemical structures similar to that of
THC that can bind to synthetic cannabinoid receptor agonists. Marijuana is often inhaled; it also
can be ingested (as edibles in the form of candy, gummies, and baked goods). When inhaled, the
primary active component, THC, which is lipophilic, is rapidly distributed to the brain and
peripheral tissues. Tetrahydrocannabinol can be detected in the blood within seconds after
inhalation, and its effects last for 1 to 2 hours. If marijuana is ingested, peak concentrations are
reached in 1 to 2 hours, with effects lasting up to 6 hours. Tetrahydrocannabinol is metabolized
by the liver, and its metabolites can be detected in the urine 3 to 5 days after single use—and for
weeks after use in chronic smokers.

Adolescents have more accessibility to marijuana in the United States with legalization of
marijuana for medicinal and recreational use. Marijuana products currently available can have
concentrations of THC between 20% and 70%, whereas products in the past contained 5% THC.
Adolescents who consume edibles may be at increased risk of becoming intoxicated if they
ingest large quantities because they do not notice an immediate effect (as in inhalation);
however, they may be intoxicated in 1 to 2 hours. Symptoms in any age group would be similar,
including in an accidental ingestion in children.

Signs of marijuana intoxication include euphoria, increased appetite, anxiety, and paranoia.
Physical signs include tachycardia, dry mouth, and conjunctival injection. Individuals who have
overdosed may exhibit vomiting, tachycardia, and lethargy. Managing symptoms of marijuana
intoxication in adolescents is primarily supportive, as the effects will dissipate in a few hours.
The synthetic marijuana compounds (eg, Spice or K2) can have more severe effects. These
products are not regulated and usually are sold online and in convenience stores. Intoxication
with a synthetic cannabinoid may include aggressive behavior, dystonia, and seizures. Lastly, it
American Academy of Pediatrics 785
PREP ® Self-Assessment PREPSA 2021
is important to be aware of cannabis hyperemesis syndrome, in which an adolescent who has
acute intoxication may experience intractable vomiting.

PREP Pearls
• Marijuana has a rapid effect when inhaled and may have a delayed response of 1 to 2
hours when ingested. Pediatricians should counsel patients about potential effects if they
ingest multiple edibles at once.
• Withdrawal from marijuana usually occurs within 1 week after abrupt cessation in
chronic users and can last up to 2 weeks.
• Tetrahydrocannabinol can be detected in the urine 3 to 5 days after single use, and it can
be detected for weeks after use in chronic smokers.

ABP Content Specifications(s)


• Recognize the major behavioral consequences of marijuana use/abuse
• Identify the major physiologic consequences associated with marijuana use/abuse

Suggested Readings
• Braymiller JL, Masters LD, Linden-Carmichael AN, Lanza ST. Contemporary patterns of
marijuana use and attitudes among high school seniors: 2010-2016. J Adolesc Health.
2018;63(4):394-400. doi:10.1016/j.jadohealth.2018.06.005.
• Gray KM, Roper AP. Marijuana. In: Neinstein LS, Katzman DK, Callahan ST, Gordon
CM, Joffe A, Rickert VI, eds. Neinstein’s Adolescent and Young Adult Health Care: A
Practical Guide. 6th ed. Philadelphia, PA: Wolters Kluwer; 2016:547-550.
• Grigsby TM, Hoffmann LM, Moss MJ. Marijuana use and potential implications of
marijuana legalization. Pediatr Rev. 2020;41(2):61-72. doi:10.1542/pir.2018-0347.

American Academy of Pediatrics 786


PREP ® Self-Assessment PREPSA 2021
Question 216
A 6-year-old girl with a deep vein thrombosis is being treated with twice-daily injectable
anticoagulant, which is the standard of care. Her parents report that every time she receives a
dose of the drug, she screams and cries hysterically and will push, punch, and bite her parents as
they try to give her the injection. In researching possible alternative treatments, the pediatrician
identifies a large, randomized, controlled clinical trial in which the injectable standard-of-care
drug was compared with a once-daily oral anticoagulant approved by the US Food and Drug
Administration. There was no statistical difference in efficacy between the two therapeutic
options. However, the rate of bleeding complications was 0.5% in the oral anticoagulant group
versus 0.3% in the standard-of-care injectable drug group (P value = .04).

Of the following, the MOST accurate statement about the use of the oral anticoagulant for this
patient is that it
A. can be considered because the increased risk for bleeding complications is not clinically
important despite the statistical significance
B. requires a waiver from the US Food and Drug Administration given the statistically
significant increased risk of bleeding complications
C. should be considered as first-line therapy because there is no statistical difference in the
risk of bleeding complications
D. should never be considered given the statistically significant increased risk of bleeding
complications

American Academy of Pediatrics 787


PREP ® Self-Assessment PREPSA 2021
Correct Answer: A
The practice of evidence-based medicine involves making decisions about the care of individual
patients using the best evidence available. For the girl in the vignette, the use of an injectable
anticoagulant has effectively managed the deep vein thrombosis but concomitantly has
dramatically decreased the child’s quality of life, as well as that of her parents, presumably
owing to the child’s fear of needles. The results of the large clinical trial reported in the vignette
show that an oral anticoagulant is equally as efficacious as the injectable agent but carries a
statistically significant, but clinically minimal, increase in risk of bleeding events (in terms of
absolute numbers). As the risk of bleeding events was 0.5% in the oral anticoagulant study arm
versus 0.3% in the injectable anticoagulant arm, this means that for every 1,000 patients treated,
two additional patients will experience a bleeding event when receiving the oral anticoagulant
versus those receiving the injectable anticoagulant. When considering the best therapeutic option
for this individual patient, the small increase in risk of a bleeding event without any sacrifice in
efficacy must be weighed against the gain achieved in quality of life. In other words, there needs
to be a distinction between a statistically significant finding in a population and a clinically
relevant finding for an individual patient. For this patient, it would be appropriate to prescribe
the oral anticoagulant.

Use of the oral anticoagulant would not require a waiver from the Food and Drug
Administration, as the drug has already been approved. It would be considered an off-label use if
it has not been approved for use in children. Although the oral anticoagulant may be the best
therapeutic option for the child in the vignette, it would not necessarily be considered the first-
line agent owing to the statistically significantly increased risk of bleeding events. Generally,
a P value of < .05 is considered statistically significant (ie, the likelihood that the difference
between the study arms was due to chance is less than 5%). All other things being equal, the
injectable anticoagulant, therefore, would be the preferred option.

PREP Pearls
• The practice of evidence-based medicine involves making decisions about the care of
individual patients using the best evidence available.
• It is important to be able to distinguish between a statistically significant finding in a
population and a clinically relevant finding for an individual patient.

ABP Content Specifications(s)


• Distinguish statistical significance from clinical importance

Suggested Readings
• Ghazarian SR. Distinguishing statistical significance from clinical importance: the value
of the P value. Pediatr Rev. 2011;32(2):73-74. doi:10.1542/pir.32-2-73.
• Smith TK, Johnson SB. Research and statistics: distribution, variability, and statistical
significance. Pediatr Rev. 2010;31(10):431-432. doi:10.1542/pir.31-10-431.

American Academy of Pediatrics 788


PREP ® Self-Assessment PREPSA 2021
Question 217
A 2-day-old, full-term female newborn is evaluated in the nursery for decreased feeding and
somnolence. Her parents report that she has been difficult to arouse over the last few hours. She
took 1.5 ounces of formula 3 hours ago. She is hypotonic and has a blood pressure of 70/40 mm
Hg, a temperature of 37.2°C, a heart rate of 180 beats/min, and a respiratory rate of 80
breaths/min. Laboratory data are shown:

Laboratory Test Result


White blood cell count 7,000/µL (7.0 × 109/L)
Ammonia 210 µg/dL (150 µmol/L)
Glucose 40 mg/dL (2.2 mmol/L)
Creatinine 0.2 mg/dL (18 µmol/L)
Blood urea nitrogen 15 mg/dL (5.4 mmol/L)
Carbon dioxide 12 mEq/L (12 mmol/L)
Anion gap 24 mEq/L (24 mmol/L)
Lactate 41 mg/dL (4.6 mmol/L)

Results of a urinalysis are positive for ketones.

Of the following, the MOST likely diagnosis is


A. fatty acid oxidation disorder
B. organic acid disorder
C. phenylketonuria
D. urea cycle defect

American Academy of Pediatrics 789


PREP ® Self-Assessment PREPSA 2021
Correct Answer: B
Laboratory data for the neonate in the vignette show hyperammonemia with anion gap acidosis;
in the presence of urine ketones, an organic acid disorder should be suspected. Organic
acidemias are caused by disordered catabolism of certain amino acids, which leads to
accumulation of organic acids. An approach to metabolic diagnosis of a neonate with decreased
feeding and somnolence is based on clues seen in laboratory data (Item C217).

Item C217: Differentiation of conditions associated with neonatal hyperammonemia.


Adapted and reprinted with permission from Burton BK. Inborn errors of metabolism in infancy:
a guide to diagnosis. Pediatrics. 1998;102(6):E69.

Common organic acid disorders include isovaleric aciduria, methylmalonic aciduria, propionic
aciduria, and maple syrup urine disease. These are all autosomal recessive in their inheritance
pattern. Clinical presentation of this group of conditions can take one of three forms:
• severe neonatal onset, characterized by acute metabolic decompensation and
encephalopathy
American Academy of Pediatrics 790
PREP ® Self-Assessment PREPSA 2021
• acute intermittent late onset, characterized by recurrent episodes of metabolic
decompensation
• chronic progressive, characterized by failure to thrive and developmental delay

Diagnosis of the specific type of organic acidemia is made via metabolic abnormalities seen in
findings of plasma amino acid, urine organic acid, and acylcarnitine profile tests. Confirmation is
provided first by enzymatic analysis and then by gene analysis of the suspected disorder.
Intervention during acute metabolic decompensation is aimed at eliminating intake of additional
protein, reducing further catabolism by administering high-rate intravenous glucose and, if
needed, intravenous fat emulsions. In addition, the clinician should consider carnitine
supplementation, which binds toxic metabolites, and hemodialysis for removal of ammonia and
other toxic metabolites. Finally, the clinician must consider restarting protein via either gradual
oral intake or total parenteral nutrition after 24 hours. The goal of restarting protein is to prevent
the body’s own muscle breakdown to obtain essential amino acids.

Newborn screening via dried blood spot on filter paper obtained from heel prick is an important
tool to detect the majority of organic acidemias. The primary target is identification of the severe
neonatal forms of the disorders available on the newborn screening. Milder and intermittent
forms can be missed owing to the absence of metabolite in the collected blood spot. Hence,
appropriate evaluation should be carried out for patients who are suspected to have an inborn
error of metabolism, even in the presence of a normal newborn screening result.

Fatty acid oxidation disorder would not show urine ketones in the setting of acute metabolic
decompensation with metabolic acidosis. Common fatty acid oxidation disorders include
medium-chain acyl-CoA dehydrogenase deficiency, very-long-chain acyl-CoA dehydrogenase
deficiency, carnitine palmitoyltransferase I and II deficiency, and carnitine transporter defects.
Newborn screening can identify the severe forms of the above conditions.

Phenylketonuria does not present with an acute metabolic decompensation in neonates. Affected
infants are healthy at birth. If untreated, patients develop acquired microcephaly, developmental
delay, psychiatric issues, and intellectual disability. Identification of phenylketonuria is one of
the biggest success stories attributable to newborn screening. The biochemical marker for
phenylketonuria on newborn screening is elevated phenylalanine, low tyrosine, and a high
phenylalanine:tyrosine ratio. Management includes a diet low in phenylalanine with
supplementation of tyrosine for the life of the patient.

Acute decompensation in urea cycle defect would present with respiratory alkalosis and not
metabolic acidosis in the setting of hyperammonemia. Common urea cycle defects include
ornithine transcarbamylase deficiency, citrullinemia, and argininosuccinic aciduria. Ornithine
transcarbamylase is an X-linked disorder of urea cycle defect that is not identified on newborn
screening. Plasma amino acid, urine organic acid, and urine orotic acid is used to distinguish
between these conditions. Long-term management includes a low-protein diet with
supplementation of essential amino acids and arginine (except in arginase deficiency), as well as
use of nitrogen scavenger medications such as sodium phenylacetate and sodium
phenylbutyrate.

American Academy of Pediatrics 791


PREP ® Self-Assessment PREPSA 2021
PREP Pearls
• Acute neonatal presentation of inborn errors of metabolism can be distinguished on the
basis of laboratory data.
• Hyperammonemia with metabolic acidosis and urine ketones is seen in organic
acidemias.

ABP Content Specifications(s)


• Plan the appropriate immediate and long-term management of organic acidemias, while
considering the long-term prognosis
• Recognize the clinical features associated with organic acidemias

Suggested Readings
• Burton BK. Inborn errors of metabolism in infancy: a guide to diagnosis. Pediatrics.
1998;102(6):E69. doi:10.1542/peds.102.6.e69.
• Rice GM, Steiner RD. Inborn errors of metabolism (metabolic disorders). Pediatr Rev.
2016;37(1):3-15. doi:10.1542/pir.2014-0122.
• Rios A, Adams DJ. Specific congenital metabolic diseases. In: McInerny TK, Adam HM,
Campbell DE, DeWitt TG, Foy JM, Kamat DM, eds. American Academy of Pediatrics
Textbook of Pediatric Care. 2nd ed. Elk Grove Village, IL: American Academy of
Pediatrics; 2017:938-962. Pediatric Care Online .

American Academy of Pediatrics 792


PREP ® Self-Assessment PREPSA 2021
Question 218
A 2-year-old girl is having a health supervision visit. She has been healthy and is on track with
her development. Her parents are getting a divorce. They are planning on living in the same
neighborhood so that they can spend as much time as possible with their daughter. They would
like to know how their daughter might respond to the upcoming transition in their lives.

Of the following, the MOST likely response is


A. apparent indifference
B. blaming herself for the situation
C. loss of motor skills
D. separation anxiety

American Academy of Pediatrics 793


PREP ® Self-Assessment PREPSA 2021
Correct Answer: D
Toddlers commonly exhibit separation anxiety as a response to the divorce of their parents.
Although toddlers and preschoolers may regress behaviorally and in certain recently attained
skills (eg, toileting independently), regression in motor function is not likely. Older preschoolers
and younger school-aged children may blame themselves for the dissolution of their parents’
marriage and some adolescents may feign indifference. These behavioral responses are not
developmentally expected in a 2-year-old child.

Divorce affects 50% of marriages and more than 1 million children per year. Children’s
responses to the multiple changes in their lives (eg, family structure, living circumstances,
routines) vary according to their developmental age. Infants may be fussier and cry more.
Toddlers can be clingier and fearful of separation. Preschoolers may have separation anxiety as
well as tantrums, be defiant, and regress in their behaviors. Infants, toddlers, and preschoolers
can have disrupted sleep and feeding patterns. School-age children may respond with anger and
have problems with school performance and peer relationships; they may have somatic
complaints as well. Adolescents may withdraw, appear indifferent, or act out. They may struggle
with depression or anxiety, have suicidal ideation, and use substances.

Children adjust best to divorce when custody arrangements allow them to maintain a meaningful
relationship with each parent and to have stability in schedules and caregivers. Although joint
physical custody is common, it is beneficial to keep structure, routine, and behavioral
expectations as consistent as possible. For younger children, this is particularly helpful for
addressing fears of abandonment and fostering secure attachment. Parents can best support their
children whatever the custodial arrangement by coparenting cooperatively, communicating
effectively, and treating each other and their children respectfully.

Long-term effects of divorce include an impact on the child’s subsequent intimate relationships.
Adolescents whose parents have divorced are more likely to be romantically and sexually active.
They date more and have sex earlier than peers whose parents are not divorced. As adults, they
have more relationship problems (eg, intimacy, commitment, divorce).

PREP Pearls
• Toddlers commonly exhibit separation anxiety as a response to the divorce of their
parents.
• Children adjust best to divorce when custody arrangements allow them to maintain a
meaningful relationship with each parent and to have stability in schedules and
caregivers.
• Adolescents whose parents have divorced are more likely to be romantically and sexually
active and to have more relationship problems (eg, intimacy, commitment, divorce) as
adults.

MOCA-Peds Objective
• Provide guidance regarding family stressors (eg, birth of a sibling, death, divorce, job
loss, relocation).

American Academy of Pediatrics 794


PREP ® Self-Assessment PREPSA 2021
ABP Content Specifications(s)
• Understand the effects of divorce on a patient’s subsequent intimate relationships
• Understand the response to divorce and/or blended families in patients of various ages
• Understand the custodial issues associated with divorce and the effect of those issues on
patients of various ages

Suggested Readings
• Acholonu RG, Legano L, Weitzman ML. Children of divorce. In: McInerny TK, Adam
HM, Campbell DE, DeWitt TG, Foy JM, Kamat DM, eds. American Academy of
Pediatrics Textbook of Pediatric Care. 2nd ed. Itasca, IL: American Academy of
Pediatrics; 2016;chap 71:601-604. Pediatric Care Online.
• Cohen GJ, Weitzman CC; Committee on Psychosocial Aspects of Child and Family
Health; Section on Developmental and Behavioral Pediatrics. Helping children and
families deal with divorce and separation. Pediatrics. 2016;138(6):1-9.
doi: 10.1542/peds.2016-3020.
• Kaplan-Sanoff. Divorce. In: Augustyn M, Zuckerman B, Caronna EB, eds. The
Zuckerman Parker Handbook of Developmental and Behavioral Pediatrics for Primary
Care. 3rd ed. Philadelphia, PA: Lippincott Williams & Wilkins; 2011:446-449.
• Kleinsorge C, Covitz LM. Impact of divorce on children: developmental
considerations. Pediatr Rev. 2012;33(4):147-155. doi: 10.1542/pir.33-4-147.

American Academy of Pediatrics 795


PREP ® Self-Assessment PREPSA 2021
Question 219
A normally developing, 10-month-old infant who has no significant medical history is being seen
in the emergency department for fussiness. When her mother picked her up from the crib at nap
time, she was crying because her left leg was stuck between the rails. She has not been able to
console her since. The girl is small for her age and has macrocephaly. She has a triangular face,
sclera with a blue hue, and a pectus excavatum. She is crying and has tenderness over the left
thigh, which is slightly swollen. Radiography confirms a fractured femur.

Of the following, the condition MOST consistent with this girl’s findings is
A. child abuse
B. hypophosphatasia
C. Menkes disease
D. osteogenesis imperfecta

American Academy of Pediatrics 796


PREP ® Self-Assessment PREPSA 2021
Correct Answer: D
The physical features of the infant described in the vignette are most consistent with osteogenesis
imperfecta (OI). Child abuse should be considered when evaluating fractures in children with OI,
but in this case the mechanism of injury fits the clinical picture. Hypophosphatasia can be
associated with fractures in infants, but this child’s facial features and bluish sclera are not
commonly seen with that condition. Menkes disease is associated with loss of developmental
milestones, seizures, and hypotonia.

Osteogenesis imperfecta results from a genetic mutation in COL1A1 or COL1A2, which leads to
abnormalities in type 1 collagen, a structural protein in bones. This in turn results in bones that
are brittle, putting the child at increased risk of experiencing fractures. There are multiple types
of OI, with varying clinical severity. Fractures of the ribs and long bones can be seen as early as
at 15 to 20 weeks’ gestation on prenatal ultrasonography in some types of OI, whereas in other
types fractures present later in childhood. Fractures in infants can be difficult to diagnose, and
infants may exhibit extreme fussiness as the primary symptom.

Abnormal findings on the newborn physical examination include blue or gray sclera, a triangular
face, macrocephaly with a large fontanelle, and a pectus excavatum or carinatum. Hernias, both
umbilical and inguinal, are more common in this population. Ultrasonography is the preferred
method for monitoring the hip joints in infants with OI, because the Ortolani and Barlow
maneuvers can cause fractures. Severely affected infants are at risk of experiencing respiratory
compromise secondary to hypotonia and a small chest cavity.

During each health supervision visit, a careful cardiac examination is important, with special
consideration given to mitral valve prolapse and aortic valve dysfunction. Anticipatory guidance
should include discussion of delayed tooth eruption and often dark-colored teeth, a risk of
experiencing hearing loss, an increased risk of developing pneumonia, and the need for
modification of activities owing to the risk of fractures.

PREP Pearls
• Osteogenesis imperfecta results from a genetic mutation in COL1A1 and COL1A2, which
in turn leads to abnormalities in type 1 collagen, a structural protein in bone.
• Abnormal physical examination findings for a newborn with osteogenesis imperfecta
include blue or gray sclera, a triangular face, macrocephaly with a large fontanelle, and a
pectus excavatum or carinatum.
• Ultrasonography is the preferred method for monitoring the hip joints in infants with
osteogenesis imperfecta, because the Ortolani and Barlow maneuvers can cause fractures.

ABP Content Specifications(s)


• Recognize the clinical findings associated with osteogenesis imperfecta

American Academy of Pediatrics 797


PREP ® Self-Assessment PREPSA 2021
Suggested Readings
• Clunie G. Prenatal diagnosis: osteogenesis imperfecta. In: McInerny TK, Adam HM,
Campbell DE, DeWitt TG, Foy JM, Kamat DM, eds. American Academy of Pediatrics
Textbook of Pediatric Care. 2nd ed. Elk Grove Village, IL: American Academy of
Pediatrics; 2017:699-709. Pediatric Care Online .
• Pereira E. Achondroplasia Pediatr Rev. 2019;40(6):316-318. doi: 10.1542/pir.2018-0009.
• Starr SR, Roberts TT, Fischer PR. Osteogenesis imperfecta: primary care. Pediatr
Rev. 2010;31(8):e54-e64. doi:10.1542/pir.31-8-e54.
• US National Library of Medicine. Osteogenesis imperfecta. Genetics Home
Reference. https://ghr.nlm.nih.gov/condition/osteogenesis-imperfecta.

American Academy of Pediatrics 798


PREP ® Self-Assessment PREPSA 2021
Question 220
A 5-year-old boy with fever and rash is seen in the emergency department. Onset of fever was 4
days ago and associated with a runny nose, cough, and red eyes. One day before presentation, he
developed a rash that started on the forehead and behind his ears and spread to the trunk and then
the extremities. There are no known sick contacts, no pets at home, and no recent travel. He is
unimmunized.

The boy is irritable and has a temperature of 40°C, a heart rate of 126 beats/min, and a
respiratory rate of 22 breaths/min. His blood pressure is normal. Physical examination findings
are notable for nonpurulent conjunctivitis, pharyngeal erythema, and a generalized
maculopapular rash over the head, trunk, and extremities. The remainder of the physical
examination findings are unremarkable.

Of the following, in addition to standard precautions, the MOST appropriate isolation


precautions for this child are
A. airborne
B. contact
C. contact and droplet
D. droplet

American Academy of Pediatrics 799


PREP ® Self-Assessment PREPSA 2021
Correct Answer: A
The history and physical examination of the unimmunized boy described in the vignette with a
febrile illness and rash is consistent with the diagnosis of measles. In hospital or ambulatory
settings, patients with suspected measles should be immediately isolated and placed on airborne
transmission precautions in addition to standard precautions. Immunocompetent patients with
measles must be isolated until 4 days after rash onset, whereas infected patients who are
immunocompromised should be isolated for the duration of illness. Droplet and contact
precautions are recommended for hospitalized patients with influenza, adenovirus, rhinovirus,
and coronavirus for the duration of their illness. Contact precautions are recommended for
hospitalized patients with parainfluenza virus and respiratory syncytial virus for the duration of
their illness.

Measles is a highly communicable acute viral illness. It begins with a prodromal stage of 2 to 4
days that is characterized by fever and at least 1 of the “3 Cs” (ie, cough, coryza, conjunctivitis)
(Item C220A). This is followed by an erythematous maculopapular rash appearing 2 to 4 days
after the onset of fever. The rash begins on the face and head and then spreads to the trunk and
extremities (Item C220B). Approximately 1 to 2 days before the onset of the rash, a
pathognomonic enanthem (ie, Koplik spots) may appear as small bluish-white plaques on the
buccal mucosa in up to 70% of cases and may persist for an additional 1 to 2 days after rash
onset. In uncomplicated cases, complete recovery occurs within a week after rash onset.
Complications include otitis media (7%-9% of patients), pneumonia (1%-6%), diarrhea (8%),
postinfectious encephalitis (~1 per 1,000), and death (~1 per 1,000). Pregnant women without
evidence of measles immunity, neonates, infants younger than 12 months of age, and severely
immunocompromised patients are at risk for developing severe disease and complications from
measles.

American Academy of Pediatrics 800


PREP ® Self-Assessment PREPSA 2021

Item C220A: Conjunctivitis.


Reprinted with permission from Kimberlin DW, et al, eds. Red Book Online. Itasca, IL:
American Academy of Pediatrics; 2018.

American Academy of Pediatrics 801


PREP ® Self-Assessment PREPSA 2021

Item C220B: Maculopapular rash.


Reprinted with permission from Kimberlin DW, et al, eds. Red Book Online. Itasca, IL:
American Academy of Pediatrics; 2018.

Although measles was eliminated in the United States in the year 2000, there were small
outbreaks between 2000 and 2018, and in 2019, a resurgence with a total of 1,276 cases of
measles were confirmed in 31 states. The 2019 cases were related to outbreaks involving close-
knit, underimmunized communities linked to imported cases from outside the United States.
Vaccine refusal and misinformation about the safety of the MMR vaccine have contributed to the
outbreaks.

Measles virus is transmitted from person to person mainly by the airborne route via aerosolized
droplets or direct contact with infected nasal or pharyngeal secretions. The measles virus is
highly infectious. Patients with measles are contagious from 4 days before to 4 days after the
onset of the rash.

The standard of care for the prevention of measles is receipt of two doses of measles-containing
vaccine. The Centers for Disease Control and Prevention has published guidance on the
administration of postexposure prophylaxis (PEP) to prevent measles in exposed susceptible
contacts. Postexposure prophylaxis with MMR vaccine (administered within 72 hours of measles
American Academy of Pediatrics 802
PREP ® Self-Assessment PREPSA 2021
exposure) or immune globulin (Ig) (administered within 6 days of exposure) to susceptible
contacts can confer protection or modify the clinical course of illness.

For infants aged 6 months or younger who are exposed to measles, PEP with intramuscular Ig is
recommended. Intramuscular Ig must be administered as soon as possible after exposure but may
be given within 6 days of exposure. For exposed infants aged 6 to 11 months, MMR vaccine can
be given in place of intramuscular Ig and would offer protection if given within 72 hours of
exposure. Infants receiving MMR vaccine before their first birthday should be revaccinated at 12
to 15 months of age and receive a third dose at least 28 days later, usually at 4 to 6 years of age.

The MMR vaccine as PEP must be considered in all exposed, vaccine-eligible contacts who are
unvaccinated or have received only one MMR dose. In community-wide measles outbreaks
involving households, schools, and day care centers where the transmission risk is very high,
PEP with MMR vaccine is recommended for exposed infants aged 12 months and older.

Individuals who receive MMR vaccine or Ig as PEP should be monitored closely for symptoms
of measles for at least one incubation period (through 21 days after exposure).

Clinicians must report every case of suspected measles to the health department. The continued
threat of imported measles cases, coupled with large outbreaks in communities with low vaccine
coverage, warrants the urgent need for pediatricians to address vaccine hesitancy and effectively
communicate the safety and public health benefit of vaccines. The clinical diagnosis of measles
may be confirmed by either reverse-transcriptase polymerase chain reaction of nasopharyngeal
swab or by measles-specific IgM from serum samples. Treatment of measles is supportive.
Vitamin A is recommended for hospitalized children with measles in the United States.

PREP Pearls
• Measles is a highly communicable acute viral illness that begins with a prodromal stage
characterized by fever and at least one of the “3 Cs” (ie, cough, coryza, conjunctivitis),
followed by an erythematous maculopapular rash.
• The diagnosis of measles must be suspected in individuals with a febrile rash illness iho
are unvaccinated, have recently traveled to another country, or reside in a community
with low vaccine uptake.
• In hospital or ambulatory settings, patients with suspected measles should be immediately
isolated and placed on airborne transmission precautions.

ABP Content Specifications(s)


• Plan appropriate use of intramuscular immune globulin in immunocompromised and
unimmunized patients who have been exposed to measles
• Plan appropriate control measures to prevent the spread of measles
• Recognize the clinical features associated with measles, including complications

American Academy of Pediatrics 803


PREP ® Self-Assessment PREPSA 2021
Suggested Readings
• American Academy of Pediatrics. Measles. In: Kimberlin DW, Brady MT, Jackson MA,
Long SS, eds. Red Book: 2018-2021. Report of the Committee on Infectious
Diseases. 31st ed. Elk Grove Village, IL: American Academy of Pediatrics; 2015:537-
550.
• Gastañaduy PA, Budd J, Fisher N, et al. A measles outbreak in an underimmunized
Amish community in Ohio. N Engl J Med. 2016;375(14):1343-1354.
doi:10.1056/NEJMoa1602295.
• McLean HQ, Fiebelkorn AP, Temte JL, Wallace GS; Centers for Disease Control and
Prevention. Prevention of measles, rubella, congenital rubella syndrome, and mumps,
2013 summary: recommendations of the Advisory Committee on Immunization Practices
(ACIP) (published correction appears in MMWR Recomm Rep. 2015;64[9]:259).
MMWR Recomm Rep. 2013;62(RR-04):1-
34. https://www.cdc.gov/mmwr/pdf/rr/rr6204.pdf.
• Phadke VK, Bednarczyk RA, Salmon DA, Omer SB. Association between vaccine
refusal and vaccine-preventable diseases in the United States: a review of measles and
pertussis. JAMA. 2016;315(11):1149-1158. doi:10.1001/jama.2016.1353.
• Strebel PM, Orenstein WA. Measles. N Engl J Med. 2019;381(4):349-357.
doi:10.1056/NEJMcp1905181.

American Academy of Pediatrics 804


PREP ® Self-Assessment PREPSA 2021
Question 221
A 35-week-gestation neonate is being evaluated in the nursery. She was born via cesarean
delivery because of severe maternal preeclampsia and breech presentation. Her mother has a
history of gestational diabetes and a negative group B Streptococcus screening result. In the
delivery room, the neonate required positive pressure ventilation for 30 seconds because of
decreased respiratory effort. She voided in the delivery room. In the nursery, her vitals signs
include a temperature of 36.5°C, a heart rate of 156 beats/min, a respiratory rate of 45
breaths/min, a blood pressure of 52/22 mm Hg, with a mean arterial blood pressure of 32 mm
Hg, and oxygen saturation on pulse oximetry of 95% in room air. Physical examination reveals a
comfortable neonate with normal work of breathing and +2 peripheral pulses.

Of the following, the BEST next management step for this neonate is to
A. administer normal saline bolus
B. continue routine care
C. draw blood culture and start antibiotics
D. start prostaglandin infusion

American Academy of Pediatrics 805


PREP ® Self-Assessment PREPSA 2021
Correct Answer: B
The optimal blood pressure for a neonate is difficult to define. The tools used to measure blood
pressure may be imprecise, even though most neonatal blood pressures are obtained with the
proper-sized blood pressure cuff in a calm, quiet neonate. Arterial blood pressures are more
accurate with less variability, but are not routinely available. Mean arterial blood pressure should
approximate gestational age in the first week after birth, as in the case of the neonate in the
vignette. Providers must consider clinical factors such as perfusion, end-organ perfusion, and
clinical stability before deciding whether to treat a low blood pressure. A neonate with low blood
pressure and evidence of end-organ hypoperfusion should receive volume resuscitation. If
hypotension persists, vasopressor or corticosteroid therapy may be required. The neonate in the
vignette does not have evidence of hypotension with end-organ hypoperfusion so a normal saline
bolus is not indicated.

In the neonatal period, episodes of hypotension with acute kidney injury (AKI) may contribute to
the risk of chronic kidney disease. Acute kidney injury is defined as an increase in serum
creatinine or a decrease in urine output. Risk factors for AKI include a birthweight of less than
1,500 g, a history of therapeutic hypothermia, a need for extracorporeal membrane oxygenation
support, and congenital heart disease requiring surgical intervention. Providers should consider
blood pressure screening for children with a history of neonatal AKI.

The neonate in the vignette does not have clinical signs suggestive of an infection, so a blood
culture and antibiotics are not indicated. There is no indication that the neonate has a cardiac
condition so she does not require a prostaglandin infusion.

PREP Pearls
• Mean arterial blood pressure should approximate gestational age in the first week after
birth.
• Clinical factors such as perfusion, end-organ perfusion, and clinical stability must be
accounted for when considering treatment for hypotension.

ABP Content Specifications(s)


• Understand the normal variations in blood pressure that are associated with gestational
age

Suggested Readings
• Enlow E, Greenberg JM. Clinical manifestations of diseases in the newborn period. In:
Kliegman RM, St Geme JW, Blum NJ, Shah SS, Tasker RC, Wilson, KM. Nelson
Textbook of Pediatrics. 21st ed. Philadelphia, PA: Elsevier; 2019:910-913.e4.
• Selewski DT, Charlton JR, Jetton JG, et al. Neonatal acute kidney injury. Pediatrics.
2015;136;e463. doi: 10.1542/peds.2014-3819.
• Yager P, Noviski N. Shock. Pediatr Rev. 2010;31(8):311-319. doi: 10.1542/pir.31-8-311.

American Academy of Pediatrics 806


PREP ® Self-Assessment PREPSA 2021
Question 222
A 17-year-old adolescent girl is being evaluated for leg weakness. Over the last week, she has
developed clumsiness and tripping that progressed to an inability to walk. She reports difficulty
urinating and constipation. She has no history of trauma and denies the use of birth control pills
or other medications. She notes that she had an episode of “blurry vision” which self-resolved
about 3 months ago.

Neurologic examination is notable for significant weakness of the lower extremities with absent
deep tendon reflexes in the legs and lack of sensation to pinprick, vibration, and temperature. Her
rectal tone is diminished. She has normal strength and sensation in the upper extremities.
Magnetic resonance imaging (MRI) of the spine reveals a longitudinal extensive transverse
myelitis. Brain MRI findings are normal. Cerebrospinal fluid analysis shows a lymphocytic
pleocytosis with elevated protein and normal glucose. Ophthalmologic examination reveals optic
atrophy of the right fundus. She is admitted to the pediatric intensive care unit and emergently
treated with high-dose methylprednisolone.

Of the following, the MOST likely etiology of her disorder is


A. autoimmune
B. postinfectious
C. trauma
D. vascular

American Academy of Pediatrics 807


PREP ® Self-Assessment PREPSA 2021
Correct Answer: A
The clinical presentation and spine magnetic resonance imaging (MRI) findings for the
adolescent in the vignette are consistent with transverse myelitis (TM), an autoimmune
inflammatory condition. Transverse myelitis can occur either as an isolated, often postinfectious,
condition or as part of the continuum of neuroinflammatory demyelinating diseases (eg, multiple
sclerosis [MS], acute disseminated encephalomyelitis [ADEM], or neuromyelitis optica spectrum
disorders [NMOSD]). The clinical history of vision disturbances suggests a prior episode of optic
neuritis. This makes a postinfectious etiology less likely and warrants further testing to evaluate
for a demyelinating autoimmune disease.

Demyelinating neuroinflammatory conditions often involve the spine and can acutely present
with TM. After initial diagnostic testing and management have been initiated, further testing
should be tailored to the clinical context and suspicion for the presence of a chronic
demyelinating disease. Brain MRI should be performed to evaluate for further areas of
demyelination, the pattern of which can support a diagnosis of MS, NMOSD, or ADEM
depending on the location, appearance, and chronicity of the lesions. Ophthalmologic
examination may reveal pallor suggestive of a prior episode of optic neuritis, as seen in this
patient with self-resolving vision changes preceding the development of TM. Cerebrospinal fluid
(CSF) specimens can be tested for nonspecific markers of inflammation and demyelination.
Serum and CSF antibdies specific to certain conditions should also be sent during the diagnostic
evaluation.

Neuromyelitis optica spectrum disorders (Devic disease) is a rare heterogenous


neuroinflammatory condition with high risk for neurologic morbidity. Typically, patients
develop extensive longitudinal TM and optic neuritis that can lead to permanent paralysis and
vision loss. Antibodies to the aquaporin-4 channel are elevated. There are several NMOSD
syndromes, which may include involvement of the brainstem, diencephalon, or area postrema.
Acute treatment with methylprednisolone, plasma exchange, or intravenous immunoglobulin is
often followed by initiation of prophylactic immunomodulating treatment to prevent relapses.
In addition to autoimmune or postinfectious conditions, a diverse range of etiologies can lead to
acute myelopathy. Vascular etiologies (eg, acute spinal cord infarction) often present with rapid
onset of symptoms; an MRI will reveal changes in a vascular or watershed distribution.
Traumatic myelopathies are suspected with a history of trauma, and imaging may reveal
compressive lesions necessitating emergent neurosurgical evaluation. The clinical presentation
and findings of the CSF and MRI of the girl in the vignette are more compatible with an
autoimmune process.

PREP Pearls
• The evaluation of suspected transverse myelitis should include diagnostic testing to
determine if it is part of an autoimmune neuroinflammatory condition. This evaluation
includes serum and cerebrospinal fluid studies, imaging, and ophthalmologic
examinations tailored to the individual clinical context.
• Acute myelopathy has a diverse range of etiologies including postinfectious,
autoimmune/inflammatory, trauma, vascular, and infectious.

American Academy of Pediatrics 808


PREP ® Self-Assessment PREPSA 2021
MOCA-Peds Objective
• Recognize the child with spinal cord compression from a mass.

ABP Content Specifications(s)


• Understand the various etiologies of spinal cord disease

Suggested Readings
• Absoud M, Greenburg B, Lim M, Lotze T, Thomas T, Deiva K. Pediatric transverse
myelitis. Neurology. 2016;87(9 suppl 2):S46-S52.
doi: 10.1212/WNL.0000000000002820.
• Chitnis T, Ness J, Krupp L, et al. Clinical features of neuromyelitis optica in children: US
Network of Pediatric MS Centers report. Neurology. 2016;86(3):245-252.
doi: 10.1212/WNL.0000000000002283.
• Wingerchuk DM, Banwell B, Bennett JL, et al. International consensus diagnostic criteria
for neuromyelitis optica spectrum disorders. Neurology. 2015;85(2):177-189.
doi: 10.1212/WNL.0000000000001729.

American Academy of Pediatrics 809


PREP ® Self-Assessment PREPSA 2021
Question 223
A 10 year old has had pneumonia at least once each year since the age of 2 years. She usually
develops sudden onset of fever, cough, malaise, chest pain, and difficulty breathing when she
becomes ill. Chest radiographs have shown variable locations of segmental or lobar
consolidation. She recovers after treatment with amoxicillin; her most recent follow-up chest
radiograph is shown (Item Q223). She also has had recurrent otitis media leading to
tympanostomy tube placement. Her growth parameters are normal, and other than scarred
tympanic membranes and mucoid nasal discharge, her other upper airway findings are
unremarkable. Breath sounds are clear and heart sounds are normal, heard best to the right of the
sternum. The liver edge is palpable at the left costal margin.

American Academy of Pediatrics 810


PREP ® Self-Assessment PREPSA 2021

Item Q223: Chest radiograph of the child in the vignette. Courtesy M Guill

American Academy of Pediatrics 811


PREP ® Self-Assessment PREPSA 2021
Of the following, the MOST likely pathophysiologic mechanism for this child’s pneumonia
episodes is
A. functional deficiency of immunoglobulin G
B. impaired mucociliary clearance
C. inadequate neutrophil oxidative burst
D. T-cell immunodeficiency T-cell immunodeficiency

American Academy of Pediatrics 812


PREP ® Self-Assessment PREPSA 2021
Correct Answer: B
The girl in the vignette most likely has primary ciliary dyskinesia (PCD), as suggested by her
history of recurrent respiratory infections and situs inversus noted on chest and abdomen
radiography. The hallmark of PCD is recurrent sinusitis, otitis media, bronchitis, and pneumonia
caused by impaired clearance of pathogens from the airways by abnormal respiratory cilia.
Immunodeficiency is another cause of recurrent pneumonia. Pneumonia in children with humoral
immunodeficiencies generally responds well to appropriate treatment with antibiotics, but recurs
frequently. Both quantitative and functional deficiencies of immunoglobulin are associated with
signs and symptoms similar to PCD, but in light of the radiograph showing situs inversus (Item
C223), PCD is a more likely cause of this girl’s illnesses.

American Academy of Pediatrics 813


PREP ® Self-Assessment PREPSA 2021

Item C223: Chest radiograph showing dextrocardia and situs inversus.


Courtesy M Guill

American Academy of Pediatrics 814


PREP ® Self-Assessment PREPSA 2021
Other forms of immunodeficiency, such as chronic granulomatous disease (characterized by
inadequate neutrophil oxidative burst) or a T-cell dysfunction, are very rare and less likely to
respond quickly to treatment with narrow-spectrum antibiotics. Children with chronic
granulomatous disease often have mucocutaneous candidiasis, mesenchymal lymphadenitis, deep
granuloma formation, and infections with Staphylococcus aureus, Escherichia
coli, Burkholderia, and other gram-negative organisms. Children with T-cell immunodeficiencies
characteristically have chronic failure to thrive and persistent infections with viral, fungal, and
unusual bacterial organisms.

Other causes of recurrent pneumonia include anatomic anomalies of the airway and/or
pulmonary parenchyma, poor airway clearance because of ineffective cough from neuromuscular
disease, retained airway foreign body, or other chronic illness. In young children with asthma,
exacerbations related to viral infections are often associated with fever and abnormal chest
examination findings that are frequently misinterpreted as pneumonia. When this scenario occurs
with each viral illness, the child may be mistakenly labeled as having recurrent pneumonia.

PREP Pearls
• Children with ciliary dyskinesia characteristically experience recurrent sinopulmonary
infections because of poor airway mucociliary clearance.
• Recurrent pneumonia may be caused by anatomic defects, immunodeficiencies, or
systemic illnesses.
• Pneumonia is a common complication of neuromuscular disease because of ineffective
cough with poor airway clearance.

ABP Content Specifications(s)


• Formulate a differential diagnosis of recurrent pneumonia
• Recognize the significance of pneumonia in a child with a neuromuscular disease, and
manage appropriately

Suggested Readings
• Bonilla FA, Khan DA, Ballas ZK, et al. Practice parameters for the diagnosis and
management of primary immunodeficiency. J Allergy Clin Immunol. 2015;136(5):1186-
1205.e1-78. doi: 10.1016/j.jaci.2015.04.049.
• Hernandez-Trujillo VP. Approach to children with recurrent infections. Immunol Allergy
Clin North Am. 2015;35(4):625-636. doi: 10.1016/j.iac.2015.07.005.
• Knowles MR, LA Daniels, SD Davis, et al. Primary ciliary dyskinesia: recent advances in
diagnostics, genetics, characterization. Am J Resp Crit Care Med. 2013;188(8):913-922.
doi: 10.1164/rccm.201301-0059CI.

American Academy of Pediatrics 815


PREP ® Self-Assessment PREPSA 2021
Question 224
A 6-year-old boy who is receiving maintenance chemotherapy for acute lymphocytic leukemia is
brought to the emergency department after developing a burning rash over his face 2 days ago.
His medications include monthly prednisone and vincristine pulse, daily oral 6-mercaptopurine,
and weekly oral methotrexate. His vital signs are normal for his age. He has erythematous
vesicular lesions on his upper lip, his right eyelid, and the right side of his forehead. The rest of
the physical examination findings are unremarkable. He is diagnosed with herpes zoster infection
and admitted for administration of intravenous acyclovir. His blood urea nitrogen level is 14
mg/dL (5.0 mmol/L), and his creatinine level is 0.5 mg/dL (44 µmol/L).

Of the following, the MOST appropriate management step to prevent drug-induced


nephrotoxicity in this child is
A. alkalinization of urine
B. extended-interval drug dosing
C. isotonic fluid hydration
D. serum drug level monitoring

American Academy of Pediatrics 816


PREP ® Self-Assessment PREPSA 2021
Correct Answer: C
The boy in this vignette, who is receiving maintenance therapy for acute lymphocytic leukemia
with herpes zoster infection, is at risk of experiencing acyclovir-induced nephrotoxicity due to
precipitation of crystals in the renal tubules. This is best prevented by intravenous isotonic
hydration.

Drugs are a common cause of acute kidney injury. The common pathologic mechanisms of drug-
induced nephrotoxicity are alteration of intraglomerular hemodynamics, renal tubular cell
toxicity, acute and chronic interstitial nephritis, crystal nephropathy, and thrombotic
microangiopathy. Risk factors that predispose a patient to nephrotoxicity include decreased
effective intravascular volume, use of multiple nephrotoxic medications, and underlying renal
insufficiency.

General measures to prevent drug-induced nephrotoxicity include using alternate non-


nephrotoxic drugs, correcting the risk factors, assessing baseline renal function, adjusting the
dose of the drug according to renal function, and avoiding nephrotoxic drug combinations.
Adequate hydration is important to maintain renal perfusion, and intravascular volume should be
corrected before nephrotoxic medication therapy begins. Item C224 presents specific measures
to prevent the nephrotoxicity caused by different classes of drugs.

American Academy of Pediatrics 817


PREP ® Self-Assessment PREPSA 2021

American Academy of Pediatrics 818


PREP ® Self-Assessment PREPSA 2021
The boy in this vignette is receiving multiple chemotherapy medications, including methotrexate,
which can cause nephrotoxicity when used in a high dose through the intravenous route. The
child in this vignette is receiving oral methotrexate once weekly and has normal baseline renal
function, so the drug is less likely to cause nephrotoxicity. Nephrotoxicity associated with high-
dose methotrexate is prevented via isotonic fluid hydration, alkalinization of urine, and
measurement of drug levels. Alkalinization of urine is useful in preventing contrast-induced
nephropathy. Extended-interval dosing can be used to prevent nephrotoxicity related to use of
aminoglycoside and vancomycin. Monitoring serum level of the drug is useful in preventing
nephrotoxicity due to tacrolimus, cyclosporine, aminoglycoside, and vancomycin.

PREP Pearls
• Risk factors for drug-induced nephrotoxicity are decreased effective intravascular
volume, use of multiple nephrotoxic medications, and underlying renal insufficiency.
• The clinician should assess baseline renal function and adjust medication doses according
to renal function to prevent nephrotoxicity.
• Adequate hydration is important before initiating therapy with nephrotoxic drugs.
• Intravenous acyclovir in high doses can induce nephrotoxicity by precipitating crystals in
renal tubules. Adequate hydration is important to prevent the development of
nephrotoxicity.

ABP Content Specifications(s)


• Recognize the drug classes that can cause renal toxicity

Suggested Readings
• Faught LN, Greff MJ, Rieder MJ, Koren G. Drug-induced acute kidney injury in
children. Br J Clin Pharmacol. 2015;80(4):901-909. doi:10.1111/bcp.12554.
• Naughton CA. Drug-induced nephrotoxicity. Am Fam Physician. 2008;78(6):743-
750. www.aafp.org/afp/2008/0915/p743.html.

American Academy of Pediatrics 819


PREP ® Self-Assessment PREPSA 2021
Question 225
A healthy, 15-month-old boy who was born at full term is seen for a health supervision visit. His
parents report that he can accurately point to a body part, retrieve an object when asked, and
point to a picture. He is being raised in a bilingual household and is able to say a total of 5
meaningful words besides “mama” and “baba.” They read books with him daily, engage him in
active conversations, and do not expose him to electronic media. His parents mention that their
friends’ daughter, who is the same age as their son but raised in a single-language household, can
speak 10 words. The boy’s parents ask what else they can do to optimize their son’s language
development.

Of the following, the BEST advice to give to the boy’s parents is to


A. contact early intervention services
B. continue their current parenting practices
C. introduce educational electronic media
D. speak to him in a single language only

American Academy of Pediatrics 820


PREP ® Self-Assessment PREPSA 2021
Correct Answer: B
The parents in the vignette should continue their current parenting practices because the total
number of words the boy is able to speak is age-appropriate. Educational electronic media may
actually impair his language development and should be avoided until at least 18 months of age.
He does not need to be referred to early intervention because his level of development is age
appropriate. His parents may continue to speak to him in 2 languages because it will not interfere
with his language development.

Early language development is intimately linked with subsequent cognitive outcomes.


Identifying language disorders in a timely fashion is therefore crucial to preventing future
difficulties with socialization, behavior, and academics. Item C225 lists normal receptive and
expressive language developmental milestones for toddlers aged 14 to 16 months.
Toddlers who do not point to objects of interest, say “mama” or “papa/dada/baba,” or say 6
words by the age of 18 months should be immediately referred for an audiology examination,
further evaluation for coexisting conditions, and early intervention services.

American Academy of Pediatrics 821


PREP ® Self-Assessment PREPSA 2021

Risk factors for language or speech delay include prematurity, lower socioeconomic status, and a
family history of speech or language delay. Gender, being raised in a bilingual household, birth
order, and chronic otitis media with effusion are not risk factors.
Children raised in bilingual households may actually have enhanced problem-solving skills and
greater cognitive flexibility compared with those raised in single language households. Those
who do have speech or language delay will exhibit the delay in both languages. Therefore, it is
important to count the total number of words spoken in all languages, not just in a single
language.

At 15 months of age, toddlers should also be able to climb on furniture, crawl up stairs, pick up a
toy from a standing position, start to use a spoon, turn pages in a book, and show empathy.
Delays in both language and motor skills should prompt evaluation for a global developmental
delay as well as referral for occupational, physical, and speech therapies.
American Academy of Pediatrics 822
PREP ® Self-Assessment PREPSA 2021
PREP Pearls
• A bilingual home or environment is not a risk factor for communication delays.
• Children who exhibit language and speech delays should be referred for an audiology
examination, early intervention services, and further evaluation for coexisting conditions.
• For multilingual children, it is important to count the total number of words spoken in all
languages.

ABP Content Specifications(s)


• Evaluate the cognitive and behavioral developmental progress/status of a child at 15
months of age
• Evaluate the motor developmental progress/status of a child at 15 months of age

Suggested Readings
• Feldman HM. How young children learn language and speech. Pediatr Rev.
2019;40(8):398-410. doi:10.1542/pir.2017-0325.
• Rosenberg M, Trashes N. Speech and language concerns. In: McInerny TK, Adam HM,
Campbell DE, DeWitt TG, Foy JM, Kamat DM, eds. American Academy of Pediatrics
Textbook of Pediatric Care. 2nd ed. Itasca, IL: American Academy of Pediatrics;
2016;chap 195:1607-1611. Pediatric Care Online.
• Scharf RJ, Scharf GJ, Stroustrup A. Developmental milestones. Pediatr
Rev. 2016;37(1):25-38. doi:10.1542/pir.2014-0103.

American Academy of Pediatrics 823


PREP ® Self-Assessment PREPSA 2021
Question 226
A 31-month-old boy is seen in the office for a growth check. He was born at term. The delivery
was complicated by excessive hemorrhage. He has been healthy and takes a daily multivitamin.
A comprehensive review of systems is unremarkable. His adjusted midparental height is at the
40th percentile. His growth charts are shown in Item Q226A and Item Q226B. His vital signs
are normal for age, and his physical examination findings are unremarkable.

American Academy of Pediatrics 824


PREP ® Self-Assessment PREPSA 2021

American Academy of Pediatrics 825


PREP ® Self-Assessment PREPSA 2021

Laboratory data are shown:

American Academy of Pediatrics 826


PREP ® Self-Assessment PREPSA 2021
Laboratory Test Result
White blood cell count 8,300/μL (8.3 × 109/L)
Hemoglobin 13 g/dL (130 g/L)
Platelet count 281 × 103/μL (281 × 109/L)
Sodium 138 mEq/L (138 mmol/L)
Potassium 4.1 mEq/L (4.1 mmol/L)
Bicarbonate 25 mEq/L (25 mmol/L)
Creatinine 0.3 mg/dL (26.5 µmol/L)
Glucose 90 mg/dL (5.0 mmol/L)
Thyroid-stimulating
1.4 mIU/L (reference range, 0.27-4.2 mIU/L)
hormone
1.3 ng/dL (17 pmol/L) (reference range, 0.9-1.7 ng/dL [12-22
Free thyroxine
pmol/L])

Urinalysis findings are normal except for a specific gravity of 1.020 and a pH of 6.

Of the following, the MOST likely diagnosis is


A. celiac disease
B. growth hormone deficiency
C. insufficient caloric intake
D. renal tubular acidosis

American Academy of Pediatrics 827


PREP ® Self-Assessment PREPSA 2021
Correct Answer: B
The boy in the vignette has growth hormone deficiency. His length curve shows declining
percentiles after about 12 months of age with preservation of his weight-for-length percentile.
Children with congenital growth hormone deficiency are typically of normal size at birth. Linear
growth deceleration occurs after the age of 6 to 12 months, when growth hormone becomes
important for linear growth. Weight is not affected as much as length, so weight-for-length or
body mass index percentile is preserved. There may also be evidence of altered body
composition with increased fat mass and decreased lean body mass. The history of excessive
hemorrhage at the time of delivery is a risk factor for growth hormone deficiency resulting from
a vascular injury to the pituitary gland. Of the pituitary hormones, growth hormone is the most
sensitive to pituitary insults.

Other pituitary hormone deficiencies can be associated with congenital growth hormone
deficiency. Hypoglycemia and prolonged jaundice may occur with isolated growth hormone
deficiency but are more common with multiple pituitary hormone deficiencies. The presence of
nystagmus and midline defects suggests septo-optic dysplasia. Males can have a small penis,
cryptorchidism, or both, especially if concomitant gonadotropin deficiency is present. Brain
magnetic resonance imaging may reveal pituitary gland or stalk abnormalities, optic nerve
hypoplasia, or agenesis of the septum pellucidum/corpus callosum.

Acquired growth hormone deficiency presents with linear growth deceleration after a period of
normal growth. Relative weight gain with increased fat mass is common. Acquired growth
hormone deficiency may be secondary to a brain tumor, cranial irradiation, or head trauma.
Other pituitary hormone deficiencies may be associated. Levels of insulin-like growth hormone
factor-1 and insulin-like growth factor binding protein 3 are usually low, and growth hormone
levels after stimulation remain low. Bone age is delayed. Patients with growth hormone
deficiency should undergo magnetic resonance imaging of the brain and sella turcica and testing
for other pituitary hormone deficiencies. Children with growth hormone deficiency grow very
well on relatively low doses of growth hormone therapy.

Celiac disease, insufficient caloric intake, and renal tubular acidosis are less likely causes of
growth failure for the boy in the vignette, given the preservation of his weight-for-length
percentile and the lack of supporting findings on laboratory evaluation. Anemia may be present
with celiac disease or insufficient caloric intake. Metabolic acidosis and electrolyte abnormalities
are usually present with renal tubular acidosis. Although not a response choice, hypothyroidism
can cause linear growth failure; however, it would not be the cause of this boy’s condition, given
his normal thyroid function test results.

PREP Pearls
• In those with congenital growth hormone deficiency, linear growth becomes abnormal at
about the age of 6 to 12 months.
• Patients with acquired growth hormone deficiency should undergo investigation for a
brain neoplasm and other pituitary hormone deficiencies.
• The weight-for-length or body mass index curve is important to consider in narrowing the
differential diagnosis of abnormal growth.

American Academy of Pediatrics 828


PREP ® Self-Assessment PREPSA 2021
MOCA-Peds Objective
• Recognize and plan initial evaluation of a child with a pituitary disorder.

ABP Content Specifications(s)


• Recognize the clinical features associated with acquired and congenital growth hormone
deficiency
• Recognize the effects of growth hormone therapy on growth

Suggested Readings
• Braun LR, Marino R. Disorders of growth and stature. Pediatr Rev. 2017;38(7):293-304.
doi:10.1542/pir.2016-0178.
• Kaplowitz P. Short stature. In: McInerny TK, Adam HM, Campbell DE, DeWitt TG, Foy
JM, Kamat DM, eds. American Academy of Pediatrics Textbook of Pediatric Care. 2nd
ed. Elk Grove Village, IL: American Academy of Pediatrics; 2017:1585-1588. Pediatric
Care Online.
• Rogol AD, Hayden GF. Etiologies and early diagnosis of short stature and growth failure
in children and adolescents. J Pediatr. 2014;164(5 Suppl):S1-14.e6.
doi:10.1016/j.jpeds.2014.02.027.
• Rose SR, Vogiatzi MG, Copeland KC. A general pediatric approach to evaluating a short
child. Pediatr Rev. 2005;26(11):410-20. doi:10.1542/pir.26-11-410.
• Suresh S, Santhanam I. Hypoglycemia. In: McInerny TK, Adam HM, Campbell DE,
DeWitt TG, Foy JM, Kamat DM, eds. American Academy of Pediatrics Textbook of
Pediatric Care. 2nd ed. Elk Grove Village, IL: American Academy of Pediatrics;
2017:2881-2887. Pediatric Care Online.

American Academy of Pediatrics 829


PREP ® Self-Assessment PREPSA 2021
Question 227
A previously healthy, 13-year-old adolescent boy with an abrupt onset of emesis, abdominal
cramps, and watery diarrhea is seen in the emergency department. One day ago, he visited a local
park with his younger siblings. The public park has a picnic ground, lake, paddle boats, and a
splash pad. The patient’s siblings who swam at the lake with him are also ill with a similar
gastrointestinal illness.

He has a temperature of 38.1°C, dry mucous membranes, and sunken eyes. Capillary refill time
is 3 seconds. The rest of the examination findings are unremarkable. Laboratory evaluation
reveals a serum sodium concentration of 128 mEq/L (128 mmol/L).

Of the following, the MOST likely cause of this illness is


A. Campylobacter
B. Cryptosporidium
C. norovirus
D. Salmonella

American Academy of Pediatrics 830


PREP ® Self-Assessment PREPSA 2021
Correct Answer: C
The adolescent boy in this vignette developed an abrupt onset of gastroenteritis (AGE) following
a visit to a public lake with his siblings. A similar illness is affecting other family members. This
scenario is highly suggestive of norovirus infection, given the symptom profile and short
incubation period for norovirus (12-48 hours). Untreated recreational water–associated outbreaks
may be caused by enteric pathogens, toxins, or chemicals present in lakes. Enteric pathogens
associated with untreated recreational water–associated outbreaks include norovirus, Escherichia
coli, Shigella, Cryptosporidium, and others. In one report from the US Centers for Disease
Control and Prevention, a total of 140 recreational water-associated disease outbreaks were
reported from 2010 to 2014, resulting in 4,958 illnesses and 2 deaths; enteric pathogens were
implicated in 80 of these outbreaks. Norovirus was the most frequent cause, with 21 outbreaks,
followed by E coli, Shigella, and Cryptosporidium. Transmission occurs via ingestion of fecally
contaminated water. Sources of fecal contamination include swimmers, storm water runoff,
sewage overflows or treatment plant discharges, or animal waste. Human and animal feces are a
common source of E coli and Cryptosporidium, whereas norovirus and Shigella infection are
suggestive of human fecal contamination.

Since the introduction of rotavirus vaccine in the United States, human norovirus (previously
known as Norwalk virus) is now the leading cause of both sporadic and epidemic AGE in
children younger than 5 years. With the increased availability of sensitive molecular detection
methods such as the real-time reverse transcriptase polymerase chain reaction assay, norovirus
has also been increasingly identified as a principal cause of foodborne illness and foodborne
disease outbreaks in the United States. Norovirus has been implicated in health care–acquired
infections, travel-associated diarrhea, and outbreaks with high attack rates in diverse settings
such as long-term care facilities, day care centers, schools, dormitories, military facilities, and
cruise ships.

Norovirus is a nonenveloped, single-stranded ribonucleic acid virus of the family Caliciviridae.


Immunity from norovirus infection is not permanent, and reinfections can occur throughout life.
Infection with one norovirus serotype may not necessarily confer cross-protection against other
serotypes.

Noroviruses are highly contagious and easily transmitted through the fecal-oral or vomitus-oral
routes, via consumption of contaminated water or food products, or by direct contact with
contaminated objects or surfaces. The virus is remarkably stable in the environment and can
survive on surfaces for prolonged periods. Other factors that promote the transmissibility of
norovirus include prolonged viral shedding (up to 3 weeks or more) by infected patients, high
virus burden, and the small inoculum dose (< 100 viral particles) needed to cause disease.
Norovirus has been detected in stool of healthy subjects, especially children.

Norovirus illness often begins with the abrupt onset of vomiting associated with watery diarrhea,
abdominal cramps, and nausea. Children may have diarrhea without vomiting. Other systemic
symptoms may include fever, malaise, myalgia, loss of appetite, and headache. The usual
duration of illness ranges from 1 to 5 days. Gastroenteritis caused by norovirus can be severe in
infants, the elderly, and immunocompromised hosts with a prolonged hospital course and
mortality.
American Academy of Pediatrics 831
PREP ® Self-Assessment PREPSA 2021
The management of norovirus AGE is supportive and aimed at the treatment of dehydration with
oral or intravenous rehydration solutions and maintenance of fluid and electrolyte balance.
Hospitalized patients with suspected AGE caused by norovirus must be placed on contact
precautions until 48 hours after resolution of the illness. Strict adherence to hand hygiene
practices remains the mainstay of norovirus infection prevention and control. Use of soap and
running water for a minimum of 20 seconds is recommended after contact with a patient with
suspected or confirmed norovirus gastroenteritis.

Outbreaks caused by enteric pathogens in treated recreational water (eg, pools) are very rare.
Most untreated recreational water-associated outbreaks are linked to smaller, inland lakes,
reservoirs, and ponds. Prevention of such outbreaks warrants public engagement and education
regarding healthy swimming messages, especially during the summer months
(https://www.cdc.gov/healthywater/swimming/swimmers/steps-healthy-swimming.html).
These measures include adherence to posted advisories for beach closure to swimming;
avoidance of swimming in discolored, smelly, foamy, or scummy water; avoidance of swimming
during diarrheal illness; and limiting water entry to nose when swimming in warm fresh water to
prevent infection with Naegleria fowleri.

In contrast to the onset time in the boy in the vignette, the typical incubation period
of Cryptosporidiosis is 3 to 14 days. Cryptosporidiosis is usually characterized by nonbloody
watery diarrhea with associated symptoms of abdominal pain, emesis, fever, loss of appetite, and
weight loss. Campylobacter is a leading bacterial cause of foodborne gastroenteritis in children,
often causing bloody diarrhea, but is a very unusual cause of recreational water-associated
outbreaks; its infection rate is lower than that of infections caused by norovirus, Shigella species
and Cryptosporidium. Nontyphoidal Salmonella species can cause diarrheal illness in children
after ingestion of contaminated water or food, but has not been associated with untreated
recreational water-associated outbreaks in the United States.

PREP Pearls
• Enteric pathogens associated with untreated recreational water-associated outbreaks
include norovirus, Escherichia coli, Shigella, Cryptosporidium, and other enteric
pathogens.
• Noroviruses are highly contagious and easily transmitted through fecal-oral or vomitus-
oral routes, via consumption of contaminated water or food, or by direct contact with
contaminated objects or surfaces.
• Norovirus illness often begins with the abrupt onset of vomiting associated with watery
diarrhea, abdominal cramps, and nausea.

ABP Content Specifications(s)


• Recognize the clinical features associated with calicivirus (norovirus and sapovirus)
infection
• Understand the epidemiology of human calicivirus (norovirus and sapovirus) infection

American Academy of Pediatrics 832


PREP ® Self-Assessment PREPSA 2021
Suggested Readings
• American Academy of Pediatrics. Norovirus and sapovirus infections. In: Kimberlin DW,
Brady MT, Jackson MA, Long SS, eds. Red Book: 2018-2021 Report of the Committee
on Infectious Diseases. 31st ed. Elk Grove Village, IL: American Academy of Pediatrics;
2018:577-580. Red Book Online.
• Graciaa DS, Cope JR, Roberts VA, et al. Outbreaks associated with untreated recreational
water—United States, 2000–2014. MMWR Morb Mortal Wkly Rep. 2018;67(25):701-
706. doi:10.15585/mmwr.mm6725a1.
• Payne DC, Vinje J, Szilagyi P, et al. Norovirus and medically attended gastroenteritis in
U.S. children. N Engl J Med. 2013;368(12):1121-1130. doi:10.1056/NEJMsa1206589.
• Shah MP, Wikswo ME, Barclay L, et al. Near real-time surveillance of U.S. norovirus
outbreaks by the Norovirus Sentinel Testing and Tracking Network—United States,
August 2009-July 2015. MMWR Morb Mortal Wkly Rep. 2017;66(7):185-189.
doi:10.15585/mmwr.mm6607a1.
• Zlot A, Simckes M, Vines J, et al; Centers for Disease Control and Prevention (CDC).
Norovirus outbreak associated with a natural lake used for recreation—Oregon, 2014
(published correction appears in MMWR Morb Mortal Wkly Rep.
2015;64[22];621). MMWR Morb Mortal Wkly Rep. 2015;64(18):485-490.

American Academy of Pediatrics 833


PREP ® Self-Assessment PREPSA 2021
Question 228
A 16-year-old girl is brought to the emergency department after having lost consciousness. She
is currently awake and alert. She describes suddenly feeling very hot and sweaty, nauseated, and
light-headed while standing at a crowded concert. A friend noticed that she did not seem to feel
well and was no longer responding when she then passed out and fell to the floor. Nearly
immediately on hitting the floor, she woke up. She reports no drug or alcohol use. She is
otherwise healthy, and her physical examination findings are normal. Her electrocardiogram
shows no abnormalities.

Of the following, the MOST likely diagnosis for this patient is


A. aortic stenosis
B. long QT syndrome
C. seizure disorder
D. vasovagal syncope

American Academy of Pediatrics 834


PREP ® Self-Assessment PREPSA 2021
Correct Answer: D
The patient in the vignette experienced syncope after experiencing a prodrome of light-
headedness and feeling warm, sweaty, and nauseated while in an upright position. This
presentation is consistent with vasovagal syncope, otherwise known as neurocardiogenic
syncope. Given the normal findings of both the physical examination and the electrocardiogram,
the patient does not have either aortic stenosis or long QT syndrome. Her history does not
indicate any evidence of a seizure.

Syncope can result from neurological, cardiac, metabolic, or psychiatric causes. Item
C228A provides a list of possible etiologies for syncope according to broad category. A
thorough history is the most important step in determining the etiology of someone with syncope
or presyncope. In the vast majority of pediatric patients with syncope, the condition will have a
benign cause. The history and physical examination can help to identify patients whose syncope
has a neurological or cardiac cause. Item C228B provides an algorithm for evaluation based on
history and physical examination findings. Patients whose syncope is experienced with exercise,
associated with chest pain, or associated with palpitations and those whose electrocardiographic
or cardiac examination findings are abnormal may have syncope with a cardiac etiology, and
they should be referred to a subspecialist. Item C228C provides a differential diagnosis for
syncope due to a cardiac etiology.

American Academy of Pediatrics 835


PREP ® Self-Assessment PREPSA 2021

American Academy of Pediatrics 836


PREP ® Self-Assessment PREPSA 2021

PREP Pearls
• Pediatric syncope is typically benign, and the etiology can be determined by means of a
thorough history and physical examination.
• Signs and symptoms that raise concern about a cardiac etiology of syncope include
syncope with exercise, syncope associated with chest pain or palpitations, or abnormal
findings on an electrocardiogram or a cardiac examination.

ABP Content Specifications(s)


• Plan the appropriate evaluation of a syncopal or pre-syncopal episode, including episodes
associated with exercise
• Recognize the cardiac causes of syncope

American Academy of Pediatrics 837


PREP ® Self-Assessment PREPSA 2021
Suggested Readings
• Anderson JB, Willis MW. Differentiating between postural tachycardia syndrome and
vasovagal syncope. Pediatrics. 2017;139(4):e20164161. doi:10.1542/peds.2016-4161.
• Cannon B, Wackel P. Syncope. Pediatr Rev. 2016;37(4):159-167. doi:10.1542/pir.2014-
0109.
• Ramaswamy P. Syncope. In: McInerny TK, Adam HM, Campbell DE, DeWitt TG, Foy
JM, Kamat DM, eds. American Academy of Pediatrics Textbook of Pediatric Care. 2nd
ed. Elk Grove Village, IL: American Academy of Pediatrics; 2017:1636-1640. Pediatric
Care Online .
• Willis J. Syncope. Pediatr Rev. 2000;21(6):201-203. doi:10.1542/pir.21-6-201.

American Academy of Pediatrics 838


PREP ® Self-Assessment PREPSA 2021
Question 229
A 2-year-old girl is being evaluated for a rash on her leg. Her father first noticed 2 spots a few
weeks ago; now she has several bumps on her left leg (Item Q229). They do not seem painful.
She is otherwise healthy.

Item Q229: Papules seen for the girl described in the vignette.
Courtesy of Mancini AJ, Krowchuk DP. Pediatric Dermatology: A Quick Reference Guide. 3rd
ed. Elk Grove Village, IL: American Academy of Pediatrics; 2016, p104.

Of the following, the girl’s MOST likely diagnosis is


A. cutaneous histoplasmosis
B. molluscum contagiosum
C. varicella
D. verruca vulgaris

American Academy of Pediatrics 839


PREP ® Self-Assessment PREPSA 2021
Correct Answer: B
The small, pearly, dome-shaped lesions seen on the girl’s leg are most consistent with
molloscum contagiosum. This painless lesion is commonly seen in otherwise healthy children
and can spread, especially if the child has an abnormal immune system or underlying eczema.
The lesions typically have a central punctum.

The rash is caused by a poxvirus resulting in the characteristic superficial papules. It is self-
limited, but often takes 6 to 12 months to resolve, with some cases lasting several years. The
lesions can become secondarily infected by skin bacteria, becoming red, swollen, and painful;
management of infected lesions may require topical or systemic antibiotics, depending on the
severity.

Inside the molluscum papules is a core of white material. If the lesions are scratched, children
can spread the rash (autoinoculation). If desired, the lesions can be treated with cryotherapy,
cantharidin (“beetle juice”), or curettage. This is usually recommended for children with
underlying eczema, lesions in the genital area, or compromised immune systems, including those
with human immunodeficiency virus.

Cutaneous histoplasmosis appears as papules, pustules, and nodules in the setting of systemic
histoplasmosis infection. Varicella (Item C229A) is more vesicular in appearance; the lesions
are at various stages, pruritic, and accompanied by systemic symptoms. Verruca vulgaris (Item
C229B) is known as the common wart; it does not usually present in clusters on the extremity as
is shown in the vignette.

Item C229A: Varicella vesicles.


Reprinted with permission from Murray DL, Vodzak J, Immergluck LC. Contagious
exanthematous diseases. In: McInerny TK, Adam HM, Campbell DE, DeWitt TG, Foy JM,
Kamat DM, eds. American Academy of Pediatrics Textbook of Pediatric Care. 2nd ed. Elk Grove
Village, IL: American Academy of Pediatrics; 2017.

American Academy of Pediatrics 840


PREP ® Self-Assessment PREPSA 2021

Item C229B: Common warts.


Reprinted with permission from Mancini AJ, Krowchuk DP. Pediatric Dermatology: A Quick
Reference Guide. 3rd ed. Elk Grove Village, IL: American Academy of Pediatrics; 2016.

PREP Pearls
• Molluscum contagiosum is a self-limited disease.
• To prevent the spread in immunocompromised patients or those with active eczema, early
treatment of molluscum contagiosum should be considered.
• Molluscum contagiosum lesions can become secondarily infected.

ABP Content Specifications(s)


• Recognize the clinical findings associated with molluscum contagiosum, and manage
appropriately

American Academy of Pediatrics 841


PREP ® Self-Assessment PREPSA 2021
Suggested Readings
• American Academy of Dermatology. Molluscum contagiosum: diagnosis and
treatment. https://www.aad.org/diseases/a-z/molluscum-contagiosum-treatment.
• Centers for Disease Control and Prevention. Molluscum
contagiosum. https://www.cdc.gov/poxvirus/molluscum-
contagiosum/clinical_information.html.
• Murray DL, Vodzak J, Immergluck LC. Contagious exanthematous diseases. In:
McInerny TK, Adam HM, Campbell DE, DeWitt TG, Foy JM, Kamat DM,
eds. American Academy of Pediatrics Textbook of Pediatric Care. 2nd ed. Elk Grove
Village, IL: American Academy of Pediatrics; 2017:1920-1926. Pediatric Care Online.

American Academy of Pediatrics 842


PREP ® Self-Assessment PREPSA 2021
Question 230
An 8-year-old, previously healthy boy is seen in the emergency department for symptoms that
began 2 days ago with mild sore throat. Yesterday he had worsening of throat pain and a low-
grade fever. He arrived from Venezuela 1 week ago. He is underimmunized because of
intermittent access to vaccines in his hometown. His vital signs are a temperature of 38.2°C,
heart rate of 84 beats/min, respiratory rate of 18 breaths/min, and blood pressure of 92/53 mm
Hg. The findings of the oropharyngeal examination are shown in Item Q230. The remainder of
his physical examination findings are normal.

Item Q230: Oropharyngeal examination findings for the boy described in the vignette.
Reprinted with permission from Kimberlin DW, et al, eds. Red Book Online. Itasca, IL:
American Academy of Pediatrics; 2018.

Laboratory data are as follows:


Laboratory Test Result
Group A Streptococcus rapid screen Negative
Throat culture Pending
Heterophile antibodies Negative

American Academy of Pediatrics 843


PREP ® Self-Assessment PREPSA 2021
Of the following, this clinical presentation is MOST concerning for infection with
A. Candida albicans
B. Corynebacterium diphtheriae
C. Epstein-Barr virus
D. Streptococcus pyogenes

American Academy of Pediatrics 844


PREP ® Self-Assessment PREPSA 2021
Correct Answer: B
The clinical presentation of the boy in the vignette is most concerning for infection
with Corynebacterium diphtheriae. He has adherent exudative pharyngitis and results of testing
for common pathogens (Streptococcus pyogenes and Epstein-Barr virus [EBV]) are negative.
This clinical picture, in the setting of underimmunization, raises concern for diphtheria
infection.

Corynebacterium diphtheriae can be acquired via 2 routes, inhalation or direct skin contact.
Although pharyngeal disease is the most common presentation, isolated infection of the nasal
passages or larynx can occur. Initial symptoms of respiratory tract infection include sore throat,
cervical lymphadenopathy, and low-grade fever. Respiratory diphtheria infection is caused by
toxigenic strains of the organism. The toxin can lead to formation of adherent membranes which,
in severe cases, can lead to upper airway obstruction. Affected children can have extensive neck
swelling with cervical lymphadenitis, referred to as a bull neck (Item C230A). The toxin can
have systemic effects, including involvement of the heart, nervous system, and kidneys.
Cutaneous diphtheria manifests as a chronic, nonhealing ulcer (Item C230B).

American Academy of Pediatrics 845


PREP ® Self-Assessment PREPSA 2021

Item C230A: Bull neck due to diphtheria.


Reprinted with permission from Reprinted with permission from Kimberlin DW, et al, eds. Red
Book Online. Itasca, IL: American Academy of Pediatrics; 2018

American Academy of Pediatrics 846


PREP ® Self-Assessment PREPSA 2021

Item C230B: Chronic ulcer of diphtheria.


Courtesy of the Centers for Disease Control and Prevention

Candida albicans should be considered when plaques are seen in the oropharynx. Typically,
these plaques are also present on the buccal mucosa, palate, and tongue. Oropharyngeal
candidiasis is common in young infants, but can also be seen in children being treated with
antibiotics or with primary or acquired cellular immune defects. Children with candidal
infections may be asymptomatic, have pain with swallowing, or experience loss of the sensation
of taste.

Epstein-Barr virus infection should be considered in the setting of exudative pharyngitis. In older
children, heterophile antibody test results will be positive in the majority of EBV infections.
When negative, alternative diagnoses should be considered. Heterophile antibody testing should
not be obtained for children younger than 4 years, because results are often negative in young
children even in the setting of acute EBV infection.

The adherent pharyngeal membranes shown in Item C230C distinguish diphtheria from the
typically thinner exudates of Streptococcus pyogenes (group A Streptococcus [GAS]). Rapid
antigen assays for GAS have high specificity, but sensitivity can vary. Therefore, it is
recommended that a throat culture be performed for children suspected of having GAS with
negative rapid antigen assay results.

American Academy of Pediatrics 847


PREP ® Self-Assessment PREPSA 2021

Item C230C: Oropharyngeal examination findings for the boy described in the vignette.
Reprinted with permission from Kimberlin DW, et al, eds. Red Book Online. Itasca, IL:
American Academy of Pediatrics; 2018

PREP Pearls
• Respiratory diphtheria infection is caused by toxigenic strains of the organism. The toxin
can lead to formation of adherent membranes which, in severe cases, can lead to upper
airway obstruction.
• Patients with respiratory diphtheria infection can have extensive neck swelling with
cervical lymphadenitis, referred to as a bull neck.
• Cutaneous diphtheria manifests as a chronic, nonhealing ulcer.

ABP Content Specifications(s)


• Recognize the clinical features associated with diphtheria

Suggested Readings
• American Academy of Pediatrics. Diphtheria. In: Kimberlin DW, Brady MT, Jackson
MA, Long SS, eds. Red Book: 2018-2021 Report of the Committee on Infectious
Diseases. Itasca, IL: American Academy of Pediatrics; 2018:319-323. Red Book Online .
• Steele RW. Pharyngitis and tonsillitis. In: McInerny TK, Adam HM, Campbell DE,
DeWitt TG, Foy JM, Kamat DM, eds. American Academy of Pediatrics Textbook of
Pediatric Care. 2nd ed. Itasca, IL: American Academy of Pediatrics; 2016;chap
311:2498-2501. Pediatric Care Online.
American Academy of Pediatrics 848
PREP ® Self-Assessment PREPSA 2021
Question 231
A 4-year-old boy is brought to the emergency department 1 hour after he was found playing with
an empty pill bottle. His grandmother reports that the bottle contained an unknown number of
various over-the-counter analgesics. The boy is asymptomatic. He has normal vital signs and
physical examination findings. His capillary blood glucose level is 84 mg/dL (4.6 mmol/L).
Electrocardiography shows normal sinus rhythm. An intravenous line is placed, a normal saline
bolus is started, and blood is drawn for laboratory testing. Because of concerns about possible
acetaminophen ingestion, a serum level is ordered to be sent at the appropriate interval after
ingestion.

Of the following, the MOST appropriate interval after ingestion for this testing is
A. 1 hour
B. 2 hours
C. 4 hours
D. 8 hours

American Academy of Pediatrics 849


PREP ® Self-Assessment PREPSA 2021
Correct Answer: C
Acetaminophen is a common analgesic/antipyretic that is readily available for purchase, often in
extremely large quantities that would be toxic if ingested at once. It is also a common component
of over-the-counter cough and cold medications, which makes an unintentional overdose a
distinct possibility if a parent is giving a child multiple medications for an upper respiratory tract
infection. A single dose of more than 150 mg/kg in a child or 7.5 g in an adult can cause
hepatotoxicity. To predict the risk of toxicity and guide management, the ideal time to obtain a
serum acetaminophen concentration is 4 hours after an acute ingestion.

Acetaminophen toxicity occurs because of oxidative liver injury from excessive levels of the
toxic metabolite N-acetyl-p-benzoquinoneimine (NAPQI). In therapeutic doses, acetaminophen is
metabolized by the hepatic enzymes UDP-glucuronosyl transferase and sulfotransferase;
however, in the case of an overdose, these metabolic pathways become overwhelmed, and
hepatic glutathione, which conjugates NAPQI into a nontoxic substance, is depleted. The result
is excess NAPQI and resultant oxidative injury.
There are four stages to acetaminophen toxicity (Item C231A).

American Academy of Pediatrics 850


PREP ® Self-Assessment PREPSA 2021

A serum acetaminophen level obtained 4 hours after ingestion can be plotted on the Rumack-
Matthew nomogram (Item C231B); time after ingestion is plotted on the X-axis and serum
acetaminophen concentration on the Y-axis. A single downward sloping line starts at the
intersection of 150 μg/mL and 4 hours. A 4-hour acetaminophen level that plots above or to the

American Academy of Pediatrics 851


PREP ® Self-Assessment PREPSA 2021
right of this line indicates high risk for toxicity and warrants treatment for acetaminophen
poisoning. The nomogram may display a line starting at the intersection of 200 μg/mL and 4
hours. This represents the original threshold level for treatment upon creation of the nomogram
in the 1970s. A third line with an intersection of 300 μg/mL and 4 hours represents the ‘high
risk’ line. A level plotted to the right of this line indicates massive acetaminophen overdose. The
presence of acetaminophen in a serum sample obtained before the 4-hour post-ingestion mark is
indicative of acetaminophen ingestion, but it cannot be used to accurately predict the risk of
toxicity. Levels obtained more than 4 hours after ingestion can be plotted on the nomogram and
managed accordingly; however, treatment is most effective if started as early as possible once a
potentially toxic level is identified.

Item C231B: Rumack-Matthew nomogram.


Reprinted with permission from Hodgman MJ, Garrard AR. A review of acetaminophen
poisoning. Crit Care Clin. 2012;28(4):499-516

As with the management of all emergent conditions, ensuring stability of the patient’s airway,
breathing, and circulation is paramount. If a patient presents within 2 hours of acetaminophen
ingestion, activated charcoal can be administered in an attempt to limit gastrointestinal
absorption, but its use should be avoided if mental status is altered. The antidote for
American Academy of Pediatrics 852
PREP ® Self-Assessment PREPSA 2021
acetaminophen poisoning is N-acetyl cysteine (NAC), which functions by facilitating
replenishment of hepatic glutathione, thus decreasing free NAPQI. It should be administered if
the acetaminophen level is above the treatment line on the Rumack-Matthew nomogram; if a
suspected toxic dose (>150 mg/kg or 7.5 g for patients ≥ 50 kg) has been ingested and a level
will not be available for more than 8 hours after ingestion; if a serum acetaminophen level is
more than 10 μg/mL and the time since ingestion is unknown; or if there is a known
acetaminophen ingestion and any evidence of liver injury. Higher doses of NAC may be
necessary for massive overdoses (i.e. a level to the right of the 300-line on the nomogram. N-
acetyl cysteine can be administered either intravenously or orally and is equally effective via
both routes. Intravenous NAC is associated with a risk of anaphylactoid reaction, particularly in
patients with a history of asthma.

PREP Pearls
• Acetaminophen is toxic at doses of 150 mg/kg in a child or 7.5 g for an adult.
• The ideal time to obtain a serum acetaminophen level to predict the risk of toxicity and
guide management is 4 hours after an acute ingestion.
• N-acetyl cysteine is the antidote for acetaminophen toxicity; it is equally effective when
given orally or intravenously.

ABP Content Specifications(s)


• Recognize the signs and symptoms of acetaminophen toxicity, and manage appropriately

Suggested Readings
• Fine JS. Poisoning. In: McInerny TK, Adam HM, Campbell DE, DeWitt TG, Foy JM,
Kamat DM, eds. American Academy of Pediatrics Textbook of Pediatric Care. 2nd ed.
Elk Grove Village, IL: American Academy of Pediatrics; 2017:2924-2949. Pediatric Care
Online.
• Fisher ES, Curry SC. Evaluation and treatment of acetaminophen toxicity Adv
Pharmacol. 2019;85:263-272. doi:10.1016/bs.apha.2018.12.004.
• Hendrickson RG, McKeown NJ. Acetaminophen. In: Hoffman RS, Nelson LS, Goldfrank
LR, Howland MA, Lewin NA, Smith SW, eds. Goldfrank’s Toxicologic
Emergencies. 11th ed. New York, NY: McGraw-Hill; 2019:472-491.
• Hodgman MJ, Garrard AR. A review of acetaminophen poisoning. Crit Care Clin.
2012;28(4):499-516. doi:10.1016/j.ccc.2012.07.006.
• Nadler A, Fein DM. Acetaminophen poisoning. Pediatr Rev. 2018;39(6):316-318.
doi:10.1542/pir.2017-0093.

American Academy of Pediatrics 853


PREP ® Self-Assessment PREPSA 2021
Question 232
A 15-year-old adolescent boy developed a spot on his chest 3 weeks ago. His mother thought
that he might have ringworm and applied an over-the-counter antifungal cream. However, the
lesion persisted, and within 1 to 2 weeks, many new lesions appeared on the trunk. The rash is
pruritic but the patient has been otherwise well. He has normal vital signs. Physical examination
findings are unremarkable aside from the skin. There is a widespread eruption that involves the
chest, abdomen, and back. The face, neck, and extremities are spared. The original lesion is
present on the right side of the upper chest (Item Q232).

Item Q232: Rash as described for the adolescent in the vignette. Courtesy of D Krowchuk

Of the following, the MOST appropriate treatment is


A. griseofulvin administered orally
B. hydroxyzine administered orally
C. selenium sulfide administered topically
D. terbinafine administered topically

American Academy of Pediatrics 854


PREP ® Self-Assessment PREPSA 2021
Correct Answer: B
The boy in the vignette developed a rash that began as a round patch mimicking tinea corporis.
On close inspection, however, the lesion lacks central clearing and border elevation. Within 2
weeks he developed numerous lesions on the trunk, many of which are papules. These findings
are consistent with a diagnosis of pityriasis rosea and, accordingly, symptomatic treatment with
hydroxyzine is appropriate. If tinea corporis was suspected, topical terbinafine (for one or a few
lesions) or oral griseofulvin (for large or multiple lesions) could be prescribed. Tinea versicolor
is characterized by scaling macules and patches (not papules) located on the trunk (Item
C232A); it may be treated with topical selenium sulfide.

Item C232A: Tinea versicolor.


Reprinted with permission from Mancini AJ, Krowchuk DP, eds. Pediatric Dermatology. A
Quick Reference Guide. 3rd ed. Elk Grove Village, IL: American Academy of Pediatrics; 2016.

Pityriasis rosea (PR) is a self-limited papulosquamous disorder (ie, lesions are elevated and have
scale). It may be infectious (human herpesvirus types 6 and 7 have been implicated) and usually
occurs in the spring and fall. In as many as 80% of patients, the eruption begins with a herald
patch. This lesion is round or oval, often has central clearing (although in this patient it did not),
and exhibits scaling (Item C232B). Within 2 weeks, a more widespread eruption develops that is
composed of erythematous papules and plaques. The long axes of plaques are oriented parallel to
lines of skin stress (Item C232C). On the back, this alignment of lesions may mimic the boughs
of a fir tree (ie, the “Christmas-tree” appearance) (Item C232D).

American Academy of Pediatrics 855


PREP ® Self-Assessment PREPSA 2021

Item C232B: The herald patch of pityriasis rosea is erythematous, round or oval, and has scale
(concentrated at the trailing, not leading edge, of the lesion).
Courtesy of D Krowchuk

American Academy of Pediatrics 856


PREP ® Self-Assessment PREPSA 2021

Item C232C: The classical lesions of pityriasis rosea are oval thin scaling plaques. The long
axes of plaques are oriented parallel to lines of skin stress. As with the herald patch, scale is
located at the trailing edge of lesions.
Courtesy of D Krowchuk

When examining individual lesions of PR, scale is observed at the trailing edge (ie, more
centrally) unlike in tinea corporis, in which it is located at the leading edge (ie, peripherally)
(Item C232C). In persons of color, the eruption may differ in two ways: it may have an “inverse
distribution” with lesions concentrated on the extremities and relative sparing of the trunk, or it
may be primarily papular with fewer plaques, as was the case for the boy in the vignette (see
also Item C232E). New lesions appear for 2 to 3 weeks, and the eruption resolves in 4 to 8
weeks.

American Academy of Pediatrics 857


PREP ® Self-Assessment PREPSA 2021

Item C232D: On the back, the alignment of pityriasis rosea lesions along lines of skin stress may
mimic the appearance of the boughs of a fir tree (ie,, the “Christmas-tree” appearance).
Reprinted with permission from Mancini AJ, Krowchuk DP, eds. Pediatric Dermatology. A
Quick Reference Guide. 3rd ed. Elk Grove Village, IL: American Academy of Pediatrics; 2016.

American Academy of Pediatrics 858


PREP ® Self-Assessment PREPSA 2021

Item C232E: A child who has pityriasis rosea. There are numerous small papules.
Reprinted with permission from Mancini AJ, Krowchuk DP, eds. Pediatric Dermatology. A
Quick Reference Guide. 3rd ed. Elk Grove Village, IL: American Academy of Pediatrics; 2016.

The diagnosis of PR is made clinically on the basis of the typical appearance and distribution of
the rash. In patients who have a predominantly papular presentation, clinicians should examine
the skin carefully for oval plaques, because finding these lesions helps secure the diagnosis.
Most individuals who have PR require no therapy. If pruritus is significant, a topical
corticosteroid or an emollient containing phenol or menthol (agents that act as counterirritants,
masking the perception of pruritus) may be applied as needed, or a sedating antihistamine may
be taken at bedtime. Judicious sun exposure may reduce pruritus and hasten the resolution of the
eruption. In view of the possible association of PR with human herpesvirus types 6 and 7
infection, acyclovir has been evaluated as a possible treatment. The results of a few small trials
suggest that acyclovir may offer some benefit by reducing erythema and limiting new lesion
formation in the early course of PR. However, not all trials have shown efficacy, and use of
acyclovir for this indication is neither approved by the US Food and Drug Administration nor
accepted as a mainstream therapy.

American Academy of Pediatrics 859


PREP ® Self-Assessment PREPSA 2021
PREP Pearls
• The lesions of pityriasis rosea are oval, scaling, thin plaques. The long axes of plaques
are oriented parallel to lines of skin stress. The scale is located on the trailing edge of
lesions rather than at the leading edge (border).
• On the back, the arrangement of pityriasis rosea lesions along lines of skin stress (and the
ribs) may mimic the appearance of the boughs of a fir tree (ie, the “Christmas-tree”
appearance).
• In persons of color, pityriasis rosea may have an “inverse” distribution (with lesions
concentrated on the extremities and relative sparing of the trunk); alternatively, lesions
may be papules with fewer plaques.

ABP Content Specifications(s)


• Recognize the clinical findings associated with pityriasis rosea, and manage appropriately

Suggested Readings
• Chang HC, Sung CW, Lin MH. The efficacy of oral acyclovir during early course of
pityriasis rosea: a systematic review and meta-analysis. J Dermatolog Treat.
2019;30(3):288-293. doi:10.1080/09546634.2018.1508820.
• Mancini AJ, Krowchuk DP, eds. Pediatric Dermatology. A Quick Reference Guide. 3rd
ed. Elk Grove Village, IL: American Academy of Pediatrics; 2016.

American Academy of Pediatrics 860


PREP ® Self-Assessment PREPSA 2021
Question 233
A newborn was born vaginally at term to a multiparous woman who received limited prenatal
care. The mother has a history of polysubstance abuse and was incarcerated in the middle of the
second trimester for a drug-related offense, but she reports that she has not used any drugs during
this pregnancy. The newborn remains in the mother’s custody. Child protective services are
determining the appropriate placement of the newborn after discharge. The mother’s obstetrician
ordered a urine drug screen when she arrived at the hospital, but the mother declined to provide a
sample. Child protective services have requested drug screening on the newborn, but it has not
yet been discussed with the mother.

Of the following, the MOST accurate statement about this screening is that
A. drug screening should be performed if indicated for clinical care, with the consent of the
parent or guardian
B. if the newborn’s urine drug screen is positive for opiates, she will need to be treated with
medication to prevent withdrawal
C. a negative newborn urine drug screen would confirm the accuracy of the mother’s history
D. results of this newborn’s meconium drug screening could reflect maternal exposure to
pain medications during labor

American Academy of Pediatrics 861


PREP ® Self-Assessment PREPSA 2021
Correct Answer: A
For the neonate in the vignette, the most accurate response is that drug screening should be
performed if indicated for clinical care, with the consent of the parent or guardian. Not all
newborns with positive opiate screens require medication to treat withdrawal. Because a urine
drug screen only reflects recent exposure, results cannot elucidate exposures earlier in
pregnancy. Although newborn urine drug screening can show metabolites of maternal pain
medications administered during labor, meconium drug screening reflects earlier exposures that
occurred while meconium was formed and deposited during the 2nd and 3rd trimester of
pregnancy.

Substance use and abuse during pregnancy is becoming increasingly common. There are
complex medical, ethical, and legal aspects to perinatal substance use, because it affects both the
mother and the fetus/neonate.

The American College of Obstetrics and Gynecology recommends universal substance use
screening with a validated tool early in pregnancy. Positive screens should lead to in-depth
patient interviews, laboratory testing if indicated, and if appropriate, referral to resources to assist
with psychosocial comorbidities (eg, trauma/violence exposure, psychiatric conditions,
homelessness, food insecurity). Laboratory testing (typically urine) should be performed after
informed consent is obtained. Reporting requirements differ by state. Pregnant women with
substance use disorders should be encouraged to enroll in treatment; medication-assisted
treatment with an opioid agonist (eg, methadone or buprenorphine) is recommended for pregnant
women with opioid use disorders.

Substance-exposed neonates should have a careful examination for signs of prematurity, fetal
growth restriction, congenital malformations, and/or congenital infection, and must be observed
for signs of withdrawal. Not all neonates born to women with substance use disorders need to
have drug screening performed. Screening may be useful if the maternal history is unclear or
inconsistent with clinical symptoms, or if the results would dictate a specific treatment course. In
some situations (eg, where a mother has entered medication-assisted treatment and is not using
illicit substances), neonatal drug screening may be requested by the parents to “prove” maternal
treatment adherence. Consent should be obtained from the neonate’s parent or legal guardian,
including a discussion of local reporting requirements. Neonatal drug screening can be
performed on several sample sources (Item C233).

American Academy of Pediatrics 862


PREP ® Self-Assessment PREPSA 2021

All drug screening tests (maternal and neonatal) have false-positive and false-negative rates;
positive screens require confirmatory testing.

American Academy of Pediatrics 863


PREP ® Self-Assessment PREPSA 2021
PREP Pearls
• Pregnant women should be screened for substance use and substance use disorder early in
pregnancy and referred for treatment as appropriate. Medication-assisted treatment with
an opioid agonist (eg, methadone or buprenorphine) is recommended for pregnant women
with opioid use disorders.
• Maternal and neonatal drug screening tests have false-positive and false-negative rates;
all positive screens must be followed with confirmatory testing.
• Results from neonatal urine drug screening reflect exposure in the preceding hours to
days; meconium and umbilical cord tissue screening reflect exposure during the 2nd and
3rd trimesters.
• Drug screening for neonates should be performed if indicated for clinical care, with the
consent of the parent or guardian.

ABP Content Specifications(s)


• Plan appropriate laboratory evaluation of substance use/abuse, including appropriate
collection of test specimens and interpretation of results
• Understand the limitations of drug-screening tests

Suggested Readings
• Committee on Obstetric Practice. Committee Opinion No 711: opioid use and opioid use
disorder in pregnancy. Obstet Gynecol. 2017;130(2):e81-e94.
doi: 10.1097/AOG.0000000000002235.
• Hudak ML, Tan RC; Committee on Drugs; Committee on Fetus and Newborn; American
Academy of Pediatrics. Neonatal drug withdrawal. Pediatrics. 2012:129(2):e540-e560.
doi: 10.1542/peds.2011-3212.
• Jansson LM, Velez ML. Infants of drug-dependent mothers. Pediatr Rev. 2011;32(1):5-
12. doi: 10.1542/pir.32-1-5.

American Academy of Pediatrics 864


PREP ® Self-Assessment PREPSA 2021
Question 234
A 7-year-old boy with a history of renal dysplasia and chronic kidney disease is seen for a health
supervision visit. He has a history of increased thirst and polyuria. His current medications
include sodium bicarbonate, ferrous sulfate, calcium carbonate, and calcitriol. His weight is at
the fifth percentile, and his height is at the third percentile. He has a heart rate of 89 beats/min, a
respiratory rate of 16 breaths/min, and a blood pressure of 90/60 mm Hg. The rest of the
examination findings are unremarkable.

Of the following, the MOST appropriate dietary restriction for this child is
A. phosphorus
B. protein
C. sodium
D. vitamin D

American Academy of Pediatrics 865


PREP ® Self-Assessment PREPSA 2021
Correct Answer: A
The boy in this vignette has chronic kidney disease (CKD), which can cause urinary phosphate
retention and hyperphosphatemia. He needs dietary restriction of phosphorus to maintain a serum
phosphorus level in the normal range and to control hyperparathyroidism.

In children with CKD, nutritional status should be evaluated on a periodic basis, because it
affects growth and neurocognitive development. Appropriate calorie and protein intake based on
estimated energy needs and dietary reference intake is recommended to achieve the expected
weight and height gain. Protein restriction is not recommended in infants and children with
CKD, because this has not been shown to improve kidney function and may impair growth.
Children with CKD should receive the recommended dietary reference intake of vitamins (B1,
B2, B6, B12, folic acid, A, C, E, and K) and minerals (copper and zinc).

Children with CKD are at risk of experiencing abnormalities in the metabolism of calcium,
phosphorus, and vitamin D and in parathyroid hormone levels. They also are at risk of
developing secondary hyperparathyroidism and mineral and bone disorders, previously termed
“renal osteodystrophy.” The goal of prevention of mineral and bone disorders in children with
CKD includes normalizing the serum phosphorus values via dietary restriction and phosphate
binders (calcium carbonate), controlling hyperparathyroidism with active vitamin D analogs
(calcitriol), and supplementing vitamin D to normalize 25-hydroxyvitamin D levels. The boy in
this vignette is receiving treatment with calcium carbonate and calcitriol and needs dietary
phosphorus restriction.

Sodium restriction is not indicated for the boy in the vignette, because he is normotensive. In
fact, this boy may need sodium supplementation, because children with renal dysplasia tend to
have increased urinary sodium losses. Dietary restriction of protein and vitamin D are not
recommended for the boy in the vignette. Children with CKD may also need a potassium-
restricted diet or a formula low in potassium content to prevent hyperkalemia.

PREP Pearls
• Dietary restriction of phosphorus in chronic kidney disease controls hyperphosphatemia
and secondary hyperparathyroidism.
• Protein restriction is not recommended in infants and children who have chronic kidney
disease.

ABP Content Specifications(s)


• Recognize the nutritional deficiencies associated with renal disease
• Plan the dietary management of renal insufficiency in patients of various ages

Suggested Readings
• Massengill SF, Ferris M. Chronic kidney disease in children and adolescents. Pediatr
Rev. 2014;35(1):16-29. doi:10.1542/pir.35-1-16.
• Sevilla WMA. Nutritional considerations in pediatric chronic disease. Pediatr
Rev. 2017;38(8):343-352. doi:10.1542/pir.2016-0030.

American Academy of Pediatrics 866


PREP ® Self-Assessment PREPSA 2021
Question 235
A 9-year-old girl is being evaluated for concerns about learning. She was recently diagnosed via
psychoeducational evaluation as having dyslexia and attention-deficit/hyperactivity disorder,
predominantly inattentive presentation. She is a polite girl who is well-liked by her peers and
teacher. She tries but struggles with staying on task and with reading fluency and
comprehension. Her parents are not interested in medication and have hired a tutor to work with
their daughter after school. Her parents would like to know how to best support their daughter.

Of the following, the BEST recommendation is


A. a behavioral intervention plan
B. a student success team meeting
C. sensory integration therapy
D. specialized academic instruction

American Academy of Pediatrics 867


PREP ® Self-Assessment PREPSA 2021
Correct Answer: D
The girl in the vignette should qualify for an Individualized Education Program (IEP) under the
special education eligibility category of “specific learning disability” and could possibly qualify
under “other health impairment” for attention-deficit/hyperactivity disorder (ADHD). Special
education services, particularly specialized academic instruction, are indicated to address her
difficulties with reading fluency and comprehension.

Children with developmental or learning problems should be connected to their state’s early
intervention services if they are younger than 3 years and to their local school district for special
education services if older than 3 years. Parents do not need a referral; they can directly ask their
state early intervention program or school district to evaluate their child for eligibility for
services. Children who qualify for assistance can then access needed developmental or
educational therapies and supports. Both early intervention programs and special education
programs are regulated by the Individuals with Disabilities Education Act.

An IEP is developed for children who qualify for special education. The IEP can provide
services and supports such as specialized academic instruction, speech/language therapy,
occupational therapy, and adaptive physical education. The services and supports are
individualized and based on the specific student’s strengths, weaknesses, and educational needs.

Section 504 of the Rehabilitation Act of 1973 is an important law that protects the civil rights of
children with physical or mental disabilities by providing these children with equal access to
education in institutions that receive federal funds. This is accomplished through a 504 plan
which can provide accommodations, such as changes in 1) the way material is presented (eg,
visual, auditory), 2) the manner in which the child completes assignments (eg, handwritten,
typed), 3) the setting in which the child is tested (eg, classroom, separate room), and 4) the time
allowed for assignments or tests (eg, extended time). Children with conditions such as ADHD
can qualify for a 504 plan.

A student success (or study) team meeting is convened with parents and school staff to discuss
concerns about a student’s difficulty in school and to design a plan to address these concerns in
the general education environment. However, children struggling with school failure, especially
those with or suspected to have diagnoses that have a significant impact on learning, are likely to
need special education services under an IEP. A behavioral intervention plan (BIP) can be
incorporated into an IEP to address severe problematic behaviors (eg, elopement, aggression,
behavioral outbursts) that interfere with learning. A BIP is not needed for the girl in the vignette
because she does not have disruptive behaviors. Sensory integration therapy is typically provided
by occupational therapists to address difficulties in handling sensory inputs (eg, auditory, tactile,
vestibular). The evidence to recommend sensory integration therapy for learning problems is
insufficient.

American Academy of Pediatrics 868


PREP ® Self-Assessment PREPSA 2021
PREP Pearls
• An Individualized Education Program (IEP) can provide for services and supports such as
specialized academic instruction, speech/language therapy, occupational therapy, and
adaptive physical education. A behavioral intervention plan can be incorporated into an
IEP to address severe problematic behaviors (eg, elopement, aggression, behavioral
outbursts) that interfere with learning.
• Section 504 of the Rehabilitation Act of 1973 is a law that protects the civil rights of
children with physical or mental disabilities by providing these children with equal access
to education in institutions that receive federal funds.
• A student success (or study) team meeting is convened with parents and school staff to
discuss concerns about a student’s difficulty in school and to design a plan to address
these concerns in the general education environment.

ABP Content Specifications(s)


• Identify the types of community services available to families of children who have
learning and behavioral problems
• Understand strategies to improve the self-esteem of children who have learning
disabilities

Suggested Readings
• Frankowski BL. Learning difficulty. In: McInerny TK, Adam HM, Campbell DE, DeWitt
TG, Foy JM, Kamat DM, eds. American Academy of Pediatrics Textbook of Pediatric
Care. 2nd ed. Itasca, IL: American Academy of Pediatrics; 2016;chap 172:1484-
1489. Pediatric Care Online.
• Godwin DL, Vinson SS. Social and community services for children with developmental
disabilities and/or behavioral disorders and their families. In: Voigt RG, Macias MM,
Myers SM, Tapia CD, eds. Developmental and Behavioral Pediatrics. 2nd ed. Itasca, IL:
American Academy of Pediatrics; 2018:591-610.
• Kral MC. Interpreting psychoeducational testing reports, Individualized Family Service
Plans (IFSP), and Individualized Education Program (IEP) plans. In: Augustyn M,
Zuckerman B, eds. Zuckerman Parker Handbook of Developmental and Behavioral
Pediatrics for Primary Care. 4th ed. Philadelphia, PA: Wolters Kluwer; 2019:477-493.
• Lipkin PH, Okamoto J; Council on Children with Disabilities and Council on School
Health. The Individuals with Disabilities Education Act (IDEA) for children with special
educational needs. Pediatrics. 2015;136(6):e1650-1662. doi: 10.1542/peds.2015-3409.

American Academy of Pediatrics 869


PREP ® Self-Assessment PREPSA 2021
Question 236
A 7-day-old female newborn had an abnormal result on newborn screening revealing an absence
of T-cell receptor excision circles (TREC), which raises concern regarding severe combined
immunodeficiency.

Of the following, the MOST appropriate next test to confirm the diagnosis is
A. a complete blood cell count with differential
B. flow cytometry for dihydroxy-rhodamine123 reduction
C. lymphocyte subsets via flow cytometry
D. serum immunoglobulin subsets

American Academy of Pediatrics 870


PREP ® Self-Assessment PREPSA 2021
Correct Answer: C
Severe combined immunodeficiency (SCID) is a genetic primary immunodeficiency defined by
the absence or dysfunction of T cells. Left untreated, children with SCID die early in life owing
to infection, but the immunodeficiency is curable with allogeneic hematopoietic stem cell
transplant. During lymphopoiesis, the T-cell receptor (TCR) is formed through genetic
recombination of the VDJ segments of the TCR genes. During this recombination event,
segments of intervening DNA are spliced out as the VDJ segments are combined. The spliced
section forms a circle of DNA exogenous to chromosomal DNA called a T-cell receptor excision
circle (TREC). T-cell receptor excision circles are detectable via polymerase chain reaction and
are indicative of functional lymphopoiesis. Polymerase chain reaction for TRECs is a viable
screening test for the presence of T cells in the newborn. An absence of TRECs may suggest
SCID, and confirmatory testing should be performed. The definitive test for quantifying
lymphocyte subsets is flow cytometry. Because the newborn in the vignette has a newborn
screening result that revealed an absence of TRECs, the definitive test of flow cytometry should
be performed.

Whereas SCID is defined by the absence of functional T-cells, the remainder of hematopoiesis
remains intact. The differential on a complete blood count for a patient with SCID may
demonstrate lymphopenia but would not distinguish between different types of lymphocytes.
Flow cytometry for dihydroxy-rhodamine123 reduction is a test of neutrophil function, and is the
diagnostic test of choice for chronic granulomatous disease. Although serum immunoglobulin
subsets would likely be abnormal in a patient with SCID, they would not aid in distinguishing
between SCID and other hypogammaglobulinemic immunodeficiencies.

PREP Pearls
• Severe combined immunodeficiency represents a group of genetic primary
immunodeficiencies defined by the absence or dysfunction of T cells.
• The definitive test for quantifying lymphocyte subsets is flow cytometry.
• Left untreated, children with severe combined immunodeficiency die early in life owing
to infection, but the condition is curable with allogeneic hematopoietic stem cell
transplant.

ABP Content Specifications(s)


• Plan the laboratory evaluation of antibody function
• Plan the laboratory evaluation of cell-mediated immunity
• Recognize the clinical findings associated with combined antibody and cellular
immunodeficiency

Suggested Readings
• Dosanjh A. Autoimmunity and immunodeficiency. Pediatr Rev. 2015;36(11):489-495.
doi:10.1542/pir.36-11-489.
• Fleisher TA. Back to basics: primary immune deficiencies: windows into the immune
system. Pediatr Rev. 2006;27(10):363-372. doi:10.1542/pir.27-10-363.
• Lipstein EA, Vorono S, Browning MF, et al. Systematic evidence review of newborn
screening and treatment of severe combined immunodeficiency. Pediatrics.
2010;125(5):e1226-e1235. doi:10.1542/peds.2009-1567.
American Academy of Pediatrics 871
PREP ® Self-Assessment PREPSA 2021
Question 237
A 3-year-old boy with global developmental delay is seen for a health supervision visit. He was
given a diagnosis of autism spectrum disorder at a recent neuropsychological evaluation. His
family history is significant for an older brother and a maternal uncle with developmental delay
and attention-deficit/hyperactivity disorder. The boy is nondysmorphic and has normal growth
parameters. His parents are interested in understanding the genetic etiology of developmental
delay in their children, as well as their risk of having another child with similar conditions.

Of the following, the BEST next step in evaluation is


A. inform the parents that the genetics team is being consulted and obtain whole-exome
sequencing
B. obtain chromosomal microarray and fragile X analysis
C. obtain chromosomal microarray and fragile X analysis and request a consultation for
genetic counseling once the results are available
D. perform pretest counseling, obtain chromosomal microarray and fragile X analysis, and
request a consultation for genetic counseling

American Academy of Pediatrics 872


PREP ® Self-Assessment PREPSA 2021
Correct Answer: D
The next best step in evaluation for the patient in the vignette would be to perform pretest
counseling for the genetic test being obtained (chromosomal microarray and fragile X analysis)
and to request consultation for genetic counseling to ensure proper posttest counseling,
discussion of results, and discussion of need for follow-up evaluation in a genetics clinic.

Genetic counseling is a vital part of patient care before and after the ordering of any genetic test.
As defined by the National Society of Genetic Counselors in 2006 (J Genet
Couns. 2006;15(2):77-83), “genetic counseling is the process of helping people understand and
adapt to the medical, psychological, and familial implications of genetic contributions to
disease.” Genetic counselors are trained to interpret personal and family medical history, provide
education pertaining to inheritance and recurrence risk, and perform appropriate counseling for
the patient and family so that they can make an informed decision. Genetic counselors are an
important link between the health care provider and the patient, the family, or both. They prepare
the patient and family regarding possible test outcomes and secondary finding(s), and they
discuss the appropriateness of additional testing, if indicated. Owing to the paucity of genetic
counselors and geneticists, first-line genetic testing is now routinely performed by primary care
providers. Care must be taken to provide appropriate pretest counseling and obtain insurance
approval so that the family is made aware of expected out-of-pocket expense before the genetic
testing is conducted. A genetic test can have three possible outcomes:
• positive, indicating that a disease-causing change was identified and explaining the
clinical features seen in the patient
• negative, indicating that no disease-causing change was identified
• variant(s) of unknown clinical significance, indicating that a change was identified but
that, on the basis of present knowledge, it is unclear whether the change could be disease
causing or a benign variant

Appropriate posttest counseling should be carried out for any of these outcomes. If performed in
the primary care setting, the counseling should include an explanation of the result; any related
management, surveillance, and additional investigations related to the diagnosis; recurrence risks
for other family members; and referral to a genetics clinic.

Whereas consulting the genetics team would be an appropriate first step, concomitant whole-
exome sequencing would be incorrect. The American College of Medical Genetics and
Genomics recommends chromosomal microarray and fragile X analysis as first-line testing for
any patient with autism-spectrum disorder, developmental delay, or both. Pretest counseling is
important before chromosomal microarray and fragile X analysis analysis are performed.
Obtaining the tests without conducting pretest counseling or requesting a consultation later
would be inappropriate.

American Academy of Pediatrics 873


PREP ® Self-Assessment PREPSA 2021
PREP Pearls
• Pretest and posttest counseling is important for any genetic testing.
• Pretest counseling should include discussion of need for genetic testing, possible
outcomes, expected out-of-pocket expense, and referral to a genetics clinic, if indicated.
• Posttest counseling should include discussion of results, management and surveillance
based on diagnosis, need for follow-up testing, and evaluation by a geneticist, if
indicated.

ABP Content Specifications(s)


• Recognize the need for appropriate referral for genetic counseling

Suggested Readings
• Faucett WA, Peay H, Coughlin CR. Genetic testing: consent and result disclosure for
primary care providers. Med Clin N Am. 2019;103(6):967-
976.doi:10.1016/j.mcna.2019.07.001.
• Stein Q, Loman R, Zuck T. Genetic counseling in pediatrics. Pediatr
Rev. 2018;39(7):323-331. doi:10.1542/pir.2017-0194.

American Academy of Pediatrics 874


PREP ® Self-Assessment PREPSA 2021
Question 238
A 14-year-old adolescent boy with cerebral palsy and developmental delay is seen for a health
supervision visit. He recently moved from another state, and his mother needs assistance in
getting referrals for services such as physical and occupational therapy. She was not pleased with
the special education programming provided at her son’s previous school and felt he was placed
in too many classes with the general population of students, which were too advanced for him.
The patient has an IQ of 65. He is able to interact socially with others and to complete most
activities of daily living on his own. His mother is knowledgeable about the Individuals with
Disabilities Education Act and the Americans with Disabilities Act. She asks for a letter for his
new school to use in developing his individualized education program. She would like the
pediatrician to specify that her son not be placed in mainstream classes.

Of the following, the MOST appropriate statement to include in the discussion with the mother is
A. all state and local school districts are required to provide free and appropriate education
to children with disabilities up to age 21 years without parental input
B. children with disabilities are rarely educated with children without disabilities to the
maximum extent possible
C. the parents have a right to provide input into the individualized education goals, related-
service needs, and placement decisions
D. the primary care provider would need to complete a full neurodevelopmental evaluation
before requesting that a child with disabilities not be placed in any mainstream classes

American Academy of Pediatrics 875


PREP ® Self-Assessment PREPSA 2021
Correct Answer: C
The mother of the boy in this vignette has the right to provide input into the individualized
education goals, related service needs, and placement decisions as part of free and appropriate
public education until her child is 21 years of age. However, she cannot dictate that her child not
be in classes with children who do not have disabilities. As long as the child can make progress
in mainstream classes, it is recommended that the boy be taught in the least restrictive learning
environment to the extent possible. The primary care provider would not be involved in
developing the individualized education program (IEP) or determining whether the child would
be able to make progress in mainstream classes.

The Individuals with Disabilities Education Act (IDEA) was passed in 1975 to ensure that all
children with disabilities and special health care needs (CSHCN) receive free appropriate pulic
education (FAPE) between 3 and 21 years of age. Per the IDEA, the child must have one of the
following 13 diagnoses:
• autism;
• deafness;
• visual impairment/blindness;
• deafness and blindness;
• hearing impairment;
• emotional disturbance;
• intellectual disability;
• orthopedic impairment;
• combination of multiple disabilities;
• health impairment (ie, attention-deficit/hyperactivity disorder);
• learning disability (ie, dyslexia);
• speech or language impairment; or
• traumatic brain injury.

According to the IDEA, schools are required to identify and assess students who may have
disabilities, regardless of the severity of the disability, which is known as “child find.” The
evaluation is done at no cost to the family.

Once a student is identified as having a disability, schools must provide the child with special
education and related services such as speech therapy, counseling, and transportation to help the
student make progress in school. The FAPE guidelines include providing accommodations and
modifications to help the child learn, including use of audio books, extra time for test taking, or
special seating. It also requires the development of an IEP which encompasses individualized
services, yearly goals, and monitoring of progress. The child with a disability should be taught in
the least restrictive environment alongside students without disabilities, as long as it meets the
needs of the child. Schools must collaborate with parents and their CSHCN in creating an IEP.
The parents’ input should be considered with regard to goals, objectives, and service needs. The
FAPE rules do not require a school to place a child in a specific program or guarantee a specific
program desired by the parent. In addition, FAPE does not give any preferential treatment to
CSHCN for joining sports teams or clubs; however, the child has equal opportunity to participate
as would any other student. Parents must communicate with the teachers and advocate for their

American Academy of Pediatrics 876


PREP ® Self-Assessment PREPSA 2021
child if they have concerns regarding the child’s progress. If parents are unhappy with the
services their child is receiving at school, they have a right to request a dispute resolution.

PREP Pearls
• Free and appropriate public education must be offered to all children with disabilities
between 3 and 21 years of age.
• Once a child is identified as having a disability, the school is required to evaluate the
child to see how to best meet his or her educational needs. The evaluation is done at no
cost to the family.
• Schools must collaborate and consider the parents’ input when creating an individualized
education program.

ABP Content Specifications(s)


• Understand the state and federal statutes that govern the care of intellectually challenged
adolescents

Suggested Readings
• Lipkin PH, Okamoto J; Council of Children with Disabilities; Council on School Health.
The Individuals With Disabilities Education Act (IDEA) for children with special
educational needs. Pediatrics. 2015;136(6):e1650-e1662. doi:10.1542/peds.2015-3409.
• National Center for Learning Disabilities. Learn the law:
IDEA. https://www.ncld.org/get-involved/learn-the-law/idea/.
• Teelin KL, Dosa NP. School-related issues for children with special health care needs. In:
McInerny TK, Adam HM, Campbell DE, DeWitt TG, Foy JM, Kamat DM,
eds. American Academy of Pediatrics Textbook of Pediatric Care. 2nd ed. Elk Grove
Village, IL: American Academy of Pediatrics; 2017:371-374. Pediatric Care Online.

American Academy of Pediatrics 877


PREP ® Self-Assessment PREPSA 2021
Question 239
A previously healthy 14-year-old adolescent girl is seen in the office for fatigue and intermittent
constipation that she has experienced for the past 2 months. Her last menstrual period was
heavier than usual. She reports no sexual activity. She has a temperature of 37°C, a blood
pressure of 110/68 mm Hg, and a heart rate of 56 beats/min. She has a weight at the 50th
percentile, height at the 75th percentile, and body mass index at the 40th percentile. Her thyroid
is enlarged to about twice the normal size and has a firm, heterogeneous texture, without
palpable nodules. The remainder of her physical examination findings are unremarkable. A urine
pregnancy test has a negative result, and a complete blood cell count is normal.

Of the following, the BEST next step is to obtain


A. thyroid ultrasonography
B. a thyroid uptake and scan
C. a thyroid-stimulating hormone level
D. a thyroid-stimulating immunoglobulin level

American Academy of Pediatrics 878


PREP ® Self-Assessment PREPSA 2021
Correct Answer: C
The girl in the vignette has fatigue, constipation, heavy menses, and bradycardia with a negative
pregnancy test finding and a normal complete blood count. These symptoms are a result of
hypothyroidism. Her enlarged thyroid gland, with a firm, heterogeneous texture and no nodules,
is consistent with autoimmune thyroiditis (Hashimoto thyroiditis), the most common cause of
acquired hypothyroidism. Obtaining a thyroid-stimulating hormone level is the best next test to
make the diagnosis of primary hypothyroidism (a primary problem with the thyroid gland). In
this case, the thyroid-stimulating hormone level would be elevated. Thyroid peroxidase and anti-
thyroglobulin antibodies are often detectable in Hashimoto thyroiditis, and test findings for them
would likely be positive in this girl.

During adolescence, Hashimoto thyroiditis is more common in females than males. Other
common signs and symptoms of hypothyroidism include hair loss, dry skin, cold intolerance,
poor linear growth, mild weight gain, and delayed return of deep tendon reflexes.

A thyroid-stimulating immunoglobulin level would be positive in Graves disease (autoimmune


hyperthyroidism). An enlarged thyroid gland is also characteristic of Graves disease but is
usually less firm and more homogeneous. Furthermore, symptoms would be of hyperthyroidism,
not hypothyroidism as in the girl in the vignette. Thyroid ultrasonography is not indicated for this
girl. The American Academy of Pediatrics Section on Endocrinology, in its Choosing Wisely
recommendations, indicated that clinicians should “avoid routinely ordering thyroid
[ultrasonography] in children who have simple goiters or autoimmune thyroiditis.” Thyroid
ultrasonography should be performed in patients with asymmetric thyroid enlargement, palpable
nodules, or concerning cervical lymphadenopathy. Thyroid nodules are more common with
Hashimoto thyroiditis, but the majority are benign. There is no evidence that the discovery of a
malignant nodule before it becomes palpable improves health outcomes. Thyroid uptake and
scan studies are helpful in the evaluation of hyperthyroidism and would not be indicated in this
girl, who has signs and symptoms of hypothyroidism.

PREP Pearls
• A thyroid-stimulating hormone level is the best initial test in diagnosing primary
hypothyroidism.
• Autoimmune thyroiditis (Hashimoto thyroiditis) is the most common cause of acquired
hypothyroidism.
• Clinicians should avoid routinely ordering thyroid ultrasonography in children who have
simple goiters or autoimmune thyroiditis.

ABP Content Specifications(s)


• Recognize the clinical features associated with Hashimoto thyroiditis
• Plan the appropriate diagnostic evaluation to distinguish among Hashimoto thyroiditis,
other causes of thyroid enlargement, and hypothyroidism
• Understand the natural history of Hashimoto thyroiditis

American Academy of Pediatrics 879


PREP ® Self-Assessment PREPSA 2021
Suggested Readings
• Diaz A, Lipman Diaz EG. Hypothyroidism (published correction appears in Pediatr Rev.
2014;35[10]:446). Pediatr Rev. 2014;35(8):336-347. doi:10.1542/pir.35-8-336.
• Orlowski CC. Hypothyroidism. In: McInerny TK, Adam HM, Campbell DE, DeWitt TG,
Foy JM, Kamat DM, eds. American Academy of Pediatrics Textbook of Pediatric
Care. 2nd ed. Elk Grove Village, IL: American Academy of Pediatrics; 2017:2184-
2188. Pediatric Care Online.

American Academy of Pediatrics 880


PREP ® Self-Assessment PREPSA 2021
Question 240
A 4-year-old boy is being evaluated for developmental delay. His mother reports that she first
became concerned about his development around age 18 months because he had just begun
walking. At 2 years of age he was unable to jump with 2 feet, and could not run or walk up the
stairs. Fine motor, social, and language development are normal. He has been receiving physical
therapy with minimal improvement in his gross motor skills.

On physical examination, he is alert and playful. His cranial nerve examination findings are
normal. He has prominent calf muscles bilaterally with normal muscle tone (Item Q240A).
Strength testing is normal. Deep tendon reflexes are 2+ bilaterally and toes go downward on
plantar stimulation. Sensation to light touch is intact. He has a normal gait. When asked to rise
from the floor he pushes off his legs to transition to a standing position (Item Q240B).

Item Q240A: Boy with prominent calf muscles. Courtesy of J Goldstein

American Academy of Pediatrics 881


PREP ® Self-Assessment PREPSA 2021

Item Q240B: Boy as he attempts to stand. Reprinted with permission from Biggar WD. Pediatr
Rev. 2006;27(3):84.

American Academy of Pediatrics 882


PREP ® Self-Assessment PREPSA 2021
Of the following, the BEST next step in this boy’s diagnostic workup is
A. brain magnetic resonance imaging
B. creatine kinase
C. electromyography
D. muscle biopsy

American Academy of Pediatrics 883


PREP ® Self-Assessment PREPSA 2021
Correct Answer: B
The clinical presentation of the boy in the vignette is notable for a mild gross motor delay and
proximal muscle weakness with preserved reflexes and normal tone suggestive of a primary
muscle disorder, such as Duchenne muscular dystrophy (DMD). A screening serum creatine
kinase (CK) level would be the most helpful next step in his diagnostic evaluation. Creatine
kinase can vary with age, sex, physical activity, or trauma. Pathologic elevations can occur in
inflammatory muscle disorders such as infectious or idiopathic myositis, spinal muscular
atrophy, and muscular dystrophies. Patients with DMD demonstrate dramatic elevations in CK,
up to 50 to 100 times the normal range; CK can be elevated in an affected presymptomatic child.

Duchenne muscular dystrophy is a progressive X-linked recessive neuromuscular disorder


caused by a mutation (most commonly a deletion) in the dystrophin gene encoding the
dystrophin protein, which is involved in muscle membrane stability. When dystrophin is absent,
the muscle membrane is less stable and becomes damaged, resulting in an inflammatory cascade
with necrosis and muscle loss. Common genetic mutations have become a target for disease-
specific exon-skipping therapies.

Boys develop progressive muscle weakness affecting the proximal muscles in early childhood.
Many have delayed gross motor milestones. When affected children are asked to stand from a
seated position, they will use their hands to push off their legs, a sign of proximal muscle
weakness called a Gower maneuver. Pseudohypertrophy of the calves is seen because of
accumulation of fat between muscle fibers. Weakness is progressive, and eventually results in
the inability to ambulate, and later, a need for cardiopulmonary support. Glucocorticoid
administration has been shown to improve muscle strength and prolong ambulation. Eteplirsen is
a disease-specific exon-skipping therapy approved by the US Food and Drug Administration and
is available for boys with amenable mutations. Several other disease-specific treatments are
currently under investigation.

Brain magnetic resonance imaging is not the first-line diagnostic test in a patient with myopathy
symptoms. Electromyography (EMG) can be used as an extension of the neurologic
examination; EMG can aid in the localization of a lesion affecting the peripheral nervous system,
providing electrical confirmation of myopathy. Muscle biopsy is usually reserved for when a
diagnosis remains elusive or pathologic confirmation is needed.

PREP Pearls
• Duchenne muscular dystrophy is an X-linked recessive disorder presenting with
progressive, initially proximal, muscle weakness progressing to loss of ambulation,
cardiomyopathy, and eventually death.
• Serum creatine kinase is the initial diagnostic test of choice in a child suspected to have
myopathy.
• Glucocorticoids have been shown to improve muscle strength and prolong ambulation in
children with Duchenne muscular dystrophy.

American Academy of Pediatrics 884


PREP ® Self-Assessment PREPSA 2021
ABP Content Specifications(s)
• Recognize the clinical findings associated with proximal muscle weakness
• Formulate a differential diagnosis of a muscle disorder of various etiologies
• Recognize the clinical findings associated with dystrophinopathy (eg, Duchenne/Becker
muscular dystrophy)

Suggested Readings
• American Academy of Pediatrics. Duchenne muscular dystrophy https://www.aap.org/en-
us/advocacy-and-policy/aap-health-initiatives/Pages/Duchenne-Muscular-
Dystrophy.aspx.
• Biggar WD. Duchenne muscular dystrophy. Pediatr Rev. 2006;27(3):83-88.
doi: 10.1542/pir.27-3-83.
• Darras BT, Urion DK, Ghosh PS.
Dystrophinopathies. GeneReviews. https://www.ncbi.nlm.nih.gov/books/NBK1119/.
• Moxley RT III, Ciafaloni E. Muscular dystrophy. In: McInerny TK, Adam HM,
Campbell DE, DeWitt TG, Foy JM, Kamat DM, eds. American Academy of Pediatrics
Textbook of Pediatric Care. 2nd ed. Itasca, IL: American Academy of Pediatrics;
2016;chap 293:2344-2357. Pediatric Care Online.

American Academy of Pediatrics 885


PREP ® Self-Assessment PREPSA 2021
Question 241
A 24-day-old neonate is seen in the office for her first newborn visit after an uncomplicated 23-
day stay in the neonatal intensive care unit. She was born at 34 weeks of gestation via
uncomplicated vaginal delivery to a healthy mother. At birth, she weighed 1,780 g. Per the
discharge summary, she started oral feedings on day 1 and reached full-volume feedings at 5
days of age. She currently weighs 2,335 g. She has a length of 47 cm and a head circumference
of 33 cm. Her physical examination findings are otherwise normal. Her mother prefers to feed
the child with formula and asks what formula she should give her daughter. The neonate’s
growth charts are reviewed (Item Q241 ).

American Academy of Pediatrics 886


PREP ® Self-Assessment PREPSA 2021

Of the following, the MOST appropriate caloric density of formula for this neonate is
A. 19 calories/oz
B. 20 calories/oz
C. 22 calories/oz
D. 24 calories/oz

American Academy of Pediatrics 887


PREP ® Self-Assessment PREPSA 2021
Correct Answer: C
The infant in this vignette has a history of prematurity and birth weight of less than 1,800 g. The
recommendation on discharge from the neonatal intensive care unit is feeding with a preterm
transitional formula at 22 calories/oz with a goal of 20 to 30 g/day of weight gain after having
reached a weight of 2 kg. The higher-calorie formula of 24 calories/oz is reserved for premature
infants who have higher metabolic needs, such as those with significant bronchopulmonary
dysplasia or who demonstrate poor weight gain while in the neonatal intensive care unit. For
formula-fed infants, preterm infant formulas mixed to 22 to 24 calories/oz provide the calories
and higher vitamin content these infants require; therefore, the 19- or 20-calorie/oz formulas are
reserved for term infants. The preferred milk for most premature infants is human milk, although
for very low-birth-weight infants (birth weight <1,500 g), human milk by itself may not provide
an adequate amount of calories, protein, minerals, and vitamins without supplementation.
Fortification should be individualized to optimize growth trajectory during the first year after
birth.

Infant formulas are regulated by the US Food and Drug Administration under the Infant Formula
Act. Term formulas contain 19 to 20 calories/oz when mixed as instructed. All formulas are
designed to provide sufficient amounts of vitamins and micronutrients. The major differences
among the various formulas include amounts of protein, sugar, fat sources, and
vitamins/micronutrients. Item C241 demonstrates the differences between human milk and
various infant formulas, including their indications for usage. Soy formula is nutritionally
inappropriate for preterm infants because of the associated risk of osteopenia, aluminum toxicity,
and prevention of adequate absorption of zinc, phosphorus, and iron. It is medically indicated
only for infants with galactosemia or congenital lactase deficiency.

American Academy of Pediatrics 888


PREP ® Self-Assessment PREPSA 2021
PREP Pearls
• For formula-fed infants, preterm infant formulas mixed to 22 to 24 calories/oz provide
the calories and higher vitamin content these infants require.
• Soy formula is nutritionally inappropriate for preterm infants because of the risk of
osteopenia, aluminum toxicity, and prevention of adequate absorption of zinc,
phosphorus, and iron.

ABP Content Specifications(s)


• Understand the caloric requirements for patients of various ages, including those born
prematurely, and the circumstances in which those requirements may change

Suggested Readings
• American Academy of Pediatrics Committee on Nutrition. Feeding the infant. In
Kleinman RE, Greer FR, eds. Pediatric Nutrition. 7th ed. Elk Grove Village, IL:
American Academy of Pediatrics; 2013:13-110.
• Campbell DE. Continuing care of the infant after transfer from neonatal intensive care.
In: McInerny TK, Adam HM, Campbell DE, DeWitt TG, Foy JM, Kamat DM,
eds. American Academy of Pediatrics Textbook of Pediatric Care. 2nd ed. Elk Grove
Village, IL: American Academy of Pediatrics; 2017:1018-1049. Pediatric Care Online .
• DiMaggio DM, Cox A, Porto AF. Updates in infant nutrition. Pediatr Rev.
2017;38(10):449-462. doi:10.1542/pir.2016-0239.
• Martinez JA, Ballew MP. Infant formulas. Pediatr Rev. 2011;32(5):179-189.
doi:10.1542/pir.32-5-179.
• Tender JAF. Preterm infant nutrition. Pediatr Rev. 2004;25(9):328-329.
doi:10.1542/pir.25-9-328.
• Torrazza RM, Neu J. Evidence-based guidelines for optimization of nutrition for the very
low birthweight infant. NeoReviews. 2013;14(7):e340-e349. doi:10.1542/neo.14-7-e340.

American Academy of Pediatrics 889


PREP ® Self-Assessment PREPSA 2021

Question 242
A 16-year-old patient with relapsed Ewing sarcoma is admitted to the pediatric intensive care
unit in respiratory distress. The consensus among members of the oncology service is that her
acute lung disease is caused by progression of disease and that there are no effective additional
oncologic treatments available. The patient and family previously recorded advance directives
specifically restricting endotracheal intubation and mechanical ventilation and stating that no
cardiopulmonary resuscitation is to be performed in the event of a cardiac arrest. The patient is
visibly uncomfortable and air-hungry despite trials of high-flow nasal cannula and escalation to
bilevel positive pressure ventilation. After discussion with the family, in an attempt to make the
patient more comfortable, she is given a weight-appropriate dose of morphine. Shortly thereafter,
she becomes hypopneic with shallow respiratory effort.

Of the following, the MOST appropriate next step is


A. administration of naloxone to reverse the morphine effects
B. administration of a paralytic to stop the gasping effort
C. endotracheal intubation until the morphine effect wears off
D. provision of family support and compassionate care

American Academy of Pediatrics 890


PREP ® Self-Assessment PREPSA 2021
Correct Answer: D
The patient in the vignette is receiving end-of-life care and displays symptoms of discomfort and
air hunger as a result of her terminal cancer. As such, it is appropriate for the clinical team to
administer therapies that will increase comfort. In this case, a weight-appropriate dose of
narcotic is administered, the side effect of which causes further respiratory depression. On the
basis of the advance directives in place as well as the intention to provide comfort to the patient,
the appropriate response to the hypoventilation is to provide family support and compassionate
care.

The ethical construct above requires knowledge of the doctrine of double effect. Dying children
frequently have pain or dyspnea at the end of life, and pharmacologic interventions intended to
provide symptomatic relief may also hasten death in rare cases. The doctrine of double effect
justifies using appropriate medications and doses such that symptoms may be relieved, even if
doing so will precipitate an otherwise inevitable and imminent death. There is an ethical
distinction between the intended effect of an action (eg, administration of medication that will
provide pain relief) from the foreseeable but unintended negative effect (respiratory depression
as a known side effect of the opioid). To be ethically justified, a number of criteria must exist
simultaneously:
• The intended effect must be beneficial (eg, relief of pain or suffering)
• Only the beneficial effect of the action is considered (eg, when prescribing the
medication, the desire is to relieve pain and not to hasten death)
• The negative effect is not the only means of achieving the good effect (eg, giving high-
dose potassium to precipitate death as a means to end pain and suffering)
• The good effect must outweigh the bad effect (eg, if death is otherwise inevitable and
imminent, the negative effect can be justified)

Administration of naloxone would be counter to the goal of providing pain relief and may in fact
precipitate more suffering by reversing the pain relief the opioid provides. Administration of a
paralytic would hasten death but would provide no pain relief; in addition, it would precipitate
death solely as a means to an end and therefore would be inappropriate. Finally, endotracheal
intubation would violate the advance directives noted in the vignette and is counter to the end-of-
life decisions of the patient and family.

PREP Pearls
• Managing pain and other symptoms at the end of life is required to ensure appropriate
support for dying patients and their families.
• Rarely, appropriate interventions may hasten death; however, if the intent was to provide
symptomatic relief, the intervention may be ethically justified under the doctrine of
double effect.
• The use of a medication for the sole intent of hastening death (such as high-dose
potassium chloride) is medically and ethically inappropriate.

MOCA-Peds Objective
• Recognize and apply ethical principles involved in end-of-life care.

American Academy of Pediatrics 891


PREP ® Self-Assessment PREPSA 2021
ABP Content Specifications(s)
• Recognize and apply ethical principles regarding the issue of medical futility
• Recognize and apply ethical principles involving palliative care and pain management
• Recognize and apply ethical principles involving euthanasia

Suggested Readings
• Opel DJ, Olson ME. Ethics for the pediatrician. Pediatr Rev. 2012;33(8):370-373.
doi:10.1542/pir.33-8-370.
• Orioles A, Morrison WE. Medical ethics in pediatric critical care. Crit Care
Clin. 2013;29(2):359-375. doi:10.1016/j.ccc.2012.12.002.

American Academy of Pediatrics 892


PREP ® Self-Assessment PREPSA 2021
Question 243
A 10-year-old boy is seen for a health supervision visit. He and his mother are concerned about
his short stature. He is otherwise healthy, takes no medications, and is doing well in school.
Findings of a comprehensive review of systems are unremarkable. His adjusted midparental
height is at the 25th percentile. Both parents had typical pubertal timing. His vital signs are
normal for age. The boy’s growth chart is shown in Item Q243. His body mass index is at the
66th percentile. An upper-to-lower body segment ratio is calculated as 1.2 (normal, 1). His
sexual maturity rating is 1. The remainder of his physical examination findings are
unremarkable. A bone age radiograph is read as concordant with his chronological age.

American Academy of Pediatrics 893


PREP ® Self-Assessment PREPSA 2021

American Academy of Pediatrics 894


PREP ® Self-Assessment PREPSA 2021
Of the following, the MOST likely diagnosis for this boy is
A. familial short stature
B. constitutional delay of growth and puberty
C. growth hormone deficiency
D. a skeletal dysplasia

American Academy of Pediatrics 895


PREP ® Self-Assessment PREPSA 2021
Correct Answer: D
The boy in the vignette has significant short stature, a normal body mass index, a concordant
bone age, and a high upper-to-lower body segment ratio (U/L ratio), indicating relatively longer
trunk and shorter legs. These findings are most consistent with a skeletal dysplasia.

The U/L ratio can be used to assess body proportions. The lower segment is measured from the
pubic symphysis to the floor while standing. The upper segment is calculated as height minus the
lower segment. Infants have a relatively high U/L ratio (about 1.7 at birth), which gradually
decreases to 1 around age 10 years, reaches a nadir during early puberty, and returns to about
0.9-1 by adulthood. A high U/L ratio indicates a relatively longer trunk and shorter legs, and it
can be seen in skeletal dysplasias and precocious puberty. A low U/L ratio indicates a relatively
shorter trunk and longer legs, and it can be seen in Klinefelter syndrome and delayed puberty.

There are many types of skeletal dysplasias. More common skeletal dysplasias include
achondroplasia, hypochondroplasia, and short stature homeobox-containing gene (SHOX)
deficiency, either associated with Turner syndrome or isolated. Haploinsufficiency of the SHOX
gene is responsible for the short stature and skeletal findings associated with Turner syndrome.
Features of isolated SHOX deficiency may include shortening of the forearm and lower limbs, a
Madelung wrist deformity (dorsal subluxation of the distal ulna), and a high-arched palate. The
growth chart for the boy in the vignette is consistent with isolated SHOX deficiency. This
diagnosis is made via genetic testing. Growth hormone therapy is approved by the US Food and
Drug Administration for treatment of short stature due to isolated SHOX deficiency.

The other response choices are all associated with normal body proportions. Constitutional delay
of growth and puberty is associated with a delayed bone age; the bone age of the boy in the
vignette is concordant. Family history is also common for those with constitutional delay, but the
boy’s parents both had typical pubertal timing. Growth hormone deficiency is associated with
abnormally slow growth and a delayed bone age. The boy’s height percentile is well below that
of his adjusted midparental height, so familial short stature is not the correct diagnosis.

PREP Pearls
• The upper-to-lower body segment ratio (U/L ratio) can be used to assess body
proportions.
• A high U/L ratio indicates a relatively longer trunk and shorter legs, and it can be seen in
skeletal dysplasias and precocious puberty.
• A low U/L ratio indicates a relatively shorter trunk and longer legs, and it can be seen in
Klinefelter syndrome and delayed puberty.

MOCA-Peds Objective
• Recognize the genetic syndromes that may present with abnormal growth.

ABP Content Specifications(s)


• Recognize the differences in upper body segment-to-lower body segment ratio in children
compared with that of adults

American Academy of Pediatrics 896


PREP ® Self-Assessment PREPSA 2021
Suggested Readings
• Binder G, Rappold GA. SHOX deficiency disorders. In: Adam MP, Ardinger HH, Pagon
RA, et al, eds. GeneReviews [Internet]. Seattle, WA: University of Washington, Seattle;
1993-2020. www.ncbi.nlm.nih.gov/books/NBK1215/.
• Braun LR, Marino R. Disorders of growth and stature. Pediatr Rev. 2017;38(7):293-304.
doi:10.1542/pir.2016-0178.
• Kaplowitz P. Short stature. In: McInerny TK, Adam HM, Campbell DE, DeWitt TG, Foy
JM, Kamat DM, eds. American Academy of Pediatrics Textbook of Pediatric Care. 2nd
ed. Elk Grove Village, IL: American Academy of Pediatrics; 2017:1585-1588. Pediatric
Care Online.
• Rose SR, Vogiatzi MG, Copeland KC. A general pediatric approach to evaluating a short
child. Pediatr Rev. 2005;26(11):410-420. doi:10.1542/pir.26-11-410.

American Academy of Pediatrics 897


PREP ® Self-Assessment PREPSA 2021
Question 244
A 6-month-old infant is seen for a health supervision visit. His mother has introduced some solid
foods into her son’s diet including rice cereal, oatmeal, and carrots with no adverse reactions.
The infant’s vital signs and growth parameters are within normal limits. His physical
examination findings are normal with the exception of mild eczema. There is no family history
of food allergies. His mother asks how soon she may introduce peanut-containing foods.

Of the following, the BEST response is that these foods should be introduced
A. after allergy testing is performed
B. after his rash has resolved
C. now without additional testing
D. at 12 months of age or later

American Academy of Pediatrics 898


PREP ® Self-Assessment PREPSA 2021
Correct Answer: C
The infant in the vignette may be introduced to peanut-containing foods now without further
testing, according to the 2017 guidelines from the National Institute of Allergy and Infectious
Diseases (https://www.sciencedirect.com/science/article/pii/S1939455119300067?via%3Dihub).
The boy need not undergo allergy testing because the results will not be of high clinical value.
There is no need to wait until after his rash has resolved because his eczema is mild. He should
not wait until 12 months to be introduced to peanut-containing foods because waiting may
actually increase his risk of allergy.

The most common foods associated with allergies are milk, egg, soy, nuts and peanuts, fish and
shellfish, wheat, and sesame. The prevalence of these food allergies has been increasing and
strategies to prevent the development of food allergies have varied. The Learning Early About
Peanut Allergy (LEAP) study informed the most current guidelines for the prevention of peanut
allergy in children. These recommendations are summarized in Item C244. Introduction of
peanut-containing foods at around 6 months of age has been shown to protect against future
development of peanut allergy. Healthy infants who have caregivers or siblings who are allergic
to peanuts may still follow the guidelines, but families should be aware of the potential for
accidental exposure of the allergic sibling or caregiver to peanut-containing foods.

Data regarding the introduction of other allergenic foods are not as rigorous as for peanuts; in
general, they should be introduced to healthy infants between 4 and 6 months of age and neither
earlier nor, if possible, later. Some research has shown that egg and fish introduction at this age
has been shown to be protective.

Item C244: Recommendations for Introduction of Peanuts.


Reprinted with permission from Wade TAW, Chan ES. Pediatr Rev. Infant peanut introduction
simplified. 2019;40(5):214. Adapted with permission from Togias A, Cooper SF, Acebal ML, et
al. Addendum guidelines for the prevention of peanut allergy in the United States: report of the

American Academy of Pediatrics 899


PREP ® Self-Assessment PREPSA 2021
National Institute of Allergy and Infectious Diseases-sponsored expert panel. Ann Allergy
Asthma Immunol. 2017;118(2):166–173.e7

Families often ask if their child should undergo food allergy testing before introducing foods. In
healthy infants, allergy testing of any modality more often than not yields many false-positive
results, which may lead to unnecessary elimination diets that are lacking in many key nutrients.
However, infants who have severe eczema or other red flags such as weight loss, gross blood in
the stool, or persistent vomiting should undergo consultation with a multidisciplinary team.

Children who are truly allergic to multiple foods must find alternative nutrition sources. For
example, children allergic to milk, nuts, and soy may find rice milk or oat milk a good alternative
source of vitamin D and calcium, but the family will need to recognize and address the decrease
in protein intake. Multivitamin supplements or special formulas may be needed in some cases.
Working closely with a registered dietitian is often helpful.

PREP Pearls
• Healthy infants with no eczema or mild to moderate eczema or other food allergies
should be introduced to peanut-containing foods, as well as other highly allergenic foods,
at around 6 months of age.
• Infants with severe eczema or egg allergy should undergo allergy testing before being
introduced to peanut-containing foods.
• Children who have multiple food allergies must find alternative nutrition sources, often
requiring the assistance of a nutritionist.

ABP Content Specifications(s)


• Recognize the effects of a restricted diet for multiple food allergies on the nutritional
adequacy of a patient’s diet
• Understand and apply current recommendations for feeding infants who are at risk for the
development of food allergy

American Academy of Pediatrics 900


PREP ® Self-Assessment PREPSA 2021
Suggested Readings
• Cosme-Blanco W, Arroyo-Flores E, Ale H. Food Allergies. Pediatr Rev. 2020; 41(8):
403-415. doi: 10.1542/pir.2019-0037 .
• Greer FA, Sicherer SH, Burks AW; Committee on Nutrition, Section on Allergy and
Immunology. The effects of early nutritional interventions on the development of atopic
disease in infants and children: the role of maternal dietary restriction, breastfeeding,
hydrolyzed formulas, and timing of introduction of allergenic complementary
foods. Pediatrics. 2019;143(4):e20190281. doi: 10.1542/peds.2019-0281.
• Le-Carlson, Kerner JA. Gastrointestinal allergy. In: McInerny TK, Adam HM, Campbell
DE, DeWitt TG, Foy JM, Kamat DM, eds. American Academy of Pediatrics Textbook of
Pediatric Care. 2nd ed. Itasca, IL: American Academy of Pediatrics; 2016;chap
256:2076-2080. Pediatric Care Online.
• Togias A, Cooper SF, Acebal ML, et al. Addendum guidelines for the prevention of
peanut allergy in the United States: report of the National Institute of Allergy and
Infectious Diseases–sponsored expert panel. World Allergy Organ J. 2017;10(1):1.
doi: 10.1186/s40413-016-0137-9.
• Wade TAW, Chan ES. Infant peanut introduction simplified. Pediatr
Rev. 2019;40(5):211-218. doi: 10.1542/pir.2018-0223.

American Academy of Pediatrics 901


PREP ® Self-Assessment PREPSA 2021
Question 245
A 16-year-old adolescent boy is seen in the office for evaluation of chest pain. He describes an
intermittent, sharp, stabbing pain along the left upper sternal border that has lasted for the last 3
days. He reports no trauma to the area. He states that it hurts the most when he takes a deep
breath. The pain seems to go away on its own. He has not taken any medications for the pain. He
appears comfortable and has normal vital signs. The pain is reproduced by palpation at the left
upper sternal border. His examination findings are otherwise unremarkable.

Of the following, the BEST next step in treatment is to


A. initiate ranitidine therapy
B. initiate therapy with an inhaled bronchodilator
C. perform electrocardiography
D. provide reassurance

American Academy of Pediatrics 902


PREP ® Self-Assessment PREPSA 2021
Correct Answer: D
The patient in the vignette describes sharp intermittent chest pain that is reproducible with
inspiration and palpation. This is consistent with chest wall pain, and reassurance is indicated.
Neither an electrocardiogram nor medications are indicated.

Chest pain in children is typically not caused by a serious disease. It is broadly categorized into
cardiac chest pain (Item C245A) and noncardiac chest pain (Item C245B). The vast majority of
pediatric chest pain is noncardiac in origin. Chest pain that is reproducible with inspiration and
palpation is characteristic of musculoskeletal chest pain, the most common etiology of pediatric
chest pain. Reassurance, rest, and analgesia are the recommended treatments for musculoskeletal
chest pain. Asthma or respiratory infections, gastroesophageal reflux disease, ulcers, and
cholecystitis can cause chest pain. In patients with chest pain of a respiratory or gastrointestinal
etiology, the history and physical examination may demonstrate shortness of breath, dyspnea, or
chest pain associated with eating or vomiting—symptoms not seen for the boy in this vignette.

Cardiac chest pain is rare in children. In taking the history, providers should ask about
palpitations, syncope, chest pain with exertion, a family history of genetic or connective tissue
disease (eg, Marfan syndrome), and a history of Kawasaki disease or cardiac surgery. A detailed
physical examination should include evaluation for signs and symptoms of genetic disorders. A
thorough cardiac examination should include evaluation for jugular venous distension and for a
heave, murmur, gallop, or rub. The finding of hepatomegaly can suggest a cardiac etiology. A
health history or physical examination findings that raise concern should prompt further
evaluation, including an electrocardiogram and referral to a pediatric cardiologist.

American Academy of Pediatrics 903


PREP ® Self-Assessment PREPSA 2021

American Academy of Pediatrics 904


PREP ® Self-Assessment PREPSA 2021

American Academy of Pediatrics 905


PREP ® Self-Assessment PREPSA 2021
PREP Pearls
• Chest pain in children is typically not a serious problem and may be musculoskeletal,
respiratory, gastrointestinal, or psychogenic in nature.
• A child whose history reveals chest pain with exertion or palpitations or who has
abnormal findings on cardiac examination or electrocardiogram should be referred.

ABP Content Specifications(s)


• Recognize the cardiovascular and non-cardiovascular causes of chest pain in children of
various ages

Suggested Readings
• Angoff GH, Kane DA, Giddins N, et. al. Regional implementation of a pediatric
cardiology chest pain guideline using SCAMPs methodology. Pediatrics.
2013;132(4):e1010-e1017. doi:10.1542/peds.2013-0086.
• Reddy SR, Singh HR. Chest pain in children and adolescents. Pediatr
Rev. 2010;31(1):e1-e9. doi:10.1542/pir.31-1-e1.
• Saleeb SF, Li WY, Warren SZ, Lock JE. Effectiveness of screening for life-threatening
chest pain in children. Pediatrics. 2011;128(5):e1062-e1068. doi:10.1542/peds.2011-
0408.
• Schroeder SA. Chest pain. In: McInerny TK, Adam HM, Campbell DE, DeWitt TG, Foy
JM, Kamat DM, eds. American Academy of Pediatrics Textbook of Pediatric Care. 2nd
ed. Elk Grove Village, IL: American Academy of Pediatrics; 2017:1235-1239. Pediatric
Care Online .

American Academy of Pediatrics 906


PREP ® Self-Assessment PREPSA 2021
Question 246
A 3-month-old infant is admitted to the hospital for diarrhea and dehydration; this is his second
admission in 3 weeks for the same symptoms. He has had 4 weeks of loose, watery stools 15 to
20 times every day, with persistent diaper rash. He is exclusively breastfed. Today, for the first
time, his parents have seen streaks of blood in his stools. He has had difficulty gaining weight
recently. The infant was born at term and passed meconium within the first 24 hours after birth.
His family history is notable for environmental allergies in both parents. His physical
examination findings demonstrate weight and length less than the 3rd percentile; his abdomen is
soft, distended, and without hepatosplenomegaly; he has pitting edema of both legs. Laboratory
testing shows his serum albumin to be 1.8 g/dL (18 g/L).

Of the following, the test MOST likely to identify the cause of the infant’s condition is a(n)
A. ammonia level
B. echocardiography
C. fecal a1-antitrypsin level
D. urinalysis

American Academy of Pediatrics 907


PREP ® Self-Assessment PREPSA 2021
Correct Answer: C
The infant in the vignette has a clinical picture consistent with protein-losing enteropathy (PLE)
because of eosinophilic (allergic) enteropathy, as supported by diarrhea, failure to thrive, and
edema in the context of a family history of allergies. Of the response choices listed, the test that
would diagnose PLE is a fecal α1-antitrypsin (α1AT) level. Because α1AT is not degraded in the
stomach and is not actively secreted or absorbed in the intestine, it is a reliable marker of protein
loss in the intestine. Elevated fecal α1AT levels should prompt further testing for the causes of
PLE.

Protein-losing enteropathy is defined by excessive protein loss in the intestine, which can result
in hypoalbuminemia and edema. Intestinal disorders that can cause a PLE (Item C246) can be
divided into inflammatory (resulting in increased gut protein loss), infectious, oncologic,
rheumatologic, and lymphatic obstruction.

The serum ammonia level would be increased in acute liver failure, which could also cause
edema and hypoalbuminemia; however, this diagnosis is less likely with normal liver and spleen
findings on physical examination in a child with chronic diarrhea, hematochezia, and failure to
American Academy of Pediatrics 908
PREP ® Self-Assessment PREPSA 2021
thrive. Echocardiography would diagnose cardiovascular disease that could present with ascites
and edema; however, without findings of hepatic enlargement and/or a cardiac murmur, this is
unlikely. Urinalysis could demonstrate evidence of nephrotic syndrome, a cause of
hypoalbuminemia and edema, but given the presence of diarrhea and hematochezia, this is a less
likely diagnosis than PLE.

PREP Pearls
• Protein-losing enteropathy should be suspected in children with hypoalbuminemia and
edema.
• A fecal α1-antitrypsin level is recommended to diagnose protein-losing enteropathy.
• Elevated fecal α1-antitrypsin levels should prompt further testing for a cause of protein-
losing enteropathy, based on the clinical presentation.

ABP Content Specifications(s)


• Plan the diagnostic evaluation of a patient with suspected protein-losing enteropathy,
while considering its causes
• Recognize the clinical features associated with protein or calorie deficiency, including
edema and malnutrition

Suggested Readings
• Guiraldes E. Protein-losing enteropathy. In: Wyllie R, Hyams JS, Kay M, eds. Pediatric
Gastrointestinal and Liver Disease. 5th ed. Philadelphia, PA: Elsevier; 2016:389-394.
• Zella GC, Israel EJ. Chronic diarrhea in children. Pediatr Rev. 2012;33:207.
doi: 10.1542/pir.33-5-207.

American Academy of Pediatrics 909


PREP ® Self-Assessment PREPSA 2021

Question 247
A 20-month-old girl is brought to the emergency department after 36 hours of nausea,
nonbloody, nonbilious emesis, and nonbloody diarrhea. Her parents report that she has been
unable to tolerate both liquids and solids for the past 24 hours. She has urinated 2 times in the
last 24 hours.

The girl has a heart rate of 130 beats/min and a respiratory rate of 26 breaths/min, and she is
afebrile. She is drowsy. Her mucous membranes are dry. Her abdomen has increased bowel
sounds. She has no rashes. Her capillary refill time is 3 sec.

Laboratory data are as follows:


Laboratory Test Result
Serum
Sodium 148 mEq/L (148 mmol/L)
Potassium 4.8 mEq/L (4.8 mmol/L)
Carbon dioxide 19 mEq/L (19 mmol/L)
Chloride 111 mEq/L (111 mmol/L)
Blood urea nitrogen 29 mg/dL (10.4 mmol/L)
Creatinine 0.6 mg/dL (53 µmol/L)
Glucose 72 mg/dL (4 mmol/L)
Urine
Specific gravity 1.025

Of the following, the treatment modality BEST recommended for this child is
A. banana/rice/applesauce/toast (BRAT) diet
B. enteral rehydration with lactose-containing product
C. enteral rehydration with oral rehydration solution
D. parenteral hydration

American Academy of Pediatrics 910


PREP ® Self-Assessment PREPSA 2021
Correct Answer: D
The 20-month-old girl in this vignette has acute viral gastroenteritis with moderate to severe
dehydration, as evidenced by her tachycardia, dry mucous membranes, and delayed capillary
refill as seen on physical examination. Her laboratory studies are notable for mildly elevated
sodium, acute kidney injury (elevated blood urea nitrogen and creatinine levels), metabolic
acidosis with mildly elevated anion gap (18 mEq/L; Anion gap (mEq/L) = sodium (Na+) −
[chloride (Cl−) + bicarbonate (HCO3−)], and increased specific gravity of urine.

The routine management of acute gastroenteritis in the setting of no-to-mild dehydration, given
its self-limited nature, involves supportive care with oral rehydration. However, in the setting of
moderate to severe dehydration in which the patient is unable to tolerate oral solutions,
intravenous (parenteral) fluids should be administered. Antiemetic agents, such as ondansetron,
are not recommended for the routine management of acute viral gastroenteritis; however, for
children older than 6 months with moderate dehydration and the inability to tolerate oral
rehydration, studies have shown that one dose (0.15 mg/kg) of ondansetron may facilitate oral
rehydration.

Once hydration is reestablished, early refeeding with an age-appropriate diet is recommended to


promote mucosal health and gut integrity (including disaccharidase function). Early refeeding is
associated with a shorter duration of diarrhea. Studies have not shown an increase in vomiting or
diarrhea after early refeeding. Foods high in simple sugars, such as juices, may result in osmotic
diarrhea and an increased risk of hyponatremia, given low concentrations of potassium and
sodium. Lean meats, fruits, vegetables, and yogurts may be better tolerated than foods high in
simple sugars and fats.

Lactose reduction has shown some benefit in reducing the duration of diarrhea in hospitalized
patients; however, there are no data to support lactose reduction in children in the outpatient
setting. Viral gastroenteritis is associated with mild lactose intolerance, but it is usually self-
limiting.

The banana/rice/applesauce/toast (BRAT) diet has not been studied and is not recommended.
Furthermore, such a diet is too restrictive and not nutritionally adequate in the setting of acute
gastroenteritis.

PREP Pearls
• In settings of moderate dehydration in which the patient is unable to tolerate oral
solutions, intravenous (parenteral) fluids should be administered.
• Once hydration is reestablished, early refeeding with an age-appropriate diet is
recommended to promote mucosal health and gut integrity; early refeeding is associated
with a shorter duration of diarrhea.
• Viral gastroenteritis is associated with mild lactose intolerance; however, it is usually
self-limiting, and there are no data to support lactose reduction in children in the
outpatient setting.

American Academy of Pediatrics 911


PREP ® Self-Assessment PREPSA 2021
ABP Content Specifications(s)
• Understand the importance of early refeeding in a child with gastroenteritis
• Plan dietary management for a patient with a gastrointestinal disorder

Suggested Readings
• Freedman SB, Thull-Freedman JD, Rumantir M, Atenafu EG, Stephens D. Emergency
department revisits in children with gastroenteritis. J Pediatr Gastroenterol
Nutr. 2013;57(5):612-618. doi:10.1097/MPG.0b013e3182a1dd93.
• Guarino A, Ashkenazi S, Gendrel D, et al; European Society for Pediatric
Gastroenterology, Hepatology, and Nutrition; European Society for Pediatric Infectious
Diseases. European Society for Pediatric Gastroenterology, Hepatology, and
Nutrition/European Society for Pediatric Infectious Diseases evidence-based guidelines
for the management of acute gastroenteritis in children in Europe: update 2014. J Pediatr
Gastroenterol Nutr. 2014;59(1):132-152. doi:10.1097/MPG.0000000000000375.
• Lo Vecchio A, Dias JA, Berkley JA, et al. Comparison of recommendations in clinical
practice guidelines for acute gastroenteritis in children. J Pediatr Gastroenterol
Nutr. 2016;63(2):226-235. doi:10.1097/MPG.0000000000001133.
• Lo Vecchio A, Vandenplas Y, Benninga M, et al. An international consensus report on a
new algorithm for the management of infant diarrhœa. Acta Pædiatr. 2016;105(8):e384-
e389. doi:10.1111/apa.13432.

American Academy of Pediatrics 912

You might also like